You are on page 1of 583

https://t.

me/Anesthesia_Books
1,000 Practice MTF MCQs for
the Primary and Final FRCA

https://t.me/Anesthesia_Books

Downloaded from https://www.cambridge.org/core. University of Edinburgh, on 19 Aug 2019 at 13:21:27, subject to the Cambridge Core terms of
use, available at https://www.cambridge.org/core/terms. https://doi.org/10.1017/9781108566100
Downloaded from https://www.cambridge.org/core. University of Edinburgh, on 19 Aug 2019 at 13:21:27, subject to the Cambridge Core terms of
use, available at https://www.cambridge.org/core/terms. https://doi.org/10.1017/9781108566100
1,000 Practice MTF
MCQs for the Primary
and Final FRCA

Edited by
Hozefa Ebrahim
University Hospitals, Birmingham

Michael Clarke
Worcestershire Acute Hospitals NHS Trust

Hussein Khambalia
Health Education England, North West

Insiya Susnerwala
Health Education England, North West

Richard Pierson
The Dudley Group NHS Foundation Trust

Anna Pierson
The Dudley Group NHS Foundation Trust

Natish Bindal
Queen Elizabeth Hospital Birmingham

Downloaded from https://www.cambridge.org/core. University of Edinburgh, on 19 Aug 2019 at 13:21:27, subject to the Cambridge Core terms of
use, available at https://www.cambridge.org/core/terms. https://doi.org/10.1017/9781108566100
University Printing House, Cambridge CB2 8BS, United Kingdom
One Liberty Plaza, 20th Floor, New York, NY 10006, USA
477 Williamstown Road, Port Melbourne, VIC 3207, Australia
314–321, 3rd Floor, Plot 3, Splendor Forum, Jasola District Centre, New Delhi – 110025, India
79 Anson Road, #06–04/06, Singapore 079906

Cambridge University Press is part of the University of Cambridge.


It furthers the University’s mission by disseminating knowledge in the pursuit of
education, learning, and research at the highest international levels of excellence.

www.cambridge.org
Information on this title: www.cambridge.org/9781108465830
DOI: 10.1017/9781108566100
© Cambridge University Press 2019
This publication is in copyright. Subject to statutory exception
and to the provisions of relevant collective licensing agreements,
no reproduction of any part may take place without the written
permission of Cambridge University Press.
First published 2019
Printed and bound in Great Britain by Clays Ltd, Elcograf S.p.A.
A catalogue record for this publication is available from the British Library.
Library of Congress Cataloging-in-Publication Data
Names: Ebrahim, Hozefa, editor.
Title: 1,000 practice MTF MCQs for the primary and final FRCA / edited by Hozefa Ebrahim
[and six others].
Other titles: 1000 practice MTF MCQs for the primary and final FRCA | One thousand practice
MTF MCQs for the primary and final FRCA
Description: Cambridge, United Kingdom ; New York, NY : Cambridge University Press,
2019. | Includes index.
Identifiers: LCCN 2018037315 | ISBN 9781108465830 (paperback)
Subjects: | MESH: Royal College of Anaesthetists (Great Britain) | Anesthesia – methods |
Anesthetics – pharmacology | United Kingdom | Examination Questions
Classification: LCC RD81 | NLM WO 218.2 | DDC 617.9/6–dc23
LC record available at https://lccn.loc.gov/2018037315
ISBN 978-1-108-46583-0 Paperback
Cambridge University Press has no responsibility for the persistence or accuracy of
URLs for external or third-party internet websites referred to in this publication
and does not guarantee that any content on such websites is, or will remain,
accurate or appropriate.

.........................................................................................................................................................................................
Every effort has been made in preparing this book to provide accurate and up-to-date information that is in
accord with accepted standards and practice at the time of publication. Although case histories are drawn
from actual cases, every effort has been made to disguise the identities of the individuals involved.
Nevertheless, the authors, editors, and publishers can make no warranties that the information contained
herein is totally free from error, not least because clinical standards are constantly changing through
research and regulation. The authors, editors, and publishers therefore disclaim all liability for direct or
consequential damages resulting from the use of material contained in this book. Readers are strongly
advised to pay careful attention to information provided by the manufacturer of any drugs or equipment
that they plan to use.
Downloaded from https://www.cambridge.org/core. University of Edinburgh, on 19 Aug 2019 at 13:21:27, subject to the Cambridge Core terms of
use, available at https://www.cambridge.org/core/terms. https://doi.org/10.1017/9781108566100
1,000 Practice MTF MCQs for
the Primary and Final FRCA

Downloaded from https://www.cambridge.org/core. University of Edinburgh, on 19 Aug 2019 at 13:21:27, subject to the Cambridge Core terms of
use, available at https://www.cambridge.org/core/terms. https://doi.org/10.1017/9781108566100
Downloaded from https://www.cambridge.org/core. University of Edinburgh, on 19 Aug 2019 at 13:21:27, subject to the Cambridge Core terms of
use, available at https://www.cambridge.org/core/terms. https://doi.org/10.1017/9781108566100
1,000 Practice MTF
MCQs for the Primary
and Final FRCA

Edited by
Hozefa Ebrahim
University Hospitals, Birmingham

Michael Clarke
Worcestershire Acute Hospitals NHS Trust

Hussein Khambalia
Health Education England, North West

Insiya Susnerwala
Health Education England, North West

Richard Pierson
The Dudley Group NHS Foundation Trust

Anna Pierson
The Dudley Group NHS Foundation Trust

Natish Bindal
Queen Elizabeth Hospital Birmingham

Downloaded from https://www.cambridge.org/core. University of Edinburgh, on 19 Aug 2019 at 13:21:27, subject to the Cambridge Core terms of
use, available at https://www.cambridge.org/core/terms. https://doi.org/10.1017/9781108566100
University Printing House, Cambridge CB2 8BS, United Kingdom
One Liberty Plaza, 20th Floor, New York, NY 10006, USA
477 Williamstown Road, Port Melbourne, VIC 3207, Australia
314–321, 3rd Floor, Plot 3, Splendor Forum, Jasola District Centre, New Delhi – 110025, India
79 Anson Road, #06–04/06, Singapore 079906

Cambridge University Press is part of the University of Cambridge.


It furthers the University’s mission by disseminating knowledge in the pursuit of
education, learning, and research at the highest international levels of excellence.

www.cambridge.org
Information on this title: www.cambridge.org/9781108465830
DOI: 10.1017/9781108566100
© Cambridge University Press 2019
This publication is in copyright. Subject to statutory exception
and to the provisions of relevant collective licensing agreements,
no reproduction of any part may take place without the written
permission of Cambridge University Press.
First published 2019
Printed and bound in Great Britain by Clays Ltd, Elcograf S.p.A.
A catalogue record for this publication is available from the British Library.
Library of Congress Cataloging-in-Publication Data
Names: Ebrahim, Hozefa, editor.
Title: 1,000 practice MTF MCQs for the primary and final FRCA / edited by Hozefa Ebrahim
[and six others].
Other titles: 1000 practice MTF MCQs for the primary and final FRCA | One thousand practice
MTF MCQs for the primary and final FRCA
Description: Cambridge, United Kingdom ; New York, NY : Cambridge University Press,
2019. | Includes index.
Identifiers: LCCN 2018037315 | ISBN 9781108465830 (paperback)
Subjects: | MESH: Royal College of Anaesthetists (Great Britain) | Anesthesia – methods |
Anesthetics – pharmacology | United Kingdom | Examination Questions
Classification: LCC RD81 | NLM WO 218.2 | DDC 617.9/6–dc23
LC record available at https://lccn.loc.gov/2018037315
ISBN 978-1-108-46583-0 Paperback
Cambridge University Press has no responsibility for the persistence or accuracy of
URLs for external or third-party internet websites referred to in this publication
and does not guarantee that any content on such websites is, or will remain,
accurate or appropriate.

.........................................................................................................................................................................................
Every effort has been made in preparing this book to provide accurate and up-to-date information that is in
accord with accepted standards and practice at the time of publication. Although case histories are drawn
from actual cases, every effort has been made to disguise the identities of the individuals involved.
Nevertheless, the authors, editors, and publishers can make no warranties that the information contained
herein is totally free from error, not least because clinical standards are constantly changing through
research and regulation. The authors, editors, and publishers therefore disclaim all liability for direct or
consequential damages resulting from the use of material contained in this book. Readers are strongly
advised to pay careful attention to information provided by the manufacturer of any drugs or equipment
that they plan to use.
Downloaded from https://www.cambridge.org/core. University of Edinburgh, on 19 Aug 2019 at 13:21:27, subject to the Cambridge Core terms of
use, available at https://www.cambridge.org/core/terms. https://doi.org/10.1017/9781108566100
Contents
List of Contributors vi
Foreword by Dr Tina McLeod ix
Preface xi
Acknowledgements xii
List of Abbreviations xiii

1a Physiology Questions 1 4b Physics Answers 299


1b Physiology Answers 33 5a Clinical Anaesthesia Questions 334
2a Anatomy Questions 99 5b Clinical Anaesthesia Answers 426
2b Anatomy Answers 133
3a Pharmacology Questions 178
3b Pharmacology Answers 210 Index 547
4a Physics Questions 264

v
Downloaded from https://www.cambridge.org/core. University of Edinburgh, on 19 Aug 2019 at 13:21:27, subject to the Cambridge Core terms of
use, available at https://www.cambridge.org/core/terms. https://doi.org/10.1017/9781108566100
Contributors
Suji Abraham Nick Dodds
Consultant Anaesthetist Specialist Trainee in Anaesthesia
Worcestershire Acute Hospitals and Intensive Care Medicine
NHS Trust Severn Deanery

Irmeet Banga Laura Dyal


Specialist Registrar in Anaesthesia Specialist Registrar in Anaesthesia
West Midlands Deanery West Midlands Deanery
Rajneesh Bankenahally Hozefa Ebrahim
Consultant Anaesthetist Consultant in Anaesthesia
Heart of England NHS Trust University Hospitals, Birmingham
Natish Bindal Ian Ewington
Consultant Anaesthetist Consultant Anaesthetist
Queen Elizabeth Hospital Birmingham Queen Elizabeth Hospital Birmingham
Lowri Bowen Richard Hodgson
Consultant Anaesthetist Consultant Anaesthetist
Children’s Hospital for Wales, Cardiff The County Hospital, Hereford

Toni Brunning Chaitra Holla


Specialist Registrar in Anaesthesia Specialist Registrar in Anaesthesia
Worcestershire Acute NHS Trust Heart of England
NHS Trust
Michael Clarke
Consultant Anaesthetist Phillip Howells
Worcestershire Acute Hospitals NHS Trust Specialist Trainee
Birmingham School of Anaesthesia
Ed Copley Health Education England, West Midlands
Consultant Anaesthetist
Northamptonshire Hussein Khambalia
Specialist Trainee in Transplant Surgery
Satinder Dalay Health Education England,
ST7 Anaesthetics North West
Birmingham School of Anaesthesia
Laura Kocierz
Matt Davies Specialist Registrar in Anaesthesia
Consultant Anaesthetist West Midlands Deanery
Russells Hall Hospital, Dudley

vi
Downloaded from https://www.cambridge.org/core. University of Edinburgh, on 19 Aug 2019 at 13:21:27, subject to the Cambridge Core terms of
use, available at https://www.cambridge.org/core/terms. https://doi.org/10.1017/9781108566100
List of Contributors vii

Sajith Kumar University Hospital of Coventry and


Consultant Anaesthetist Warwickshire
Heart of England NHS Trust
Philip Pemberton
Ramy Labib Specialist Registrar in Anaesthesia
Consultant Anaesthetist West Midlands Deanery
Worcestershire Acute Hospitals
NHS Trust Anna Pierson
Consultant Anaesthetist
Nicholas Lascelles The Dudley Group NHS Foundation Trust
Specialist Registrar in Anaesthesia
Gloucester Royal Hospital Richard Pierson
Consultant Anaesthetist
Katherine Laver The Dudley Group NHS Foundation
Specialist Trainee in Anaesthesia Trust
and Intensive Care Medicine
West Midlands Deanery Jane Pilsbury
Consultant Anaesthetist
Peter Lax University Hospitals Birmingham
Consultant in Anaesthetics and
Intensive Care Medicine Nagendra Pinnamaneni
RAF Tactical Medical Wing Specialist Registrar in Anaesthesia
East Midlands Deanery
Adam Low
Consultant Anaesthetist Priya Ramchandran
Queen Elizabeth Hospital, Birmingham Specialist Registrar in Anaesthesia
West Midlands Deanery
Vivienne Madden
Specialist Registrar in Anaesthesia Carla Richardson
West Midlands Deanery Consultant Anaesthetist and
Critical Care
Harsha Mistry University Hospitals Birmingham
Specialist Registrar in Anaesthesia
West Midlands Deanery Karim Rizkallah
ST8, General Surgery
Rachel Moore North West Deanery
Consultant Anaesthetist
University Hospitals Birmingham Ahmed Salama
Specialist Trainee in Anaesthesia
Singaraselvan Nagarajan West Midlands Deanery
Associate Consultant, Women’s
Anaesthesia Sandeep Sharma
KK Women’s and Children’s Hospital, Specialty Doctor
Singapore Birmingham Heartlands Hospital

Rajen Nathwani Naginder Singh


Consultant in Anaesthesia and Intensive Consultant Anaesthetist
Care Medicine Queen Elizabeth Hospital
Birmingham

Downloaded from https://www.cambridge.org/core. University of Edinburgh, on 19 Aug 2019 at 13:21:27, subject to the Cambridge Core terms of
use, available at https://www.cambridge.org/core/terms. https://doi.org/10.1017/9781108566100
viii List of Contributors

Insiya Susnerwala Laura Tulloch


Specialist Trainee in Anaesthesia Consultant in Anaesthesia and
Health Education England North West Intensive Care Medicine
Worcester Acute Hospitals NHS Trust
Robert Tipping
Consultant Anaesthetist Elenor Whittingham
Queen Elizabeth Hospital Specialist Registrar in Anaesthesia
Birmingham West Midlands Deanery

Downloaded from https://www.cambridge.org/core. University of Edinburgh, on 19 Aug 2019 at 13:21:27, subject to the Cambridge Core terms of
use, available at https://www.cambridge.org/core/terms. https://doi.org/10.1017/9781108566100
Foreword
There have been a number of single best answer books published recently, but there is a
paucity of new true–false multiple-choice books. Whilst the internet provides a welcome
educational resource, it is often unregulated and of variable quality and a book such as this,
which is accurately researched, is a valuable addition to the bookshelf. The ethos of this book
is problem-based learning, which has many advantages over the traditional textbook in that
it provides information in digestible bite-sized chunks.
This book has 1000 true–false multiple choice questions. The 600 basic science questions
comprise 150 in each of anatomy, physiology, pharmacology and physics and will be useful
for candidates sitting both the primary and the final FRCA. The 400 clinical questions are
geared toward final FRCA candidates, making this a unique MCQ book which can be used
throughout the examination journey.
Whilst the questions are useful for exam practice, the answers provide a wealth of
information, including key diagrams, and this publication is therefore a useful textbook
in its own right. It can be used by trainees and trainers as a base of knowledge for viva
practice and should be available in every department.
I congratulate Dr Ebrahim and his co-authors on the production of this book – which I
strongly recommend to all anaesthetists.

Dr Tina McLeod MBBS FRCA


Consultant Anaesthetist, Heart of England NHS Foundation Trust

ix
Downloaded from https://www.cambridge.org/core. University of Edinburgh, on 19 Aug 2019 at 13:21:26, subject to the Cambridge Core terms of
use, available at https://www.cambridge.org/core/terms. https://doi.org/10.1017/9781108566100.001
Downloaded from https://www.cambridge.org/core. University of Edinburgh, on 19 Aug 2019 at 13:21:26, subject to the Cambridge Core terms of
use, available at https://www.cambridge.org/core/terms. https://doi.org/10.1017/9781108566100.001
Preface
Revising for exams can be a period of mixed emotions. Some enjoy the challenge of learning
new material, but a great many find it a time of stress. Let us make that time easier for you.
One quiet afternoon in the coffee room, I heard some of my dear trainees stressing over
some bad questions. It is true that some questions are poorly written – not in this book, I
hope, as all of our questions have been written by seasoned educationalists and peer
reviewed by many exam candidates – but nevertheless, books and the internet are littered
with ambiguous questions. Indeed, the right answer can change with time. However, I tried
to reassure them that any question that has caused them to discuss these ambiguities would
surely have resulted in them gaining more knowledge. These words appeared to help,
although I knew that any added stress at this difficult time was far from welcome.
Studying for exams is as much about having the right positive attitude as it is about
cramming information!
The basic sciences for the FRCA exam are well defined. This book has 150 questions for
each of the four basic sciences – anatomy, physics, pharmacology and physiology. The
questions have been written to cover the entire syllabus. It is our suggestion that you only
start practising MCQs once you have spent appropriate time reading the core material.
Find some quiet time to complete a predetermined number of questions, under exam
conditions. Mark them, and then go through your results. For stems in which you are
scoring 4s and 5s, you clearly have a good grasp of the topic. Pat yourself on the back and
move on. For stems in which you are scoring 3 or less, after reading our explanation, spend
just a few more minutes concentrating on reading more about that topic. We do not advise
going back to the drawing board and spending hours rereading the entire topic, as this will
not be the best use of your time. Five minutes of targeted reading usually yields the majority
of information needed for that question.
Use this technique for the clinical questions as well.
The FRCA examination-setters are not trying to trick you. The MCQ exam is a test of
knowledge. If you find a particular question easy, it is probably because you have got the
knowledge. If a question is difficult, spend some time reading that topic. In our experience,
time well spent always pays off. Keep a positive mental attitude.
That quiet afternoon, a few of us made the decision to compile the best of our questions,
and embark upon another project. I hope this book is helpful to you. And please remember,
one day you’ll be the teacher.
Good luck.

xi
Downloaded from https://www.cambridge.org/core. University of Edinburgh, on 19 Aug 2019 at 13:21:27, subject to the Cambridge Core terms of
use, available at https://www.cambridge.org/core/terms. https://doi.org/10.1017/9781108566100.002
Acknowledgements
Firstly, I extend my gratitude to Dr Syedna Mohammed Burhanuddin for all his wisdom
throughout my life. Without him, I would not be where I am.
So many people have given us support along the way, in many different guises; I thank
those who have drafted questions and explanations, proof-read our work, given suggestions
for the content, given us encouragement along the way, and kept the project going.
Thank you!
To consultants, programme training directors, regional advisors, trainees and jobbing
consultants who have given us inspiration, experience and education throughout our years
as doctors.
To Ellie Whittingham for her help with the illustrations. Ellie is a perfect combination of
scientist and artist.
To Mike, Richard, Anna, Hussein, Insiya and Natish for being good friends and excellent
authors. Thanks for tolerating my incessant emails, phone calls and corridor-pestering!
Finally, thank you to all our families for allowing us to hide in our studies typing away.
Tasneem, Mustafa and Farida Ebrahim, Charlotte Norris, Amelia and James Clarke, George
and Henry Pierson, Umme-Hani and Abbas Khambalia, Sudesh and Munishwar Bindal,
thank you!

xii
Downloaded from https://www.cambridge.org/core. University of Edinburgh, on 19 Aug 2019 at 13:21:28, subject to the Cambridge Core terms of
use, available at https://www.cambridge.org/core/terms. https://doi.org/10.1017/9781108566100
Abbreviations
A&E accident and emergency
AAA abdominal aortic aneurysm
AAGBI Association of Anaesthetists of Great Britain and Ireland
ABG arterial blood gas
ABP arterial blood pressure
ACE angiotensin-converting enzyme
ACh acetylcholine
ACT activated clotting time
ACTH adrenocorticotrophic hormone
ADCC antibody-dependent cell-mediated cytotoxicity
ADH antidiuretic hormone
ADHD attention deficit hyperactivity disorder
AFE amniotic fluid embolism
AFLP acute fatty liver of pregnancy
AFOI awake fibreoptic intubation
AKI acute kidney injury
ALF acute liver failure
ALI acute lung injury
ALP alkaline phosphatase
ALS advanced life support
ALT alanine aminotransferase
AMPA α-amino-3-hydroxy-5-methyl-4-isoxazolepropionic acid
ANP atrial natriuretic peptide
ANS autonomic nervous system
AOP apnoea of prematurity
APACHE Acute Physiology And Chronic Health Evaluation
APTT activated partial thromboplastin time
ARDS acute respiratory distress syndrome
AS aortic stenosis
ASA American Society of Anesthesiologists
ASD atrial septal defect
ASIS anterior superior iliac spine
AST aspartate aminotransferase
ATLS advanced trauma life support
ATP adenosine triphosphate
AV atrioventricular
BBB blood–brain barrier
BCIS bone cement implantation syndrome
BG blood glucose
BiPAP bilevel positive airway pressure
BIS bispectral index
BMI body mass index
BMR basal metabolic rate
xiii
Downloaded from https://www.cambridge.org/core. University of Edinburgh, on 19 Aug 2019 at 13:21:26, subject to the Cambridge Core terms of
use, available at https://www.cambridge.org/core/terms. https://doi.org/10.1017/9781108566100
xiv List of Abbreviations

BP blood pressure
BSA burn surface area
BTS British Thoracic Society
BZD benzodiazepine
CABG coronary artery bypass graft
cAMP cyclic adenosine monophosphate
CBF cerebral blood flow
CDH congenital diaphragmatic hernia
CEPOD Confidential Enquiry into Perioperative Deaths
CFAM cerebral function analyzing monitor
cGMP cyclic guanosine monophosphate
CHEOPS Children’s Hospital of Eastern Ontario Pain Scale
CIM critical illness myopathy
CIP critical illness polyneuropathy
cLMA classic laryngeal mask airway
CLP cleft lip and palate
CMAP compound muscle action potential
CMRO2 cerebral metabolic oxygen requirement
CMV cytomegalovirus
CN cranial nerve
CNB central neuraxial block
CNS central nervous system
CO cardiac output
COETT cuffed oral endotracheal tube
COHb carboxyhaemoglobin
COMT catechol-О-methyl transferase
COPD chronic obstructive pulmonary disease
COX cyclo-oxygenase
CP cerebral palsy
CPAP continuous positive airway pressure
CPET cardiopulmonary exercise testing
CPD citrate phosphate dextrose
CPR cardiopulmonary resuscitation
CPSP chronic postsurgical pain
CRF continuous radiofrequency
CRH corticotropin-releasing hormone
CRMO2 cerebral metabolic rate for oxygen
CRPS complex regional pain syndrome
CRT cathode ray tube
CS caesarean section
CSE combined spinal–epidural
CSF cerebrospinal fluid
CT computerized tomography
CTG cardiotocography
CTPA computerized tomography pulmonary angiography
CTZ chemoreceptor trigger zone
CVC central venous catheter

Downloaded from https://www.cambridge.org/core. University of Edinburgh, on 19 Aug 2019 at 13:21:26, subject to the Cambridge Core terms of
use, available at https://www.cambridge.org/core/terms. https://doi.org/10.1017/9781108566100
List of Abbreviations xv

CVO combined ventricular output


CVP central venous pressure
CVS cardiovascular
CXR chest X-ray
CYP cytochrome P450
DA ductus arteriosus
DAG diacylglycerol
DAS Difficult Airway Society
DI diabetes insipidus
DJ duodenojejunal
DLCO diffusing capacity of the lungs for carbon monoxide
DLT double lumen tube
DNA deoxyribonucleic acid
DOPA dihydroxyphenylalanine
DP dorsalis pedis
2,3-DPG 2,3-diphosphoglycerate
DPP4 dipeptidyl peptidase 4
DVT deep vein thrombosis
DXA dual-energy X-ray absorptiometry
EBP epidural blood patch
ECF extracellular fluid
ECG electrocardiogram
ECMO extracorporeal membrane oxygenation
ECST European Carotid Surgery Trial
ECT electroconvulsive therapy
ED emergency department
ED50 effective dose 50%
EDV end diastolic volume
EEG electroencephalogram
EMG electromyography
EMLA eutectic mixture of local anaesthetics
ENT ear nose and throat
ESV end systolic volume
ESWL extracorporeal shock-wave lithotripsy
ETCO2 end tidal carbon dioxide
ETT endotracheal tube
EVAR endovascular aortic aneurysm repair
FAD+ flavin adenine dinucleotide (oxidized form)
FADH flavin adenine dinucleotide (reduced form)
FBC full blood count
FCP final common pathway
FDG fluorodeoxyglucose
FEF forced expiratory flow
FEV1 forced expiratory volume in 1 second
FFP fresh frozen plasma
FiO2 fraction of inspired oxygen
FRC functional residual capacity

Downloaded from https://www.cambridge.org/core. University of Edinburgh, on 19 Aug 2019 at 13:21:26, subject to the Cambridge Core terms of
use, available at https://www.cambridge.org/core/terms. https://doi.org/10.1017/9781108566100
xvi List of Abbreviations

FSH follicle-stimulating hormone


FVC forced vital capacity
GA general anaesthetic
GABA γ amino-butyric acid
GBS Guillain–Barré syndrome
GCS Glasgow coma score
GDP guanosine diphosphate
GFR glomerular filtration rate
GH growth hormone
GHB γ-hydroxybutyrate
GI gastrointestinal
GLP glucagon-like peptide
GnRH gonadotropin-releasing hormone
GORD gastro-oesophageal reflux disease
GPCR G-protein-coupled receptor
GTN glyceryl trinitrate
GTP guanosine triphosphate
GU genitourinary
HALF hyperacute liver failure
Hb haemoglobin
HbA adult haemoglobin
HbF fetal haemoglobin
hCG human chorionic gonadotropin
HDL high-density lipoprotein
HDU high-dependency unit
HELLP haemolysis, elevated liver enzymes and low platelets
HES hydroxyethyl starch
HFOV high-frequency oscillatory ventilation
HII high-impact interventions
HIV human immunodeficiency virus
HLA human leucocyte antigen
HLHS hypoplastic left heart syndrome
HME heat and moisture exchanger
HOCM hypertrophic obstructive cardiomyopathy
hPL human placental lactogen
HPV hypoxic pulmonary vasoconstriction
HR heart rate
HRT hormone replacement therapy
5-HT 5-hydroxytryptamine
HZ herpes zoster
IABP intra-aortic balloon pump
IAP intra-abdominal pressure
IBW ideal body weight
ICA internal carotid artery
ICD intercostal chest drain
ICF intracellular fluid
ICP intracranial pressure

Downloaded from https://www.cambridge.org/core. University of Edinburgh, on 19 Aug 2019 at 13:21:26, subject to the Cambridge Core terms of
use, available at https://www.cambridge.org/core/terms. https://doi.org/10.1017/9781108566100
List of Abbreviations xvii

ICU intensive care unit


IE infective endocarditis
IF intrinsic factor
IJV internal jugular vein
IM intramuscular
INR international normalized ratio
IOP intraocular pressure
IP3 inositol triphosphate
IPPV intermittent positive pressure ventilation
ISF interstitial fluid
ITP3 inositol triphosphate
IV intravenous
IVC inferior vena cava
IVF intravascular fluid
LA local anaesthetic
LAD left anterior descending artery
LAP left atrial pressure
LBBB left bundle branch block
LD50 lethal dose 50%
LDH lactate dehydrogenase
LFJV low-frequency jet ventilation
LH luteinizing hormone
LIF left iliac fossa
LMA laryngeal mask airway
LMWH low molecular weight heparin
LOAF lateral two lumbricals, opponens pollicis, abductor pollicis brevis and flexor
pollicis brevis
LOR loss of resistance
LOS lower oesophageal sphincter
LRA locoregional anaesthesia
LV left ventricle
LVEDV left ventricular end diastolic volume
LVEF left ventricular ejection fraction
LVESV left ventricular end systolic volume
LVH left ventricular hypertrophy
LVRS lung volume reduction surgery
MA maximum amplitude
MAC minimum alveolar concentration
MAO monoamine oxidase
MawP mean airway pressure
MDMA 3,4-methylenedioxy-N-methylamphetamine (Ecstacy)
MEN multiple endocrine neoplasia
MEP motor evoked potentials
MET metabolic equivalent
MG myasthenia gravis
MH malignant hyperpyrexia
MHRA Medicines and Healthcare Products Regulatory Agency

Downloaded from https://www.cambridge.org/core. University of Edinburgh, on 19 Aug 2019 at 13:21:26, subject to the Cambridge Core terms of
use, available at https://www.cambridge.org/core/terms. https://doi.org/10.1017/9781108566100
xviii List of Abbreviations

MI myocardial infarction
MPAP mean pulmonary artery pressure
MR magnetic resonance
MRI magnetic resonance imaging
NAD+ nicotinamide adenine dinucleotide (oxidized form)
NADH nicotinamide adenine dinucleotide (reduced form)
NAP3 Third National Audit Project
NASCET North American Symptomatic Carotid Endarterectomy Trial
NCA nurse-controlled analgesia
NDMR non-depolarizing muscle relaxant
NDNMB non-depolarizing neuromuscular block
NEC necrotizing enterocolitis
NG nasogastric
NICU neonatal intensive care unit
NK neurokinin
NKCC Na-K-2Cl co-transporter
NMDA N-methyl-D-aspartate
NMJ neuromuscular junction
NNBC National Network for Burn Care
NNT number needed to treat
NR Reynold’s number
NRS numerical rating scale
NSAID non-steroidal anti-inflammatory drug
NTS nucleus tractus solitarius
NYHA New York Heart Association
OA osteoarthritis
ODP operating department practitioner
OHDC oxygen–haemoglobin dissociation curve
OLV one-lung ventilation
ORIF open reduction internal fixation
OSA obstructive sleep apnoea
PA pulmonary artery
PAC pulmonary artery catheter
paCO2 arterial partial pressure of carbon dioxide
pACO2 alveolar partial pressure of carbon dioxide
PAFC pulmonary artery flotation catheter
paO2 arterial partial pressure of oxygen
pAO2 alveolar partial pressure of oxygen
PAP pulmonary arterial pressure
PAWP pumonary artery wedge pressure
PCA patient-controlled analgesia
PCEA patient-controlled epidural analgesia
PCI percutaneous coronary intervention
pcjO2 conjunctival oxygen tension
PCWP pulmonary capillary wedge pressure
PD Parkinson’s disease
PDA patent ductus arteriosus

Downloaded from https://www.cambridge.org/core. University of Edinburgh, on 19 Aug 2019 at 13:21:26, subject to the Cambridge Core terms of
use, available at https://www.cambridge.org/core/terms. https://doi.org/10.1017/9781108566100
List of Abbreviations xix

PDE phosphodiesterase
PDPH post-dural-puncture headache
PEEP positive end-expiratory pressure
PEFR peak expiratory flow rate
PET positron emission tomography
PFO patent foramen ovale
PGA postgestational age
PGE2 prostaglandin E2
PGI2 prostacyclin
PH pulmonary hypertension
PHN postherpatic neuralgia
PICU paediatric intensive care unit
PIP2 phosphatidylinositol
PMCS perimortem caesarean section
PNMT phenylethanolamine N-methyl transferase
PNS peripheral nervous system
POCD postoperative cognitive dysfunction
PONV postoperative nausea and vomiting
PPAR peroxisome proliferator-activated receptor
PPH postpartum haemorrhage
PPI proton pump inhibitor
ppoFEV1% predicted postoperative FEV1 percentage
PRF pulsed radiofrequency
PRL prolactin
PSIS posterior superior iliac spine
PSNS parasympathetic nervous system
PT prothrombin time
PTH parathyroid hormone
PVR pulmonary vascular resistence
QAI quaternary ammonium ion
QTc corrected QT
RA right atrium
RCOG Royal College of Obstetricians and Gynaecologists
REM rapid eye movement
RER respiratory exchange ratio
RF radiofrequency
RMP resting membrane potential
RMS root mean square
RNA ribonucleic acid
ROS reactive oxygen species
ROSC return of spontaneous circulation
RQ respiratory quotient
RR respiratory rate
RRT renal replacement therapy
RSI rapid sequence induction
rSO2 regional cerebral oxygen saturation
RTA road traffic accident

Downloaded from https://www.cambridge.org/core. University of Edinburgh, on 19 Aug 2019 at 13:21:26, subject to the Cambridge Core terms of
use, available at https://www.cambridge.org/core/terms. https://doi.org/10.1017/9781108566100
xx List of Abbreviations

RUL right upper lobe


RUQ right upper quadrant
RV right ventricle
RVLM rostral ventrolateral medulla
RVOT right ventricular outflow obstruction
SA sinoatrial
SAD supraglottic airway device
SAH subarachnoid haemorrhage
SCS spinal cord stimulation
SCUF slow continuous ultrafiltration
SE status epilepticus
SIP sympathetically independent pain
SIRS systemic inflammatory response syndrome
SLED sustained low-efficiency dialysis
SMP sympathetically maintained pain
SNAP sensory nerve action potential
SNP sodium nitroprusside
SNR signal-to-noise ratio
SP stump pressure
SPECT single-photon emission computed tomography
SSEP somatosensory evoked potential
STP standard temperature and pressure
SVP saturated vapour pressure
SVR systemic vascular resistance
SVT supraventricular tachycardia
T3 triiodothyronine
T4 thyroxine
TACO transfusion-associated circulatory overload
TAP transversus abdominis plane
TBSA total body surface area
TCI target-controlled infusion
TCD transcranial Doppler ultrasound
TEG thromboelastogram
TENS transcutaneous electrical nerve stimulation
TFPI tissue factor pathway inhibitor
TH cells T-helper cells
TIA transient ischaemic attack
TIVA total intravenous anaesthesia
TLC total lung capacity
TMJ temporomandibular joint
TN trigeminal neuralgia
TOF train of four or tracheo-oesophageal fistula
TPN total parenteral nutrition
TRALI transfusion-related acute lung injury
TRAM transverse rectus abdominis myocutaneous
TRH thyrotropin-releasing hormone
TRVP transient receptor potential vanilloid

Downloaded from https://www.cambridge.org/core. University of Edinburgh, on 19 Aug 2019 at 13:21:26, subject to the Cambridge Core terms of
use, available at https://www.cambridge.org/core/terms. https://doi.org/10.1017/9781108566100
List of Abbreviations xxi

TSH thyroid-stimulating hormone


TURP transurethral resection of the prostate
UFH unfractionated heparin
URTI upper respiratory tract infection
USS ultrasound scan
VAE venous air embolism
VAS visual analogue scale
VC vital capacity
VD volume of distribution
VF ventricular fibrillation
VRIII variable rate intravenous insulin infusion
VRS visual rating scale
VSD ventricular septal defect
VT ventricular tachycardia
VTE venous thromboembolism
VZV varicella zoster virus
WFNS World Federation of Neurosurgeons
WPW Wolff–Parkinson–White syndrome

Downloaded from https://www.cambridge.org/core. University of Edinburgh, on 19 Aug 2019 at 13:21:26, subject to the Cambridge Core terms of
use, available at https://www.cambridge.org/core/terms. https://doi.org/10.1017/9781108566100
Downloaded from https://www.cambridge.org/core. University of Edinburgh, on 19 Aug 2019 at 13:21:26, subject to the Cambridge Core terms of
use, available at https://www.cambridge.org/core/terms. https://doi.org/10.1017/9781108566100
Chapter
Physiology Questions

1a
Question 1
Regarding cardiac muscle structure, which of the following statements are correct?:
a. The volume of the left ventricle is maximal at the atrial end systolic pause
b. The right coronary artery usually supplies both the right atrium and ventricle, and part
of the left atrium
c. Cardiac muscle cells have one nucleus, but many mitochondria
d. Striated cardiac muscle fibres are more structured than skeletal muscle fibres
e. The sarcoplasmic reticulum sequesters calcium via a Na+/K+-ATPase pump

Question 2
Regarding conduction through the heart:
a. Conduction through the cardiac septum is usually uni-directional from left to right
b. Left bundle branch block usually produces left axis deviation on the 12-lead ECG
c. Right bundle branch block usually produces right axis deviation on the 12-lead ECG
d. Stimulation of the tenth cranial nerve induces slowing of AV conduction
e. Wolff–Parkinson–White syndrome is always associated with an accessory conducting
bundle

Question 3
With regard to the cardiac action potential:
a. Sodium influx via fast sodium channels occurs during phase 0 of the nodal cardiac action
potential
b. The absolute refractory period extends into phase 3 of the action potential
c. The plateau phase is due to a decrease in cell membrane permeability of calcium
d. The Na+/K+ pump is involved in the restoration of ionic gradients in phase 4 of the
nodal action potential
e. Slow L-type Ca2+ channels are involved in both nodal and conduction system action
potentials

Question 4
Regarding automaticity with the sinoatrial (SA) node and the atrioventricular (AV) node:
a. The SA node is principally responsible for the heart’s automaticity
b. The threshold potential for the nodal action potential is –90 mV
1
Downloaded from https://www.cambridge.org/core. University of Edinburgh, on 19 Aug 2019 at 13:21:26, subject to the Cambridge Core terms of
use, available at https://www.cambridge.org/core/terms. https://doi.org/10.1017/9781108566100.003
2 Chapter 1a: Physiology Questions

c. Parasympathetic stimulation of the SA node causes a slowing of heart rate via an increase
in membrane Ca2+ permeability
d. The SA node does not have an absolute refractory period
e. The AV node has a longer phase 4 than the SA node

Question 5
With regard to the cardiac cycle:
a. The first heart sound represents the closure of the aortic valve
b. The second heart sound occurs at the beginning of the T wave on the ECG
c. The peak of left ventricular pressure occurs with the QRS complex on the ECG
d. Ventricular volume begins to increase when the atrioventricular valves open
e. The peak of aortic pressure corresponds with the T wave

Question 6
This question concerns the cardiac cycle – diastole, perfusion, lusitropy:
a. The myocardium is entirely dependent on perfusion occurring during diastole
b. Lusitropy refers to the myocardial relaxation
c. At rest diastole accounts for 0.5 seconds of a cardiac cycle lasting 0.8 seconds
d. Ventricular filling is rapid during early diastole
e. Atrial contraction during late diastole accounts for the majority of end diastolic ven-
tricular volume

Question 7
With respect to the CVP waveform:
a. Irregular cannon ‘a waves’ are due to complete heart block
b. The ‘v wave’ is smaller in tricuspid incompetence
c. Normal CVP is 0–8 mmHg
d. The ‘y descent’ is demonstrating passive ventricular filling
e. The ‘c wave’ is the tallest wave

Question 8
This question concerns the P-V relationship, and the Frank–Starling curve:
a. The Frank–Starling law states that the force of contraction is related to the initial fibre
length
b. The force of myocardial contraction is proportional to the initial fibre length, until an
upper limit is reached
c. Before the mitral valve opens there is a decrease in pressure in the ventricle, but no
change in volume
d. Before aortic valve closure there is an increase in volume with no associated change in
pressure
e. The aortic valve opens when the pressure in the ventricle is lower than that in the aorta

Downloaded from https://www.cambridge.org/core. University of Edinburgh, on 19 Aug 2019 at 13:21:26, subject to the Cambridge Core terms of
use, available at https://www.cambridge.org/core/terms. https://doi.org/10.1017/9781108566100.003
Chapter 1a: Physiology Questions 3

Question 9
With respect to cardiac output formulae:
a. CO = HR × (ESV – EDV)
b. The Fick principle cannot be used to calculate blood flow through the liver
c. Only calculated values are used in the Fick equation
d. The oxygen uptake forms the denominator in the Fick equation
e. Shunts do not affect the accuracy of the calculation of cardiac output via the Fick
principle

Question 10
Regarding preload, afterload and contractility:
a. Afterload is increased by peripheral vasoconstriction
b. Increased afterload causes an increased stroke volume
c. Preload can be likened to end systolic volume
d. Preload can be estimated by measurement of CVP
e. Preload and afterload are the only factors affecting contractility

Question 11
Regarding heart rate and coronary blood flow:
a. The sympathetic outflow controlling heart rate is via T1–T8
b. The right coronary artery is the dominant vessel in half the population
c. The right coronary artery arises from the posterior aortic sinus
d. Atrial natriuretic peptide (ANP) is a vasodilator
e. The nucleus ambiguus is involved in integration of the afferent inputs affecting heart rate
from baroreceptors, chemoreceptors and higher centres

Question 12
With regard to the Valsalva manoeuvre:
a. A square wave response is seen in autonomic neuropathy
b. A diminished chemoreceptor reflex causes the abnormal response in autonomic
neuropathy
c. It can be used to terminate supraventricular tachycardia
d. The fall in blood pressure is exaggerated in patients under spinal anaesthesia
e. It decreases the intensity of most heart murmurs on auscultation

Question 13
With regard to the physiological control of blood pressure:
a. Baroreceptors in the carotid sinus are innervated by the vagus nerve
b. The vasomotor centres are found in the hypothalamus and medulla
c. Higher centres have no influence on the vasomotor centres
d. Low pressure baroreceptors are found in the atria, ventricles and pulmonary vessels
e. The Bainbridge reflex causes a reflex bradycardia

Downloaded from https://www.cambridge.org/core. University of Edinburgh, on 19 Aug 2019 at 13:21:26, subject to the Cambridge Core terms of
use, available at https://www.cambridge.org/core/terms. https://doi.org/10.1017/9781108566100.003
4 Chapter 1a: Physiology Questions

Question 14
With regard to the left ventricular end diastolic volume:
a. In a normal heart it is approximately 30 ml
b. It is a measure of preload
c. It is reduced in exercise
d. It is independent of ventricular compliance
e. It is increased in diastolic heart failure

Question 15
The following will cause arterioles to constrict:
a. Direct injury to the vessel
b. Decreased tissue pH
c. Decreased tissue oxygen tension
d. Thromboxane A2
e. Bradykinin

Question 16
The following mediators cause vasoconstriction in vascular smooth muscle:
a. Epinephrine
b. PGF2α
c. Serotonin
d. PGI2
e. Adenosine

Question 17
The following factors may predispose to turbulent flow within a tube:
a. Small diameter
b. Large diameter
c. High viscosity
d. Low velocity
e. High density

Question 18
The following statements are true regarding the lymphatic system:
a. Lymph contains clotting factors
b. Protein content of lymph is generally more than that of plasma
c. The lymphatic system contains valves
d. Skeletal muscle contraction aids lymphatic flow
e. The thoracic duct is the largest lymphatic vessel

Downloaded from https://www.cambridge.org/core. University of Edinburgh, on 19 Aug 2019 at 13:21:26, subject to the Cambridge Core terms of
use, available at https://www.cambridge.org/core/terms. https://doi.org/10.1017/9781108566100.003
Chapter 1a: Physiology Questions 5

Question 19
Regarding blood flow in arterioles and capillaries:
a. Arterioles are the main site of resistance to blood flow
b. Blood flow in capillaries is pulsatile
c. Capillaries have no smooth muscle
d. Changes in temperature can affect flow
e. Precapillary sphincters have rich sympathetic innervation

Question 20
In a rigid tube:
a. Flow is directly proportional to the fourth power of the radius
b. Flow is inversely proportional to the pressure difference
c. Resistance is directly proportional to the length
d. Resistance is directly proportional to the square of the radius
e. If the radius is doubled, the resistance is increased by 16 times

Question 21
The following lung volumes or capacities can be measured by spirometry:
a. Functional residual capacity
b. Vital capacity
c. Total lung capacity
d. Inspiratory reserve volume
e. Expiratory reserve volume

Question 22
Regarding lung compliance:
a. The normal total lung compliance is 200 cmH2O.ml−1
b. Static compliance is greater than dynamic compliance
c. Compliance is increased when a patient is supine
d. It is determined by the gradient of the pressure–volume curve
e. It is greatly reduced in acute respiratory distress syndrome (ARDS)

Question 23
Regarding respiratory dead space:
a. In dead space, the V/Q ratio is zero
b. As dead space increases, paCO2 falls
c. It is increased by general anaesthesia
d. Total dead space is determined using the Bohr equation
e. It is greater in the apices of the lungs than the bases

Downloaded from https://www.cambridge.org/core. University of Edinburgh, on 19 Aug 2019 at 13:21:26, subject to the Cambridge Core terms of
use, available at https://www.cambridge.org/core/terms. https://doi.org/10.1017/9781108566100.003
6 Chapter 1a: Physiology Questions

Question 24
When referring to work of breathing:
a. It is determined by the area inside a pressure–volume curve
b. Inspiratory work is that which overcomes the elastic recoil of the thoracic wall
c. Expiratory work is that which overcomes airway resistance
d. Work to overcome non-elastic forces is lost as heat
e. Respiratory work increases in a ventilated patient

Question 25
Regarding the functional residual capacity (FRC):
a. It is approximately 30 ml.kg−1
b. Pulmonary vascular resistance is highest at FRC
c. It may be less than the closing capacity
d. It is increased under anaesthesia
e. It is decreased in pregnancy

Question 26
Concerning respiratory mechanics:
a. The diaphragm is responsible for 50% of the air that enters the lungs during spontaneous
respiration
b. A third of the diaphragmatic fibres are slow twitch fibres
c. The transpulmonary pressure is equal to the difference between the pressure within the
lungs and the intrapleural pressure
d. The accessory muscles of respiration serve to stabilize the upper rib cage and to prevent
in-drawing in normal respiration
e. Compliance of the lung is defined as the change in pressure per unit change in volume

Question 27
Concerning surfactant:
a. Before 32–34 weeks’ gestation, its production is inadequate and this predisposes to
respiratory distress syndrome
b. Type II alveolar epithelial cells are responsible for its production
c. Less fluid is drawn from capillaries into alveoli as a result of its action
d. The hysteresis area of the pressure–volume loop is increased as a result of its action in
reducing surface tension
e. Larger alveoli are seen to collapse more readily as a result of its action

Question 28
Regarding ventilation and perfusion matching in the upright lung:
a. From apex to base, ventilation increases; blood flow also increases, but less rapidly
b. The ventilation/perfusion ratio is higher at the apex of the lung and decreases progres-
sively towards the base of the lung

Downloaded from https://www.cambridge.org/core. University of Edinburgh, on 19 Aug 2019 at 13:21:26, subject to the Cambridge Core terms of
use, available at https://www.cambridge.org/core/terms. https://doi.org/10.1017/9781108566100.003
Chapter 1a: Physiology Questions 7

c. The difference in partial pressures between the apex and base of the lung is greater for
carbon dioxide compared with that for oxygen
d. Hypoxaemia that results from ventilation/perfusion inequality can be corrected by an
increase in ventilation
e. Pulmonary emboli result in an increase in the ventilation/perfusion ratio

Question 29
Concerning alveolar ventilation and the alveolar gas equation:
a. At rest, the level of alveolar ventilation is the main determinant of the pO2 of
alveolar gas
b. Hypoventilation always results in an increased arterial pressure of carbon dioxide in the
blood stream
c. The respiratory quotient is calculated by the oxygen consumption divided by the carbon
dioxide production
d. Faced with hyperventilation, it takes longer for pCO2 to reach equilibrium as compared
with pO2
e. Shunt refers to areas of the lungs where ventilation is adequate, but perfusion is
deficient

Question 30
Concerning the distribution of blood flow in the lung described by West:
a. Zone 1 does not exist under normal conditions
b. In zone 2, the difference between alveolar and arterial pressures determines
blood flow
c. In zone 3: Pa > PA > Pv
where Pv = venous pressure, Pa = arterial pressure and PA = alveolar pressure
d. From apex to base, the pressure responsible for driving blood flow increases
e. In zone 2, the arteriovenous pressure difference determines blood flow

Question 31
With reference to intermittent positive pressure ventilation (IPPV):
a. The addition of positive end-expiratory pressure (PEEP) increases the dead space
b. It increases the functional residual capacity (FRC)
c. It reduces V/Q mismatch
d. It results in an increase in antidiuretic hormone (ADH) secretion
e. High airway pressures cause a decrease in pulmonary vascular resistance

Question 32
At high altitude (2500 m above sea level):
a. The FiO2 is 20.9%
b. The oxygen–haemoglobin dissociation curve (OHDC) is moved to the
right initially
c. Hypoxic pulmonary vasoconstriction is beneficial

Downloaded from https://www.cambridge.org/core. University of Edinburgh, on 19 Aug 2019 at 13:21:26, subject to the Cambridge Core terms of
use, available at https://www.cambridge.org/core/terms. https://doi.org/10.1017/9781108566100.003
8 Chapter 1a: Physiology Questions

d. There is increased 2,3-DPG production


e. Polycythaemia is the most effective feature of acclimatization

Question 33
The non-respiratory functions of the lungs include:
a. Immune function mediated by pulmonary alveolar macrophages
b. Epinephrine breakdown
c. Angiotensin I production
d. Fibrinolysis of blood clots in the pulmonary circulation
e. Drug metabolism by the cytochrome p450 system

Question 34
Increased oxygen binding to haemoglobin occurs with:
a. 2,3-DPG
b. HbF
c. Methaemoglobin
d. Bohr effect
e. Haldane effect

Question 35
Central chemoreceptors directly increase minute ventilation in response to:
a. Hypercarbia
b. Hypoxia
c. Acidosis
d. Hyperthermia
e. Anaemia

Question 36
Regarding the haemoglobin buffering system:
a. Haemoglobin is a weak acid
b. It increases plasma bicarbonate
c. It increases plasma chloride
d. It has a pKa of 8.1 when deoxygenated
e. It is facilitated by plasma carbonic anhydrase

Question 37
Prolonged oxygen therapy at atmospheric pressure can cause:
a. Cough
b. Retrolental fibroplasia
c. Pulmonary oedema
d. Renal failure
e. Tremors

Downloaded from https://www.cambridge.org/core. University of Edinburgh, on 19 Aug 2019 at 13:21:26, subject to the Cambridge Core terms of
use, available at https://www.cambridge.org/core/terms. https://doi.org/10.1017/9781108566100.003
Chapter 1a: Physiology Questions 9

Question 38
Acute respiratory failure can be a feature of:
a. Aspirin overdose
b. Tetanus
c. Hypersensitivity pneumonitis
d. Poliomyelitis
e. Guillain–Barré syndrome

Question 39
Diagnostic criteria for acute lung injury include:
a. Acute onset
b. Air bronchograms on chest radiograph
c. Pulmonary artery wedge pressure (PAWP) <20 mmHg
d. Hypoxaemia with paO2 /FiO2 <27
e. Bilateral infiltrates on chest radiograph

Question 40
In chronic respiratory failure, the following are commonly seen on an arterial blood
gas (ABG):
a. paO2 under 8 kPa
b. Bicarbonate greater than 30 mEq.l−1
c. paCO2 greater than 6 kPa
d. Base excess greater than +2
e. COHb >15%

Question 41
The rate of diffusion of a gas through a tissue membrane is:
a. Directly proportional to the surface area of the membrane
b. Inversely proportional to the square root of the thickness of the membrane
c. Directly proportional to the difference in gas partial pressures either side of the
membrane
d. Directly proportional to the solubility of the gas in the tissue
e. Inversely proportional to the square root of the molecular weight of the gas

Question 42
The alveolar–arterial (A–a) oxygen gradient in hypoxaemia:
a. Is normal in alveolar hypoventilation
b. Is elevated at high altitude
c. Is decreased in diffusion defects
d. Is increased in right-to-left shunt
e. Is decreased in V/Q mismatch

Downloaded from https://www.cambridge.org/core. University of Edinburgh, on 19 Aug 2019 at 13:21:26, subject to the Cambridge Core terms of
use, available at https://www.cambridge.org/core/terms. https://doi.org/10.1017/9781108566100.003
10 Chapter 1a: Physiology Questions

Question 43
Regarding the oxyhaemoglobin dissociation curve:
a. The curve is shifted to the right with an increase in pH
b. The curve is shifted to the left in stored blood
c. P50 is shifted to the right in chronic anaemia
d. P50 is shifted to the left in HbS (sickle cell)
e. P50 is shifted to the left in HbF

Question 44
Regarding carbon dioxide transport in blood:
a. CO2 is 20 times more soluble in blood than oxygen
b. The majority of CO2 is transported as bicarbonate
c. About 10% of CO2 is dissolved unchanged in blood
d. CO2 combines with water to form carbonic acid catalyzed by carbonic anhydrase in
plasma
e. Binding of oxygen to haemoglobin reduces its affinity for CO2

Question 45
Regarding peripheral and central chemoreceptors:
a. Central chemoreceptors respond to changes in pO2, pCO2 and [H+]
b. Peripheral chemoreceptors respond to changes in oxygen content
c. Central chemoreceptor sensitivity to CO2 may be lost in chronic lung disease
d. Aortic body chemoreceptors respond to changes in pH, pO2 and pCO2
e. Carotid body response to low pO2 is potentiated by low pH

Question 46
The membrane potential of a neurone at rest:
a. Is more negative on the outside of the cell than the inside
b. Is maintained by the active transport of potassium ions out of the cell and sodium ions
into the cell
c. Is more permeable to potassium ions than sodium ions
d. Is impermeable to anions
e. Is –50 mV

Question 47
The Nernst equation:
a. Calculates the potential difference that any ion would produce if the membrane was
permeable to it
b. Calculates the value of the overall membrane potential
c. Requires knowledge of the absolute temperature
d. Calculates similar potentials to the real potential for all ions
e. Takes into account the electrostatic attraction of impermeable ions

Downloaded from https://www.cambridge.org/core. University of Edinburgh, on 19 Aug 2019 at 13:21:26, subject to the Cambridge Core terms of
use, available at https://www.cambridge.org/core/terms. https://doi.org/10.1017/9781108566100.003
Chapter 1a: Physiology Questions 11

Question 48
Regarding the action potential generated within a nerve cell:
a. It is initiated by the influx of sodium ions
b. Will not occur unless the resting membrane potential rises to over –35 mV
c. It contains a period of hyperpolarization
d. Does not allow for tetany
e. Can only flow in one direction

Question 49
The following are true regarding neuronal action potentials:
a. Depolarization is a rapid process
b. A plateau appears due to the opening of calcium channels
c. There are three distinct phases
d. An inactivated sodium channel cannot reopen until it has returned to near resting
potential
e. The peak membrane potential is approximately +30 mV

Question 50
Concerning the sodium channel:
a. It is an integral membrane protein
b. It consists of four domains around a central pore
c. It is blocked externally by local anaesthetics
d. It only allows sodium to pass through
e. Function is increased by high hydrogen concentration

Question 51
With regard to neurotransmitters:
a. They are always either inhibitory or excitatory
b. They are all proteins
c. They act via specific receptors
d. Excitatory neurotransmitters open sodium channels
e. They are stored in the presynaptic axon terminal of neurones

Question 52
The following are true concerning acetylcholine:
a. It is synthesized in the cytoplasm of the nerve endings
b. It is the neurotransmitter at all sympathetic postganglionic nerve endings
c. It is broken down by plasma cholinesterase
d. It binds to muscarinic receptors at the neuromuscular junction
e. It is involved in memory formation

Downloaded from https://www.cambridge.org/core. University of Edinburgh, on 19 Aug 2019 at 13:21:26, subject to the Cambridge Core terms of
use, available at https://www.cambridge.org/core/terms. https://doi.org/10.1017/9781108566100.003
12 Chapter 1a: Physiology Questions

Question 53
The following are amino-acid neurotransmitters:
a. γ amino-butyric acid (GABA)
b. Substance P
c. Glutamate
d. Histamine
e. Glycine

Question 54
The following are true of γ amino-butyric acid (GABA) receptors:
a. They are all G-protein coupled receptors
b. Propofol is likely to work at GABAA receptors
c. They are only found in the central nervous system
d. Ketamine is likely to work at the GABAB receptor
e. They are present on the postsynaptic membrane of the neurone

Question 55
Regarding neuroreceptors within the autonomic nervous system:
a. Receptors within autonomic ganglia are nicotinic acetylcholine receptors
b. All receptors on effector organs innervated by the parasympathetic nervous system are
adrenergic receptors
c. Receptors on the effector organs innervated by the sympathetic nervous system are
muscarinic acetylcholine receptors
d. Sweat glands have muscarinic acetylcholine receptors
e. There are adrenergic receptors within the adrenal medulla

Question 56
Examples of second messengers include:
a. Cyclic adenosine monophosphate (cAMP)
b. Inositol triphosphate
c. Diacylglycerol
d. Adenylyl cyclase
e. Glycine

Question 57
The following statements about G proteins are true:
a. α1 adrenoceptors are associated with Gi proteins
b. Gi proteins inhibit phospholipase C
c. β1 adrenoceptors are associated with Gs proteins
d. Opiate receptors are associated with Gq proteins
e. Gs proteins activate protein kinase A

Downloaded from https://www.cambridge.org/core. University of Edinburgh, on 19 Aug 2019 at 13:21:26, subject to the Cambridge Core terms of
use, available at https://www.cambridge.org/core/terms. https://doi.org/10.1017/9781108566100.003
Chapter 1a: Physiology Questions 13

Question 58
The consequences of anaerobic respiration include:
a. Activation of the Cori cycle
b. Use of six ATP molecules during gluconeogenesis
c. Increased hepatic lactate metabolism
d. Increased renal lactate metabolism
e. A build-up of NAD+

Question 59
During cellular respiration:
a. ATP releases energy when reduced to ADP
b. Oxidative phosphorylation occurs in the mitochondria
c. ATP formation is limited below the Pasteur point
d. Carbon dioxide is a by-product of the electron transfer chain
e. Erythrocytes generate a large total of the body’s ATP

Question 60
The following statements are true regarding bodily fluids:
a. Intracellular fluid makes up a third of total body water
b. Interstitial fluid is the main component of extracellular fluid
c. Intracellular fluid contributes 60% of the male body weight
d. Compared with a preterm infant, the full-term neonate has a higher total body water
contribution to body weight
e. CSF is hyperchloraemic compared with plasma

Question 61
Regarding the physiology of the kidney:
a. The thick ascending limb of the loop of Henle is lined by cuboidal epithelial cells
b. Active transport reabsorbs at least 40% of filtered sodium
c. The majority of the loops of Henle act to increase medullary tonicity
d. Solute reabsorption occurs predominantly in the loop of Henle
e. The distal convoluted tubule is permeable to water throughout its length

Question 62
Renal blood flow is increased by:
a. High protein diet
b. Prostaglandin
c. Adenosine
d. Sympathetic nerve stimulation
e. Renin

Downloaded from https://www.cambridge.org/core. University of Edinburgh, on 19 Aug 2019 at 13:21:26, subject to the Cambridge Core terms of
use, available at https://www.cambridge.org/core/terms. https://doi.org/10.1017/9781108566100.003
14 Chapter 1a: Physiology Questions

Question 63
Regarding renal blood flow:
a. The kidney receives 20% of the cardiac output
b. The renal medulla receives 20% of renal blood flow
c. The vasa recta lie in the medullary pyramids
d. Capillaries drain into arterioles only in the kidneys
e. Efferent arterioles contain oxygenated blood

Question 64
Regarding the glomerulus and Bowman’s capsule:
a. All the glomeruli are situated within the renal cortex
b. The cells of the basal lamina are fenestrated
c. The filtration fraction is 20%
d. The renal tubular oncotic pressure is approximately 35 mmHg
e. Afferent glomerular hydrostatic pressure is approximately 45 mmHg

Question 65
Daily dietary requirements include:
a. 0.8 g.kg−1.day−1 of protein
b. 0.15 g.kg−1.day−1 of nitrogen
c. Cobalamin to prevent megaloblastic anaemia
d. 0.02 mmol.kg−1.day−1 of calcium
e. 1 mmol.kg−1.day−1 of potassium

Question 66
With regard to glomerular filtration rate:
a. It is calculated by measuring insulin clearance
b. Creatinine clearance is the most accurate measure
c. Is reduced by most anaesthetic agents
d. Is normally 120 ml.min−1
e. Vasopressin reduces GFR

Question 67
With regard to buffers:
a. A buffer solution consists of a strong acid and its conjugate base
b. A buffer functions best if its pKa is within 1 unit of the desired pH
c. Deoxygenated haemoglobin is a more effective buffer than oxygenated
haemoglobin
d. The bicarbonate/carbonic acid buffer system has a pKa of 6.1
e. In chronic renal failure excess acid may be buffered by calcium carbonate in bone

Downloaded from https://www.cambridge.org/core. University of Edinburgh, on 19 Aug 2019 at 13:21:26, subject to the Cambridge Core terms of
use, available at https://www.cambridge.org/core/terms. https://doi.org/10.1017/9781108566100.003
Chapter 1a: Physiology Questions 15

Question 68
With regard to the kidney:
a. Aldosterone is produced by the kidney
b. Vasopressin (ADH) acts on the aquaporin-2 receptor
c. Renin converts angiotensin I to angiotensin II
d. Erythropoietin is only secreted by the kidney
e. 1-α hydroxylase activates vitamin D in the distal tubules

Question 69
With regard to acid–base balance:
a. pH = log10 [H+]
b. Siggaard–Anderson nomogram is used to measure paCO2
c. pH is temperature dependent
d. Base deficit is positive in alkalosis
e. In methanol ingestion the anion gap is likely to be low

Question 70
Regarding compensation in acid–base balance:
a. Renal compensation usually takes several weeks to occur
b. 5% of bicarbonate is reabsorbed in the proximal tubule
c. The lungs excrete more acid than the kidneys
d. Kidneys cannot produce urine with pH <4.4
−1
e. Normal plasma HCO 3 is 30–40 mEq.l

Question 71
Regarding antidiuretic hormone (ADH):
a. Maximal ADH release will result in urinary [Na+] of approx. 600 mmol.l−1
b. Release is increased with nicotine
c. Release is increased with alcohol
d. Deficiency results in nephrogenic diabetes insipidus
e. Causes active reabsorption of water at the collecting ducts

Question 72
The loop of Henle:
a. Extends into the renal medulla
b. The ascending (thick) limb passively extrudes Na+ and is impermeable to water
c. The vasa recta act as countercurrent multipliers
d. Is the site of action of amiloride
e. Tubular fluid leaving the ascending limb is hypotonic to plasma

Downloaded from https://www.cambridge.org/core. University of Edinburgh, on 19 Aug 2019 at 13:21:26, subject to the Cambridge Core terms of
use, available at https://www.cambridge.org/core/terms. https://doi.org/10.1017/9781108566100.003
16 Chapter 1a: Physiology Questions

Question 73
Concerning regulation of plasma potassium:
a. Amiloride causes hypokalaemia
b. Aldosterone release causes decreased plasma K+
c. Acidosis can cause hyperkalaemia
d. 70% of potassium ions are actively reabsorbed in the proximal tubule
e. K+ secretion in the distal tubule is dependent on distal tubular flow rate

Question 74
Concerning sodium handling in the kidney:
a. <1% of filtered sodium is excreted in the urine
b. It is regulated by ADH
c. 70% of filtered sodium is reabsorbed in the loop of Henle
d. Aldosterone promotes sodium reabsorption in the distal tubule
e. Plasma sodium concentration controls effective blood volume

Question 75
Concerning response to haemorrhage, compensatory mechanisms include:
a. Increased ADH release
b. Decreased cardiac output
c. Increased renal sodium reabsorption
d. Inhibition of the renin–angiotensin–aldosterone system
e. Increased atrial natriuretic peptide (ANP) levels

Question 76
Concerning neuronal transmission within the brain and spinal cord:
a. GABA is the most important excitatory neurotransmitter
b. The resting membrane potential of a nerve fibre is maintained by the electropositive
charge on the interior of the cell
c. Amino acids and neuropeptides activate AMPA and NK-1 receptors to produce hyper-
polarization of nerve cells at the dorsal horn
d. Glycine is an antinociceptive transmitter at the dorsal horn
e. The effects of opiates can be potentiated by α-agonists at the spinal level

Question 77
With regards to cerebral functional anatomy:
a. There are six histologically distinct layers of the cerebral cortex
b. Damage to specific areas of the hypothalamus causes loss of a component of
emotion
c. The amygdala is responsible for the subconscious physiological response to an unrelated
somatic stimulation from a previously learned experience

Downloaded from https://www.cambridge.org/core. University of Edinburgh, on 19 Aug 2019 at 13:21:26, subject to the Cambridge Core terms of
use, available at https://www.cambridge.org/core/terms. https://doi.org/10.1017/9781108566100.003
Chapter 1a: Physiology Questions 17

d. Carbon monoxide causes damage to the longitudinal fasciculus causing object agnosia
and prosopagnosia
e. Destruction of the corpus callosum causes alexia, but without agraphia

Question 78
With reference to the blood–brain barrier (BBB) and cerebrospinal fluid:
a. The BBB is a physical barrier made up of zona central cells within the cranium
b. The BBB is permeable to water, CO2 and sodium ions, but not to mannitol
c. 20% of the drainage of fluid from the brain is via the lymphatic system
d. Hypotonic solutions normally cause increased brain water in a healthy individual
e. The choroid plexus produces approximately 500 ml of CSF per day

Question 79
With regards to cerebral blood flow:
a. 15% of the cardiac output is received by the brain
b. 50% of the energy provision to the brain comes from oxidative phosphorylation
c. Both general anaesthesia and hypothermia decrease the cerebral metabolic rate for
oxygen (CMRO2)
d. Subclinical seizure activity, as seen on EEG, increases both CMRO2 and cerebral blood
flow (CBF)
e. Patients who are comatose from local anaesthetic toxicity have an increased CMRO2, but
a decreased CBF

Question 80
Regarding cerebral blood flow (CBF):
a. In a healthy brain, cerebral autoregulation provides a constant CBF between paCO2
measurements of 2 kPa to 8 kPa
b. Severe hypoxia causes cerebral vasodilatation and an increase in CBF
c. Cerebral perfusion pressure is approximately equal to the difference between mean
arterial pressure and the intracranial pressure
d. Inhalational anaesthetics decrease CMRO2, but increase CBF
e. Inhalational anaesthetics abolish vasoconstrictive CO2 reactivity more than intravenous
opiates in patients without head injury

Question 81
Regarding the physiology of pain:
a. ‘Fast’ pain stimuli are those that pass via peripheral nociceptors, along primary
afferent Aδ fibres, into the dorsal horn of the spinal cord, and synapse in laminae I,
V and X
b. Aδ fibres decussate in the medulla oblongata
c. Potassium is the most potent allogenic nociceptive agent, and acts on the bradykinin
receptor

Downloaded from https://www.cambridge.org/core. University of Edinburgh, on 19 Aug 2019 at 13:21:26, subject to the Cambridge Core terms of
use, available at https://www.cambridge.org/core/terms. https://doi.org/10.1017/9781108566100.003
18 Chapter 1a: Physiology Questions

d. C-fibres are unmyelinated and conduct at a speed of 2 m.s−1


e. The periaqueductal area of the midbrain provides the majority of the inhibitory control
of pain

Question 82
Regarding the dorsal column:
a. It is located between the ventral median fissure and the ventral horn of the spinal cord
b. It is the primary component of the dorsal horn
c. It comprises two tracts: the fasciculus cuneatus, and the fasciculus gracilis medially
d. A unilateral lesion at the level of the sixth thoracic vertebra will cause proprioceptive
disturbance in the ipsilateral leg
e. Unilateral dissection of the dorsal nerve roots causes motor paralysis

Question 83
Acetylcholine is a neurotransmitter at:
a. Sweat glands
b. The adrenal medulla
c. The parotid gland
d. Parasympathetic ganglia
e. The neuromuscular junction

Question 84
Regarding neuromodulation techniques for chronic pain relief:
a. These techniques are especially beneficial in patients who have otherwise shown drug-
seeking behaviours with conventional medications
b. Stimulation of Aβ proprioceptive fibres causes suppression of small-diameter, high-
threshold Aδ and C sensory fibres in the dorsal horn
c. An intracranial pain relief technique can involve stimulation of the motor cortex
d. A pulse generator is required for electrical stimulation methods, and is usually located
within the abdominal cavity
e. Randomized controlled trials have shown improvement in pain control with neuro-
modulation techniques

Question 85
With regards to the spinothalamic tract, which of the following are true?:
a. It lies anterolateral to the ventral horn
b. It carries information regarding crude touch
c. It is an ascending pathway
d. It decussates in the medulla oblongata
e. Pain and temperature both have a bias towards the lateral spinothalamic tract, rather
than the anterior tract

Downloaded from https://www.cambridge.org/core. University of Edinburgh, on 19 Aug 2019 at 13:21:26, subject to the Cambridge Core terms of
use, available at https://www.cambridge.org/core/terms. https://doi.org/10.1017/9781108566100.003
Chapter 1a: Physiology Questions 19

Question 86
Regarding the central structures involved in motor control:
a. The motor cortex is found in the frontal lobe
b. The spinocerebellum is concerned with planned execution of voluntary movements
c. In the primary motor cortex, parts of the body performing finer movements have larger
representation
d. The basal ganglia influence involuntary movements
e. Dysdiadochokinesia is a feature of cerebellar dysfunction

Question 87
Regarding the electroencephalogram (EEG):
a. Measures biological signals in the order of approximately 1 mV
b. Has a high amplitude and low frequency in an awake patient
c. Deepening anaesthesia leads to decrease in signal amplitude and frequency
d. The signal from two pairs of electrodes is combined to form the cerebral function
analyzing monitor
e. Cortical θ (theta) waves are always abnormal

Question 88
Regarding auditory evoked potentials:
a. They are signals of very low amplitude
b. The timing of the signal represents the area of brain it arises from
c. They have 24 distinguishable peaks
d. They are significantly affected by intravenous anaesthetic agents
e. They correlate with recall of events under anaesthesia

Question 89
Functions of the hypothalamus include:
a. Control of body temperature
b. Control of sexual activity
c. Control of appetite
d. Control of behaviour
e. Control of sleep

Question 90
Regarding the bispectral index (BIS):
a. It is measured in Hertz (Hz)
b. It requires expert interpretation
c. It predicts movement to surgical stimulus
d. Higher values represent more cortical activity
e. It does not distinguish between causes of decreased brain activity

Downloaded from https://www.cambridge.org/core. University of Edinburgh, on 19 Aug 2019 at 13:21:26, subject to the Cambridge Core terms of
use, available at https://www.cambridge.org/core/terms. https://doi.org/10.1017/9781108566100.003
20 Chapter 1a: Physiology Questions

Question 91
Acetylcholine receptors at the neuromuscular junction in adult mammals:
a. Have a molecular weight of 250 000 Daltons
b. Consist of two α subunits, one β, one γ and one δ
c. Appear outside the neuromuscular junction following denervation injuries
d. Are found in the terminal button of a motor neurone
e. Are G-protein coupled receptors

Question 92
Regarding the basic contractile unit of a skeletal myofibril:
a. It is composed of thick actin filaments and thin myosin filaments
b. Myosin filaments occupy the central part of the sarcomere and comprise the A band
c. Myosin filaments are kept in side-by-side alignment by the M-line
d. The H-zone is the area of myosin filaments in the centre of the sarcomere not overlapped
by actin filaments
e. Tropomyosin is associated with myosin filaments

Question 93
Regarding muscle composition:
a. All muscles have a motor end plate
b. Skeletal muscles act as a syncytium
c. Smooth muscle contains troponin
d. Skeletal muscle cells contain more mitochondria than cardiac muscle cells
e. Calcium plays an important role in the contraction of all muscles

Question 94
Regarding control of muscle movement:
a. Muscle spindles receive sensory and motor input
b. Muscle spindles provide static and dynamic signals
c. Muscle spindles are located in the muscle tendons
d. Golgi tendon organs respond to changes in muscle length
e. Golgi tendon organs protect against mechanical rupture of muscles

Question 95
Regarding stretch reflexes:
a. Anterior spinal nerves contain sensory fibres and posterior nerves contain motor fibres
b. The knee jerk reflex is an example of a polysynaptic reflex
c. Nociceptive stimulation propagates impulses along Aδ and C fibres
d. Monosynaptic reflexes synapse in the dorsal horn of the spinal cord
e. Glutamate is the neurotransmitter released at the synapse within the spinal cord

Downloaded from https://www.cambridge.org/core. University of Edinburgh, on 19 Aug 2019 at 13:21:26, subject to the Cambridge Core terms of
use, available at https://www.cambridge.org/core/terms. https://doi.org/10.1017/9781108566100.003
Chapter 1a: Physiology Questions 21

Question 96
During pregnancy, blood flow is increased to the following organs:
a. Uterus
b. Kidneys
c. Liver
d. Brain
e. Skin

Question 97
The following substances are transferred across the placental membrane by simple
diffusion:
a. Oxygen
b. Glucose
c. Carbon dioxide
d. Amino acids
e. Fatty acids

Question 98
Regarding eclampsia in pregnancy:
a. Up to 45% of eclamptic seizures occur postpartum
b. Seizures are usually self-limiting
c. Magnesium sulfate is the prophylactic agent of choice
d. Eclamptic seizures occur in less than 2% of cases of pre-eclampsia
e. The therapeutic range of magnesium is 3–6 mmol.l−1

Question 99
With reference to pethidine use in labour:
a. Pethidine is a weak acid
b. Pethidine is lipophilic and readily crosses the placenta
c. Norpethidine is a proconvulsant
d. Pethidine inhibits uterine contraction
e. Pethidine has local anaesthetic properties

Question 100
With reference to cardiovascular system changes in pregnancy:
a. Cardiac output increases by up to 50% by the start of the third trimester
b. Stroke volume increases by up to 30%, predominantly due to an increase in contractility
c. Serum colloid osmotic pressure decreases by up to 15–20%
d. During labour the cardiac output may increase by up to a further 20% or more
e. The blood volume returns to normal by seven days postpartum

Downloaded from https://www.cambridge.org/core. University of Edinburgh, on 19 Aug 2019 at 13:21:26, subject to the Cambridge Core terms of
use, available at https://www.cambridge.org/core/terms. https://doi.org/10.1017/9781108566100.003
22 Chapter 1a: Physiology Questions

Question 101
The following drugs are readily transferred across the placenta:
a. Nitrous oxide
b. Sevoflurane
c. Suxamethonium
d. Etomidate
e. Neostigmine

Question 102
The ECG in a pregnant woman may commonly show the following changes:
a. Sinus tachycardia
b. Right axis deviation
c. ST segment depression
d. T wave inversion
e. Atrial fibrillation

Question 103
The following changes occur in the respiratory system during pregnancy:
a. Tidal volume is increased
b. Compensatory respiratory alkalosis is observed
c. Lung compliance is reduced
d. Expiratory reserve volume is increased
e. Respiratory rate is increased

Question 104
Common hepatobiliary system changes in pregnancy may include:
a. Increased hepatic blood flow by up to 20%
b. A 25% reduction in plasma cholinesterase levels
c. A threefold increase in alkaline phosphatase levels
d. Presence of spider naevi and palmar erythema
e. Biliary stasis and increased bile acid production

Question 105
The risk factors associated with the development of pre-eclampsia include:
a. Advanced maternal age
b. Multiparity
c. Obesity
d. Smoking
e. Twins

Downloaded from https://www.cambridge.org/core. University of Edinburgh, on 19 Aug 2019 at 13:21:26, subject to the Cambridge Core terms of
use, available at https://www.cambridge.org/core/terms. https://doi.org/10.1017/9781108566100.003
Chapter 1a: Physiology Questions 23

Question 106
The following hormones are secreted by the placenta:
a. Oestrogen
b. Progesterone
c. Relaxin
d. Thyroxine
e. Human chorionic gonadotropin

Question 107
The following changes may be seen in the renal tract during normal pregnancy:
a. Glomerular filtration rate increases by up to 50%
b. Serum creatinine levels of 90 μmol.l−1 are considered normal
c. Glycosuria is relatively common
d. Kidneys increase in length by 1 cm
e. Plasma osmolality is reduced

Question 108
Apnoeic episodes in a preterm infant can be triggered by:
a. Hypoxia
b. Hypercarbia
c. Sepsis
d. Anaemia
e. Hypoglycaemia

Question 109
Regarding non-shivering thermogenesis in a term infant:
a. Brown fat makes up about 6% of total body weight
b. Brown fat is found in the interscapular region, axilla and mediastinum
c. Brown fat is highly vascular with rich mitochondrial content
d. Non-shivering thermogenesis may double heat production
e. Brown fat is increased in preterm babies

Question 110
With reference to the nervous system in a newborn:
a. It accounts for 10% of body weight at birth
b. Brain size increases threefold during the first year of life
c. Neonatal cerebral circulation receives one-third of the cardiac output
d. The blood–brain barrier is fully mature at six months of age
e. Cerebral autoregulation is relatively well developed at term

Downloaded from https://www.cambridge.org/core. University of Edinburgh, on 19 Aug 2019 at 13:21:26, subject to the Cambridge Core terms of
use, available at https://www.cambridge.org/core/terms. https://doi.org/10.1017/9781108566100.003
24 Chapter 1a: Physiology Questions

Question 111
Which of the following statements are true regarding lung surfactant?
a. Type II pneumocytes secrete lung surfactant
b. Surfactant production in a fetus starts after 24 weeks
c. Maternal steroids can increase surfactant production
d. Alveolar distension inhibits surfactant production
e. Up to 50% of the alveolar epithelial cells are Type II pneumocytes

Question 112
Regarding fetal haemoglobin (HbF):
a. It has 2 α and 2 δ chains
b. The oxyhaemoglobin dissociation curve is shifted to the right
c. HbF is fully replaced by adult haemoglobin (HbA) by 12 months of age
d. At term, HbF makes up to 80% of total haemoglobin
e. P50 of HbF is 3.6 kPa

Question 113
In a term fetus:
a. There is one umbilical artery and two umbilical veins
b. The umbilical artery arises from the iliac artery
c. The umbilical vein empties directly into the inferior vena cava
d. The left ventricle provides 35% of the cardiac output
e. The foramen ovale directs oxygen-rich blood from the inferior vena cava to the left
atrium

Question 114
With regards to exercise in the normal healthy adult:
a. The surface area of the lung is approximately 300 times that of the skin
b. Typically, the respiratory exchange ratio decreases from 1.0 to 0.8 as a subject
exercises
c. An increase in work above the VO2 max can only occur through anaerobic glycolysis
d. The oxygen dissociation curve shifts to the right in muscles during exercise due to an
increase in temperature, pCO2 and H+ concentration
e. At high levels of exercise, the pO2 rises, but the pCO2 and pH fall

Question 115
With regards to breathing at high altitude:
a. Acclimatisation most effectively occurs through hyperventilation
b. An increase in 2,3-diphosphoglycerate concentration causes better oxygen tissue
unloading
c. A lower haematocrit results, to provide better blood flow

Downloaded from https://www.cambridge.org/core. University of Edinburgh, on 19 Aug 2019 at 13:21:26, subject to the Cambridge Core terms of
use, available at https://www.cambridge.org/core/terms. https://doi.org/10.1017/9781108566100.003
Chapter 1a: Physiology Questions 25

d. Pulmonary hypertension and right ventricular hypertrophy are common in high alti-
tude inhabitants
e. Hypoxic pulmonary vasoconstriction provides better V/Q matching during exercise at
altitude

Question 116
Which of the following statements are true, with respect to diving and underwater
physiology?
a. The mass of gas dissolved in body fluids increases with depth
b. The ambient pressure at a depth of 10 m is approximately 1 bar
c. Submersion in cold water increases stroke volume
d. During a ‘breath-hold’ dive, intra-alveolar pressure may become negative
e. Confusion from high-pressure neurological syndrome occurs due to compression of
lipids at great depths

Question 117
With regards to hyperbaric gases:
a. SCUBA stands for self-contained underwater breathing apparatus
b. The use of helium in SCUBA diving decreases the incidence of decompression
sickness
c. Oxygen toxicity can occur at oxygen pressures of 0.5 bar, when diving to depths
of 20 m
d. Decompression sickness symptoms typically occur whilst ascending
e. Decompression sickness occurs as dissolved oxygen forms bubbles in the tissues

Question 118
The following environmental changes occur with altitude:
a. Relative humidity increases with altitude due to mist
b. The saturated water pressure in the lungs remains constant despite increasing
altitude
c. The oxygen concentration of air decreases with altitude
d. The alveolar partial pressures of oxygen and carbon dioxide of a climber breathing air on
top of Everest would be close to zero
e. Barometric pressure is approximately halved for every increase in altitude of 5500 m

Question 119
With regards to the physiological responses to altered temperature:
a. The thermoneutral zone refers to an environment with temperatures ranging between
36 and 38 °C
b. The anterior hypothalamus is concerned with heat production and conservation
c. Glucocorticoids act upon mitochondrial receptors within tissues to increase Na+/K+
ATPase heat production

Downloaded from https://www.cambridge.org/core. University of Edinburgh, on 19 Aug 2019 at 13:21:26, subject to the Cambridge Core terms of
use, available at https://www.cambridge.org/core/terms. https://doi.org/10.1017/9781108566100.003
26 Chapter 1a: Physiology Questions

d. The shivering response is the principal method of thermoregulation in neonates


through utilization of brown adipose fat
e. The shivering response in adults increases 25-fold below a core temperature of 34 °C,
compared to 37 °C

Question 120
When considering the physiology of space travel, which of the following statements are
true?
a. Whilst standing on the launch pad, the blood pressure is lower in the legs than in the
arms
b. On rapid vertical acceleration, cardiac output increases due to increased epinephrine
production
c. On rapid vertical acceleration, hypoxaemia and hypocarbia occur
d. Weightlessness usually increases cardiac output
e. Ventricular hypertrophy is seen in astronauts

Question 121
During metabolism and energy formation:
a. Under aerobic conditions, 36 molecules of ATP can be generated by the metabolism of 1
molecule of glucose
b. Conversion of 1 molecule of glucose to 2 molecules of pyruvate (glycolysis) generates a
net gain of 4 molecules of ATP
c. The Cori cycle requires 6 molecules of ATP to produce 1 molecule of glucose
d. Metabolism of 1 molecule of fatty acid under aerobic conditions generates 44 molecules
of ATP
e. The Cori cycle is another term for the Krebs cycle

Question 122
The following are consequences of starvation:
a. Fall in triiodothyronine levels
b. Initial fall in glycogenolysis
c. Creatinuria
d. Fall in cortisol level
e. A threefold decrease in protein breakdown after three weeks

Question 123
Regarding the liver:
a. Portal veins join to form the hepatic vein
b. Kupffer cells are a part of the reticuloendothelial system
c. 50% of hepatic oxygen delivery occurs via the portal vein
d. Bile drains into the central vein prior to the common bile duct
e. 30% of hepatic blood supply occurs via the hepatic artery

Downloaded from https://www.cambridge.org/core. University of Edinburgh, on 19 Aug 2019 at 13:21:26, subject to the Cambridge Core terms of
use, available at https://www.cambridge.org/core/terms. https://doi.org/10.1017/9781108566100.003
Chapter 1a: Physiology Questions 27

Question 124
Regarding bile salts:
a. Bile salts are formed from cholesterol
b. Bile acids are amphipathic
c. More than 90% of bile salts are absorbed from the small intestine
d. Bile salts are conjugated to increase lipid solubility
e. Bile salts are important in the absorption of vitamins A, B, D, E and K

Question 125
Regarding the metabolism and excretion of bilirubin:
a. The small intestine is impermeable to unconjugated bilirubin
b. Accumulation of free bilirubin causes jaundice
c. Unconjugated bilirubin is secreted in bile
d. Bilirubin is bound to albumin in the plasma
e. Increased levels of urobilinogen are seen in obstructive jaundice

Question 126
Regarding gastric secretion:
a. Chief cells secrete pepsin
b. Oxyntic cells secrete intrinsic factor
c. Hydrochloric acid is important in duodenal calcium absorption
d. Parietal cells are most prevalent in the body and antrum
e. Acetylcholine, histamine and gastrin all stimulate a reduction in stomach pH

Question 127
During vomiting:
a. It begins with a retrograde giant contraction in the small intestine
b. The body of the stomach relaxes
c. The diaphragm contracts
d. Efferent signals from the vomiting centre are carried by cranial nerves V, VII, IX and XII
e. D2 and 5HT3 receptors mediate afferent impulses between the chemoreceptor trigger
zone and the vomiting centre

Question 128
During hypothermia:
a. The posterior hypothalamus is sensitive to cold afferent signals
b. Efferent temperature signals travel via the spinothalamic tract
c. Brown fat is found between the scapulae
d. Shivering increases the basal metabolic rate by up to 10%
e. Temperature is lowest in the evening and peaks in the morning

Downloaded from https://www.cambridge.org/core. University of Edinburgh, on 19 Aug 2019 at 13:21:26, subject to the Cambridge Core terms of
use, available at https://www.cambridge.org/core/terms. https://doi.org/10.1017/9781108566100.003
28 Chapter 1a: Physiology Questions

Question 129
Regarding swallowing:
a. Saliva has a lower potassium concentration than plasma
b. Contraction of cricopharyngeus opens the upper oesophageal sphincter
c. Lower oesophageal sphincter tone is maintained by the right crus of the diaphragm
d. Peristaltic waves in the oesophagus travel at 2–4 cm.s−1
e. The resting pressure of the lower oesophageal sphincter is 15–25 mmHg above gastric
pressure

Question 130
In a normal diet:
a. The caloric value of carbohydrate is 9 kcal.g−1
b. Linoleic, linolenic and arachidonic acids are essential fatty acids
c. A respiratory quotient of 1.0 denotes all energy production is derived from carbohydrate
d. The average protein requirement per day is 0.5–1.0 g.kg−1
e. 1 kilocalorie is equal to 4.18 J of energy

Question 131
With regard to the pituitary gland and its hormones:
a. Its blood supply originates from the circle of Willis
b. The posterior pituitary and the hypothalamus are connected by a portal circulation
c. ADH is synthesized in the hypothalamus
d. The anterior pituitary is regulated directly by hypothalamic neurones
e. Pituitary tumours may cause hydrocephalus

Question 132
With regard to the pituitary hormones:
a. Excessive growth hormone secretion causes gigantism in adults
b. Cushing’s disease is caused by hyposecretion of ACTH
c. Central diabetes insipidus is associated with dehydration and hyponatraemia
d. ADH release is inhibited by alcohol
e. When managing a heart-beating donor it may be necessary to administer synthetic
pituitary hormones

Question 133
With regard to the adrenal cortex:
a. It forms 70% of the adrenal gland
b. Its embryological origins are from the ectodermal cells of the neural crest
c. Aldosterone production only occurs in the zona glomerulosa
d. Glucocorticoids promote gluconeogenesis
e. Mineralocorticoids cause potassium reabsorption from the distal convoluted tubule of
the kidney

Downloaded from https://www.cambridge.org/core. University of Edinburgh, on 19 Aug 2019 at 13:21:26, subject to the Cambridge Core terms of
use, available at https://www.cambridge.org/core/terms. https://doi.org/10.1017/9781108566100.003
Chapter 1a: Physiology Questions 29

Question 134
With regard to the adrenal medulla:
a. It is innervated by preganglionic, cholinergic and parasympathetic nerve fibres
b. It produces equal amounts of noradrenaline and adrenaline
c. The rate-limiting step in catecholamine synthesis is the conversion of DOPA to
dopamine
d. Phenylalanine is the precursor for all catecholamines
e. Conn’s syndrome is caused by adrenal medulla tumours

Question 135
With regard to hormones:
a. Chromaffin cells are an example of neuroendocrine cells
b. Catecholamines are steroid hormones
c. Steroid hormones bind to G-protein coupled receptors
d. Insulin binds with tyrosine kinase receptors
e. Cholesterol is the precursor for all corticosteroids

Question 136
Calcitonin:
a. Is essential to calcium homeostasis
b. Increases plasma phosphate levels
c. Inhibits osteoclasts
d. Decreases plasma calcium
e. Is secreted from the parathyroid gland

Question 137
With regard to calcium homeostasis:
a. 10% of plasma calcium is bound to albumin
b. Hypercalcaemia causes tetany and laryngospasm
c. Parathyroid hormone (PTH) is released from chief cells
d. Vitamin D increases plasma levels of calcium and phosphate
e. PTH decreases plasma calcium and increases plasma phosphate

Question 138
With regard to the thyroid hormones:
a. Thyroxine (T4) is more potent than triiodothyronine (T3)
b. They are 99% protein bound
c. They are synthesized in follicular cells by successive iodination of tryptophan
d. Iodide enters follicular cells via diffusion
e. They increase the number and the sensitivity of β adrenoreceptors

Downloaded from https://www.cambridge.org/core. University of Edinburgh, on 19 Aug 2019 at 13:21:26, subject to the Cambridge Core terms of
use, available at https://www.cambridge.org/core/terms. https://doi.org/10.1017/9781108566100.003
30 Chapter 1a: Physiology Questions

Question 139
Concerning erythropoietin:
a. It is produced in the liver
b. It is expressed in cells in the renal medulla
c. Levels are reduced in acute renal failure
d. Therapy is associated with thrombosis
e. It causes increased red cell survival

Question 140
With regard to the pancreas:
a. Insulin is a catabolic hormone
b. Insulin is essential for glucose uptake in the brain
c. Normal insulin production is 40–50 units.day−1
d. D cells secrete pancreatic polypeptide
e. Somatostatin inhibits release of insulin and glucagon

Question 141
Regarding ABO blood groups and antigens:
a. Group A occurs with a frequency of 24% in the UK population
b. Group B occurs with a frequency of 8% in the UK population
c. Rhesus D antibodies are IgG antibodies
d. Donor blood of group A can be safely administered to a recipient whose blood group
is O
e. There are no naturally occurring antibodies in the serum of individuals with blood
group AB

Question 142
Regarding the pathways of coagulation:
a. The classical model incorporating the extrinsic and intrinsic pathways of clotting
represents the currently accepted model of clotting in vivo
b. A prolonged APTT is always predictive of a clinical bleeding tendency
c. The INR is sensitive to deficiencies in all of the following: FI, FII, FV, FVII, FX
d. The amplification phase of the cell-based model of clotting includes the ‘thrombin
burst’
e. Tissue factor pathway inhibitor (TFPI) contributes to the prevention of overwhelming
pathological thrombosis

Question 143
Regarding the production and function of antibodies:
a. The interaction of B cells with cytotoxic T cells results in the production of plasma cells
capable of antibody secretion

Downloaded from https://www.cambridge.org/core. University of Edinburgh, on 19 Aug 2019 at 13:21:26, subject to the Cambridge Core terms of
use, available at https://www.cambridge.org/core/terms. https://doi.org/10.1017/9781108566100.003
Chapter 1a: Physiology Questions 31

b. Their constituent heavy chain portion determines the antigen-binding site of


antibodies
c. Rearrangement of variable, joining and diversity genes at the DNA level produces
diversity of the constant portion of antibodies
d. A brisk and augmented immune response upon re-exposure to a previously encountered
antigen is termed a secondary response
e. Mononuclear phagocytes are an example of effector cells capable of mediating antibody-
dependent cell-mediated cytotoxicity

Question 144
Regarding the normal constituents and composition of plasma:
a. The total protein content is 7 g.100 ml−1
b. Albumin accounts for 40% of the total protein
c. Immunoglobulins account for 20% of the total protein
d. The total plasma volume is 10% of body weight
e. Osmolality is within the range of 280–305 mosmol.l−1

Question 145
Regarding allergies in anaesthesia:
a. Allergic anaphylaxis is characterized at the molecular level by the cross-linking of IgE
antibodies
b. Cough medicines are a potential source of the sensitizing allergens that may predispose
to anaesthesia-related anaphylaxis
c. In suspected anaphylaxis, serum samples for mast cell tryptase should be taken imme-
diately, at 6 hours and at 24 hours after exposure to the allergen
d. Intradermal testing for potential allergens is less sensitive but more specific as compared
with skin prick tests
e. The clinical features of allergic as compared with non-allergic anaphylaxis can be
identical

Question 146
Concerning physiological changes that occur with advancing age:
a. Total body water and total body fat are increased leading to the prolonged duration of
action of both water- and lipid-soluble drugs
b. Beyond the age of 66, closing volume increases to exceed the functional residual
capacity, resulting in venous admixture
c. Urea and creatinine may be within the normal range representing normal renal function
d. When using volatile anaesthetic agents, a shorter onset time is due to reduced oil–gas
partition coefficients and a reduction in cardiac output
e. Maximal cardiac output with exercise is likely to be reduced by at least 40% in a 95-year-
old patient

Downloaded from https://www.cambridge.org/core. University of Edinburgh, on 19 Aug 2019 at 13:21:26, subject to the Cambridge Core terms of
use, available at https://www.cambridge.org/core/terms. https://doi.org/10.1017/9781108566100.003
32 Chapter 1a: Physiology Questions

Question 147
Concerning anaesthesia and cognitive dysfunction in the elderly:
a. Dementia affects 20% of patients over the age of 80 years
b. At 2 years post major surgery, 10% of elderly patients suffer unresolved postoperative
cognitive dysfunction (POCD)
c. Serum concentrations of neurone-specific enolase correlate with the development of
POCD
d. POCD is less likely to occur in patients with a higher level of intellectual performance
preoperatively
e. Regional anaesthesia provides protection against the development of POCD in the early
postoperative phase

Question 148
In perioperative care of the elderly, regarding the cardiovascular system:
a. Atrial fibrillation results in a reduction in cardiac output of approximately 30%
b. Reduced response to α-agonists is attributed to reduced α receptor sensitivity
c. A fall in blood pressure produces an exaggerated carotid baroreceptor response as
compared with a younger patient
d. At three months, regional anaesthesia for hip surgery is associated with a lower
incidence of deep vein thrombosis compared with general anaesthesia
e. Hypotension at induction is largely the result of impaired autonomic homeostasis

Question 149
The following respiratory physiological parameters are decreased in the elderly:
a. Functional residual capacity
b. Closing capacity
c. Shunt
d. Residual volume
e. Total lung capacity

Question 150
In a 70-year-old male, the following physiological parameters are decreased by more than
25% as compared with a young adult:
a. Muscle mass
b. Cardiac output at rest
c. Cerebral blood flow
d. Oxygen consumption at rest
e. Renal blood flow

Downloaded from https://www.cambridge.org/core. University of Edinburgh, on 19 Aug 2019 at 13:21:26, subject to the Cambridge Core terms of
use, available at https://www.cambridge.org/core/terms. https://doi.org/10.1017/9781108566100.003
Chapter
Physiology Answers

1b
Question 1: TFTFF
Atrial contraction accounts for 10% of ventricular filling at rest. At high heart rates this ratio
increases to 40% due to limited time available for passive filling. After atrial contraction is
complete, the atrial pressure starts to fall. This causes the valve to float upwards before
closure. At this time the ventricular volume is maximal, which is termed the end diastolic
volume (EDV).
The two main coronary arteries are the left main and right coronary arteries. The left
main coronary artery supplies blood to the left side of the heart muscle (the left ventricle and
left atrium). The right coronary artery supplies blood to the right ventricle, the right atrium
and the SA (sinoatrial) and AV (atrioventricular) nodes, which regulate the heart rhythm.
Each cardiac myocyte is surrounded by a cell membrane called the sarcolemma and
contains one nucleus. The cells are packed with mitochondria to provide the steady supply
of ATP required to sustain cardiac contraction. As with skeletal muscle, cardiac myocytes
contain the contractile proteins actin (thin filaments) and myosin (thick filaments), together
with the regulatory proteins troponin and tropomyosin. Cardiac muscle is striated,
although the pattern is not as ordered as in skeletal muscle.
Calcium has an essential role in myocardial contraction. A raised intracellular calcium
concentration is the trigger that activates contraction. Diastolic relaxation is an active (ATP-
dependent) process. Calcium transport out of the cytosol occurs via a sarcoplasmic reticu-
lum Ca2+ ATPase, through sarcolemmal Na+/Ca2+ exchange.

Question 2: TTFTT
The sinoatrial (SA) node, atrioventricular (AV) node, bundle of His and other atrial centres all
have inherent pacemaker activity. From the SA node, impulses spread throughout the atria to
the AV node. Depolarization spreads from the AV node to the bundle of His in the interven-
tricular septum. The bundle splits into right and left bundle branches, supplying the respective
ventricles. As the left bundle branch is activated first, the depolarization proceeds from left to
right and may give rise to a small negative deflection within the ECG, referred to as the Q-wave.
In LBBB, the normal direction of septal depolarization is reversed (becomes right to left),
as the impulse spreads first to the RV via the right bundle branch and then to the LV via the
septum. This sequence of activation extends the QRS duration to >120 ms and eliminates
the normal septal Q waves in the lateral leads. The overall direction of depolarization (from
right to left) produces tall R waves in the lateral leads (I, V5–6) and deep S waves in the right
precordial leads (V1–3), and usually leads to left axis deviation.

33
Downloaded from https://www.cambridge.org/core. University of Edinburgh, on 19 Aug 2019 at 13:21:27, subject to the Cambridge Core terms of
use, available at https://www.cambridge.org/core/terms. https://doi.org/10.1017/9781108566100.004
34 Chapter 1b: Physiology Answers

In RBBB, activation of the right ventricle is delayed as depolarization has to spread across
the septum from the left ventricle. The left ventricle is activated normally, meaning that the
cardiac axis is not affected by RBBB and the early part of the QRS complex is unchanged.
The conduction of electrical impulses throughout the heart, and particularly in the
specialized conduction system, is influenced by the autonomic nervous system. This auto-
nomic control is most apparent at the AV node. Sympathetic activation increases conduc-
tion velocity in the AV node. Parasympathetic (vagal) activation decreases conduction
velocity (negative dromotropy) at the AV node. This leads to slower depolarization of
adjacent cells, and reduced velocity of conduction.
Wolff–Parkinson–White syndrome (WPW) was first described in 1930 by Louis Wolff,
John Parkinson and Paul Dudley White. WPW syndrome is a combination of the presence
of a congenital accessory pathway and episodes of tachyarrhythmia. The incidence rate is
0.1–3.0 per 1000. It is associated with a small risk of sudden cardiac death. It has two
characteristic components: pre-excitation and accessory pathways. Pre-excitation refers to
early activation of the ventricles due to impulses bypassing the AV node via an accessory
pathway. Accessory pathways, also known as bypass tracts, are abnormal conduction path-
ways formed during cardiac development and can exist in a variety of anatomical locations,
and in some patients there may be multiple pathways. In WPW the accessory pathway is
often referred to as the bundle of Kent, or atrioventricular bypass tract.

Question 3: FTFFT
When considering the cardiac action potential it is important to differentiate between the
nodal (pacemaker) action potential (Figure 1.3.1) and the conduction system action poten-
tial (Figure 1.3.2).
They both last approximately 250 ms and have an absolute refractory period (during
which no further action potentials can be generated) and a relative refractory period (during
which an action potential can be initiated by a supramaximal stimulus).

20
Membrane potential (mV)

Phase 0
2+ 2+
L-type Ca channels open (Ca influx)
0
3 Phase 3
0 + +
Repolarization via opening of K (K efflux)
Phase 4
+
Hyperpolarization before K efflux stops
Baseline +drift towards the threshold potential (–40 mV)
Na leak
T-type Ca2+
–40 Na+/Ca2+ pump

–80
0 100 200 300 400
Time (msec)

Figure 1.3.1 Nodal action potential Nodal action potential i.e. the nodal action potential is mostly to do with
changes in permeability to Ca2+ and K+.

Downloaded from https://www.cambridge.org/core. University of Edinburgh, on 19 Aug 2019 at 13:21:27, subject to the Cambridge Core terms of
use, available at https://www.cambridge.org/core/terms. https://doi.org/10.1017/9781108566100.004
Chapter 1b: Physiology Answers 35

30
1
2

Phase 0
0 + +
Fast Na channels open (Na influx)
Phase 1
Membrane potential (mV)

Na+ channels close


0 Some K+ channels open (K+ efflux) →
3 partial repolarization
Phase 2 (plateau phase)
2+ 2+
L-type Ca open (Ca influx)
Phase 3
+ +
K channels open (K efflux)
2+
L-type Ca close
Absolute refractory period Phase 4
Relative refractory Resting membrane potential
+ +
period Na /K pump resets ionic
gradient
4
–90

–100
0 100 200 300 400 500
Time (msec)

Figure 1.3.2 Conduction action potential.

Question 4: TFFFT
Automaticity refers to the ability of a cell to depolarize without stimulus. In the heart the
sinoatrial (SA) node, atrioventricular (AV) node, His–Purkinje system and some cardiac
myocytes contain cells that display automaticity. The SA node is the predominant pace-
maker for the heart. Cells with automaticity do not have a resting membrane potential, but
rather exhibit baseline drift due to Na+ leak, open T-type Ca2+ channels and a Na+/Ca2+
pump. This allows cations to move intracellularly and gradually increases the membrane
potential. At –40 mV an action potential is triggered as the cell reaches the threshold
potential. The SA node can be affected by the sympathetic nervous system to increase or
decrease the heart rate. Sympathetic nervous input (β receptors) causes Ca2+ channels to open,
causing the cells to reach the threshold potential more quickly and thereby increasing the rate
at which the SA node fires, whereas parasympathetic input (muscarinic receptors) causes an
increase in K+ permeability which hyperpolarizes the cell and reduces the rate of SA node
firing.

Question 5: FFFTF
The events of the cardiac cycle are lengthy to describe and therefore they are often charted in
a diagram. This diagram is also known as Wigger’s diagram (Figure 1.5.3). It relates the ECG
trace, heart sounds, pressure waves of the left ventricle and atria, and the aorta, the left
ventricular volumes and valvular activity together to give a picture of the cardiac cycle as a
whole. It can be a daunting diagram, but when taken part by part it becomes much easier to
understand and therefore learn (Figures 1.5.1 and 1.5.2).

Downloaded from https://www.cambridge.org/core. University of Edinburgh, on 19 Aug 2019 at 13:21:27, subject to the Cambridge Core terms of
use, available at https://www.cambridge.org/core/terms. https://doi.org/10.1017/9781108566100.004
36 Chapter 1b: Physiology Answers

QRS

P T

S1 S2

Atrioventricular
Atrioventricular valves open Atrioventricular valves open
valves closed

Aortic
Aortic valve closed valve Aortic valve closed
open

Diastole Systole Diastole

Figure 1.5.1 Wigger’s diagram of ECG, heart sounds, systole, diastole and valvular activity.

Downloaded from https://www.cambridge.org/core. University of Edinburgh, on 19 Aug 2019 at 13:21:27, subject to the Cambridge Core terms of
use, available at https://www.cambridge.org/core/terms. https://doi.org/10.1017/9781108566100.004
Chapter 1b: Physiology Answers 37

Atrial Aortic

QRS QRS

P T P T Aortic valves
Aortic closes
valves open
120 S2 S2 120 S2 S2
Pressure (mmHg)

Pressure (mmHg)
Aortic
pressure

a x v

0 Atrial Pr 0
Atrioventricular Atrioventricular Atrioventricular Atrioventricular
Atrioventricular valves open valves closed Atrioventricular valves open
valves open valves closed valves open
Aortic Aortic
Aortic valves closed Aortic valves closed Aortic valves closed Aortic valves closed
valves open valves open

Diastole Systole Diastole Diastole Systole Diastole

Left ventricular volume


Ventricular
QRS
EDV
QRS

P T
P T
ESV

S2 S2
S2 S2
Ventricular volume (ml)

Strovevolume

120

60

Atrioventricular Atrioventricular
Atrioventricular valves open
valves closed valves open Ventricular
pressure
Aortic Atrioventricular Atrioventricular
Aortic valves closed Aortic valves closed Atrioventricular valves open
valves open valves closed valves open

Aortic
Aortic valves closed Aortic valves closed
valves open
Diastole Systole Diastole

Diastole Systole Diastole

Figure 1.5.2 Wigger’s diagram of pressure waves.

Downloaded from https://www.cambridge.org/core. University of Edinburgh, on 19 Aug 2019 at 13:21:27, subject to the Cambridge Core terms of
use, available at https://www.cambridge.org/core/terms. https://doi.org/10.1017/9781108566100.004
38 Chapter 1b: Physiology Answers

QRS

P T

120 S1 S2

Aortic pressure
Pressure (mmHg)

Atrial pressure
0 Ventricular pressure
Ventricular volume (ml)

120 Left ventricular volume

60

0
Atrioventricular
Atrioventricular valves open Atrioventricular valves open
valves closed

Aortic valve
Aortic valve closed Aortic valve closed
open

Diastole Systole Diastole

Figure 1.5.3 Complete Wigger’s diagram.

Downloaded from https://www.cambridge.org/core. University of Edinburgh, on 19 Aug 2019 at 13:21:27, subject to the Cambridge Core terms of
use, available at https://www.cambridge.org/core/terms. https://doi.org/10.1017/9781108566100.004
Chapter 1b: Physiology Answers 39

Question 6: FTTTF
At rest, the cardiac cycle lasts approximately 0.8 s. The diastolic period is 0.5 s, while the
systolic period is 0.3 s. During tachycardia the diastolic time is shortened and becomes
closer to the systolic time in duration. Many events occur during diastole.
Diastole can be divided into several phases: beginning, early, mid and late.
The start of diastole is denoted by the closure of the aortic and pulmonary valves (second
heart sound). As the ventricular muscle relaxes the intraventricular pressure falls (isovolu-
metric relaxation) until the pressure in the ventricles is less than that in the atria, the
atrioventricular valves open (marking early diastole) and the ventricles fill rapidly. This
rapid filling is responsible for approximately 80% of the ventricular volume. As the
ventricles fill, the pressure difference between the atria and ventricles decreases and the
filling therefore slows (mid-diastole). It is during late diastole that the atria contract and
force blood from the atria into the ventricles.
The coronary blood flow and therefore perfusion vary during the cardiac cycle and by
region. Most of the blood flow to the coronary arteries is during diastole. Some blood flow
occurs during systole, while the left ventricle is dependent on diastolic flow only due to
higher pressures generated during contraction, causing compression of the overlying
vessels. This means that the left ventricle is more susceptible to ischaemia.
Lusitropy refers to the relaxation of myocardium following contraction. Positive lusi-
tropic factors include low calcium and β- adrenergic stimulation.

Question 7: TFFTF
CVP stands for central venous pressure and the waveform can be seen if a central venous
catheter is transduced (Figure 1.7.1). Normal CVP in a spontaneously breathing patient is
0–8 cmH2O (not mmHg). It is useful as an estimate of right ventricular preload and to
monitor adequacy of volume replacement – serial measurements are vastly more useful than
single results.

a
c

x
y

Figure 1.7.1 CVP wave.

The waveform is divided into ‘waves’. There are three positive waves and two ‘descent’ or
negative waves.
The ‘a wave’ is the pressure increase secondary to atrial contraction. This is increased in
atrial hypertrophy and absent in atrial fibrillation. Cannon a waves are seen when the atrium
contracts against a closed tricuspid valve, if the a waves are irregular it indicates complete
heart block, as opposed to regular when it suggests a nodal rhythm.
The ‘c wave’ represents the increase in pressure caused by the bulging of the tricuspid
valve into the right atrium during ventricular contraction.

Downloaded from https://www.cambridge.org/core. University of Edinburgh, on 19 Aug 2019 at 13:21:27, subject to the Cambridge Core terms of
use, available at https://www.cambridge.org/core/terms. https://doi.org/10.1017/9781108566100.004
40 Chapter 1b: Physiology Answers

The ‘v wave’ denotes atrial filling (closed tricuspid valve). Large v waves suggest tricuspid
incompetence.
The ‘x descent’ occurs during relaxation of the atrium. The x descent can be absent in
tricuspid incompetence.
The ‘y descent’ demonstrates passive ventricular filling (open tricuspid valve).

Question 8: TTTFF
The Frank–Starling law relates the fibre length at initiation of contraction to the force of
myocardial contraction produced. The two are proportionally related up to a point at which
the relationship fails (consider an overstretched elastic band – it doesn’t recoil with the same
force as a normally stretched elastic band). This can be represented in graphical form for
both normal and pathological states.
The pressure–volume relationship of the ventricles can be denoted in graphical form and can
be used to demonstrate the workload of the heart. In order for a valve to open, the pressure in the
chamber behind it must be greater than that of the chamber or space in front of it (i.e. the left
ventricular pressure must be greater than the aortic pressure for the aortic valve to open). During
the cardiac cycle there are two occasions where the pressure and volume are not inversely related.
This is referred to as isovolumetric relaxation or contraction, and reflects the period of time
during which the ventricles are essentially sealed with closed valves both from the atria and to the
great vessels. For example, as the ventricle relaxes the pressure within will fall; however, with
closed valves the volume will not change (isovolumetric relaxation)

Question 9: FFFFF
There are many methods used to calculate the cardiac output.
The simplest formula is: cardiac output (CO) = heart rate (HR) × stroke volume (SV). As
stroke volume = end diastolic volume (EDV) – end systolic volume (ESV) it can also be
written as: CO = HR × (EDV – ESV).
One of the most commonly used methods to calculate cardiac output is the Fick method,
which relies on the Fick principle. The Fick principle may be used to calculate blood flow to
individual organs and by considering the whole body as an organ, the cardiac output can be
calculated. It relies on the use of a marker substance (e.g. dye, O2, CO2) and states that the
blood flow through the organ in question is equal to the marker substance uptake divided by
the arteriovenous difference of that substance. For cardiac output calculation, the marker
used is often O2.
The Fick equation for cardiac output:

_ 2
VO
CO ¼
CaO2  CvO2

CaO2: arterial sample


CvO2 : mixed venous O2 saturation from PAFC sample
All of the variables can be directly measured; however, measuring oxygen uptake is
laborious and consequently an assumed value for VO _ 2 is used. This is called the Fick
determination and the assumed value used is 250 ml.min−1 (125 ml.min−1.m−2). Another
assumption made when using this calculation is that there are no intracardiac or

Downloaded from https://www.cambridge.org/core. University of Edinburgh, on 19 Aug 2019 at 13:21:27, subject to the Cambridge Core terms of
use, available at https://www.cambridge.org/core/terms. https://doi.org/10.1017/9781108566100.004
Chapter 1b: Physiology Answers 41

intrapulmonary shunts and therefore that the pulmonary blood flow is equivalent to the
systemic blood flow.

Question 10: TFFTF


Preload refers to the ventricular wall tensions before the start of systole (at the end of
diastole) and is therefore related to the length of the cardiac muscle fibre at this time. As
such it is described by the Frank–Starling relationship. It is associated with the end diastolic
volume, but its clinical measurement is usually taken as the CVP (if looking at the right side
of the heart) or the pulmonary capillary wedge pressure (left side of the heart).
Afterload is the ventricular wall tension required to eject blood during systole. It is the
work required to overcome resistance beyond the ventricle and is often therefore considered
clinically in terms of systemic vascular resistance (SVR). Anything that increases resistance
in the arterial circulation will increase afterload. An increased afterload means increased
myocardial work, decreased stroke volume and increased end systolic volume.
Contractility refers to the ability of myocardial muscle fibres to shorten (contract). It is
affected by extrinsic factors (preload and afterload) and intrinsic factors (autonomic activity
and catecholamines). Drugs affecting contractility are said to have an inotropic effect; this
may be positive or negative. It is difficult to measure contractility and therefore indirect
methods are used, such as stroke volume, cardiac output and ratio of left end diastolic
pressure to left end diastolic volume.

Question 11: FTFTF


The normal range of heart rate varies by individual but is stated as 60–100 bpm. Tonic
activity from the sympathetic outflow to the heart is mediated via T1–5 (usually dominant)
and parasympathetic tone via the vagus nerve. The balance of sympathetic and parasympa-
thetic tone determines heart rate. The heart rate can also be affected by hormonal mechan-
isms including adrenaline (via β1 receptors). There are many afferent inputs to the medulla
that influence the balance of autonomic tone and these are integrated by the nucleus tractus
solitarius (NTS) and the rostral ventrolateral medulla (RVLM).
The arterial supply is via the right coronary artery (RCA) from the anterior aortic sinus
and the left coronary artery (LCA) from the posterior aortic sinus. The RCA supplies the
right side of the heart, the sinoatrial node and (in most) the atrioventricular node. The LCA
supplies the left side of the heart and the interventricular septum. The RCA is the dominant
vessel in 50% of people and in 30% the RCA and LCA have equal dominance.
Coronary blood flow is autoregulated (60–80 mmHg) via many mechanisms: myogenic,
metabolic, endothelial vasoactive substances and hormones (e.g. ANP is a vasodilator and
ADH and angiotensin II are vasoconstrictors)

Question 12: FFTTT


The Valsalva manoeuvre is the action of forced expiration against a closed glottis after full
inspiration. A pressure of 40 mmHg should be achieved and held for 10 s. Four phases occur
during the manoeuvre where changes in the blood pressure (BP) and heart rate are observed.
• Phase 1: Increase in intrathoracic pressure causes an initial increase in venous return,
resulting in increased BP and lowering of the heart rate.

Downloaded from https://www.cambridge.org/core. University of Edinburgh, on 19 Aug 2019 at 13:21:27, subject to the Cambridge Core terms of
use, available at https://www.cambridge.org/core/terms. https://doi.org/10.1017/9781108566100.004
42 Chapter 1b: Physiology Answers

• Phase 2: The sustained high intrathoracic pressure reduces venous return and BP falls.
Baroreceptors sense a reduction in stretch and the baroreceptor reflex is stimulated to
restore normotension.
• Phase 3: At the end of the manoeuvre, the release of intrathoracic pressure creates a large
empty venous reservoir. The venous return pools in the pulmonary vessels which
reduces preload and causes the BP to fall.
• Phase 4: The baroreceptors respond to the drop in BP by reducing their inhibitory effect
on the pressor centre, causing vasoconstriction and tachycardia. Once venous return is
restored, there is a compensatory overshoot resulting in hypertension and a
baroreceptor-mediated bradycardia before both parameters eventually return to
normal.
The Valsalva manoeuvre can be used to assess autonomic function or to terminate a
supraventricular tachycardia. The manoeuvre also increases the intensity of hypertrophic
cardiomyopathy murmurs and decreases the intensity of most other murmurs.
In autonomic neuropathy there is an exaggerated drop in BP during phase 2 with no
overshoot or bradycardia in phase 4, this is due to diminished baroreceptor reflexes. A
square-wave response is seen in congestive cardiac failure, tamponade and valvular disease.
The square wave response occurs when the CVP is markedly raised and is characterized by a
rise instead of a drop in BP during phase 2.
The BP fall in phase 2 will be exaggerated in hypovolaemic patients and those with
sympathetic block (e.g. β-blockers and regional anaesthesia).

Question 13: FFFTF


The blood pressure is tightly controlled by both short- and long-term regulatory mechan-
isms. Short-term blood pressure regulation is largely mediated by arterial and cardiac
baroreceptors and the vasomotor centre, which alters the balance between sympathetic
and parasympathetic nervous systems. Long-term blood pressure control is mediated by
neurohumoral, renal, metabolic and genetic factors.
Baroreceptors are stretch receptors; they are classified into high- and low-pressure
receptors. High-pressure receptors are located within the aortic arch and the carotid
sinus. They are responsible for rapid, short-term blood pressure control. Low-pressure
baroreceptors are found within the heart, in large systemic veins and the pulmonary
vasculature. These receptors bring about changes in blood volume and are responsible for
slower changes in blood pressure.
The carotid sinus is situated at a dilatation at the bifurcation of the internal carotid.
Carotid sinus baroreceptors discharge impulses back to the vasomotor centre via a branch of
the glossopharyngeal nerve (IX). The baroreceptors in the aortic arch transmit impulses via
the vagus nerve (X).
The vasomotor centres are located in the medulla. The pressor area is located in the
ventrolateral medulla and the depressor area (cardioinhibitory centre) in the ventromedial
medulla. In hypotension, there is decreased baroreceptor stretch, the pressor area is
stimulated (via glossopharyngeal and vagus afferents) and efferent signals to sympathetic
preganglionic neurones in the spinal cord result in vasoconstriction, tachycardia and
increased blood pressure. In hypertension, the resultant increase in baroreceptor discharge
results in the stimulation of the depressor area and inhibition of the vasomotor centre,
causing a reduction in sympathetic tone and increased vagal activity.

Downloaded from https://www.cambridge.org/core. University of Edinburgh, on 19 Aug 2019 at 13:21:27, subject to the Cambridge Core terms of
use, available at https://www.cambridge.org/core/terms. https://doi.org/10.1017/9781108566100.004
Chapter 1b: Physiology Answers 43

Higher centres, such as the hypothalamus, cerebral cortex and limbic system, exert some
influence on the vasomotor centres. The nucleus tractus solitarius (NTS) is the sensory
nucleus for both the vagus and the glossopharyngeal nerves. Afferent signals from chemo-
and baroreceptors travel via the NTS and provide inhibitory signals to the vasomotor centre.
The Bainbridge reflex is an increase in heart rate due to an increase in central venous
pressure.

Question 14: FTFFF


The normal left ventricular end diastolic volume (LVEDV) is 120–150 ml in an adult. The
LVEDV is often used as a measure of preload. Preload is the LV wall tension at the end of
diastole and reflects maximal length of the ventricular sarcomeres just prior to contraction.
In clinical practice, surrogates for preload measurement include LVEDV (measured by
echocardiogram), pulmonary artery occlusion pressure when studying the LV, and central
venous pressure when studying the right ventricle.
The Frank–Starling law of the heart states that the stroke volume of the heart increases in
response to an increase in the volume of blood filling the heart (LVEDV) when all other factors
remain constant. A normal stroke volume is 70–80 ml. Stroke volume is calculated by end
diastolic volume (EDV) minus end systolic volume (ESV). Ejection fraction is the percentage of
ventricular volume that is ejected from the ventricle during systolic contraction. A normal
ejection fraction is 55–70% and is a measure of contractility. At rest a normal LVESV is 50–70 ml.
The LVEDV (or preload) is determined by venous return, heart rate, atrial contraction
and ventricular compliance. Poorly compliant ventricles, seen post myocardial infarction
and in ventricular hypertrophy for example, take longer to fill, relying more upon the atrial
‘kick’ for filling.
In systolic heart failure, the ejection fraction will be decreased and the LVEDV will be
increased. In diastolic failure the ejection fraction is normal and the LVEDV will be normal
to low since the ventricle is non-compliant and difficult to fill in diastole.
During strenuous exercise, oxygen consumption may increase up to 4000 ml.min−1.
Heart rate and stroke volume increase to about 90% of their maximum values in order to
increase cardiac output. Venous return increases due to the pumping action of the muscles.
Locally released mediators cause vasodilation in the contracting muscles resulting in
decreased afterload. The increased inotropy and chronotropy in the heart results in a
reduction in LVESV (due to more forceful ventricular contractions) and there is an
increased LVEDV due to increased venous return. Trained athletes have lower heart
rates, greater LVEDV and greater stroke volumes at rest.

Question 15: TFFTF


The cardiovascular system is closely controlled by both systemic and local control mechan-
isms. Systemic regulation is achieved by neural regulatory mechanisms, using baroreceptors
and the vasomotor centre in the medulla to adjust sympathetic tone and vessel calibre, and
the neurohumoral system. Local regulatory mechanisms also exist and allow tissues to
autoregulate their own blood flow; examples of this occur in the kidney, abdominal
mesentery, skeletal muscle, brain, liver and myocardium.
Autoregulation is thought to occur as a result of the intrinsic contractile response of
smooth muscle to stretch (myogenic theory) and due to the accumulation of vasodilator

Downloaded from https://www.cambridge.org/core. University of Edinburgh, on 19 Aug 2019 at 13:21:27, subject to the Cambridge Core terms of
use, available at https://www.cambridge.org/core/terms. https://doi.org/10.1017/9781108566100.004
44 Chapter 1b: Physiology Answers

substances in active tissues (metabolic theory). When blood flow decreases, these vasodi-
lator substances accumulate, causing vessel dilatation, thus improving blood flow.
Metabolic changes in the tissues that produce vasodilation include decreases in oxygen
tension and pH. Increases in carbon dioxide tension, lactate, osmolality and K+ also dilate
vessels. A rise in temperature exerts a direct vasodilator effect. Adenosine has a vasodilatory
effect in cardiac muscle, but not in skeletal muscle.
Injured vessels constrict strongly. A drop in tissue temperature causes vasoconstriction;
this process is important in temperature regulation.
Endothelial cells secrete a number of vasoactive substances, including prostaglandins,
thromboxanes, nitric oxide and endothelins. Prostacyclin is produced by endothelial cells. It
inhibits platelet aggregation and promotes vasodilation. Thromboxane A2 opposes the
actions of prostacyclin on platelets and promotes vasoconstriction. Nitric oxide plays a
key role in vasodilation. Endothelin-1 is a local, paracrine regulator of vascular tone and
causes vasoconstriction.
Other circulating vasodilator hormones include kinins, VIP, substance P and atrial
natriuretic peptide (ANP). Circulating vasoconstrictor hormones include vasopressin,
noradrenaline, adrenaline and angiotensin II.

Question 16: TTFFF


Vasoconstrictors: epinephrine, norepinephrine, angiotensin II, vasopressin, endothelins,
prostaglandin F2α and thromboxane.
Vasodilators: acetylcholine, atrial natriuretic peptide, bradykinin, serotonin, adenosine,
prostacyclin (PGI2) and prostaglandin E2 (PGE2).

Question 17: FTFFT


Reynold’s number (NR) is a dimensionless number and predicts whether the flow is laminar
or turbulent within a tube. If NR <2000, the flow is usually laminar. If NR >4000, the flow is
turbulent. For NR between 2000 and 4000, the flow could either be laminar or turbulent.
ρvD
NR ¼
η
where ρ = density, v = mean velocity, D = diameter of the tube, η = viscosity.
Thus the equation indicates that large diameter, high velocity and low viscosity predis-
pose to a higher NR and hence a turbulent flow.

Question 18: TFTTT


The lymphatic system serves to collect the exuded plasma and its constituents from the
interstitial space. The protein content of lymphatic fluid is lower than in plasma (1.5 versus
6%). The lymph vessels are blind-ended, thin-walled and lack tight junctions in their endothe-
lium. They have one-way valves and drain via a system of lymphatic vessels, eventually entering
the subclavian veins at their junctions with the respective internal jugular veins.
The thoracic duct is the largest lymphatic vessel and transports up to 4 litres of lymph per
day. In adults, the thoracic duct is typically 38–45 cm in length and usually starts from the
level of the second lumbar vertebra, extending to the root of the neck. It drains into the
systemic circulation at the left subclavian vein and collects most of the lymph in the body,

Downloaded from https://www.cambridge.org/core. University of Edinburgh, on 19 Aug 2019 at 13:21:27, subject to the Cambridge Core terms of
use, available at https://www.cambridge.org/core/terms. https://doi.org/10.1017/9781108566100.004
Chapter 1b: Physiology Answers 45

other than from the right thorax, arm, head and neck, which are drained by the right
lymphatic duct.
Lymphatic flow is aided by skeletal muscle contractions, negative intrathoracic pressure
(i.e. by the patient breathing), and pressure created by the blood flow from adjacent arteries
and veins.

Question 19: TFTTF


The arterioles are the major site of the resistance to blood flow, and small changes in their
calibre cause large changes in the total peripheral resistance. The capillaries range in
diameter from 5–10 μm. The capillary wall is made of single layer of endothelium.
Blood is a non-Newtonian fluid; its viscosity is affected by changes in temperature, which
can affect flow.
Precapillary sphincters are not innervated and they respond to local or circulating
mediators.

Question 20: TFTFF


Poiseuille’s equation:

ΔPπr4
Flow ¼
8ηl
Where ΔP = pressure difference, r = radius of the tube, η = viscosity of the fluid, l = length of
the tube
8ηl
Resistance ¼
πr4

Question 21: FTFTT


Any lung volume that encompasses the residual volume cannot be measured with spiro-
metry, as by definition, it is the volume of air remaining in the lungs after maximal
expiration. Therefore, spirometry can measure vital capacity, tidal volume, inspiratory
reserve volume and expiratory reserve volume. Functional residual capacity (FRC) is the
volume of air remaining in the lungs after a normal expiratory breath. It is measured using
the nitrogen washout technique, helium dilution technique or body plethysmography. Total
lung capacity (TLC), the total volume of air in the lungs after maximal inspiration, is
measured by body plethysmography.

Question 22: FTFTT


Compliance is defined as the change in volume per unit change in transpulmonary pressure.
Therefore the units are ml.cmH2O−1, and compliance is determined by the slope of
the pressure–volume curve. Normal values for both static and dynamic compliance are
60–100 ml.cmH2O–1. Static compliance is always greater than dynamic compliance due to
the variation in time constants in different areas of the lung (i.e. how quickly a lung
compartment can react to an alteration in pressure). Compliance is best at the FRC and is

Downloaded from https://www.cambridge.org/core. University of Edinburgh, on 19 Aug 2019 at 13:21:27, subject to the Cambridge Core terms of
use, available at https://www.cambridge.org/core/terms. https://doi.org/10.1017/9781108566100.004
46 Chapter 1b: Physiology Answers

influenced by pathological and physiological factors. Physiological factors which decrease


compliance include supine posture, pregnancy and extremes of age. Pathological factors
which decrease compliance can be classified into extrapulmonary, which include the chest
wall (kyphoscoliosis), and intrapulmonary, which include pulmonary fibrosis, ARDS and
atelectasis. The only pathological factor that increases the lung compliance is emphysema.

Question 23: FFFFT


Respiratory dead space (physiological dead space) is defined as the volume of inspired gas that
does not take part in gas exchange. It is determined by the totals of the anatomical dead space
(calculated by Fowler’s method) and the alveolar dead space (determined by the Bohr equation).
Physiological dead space = anatomical dead space + alveolar dead space
The V/Q ratio is therefore infinity. The respiratory dead space is decreased by general
anaesthesia due to increased shunt. As dead space increases, alveolar ventilation decreases,
therefore paCO2 will increase.
The V/Q ratio at the apices of the lungs is relatively higher, as they are more well-
ventilated than perfused compared to the lung bases, where ventilation and perfusion are
highest. Thus, alveolar dead space in the apices is increased.

Question 24: TTTTT


The work of breathing is the work required by the respiratory muscles in order to overcome
the mechanical impedance to respiration caused by the lung, chest wall and abdominal
contents during breathing. It is the sum of the work required to overcome both elastic
properties and flow resistance, and is determined by the area contained inside a pressure–
volume curve. The elastic forces are stored as potential energy during inspiration.
Although expiration is usually passive, i.e. does not require energy expenditure (utilizes
the potential energy stored in the respiratory muscle during inspiration), during forced
expiration, or in a patient with airway disease, additional expiratory mechanical work may
be required, even in tidal breathing. The frictional forces of expiration are due to airway
resistance. For example, in asthma, there is an increase in flow-resistive work as a result of
the airway narrowing. Here, the elastic energy stored during inspiration will not produce
enough airflow during expiration and the expiratory muscles must cope with an increased
load and perform extra work.
Any work to overcome non-elastic forces is lost as heat. In a ventilated patient, the
ventilator tubing and the endotracheal tube increase the airway resistance, therefore
increasing the work of breathing.

Question 25: TFTFT


The FRC is the volume of air left in the lungs at the end of a normal tidal breath. It is
approximately 30 ml.kg−1 and effectively provides an oxygen reservoir as a buffer against
injury. Here, gas exchange continues to occur throughout the respiratory cycle. It prevents
airway collapse and atelectasis, therefore pulmonary vascular resistance is lowest at FRC.
The closing capacity is the volume of the lung at the point when small airways begin to
collapse during expiration. It is the sum of the residual volume and the closing volume.
Closing capacity increases with age and so encroaches upon the FRC. This means that there
is an increased ventilation/perfusion (V/Q) mismatch.

Downloaded from https://www.cambridge.org/core. University of Edinburgh, on 19 Aug 2019 at 13:21:27, subject to the Cambridge Core terms of
use, available at https://www.cambridge.org/core/terms. https://doi.org/10.1017/9781108566100.004
Chapter 1b: Physiology Answers 47

FRC decreases under general anaesthesia due to the loss of muscle tone reducing the
bucket handle action of the rib cage.
FRC typically reduces by approximately 18–20% during pregnancy, due to compression
of the diaphragm by the gravid uterus. In the pregnant woman, in order to overcome the
reduction in FRC, total lung capacity, expiratory reserve volume and tidal volume increase
by 30–40%. Minute ventilation increases, giving an increase in pulmonary ventilation
necessary to meet the increase in oxygen requirement.

Question 26: FFTTF


The main muscle of respiration is the diaphragm, which during normal respiration is
responsible for 75% of the air that is drawn into the lungs. Two-thirds of diaphragmatic
muscle fibres are of the slow twitch variety so as to reduce its fatigability. The lungs and the
thoracic cage are the two main mechanical components of the respiratory system.
A pressure gradient exists across the surface of the lungs known as the transpulmonary
pressure, which is equal to the difference between the airway pressure and the pressure at the
lung surface (intrapleural pressure). The natural tendency of the lungs is to collapse, while
that of the rib cage is to spring outwards. This produces a pressure differential between the
two pleural layers, parietal and visceral – the intrapleural pressure.
The accessory muscles include the sternocleidomastoids, the scalene muscles and strap
muscles of the neck. While these are relatively inactive in normal respiration, they do serve
to stabilize the upper rib cage and prevent in-drawing of the ribs.
Compliance is defined as the change in volume per unit change in pressure.

Question 27: TTTFF


Type II alveolar epithelial cells secrete surfactant. An important constituent of surfactant is
dipalmitoylphosphatidylcholine (DPPC), synthesized from fatty acids. Surfactant synthesis
and its turnover are rapid, production begins relatively late in gestation; hence its deficiency
before 32–34 weeks’ gestation can predispose to respiratory distress syndrome. Surfactant
serves to reduce surface tension and, in so doing, it improves the pulmonary compliance
and reduces the tendency of the alveoli to draw in fluid from interstitial capillaries, keeping
the alveoli relatively dry.
Hysteresis (defined as the change in the measurement value depending on whether the
value is increasing or decreasing) of the pressure–volume loop is the result of the energy
expended as heat in the respiratory cycle. Surfactant reduces not only the work of breathing,
but also the heat expended and therefore results in a reduction in the volume of the
hysteresis area of the pressure–volume curve.
Surfactant also serves to stabilize the smaller alveoli, as these are most prone to collapse
due to surface tension. In stabilizing the smaller alveoli, surfactant makes them less likely to
collapse and empty their contents to adjacent larger alveoli, but has no effect on the stability
of larger alveoli per se.

Question 28: FTFFT


From apex to base, both ventilation and perfusion increase in the upright lung; however, the
increase in perfusion is much greater. This results in a ventilation/perfusion ratio (V/Q

Downloaded from https://www.cambridge.org/core. University of Edinburgh, on 19 Aug 2019 at 13:21:27, subject to the Cambridge Core terms of
use, available at https://www.cambridge.org/core/terms. https://doi.org/10.1017/9781108566100.004
48 Chapter 1b: Physiology Answers

ratio) that is greater at the apex than it is at the base, being approximately equal to 1 at the
level of the third rib.
The difference in partial pressure between the apex and base of the lung is greater for
oxygen; this is because the uptake of oxygen at the apex tends to be poor due to the low
blood flow at rest. This increases in exercise when blood flow throughout the lung is more
uniform. Carbon dioxide partial pressures are less affected by blood flow and influenced
more by ventilation; the difference in CO2 output between apex and base is much less
marked.
While hypercarbia can be compensated for by an increase in ventilation (in order to
reduce the partial pressure of carbon dioxide in the alveoli and in turn in the blood),
hypoxaemia cannot be corrected in this way. The main way to increase the partial pressure
of oxygen in the bloodstream is to increase the fraction of inspired oxygen. Pulmonary
emboli obstruct blood flow to the lungs, i.e. perfusion is reduced. Ventilation throughout
the lung remains constant, thus the ventilation/perfusion ratio is increased.

Question 29: TTFTF


The balance between oxygen supply (alveolar ventilation) and demand (removal by pul-
monary capillaries to supply the tissues) determines the pO2 of alveolar gas. At rest, the
removal of oxygen from the lungs (i.e. tissue oxygen usage) is relatively constant and so
alveolar ventilation becomes the main determinant of pAO2. Hypoventilation results in
inefficient gas exchange and so pO2 falls while PCO2 is seen to rise, in turn causing an
increase in paCO2.
The respiratory quotient is used in the alveolar gas equation, which uses the composition
of inspired gas and the respiratory quotient to calculate alveolar pO2:

pA O2 ¼ pI O2  ðpA CO2 =RÞ þ F

(Where pIO2 is the partial pressure of inspired oxygen, F is a small correction factor and R is
the respiratory quotient, which normally = 0.8.) R is calculated by CO2 production/O2
consumption (200/250 = 0.8), and is dependent on the type of meal, i.e increased in high
carbohydrate meal.
paCO2 takes longer to reach equilibrium as compared with pAO2 because body CO2
stores are much greater than O2 stores and so reaching a steady state value takes longer.
Shunt refers to regions of lung that are perfused but where ventilation is deficient. This
does occur in health but can be exacerbated in disease and also partly explains why paO2 is
less than pAO2.
Shunts can be classified into:
1. Physiological (5%): intrapulmonary (bronchial vein) and extrapulmonary (thebesian
veins)
2. Pathological: intrapulmonary (ARDS) and extrapulmonary (PFO and PDA).

Question 30: TTFTF


This question relates to changes in blood flow occurring in the lung from the apex to the
base described by West as Zones 1–3. Where Pv = venous pressure, Pa = arterial pressure
and PA = alveolar pressure. In Zone 1: PA>Pa>Pv. This situation does not arise under

Downloaded from https://www.cambridge.org/core. University of Edinburgh, on 19 Aug 2019 at 13:21:27, subject to the Cambridge Core terms of
use, available at https://www.cambridge.org/core/terms. https://doi.org/10.1017/9781108566100.004
Chapter 1b: Physiology Answers 49

normal conditions, but can occur with positive pressure ventilation or, alternatively, when
arterial blood flow is compromized (massive haemorrhage).
In Zone 2: Pa>PA>Pv and, so, it is here that the difference in arterial and alveolar pressures
determines blood flow. While the alveolar pressure is relatively constant throughout the lung, the
arterial pressure and therefore the pressure driving blood flow increase down the zone. This effect
observed in Zone 2 is referred to as the Starling resistor, sluice or waterfall effect.
In Zone 3, Pa>Pv>PA, i.e. the venous pressure now exceeds the alveolar pressure and so it
is here that the arteriovenous pressure difference determines blood flow in the usual way.
Like Zone 2, an increase in blood flow is also observed down this zone, attributed to
increased capillary dilatation moving downward through the zone.

Question 31: TFFTF


IPPV results in some unfavourable pulmonary physiological effects such as: maldistribution
of gas, progressive atelectasis with a reduced FRC, increased ventilation/perfusion (V/Q)
mismatch, decreased compliance and a reduction in surfactant.
In spontaneously breathing patients, both ventilation and perfusion are preferentially dis-
tributed to the dependent zones of the lungs. With IPPV, preferential ventilation of the non-
dependent regions occurs (due to lower resistance to flow), resulting in increased V/Q mismatch.
IPPV in the supine position leads to decreased FRC, due in part to decreased lung volume
from cephalad displacement by the diaphragm and abdominal contents. The loss of lung
volume contributes to atelectasis and reduced compliance.
Pulmonary hypoperfusion from IPPV, especially in the non-dependent regions with
maldistribution of gas, leads to increased alveolar dead space ventilation. Dead space
ventilation increases with rapid respiratory rates, age and lung pathology.
Pulmonary vascular resistance (PVR) is increased at very high lung volumes due to
stretch of the pulmonary vessels and low lung volumes due to compression of the pulmon-
ary vessels. PVR is lowest at FRC.
PEEP is used to reduce airways resistance (wider calibre airways at higher FRC), increase
FRC and to prevent or reverse lung collapse. PEEP increases anatomical dead space.
Most cardiovascular side effects of IPPV correlate with mean intrathoracic pressure and
reduced venous return.
Humoral effects of IPPV include an increase in ADH, renin–angiotensin and atrial
natriuretic peptide, leading to overall retention of sodium and water.

Question 32: TFFTF


Barometric pressure decreases exponentially as the distance from the Earth’s surface
increases. The partial pressure of oxygen decreases at increasing altitude, but the composi-
tion of air (e.g. the FiO2) does not change.
The compensatory changes to high altitude can be divided into acute and chronic. The
most important acute physiological response to high altitude is hyperventilation.
Hyperventilation is triggered by hypoxic stimulation of the peripheral chemoreceptors.
The resultant hypocarbia and respiratory alkalosis lead to a leftward shift of the oxygen
dissociation curve. The respiratory alkalosis is slowly corrected by renal compensation. The
respiratory alkalosis stimulates increased 2,3-DPG production and eventually causes the
OHDC to move towards the right, thus improving oxygen delivery to the tissues. Pulmonary

Downloaded from https://www.cambridge.org/core. University of Edinburgh, on 19 Aug 2019 at 13:21:27, subject to the Cambridge Core terms of
use, available at https://www.cambridge.org/core/terms. https://doi.org/10.1017/9781108566100.004
50 Chapter 1b: Physiology Answers

vasoconstriction also occurs in response to alveolar hypoxia. This increases the pulmonary
arterial pressures and increases the strain on the right heart.
One of the body’s chronic responses to high altitude includes polycythaemia due to
increased erythropoietin release from the kidney. Although the additional oxygen-carrying
capacity is beneficial, polycythaemia also causes increased blood viscosity, which negates
some of its beneficial effects. Chronic hypoxic pulmonary vasoconstriction causes right
ventricular hypertrophy. Myoglobin concentration increases and peripheral capillary pro-
liferation occurs.

Question 33: TFFTT


The lung is responsible for several important non-respiratory functions. These functions
include:
• Vascular reservoir: The pulmonary circulation has extremely distensible vasculature;
this enables it to cope with large fluxes in venous return, particularly during exercise and
postural changes. Changes in pulmonary vascular volume are also influenced by the
sympathetic nervous system. 70–100 ml of blood is contained within the pulmonary
capillaries, which takes part in gas exchange.
• Filter for blood: Clots, fibrin clumps and air bubbles are all filtered from peripheral
venous blood by the lungs, preventing them from entering the systemic circulation.
Pulmonary endothelium also produces substances (fibrinolysin activator) that break
down blood clots in the pulmonary circulation.
• Immune function: The mucociliary escalator is the first of line defence against inhaled
physical substances. As well as a physical barrier, the lungs provide an immune function
that is mediated by pulmonary alveolar macrophages and a variety of immune
mediators. Immunoglobulins (IgA) are also present in the bronchial secretions.
• Endocrine and metabolic function: The pulmonary endothelium selectively takes up
norepinephrine and serotonin from circulating blood while sparing histamine,
dopamine and epinephrine; 30% of norepinephrine and 98% of serotonin is removed by
this process. Angiotensin-converting enzyme (ACE) is present in large quantities in the
pulmonary vascular endothelium. Angiotensin I is converted to the vasoactive peptide
angiotensin II by ACE.
• Drug metabolism: The lung is a small but important extrahepatic site for drug
metabolism by the cytochrome P450 system. The system is easily saturated, but unlike
the hepatocytes, cannot be induced.
• Major role in acid/base balance

Question 34: FTFFF


Haemoglobin (Hb) is the protein responsible for carrying almost all of the oxygen within the
blood; only a small percentage of oxygen is dissolved in solution. Hb is composed of four
subunits, each consisting of a haem group and a globin chain. The haem group itself is
composed of a protoporphyrin ring containing iron in its ferrous state (Fe2+) at the centre.
The Hb molecule has four binding sites for oxygen (the iron atoms in the four haem groups)
and therefore is able to cooperatively and reversibly bind up to four oxygen molecules.
Oxygen can only bind to iron in the ferrous state. Methaemoglobin contains iron in the
ferric/oxidized state (Fe3+) and hence cannot carry oxygen.

Downloaded from https://www.cambridge.org/core. University of Edinburgh, on 19 Aug 2019 at 13:21:27, subject to the Cambridge Core terms of
use, available at https://www.cambridge.org/core/terms. https://doi.org/10.1017/9781108566100.004
Chapter 1b: Physiology Answers 51

Adult haemoglobin (HbA) has two α and two β globin chains, whereas fetal haemoglobin
(HbF) contains two γ and two α chains. When fully saturated, each gram of HbA carries
1.306 ml of oxygen. However, when fully saturated, HbF will carry more, at 1.312 ml.g−1.
This allows placental transfer of oxygen from mother to fetus.
Increases in 2,3-DPG, hydrogen ion and carbon dioxide concentrations all reduce the
access of oxygen to the haem portion as a result of a conformational change within the Hb
molecule (by manipulating bonds between amino acids). This results in decreased affinity of
Hb for oxygen. 2,3-DPG is a by-product of glycolysis, specifically reducing oxygen affinity
by binding to the β chains. Therefore, more oxygen may be offloaded to the tissues, a
survival benefit.
The reduction in oxygen–Hb affinity in the presence of an increased carbon dioxide or
hydrogen ion concentration, or an increase in temperature, is known as the Bohr effect. The
Haldane effect has no bearing on oxygen binding, but refers to the increased ability of
deoxygenated blood to carry carbon dioxide. Conversely, oxygenated blood has a reduced
capacity for carbon dioxide.

Question 35: TFFFF


Minute ventilation is influenced by information from both central and peripheral chemo-
receptors, which relay signals to the medullary respiratory centre.
Central chemoreceptors are situated in the ventrolateral medulla near the respiratory
centre and are stimulated predominantly by a rise in hydrogen ion concentration within the
nearby cerebrospinal fluid (CSF). However, circulating H+ ions are unable to cross the tight
junctions of the blood–brain barrier (BBB), and therefore, central chemoreceptors are not
influenced by plasma pH. Their predominant mode of action is to increase ventilation in
response to hypercarbia. Unlike H+ ions, circulating CO2, which is raised in respiratory
acidosis, is able to cross the BBB. Within the CSF, the excess CO2 combines with H2O to
form carbonic acid. This dissociates to form H+ ions, which then stimulate the central
chemoreceptors. Central chemoreceptors are unaffected by hypoxia.

CO2 þ H2 ⇌ H2 CO3 ⇌ Hþ þ HCO


3

Hypoxia is the predominant stimulus for the peripheral chemoreceptors. These are
situated in the carotid and aortic bodies and relay information to the medulla via the
glossopharyngeal and vagus nerves, respectively. These specialized receptors are richly
invested with capillary networks and derive their oxygen needs from dissolved oxygen;
hence they are very sensitive to fluctuations in low oxygen tensions. Do not confuse this with
conditions where the content of oxygen is decreased, e.g. anaemia (where oxygen tension
may be normal, despite the low content). The carotid and aortic bodies also respond to the
carbon dioxide tension and plasma pH, but these are less influential than hypoxia.
Control of ventilation is mainly influenced by the central chemoreceptors, which are very
sensitive to small fluctuations in CSF pH. Significant degrees of hypoxia are required to
stimulate the peripheral receptors. However, their role becomes more significant at altitude
or in cases of chronic carbon dioxide retention, where there is blunting of the central
receptor response.
An increase in body temperature does stimulate ventilation, but this is via the respiratory
centre directly, not the chemoreceptors.

Downloaded from https://www.cambridge.org/core. University of Edinburgh, on 19 Aug 2019 at 13:21:27, subject to the Cambridge Core terms of
use, available at https://www.cambridge.org/core/terms. https://doi.org/10.1017/9781108566100.004
52 Chapter 1b: Physiology Answers

Question 36: TTFTF


Haemoglobin is the primary non-bicarbonate buffer in the extracellular fluid. It is a weak
acid and acts as a buffer by accepting hydrogen ions (through its histidine residue). This is
another example of the action of carbonic anhydrase; during acidic conditions, plasma
carbon dioxide diffuses into the erythrocyte where it combines with water to form carbonic
acid in a reaction catalyzed by erythrocyte carbonic anhydrase. Carbonic acid then dis-
sociates to form hydrogen ions and bicarbonate. H+ binds to histidine residues on the
globulin chains. Bicarbonate then diffuses out of the erythrocytes into the plasma. To
maintain electrical neutrality of the cell, chloride ions diffuse into the erythrocytes from
the plasma (‘chloride shift’) (Hamburger effect).
Deoxygenated haemoglobin is a more powerful buffer than its oxygenated counterpart. It
has a pKa of 8.1 versus 6.8, hence it is less acidic and acts as a more effective base to accept
the protonated H+. Deoxygenated blood is therefore a more effective carrier of carbon
dioxide at the tissue level. The converse is seen at the lungs, where oxygenated blood is less
able to bind CO2, which is ‘offloaded’ and eliminated from the body. This is known as the
Haldane effect. Although there is a significant difference in the amount of CO2 carried in
arterial and venous blood, haemoglobin is such an effective buffering system that there is
only a small difference between the pH of arterial and venous blood.

Question 37: TTTFF


Oxygen therapy should be administered and prescribed appropriately; under certain conditions
it can be toxic, with a host of deleterious side effects. Oxygen toxicity is caused by exposure to
oxygen at partial pressures greater than those to which the body is generally accustomed. This
occurs in three principal settings: underwater diving, hyperbaric oxygen chambers and supple-
mental oxygen therapy. Toxicity is related to free radical production. These reactive oxygen
species can damage cell structures and result in oxidative stress to certain tissues.
The goal of supplemental oxygen therapy is usually to use the lowest FiO2 possible for the
minimum time in order to ensure adequate tissue oxygenation. With regards to duration of
oxygen therapy and risk of toxicity, FiO2 1.0 should be limited to a period of less than
12 hours, FiO2 0.8 to less than 24 hours and FiO2 0.6 to less than 36 hours.
Pulmonary effects of toxicity are more common, and occur with exposure to FiO2 of 0.5
or more at atmospheric pressure. Symptoms result from airway and pulmonary inflamma-
tion and include a ‘tickle’ leading to frequent coughing. If oxygen is not discontinued, this
can progress to dyspnoea, substernal discomfort, pulmonary oedema and finally ARDS due
to diffuse alveolar damage. Historically, this was known as the Lorrain–Smith effect.
Breathing 100% oxygen eventually leads to absorption atelectasis.
The neonate is especially at risk following exposure to high inspired oxygen concentra-
tions. Retrolental fibroplasia is strongly associated with hyperoxia in the developing infant
(although the mechanism differs). Preterm newborns are known to be at higher risk for
bronchopulmonary dysplasia. Others at higher risk include patients on mechanical ventila-
tion (with FiO2 >0.5) and those on particular chemotherapy agents, such as bleomycin.
Exposure to partial pressures of oxygen above 160 kPa, i.e. supra-atmospheric, for as little
as a few minutes is associated with central nervous system toxicity. Therefore, those most at
risk are underwater divers and patients having hyperbaric oxygen therapy. Toxicity is
characterized by perioral twitching, tinnitus, confusion, seizures and drowsiness (Paul

Downloaded from https://www.cambridge.org/core. University of Edinburgh, on 19 Aug 2019 at 13:21:27, subject to the Cambridge Core terms of
use, available at https://www.cambridge.org/core/terms. https://doi.org/10.1017/9781108566100.004
Chapter 1b: Physiology Answers 53

Bert effect). Tremors are usually associated with hypercarbia and renal failure is not known
to occur as a result of hyperoxia.

Question 38: FFTTT


Acute respiratory failure is a common reason for admission to critical care units. It is often
secondary to lower respiratory tract infection, but there are other important causes.
The concept of a critical care unit, with respiratory support, arose from the polio
epidemics of the 1950s. The polio virus can damage motor neurones, including those that
supply intercostal nerves. Polio patients with respiratory failure were originally treated with
negative pressure ventilators (‘iron lungs’). Poliomyelitis is now almost completely eradi-
cated in the UK due to vaccination.
Tetanus results from infection with tetanus toxoid produced by Clostridium tetani. This
potent neurotoxin affects striated, skeletal muscle and causes tetanic contractions such as
trismus and opisthotonus. Acute respiratory failure is not a feature.
Guillain–Barré syndrome is an acute, ascending polyneuropathy, which has a sudden
onset and affects the peripheral nervous system. This can involve paralysis of the diaphragm,
subsequent respiratory failure, and potentially a requirement for intubation and ventilation.
Hypersensitivity pneumonitis is an alternative term for extrinsic allergic alveolitis. It may
be triggered by a range of allergens (including plastics, hay and pigeon feathers), which
cause an acute hypersensitivity reaction similar to asthma, but which may progress to
chronic pulmonary fibrosis.
Aspirin in overdose stimulates the respiratory centre in the medulla, causing hyperventi-
lation and respiratory alkalosis. If a patient with known aspirin overdose shows signs of
respiratory failure, this should raise the suspicion of ingestion of other drugs.

Question 39: TFFFT


Acute lung injury (ALI) is a common condition that is characterized by acute severe hypoxia
that is not due to left atrial hypertension. The term ALI encompasses a continuum of clinical
and radiographic pulmonary changes, with acute respiratory distress syndrome (ARDS)
being at the most severe end of the spectrum. ALI is still associated with a high mortality
and, in essence, is the presence of non-cardiogenic pulmonary oedema and respiratory
failure in the critically ill patient.
Correct diagnosis of ALI is essential as other causes of hypoxaemia may be present that
are more easily treated. The definition in current use was described in 1994 at the
American–European Consensus Conference and was created in order to make it easier to
classify and study the diseases epidemiologically. Strengths of these criteria are that they are
clinically relevant and easy to use. Weaknesses are that they are non-specific and subject to
interpretation.
Diagnostic criteria are: acute onset, bilateral infiltrates on chest X-ray consistent with
oedema (air bronchograms may be seen, but these are not diagnostic), pulmonary artery
wedge pressure (PAWP) <18 mmHg or clinical absence of left atrial hypertension and
hypoxaemia with a paO2 /FiO2 <40 (if paO2/FiO2 <27, this indicates ARDS).

Downloaded from https://www.cambridge.org/core. University of Edinburgh, on 19 Aug 2019 at 13:21:27, subject to the Cambridge Core terms of
use, available at https://www.cambridge.org/core/terms. https://doi.org/10.1017/9781108566100.004
54 Chapter 1b: Physiology Answers

Question 40: TTTTF


Respiratory failure occurs when there is inadequate gas exchange by the respiratory system.
It is diagnosed by a paO2 <8.0 kPa in the arterial blood. p may be low or normal (type 1
respiratory failure) or elevated (type 2 respiratory failure).
Type 1 respiratory failure is usually acute, and is caused by factors that impair oxygena-
tion, although alveolar ventilation may be maintained. Examples would include low ambi-
ent oxygen partial pressure (e.g. altitude), and ventilation/perfusion mismatches such as
consolidation (worsening shunt) and pulmonary embolism (worsening dead space).
Type 2 respiratory failure can be acute or chronic and reflects a failure of alveolar
ventilation; consequently, paCO2 rises. Causes include central hypoventilation, chest wall
abnormalities, neuromuscular disease and increased airways resistance.
Acutely raised paCO2 results in a respiratory acidosis with a reduced pH, due to the action
of carbonic anhydrase catalyzing the following reversible reaction:
CO2 þ H2 O ⇌ H2 CO3 ⇌ Hþ þ HCO
3

In chronic respiratory failure, there is renal compensation for the prolonged rise in CO2
via retention of bicarbonate (a base), to maintain a normal pH. This is seen as an increase in
the bicarbonate level on the ABG with a positive base excess value.
Carboxyhaemoglobin (COHb) levels are often available on ABG reports and are a
measure of the amount of carbon monoxide in the blood. COHb is not a sign of respiratory
failure, but may be raised if the patient is a smoker. The normal value for non-smokers is
1.5–3%, but this can be increased to 5–15% in heavy smokers. Levels of 15–20% suggest
significant carbon monoxide poisoning, as may occur after smoke inhalation. Levels over
40% indicate severe poisoning, with seizures and loss of consciousness likely.

Question 41: TFTTT


Fick’s law states that the rate of transfer of a gas through a sheet of tissue is proportional to
tissue area (A) and the difference in gas partial pressures either side of the membrane
(P1 – P2), and inversely proportional to the tissue thickness (T). The rate of transfer is also
proportional to the diffusion constant (D), which is proportional to the solubility of the gas
and inversely proportional to the square root of the molecular weight.

A
Rate of diffusion ∝ :D:ðP1  P2 Þ
T

pffiffiffiffiffiffiffi
Solubility
Where D ∝ MW

Question 42: TFFTF


The A–a gradient is a measure of the difference between alveolar and arterial partial
pressures of oxygen (pAO2 – paO2). It is used in diagnosing the cause of hypoxaemia; the
measurement helps isolate the problem as either intra- or extrapulmonary. A normal A–a
gradient for a young adult, non-smoker, breathing air is between 5 and 10 mmHg. It
increases with age; a rough estimate can be calculated by (age in years/4) + 4.
An abnormally raised A–a gradient indicates that the hypoxaemia may be due to a defect
in diffusion, V/Q mismatch or right-to-left shunt.

Downloaded from https://www.cambridge.org/core. University of Edinburgh, on 19 Aug 2019 at 13:21:27, subject to the Cambridge Core terms of
use, available at https://www.cambridge.org/core/terms. https://doi.org/10.1017/9781108566100.004
Chapter 1b: Physiology Answers 55

The A–a gradient is normal in hypoventilation and at high altitude, as the lung parenchyma
can be assumed to be normal. Hypoxaemia in the face of a normal A–a gradient implies
hypoventilation with displacement of alveolar oxygen by CO2 or another substance. At
altitude, paO2 is low but only because pAO2 is low; transfer of gas is within normal limits.

Question 43: FTTFT


The oxyhaemoglobin dissociation curve is an important tool for understanding how our
blood transports and releases oxygen. It specifically relates oxygen saturation and partial
pressure of oxygen in the blood, and is determined by the affinity of haemoglobin for oxygen.
The affinity is affected by several factors; these shift or reshape the curve. A rightward
shift indicates the Hb has a decreased affinity for oxygen, but it makes it easier for the Hb to
release oxygen bound to it. The effect here is to increase the partial pressure of oxygen in the
tissues where it is most needed. Conversely, a leftward shift indicates an increased affinity
for oxygen so that Hb binds it more easily, but unloads it more reluctantly.
The term P50 is defined as the partial pressure of oxygen at which the oxygen binding
protein (usually Hb) is 50% saturated. P50 is used to specify the position of the curve, i.e. it is
a measure of oxygen affinity. The P50 is located on the steepest part of the curve and
therefore shifting the curve left or right has a greater effect on SpO2 here. P50 is shifted to the
right with an increase in hydrogen ion concentration, pCO2, temperature and 2,3-DPG.
In stored blood there is a decrease in 2,3-DPG levels, hence P50 is shifted to the left
(increased affinity of Hb for O2).
In chronic anaemia there is an increase in 2,3-DPG, facilitating more tissue release of oxygen.
HbS has reduced affinity for O2, thus P50 is shifted to the right.
HbF has high O2 affinity and the curve is shifted to the left. There is also leftward shift in
carbon monoxide (CO) ingestion as Hb binds with CO 200–250 times more readily than
with oxygen.

Question 44: TTFFT


Once CO2 is released from the cells, it is transported in the blood in three ways:
• As bicarbonate ions (85%)
• Dissolved in solution (5%)
• As carbamino compounds bound to haemoglobin (10%)
CO2 combines with water in erythrocytes to form carbonic acid. This reaction is catalyzed
by carbonic anhydrase, which is absent in plasma.
CO2 combines reversibly with haemoglobin to form carbaminohaemoglobin. CO2 does
not bind to iron, as oxygen does, but to amino groups on the polypeptide chain of
haemoglobin (and to amino groups on plasma proteins).
Deoxyhaemoglobin has higher affinity for binding CO2. This is known as the Haldane
effect. Conversely, oxygenated blood has a reduced capacity for CO2.

Question 45: FFTFT


Central chemoreceptors are medullary neurones, located on the ventrolateral medullary
surface near to the exit of the glossopharyngeal and vagus nerves. They are sensitive to the
pH of their environment. The central chemoreceptors detect changes in pH of the nearby

Downloaded from https://www.cambridge.org/core. University of Edinburgh, on 19 Aug 2019 at 13:21:27, subject to the Cambridge Core terms of
use, available at https://www.cambridge.org/core/terms. https://doi.org/10.1017/9781108566100.004
56 Chapter 1b: Physiology Answers

CSF that are indicative of altered oxygen and carbon dioxide concentrations in brain tissue.
An increase in pCO2 indirectly causes the blood to become more acidic; the CSF pH is
closely comparable to plasma, as CO2 diffuses easily across the blood–brain barrier. It
should be noted that a change in plasma pH alone will not stimulate central chemorecep-
tors, as H+ cannot cross the blood–brain barrier into the CSF. Only CO2 levels affect this due
to its diffusion and subsequent reaction with water to form carbonic acid, thus decreas-
ing pH.
Peripheral chemoreceptors help maintain homeostasis in the cardiorespiratory system by
monitoring concentrations of substances within the blood. They consist of the carotid and
aortic bodies and they respond to changes in pO2, pCO2 and glucose concentration. The
carotid bodies are located on the external carotid arteries near their bifurcation with the
internal carotid artery. Each carotid body is only a few millimetres in diameter and yet has
the highest blood flow per tissue weight of any organ in the body. Afferent fibres enter the
glossopharyngeal nerve. Unlike the aortic bodies, the carotid bodies also respond to changes
in pH. Hypoxaemia, hypercapnia and acidosis lead to an increase in carotid body receptor
firing (along with a decrease in blood flow to the carotid body).
Peripheral chemoreceptors are sensitive to changes in oxygen tension (pO2), not content.
If hypoventilation becomes chronic, as in COPD, chemoreceptors lose their sensitivity
and inadequately respond to changes in CO2. When central chemoreceptors fail, peripheral
chemoreceptors attempt to regulate respiratory function and restore acid–base balance.
Since peripheral chemoreceptors are sensitive to hypoxaemia, the patient’s stimulus to
breathe now becomes reliant on low pO2. If the pO2 is increased significantly by giving
supplemental oxygen, the peripheral chemoreceptors will not stimulate breathing, resulting
in apnoea.

Question 46: FFTTF


The resting membrane potential (RMP) is not an equilibrium potential as it relies on the
constant expenditure of energy (for ionic pumps) as well as the diffusion of ions down their
concentration gradients through ion channels and, finally, permeability of the cell mem-
brane to ions. The RMP is dominated by the ion in the system with the greatest conductance;
for most cells this is potassium.
The electrical potential of a neurone is negative at rest, which is common to all excitable
tissues. This polarity is maintained by the active transport of sodium ions out of the cell and
potassium ions into the cell. Because neuronal cells could not afford to have equal and large
permeability to both ions, RMP is determined by a predominant high permeability to
potassium. Therefore, although the cell membrane is permeable to both ions (Na+ has
only 5% of the permeability of K+), many more potassium ions are able to leak out compared
to the number of sodium ions being transported in. This, in conjunction with the membrane
being impermeable to anions, means that the inside of a neurone is more electronegative
than the outside of the cell.
In a more formal notation, the RMP is the weighted average of each contributing ion’s
equilibrium potential (Goldman equation). Taking all this into account, RMP is –70 mV
(close to the equilibrium potential of K+).

Downloaded from https://www.cambridge.org/core. University of Edinburgh, on 19 Aug 2019 at 13:21:27, subject to the Cambridge Core terms of
use, available at https://www.cambridge.org/core/terms. https://doi.org/10.1017/9781108566100.004
Chapter 1b: Physiology Answers 57

Question 47: TFTFF


The Nernst equation calculates the electrical potential for an individual ion, and therefore
helps to predict how much each ion contributes to the cell membrane potential. At rest, the
Nernst potentials for potassium and chloride are similar to the real potential, but not for
sodium. This is because the resting membrane is relatively impermeable to sodium com-
pared to other ions. The electrostatic attraction of impermeable ions plays no part in the
Nernst equation.
The Nernst equation states that:

RT ½ionout
Em ¼ ×ln
ZF ½ionin

where Em = membrane potential, R = Universal gas constant, T = absolute temperature,


Z = valency, F = Faraday’s constant.

Question 48: TFTFT


When a stimulus reaches a neurone at its resting membrane potential, the gated ion
channels on the neuronal membrane open suddenly, causing an influx of sodium ions
resulting in depolarization. This influx of sodium alters the overall electric charge within
the cell, i.e. the membrane potential becomes less negative. If the neuronal membrane
potential reaches a threshold level of –55 mV, then an action potential is generated.
An action potential is an ‘all or nothing’ event, i.e. it will only arise if there is sufficient
influx of sodium ions to raise the membrane potential above the critical threshold and will
not respond to any subthreshold stimulus (this is known as the all or nothing law).
As soon as depolarization is complete, the cell ‘resets’ back to resting potential: repolariza-
tion. The Na+ channels close, beginning the neurone’s refractory period. Simultaneously,
voltage-gated K+ channels open, allowing potassium efflux and meaning the membrane
potential becomes more negative. Diffusion of potassium ions out of the cell results in
hyperpolarization after the resting potential has been restored. At this point, the Na+ channels
return to their resting state, ready to open again if the threshold potential is reached.
Tetany is a disorder of increased neuronal excitability resulting in involuntary muscle
contraction caused by conditions that increase the action potential frequency of muscle cells
or the nerves that innervate them. A tetanic contraction may be induced within skeletal
muscle if a supramaximal stimulus is applied to the corresponding neurones during the
relative refractory period. It is associated with potentiation and makes the postsynaptic
neurone more likely to fire an action potential – this is used in anaesthesia to detect a non-
depolarizing or depolarizing neuromuscular block.
Action potentials may only be propagated in one direction.

Question 49: TFFTT


There are five distinct phases in the formation of an action potential within the nerve cell:
1. A stimulus from a sensory cell/neurone causes the target cell to depolarize towards the
threshold potential (voltage-gated Na+ channels open, causing influx of sodium ions).

Downloaded from https://www.cambridge.org/core. University of Edinburgh, on 19 Aug 2019 at 13:21:27, subject to the Cambridge Core terms of
use, available at https://www.cambridge.org/core/terms. https://doi.org/10.1017/9781108566100.004
58 Chapter 1b: Physiology Answers

2. If the excitation threshold is reached, all Na+ channels open and the membrane rapidly
depolarizes.
3. At the peak action potential (approximately 30 mV), K+ channels open resulting in rapid
efflux of potassium ions. At the same time, Na+ channels close leading to the phase of
repolarization.
4. The membrane becomes hyperpolarized as potassium ions continue to leave the cell.
The hyperpolarized membrane is in a refractory period and cannot fire. The inactivated
Na+ channels cannot reopen until the membrane has approached its resting potential.
This explains the absolute refractory period – no action potential can be fired, regardless
of the magnitude of the stimulus.
5. The K+ channels close and the Na+/K+ transporter restores the resting potential.
There is no plateau phase in the neuronal action potential; this is a feature of cardiac action
potentials.

Question 50: TTFFF


Sodium channels are integral membrane proteins that form ion channels, conducting Na+
ions through the plasma membrane. They are classified according to the opening trigger; they
may be voltage gated or ligand gated. The ion channel consists of a highly processed α subunit
(plus a number of β subunits) organized into four homologous domains, each containing six
transmembrane α-helices around a central core. Three helices are negatively charged and one
is positively charged. When the membrane is depolarized, the positively charged segments
move outwards, changing the conformation of the entire structure, and the pore opens.
Although on the whole the sodium channel is selective and filters the ions passing
through it, it will also allow the passage of lithium and thallium to varying degrees.
Selection against potassium ions is vital to the function of the sodium channel; nevertheless,
potassium ions can pass through in small numbers. The channel is impermeable to bivalent
cations, such as calcium.
Local anaesthetics act by inhibiting sodium influx through voltage-gated sodium chan-
nels in the neuronal cell membrane. The receptor site is thought to be located at the
cytoplasmic (or inner) portion of the channel, i.e. internally. Therefore, local anaesthetics
must first cross the cell membrane before they are able to block the ion channel.
Sodium channel function is reduced by high hydrogen concentrations.

Question 51: FFTTT


Neurotransmitters are endogenous substances that transmit signals across a synapse from
one neurone to another. A variety of chemicals may act as neurotransmitters: amines (e.g.
norepinephrine), amino acids (e.g. glycine), polypeptides (e.g. substance P), hormones (e.g.
vasopressin) or others, such as acetylcholine or nitrous oxide.
They are packaged and stored in synaptic vesicles clustered beneath the presynaptic
membrane of the axon terminal. In response to a threshold action potential or a graded
electrical potential, a neurotransmitter is released and diffuses across the synaptic cleft,
where it binds to a specific receptor on the postsynaptic membrane of the adjacent neurone.
Neurotransmitter receptors in the postsynaptic membrane are either ligand-gated ion
channels or G-protein coupled receptors; binding causes a conformational change that
allows opening of the channels.

Downloaded from https://www.cambridge.org/core. University of Edinburgh, on 19 Aug 2019 at 13:21:27, subject to the Cambridge Core terms of
use, available at https://www.cambridge.org/core/terms. https://doi.org/10.1017/9781108566100.004
Chapter 1b: Physiology Answers 59

Neurotransmitters mostly fall into being either excitatory or inhibitory, but this is not
exclusive. Dopamine, for example, may be inhibitory at one synapse and excitatory at
another. In general, excitatory neurotransmitters open sodium channels; this leads to
depolarization of the postsynaptic membrane, promoting the generation of an action
potential. In contrast, binding of an inhibitory neurotransmitter to postsynaptic receptors
causes opening of potassium or chloride channels. The resultant hyperpolarization inhibits
the generation of an action potential.

Question 52: TFFFT


Acetylcholine is a neurotransmitter, synthesized in the cytoplasm of nerve endings from
acetyl-CoA and choline, a reaction that is catalyzed by choline acetyltransferase. It is stored
in vesicles. It is an important neurotransmitter, and acts upon:
• autonomic ganglia
• parasympathetic postganglionic nerve endings
• sympathetic nerve endings at sweat glands and some muscle blood vessels
• the neuromuscular junction
• many parts of the CNS, where it is thought to play a role in memory, cognition,
perception, attention and arousal.
It is broken down to inactive metabolites, choline and acetate, by acetylcholinesterase on the
postsynaptic membrane.

Question 53: TFTFT


GABA and glycine are inhibitory amino-acid neurotransmitters. Glutamate is by far the
most prevalent neurotransmitter, which is excitatory at well over 90% of the synapses in the
human brain; it too is an amino acid.
Substance P is a polypeptide, often co-existing with glutamate and thought to play a role
in response to painful stimuli, as well as in mood modulation. Histamine is an amine that is
involved in inflammatory responses. It is thought to be a neurotransmitter, although its role
is unclear.

Question 54: FTFFT


There are two types of GABA receptors: GABAA receptors are ligand-gated ion channels
and GABAB receptors are G-protein coupled receptors. They are located on the postsynap-
tic membrane and are found throughout the central nervous system, as well as in more
remote sites, such as placental and endocrine tissues.
Most anaesthetic agents, including propofol, thiopentone and etomidate, potentiate the
inhibitory effect of GABAA at the GABAA receptor. In addition, benzodiazepines and
volatile anaesthetic agents are also thought to target allosteric binding sites on the
GABAA receptor, modulating it indirectly.
Ketamine, however, acts as an antagonist at the NMDA receptor. Examples of drugs
acting at the GABAB receptor include baclofen and γ-hydroxybutyrate (GHB).

Downloaded from https://www.cambridge.org/core. University of Edinburgh, on 19 Aug 2019 at 13:21:27, subject to the Cambridge Core terms of
use, available at https://www.cambridge.org/core/terms. https://doi.org/10.1017/9781108566100.004
60 Chapter 1b: Physiology Answers

Question 55: TFFTF


There are multiple receptor types within the autonomic nervous system (ANS). These may
be broadly divided into either cholinergic or adrenergic receptors. Cholinergic neurones
release acetylcholine (ACh) and act on either nicotinic or muscarinic receptors. Nicotinic
ACh receptors are found in the ganglia of the ANS, in the adrenal medulla, on
postganglionic neurones in both the sympathetic and parasympathetic nervous systems
and at the motor end plate in skeletal muscle. Note that the preganglionic neurones of the
ANS directly innervate the adrenal medulla, causing release of catecholamines into the
circulation, which then act at adrenergic receptors.
Muscarinic ACh receptors are found on cells of the effector organs of the parasympathetic
nervous system. Stimulation of parasympathetic postganglionic fibres leads to ACh release
at the effector organ/cell.
Adrenergic neurones release norepinephrine (along with epinephrine and dopamine)
and are found at effector sites (i.e. postganglionic fibres) within the sympathetic nervous
system. Adrenergic receptors may be subdivided into either α or β.
Sweat glands are innervated by postganglionic fibres of the sympathetic nervous system
and contain muscarinic ACh receptors.

Question 56: TTTFF


Chemical information is transmitted intracellularly via receptors by three main
mechanisms:
• Binding of a ligand directly to ion channels, e.g. nicotinic ACh and GABA receptors
• Indirectly via a G-protein bound to the receptor that subsequently stimulates an ion
channel, e.g. muscarinic ACh
• Indirectly via a second messenger. The receptor (e.g. adrenoceptor) is still bound to a
G-protein, which in turns stimulates the production of an intracellular messenger by an
enzymatic process, e.g. cyclic adenosine monophosphate (cAMP).
Activation of G-proteins can stimulate second messengers such as cAMP, cGMP, inositol
triphosphate (ITP3) and diacylglycerol (DAG). These secondary messengers bind to, and
activate, protein kinases and ion channels, thus continuing the signalling cascade. They are
all synthesized and controlled by the enzymes adenylyl cyclase, tyrosine kinase and guanylyl
cyclase.
Glycine is a postsynaptic amino acid transmitter in the central nervous system affecting
chloride channels and is not a second messenger.

Question 57: FFTFT


G-proteins are made up of α, β and γ subunits bound to guanosine diphosphate (GDP).
When the receptor is activated by ligand binding, it causes a conformational change
allowing binding with guanosine triphosphate (GTP) in exchange for GDP. This causes
the α-GTP unit to dissociate and to further affect intracellular signalling proteins or target
functional proteins directly. The signalling pathways involve activation of a second mes-
senger. The activated protein will deactivate itself via an intrinsic GTPase; the α subunit will
then reassociate with the β and γ subunits to re-form the G-protein complex. There are three
different types of G-proteins:

Downloaded from https://www.cambridge.org/core. University of Edinburgh, on 19 Aug 2019 at 13:21:27, subject to the Cambridge Core terms of
use, available at https://www.cambridge.org/core/terms. https://doi.org/10.1017/9781108566100.004
Chapter 1b: Physiology Answers 61

• Gs stimulates/activates cAMP. It is activated via histamine and β adrenergic receptors


and binds to adenylyl cyclase, which promotes the conversion of ATP into cAMP. This
acts as a second messenger in many systems, including the synthesis and release of
hormones, e.g. thyroxine. It can also activate protein kinase A, which has a role in gene
transcription.
• Gi inhibits the cAMP pathway by inhibiting adenylyl cyclase activity. Of relevance to
anaesthesia, the following G-protein coupled receptors are coupled to the Gi subunit:
adrenergic α2, opioid, dopamine and serotonin.
• Gq is linked with α1 adrenoceptors and stimulates phospholipase C, which cleaves
phosphatidylinositol (PIP2) into two second messengers: inositol triphosphate (IP3) and
diacylglycerol (DAG). IP3 opens calcium channels and DAG stimulates protein kinases
and controls potassium, calcium and chloride channels.

Question 58: TTTTT


During anaerobic respiration, there is no contribution from the citric acid cycle or oxidative
phosphorylation, therefore glycolysis (in muscles) is the only process by which ATP may be
generated. It yields only two ATP molecules. The resultant excess of pyruvate is subse-
quently reduced to lactate (by accepting H+ from NADH, forming NAD+). Lactate is
buffered intracellularly by bicarbonate up to saturation point; further metabolism occurs
predominantly in the liver (50%), with the kidneys metabolizing a further 30%.
Under aerobic conditions, lactate is reconverted to pyruvate and enters the citric acid
cycle. If anaerobic conditions persist, it is converted to glucose via gluconeogenesis in the
liver (Cori cycle). The Cori cycle consumes a net four ATP (two made during glycolysis and
six used for gluconeogenesis). As a result, it cannot be sustained indefinitely and the
resulting plasma lactate levels rely heavily on renal excretion.
Metformin inhibits the Cori cycle and hence patients who have renal failure have reduced
lactate clearance. The ongoing glycolysis helps to generate ATP, but cannot deal with the
protons that are released on hydrolysis of ATP. Thus these concentrations rise and, unless
adequately buffered, acidosis ensues.

Question 59: FTTFF


Energy for cellular metabolism is stored within molecules of adenosine triphosphate (ATP),
specifically within the high energy bonds binding the phosphate groups to the adenosine group.
ATP is generated from several sources: glycolysis (which occurs in the cytosol), the citric
acid cycle/oxidative phosphorylation (which occur in the mitochondria), and β oxidation of
fatty acids, which also produces substrates for the citric acid cycle. Erythrocytes do not
possess mitochondria and hence cannot produce ATP.
Glycolysis involves the enzymatic conversion of the six-carbon glucose to the three-
carbon pyruvate, and the free energy provided by these reactions is used to generate the
high-energy compounds ATP and NADH.
Pyruvate is converted into the two-carbon acetyl-CoA via pyruvate dehydrogenase.
Acetyl-CoA can also be formed from fatty acid metabolism.
Acetyl-CoA is then fed into the citric acid cycle. This comprises a series of enzymatic
reactions that oxidize acetyl-CoA into two molecules of carbon dioxide. Other products of
these reactions include conversion of NAD+ to NADH and FAD+ to FADH.

Downloaded from https://www.cambridge.org/core. University of Edinburgh, on 19 Aug 2019 at 13:21:27, subject to the Cambridge Core terms of
use, available at https://www.cambridge.org/core/terms. https://doi.org/10.1017/9781108566100.004
62 Chapter 1b: Physiology Answers

These molecules then enter the pathway of oxidative phosphorylation, where they
become oxidized (lose electrons), and so release electrons on to a series of protein complexes
called the electron transfer chain. These electrons finally reduce oxygen to water. The
movement of electrons helps to create an electrochemical gradient across the inner mito-
chondrial membrane, which favours the action of ATP synthase, catalyzing the phosphor-
ylation of ADP to ATP.
ATP stores are continually regenerated, as the human body can only store enough to last
90 s. ATP releases energy when hydrolyzed to adenosine diphosphate (ADP) and again with
the loss of a second phosphate group to become adenosine monophosphate (AMP).
Hydrolysis of each bond releases up to 10–12 kcal.mol−1.
ATP synthesis is oxygen dependent and does not occur below the Pasteur point (the
critical mitochondrial pO2 for aerobic metabolism), which is approximately 0.15–0.3 kPa.

Question 60: FTFFT


Approximately 60% of male body weight is made of water but in females, it only contributes
50–55%. Intracellular fluid (ICF) makes up two-thirds of total body water (28 l) and equates
to 40% of the average male body weight. Extracellular fluid (ECF) makes up the remaining
third (14 l) and is composed of several components: interstitial fluid (9.5 l), plasma (3.5 l)
and transcellular fluid (1 l). Body water is higher in the neonate, and is distributed in
different proportions. It is more pronounced in prematurity, when ECF is greater than ICF.
The full-term neonate has a total body water of approximately 75%, compared with over
80% in the preterm neonate. Over the first few years of life, the distribution of body water
changes to match that of the adult.
Transcellular fluids are those separated from the plasma by an epithelium, e.g. bile, CSF,
pleural, peritoneal, pericardial and intraocular fluid. CSF is an ultrafiltrate of plasma which
is modified by ependymal cells. It is hypokalaemic, hypoglycaemic, hyperchloraemic and
hypoproteinaemic relative to plasma. The sodium and bicarbonate are similar in both, but
CSF is more acidic, despite having the same osmolality.

Question 61: TFFFF


The functional unit of the kidney is the nephron, and there are in excess of 1 million
nephrons in each kidney. The nephron is comprised of a glomerulus, a renal tubule and a
collecting duct. The renal tubule is further divided into proximal and distal convoluted
tubules, which are connected by the loop of Henle.
The glomerulus, proximal and distal convoluted tubules are situated within the renal cortex.
Blood is filtered at the glomerulus to form an ultrafiltrate of water and solutes (e.g. sodium,
potassium, chloride, urea, glucose) which passes into the proximal tubule. This is the predomi-
nant site for solute reabsorption (70%) and the surface area available for this process is increased
by the presence of microvilli on the luminal side of the epithelial lining (the brush border).
The distal end of the proximal tubule becomes the loop of Henle, which extends into the
renal medulla before joining with the distal tubule. The loop has a water-permeable
descending limb, and an ascending limb (which has thin and thick segments), lined by
cuboidal epithelial cells rich in mitochondria, which are impermeable to water.
These cells actively transport approximately 20% of filtered sodium against its concen-
tration gradient into the medulla via a Na+−K+−2Cl− co-transporter. This process helps to

Downloaded from https://www.cambridge.org/core. University of Edinburgh, on 19 Aug 2019 at 13:21:27, subject to the Cambridge Core terms of
use, available at https://www.cambridge.org/core/terms. https://doi.org/10.1017/9781108566100.004
Chapter 1b: Physiology Answers 63

generate a hypertonic medulla, so that when the increasingly dilute ultrafiltrate passes
through the collecting duct, water can be reabsorbed via aquaporins under the control of
antidiuretic hormone. This ultimately controls the volume of urine produced and maintains
plasma osmolarity. Only 15% of the loops of Henle are intramedullary and thus able to
contribute to increasing medullary tonicity. The remaining 85% have short loops that
remain within the cortex and do not make a significant contribution.
The distal tubule reabsorbs approximately 12% of filtered sodium and secretes potassium
and hydrogen into the filtrate. Only the distal part of the distal tubule is water permeable.

Question 62: TTFFF


Renal blood flow is constant and independent of perfusion pressure between mean arterial
pressures of 80–180 mmHg. This process is called autoregulation. The mechanism is still
debated. The myogenic theory proposes that arteriolar tone changes in response to trans-
mural pressures and tubuloglomerular feedback. Autoregulation continues even in the
denervated kidney; the sympathetic nerve supply does, however, have some capacity to
constrict both afferent and efferent arterioles, as well as decrease renal blood flow. There is
little effect on the GFR. Indirectly, the sympathetic nerves have an effect via β1 receptors by
stimulating the release of renin from the juxtaglomerular apparatus. Consequently, angio-
tensin II is released, which vasoconstricts the efferent arterioles and decreases blood flow.
Catecholamines only constrict the afferent arterioles.
The tubuloglomerular feedback loop incorporates the juxtaglomerular apparatus and the
macula densa in the ascending limb of the loop of Henle. The afferent arteriole contracts
under the influence of adenosine released from here.
Prostaglandins and prostacyclins increase renal cortical flow, which is disrupted by
NSAIDs. A high-protein diet raises glomerular capillary pressures and increases renal
blood flow. Weight-lifters on a high-protein diet need to drink a increased volume of
water to compensate.

Question 63: TFTTT


The kidneys receive 20–25% of the cardiac output, despite only weighing 0.5% of total body
weight. Renal blood flow to the cortex is more than ten times that of the medulla. The renal
arteries enter the hilum where they branch into the segmental arteries, dividing further into
interlobar arteries. These arteries then supply blood to arcuate arteries that run through the
boundary of cortex and medulla. Branches of interlobular arteries from the arcuate arteries
travel to the glomerular capsule, where they form afferent arterioles and divide into
glomerular capillaries. These collect and drain into the efferent arteriole. The efferent
arteriole still carries oxygenated blood and nutrients to supply the renal cells. It branches
into two capillary networks: the peritubular capillaries and the vasa recta.
The peritubular capillaries are predominantly cortical and supply the proximal and distal
convoluted tubules. The vasa recta run parallel to the loop of Henle in the medullary
pyramids to take part in the countercurrent multiplication mechanism. They then drain
into the interlobar veins and on to the renal vein. Thus, technically, the efferent arterioles
are portal vessels, i.e. they carry blood from a capillary system (glomeruli) to a secondary
capillary network (peritubular). Uniquely, the kidney is the only site in the human body
where capillaries drain into arterioles.

Downloaded from https://www.cambridge.org/core. University of Edinburgh, on 19 Aug 2019 at 13:21:27, subject to the Cambridge Core terms of
use, available at https://www.cambridge.org/core/terms. https://doi.org/10.1017/9781108566100.004
64 Chapter 1b: Physiology Answers

Question 64: TFTFT


The glomerulus consists of a network (or ‘knot’) of capillaries encased in a cupped end of the
renal tubule called the Bowman’s capsule. Blood is transported to the glomerulus from the
renal artery via the afferent arterioles, and leaves via the efferent arterioles. These then form
the vasa recta, which are involved in the countercurrent exchange system, before draining
into venules and ultimately the renal vein. This is an example of a portal system; a system of
blood vessels that begins and ends with capillaries.
The functional unit formed by the glomerulus and Bowman’s capsule is the site of
filtration of blood, across the capsule and into the tubule.
Renal blood flow is approximately 20% of the cardiac output (around 1 l.min−1) and renal
plasma flow is approximately 600 ml.min−1. Glomerular filtrate is produced across
Bowman’s capsule at approximately 120 ml.min−1 (the glomerular filtration rate), which
represents a filtration fraction of 20%. This means that around 180 litres of glomerular
filtrate is produced over 24 hours, the vast majority of which is reabsorbed from the renal
tubules, with the minority excreted as urine.
The glomerular capillary endothelium is fenestrated, but the basal lamina of Bowman’s
capsule does not contain any pores. The main determinant of filtration is molecular size.
Hence, glomerular filtrate has a similar concentration of small ions to plasma, but is
essentially protein-free in health.
Filtration through the glomerulus depends on Starling’s forces; passage of fluid out of the
capillaries is favoured by high capillary hydrostatic pressure and low oncotic pressure
within Bowman’s capsule. This is the case at the afferent arteriolar end, where hydrostatic
pressure (45 mmHg) is higher than the oncotic pressure (25 mmHg) and there is net flow
outwards. At the efferent arteriolar end of the capillary, plasma oncotic pressure is higher
(35 mmHg). This is because proteins are unable to permeate through the basement
membrane and are retained within the plasma. Although the capillary hydrostatic pressure
falls as fluid and solutes leave, the portal system means that this pressure drop is not as
significant as that across normal capillaries, and it remains high (>30 mmHg). Since the
oncotic and hydrostatic pressures are in equilibrium, there is zero net filtration.

Question 65: TTTFT


The body needs a daily balanced supply of proteins, carbohydrates and fats, as well as
vitamins, minerals, electrolytes, trace elements and water. The average daily adult require-
ments are thought to be: 30–40 ml.kg−1 water, 1 mmol.kg−1 sodium and potassium, 0.1–
0.2 mmol.kg−1 calcium and magnesium, 0.2 mg.kg−1 iron and zinc and finally, vitamins in
small amounts. Patients with increased metabolic rates as seen in trauma, sepsis, surgery or
burns will therefore have increased energy and nutrient requirements.
The daily requirement of nitrogen is 0.15 g.kg−1.day−1 and protein is 0.8–1 g.kg−1.day−1.
6.25 g of protein yields 1 g of nitrogen. Vitamins are essential, i.e. they cannot be synthesized
by the body. These include the fat-soluble vitamins A, D, E and K, and the water-soluble
family of vitamin B, which can be found in the majority of fortified cereals.
Vitamin B deficiencies give rise to many well-known conditions: vitamin B1 (thiamine)
deficiency = Wernicke’s encephalopathy or beriberi, vitamin B3 (niacin) deficiency =
pellagra, vitamin B6 (pyridoxene) deficiency = peripheral neuropathy and stomatitis and
vitamin B12 (cobalamin) deficiency = megaloblastic anaemia.

Downloaded from https://www.cambridge.org/core. University of Edinburgh, on 19 Aug 2019 at 13:21:27, subject to the Cambridge Core terms of
use, available at https://www.cambridge.org/core/terms. https://doi.org/10.1017/9781108566100.004
Chapter 1b: Physiology Answers 65

Question 66: FFTTT


The glomerular filtration rate (GFR) measures the rate at which blood is filtered by the
kidneys. Clearance is defined as the volume of plasma that is cleared of substance per unit
time. Clearance or GFR is measured most accurately by calculating inulin clearance, but
creatinine is a useful surrogate. Inulin is a polysaccharide that is easily filtered, but is not
reabsorbed, secreted, synthesized or metabolized by the kidney. This means any inulin
present in the urine must have been filtered by the glomerulus.
The formula for calculating clearance is:
Cx ¼ Ux V=Px

where C is clearance, x is the substance to be cleared, U is the urinary concentration, V is


urine flow and P is plasma concentration.
Normal GFR is 120 ml.min−1, which is equal to 180 l.day−1. GFR is dependent on the
same factors that govern filtration across any capillary bed: effective glomerular surface area,
permeability of the capillary wall, and hydrostatic and osmotic gradients across the capillary
walls. The mesangial cells located between the capillary endothelium and the basement
membrane have a contractile function that alters the effective glomerular surface area. The
mesangial cells contract in response to angiotensin II, vasopressin, noradrenaline, leuko-
trienes, histamine and certain prostaglandins, reducing GFR. Relaxation occurs in response
to dopamine and atrial natriuretric peptide, thus increasing GFR. Anaesthetic agents tend to
decrease the SVR; the subsequent reduction in mean arterial pressure and renal blood flow
will reduce GFR.
GFR is low in infants. It decreases in old age and increases in pregnancy. GFR is
remarkably constant in humans; variation in water excretion and solutes depends on
changes in tubular reabsorption and secretion, not on changes in GFR.

Question 67: FTTTT


A buffer is a solution that resists changes in pH when an acid or a base is added to it. It
consists of a weak acid and its conjugate base.
There are a number of physiological buffer systems in the body. The bicarbonate/
carbonic acid system is the most important; other systems include haemoglobin, plasma
proteins, phosphate, calcium carbonate (in bone) and ammonia (in urine).
The bicarbonate/carbonic acid buffer system is the main buffer in the blood. The
Henderson–Hasselbalch equation for this buffer system is:
½HCO 3
pH ¼ 6:1 þ log
½H3 CO3 

The ideal buffer has a pKa within one unit of the desired pH. Despite having a low pKa
(6.1) relative to blood pH, the bicarbonate/carbonic acid buffer system is still effective
due to the ready excretion of carbonic acid in the form of CO2 by the lungs, and the
continuous regeneration of bicarbonate by the kidneys. The bicarbonate/carbonic acid
buffer system is more efficient at buffering acids since its efficiency increases as the pH
falls.
Haemoglobin acts as a blood buffer by dissociation of the imidazole groups of its histidine
residues. Deoxygenated blood is a better buffer than oxygenated haemoglobin because its

Downloaded from https://www.cambridge.org/core. University of Edinburgh, on 19 Aug 2019 at 13:21:27, subject to the Cambridge Core terms of
use, available at https://www.cambridge.org/core/terms. https://doi.org/10.1017/9781108566100.004
66 Chapter 1b: Physiology Answers

imidazole groups dissociate less (the Haldane effect). Haemoglobin has six times the buffering
capacity of plasma proteins. Urinary buffering occurs in the proximal and distal convoluted
tubules and collecting ducts. The main buffers in the urinary system are the bicarbonate/
carbonic acid system, phosphate (HPO 4 =H2 PO4 ) and ammonia (NH3 =HN4 ).
2 þ

In chronic renal failure, more acid is produced than excreted. Extracellular buffers
become depleted and plasma bicarbonate levels are reduced. Bicarbonate reabsorption
and regeneration are also reduced. Haemoglobin levels are low, secondary to the reduced
production of erythropoietin, which also reduces buffering capability in the blood. Less
ammonia is produced by the damaged nephrons, which in turn reduces the buffering
capability of the urine. Excess acid may be buffered by calcium carbonate in bone, con-
tributing to renal osteodystrophy.

Question 68: FTFFF


The kidney has a significant endocrine function; it produces and excretes several hormones
(renin, erythropoietin, active vitamin D) as well as being the target site for others (ADH,
aldosterone, PTH, calcitonin, ANP).
Renin is produced and secreted in the juxtaglomerular apparatus of the kidney in
response to hypovolaemia, hyponatraemia, cardiac failure and cirrhosis. Renin converts
angiotensinogen to angiotensin I. Angiotensin converting enzyme converts angiotensin I to
angiotensin II in the lungs. Angiotensin II acts as a potent vasoconstrictor and stimulates
production of aldosterone by the adrenal cortex and release of vasopressin from the poster-
ior pituitary. Aldosterone acts on the distal tubules of the kidney and promotes sodium and
water reabsorption in exchange for K+ and H+. Vasopressin causes increased permeability of
the collecting ducts to water via an action on the aquaporin-2 receptor. The result is
conservation of water from the distal nephron and less urine production.
Erythropoietin is a glycoprotein hormone secreted mainly by the kidneys, but also by the
liver. Erythropoietin secretion is increased in response to hypoxia and haemorrhage and
inhibited by increased numbers of circulating red blood cells. It causes an increase in
erythropoiesis in the bone marrow.
1-α hydroxylase hydroxylates 25-hydroxycholecalciferol to the active form of vitamin D
(1,25-dihydroxycholecalciferol) in the proximal tubule. Active vitamin D is actually a
hormone; it helps to regulate bone calcium and phosphorus levels by increasing absorption
of dietary calcium and phosphorus.

Question 69: FTTTF


pH is used as an indicator of acidity and is defined as the negative logarithm to the base 10 of
hydrogen ion concentration, i.e. pH= –log10[H+]. Each decrease of one pH unit is equivalent
to a ten-fold increase of [H+]. pH of normal arterial blood is 7.35–7.45 and this corresponds
to [H+] of 35–45 nmol.l−1. Temperature affects pH: pH decreases as the temperature
increases. The Rosenthal correction factor is used to correct the pH measured in a blood
gas analyzer according to the patient temperature.
Base deficit is the amount of acid or base required to restore 1 litre of blood to normal pH
at a pCO2 of 5.3 kPa at body temperature. Base deficit is positive in alkalosis and negative in
acidosis.

Downloaded from https://www.cambridge.org/core. University of Edinburgh, on 19 Aug 2019 at 13:21:27, subject to the Cambridge Core terms of
use, available at https://www.cambridge.org/core/terms. https://doi.org/10.1017/9781108566100.004
Chapter 1b: Physiology Answers 67

Anion gap is the difference between the measured cation and anion concentrations in the
plasma. The normal anion gap is 8–16 mmol.l−1 and can be calculated roughly as:

½Naþ þ Kþ   ½Cl þ HCO


3

Calculating the anion gap is useful when differentiating the cause of metabolic acidosis. A
high anion gap acidosis indicates the presence of unmeasured anions such as alcohol,
methanol, ketones, lactate and exogenous acids. A normal anion gap acidosis is caused by
hyperchloraemia, bicarbonate loss (GI loss, acute tubular acidosis) or retention of H+ ions.
The Siggaard–Anderson nomogram is a graph showing the log of the arterial CO2 on the
y-axis and plasma pH on the x-axis. Base excess, buffer base (haemoglobin) and bicarbonate
are added as separate lines. The nomogram can be used to calculate paCO2, although now
modern blood gas analyzers automatically perform the required calculations.

Question 70: FFTTF


Maintenance of stable pH in body fluids is necessary for normal enzyme function, ion
distribution and protein structure. pH homeostasis is achieved by three mechanisms:
buffering, compensation and correction.
Buffers work immediately to maintain a normal pH, whereas compensation and eventual
correction occur much more slowly. The body accommodates disturbances in acid–base
balance by using respiratory and renal compensation.
Respiratory compensation occurs quickly, but the acid–base state can never be fully
corrected by this mechanism. The lungs excrete a huge amount of respiratory acid (12 000–
13 000 mmol.day−1) compared to the kidneys (70–100 mmol.day−1).
Renal compensation takes longer to occur (over hours to days), but it can fully correct the
acid–base disturbance. The kidney’s role in acid–base balance is mainly concerned with acid
excretion and reabsorption of filtered bicarbonate.
Normal plasma bicarbonate levels are 22–26 mEq.l−1. The proximal tubule is the site for
bicarbonate reabsorption (90%) and ammonium production. Under normal circumstances,
less than 0.1% of filtered bicarbonate is excreted in the urine. The distal tubule is where the
majority of hydrogen ions are excreted into the urine. Normal urine has a pH of 4.5–5; the
kidneys are unable to produce urine with pH <4.4.

Question 71: TTFFF


Antidiuretic hormone (ADH) or vasopressin is a neuropeptide hormone produced in the
hypothalamus (supraoptic nucleus) and stored in the posterior pituitary. ADH release is
triggered by increased plasma osmolality, haemorrhage, pain, stress response and angio-
tensin II. ADH release is increased by nicotine, morphine and barbiturates. ADH release is
inhibited by alcohol.
ADH increases water permeability by activating the V2 receptor (G-protein coupled) on
the peritubular side of the collecting tubule cell which opens water channels (aquaporin-2)
on the luminal side of the collecting tubule cell. Maximal ADH release will cause water
retention, highly concentrated urine (minimum 300 ml.24h−1) with a high [Na+] of
approximately 600 mmol.l−1. In the absence of ADH, urinary [Na+] may be as low as 80–
100 mmol.l−1.

Downloaded from https://www.cambridge.org/core. University of Edinburgh, on 19 Aug 2019 at 13:21:27, subject to the Cambridge Core terms of
use, available at https://www.cambridge.org/core/terms. https://doi.org/10.1017/9781108566100.004
68 Chapter 1b: Physiology Answers

Diabetes insipidus (DI) can be divided into central (a deficiency in ADH secretion) or
nephrogenic (kidneys are unresponsive to ADH). DI is characterized by symptoms of
polyuria and polydipsia (large volumes of dilute urine) with raised plasma osmolality.
Patients are unable to concentrate their urine in response to water deprivation. Central
DI occurs in head injury, neurosurgery and, in a small number of cases, it can be familial.
Treatment is with synthetic ADH replacement. Drugs can also cause nephrogenic DI e.g.
lithium, gentamicin or demeclocycline. Rarely, it may be the result of an X linked recessive
disorder; its treatment is with thiazide diuretics.

Question 72: TFFFT


The function of the loop of Henle is to enable the body to produce concentrated urine. It
achieves this by employing a sodium countercurrent multiplier mechanism to produce
hypertonic conditions in the renal medulla. This provides a gradient for water reabsorption
from the collecting duct (under the control of ADH). Fluid entering the loop of Henle is
isotonic to plasma, but after traversing the loop of Henle the fluid becomes increasingly
dilute. As the tubular fluid enters the distal tubule, it is hypotonic compared to plasma.
The active part of the countercurrent multiplication mechanism occurs in the ascending
limb of the loop of Henle. In the ascending limb, sodium and chloride (Na+/2Cl−/K+ co-
transporter) are actively pumped out of the tubule into the interstitium. This process is
inhibited by loop diuretics. As the interstitium becomes hypertonic, water from the des-
cending limb moves out down a concentration gradient, concentrating the urine within the
tubules. The ascending limb is impermeable to water, but is able to actively transport Na+
and Cl− out, resulting in a dilute, hypotonic tubular fluid reaching the distal tubule.
The glomeruli, proximal tubules and distal tubules are situated in the cortex, whilst the loop
of Henle and collecting ducts extend down through the medulla. The length of a nephron’s loop
of Henle depends on the location of the glomerulus – if the glomerulus is in the outer two-thirds
of the cortex they are called cortical nephrons (85%) and have very short loops of Henle.
Nephrons whose glomeruli are in the inner one-third of the cortex (juxtamedullary nephrons –
15%) have long loops of Henle. Cortical nephrons do not make a significant contribution to the
manufacture of medullary hypertonicity that leads to concentration of urine.
The vasa recta are capillaries derived from the efferent arterioles of the juxtamedullary
nephrons. The function of the vasa recta is to provide nutrients to the renal medulla and
remove waste products without washing away the solutes responsible for medullary hyper-
tonicity. The ‘hairpin’ arrangement of the vasa recta means that the osmotic gradient built
up by the loops of Henle is not dissipated and also allows the vasa recta to function as
countercurrent exchangers (distinguished from countercurrent multipliers by the fact that
no energy is required).

Question 73: FTTFT


The maintenance of potassium balance is bought about by regulation of potassium ion
excretion. The majority of potassium ions are stored intracellularly.
In the proximal tubule, approximately 70% of filtered K+ is passively reabsorbed. The
distal tubule and collecting ducts passively secrete potassium ions into the tubular fluid,
which results in potassium appearing in the urine. Excretion of K+ is enhanced by increased
cellular potassium concentration and aldosterone release. Potassium excretion is also

Downloaded from https://www.cambridge.org/core. University of Edinburgh, on 19 Aug 2019 at 13:21:27, subject to the Cambridge Core terms of
use, available at https://www.cambridge.org/core/terms. https://doi.org/10.1017/9781108566100.004
Chapter 1b: Physiology Answers 69

dependent on distal tubular flow rate. Diuretics increase distal tubular flow and cause
increased K+ secretion by lowering the tubular K+ concentration.
Acidosis can cause hyperkalaemia even when the body’s stores of potassium are normal.
Insulin promotes potassium ion entry into cells and is used in the initial treatment of
hyperkalaemia.
Aldosterone antagonists (spironolactone) and Na+ channel blockers (amiloride) are
termed potassium-sparing diuretics. They have little diuretic action, but do cause natriur-
esis and reduced H+ and K+ secretion in the distal tubule. Potassium-sparing diuretics can
help to reduce the disturbances of K+ balance that occur with the use of loop diuretics alone.

Question 74: TFFTT


Virtually all the sodium that is filtered into the nephron is reabsorbed back into the
circulation and <1% is excreted into the urine.
70% of filtered sodium is reabsorbed in the proximal tubule; the primary transporter
involved is Na+/K+ ATPase.
20% of filtered sodium is reabsorbed via the Na+/Cl−/K+ pump in the loop of Henle.
5% of filtered sodium is reabsorbed in the early distal tubules via the Na+/ Cl− symport.
5% of filtered sodium is reabsorbed in the late distal tubule and collecting ducts via
sodium channels.
The regulatory hormones involved in sodium reabsorption are angiotensin II, aldoster-
one and atrial natriuretic peptide.
Reabsorption of other solutes is dependent on sodium reabsorption. Sodium reabsorp-
tion leads to electrical, concentration and osmotic gradients for the passive reabsorption of
such solutes as chloride, potassium and urea, and for water reabsorption. It is also important
for the reabsorption of glucose, amino acids, phosphate, calcium and bicarbonate, and for
the secretion of H+.

Question 75: TFTFF


There are six compensatory mechanisms that are described in response to acute haemor-
rhage: baroreceptor reflexes, chemoreceptor reflexes, cerebral ischaemic response, reab-
sorption of tissue fluid into the plasma compartment, release of endogenous catecholamines
and renal conservation of sodium and water.
The baro- and chemoreceptor reflexes result in activation of the sympathetic nervous
system, which serves to increase systemic vascular resistance and cardiac output. Reduced
effective circulating volume and decreased venous return decrease atrial stretch. This
reduces ANP levels and increases ADH release. Decreased renal perfusion results in activa-
tion of the renin–angiotensin–aldosterone system, causing an increase in renal sodium
reabsorption and vasoconstriction. Sodium and water reabsorption leads to restoration of
the effective circulating volume after haemorrhage.

Question 76: FFFTT


Glutamate is the most prevalent excitatory neurotransmitter within the brain, with glycine
predominating in the brainstem. However, GABA (γ-aminobutyric acid) receptors are the
most important inhibitory receptors. Cells have an electronegative internal charge,

Downloaded from https://www.cambridge.org/core. University of Edinburgh, on 19 Aug 2019 at 13:21:27, subject to the Cambridge Core terms of
use, available at https://www.cambridge.org/core/terms. https://doi.org/10.1017/9781108566100.004
70 Chapter 1b: Physiology Answers

maintaining the membrane potential. Within the dorsal horn, amino acids and neuropep-
tides activate AMPA and NK-1 receptors to produce depolarization of nerve cells. They
subsequently cause secondary NMDA receptors to be activated indirectly. The effects of
opiates can indeed be potentiated by α-agonists at the spinal level. Drugs include clonidine
and dexmedetomidine.

Question 77: TFTTT


The limbic system is concerned with the expression and recall of emotions such as
fear, anger and pleasure. Damage to specific areas of the limbic system causes loss of
components of emotion. The hypothalamus is part of the limbic system, but is
concerned with unconscious regulation of homeostasis. The amygdala is an interesting
part of the brain. It is responsible for the subconscious physiological response to an
unrelated somatic stimulation from a previously learned experience. As an example, I
get an overwhelming sense of nausea when I see the RAC centre on the M6 – as it
reminds me of long car journeys as a child when I used to suffer terribly from travel
sickness! Carbon monoxide causes many physiological disturbances, one being damage
to the longitudinal fasciculus. This leaves an individual unable to identify objects,
despite ability to view them clearly. This is known as object agnosia. In addition, it
causes prosopagnosia – an inability to recognize faces. Alexia – where an individual is
unable to understand the written word – may come in several forms. Pure alexia
(alexia without agraphia, the inability to write) may be seen in lesions of the corpus
callosum, left occipital lobe or visual word form area.

Question 78: FFFTT


The blood–brain barrier (BBB) is indeed a physical barrier within the brain. It provides a
healthy environment for neurones. It is made up of tight junctions (zona occludens). The
Zona Central refers to an area in Chile. It is the home of the majority of the Chilean
population, and therefore houses many blood–brain barriers! The BBB is permeable to
water, oxygen and carbon dioxide, but not to ions such as sodium, potassium or proteins.
Due to the structure of the BBB, all the drainage of the cranium is via the venous system –
there is no lymphatic drainage. As the barrier is semi-permeable, administration of hypo-
tonic solutions causes an increase in brain water. There is approximately 150 ml of CSF in a
healthy adult, split equally between the cranium and spinal cord. It is produced by the
choroid plexus at 500 ml.day−1.

Question 79: TFTTF


Almost all the energy supplied to the brain is provided by oxidative phosphorylation of
glucose to produce ATP. Cerebral metabolic rate for oxygen (CMRO2) is generally kept
constant. However, hypothermia, unconsciousness and anaesthesia reduce cerebral activity,
and therefore CMRO2 falls accordingly. CMRO2 and CBF are proportional. A drop in body
temperature from 37 °C to 27 °C halves CMRO2. Almost all cerebral activity ceases when
body temperature drops below 20 °C. A patient with local anaesthetic toxicity will have a
decreased CMRO2 if comatose, or an increased CMRO2 if fitting. However, the CBF will be
proportionally increased or decreased along with the CMRO2.

Downloaded from https://www.cambridge.org/core. University of Edinburgh, on 19 Aug 2019 at 13:21:27, subject to the Cambridge Core terms of
use, available at https://www.cambridge.org/core/terms. https://doi.org/10.1017/9781108566100.004
Chapter 1b: Physiology Answers 71

Question 80: FTTTF


CBF is proportional to paCO2. Cerebral autoregulation refers to the maintenance of a
constant blood flow over a range of blood pressures (Figure 1.80.1). Hypoxia (paO2 <6
kPa) causes cerebral vasodilatation secondary to metabolic acidosis and an increase in CBF.
Cerebral perfusion pressure is indeed approximately equal to the difference between mean
arterial pressure and intracranial pressure. Inhalational anaesthetics decrease CMRO2 by
decreasing cerebral activity. This produces the small corresponding decrease in blood flow
required. However, inhalational anaesthetics cause an uncoupling of the CBF–CMRO2
relationship. Cerebral vasodilation that results ultimately increases CBF. CO2 reactivity is
normal with inhalational anaesthetics in the absence of head injury. However, in the
presence of cerebral oedema, the vasoconstrictive effect of hypocarbia is lost when com-
pared with intravenous opiates. This is an important consideration when anaesthetizing
patients for ICU or surgery. CBF is also regulated by the autonomic nervous system.

Cerebral blood flow maintained


100 at approx. 50 ml/100g/minute
over a range of cerebral
Cerebral blood flow (ml/100g/min)

perfusion pressures
80

60

40

20

30 60 90 120 150 180


Cerebral perfusion pressure (mmHg)

Figure 1.80.1 Cerebral autoregulation.

Question 81: TFFTT


‘Fast’ pain stimuli are those that pass via peripheral nociceptors along primary afferent Aδ
fibres into the dorsal horn of the spinal cord (1st order neurone), and synapse in laminae I,
V and X. They enter the spinal cord, decussate at the same level, and largely travel towards
the brain in the contralateral spinothalamic tracts (2nd order neurone).
There are many compounds that activate and sensitize nociceptors in the periphery.
Potassium, serotonin, bradykinin, protons, histamine and ATP are some of the common
nociceptive agents, of which bradykinin is the most potent. It acts on the bradykinin (B2)
receptor. Some of the compounds known to sensitize peripheral nociceptors include

Downloaded from https://www.cambridge.org/core. University of Edinburgh, on 19 Aug 2019 at 13:21:27, subject to the Cambridge Core terms of
use, available at https://www.cambridge.org/core/terms. https://doi.org/10.1017/9781108566100.004
72 Chapter 1b: Physiology Answers

prostaglandins, leukotrienes, cytokines, nerve growth factor, neuropeptides, noradrenaline


and nitric oxide.
C-fibres are known as ‘slow pain fibres’, are unmyelinated and conduct impulses at a
speed of 2 m.s-1. The periaqueductal area of the midbrain provides the majority of the
inhibitory control of pain (descending inhibitory pathways).

Question 82: FFTTF


The dorsal column is located at the posterior aspect of the spinal cord, between the dorsal
median sulcus and the dorsal horn. It is not within the dorsal horn, but posterior to it. It
comprises two tracts: the fasciculus cuneatus (taking fibres primarily from the upper limbs)
and the fasciculus gracilis (serving the lower limbs). A unilateral lesion at the level of the
sixth thoracic vertebra will indeed cause proprioceptive disturbance in the ipsilateral leg.
Fibres run in the ipsilateral side of the dorsal column carrying primary afferent neurones,
ascending uninterrupted to the medulla oblongata where they terminate in the nuclei
gracilis and cuneatus. Dorsal nerve roots must not be confused with dorsal columns.
Dorsal nerve roots do not involve motor control, rather sensory function.

Question 83: TTTTT


All of the above are true. Acetylcholine (ACh) acts within the peripheral (PNS) and central
(CNS) nervous systems. Within the PNS it acts on the neuromuscular junction to cause
skeletal muscle contraction and the muscarinic cardiac fibres to inhibit cardiac muscle
contraction. In the autonomic nervous system (ANS) it is released from all pre- and
postganglionic parasympathetic neurones, all preganglionic sympathetic neurones, and
postganglionic sweat glands. This is an exam favourite!

Question 84: FTTFT


Neuromodulation techniques involve stimulation of a peripheral nerve, spinal cord, deep
brain or motor cortex and drug administration via the intraspinal or intra-cerebroventri-
cular route. Unsuitable patients include those with a major psychiatric disorder, poor
understanding or compliance, those with poor social support and patients who abuse
recreational and prescribed drugs. Peripherally, stimulation of Aβ proprioceptive fibres
causes suppression of small-diameter, high-threshold Aδ and C sensory fibres in the dorsal
horn. For spinal cord stimulation techniques, a pulse generator is required, but is usually
implanted within the abdominal wall, not the abdominal cavity. As you would expect with
such invasive procedures, randomized control trials have been conducted. They have shown
an improvement in pain control, quality of life and decreased social dependence with the use
of neuromodulation techniques. In addition, they have been shown to be cost effective due
to the reduction in hospitalization and medicinal use.

Question 85: TTTFT


The spinothalamic tract is a sensory pathway originating in the spinal cord, situated
anterolateral to the ventral horn. The axons ascend the spinal cord, carrying information
regarding pain, temperature, itch, non-discriminative touch and pressure. Some authorities
suggest that the spinothalamic tract is split into lateral and anterior components. Pain and

Downloaded from https://www.cambridge.org/core. University of Edinburgh, on 19 Aug 2019 at 13:21:27, subject to the Cambridge Core terms of
use, available at https://www.cambridge.org/core/terms. https://doi.org/10.1017/9781108566100.004
Chapter 1b: Physiology Answers 73

temperature are preferentially carried by the lateral tract, whereas non-discriminative touch
and pressure are transmitted via the anterior component. Both tracts decussate at the level
of the spinal cord, rather than in the brainstem, and this usually occurs one to two spinal
nerve segments above the point of entry. The axons, once in the rostral ventromedial
medulla, move dorsally and synapse with 3rd order neurones in several thalamic nuclei.

Question 86: TFTFT


Control of movement is governed by the cerebral cortex, the cerebellum and the basal
ganglia.
The motor cortex is situated in the frontal lobe and is divided into three parts:
• The primary motor cortex in the precentral gyrus, where different parts of the body are
represented; those with finer movements have larger representation
• The premotor area, concerned with postural movement at the beginning of voluntary
movements
• The supplemental motor area concerned with planning of complex movements.
The cerebellum, situated in the posterior fossa, is divided into three parts:
• The vestibulocerebellum, which maintains the body’s equilibrium in motion
• The spinocerebellum, which is mainly concerned with proprioception
• The neocerebellum, which is involved in planned execution of voluntary movement.
The basal ganglia are situated at the base of the forebrain and comprise multiple subcortical
nuclei: the caudate nucleus, putamen, globus pallidus, subthalamic nucleus, nucleus accum-
bens and substantia nigra. Together with the cerebellum, they influence voluntary move-
ment by carrying out the various subconscious movements necessary to perform a voluntary
activity.

Question 87: FFFTF


The electroencephalogram (EEG) is an example of how we use biological signals to monitor
organ function. The brain produces small, complex signals and the summated product of
inhibitory and excitatory postsynaptic activity is combined to form the EEG. These are small
amplitude, varying voltage signals of the order of 1–500 μV. The EEG of an awake patient
will have low amplitude and high frequency signals, and deepening anaesthesia will lead to a
progressive increase in the amplitude and decrease in frequency. For ease of interpretation,
the frequency spectrum of the EEG is divided into four bands:
α – normal, 8–13 Hz
β – normal, >13 Hz
δ – abnormal, <4 Hz
θ – sometimes abnormal, 4–7 Hz
Signals from two symmetrical pairs of electrodes are combined to form the cerebral function
analyzing monitor (CFAM), which will show the overall mean amplitude and relative power
in each frequency band. Cortical θ (theta) waves are frequently observed in young children.
In older children and adults, they tend to appear during meditative, drowsy or sleeping
states, but not during the deepest stages of sleep. Several types of brain pathology can give
rise to abnormally strong or persistent cortical theta waves.

Downloaded from https://www.cambridge.org/core. University of Edinburgh, on 19 Aug 2019 at 13:21:27, subject to the Cambridge Core terms of
use, available at https://www.cambridge.org/core/terms. https://doi.org/10.1017/9781108566100.004
74 Chapter 1b: Physiology Answers

Question 88: TTFFT


Auditory evoked potentials are very low-amplitude signals (1–2 μV) extracted from the
background EEG and generated using an auditory stimulus. The signals are a series of peaks
and troughs; the timing reflects the signal origin within a specific area of the brain. The early
signal (0–10 ms) is from the brainstem, the middle signal (10–100 ms) is the early cortical
response and the late signals (100–1000 ms) represent the late cortical response.
The signal is thought to have six peaks, although this is up for debate. The signals are not
significantly affected by intravenous anaesthetic agents, thereby limiting their use in total
intravenous anaesthesia. The early cortical waves of auditory evoked potentials correlate
with the implicit and explicit memory of events under anaesthesia.

Question 89: TTTTT


The hypothalamus is a gland located in the floor of the third ventricle. It has connections to the
brainstem, pituitary gland and cerebrum. Together with the limbic structures and the endocrine
system, it controls the endocrine system, as well as many aspects of emotional behaviour.
Central thermoreceptors within the hypothalamus detect changes in temperature and initiate
physiological and behavioural changes in response to these. Stimulation of the extreme anterior
and posterior areas of the hypothalamus increases sexual drive. Appetite is regulated by the
feeding and satiety centres of the hypothalamus. Stimulation of the ventromedial hypothalamus
leads to placidity, whereas stimulation of the lateral hypothalamus leads to rage and restlessness.
Sleep is controlled via the suprachiasmatic nuclei in the hypothalamus.

Question 90: FFFTT


The bispectral index is a statistically based, empirically derived complex parameter. It is a
weighted sum of several EEG parameters and provides the user with a dimensionless
number which ranges from 0 (equivalent to EEG silence) to 100 (normal cortical electrical
activity). It records the cortical activity of a patient’s brain and, using Fourier analysis, gives
a raw figure, higher values representing higher levels of cortical activity (Table 1.90.1):

Table 1.90.1 Bispectral index values

85–100 Awake, capable of specific recall


60–85 Increased sedation, impaired memory but rousable
40–60 Surgical plane of anaesthesia. Auditory processing
and reflex movement are possible, but memory
unlikely
0–40 Burst suppression

Many published BIS studies have controlled for surgical stimulation by excluding it, so
the applicability to patients undergoing surgery remains uncertain, and BIS values may not
accurately represent surgical anaesthesia. BIS is best described as a monitor of the depth of
the hypnotic component of anaesthesia or sedation. BIS values also display interpatient
variability, therefore an absolute number cannot be relied upon in isolation to guarantee
anaesthesia.

Downloaded from https://www.cambridge.org/core. University of Edinburgh, on 19 Aug 2019 at 13:21:27, subject to the Cambridge Core terms of
use, available at https://www.cambridge.org/core/terms. https://doi.org/10.1017/9781108566100.004
Chapter 1b: Physiology Answers 75

There are broad guidelines to aid the interpretation of BIS values; in general, the chance
of postoperative recall is highly unlikely if the BIS value is kept below 60. However,
anaesthetists will tend to overanaesthetize patients with a BIS value of 30–40 in order to
prevent awareness. No BIS value predicts an individual’s threshold for loss or recovery of
consciousness. BIS values only give an idea of cortical activity – they cannot distinguish
whether a low value is due to anaesthesia or pathological causes of reduced consciousness.
In addition, it is worth bearing in mind that, in the absence of surgical stimulation, the use of
opioids produces clinical changes in depth of sedation or anaesthesia that are not reflected
by a decrease in BIS. This is significant drawback in using BIS to assess depth of balanced
anaesthesia. However, when using opioids during surgery, BIS does appear to decrease,
perhaps showing how pain can counteract arousal.

Question 91: TFTFF


The nicotinic acetylcholine receptor is a transmembranous ligand-gated ion channel found
in the postjunctional membrane of a motor end plate. It consists of five subunits (two α, one
β, one ε and one δ). Much of the early work into these receptors was done in the electric eel,
where a γ subunit exists rather than a ε subunit. Acetylcholine binds to the α subunits and
induces a conformational change in the subunits, opening the ion channel and allowing flow
of cations through the channel according to their concentration gradients, thus allowing
propagation of action potentials. Between 24 hours and 2 years after denervation or major
burns injuries, acetylcholine receptors appear at extra-junctional locations. It is for this
reason that depolarizing muscle relaxants should be avoided in these patients.

Question 92: FTTTF


I band A band I band
H Zone

Myosin
Actin

Z line M Line Z line

Figure 1.92.1 Sarcomere.

Downloaded from https://www.cambridge.org/core. University of Edinburgh, on 19 Aug 2019 at 13:21:27, subject to the Cambridge Core terms of
use, available at https://www.cambridge.org/core/terms. https://doi.org/10.1017/9781108566100.004
76 Chapter 1b: Physiology Answers

Muscle cells are composed of tubular myofibrils, which in turn are composed of repeating
sections of sarcomeres. A sarcomere is the basic unit of cross-striated myofibril in a muscle.
It is composed of thin actin filaments and thick myosin filaments (Figure 1.92.1). Myosin
has a long, fibrous tail and a globular head, which binds to actin. Myosin filaments are
crosslinked at the centre by the M line. The myosin head also binds to ATP, which is the
source of energy for muscle movement. Myosin may only bind to actin when the actin
binding sites are exposed by calcium ions. Actin molecules are bound to the Z line, which
forms the borders of the sarcomere. Other bands appear when the sarcomere is relaxed.
Tropomyosin is a dimer that coils around the core of the thin filaments and plays an
important role in regulating muscle contraction.

Question 93: FFFFT


The basic contractile mechanism of all types of muscle is based on the interaction between
actin and myosin, but their physiological function varies. This is mirrored in their micro-
scopic make-up. Skeletal and smooth muscle cells have a motor end plate to propagate
action potentials. However, cardiac muscle cells (and some smooth muscle cells) act as a
syncytium and, therefore, motor end plates are not a feature in these cells. Cardiac muscle
cells have the greatest number of mitochondria and have a rich capillary supply, as they
cannot afford to incur an oxygen debt through anaerobic metabolism.
Calcium binds to troponin in skeletal muscle cells to facilitate contraction. In smooth
muscle cells, it binds to calmodulin. Calcium is an important feature in the contraction of all
muscles.

Question 94: FTFFT


Control of muscle movements is essential for tone, posture and for the accuracy of move-
ments. Stretch receptors in the fibres of skeletal muscle are known as muscle spindles. They
provide a feedback loop, receiving input from sensory (Ia and IIa) afferent nerves and
eliciting motor responses via γ efferent nerves. When the position of a muscle is changed by
stretching, the muscle spindles stretch and increase the feedback signals to the spinal cord.
This increases the motor output and skeletal muscle tone opposes the original stretch. There
are static signals to control posture, and dynamic components to control the rate of muscle
contraction and therefore the smoothness of the movement.
Golgi tendon organs are located in the muscle tendon adjacent to the muscle fibres. They
respond to changes in muscle tension, creating a positive feedback loop to reduce muscle
tone and stimulate contraction in the antagonist muscles of opposing groups. They cause a
virtual relaxation of the muscle and protect against excessive tension and therefore rupture
of the muscle.

Question 95: FFTFT


A reflex is a neuronal pathway that produces a predictable, repetitive, automatic response to
a sensory stimulus. They may be monosynaptic, such as the knee jerk reflex, or polysynaptic,
such as the withdrawal reflex. The sensory fibres travel in the posterior nerve roots, synapse
in the spinal cord (in the ventral horn for monosynaptic reflexes) and the motor response
fibres travel in the anterior spinal nerves. This is known as the Bell–Magendie law.

Downloaded from https://www.cambridge.org/core. University of Edinburgh, on 19 Aug 2019 at 13:21:27, subject to the Cambridge Core terms of
use, available at https://www.cambridge.org/core/terms. https://doi.org/10.1017/9781108566100.004
Chapter 1b: Physiology Answers 77

Fast pain signals travel via type Aδ fibres to terminate in the dorsal horn, where they
synapse with dendrites of the neospinothalamic tract. Fast pain can be felt within a tenth of a
second, whereby a withdrawal response results. Slow pain is transmitted via slower type C
fibres to laminae II and III of the dorsal horn (substantia gelatinosa).

Question 96: TTFTT


During pregnancy, there is a significant increase in cardiac output; this is thought to be
between 30–50% by the end of the second trimester. Distribution of cardiac output during
pregnancy is different from the non-gravid state.
There is increased blood flow to the uterus, kidneys and skin. By the end of pregnancy, the
uterus receives up to 20% of the total cardiac output simply to cope with the increasing
demands of the growing fetus. Like the heart, the kidneys work harder during pregnancy to
filter the increasing blood volume, which reaches a maximum at 16–24 weeks’ gestation.
Skin changes are common in pregnancy; the increase in blood flow may make pregnant
women feel warmer and may increase sweating (‘pregnancy glow?’). In addition, this may
contribute to nose bleeds and bleeding gums.
Absolute hepatic blood flow remains unchanged, but the percentage of cardiac output to
the liver decreases.
Cerebral blood flow increases during pregnancy and this has been demonstrated using
internal carotid artery (ICA) Doppler studies. Cerebral blood flow changes were associated
with progressive decreases in cerebral vascular resistance and a moderate increase in ICA
diameter. This may be as a result of the vasodilating properties of oestrogen and other factors
on cerebral vessels. The cerebral autoregulation curve shifts to the right in pregnancy.

Question 97: TFTFT


Oxygen, carbon dioxide and fatty acids are transferred down their concentration gradient
by simple diffusion across the lipid bilayer. Fick’s law determines the transplacental move-
ment of these molecules.
The human fetus is almost totally dependent on maternal glucose passing through the
placenta, as its own production is minimal. Overall glucose flux follows a maternal-to-fetal
concentration gradient. Glucose crosses the placenta by facilitated diffusion. This transport
system has a high capacity and three transporter isoforms have been identified: GLUT 1, 3
and 4. It is thought that transplacental glucose transport is limited by placental blood flow,
rather than being diffusion limited.
Amino acids cross the placenta by secondary active transport. A ubiquitous active
transport protein is the sodium:potassium pump. By moving sodium out and potassium
into the cell, an electrochemical gradient is formed across the membrane. The cell may then
use the energy ‘stored’ in this electrochemical gradient to secondarily pump molecules
against a concentration gradient. Cells concentrate amino acids by linking the movement of
sodium (from high to low concentrations) to transport of an amino acid (from low to high).

Question 98: TTTTF


Magnesium sulfate is the agent most commonly used for treatment of eclampsia and
prophylaxis of eclampsia in patients with severe pre-eclampsia. Magnesium sulfate is a

Downloaded from https://www.cambridge.org/core. University of Edinburgh, on 19 Aug 2019 at 13:21:27, subject to the Cambridge Core terms of
use, available at https://www.cambridge.org/core/terms. https://doi.org/10.1017/9781108566100.004
78 Chapter 1b: Physiology Answers

well-known membrane stabilizer and vasodilator. It may also act as a central anticonvulsant
via its inhibitory action on the NMDA receptor.
Eclamptic seizures are rare and occur in less than 2% of mothers with pre-eclampsia. It is
important to realize that despite the fact that delivery of the baby is the only definitive treatment
for pre-eclampsia, delivery does not remove the risk of seizures; seizures can occur in the
immediate postpartum period in up to 45% of cases. They are usually self-limiting.
The therapeutic range of magnesium is 2–4 mmol.l−1, but the actual magnesium dose and
concentration needed for prophylaxis have never been estimated. Maternal toxicity is rare
when magnesium sulfate is carefully administered and monitored.

Question 99: FTTFT


Pethidine is a synthetic opioid, once indicated for the treatment of moderate to severe pain.
It was once widely used as an analgesic in labour and delivery, but is now used much less
often due to its numerous disadvantages over other opioids and potential drug interactions
(especially with serotonergics). It is a weak base with anticholinergic and local anaesthetic
properties (related to its interactions with sodium ion channels). It exerts its analgesic effect
by its agonist action on the OP3 (μ) opioid receptors.
Pethidine is metabolized to norpethidine and pethidinic acid. Norpethidine is an active
metabolite with sedative and proconvulsant actions. Other severe side effects of norpethi-
dine include serotonin syndrome, delirium, tremor and dysphoria. The neurotoxicity of
pethidine metabolites is a unique feature compared to other opioids. Pethidine has no
action on the myometrium.

Question 100: TFTTF


The increase in stroke volume is predominantly due to increase in the circulating blood
volume. The peripheral vascular resistance drops by up to 50% in early pregnancy due to
increased levels of circulating oestrogen and progesterone. The combination of low periph-
eral vascular resistance and increased circulatory volume results in an increased resting
heart rate of about 90 bpm.
During labour, the contracting uterus contributes to the increase (about 20%) in cardiac
output (autotransfusion). There is a transient increase in cardiac output just after delivery
owing to placental autotransfusion and release of aortocaval compression.
The 50% increase in plasma volume combined with an increase in the red cell volume
(30%) results in a physiological anaemia of pregnancy and low viscosity, thus offsetting the
prothrombotic effects of pregnancy. The decrease in colloid osmotic pressure is mainly due
to haemodilution rather than altered protein production.
The blood volume returns to normal in 10–14 days.

Question 101: TTFFF


Inhaled anaesthetics, including nitrous oxide, are lipid soluble and readily cross the placenta
rapidly.
Suxamethonium and other muscle relaxants are highly ionized and therefore do not pass
across the placenta easily. Etomidate is less lipophilic and crosses the placenta slowly.

Downloaded from https://www.cambridge.org/core. University of Edinburgh, on 19 Aug 2019 at 13:21:27, subject to the Cambridge Core terms of
use, available at https://www.cambridge.org/core/terms. https://doi.org/10.1017/9781108566100.004
Chapter 1b: Physiology Answers 79

Neostigmine, a quartenary ammonium compound, does not cross the placenta easily
owing to its polar nature.

Question 102: TFTTF


As we know, the cardiac output increases by up to 30–50% by the third trimester. The increase in
blood volume results in an increased stroke volume by up to 35%. The increase in circulating
oestrogen and progesterone causes vasodilatation and a fall in peripheral vascular resistance. This
leads to an increased heart rate by 15–25%, which commonly gives rise to sinus tachycardia seen
on ECG. Premature atrial and ventricular complexes are also common, unlike atrial fibrillation.
This is rare in pregnancy, unless the woman has pre-existing cardiac disease.
Left ventricular hypertrophy (LVH), along with vasodilatation, facilitates the increase in
cardiac output. The LVH combined with the upward shift of the diaphragm causes a
rotation of the cardiac axis to the left along with ST segment depression and T wave
flattening or inversion, particularly in lead III.

Question 103: TTFFF


Changes in the respiratory system are of great significance to the anaesthetist and may be
considered as anatomical or physiological. The physiological changes are mediated pre-
dominantly by the effects of progesterone. The increase in minute ventilation during
pregnancy is achieved mainly by increasing the tidal volume by up to 45%. Respiratory
rate is essentially unaltered during pregnancy. This change is due to progesterone-mediated
hypersensitivity to CO2. Since dead space remains unchanged, alveolar ventilation is higher
and, consequently, there is a fall in paCO2. The resultant respiratory alkalosis is offset by a
corresponding reduction in serum bicarbonate.
The inspiratory reserve volume is increased, but vital capacity and FEV1 remain
unchanged. From the middle of the second trimester, expiratory reserve volume and
residual volume are progressively decreased. Function residual capacity begins to fall by
the fifth month of pregnancy and is decreased by 20% at term. It should be remembered that
as a result, preoxygenation is less effective due to increased oxygen consumption and the
decreased FRC, leading to a rapid fall in arterial oxygen tension.
Lung compliance is relatively unaffected, but chest wall compliance is reduced, especially
in the lithotomy position.

Question 104: FTTTT


Physiological changes of normal pregnancy with regards to the hepatobiliary system can be
confused with the signs and symptoms of liver disease.
Blood flow to the liver remains constant and the liver usually remains impalpable during
pregnancy. If anything, the liver may be pushed upward and backward as the gravid uterus
enlarges. Plasma concentration of ALP is increased 2–4-fold due to placental secretion.
Plasma cholinesterase levels fall up to 25% at term. Biliary stasis and decreased contractile
response of the gall bladder (reduced gall bladder emptying) may lead to an increased
incidence of gall stones. This is mediated by the effect of progesterone. Spider naevi
(telangiectasia) and palmar erythema are common in pregnant women and occur as a result
of raised oestrogen levels. Their presence does not indicate hepatic failure.

Downloaded from https://www.cambridge.org/core. University of Edinburgh, on 19 Aug 2019 at 13:21:27, subject to the Cambridge Core terms of
use, available at https://www.cambridge.org/core/terms. https://doi.org/10.1017/9781108566100.004
80 Chapter 1b: Physiology Answers

Question 105: TFTFT


Pre-eclampsia is a major cause of maternal and fetal mortality and morbidity. The incidence
of pre-eclampsia is somewhere between 2% and 10%. One of the most common risk factors
for pre-eclampsia is nulliparity and it is thought to almost triple a woman’s risk. Multiparity
on its own is not thought to be a risk factor. Women with history of pre-eclampsia in a
previous pregnancy are at increased risk (up to seven times) of pre-eclampsia in subsequent
pregnancies.
Advanced maternal age (>40), obesity, diabetes, pre-existing hypertension, chronic
autoimmune disease, family history of pre-eclampsia, abnormal placenta and multiple
pregnancies are some of the other risk factors. In particular, the presence of antipho-
spholipid antibodies significantly increases the risk.
Cigarette smoking during pregnancy has been associated with a decreased risk of pre-
eclampsia.

Question 106: TTTFT


The placenta is a functional endocrine organ: it secretes a number of hormones from the
syncytial layer of chorionic villi. Apart from oestrogen and progesterone, the placenta
secretes human chorionic gonadotropin (hCG) and human placental lactogen (hPL). hCG
is vital for progression of the pregnancy; it ensures the corpus luteum secretes enough
oestrogen and progesterone until the placenta is able to produce them in sufficient volume
itself; it also suppresses the maternal immunological response so that the placenta is not
rejected. hPL is secreted as the placenta increases in size. Raised levels of hPL promote
maternal mammary growth and also regulate maternal glucose, protein and fat availability
to the fetus.
Relaxin is a protein hormone secreted by the corpus luteum (in non-pregnant women)
and the placenta. Relaxin facilitates the softening of ligaments, including the pubic sym-
physis (to aid passage of the fetus through the birth canal) and intercostal ligaments
(increasing the volume of the thoracic cage). Relaxin may have a role in the reduction of
systemic vascular resistance in normal pregnancy by its action on the arterial smooth
muscles.
Thyroxine is not secreted by the placenta.

Question 107: TFTTT


Interestingly, both kidneys increase in size up to 1–1.5 cm during pregnancy; kidney volume
increases by up to 30%, mainly due to an increase in renal vascular and interstitial volume.
There is a 50% increase in renal plasma flow and GFR. This manifests as an increase in
creatinine clearance. As a result, creatinine levels drop to around 60–70 μmol.l−1 in
pregnancy. Creatinine levels above 85 μmol.l−1 are usually the result of renal insufficiency.
The increase in GFR is not matched by tubular reabsorption rates; as a result, mild
glycosuria and proteinuria are relatively common.
Plasma osmolality falls as a result of water retention due to the enhanced activity of
renin–angiotensin–aldosterone pathways.

Downloaded from https://www.cambridge.org/core. University of Edinburgh, on 19 Aug 2019 at 13:21:27, subject to the Cambridge Core terms of
use, available at https://www.cambridge.org/core/terms. https://doi.org/10.1017/9781108566100.004
Chapter 1b: Physiology Answers 81

Question 108: TTTTT


Apnoea of prematurity (AOP) is a common problem affecting preterm infants, likely
secondary to a ‘physiological’ immaturity of respiratory control that may be exacerbated
by neonatal disease. These include altered ventilator responses to hypoxia, hypercapnia and
altered sleep states.
The most widely used definition of AOP specifies a pause in breathing for more than 15–
20, or accompanied by desaturation (SpO2 ≤80% for ≥4 s) and bradycardia (heart rate <2/3
baseline for ≥4 s), in infants born at less than 37 weeks’ gestation. The incidence is inversely
correlated with gestational age and birth weight: nearly all infants born at <29 weeks’
gestation and/or weighing <1000 g exhibit AOP. It usually self-resolves.
While AOP is a developmental disorder, the reasons behind the propensity to apnoea are
not entirely clear. It is thought that AOP may be attributed to an immature respiratory
centre, decreased chemoreceptor sensitivity and parasympathetic-dominant autonomic
nervous system, in addition to immature pulmonary reflexes. Also of interest is the role
of thermoregulation in apnoea. Exposure to cooler temperatures has been shown to
decrease the duration and frequency of AOP. Hypoglycaemia and other electrolyte imbal-
ances are associated with AOP.

Question 109: TTTTF


The newborn term infant is at risk of hypothermia – the main process to counteract this is
non-shivering thermogenesis, a process that occurs primarily in the liver, brain and brown
fat. Cold stress triggers an increase in levels of epinephrine and TSH. Working in parallel,
they seek to increase circulating levels of T3, which acts locally in brown fat to uncouple
mitochondrial oxidation from phosphorylation, thereby allowing energy to be dissipated as
heat.
Brown fat is found in the interscapular region, mediastinum, axilla, around the vessels of
the neck and in perinephric fat. It makes up about 6% of total body weight of a term infant.
It is highly vascular with sympathetic innervation and increased mitochondrial content to
facilitate heat generation. Norepinephrine released from the sympathetic neurones stimu-
lates release of lipases, which facilitate lipid breakdown and heat generation.
This mechanism may double the heat production, but there is a marked increase in
oxygen demand. Preterm babies have a reduced amount of brown fat and are therefore more
at risk of hypothermia.

Question 110: TTTTT


The nervous system accounts for 10% of total body weight at birth. The brain increases in
size threefold during the first year of life, reflected in the high metabolic demand of brain
tissue during this time. In order for this rapid increase in size to occur, the neonatal brain
receives one-third of the cardiac output, compared with one-sixth of cardiac output in
adults.
Blood–brain barrier maturity is not attained until six months of age. Cerebral autoregu-
lation is fully developed at term.

Downloaded from https://www.cambridge.org/core. University of Edinburgh, on 19 Aug 2019 at 13:21:27, subject to the Cambridge Core terms of
use, available at https://www.cambridge.org/core/terms. https://doi.org/10.1017/9781108566100.004
82 Chapter 1b: Physiology Answers

Question 111: TTTFF


Type I and Type II pneumocyte differentiation occurs at approximately 20–22 weeks’
gestation and from 24 weeks’ gestation, Type II pneumocytes begin to produce surfactant.
Surfactant is an agent that decreases surface tension at the interface of gaseous–aqueous
components within the lungs. It is essential for efficient exchange of gases and for main-
taining the structural integrity of alveoli. Surfactant is a secretory product, composed of
lipids and proteins. Its functions are to increase pulmonary compliance, to prevent atelec-
tasis at the end of expiration and to facilitate recruitment of collapsed airways. It also plays a
role in innate immunity; surfactant degradation/inactivation may contribute to enhanced
susceptibility to lung inflammation and infection.
Alveolar distension, cortisol and epinephrine stimulate surfactant production in the
pneumocytes. Maternal dexamethasone increases surfactant production if given after 24
weeks, if preterm delivery is anticipated. This is to try and prevent or lessen morbidity
associated with infant respiratory distress syndrome as a result of prematurity.
Type I pneumocytes make up 95% of the alveolar epithelium while the remaining 5%
accounts for the Type II pneumocytes.

Question 112: FFFTF


HbF has two α and two γ globin chains (HbA is comprised of two α and two β chains). The
presence of γ chains provides a greater affinity for oxygen binding and maintains the
structure and integrity of the haemoglobin molecule in a highly acidic environment.
The oxyhaemoglobin dissociation curve is shifted to the left, enabling more oxygen
transfer across the placenta from the maternal circulation (double Bohr effect). The P50
of HbF is 2.5 kPa (19 mmHg).
In a newborn baby, the HbF accounts for up to 80% of the total haemoglobin. By six
months of age, HbF is fully replaced by adult haemoglobin.

Question 113: FTFTT


There is a single umbilical vein and two umbilical arteries (arising from the left and right
iliac arteries).
The oxygenated blood from the umbilical vein passes through the portal vein and ductus
venosus before reaching the inferior vena cava.
In the adult circulation, the stroke volume of the right ventricle (RV) should equal that of the
left (LV), assuming that there are no shunts – the circulatory system works in series. Here,
cardiac output can be defined as the volume of blood ejected by one ventricle in 1 min. This
does not occur in the fetus. Here, as a result of intra- and extracardiac shunting, the LV stroke
volume does not equal that of the RV. The RV receives approximately 65% of the venous return
and the LV about 35%. Thus, the cardiac output must be considered in different terms – it can
only be thought of in terms of the combined ventricular output (CVO). About 45% of the CVO
is directed to the placental circulation, with only 8% entering the pulmonary circulation.
At the junction of the IVC and the right atrium (RA) is a tissue flap known as the Eustachian
valve. This flap tends to direct the more highly oxygenated blood across the foramen ovale and
into the left atrium. Here, the oxygen saturation is 65%. The blood is then ejected into the
ascending aorta. The majority of the LV blood is delivered to the brain and coronary circulation.

Downloaded from https://www.cambridge.org/core. University of Edinburgh, on 19 Aug 2019 at 13:21:27, subject to the Cambridge Core terms of
use, available at https://www.cambridge.org/core/terms. https://doi.org/10.1017/9781108566100.004
Chapter 1b: Physiology Answers 83

Because of the high pulmonary vascular resistance, only about 12% of the RV output enters the
pulmonary circulation, with the remaining 88% crossing the ductus arteriosus into the descend-
ing aorta. The lower half of the body is therefore supplied with relatively deoxygenated blood.

Question 114: FFTTT


The surface area of the lung is indeed many times greater than the skin, but by a factor of about
30, not 300. The respiratory exchange ratio actually increases within the normal limits of
exercise, from 0.8 to 1.0. VO2 max is the maximum rate of utilization of oxygen during exercise.
If an increase in work is to be achieved above the level of the VO2 max, then this will be via
processes that do not utilize oxygen, i.e. anaerobic glycolysis. During moderate exercise, there is
usually very little change in pO2, pCO2 and pH, but during high levels of exercise, lactic acid
production causes a relative compensatory hyperventilation resulting in hypocarbia.

Question 115: TTFTF


A lower haematocrit may improve rheology in cardiac patients, but it is polycythaemia that
the climber needs. Therefore, there is a physiological rise in the haemoglobin concentration.
Polycythaemia and hypoxic pulmonary vasoconstriction (HPV) cause a degree of pulmon-
ary hypertension and subsequent right ventricular hypertrophy. This is a common finding
in high-altitude inhabitants. There seems to be no beneficial effect of HPV, however.

Question 116: TFTTT


There are many interesting physiological changes that occur during diving, not limited to
physiology of the respiratory system. Applying Henry’s law, the mass of gas dissolved in
body fluids increases with depth. Dalton’s law states that an increase in total gas pressure is
associated with an increase in the partial pressures of all the constituent gases in that
mixture. The proportion of gases stays the same.
Atmospheric pressure is approximately 1 bar, but increases by 1 bar for every 10 m a diver
descends. Therefore, at 10 m depth, the absolute pressure would actually be 2 bar.
Interestingly, when a diver initially submerses in water a pressure gradient develops down
the length of the body due to the pressure gradient discussed above. This opposes the
hydrostatic pressures in the lower, dependent parts of the body. Additionally, the cold
environment causes significant vasoconstriction of the cutaneous vessels. These actions
decrease the degree of venous pooling in the dependent areas of the body, with a resultant
increase in venous return of approximately 500 ml. This increases the stroke volume.
During a ‘breath-hold’ dive, the mass of air within the lungs remains constant. However, as
the diver descends, the ambient pressure increases; Boyle’s law dictates that the volume of air
decreases. This compression of gases causes the volume to decrease until it may drop below
the residual volume, producing a negative intra-alveolar pressure, resulting in pulmonary
oedema or even haemorrhage. At depths of 200 m and more, lipids are compressed. They are
more compressible than water. This leads to a syndrome of tremor, dizziness and confusion.

Question 117: TTTFF


Helium is used for many reasons. Firstly, as depth increases, the turbulence of gas within the
airways increases with density. Helium, being a less dense gas, decreases turbulence. Secondly,

Downloaded from https://www.cambridge.org/core. University of Edinburgh, on 19 Aug 2019 at 13:21:27, subject to the Cambridge Core terms of
use, available at https://www.cambridge.org/core/terms. https://doi.org/10.1017/9781108566100.004
84 Chapter 1b: Physiology Answers

helium is 40 times less soluble than nitrogen, leading to less absorption within the tissues, and
subsequently fewer ‘bubbles’ within the tissues on ascent. Symptoms of oxygen toxicity
include vertigo, paraesthesia and muscle twitching, and can occur at a depth of 8 m when
breathing 100% O2. The symptoms of decompression sickness typically occur within hours of
ascent. Symptoms are not usually experienced during the ascent. Decompression sickness
occurs as dissolved nitrogen forms bubbles in the tissues as the diver ascends.

Question 118: FTFTT


Relative humidity decreases with altitude. As a result, insensible water loss through respira-
tion and skin evaporation increases. The oxygen concentration remains constant at 21%,
and is independent of altitude. However, as atmospheric pressure decreases with altitude,
Dalton tells us that the partial pressure of the constituent gases will decrease proportionally.
The saturated vapour pressure at body temperature remains constant at 47 mmHg. The
atmospheric pressure at 19000 m (the height of Everest) is approximately 47 mmHg.
Therefore, alveolar gases would almost entirely comprise water vapour.

Question 119: FFFFF


The thermoneutral zone refers to the range of environmental temperatures over which the
body does not rely on metabolism to maintain body temperature. Instead, core body tem-
perature is controlled by vasomotor activity. This is typically 27–31 °C. The preoptic region
and anterior hypothalamus act to increase heat loss, mainly through peripheral vasodilatation.
The posterior hypothalamus is concerned with heat production and conservation. Thyroid
hormones, not glucocorticoids, act upon mitochondrial receptors within tissues to increase
Na+/K+ ATPase heat production. The shivering response increases heat production fivefold in
adults, but is poorly developed in neonates. Below a core temperature of 35 °C, muscle
weakness occurs, and there follows decreased mobility and shivering, which cease at 32 °C.

Question 120: FFFTF


For all healthy individuals, the simple laws of physics of fluid in a column suggest that the
dependent parts of the body will have a higher pressure exerted upon them. This makes
various assumptions, one of which is the lack of valves within the column. Whilst excite-
ment is likely to increase circulating catecholamines during take-off, pooling of blood
occurs in the dependent parts of the body with a subsequent decrease in venous return.
This is turn leads to a decreased cardiac output. At the same time, relative pooling of blood
occurs in the dependent areas of the lung, but the apices are fully dilated and distended. This
increases V/Q mismatch, resulting in hypoxaemia, but hypercarbia.
Weightlessness may be associated with as much as 2000 ml of fluid redistributed from the
lower limbs, increasing venous return to the heart, and subsequently cardiac output.
However, due to reduced sympathetic activity and increased parasympathetic tone, a degree
of ventricular atrophy occurs, not hypertrophy.

Question 121: FFTTF


Carbohydrate metabolism generates glucose which can be polymerized into glycogen for
storage or catabolized via glycolysis.

Downloaded from https://www.cambridge.org/core. University of Edinburgh, on 19 Aug 2019 at 13:21:27, subject to the Cambridge Core terms of
use, available at https://www.cambridge.org/core/terms. https://doi.org/10.1017/9781108566100.004
H2O
Electron transport chain

Electron transport
ATP
O2

Krebs cycle (aka Citric acid cycle)


FADH2
NADH

Krebs
cycle

or GTP
ATP
Acetyl
CoA

Pyruvate processing
NADH

CO2
2 for every glucose
Pyruvate
NADH

ATP

Glycolysis

Figure 1.121.1 Cellular respiration.


Glucose

Downloaded from https://www.cambridge.org/core. University of Edinburgh, on 19 Aug 2019 at 13:21:27, subject to the Cambridge Core terms of
use, available at https://www.cambridge.org/core/terms. https://doi.org/10.1017/9781108566100.004
86 Chapter 1b: Physiology Answers

When one molecule of glucose undergoes glycolysis to form two molecules of pyruvate,
four ATP and two NADH are generated. However, two ATP are used in the process
(including one to convert glucose to glucose-6-phosphate), and hence there is a net gain
of two ATP. Using glycogen yields a net three ATP as it doesn’t use ATP to form glucose-6-
phosphate.
The citric acid cycle (or Krebs cycle) is an important aerobic pathway and is a common
end point for energy production from breakdown products of carbohydrate, fat and protein.
Carbohydrate enters the cycle as pyruvate, is converted to acetyl-CoA (two carbon atoms),
which combines with oxaloaceteate (four carbon atoms) to form citric acid (six carbon
atoms). The cycle generates ATP and NADH, which is fed into the oxidative phosphoryla-
tion system to produce further ATP (Figure 1.121.1). Under aerobic conditions, one
molecule of glucose metabolized via glycolysis, the citric acid cycle and oxidative phosphor-
ylation yields 38 ATP. Catabolism of one molecule of fatty acid via the citric acid cycle yields
44 molecules of ATP.
Under anaerobic conditions, where there is no oxygen to drive the citric acid cycle or
oxidative phosphorylation, there is a build-up of pyruvate and NADH (which is not
reduced). In such circumstances, pyruvate is reduced to lactate by accepting a proton,
allowing the reformation of NAD+. In this way metabolism and energy production can
continue without oxygen, albeit in a less efficient manner, with net ATP production being
limited to two molecules of ATP from one molecule of glucose. However, the accumulated
lactate can then be converted back into pyruvate and then glucose within the liver. This is
known as the Cori cycle, and requires expenditure of six molecules of ATP to convert two
molecules of pyruvate to one molecule of glucose.

Question 122: TFTFT


During starvation, the body must adapt to utilize stored energy. Initially, this is via
glycogenolysis, but glycogen stores within the liver and skeletal muscle are rapidly depleted
(within 24 hours), and an alternative source of glucose is required to prevent hypoglycae-
mia. Gluconeogenesis describes processes where glucose is synthesized from non-carbohy-
drate sources, including lactate and amino acids, e.g. alanine from muscles (via pyruvate)
and glycerol released from fats. Gluconeogenesis is stimulated by various hormones,
including cortisol, growth hormone and epinephrine, and is inhibited by rising insulin
levels.
Increasing cortisol and epinephrine levels encourage lipolysis, releasing large amounts of
free fatty acids and glycerol, which not only supply glucose via gluconeogenesis but also
contribute to ketone body formation. Other features of starvation, including falling insulin
and triiodothyronine levels, also stimulate lipolysis.
During lipolysis fatty acids undergo β oxidation to acetyl-CoA. Large volumes of acetyl-
CoA can overwhelm the citric acid cycle, and acetyl-CoA molecules are diverted to form
ketone bodies: acetone, acetoacetic acid and β-hydroxybutyric acid. The liver is the only
organ that is able to produce ketone bodies, but isn’t able to use them for energy; however,
most other organs, including the brain and kidneys, can. In established starvation, ketone
bodies supply up to 50% of the brain’s energy. After 3–5 days gluconeogenesis declines as
the body adjusts to ketone bodies as the predominant energy supply. By three weeks the rate
of protein breakdown is a third of that originally seen.

Downloaded from https://www.cambridge.org/core. University of Edinburgh, on 19 Aug 2019 at 13:21:27, subject to the Cambridge Core terms of
use, available at https://www.cambridge.org/core/terms. https://doi.org/10.1017/9781108566100.004
Chapter 1b: Physiology Answers 87

Question 123: FTTFT


70% of hepatic blood flow is delivered via the portal vein. This system conveys blood from
the gut to the liver. This blood contains dietary breakdown products from the gut but is
poorly oxygenated, with saturations of 65–70%. It accounts for approximately 50% of
hepatic oxygen supply. The remaining 30% of blood flow is via the hepatic artery, which
is well oxygenated with saturations of 98%, accounting for the other 50% of hepatic oxygen
supply.
Anatomically, the liver is divided into four lobes and further subdivided into lobules.
Each lobule is hexagonal and approximately 1 mm across. At each point of the hexagon,
there is a portal triad containing a hepatic arteriole (originating from the hepatic artery), a
portal venule (originating from the portal vein) and a bile ductule (which drains into the bile
duct). Within the hexagon are sheets of hepatocytes, which have numerous roles, including
bile production. In the centre is a central vein that drains ultimately into the hepatic vein.
Blood drains slowly from the two blood vessels of the portal triad, across the sheet of
hepatocytes, via sinusoids, and into the central vein; this takes approximately 8.4 seconds.
Blood supplies oxygen to the metabolically active hepatocytes, and food and drugs taken
from the gut are subjected to breakdown processes. Simultaneously, the hepatocytes
synthesize bile, which drains in the opposite direction towards the portal triad, via bile
canaliculi and into the ductules. Lining the sinusoids are Kupffer cells, which are hepatic
macrophages and form part of the reticuloendothelial system. They play an important role
in phagocytosis and denaturing proteins.
Functionally, the liver is divided into units is called acini. Each acinus is diamond-shaped,
with a central vein at the top and bottom points, and a portal triad at the left and right
points.
Within the acinus, hepatocytes are divided into three zones according to their position:
periportal (zone I), intermediate (zone II) and perivenous (zone III). Nutrient and oxygen
content decrease as blood travels from zone I to zone III hepatocytes. This is reflected in
zone I being the most metabolically active, mediating both anabolism and catabolism of
proteins. Interestingly, zone III, which has the least oxygen supply, contains the cytochrome
P450 system responsible for drug biotransformation.

Question 124: TTTFF


Bile acids are formed in the liver from cholesterol. They are then conjugated with amino
acids to form bile salts in order to make them more water soluble. Bile salts are the principle
organic component of bile, which is stored in the gall bladder before being released into the
small intestine. A major role of bile salts is to emulsify dietary fat to enhance its absorption
from the GI tract. They are more able to fulfil this role when rendered water soluble.
Over 90% of excreted bile salts are taken up from the ileum via active transport and
returned to the liver via the portal system (enterohepatic circulation). A small proportion
pass into the colon where they are converted to secondary bile salts by intestinal flora and
reabsorbed; the remainder are excreted in the faeces. Enterohepatic circulation ensures that
the total pool of circulating bile acids remains constant (between 2.5 g and 4.0 g), with a low
rate of synthesis required. The whole pool recirculates around the body between six and
eight times per day.

Downloaded from https://www.cambridge.org/core. University of Edinburgh, on 19 Aug 2019 at 13:21:27, subject to the Cambridge Core terms of
use, available at https://www.cambridge.org/core/terms. https://doi.org/10.1017/9781108566100.004
88 Chapter 1b: Physiology Answers

Other functions of bile include absorption of the fat-soluble vitamins A, D, E and K. It


provides an excretory route for bile pigments, cholesterol, steroids and some drugs, e.g.
morphine. Bile salts are amphipathic, which means they have both hydrophilic and hydro-
phobic areas. This allows them to form micelles with hydrophobic cores and hydrophilic
exteriors. This way, lipids can be transported in solution and delivered to the small intestine
for absorption.

Question 125: FTFTF


Bilirubin is a product of haemoglobin breakdown, which occurs in the spleen. Haem is
cleaved into iron (recycled to form new haemoglobin) and a porphyrin ring which is
converted to biliverdin and then reduced to unconjugated bilirubin. This is then bound to
albumin and transported in the plasma to the liver. Here, bilirubin is conjugated to make it
water soluble and therefore able to be taken up into bile.
Bile is released into the small intestine where it has a role in the metabolism of dietary
lipids. Bile acids are reabsorbed in the ileum and returned to the liver via the enterohepatic
circulation, but the conjugated bilirubin is not reabsorbed and passes into the colon. Here,
bacteria convert it to urobilinogen. Urobilinogen is converted to urobilin and stercobilin.
Urobilin is taken up into the systemic circulation and delivered to the kidneys, where it gives
a yellow colour to urine. Stercobilin remains within the colon and gives a brown colour to
faeces.
Jaundice can be classified as prehepatic, intrahepatic or posthepatic. Prehepatic jaundice
arises from haemolysis and the excessive breakdown of haemoglobin. Causes include sickle
cell anaemia, thalassaemia and malaria. Large amounts of unconjugated bilirubin accumu-
late in the plasma and tissues, to give rise to the characteristic yellow appearance.
Posthepatic jaundice is also called obstructive jaundice, and results from the obstruction
of bile drainage from the liver. Common causes are gall stones and cancer of the head of the
pancreas. In this situation, bilirubin is conjugated, but then unable to leave the liver.
Conjugated bilirubin leaks into the systemic circulation, and is delivered to the kidneys,
where it turns the urine dark. No urobilinogen is delivered to the colon, so there is a
reduction in stercobilin levels, and the faeces become pale.

Question 126: FTTTT


The stomach secretes a variety of substances, including water, ions, hydrochloric acid (HCl),
pepsin, mucus, gastrin and intrinsic factor (IF). Regulation of gastric secretion is under
neural and humoral control.
HCl is secreted from parietal (oxyntic) cells which are found in the body and antrum of
the stomach. HCl both converts pepsinogen (from chief cells) into pepsin and provides an
acidic environment in the duodenum to facilitate iron and calcium absorption. Pepsin
converts up to 20% of ingested protein into smaller peptides. It is maximally active at pH 2.0,
and is progressively inactivated as pH rises.
Parietal cells also secrete intrinsic factor, which is required for absorption of vitamin B12
in the terminal ileum. Parietal cells are stimulated to release HCl and IF by histamine,
gastrin and acetylcholine (from the vagus nerve).

Downloaded from https://www.cambridge.org/core. University of Edinburgh, on 19 Aug 2019 at 13:21:27, subject to the Cambridge Core terms of
use, available at https://www.cambridge.org/core/terms. https://doi.org/10.1017/9781108566100.004
Chapter 1b: Physiology Answers 89

Gastrin is secreted by G cells in the antrum of the stomach and stimulates both parietal
and chief cells. Gastrin enhances gastric motility and increases lower oesophageal sphincter
tone; its release is stimulated by antral distension, caffeine and alcohol.
Mucus is secreted from cells around the pylorus; it acts as a protective layer from the acid
and as a lubricant for propulsion.

Question 127: TTTTT


Vomiting is a reflex with afferent and efferent limbs and a central control area. Afferent
signals travel via the vagus nerve and are activated by gastrointestinal mechano-and
chemoreceptors. These stimulate the vomiting centre in the medulla, which also has input
from the cortex, vestibular system, chemoreceptor trigger zone and other viscera. The
cortical afferents to the vomiting centre are stimulated by sight, smell or emotion, as well
as direct insults, e.g. head injuries. Afferent input from the vestibular apparatus is emeto-
genic during vertigo and motion sickness.
The chemoreceptor trigger zone (CTZ) is located on the floor of the fourth ventricle
where the blood–brain barrier is poorly developed. As such, it is highly sensitive to blood-
borne toxins, including drugs. It contains a variety of neurotransmitter receptors, including
5HT3, D2, H2 and ACh.
Efferent impulses from the vomiting centre pass via cranial nerves V, VII, IX, XII and also
spinal nerves to the intercostal, diaphragm and abdominal muscles.
The act of vomiting begins with a giant retrograde contraction from the small intestine,
following profound relaxation of the stomach via the vagus nerve. A deep inspiratory breath
is then taken before the glottis closes and the soft palate elevates to block the nasopharynx.
This then prompts the lower oesophageal sphincter and body of the stomach to relax as the
abdominal, thoracic and diaphragm muscles contract, raising the intra-abdominal pressure
and forcing expulsion of gastric contents through the mouth.

Question 128: TFTFF


Body temperature displays diurnal variation and is lowest in the morning. Control of core
temperature is regulated by the hypothalamus, which receives afferent temperature signals
from cutaneous receptors via the spinothalamic tract. The anterior hypothalamus is sensi-
tive to warm signals and the posterior hypothalamus is sensitive to cold signals. The
hypothalamus processes the information and generates an appropriate response.
The core temperature range in which no response is required is 36.8 to 37.2 °C. This
central regulation is less efficient in older people and critical illness. It is also impaired by
anaesthesia, drugs and hormones, e.g. thyroxine.
If the core temperature falls below this range, a range of adaptive measures are initiated.
Shivering increases the basal metabolic rate (BMR) by 600%, but is ineffective due to the
energy expenditure involved in increased muscle blood flow. Non-shivering thermogenesis
can increase BMR two- to threefold. It occurs from metabolism of brown fat, which is
located between the scapulae and perinephric regions. Brown fat is heavily laden with
mitochondria and increased lipolysis results in ATP formation. This system is particularly
important in neonates.
When the core temperature is raised, the body responds by sweating. Heat is lost through
the latent heat of vaporization. The maximum sweat production is limited to 12 litres per

Downloaded from https://www.cambridge.org/core. University of Edinburgh, on 19 Aug 2019 at 13:21:27, subject to the Cambridge Core terms of
use, available at https://www.cambridge.org/core/terms. https://doi.org/10.1017/9781108566100.004
90 Chapter 1b: Physiology Answers

day. Sodium is also lost during sweating, and great sweat loss may result in cramps, fatigue
and dehydration. Cutaneous vasodilation also occurs in order to increase the surface area of
heat loss available to the body. Vasodilation can increase capillary blood flow up to 7 litres
per minute in extreme cases. As temperatures rise above 40 °C, there is protein denatura-
tion, cellular damage and ultimately death.

Question 129: FFTTT


Swallowing is divided into the oral, pharyngeal and oesophageal stages. It is initially
voluntary, but subsequently comes under autonomic control. Food in the oral cavity
stimulates the salivary glands to secrete saliva, which aids lubrication of food and contains
enzymes that begin the digestive process. Saliva composition is variable, but it is generally
more hypotonic than plasma, with lower sodium concentrations (50 mmol.l−1), and higher
potassium (15 mmol.l−1) and bicarbonate (50 mmol.l−1) concentrations.
To initiate swallowing, the tongue propels the food bolus into the oropharynx against the
hard palate. This stimulates the swallowing centre in the medulla via afferent fibres in
cranial nerves V, IX and X. The autonomic swallowing reflex is then activated via efferent
fibres in cranial nerves V, VII, X and XII.
During the pharyngeal phase the nasopharynx and glottis are closed to prevent aspira-
tion, and breathing briefly stops. The upper oesophageal sphincter (formed by the crico-
pharyngeus muscle) relaxes, and food is propelled towards the oesophagus by rhythmic
contractions of the superior, middle and inferior constrictors of the pharynx.
After the bolus enters the oesophagus, the upper sphincter closes and the lower oesopha-
geal sphincter (LOS) relaxes. Pressure in the distal oesophagus is usually 15–25 mmHg
above that of the stomach, to prevent regurgitation. LOS relaxation occurs 1–2 s after the
start of swallowing and continues for up to 10 s. The bolus moves along at 2–4 cm.s−1 with
the primary peristaltic wave. The LOS is a purely a physiological sphincter and maintains
competency by being compressed by the right crus of the diaphragm and by the acute entry
angle of the oesophagus into the stomach.

Question 130: FFTTT


The majority of Western diets contain large proportions of carbohydrates. The caloric value
of carbohydrates and proteins is 4 kcal.g−1. This is a representation of the amount of energy
stored in different foodstuffs; 1 kcal is equivalent to 4.18 J of energy.
A daily protein intake of 0.5–1.0 g.kg−1.day−1 is required to replace that lost to catabolism.
Proteins are metabolized to amino acids, some of which can be synthesized de novo in the
body. Those that can only be obtained from dietary protein are known as essential amino
acids. There are nine in total: leucine, valine, phenylalanine, histidine, isoleucine, lysine,
methionine, threonine and tyrosine.
Fats have a higher caloric value of 9 kcal.g−1. Essential fatty acids required in the diet are
linoleic and linolenic acid.
The respiratory quotient (RQ) gives a ratio of the amount of carbon dioxide eliminated
from the body per amount of oxygen used. It is used to calculate the basal metabolic rate and
varies depending on which type of food is the predominant source of energy. An RQ of 1.0
denotes all the energy is derived from carbohydrate; fat has a RQ of 0.7 and protein 0.8–0.9.
Usually, the body uses a combination of substrates, so the RQ is <1.0. An estimation of the

Downloaded from https://www.cambridge.org/core. University of Edinburgh, on 19 Aug 2019 at 13:21:27, subject to the Cambridge Core terms of
use, available at https://www.cambridge.org/core/terms. https://doi.org/10.1017/9781108566100.004
Chapter 1b: Physiology Answers 91

RQ is required to calculate the alveolar gas equation. The RQ is a measure of what is


happening at a cellular level and as such is difficult to measure directly. However, it can be
estimated from the respiratory exchange ratio (RER), which can be calculated by measuring
the O2 and CO2 content of air expired from the lungs.

Question 131: FFTFT


The pituitary gland is situated in the middle cranial fossa and occupies the sella turcica of
the sphenoid bone at the base of the skull. The pituitary is related to the third ventricle,
hypothalamus and visual pathways superiorly and the cranial nerves III, IV, V and VI,
cavernous sinus and the internal carotid artery laterally. Pituitary tumours causing mass
effect commonly cause headache and visual field defects. Larger tumours may cause hydro-
cephalus (due to blocking the outflow of the third ventricle), cranial nerve palsies and
hypopituitarism.
The pituitary has two lobes: anterior and posterior. The anterior pituitary (adenohypo-
physis) secretes six hormones: ACTH, GH, FSH, LH, TSH and PRL. The posterior pituitary
(neurohypophysis) secretes two hormones: ADH and oxytocin. ADH and oxytocin are both
synthesized in the hypothalamus, in the supraoptic nucleus and the paraventricular nucleus,
respectively, and then travel down nerve axons to be secreted by the posterior pituitary.
The anterior pituitary is regulated by hypothalamic tropic hormones (TRH, CRH, GnRH,
and PRLH) that reach the anterior pituitary via the portal venous system. The posterior
pituitary is regulated directly by axons from the hypothalamus that synapse with its cells.
Pituitary hormone release is regulated by a negative feedback system.
The blood supply to the anterior pituitary arises from the superior hypophyseal artery
and the posterior pituitary from the inferior hypophyseal artery – both of which are
branches of the internal carotid artery. The hypothalamus receives its blood supply from
the circle of Willis.

Question 132: FFFTT


Oversecretion of growth hormone prior to puberty causes gigantism rather than acrome-
galy. Acromegaly is caused by an excess of growth hormone (GH) after puberty. It usually
occurs as a result of a functioning pituitary tumour causing excess GH release. Acromegaly
has an insidious onset and presents in middle age, affecting men and women equally.
Characteristic features include: prognathism, prominent supraorbital ridges, increase in
the size of hands and feet, macroglossia, macrognathia, coarse oily skin and occasionally
deepening of the voice. Acromegaly is commonly associated with obstructive sleep apnoea
(OSA), osteoarthritis, hypertension and diabetes mellitus.
Cushing’s disease refers to an excess of glucocorticoid due to hypersecretion of ACTH
from a pituitary corticotroph adenoma. Cushing’s syndrome refers to the non-specific state
of chronic glucocorticoid excess, regardless of its cause. Typical characteristics of Cushing’s
syndrome include truncal obesity, moon facies and friable skin. OSA, glucose intolerance
and systemic hypertension are common.
ADH causes water to be reabsorbed in the distal tubules and collecting ducts in the
kidney. ADH release is inhibited by alcohol and reduced osmolarity of extracelluar fluid and
is stimulated by increased extracelluar osmolality, pain, haemorrhage, stress and thirst.
Central diabetes insipidus (DI) results from a failure of ADH release from the

Downloaded from https://www.cambridge.org/core. University of Edinburgh, on 19 Aug 2019 at 13:21:27, subject to the Cambridge Core terms of
use, available at https://www.cambridge.org/core/terms. https://doi.org/10.1017/9781108566100.004
92 Chapter 1b: Physiology Answers

hypothalamic–pituitary axis. DI is associated with traumatic brain injury, subarachnoid


haemorrhage, intracerebral haemorrhage and pituitary surgery. Patients with DI are unable
to concentrate their urine, so produce large volumes of dilute urine. Inappropriate water
loss leads to hypernatraemia with high serum osmolality (>305 mmol.kg−1) and dehydra-
tion. Treatment consists of water replacement and synthetic ADH replacement.
In brainstem death, DI is common. Hypothalamic dysfunction results in loss of thermo-
regulation and functional hypothyroidism that may contribute to the ensuing multiorgan
failure. T3 replacement and vasopressin infusions are commonly used when optimizing
patients for heart-beating donation.

Question 133: TFTTF


The adrenal glands consist of two endocrine organs: the inner adrenal medulla and the outer
adrenal cortex. The adrenal cortex and medulla have distinct embryological origins. The
adrenal cortex is derived from mesodermal cells of the neural crest and the chromaffin cells
in the adrenal medulla from the ectoderm.
The adrenal cortex forms 70% of the adrenal gland and is responsible for the secretion of
three types of steroid hormones: glucocorticoids, mineralocorticoids and androgens. The
adrenal cortex has three distinct layers; from outermost to innermost are the zona glomer-
ulosa, zona fasiculata and the zona reticularis. The zona glomerulosa secretes aldosterone.
The zona fasiculata is the largest layer and secretes mainly cortisol with some sex hormone
production. The zona reticularis mainly secretes sex hormones and a small amount of
cortisol.
Glucocorticoids (mainly cortisol) have catabolic, cardiovascular and anti-inflammatory
effects, and promote glycogen storage in the liver. Glucocorticoids also antagonize the
action of insulin. Mineralocorticoids are essential for electrolyte and fluid balance.
Aldosterone release stimulates sodium reabsorption from the distal convoluted tubule of
the kidney and plasma expansion. There is also resultant potassium and hydrogen ion loss
in the urine. Aldosterone secretion is regulated by the renin–angiotensin system, a fall in
plasma sodium, a rise in plasma potassium and ACTH release from the pituitary.

Question 134: FFFFF


The adrenal medulla is a modified sympathetic ganglion and is innervated by preganglionic,
cholinergic and sympathetic nerve fibres.
The precursor for all catecholamine synthesis is tyrosine, which undergoes a series of
enzyme-catalyzed steps involving hydroxylation and decarboxylation to produce catecho-
lamines. Tyrosine is first converted to dihydroxyphenylalanine (DOPA) via tyrosine dec-
arboxylase – this is the rate limiting step in catecholamine synthesis. DOPA is then
converted to dopamine by DOPA decarboxylase. Dopamine is further converted to nor-
epinephrine via dopamine β hydroxylase. Norepinephrine is finally converted to epinephr-
ine by phenylethanolamine N-methyl transferase (PNMT). Epinephrine usually accounts
for 80% of the adrenal medulla output, with norepinephrine contributing 20%. The adrenal
medulla also produces small quantities of dopamine, chromogranin A, acetylcholine and
metenkephalin.
Adrenal medulla tumours affecting the chromaffin cells are called phaeochromocytomas.
Conn’s syndrome is a term for primary hyperaldosteronism.

Downloaded from https://www.cambridge.org/core. University of Edinburgh, on 19 Aug 2019 at 13:21:27, subject to the Cambridge Core terms of
use, available at https://www.cambridge.org/core/terms. https://doi.org/10.1017/9781108566100.004
Chapter 1b: Physiology Answers 93

Question 135: TFFTT


The endocrine system is responsible for regulating the basal processes such as metabolism
and growth. A hormone is a substance that is secreted into the bloodstream and exerts its
effects on a target tissue distant to its site of release. In general, hormones transmit messages
slowly and to multiple target sites, whereas the nervous system acts more quickly and at
more specific sites. A neuroendocrine cell releases transmitter substances into the blood
stream when it is stimulated by the nervous system.
Hormones can be categorized by their structure: steroid hormones, e.g. aldosterone and
cortisol, single amino-acid derivatives, e.g. thyroxine and catecholamines, and polypeptides,
e.g. insulin, growth hormone and ADH. The precursor for corticosteroid production is
cholesterol, whereas tyrosine is the precursor in catecholamine synthesis.
Hormones exert their effects by attaching to specific receptors on their target cells. There
are two main groups of receptor: intracellular receptors and protein-bound receptors. Fat-
soluble hormones, such as the steroid hormones, can easily diffuse through the cell mem-
brane and bind to receptors in the nucleus or cytoplasm of the cell. The hormone receptor
complex then causes a change in gene expression that alters protein synthesis. Water-
soluble hormones, such as the amino-acid derivatives and polypeptides, use membrane-
bound receptors, commonly G-protein-linked (catecholamines, ADH, glucagon) or tyro-
sine kinase-linked receptors (insulin), which rely on secondary messengers to simulate
cellular processes within the cell.

Question 136: FFTTF


Calcitonin is a polypeptide produced by the parafollicular C cells in the thyroid. It has a
minor role in calcium homeostasis and this is why after thyroidectomy, calcium balance is
not generally affected unless the parathyroid glands have also been removed. Calcitonin
decreases plasma calcium levels by inhibiting calcium release from bone (by inhibiting
osteoclasts) and increasing calcium and phosphate excretion via the kidneys.

Question 137: FFTTF


Calcium plays a vital role in a number of physiological processes. It has an essential role in
muscle contraction, nerve conduction, the clotting cascade and acts as an important
intracellular messenger. Symptoms of hypocalcaemia include confusion, tetany, laryngo-
spasm, hypotension and arrhythmias.
In the body, 99% of the calcium is stored in bone, 0.3% in muscle and 0.7% in other
tissues. Of this small proportion of extra-skeletal calcium, 40% of plasma calcium is bound
to albumin, 10% exists in complexes with other molecules and the remaining 50% is ionized.
It is the ionized calcium in the plasma that is active and under tight homeostatic control.
Calcium homeostasis is achieved by the balance of three hormones: PTH, calcitonin and
cholecalciferol or vitamin D3. These hormones control plasma calcium levels by exerting
effects on the absorption or secretion of calcium from the GI tract, as well as reabsorption of
calcium from the bones and removal of calcium via urinary excretion.
PTH is produced by chief cells in the parathyroid glands. It causes increased plasma
calcium and decreased plasma phosphate levels. PTH increases plasma calcium levels by
mobilizing bone stores and increasing calcium reabsorption in the renal tubules. It also

Downloaded from https://www.cambridge.org/core. University of Edinburgh, on 19 Aug 2019 at 13:21:27, subject to the Cambridge Core terms of
use, available at https://www.cambridge.org/core/terms. https://doi.org/10.1017/9781108566100.004
94 Chapter 1b: Physiology Answers

increases calcium and phosphate absorption from the gastrointestinal tract by activating
1,25-hydroxycholecalciferol (active vitamin D3). PTH causes increased phosphate absorp-
tion from bone and the gastrointestinal tract, but because it also increases phosphate
excretion in the renal tubules, the net effect of PTH on phosphate is to lower plasma levels.

Question 138: FTFFT


Thyroid hormones control basal metabolism and heat production. Thyroid hormones
enhance the effects of catecholamines by increasing the number and sensitivity of
β-adrenoreceptors. They also enhance the glycogenic effect of insulin, increase growth
hormone levels and are integral to normal growth and nervous system development.
Thyroid hormone release is controlled by TSH from the anterior pituitary and TRH from
the hypothalamus.
The thyroid secretes two forms of thyroid hormone: 90% thyroxine (T4) and 10%
triiodothyronine (T3). T3 is three to five times more potent than T4, but has a shorter
half-life. Tissues are able to convert T4 into T3. Thyroid hormones are 99% protein bound,
mainly by thyroid-binding globulin and to a lesser extent by albumin.
T3 and T4 are synthesized in the follicular cells of the thyroid by successive iodination of
tyrosine bound to thyroglobulin by the enzyme thyroperoxidase. Iodide is actively trans-
ported into follicular cells and oxidized to iodine, which then binds with tyrosine to form
mono- and diiodothyronine. Mono- and diiodothyronine combine to form triiodothyro-
nine (T3) and tetraiodothyronine (T4).

Question 139: TFFTT


Erythropoietin is a haemopoietic, glycoprotein growth factor produced in the renal cortex
(85%) and liver (15%). It causes increased red cell production in the bone marrow.
Production is reduced in chronic renal failure and may be increased in renal tumours,
causing polycythaemia. Erythropoietin secretion is stimulated by hypoxia, haemorrhage,
catecholamines, androgens, alkalosis, cobalt salts and adenosine. Erythropoietin secretion is
inhibited by increased red cell volume and theophylline. Treatment with erythropoietin is
associated with myocardial infarction, stroke, venous thromboembolism and tumour
recurrence.

Question 140: FFTFT


The pancreas is a secretory organ with endocrine and exocrine function. The main func-
tional unit is the acinar cell. The islets of Langerhans are formed by the endocrine cells
consisting of β (B) cells secreting insulin, α (A) cells secreting glucagon, δ (D) cells secreting
somatostatin and F cells secreting pancreatic polypeptide. These hormones are secreted into
the portal circulation.
Insulin is an anabolic hormone responsible for glucose uptake into muscle and fat. (It
should be noted that some tissues do not require insulin for effective uptake of glucose, like
the brain and the liver. This is because these organs use a different glucose transporter
protein, which is insulin independent.) Insulin consists of 51 amino acids organized in two
chains joined by two disulfide bridges. Normal insulin production is in the region of 40–50
units per day. Insulin secretion is stimulated by hyperglycaemia, β-agonists and glucagon.

Downloaded from https://www.cambridge.org/core. University of Edinburgh, on 19 Aug 2019 at 13:21:27, subject to the Cambridge Core terms of
use, available at https://www.cambridge.org/core/terms. https://doi.org/10.1017/9781108566100.004
Chapter 1b: Physiology Answers 95

Insulin secretion is inhibited by hypoglycaemia, β-blockers, α-agonists, somatostatin, thia-


zides and volatile agents.
Somatostatin, also called growth hormone-inhibiting factor, inhibits insulin and gluca-
gon release. Somatostatin also inhibits gastric acid production and gall bladder contraction
and reduces splanchnic blood flow. Somatostatin analogues, such as octreotide, are used to
treat a range of conditions, including acromegaly, hormone-secreting tumours of the
gastrointestinal tract (e.g. carcinoid) and variceal bleeding.
Glucagon is a catabolic hormone consisting of 29 amino acids. It opposes the effects of
insulin and is also a direct positive inotrope. Glucagon release is stimulated by hypoglycae-
mia, increased amino acids, β-agonists, sepsis, stress and trauma. Glucagon secretion is
inhibited by hyperglycaemia, decreased amino acids, increased free fatty acids, insulin,
somatostatin and α-agonists.

Question 141: FTTFT


In the UK, the ABO system of blood groups occur with the following frequency: O (47%), A
(42%), B (8%) and AB (3%). The ABO system describes the surface antigens present on
erythrocytes and antibodies found in the plasma; its importance is in determining blood
transfusion compatibility.
The antigens carried by the various blood group erythrocytes are as follows: A – A, B – B,
AB – A and B, O – no antigens. The antibodies present in the blood stream for each group
are as follows: A – anti-B, B – anti-A, AB – nil, O – anti-A and anti-B. It is because group O
erythrocytes have no surface antigens that blood group O is described as the ‘universal
donor’. In subjects who are group AB, the serum is devoid of ABO antibodies making this
group the ‘universal recipient’.
The rhesus system is the second most important in determining compatibility, with the
terms rhesus positive and negative describing the presence or absence of rhesus antigen,
respectively. A rhesus negative subject exposed to rhesus positive blood (e.g. through blood
transfusion or pregnancy of a rhesus negative mother with a rhesus positive fetus) can
develop antirhesus antibodies (IgG) that have the potential to cause harm, e.g. haemolytic
disease of the newborn.

Question 142: FFTFT


The cell-based model has now superseded the classical model (extrinsic and intrinsic
pathways) as the accepted model for clotting in vivo. The classical model does not explain
some clinical observations, e.g. haemophiliacs bleed, whereas patients with factor XII
deficiency do not. In fact, patients with deficiencies in factors XII, prekalikrein or high
molecular weight kininogen all have prolonged APTTs, but no bleeding tendency.
The classical pathways lead to a final common pathway (FCP). The factors mentioned (I,
II, V, VII and X) all affect INR as they form part of the extrinsic pathway. Deficiencies in
factors in the intrinsic pathway affect the APTT. Deficiencies in factors X, V and II (part of
the FCP) cause prolongation of both APTT and PT (INR is a standardized derivative of PT).
The cell-based model comprises the following stages: initiation, amplification, propaga-
tion, stabilization and inhibition. Initiation generates a small amount of thrombin; feedback
causes more to be generated in amplification. The ‘thrombin burst’ occurs as part of the
propagation phase.

Downloaded from https://www.cambridge.org/core. University of Edinburgh, on 19 Aug 2019 at 13:21:27, subject to the Cambridge Core terms of
use, available at https://www.cambridge.org/core/terms. https://doi.org/10.1017/9781108566100.004
96 Chapter 1b: Physiology Answers

Haemostasis is countered by naturally occurring anticoagulants to ensure that the process


is localized and not overwhelming. The most important naturally occurring anticoagulants
are tissue factor pathway inhibitor (TFPI), as well as co-factor inhibitors (proteins C and S)
and serine protein inhibitors e.g. antithrombin.

Question 143: FFTTT


The production of antibodies is facilitated by specific T-helper cells (TH cells) already
activated by antigen-presenting cells. Cytokines released by TH cells augment the transition
of B cells to antibody-secreting plasma cells and their subsequent proliferation.
Antibodies consist of four protein chains; two are heavy and two are light in a ‘Y’ shaped
arrangement. The heavy chains form the base of the ‘Y’ and interact with other cells (Fc
portion), whereas the light chains form the arms of the ‘Y’ and contain antigen-binding
sites. At the DNA level, rearrangement of variable, joining and diversity genes produces a
great variety of antigen binding sites.
A brisk and augmented immune response upon re-exposure to a previously encountered
antigen is termed a secondary response or a memory response. The memory response is
characterized by a shorter lag, higher antibody titre, prolonged plateau phase and slower
decline, with greater affinity of the antibodies (IgG type) for the antigen.
Antibody-dependent cell-mediated cytotoxicity (ADCC) describes the action of effector
cells that can recognize and destroy pathogens that have been coated in antibody (opso-
nized). Effector cells recognising the Fc portion achieve ADCC; examples include mono-
nuclear phagocytes, T cells and NK cells.

Question 144: TFTFF


Plasma is the non-cellular component of blood, i.e. the extracellular fluid that is contained
intravascularly. One key feature of plasma make-up is its high protein content, which is
approximately 7 g per 100 ml or 70 g.l−1. Of the total plasma protein albumin constitutes
60% (35–50 g.l−1), immunoglobulins account for 20% (25–30 g.l−1) and fibrinogen accounts
for 7%. The total volume of plasma is approximately 3.5 l in adults, which equates to 5% of
total body weight. Plasma osmolality is measured in mosmol.kg−1 and falls within the range
of 280–305 mosmol.kg−1. Osmolarity is measured in mosmol.l−1.

Question 145: TTFFT


Anaphylaxis is defined as a severe, life-threatening, generalized or systemic hypersensitivity
reaction. Allergic reactions in anaesthesia were previously categorized as anaphylactic (with
IgE cross-linking) or anaphylactoid (without IgE cross-linking). The term anaphylactoid is
now used less in the European literature and anaphylaxis itself is described as allergic where
an immune mechanism, e.g. IgE cross-linking, is involved, or as non-allergic where there is
no such mechanism. The clinical features of both allergic and non-allergic anaphylaxis can
be identical.
Approximately 60% of anaesthesia-related anaphylaxis is triggered by non-depolarizing
muscle relaxants (NDMRs), based on the results of skin prick tests. The antigens triggering
anaphylaxis to NDMRs are quaternary ammonium ions (QAIs); these are found in tooth-
pastes, washing detergents, shampoos and cough medicines, as well as NDMRs. Prior

Downloaded from https://www.cambridge.org/core. University of Edinburgh, on 19 Aug 2019 at 13:21:27, subject to the Cambridge Core terms of
use, available at https://www.cambridge.org/core/terms. https://doi.org/10.1017/9781108566100.004
Chapter 1b: Physiology Answers 97

sensitization to QAIs that can later trigger anaphylaxis upon exposure to NDMRs is very
feasible.
Current guidelines stipulate the collection of mast cell tryptase samples should be during
the initial resuscitation, at 1–2 hours after the onset of symptoms and lastly at 24 hours/in
convalescence. Further tests at an anaesthesia allergy clinic may include skin prick tests
(usually the initial investigation) and intradermal skin tests, which are more sensitive but
less specific in comparison.

Question 146: FTFFT


Physiological changes in the elderly affect all body systems. The volume of distribution of
water-soluble drugs is decreased, allowing dose reduction, while that of lipid-soluble drugs
is increased, resulting in their prolonged clearance.
Closing capacity exceeds functional residual capacity at age 40 when supine, and at age 65
when upright, resulting in the closure of smaller alveoli. Collapsed alveoli are perfused, but
not necessarily ventilated, causing ventilation/perfusion mismatch and in turn venous
admixture.
Deterioration in renal function is related to a reduction in the number and mass of
glomeruli (by 30% in the 8th decade) as well as impairment in tubular function. The
measurement of serum creatinine is, however, dependent on body mass and so a frail
elderly person may have a normal creatinine and yet significant renal dysfunction.
A shorter onset time with volatile agents is observed in the elderly due to a reduction in
cardiac output and lower blood gas partition coefficients, i.e. greater solubility of the volatile
agent in blood. Oil–gas partition coefficients affect the potency of volatile agents.
Maximal cardiac output with exercise declines by 1% per year over the age of 50 and so a
45% reduction would be expected in this patient.

Question 147: TTFTT


Delirium and longer-term cognitive dysfunction are common complications of anaesthesia
in the elderly.
Dementia refers to chronic and irreversible cognitive dysfunction that can have varied
aetiology, e.g. vascular dementia, Alzheimer’s disease. Dementia has a prevalence of 10% in
people aged 60 and above and 20% in those aged 80 and above. Pre-existing dementia
predisposes to perioperative cognitive dysfunction, e.g. delirium. An important distinction
between these two diagnoses is that delirium is acute in onset, short-term in duration and
potentially reversible, dementia is not.
Postoperative cognitive dysfunction (POCD) is defined as cognitive dysfunction that is a
long-term, possibly permanent and disabling deterioration in cognitive function after an
operation. Although it is difficult to quantify accurately, it is suggested that its incidence is
10% at three months postoperatively and 1% suffer unresolved POCD at two years
postoperatively.
POCD has a multifactorial aetiology; cerebral emboli and perioperative biochemical and
physiological dysfunction are likely to be contributory. Serum concentrations of markers
known to signify brain damage, e.g. neurone-specific enolase, correlate poorly with POCD.
Factors protective against developing early POCD (but not necessarily late POCD) include
regional anaesthesia and a higher level of preoperative intellectual performance.

Downloaded from https://www.cambridge.org/core. University of Edinburgh, on 19 Aug 2019 at 13:21:27, subject to the Cambridge Core terms of
use, available at https://www.cambridge.org/core/terms. https://doi.org/10.1017/9781108566100.004
98 Chapter 1b: Physiology Answers

Question 148: TFFFT


In atrial fibrillation, loss of the ‘atrial kick’ results in a fall in cardiac output of approxi-
mately 30%.
While β-receptor sensitivity is blunted in the elderly, sensitivity to α-agonism remains
relatively unaffected.
The response of the carotid baroreceptors is blunted as compared with younger subjects,
and this can compound the hypotensive effect of general anaesthesia. The main reason for
the exaggerated hypotension at induction that results from general anaesthesia is impaired
autonomic homeostasis, compounded by the effect of inhalational and intravenous anaes-
thetic agents.
In the early postoperative period (up to 30 days), use of regional anaesthesia has been
shown to confer some benefit in terms of a reduction in the rate of deep venous thrombosis,
pulmonary emboli and reduced transfusion requirements, but these benefits are not seen at
three months or beyond.

Question 149: FFFFT


Decreased compliance of the lungs and chest wall, as well as a reduction in the elastic
support of the airways leads to a reduction in many of the measured lung volumes and
capacities (TLC, FEV1, FVC), except for residual volume and closing capacity, which are
increased. Functional residual capacity (FRC) remains unchanged or may even increase
(due to an increase in residual volume) with advancing age.
In quiet breathing, if closing capacity encroaches upon FRC, smaller airways and alveoli
are seen to close. Therefore, rising closing capacity is seen with advancing age and can
contribute to hypoxaemia of old age by causing the collapse and closure of small alveoli.
Collapsed alveoli remain perfused, but not adequately ventilated, resulting in an increase in
the shunt fraction and venous admixture.

Question 150: TTFTT


Advancing age causes deterioration in function in all body systems; some physiological
parameters are affected to a greater extent than others (Table 1.150.1).
Table 1.150.1 As a % of the value for a young
adult, the following apply to a 70-year-old male:

Muscle mass 74
Cardiac output at rest 64
Cerebral blood flow 80
Oxygen consumption at rest 74
Renal blood flow 60

Downloaded from https://www.cambridge.org/core. University of Edinburgh, on 19 Aug 2019 at 13:21:27, subject to the Cambridge Core terms of
use, available at https://www.cambridge.org/core/terms. https://doi.org/10.1017/9781108566100.004
Chapter
Anatomy Questions

2a
Question 1
Veins in the neck are often used for intravascular access. The anatomical landmarks are
important in identifying the vessel in scenarios without ultrasound guidance. Which of the
following statements are true?
a. The internal jugular vein begins at the jugular foramen and is accompanied by the
glossopharyngeal, vagus and accessory nerves as they exit the skull
b. The internal jugular vein terminates at the subclavian vein
c. The carotid sheath contains the internal jugular vein, the vagus nerve and the common
carotid artery
d. The anterior triangle of the neck is formed by the sternocleidomastoid, the mandible and
the mid-line of the neck
e. The superior thyroid vein is one of the tributaries of the internal jugular vein and must
be divided first during a thyroidectomy in order to stop bleeding

Question 2
With regards to the large veins of the neck, which of the following are true?
a. The subclavian vein is a continuation of the axillary vein, beginning at the lateral border
of the first rib and ending just medial to the sternocleidomastoid
b. The external jugular vein, which lies in the posterior triangle of the neck, drains directly
into the subclavian vein
c. The right brachiocephalic vein is almost double the length of its left-sided
counterpart
d. Cannulation of the subclavian vein is best achieved by inserting the needle below the
mid-point of the clavicle and aiming towards to the manubriosternal joint
e. Cannulation of the internal jugular vein can be achieved by inserting the needle half way
between the mastoid process and the sternal notch – in line with C6

Question 3
With regards to the cervical plexus:
a. It is formed from the anterior rami of C1–C5
b. The phrenic nerve originates from C3–C5 and travels down the neck behind the internal
jugular vein
c. Superficial branches supply structures to the back of the head and the skin at the front of
the neck

99
Downloaded from https://www.cambridge.org/core. University of Edinburgh, on 19 Aug 2019 at 13:21:27, subject to the Cambridge Core terms of
use, available at https://www.cambridge.org/core/terms. https://doi.org/10.1017/9781108566100.005
100 Chapter 2a: Anatomy Questions

d. Deeper branches supply muscles of the neck, including the sternocleidomastoid and
trapezius
e. Cervical plexus blockade can lead to blockade of the cervical sympathetic chain, leading
to Horner’s syndrome

Question 4
With regards to the pharynx, which of the following are true?
a. The pharynx originates at the skull base, and extends to the level of the C4 vertebra
b. The walls of the pharynx are coated in four different layers – mucous, fibrous, muscular
and fascial
c. The muscular coat consists of the constrictor muscles – superior, middle and inferior
d. A pharyngeal pouch is formed between differences in the two functional components of
the muscle
e. Pharyngeal muscles involved in swallowing are supplied by the vagus and glossophar-
yngeal nerves

Question 5
The nasopharynx, oropharynx and laryngopharynx lie between the pharynx and larynx.
a. The Eustachian tube opens into the nasopharynx
b. Another name for the adenoids is the nasopharyngeal tonsils
c. The oropharynx extends to the level of the epiglottis where it joins with the nasopharynx
d. The laryngopharynx begins at the epiglottis and reaches the lower border of the cricoid
cartilage
e. The piriform fossae are recesses in the larynx that are often implicated in the accidental
swallowing of fish bones

Question 6
The larynx is a complex structure containing a number of cartilages and ligaments.
a. The thyroid cartilage has two plates which join to form the Adam’s apple
b. The cricoid cartilage at the level of C6 articulates with the inferior horn of the thyroid
cartilage
c. The arytenoid cartilages are paired and each projects anteriorly as the vocal process
d. The valleculae are depressions formed on either side of the epiglottis as it communicates
with the tongue
e. The corniculate and cuneiform cartilages lie in the aryepiglottic folds

Question 7
The membranes and ligaments within the larynx are as follows:
a. The aryepiglottic fold gives rise to the false vocal cord
b. The cricothryoid ligament is the recommended site for insertion of an emergency
tracheostomy
c. The thyrohyoid membrane contains the passage of the recurrent laryngeal nerve and the
laryngeal artery

Downloaded from https://www.cambridge.org/core. University of Edinburgh, on 19 Aug 2019 at 13:21:27, subject to the Cambridge Core terms of
use, available at https://www.cambridge.org/core/terms. https://doi.org/10.1017/9781108566100.005
Chapter 2a: Anatomy Questions 101

d. The conus elasticus extends to form the vocal ligament as it attaches with the cricoid
cartilage
e. During a grade 1 laryngoscopy, a direct view should be obtained of the vallecula,
epiglottis and the true vocal cords

Question 8
The cartilages and ligaments within the larynx are controlled by the intrinsic and extrinsic
laryngeal muscles.
a. The sternothyroid attaches to the thyroid cartilage and elevates the larynx
b. The thyrohyoid attaches the thyroid cartilage to the hyoid bone and depresses the larynx
c. The posterior cricoarytenoids cause abduction of the vocal cords
d. The lateral cricoarytenoids cause closure of the glottis
e. The transverse arytenoid is the only unpaired, intrinsic muscle

Question 9
With respect to the blood and nerve supply within the larynx:
a. The arterial supply to the larynx is from the superior and inferior laryngeal arteries,
which are branches of the superior thyroid artery
b. Venous drainage empties into the internal jugular vein
c. All muscles, except the sternothyroid, are supplied by the recurrent laryngeal nerve
d. The recurrent laryngeal nerve and the superior laryngeal artery traverse the thyrohyoid
membrane
e. Lymphatic drainage from the larynx drains into the deep cervical chain

Question 10
The orbital cavity is frequently operated on under the provision of local anaesthetic
blockade. It is therefore useful to have a good understanding of the structures held within
the orbit and the globe.
a. The orbit is pyramidal in shape
b. The orbit contains two posterior openings: the superior and the inferior orbital fissures
c. The globe is located anteriorly within the orbit and consists of three layers
d. The vascular layer of the globe contains the choroid, the iris and the ciliary bodies
e. Each retina is made up of approximately 120 million rods and 7 million cones

Question 11
Control of vision is determined by the following:
a. Action of the ciliary muscle, which alters tension in the suspensory ligaments
b. The smooth muscles fibres in the iris control the size of the pupil
c. Ganglion cells in the retina give rise to axons, which gather at the optic disc and lead to
the optic nerve
d. Axons from the nasal half remain ipsilateral, whereas axons from the temporal half
decussate at the optic chiasm
e. The superior colliculus regulates movement of the eye

Downloaded from https://www.cambridge.org/core. University of Edinburgh, on 19 Aug 2019 at 13:21:27, subject to the Cambridge Core terms of
use, available at https://www.cambridge.org/core/terms. https://doi.org/10.1017/9781108566100.005
102 Chapter 2a: Anatomy Questions

Question 12
Regarding the nerves of the orbit:
a. CN III carries sympathetic fibres to constrict the pupil
b. CN IV supplies the superior rectus muscle
c. CN V is implicated in the corneal reflex
d. CN VI supplies the lateral rectus muscle
e. CN VII has an ophthalmic division, damage to which can lead to unilateral ptosis

Question 13
The mouth is the most frequently assessed component of an airway assessment. Variations
in structures within the mouth can often pose significant difficulties in obtaining an
adequate airway.
a. On a grade 1 laryngoscopy view, one would be able to see the palatoglossal and the
palatopharyngeal arches
b. The tongue is located in the oral cavity and is controlled by actions of the intrinsic and
extrinsic muscles
c. The nerve supply to muscles of the tongue is primarily from cranial nerve IX, the
hypoglossal nerve
d. The anterior two-thirds of the tongue are innervated by the glossopharyngeal nerve
e. Lingual nerve fibres pass into the facial nerve via the chorda tympani

Question 14
With regards to anatomical knowledge of the nose:
a. The nasal cavity is divided by the nasal septum and contains a roof, a floor and two walls
b. The cribiform plate of the ethmoid bone contributes to the roof of the nasal cavity
c. The nasal septum makes up the medial wall and receives openings from the paranasal
sinuses
d. The lateral wall contains three conchae, anterior, middle and posterior, which act to
increase the surface area
e. Arterial supply to the nose is via the ophthalmic arteries and venous drainage is via the
maxillary veins

Question 15
With regards to the thyroid gland:
a. The thyroid gland is located at the level of C5–T1
b. The thyroid isthmus overlies the trachea with two lobes on either side
c. The acini within the gland contain colloid which stores thyroglobulin
d. Parafollicular cells within the gland are responsible for the secretion of calcitriol
e. Arterial supply to the thyroid is via the superior, middle and inferior thyroid arteries

Downloaded from https://www.cambridge.org/core. University of Edinburgh, on 19 Aug 2019 at 13:21:27, subject to the Cambridge Core terms of
use, available at https://www.cambridge.org/core/terms. https://doi.org/10.1017/9781108566100.005
Chapter 2a: Anatomy Questions 103

Question 16
The parathyroid hormone is involved with calcium homeostasis.
a. There are four parathyroid glands located in the upper poles of the thyroid gland
b. Chief cells within the gland produce parathyroid hormone
c. Parathyroid hormone is a polydisaccharide hormone that is stored in the parathyroid
gland prior to being released into the bloodstream
d. Increase in the level of parathyroid hormone causes an overall increase in serum calcium
levels
e. Calcitonin, produced by the parafollicular cells, augments the activity of parathyroid
hormone in increasing serum calcium levels

Question 17
The following structures are present within the carotid sheath:
a. Common carotid artery
b. Internal carotid artery
c. Internal jugular vein
d. Vagus nerve
e. Hypoglossal nerve

Question 18
The internal jugular vein lies:
a. Anterolateral to the carotid artery
b. Lateral to the vagus nerve
c. Posterior to the thoracic duct
d. Medial to the hypoglossal nerve
e. Posterior to the sympathetic chain

Question 19
Complications associated with a deep cervical plexus block include:
a. Total spinal
b. Phrenic nerve palsy
c. Horner’s syndrome
d. Pneumothorax
e. Recurrent laryngeal nerve palsy

Question 20
When performing a carotid endarterectomy under local anaesthesia, the following nerves
need to be blocked:
a. Supraclavicular nerve
b. Trigeminal nerve
c. Transverse cervical nerve
d. Greater occipital nerve
e. Lesser auricular nerve

Downloaded from https://www.cambridge.org/core. University of Edinburgh, on 19 Aug 2019 at 13:21:27, subject to the Cambridge Core terms of
use, available at https://www.cambridge.org/core/terms. https://doi.org/10.1017/9781108566100.005
104 Chapter 2a: Anatomy Questions

Question 21
A maxillary nerve block can be administered in the pterygopalatine fossa. The following
structures are present in the fossa:
a. Maxillary nerve
b. Maxillary artery
c. Greater petrosal nerve
d. Lingual nerve
e. Lingual artery

Question 22
With regards to a sub-Tenon’s block:
a. The Tenon’s capsule is a thin layer of connective tissue separating the globe from the
orbital fat
b. Local anaesthetic and aqueous iodine are applied to the eye prior to incision
c. An incision is made in the lateral aspect of the conjunctiva
d. The injection is made on the posterior aspect of the sclera
e. A subconjuctival haemorrhage is seen in 50% of patients following insertion of the block

Question 23
The following complications are associated with a partial thyroidectomy:
a. Haematoma
b. Stridor
c. Hypercalcaemia
d. Recurrent laryngeal nerve palsy
e. Thyroid storm

Question 24
Common preoperative features associated with a thyroid goitre include:
a. Tracheomalacia
b. Engorged nasopharyngeal vein
c. Increased venous return
d. Hoarse voice due to vocal cord abduction
e. Stridor

Question 25
With regards to recurrent laryngeal nerve injury during thyroid surgery:
a. The left is more prone to injury than the right
b. Unilateral nerve injury will lead to hoarseness
c. Bilateral nerve injury will lead to stridor
d. Corrective surgery should be performed immediately
e. Unilateral vocal cord paralysis will cause adduction of the cord

Downloaded from https://www.cambridge.org/core. University of Edinburgh, on 19 Aug 2019 at 13:21:27, subject to the Cambridge Core terms of
use, available at https://www.cambridge.org/core/terms. https://doi.org/10.1017/9781108566100.005
Chapter 2a: Anatomy Questions 105

Question 26
This question concerns the structure of the trachea. Which of the following statements are
true?
a. It originates at the inferior end of the larynx, at approximately level C6
b. It bifurcates into bronchioles at the level of the sternal angle, T5
c. It has approximately 20 complete, circular, cartilaginous rings for support
d. Is lined with pesudostratified ciliated columnar epithelium cells
e. The trachealis muscle provides support to the cartilaginous rings

Question 27
The following question concerns the relationships of the trachea.
a. The oesophagus lies posterior to the trachea
b. The carotid sheath lies lateral to the trachea bilaterally
c. The arch of the aorta runs posterior to the trachea
d. The thyroid ima artery runs posterior to the trachea
e. The inferior thyroid veins run lateral to the trachea bilaterally

Question 28
The following question is regarding the anatomy related to the insertion of a tracheostomy.
a. A tracheostomy is normally placed superior to the thyroid gland
b. A tracheostomy should be placed between the second and third tracheal rings
c. The trachealis muscle should be divided longitudinally and retracted laterally
d. The inferior thyroid artery is often encountered anterior to the thyroid and should be
dissected clear and divided, to protect against bleeding
e. The inferior thyroid veins often form a plexus on the anterior surface of the trachea and
may lead to bleeding if divided

Question 29
With regards to tracheostomies:
a. Dual antiplatelet therapy is an absolute contraindication to performing a tracheostomy
b. Damage to the superior laryngeal nerve can lead to vocal cord paralysis
c. The thyroid isthmus lies over the third and fourth tracheal rings
d. A platelet count <50,000 μl–1 is an absolute contraindication for insertion of
a percutaneous tracheostomy
e. Wound infections are higher with surgical tracheostomies than with percutaneous
tracheostomies

Question 30
Early complications of tracheostomy insertion may include:
a. Pneumothorax
b. Tracheal cartilage fracture

Downloaded from https://www.cambridge.org/core. University of Edinburgh, on 19 Aug 2019 at 13:21:27, subject to the Cambridge Core terms of
use, available at https://www.cambridge.org/core/terms. https://doi.org/10.1017/9781108566100.005
106 Chapter 2a: Anatomy Questions

c. Tracheomalacia
d. Persistant tracheal stoma
e. Bleeding from the innominate artery

Question 31
The intercostal space typically contains three muscles and a neurovascular bundle.
The following question concerns the relations of these structures with the ribs.
a. The most superficial muscle layer is the external intercostal muscle
b. The deepest muscle layer is the internal intercostal muscle
c. The intercostal neurovascular bundle runs posterior and inferior to the rib above
d. The dorsal rami of T1–T11 form the intercostal nerve
e. The arterial supply of the intercostal spaces comes from the anterior and posterior
intercostals

Question 32
The following question is regarding the nerve supply to the intercostal spaces.
a. There are 12 pairs of thoracic spinal nerves that supply the intercostal spaces
b. Along its course, the intercostal nerve gives off a number of branches that supply the
intercostal muscles and skin
c. In supplying the skin in this manner, each intercostal nerve supplies its own dermatome
d. A dermatome is the cutaneous area supplied by the ventral rami of a spinal nerve root
e. Rami communicantes connect each intercostal nerve to the parasympathetic trunk

Question 33
Concerning the left main bronchus:
a. It is more vertical than the right
b. It passes inferior to the arch of the aorta
c. It passes anterior to the oesophagus
d. It passes posterior to the arch of the aorta
e. It is more common for inhaled foreign bodies to become lodged within in it

Question 34
Concerning the bronchial tree:
a. Each main bronchus is accompanied by pulmonary artery and vein, bronchial vessels,
lymph vessels and nerves at the root of the lung
b. Each main bronchus divides into secondary and then tertiary bronchi
c. There are two secondary bronchi on the left and three on the right
d. Each tertiary bronchus supplies a bronchopulmonary segment
e. Oblique fissures separate the superior and inferior lobes bilaterally

Downloaded from https://www.cambridge.org/core. University of Edinburgh, on 19 Aug 2019 at 13:21:27, subject to the Cambridge Core terms of
use, available at https://www.cambridge.org/core/terms. https://doi.org/10.1017/9781108566100.005
Chapter 2a: Anatomy Questions 107

Question 35
Concerning the surfaces of the lung:
a. Each lung has three surfaces
b. The cardiac surface is adjacent to the mediastinum
c. The costal surface is adjacent to the sternum
d. The diaphragmatic surface is adjacent to the diaphragm
e. The root of the lung is on the mediastinal surface

Question 36
Concerning the borders of the lungs:
a. Each lung has three borders
b. Each lung has an apex
c. The superior border is at the apex, which extends through the superior thoracic
aperture
d. The inferior border runs along the diaphragmatic surface
e. The medial border runs along the mediastinum and forms the cardiac notch on the left

Question 37
With regards to the blood supply and lymph drainage of the lungs and bronchi:
a. The right lung is supplied by two arteries and the left lung is supplied by one artery
b. Each lung is supplied by a superior and corresponding lobar branches from the pul-
monary arteries
c. The bronchial arteries carry deoxygenated blood from the pulmonary arteries
d. The superficial and deep lymphatic plexuses drain both of the lungs
e. The deep lymphatic plexus drains directly into the thoracic duct

Question 38
The nerve supplies to the lungs and visceral pleura are closely related. The following
question therefore considers the nerve supply to both organs:
a. The pulmonary plexuses contain parasympathetic and sympathetic nerve fibres
b. The parasympathetic nerve fibres arise from the vagus nerve (CN XI)
c. The sympathetic fibres arise from the sympathetic trunk
d. The plexuses are located at the roots of the lung
e. Afferent fibres from the pleura and bronchi accompany the parasympathetic fibres

Question 39
Regarding the nerve supply to the lungs and surrounding structures, which of the following
statements are correct?
a. Parasympathetic innervation leads to bronchoconstriction
b. Parasympathetic innervation leads to vasoconstriction
c. Sympathetic innervation is secretomotor to the bronchial tree

Downloaded from https://www.cambridge.org/core. University of Edinburgh, on 19 Aug 2019 at 13:21:27, subject to the Cambridge Core terms of
use, available at https://www.cambridge.org/core/terms. https://doi.org/10.1017/9781108566100.005
108 Chapter 2a: Anatomy Questions

d. The nerve supplies to both lung and visceral pleura arise from intercostal and phrenic
nerves
e. Nerves to the parietal pleura arise from intercostal and phrenic nerves

Question 40
The pleurae are sacs, which line the inner surface of the thoracic cavity and form
a lubricating layer over the lungs.
a. The visceral pleura covers the surface of the lungs
b. The parietal pleura covers the thoracic wall
c. The parietal pleura is continuous with the abdominal peritoneum
d. The parietal pleura is continuous with the cervical pleura
e. The parietal and visceral pleurae are continuous at the root of the lung

Question 41
The thoracic diaphragm forms a dome-shaped, muscular division between the abdominal and
thoracic cavities. The following statements relate to the function and structure of the
diaphragm.
a. The main function of the diaphragm is in expiration
b. The main function of the intercostal muscles is in inspiration
c. The diaphragm is lined by the visceral peritoneum
d. The diaphragm is entirely muscular
e. The diaphragm is anatomically continuous with the structures that pass through it
between the thorax and abdomen

Question 42
Given that the diaphragm spans the floor of the thorax, there are a number of important
attachments related to it. Which of the following statements are true?
a. The anterior-most section of the muscular portion of the diaphragm attached to the
xiphoid process forms two foramina laterally to it, between the sternal part and the costal
part of the muscle
b. The costal margins insert into the lower border of the ribs and costal cartilages 6–12
c. Posteriorly, the diaphragm attaches to the lumbar vertebrae via the crura
d. The right crus of the diaphragm inserts into the anterior surface of L1–L3
e. The left crus of the diaphragm inserts in the anterior surface of L1–L2

Question 43
The diaphragm acts as the division between the abdomen and thorax. However, it allows the
passage of a number of structures from thorax to abdomen and vice versa. Which of the
following statements are true?
a. The aortic hiatus is formed by the median arcuate ligament at the level of T12
b. The oesophagus passes through the central muscular portion of the diaphragm
c. With the oesophagus, the vagus and oesophageal branches of the left gastric vessels also
pass through the oesophageal foramen

Downloaded from https://www.cambridge.org/core. University of Edinburgh, on 19 Aug 2019 at 13:21:27, subject to the Cambridge Core terms of
use, available at https://www.cambridge.org/core/terms. https://doi.org/10.1017/9781108566100.005
Chapter 2a: Anatomy Questions 109

d. The vena cava passes through the vena caval foramen at level T10
e. With the vena cava, the right phrenic nerve also passes from thorax to abdomen

Question 44
The following question relates to the blood supply and lymphatic drainage of the diaphragm.
This is conveniently divided into supply to the superior and inferior surfaces of the diaphragm.
a. The arterial supply to the superior and inferior surfaces of the diaphragm is from the
respective phrenic arteries, via the aorta
b. The venous drainage of the superior and inferior surfaces of the diaphragm is via the
respective phrenic veins, directly to the IVC
c. The lymphatic drainage of the superior surface is to the diaphragmatic lymph nodes
d. The lymphatic drainage of the inferior surface is to the superior lumbar lymph nodes
e. There is no communication between the lymph plexuses between the superior and
inferior surfaces of the diaphragm

Question 45
The following question relates to the innervation of the diaphragm.
a. The entire motor supply of the diaphragm is from the phrenic nerve
b. The sensory supply is from the phrenic nerve centrally and the intercostal nerves
peripherally
c. Both the sensory and motor supplies from the phrenic nerve arise from the ventral rami
of C2–C5
d. Commonly, following intra-abdominal surgery, pain is felt in the shoulder region, in the
dermatome C5, due to referred pain from the phrenic nerve
e. Intra-abdominal irritation peripherally on the diaphragm also refers pain to the
shoulder due to the phrenic nerve

Question 46
The following question relates to the structure of the right atrium.
a. It forms the base of the heart, as seen on the chest X-ray
b. It receives the inferior and superior vena cavae and the coronary sinus
c. The right atrium has a more muscular wall, when compared to the left atrium
d. There are two parts to the internal wall of the atrium, divided by the crista terminalis
e. The tricuspid valve is between the right atrium and right ventricle

Question 47
The following question is concerned with the structure of the left atrium.
a. It forms the left and superior borders of the heart as viewed on chest X-ray
b. It drains oxygenated blood via the pulmonary artery
c. Is divided from the left ventricle by the semi-lunar mitral valve
d. The cusps of the mitral valve are attached to the internal wall via the papillary muscle
e. The left atrium is a common site for the formation of thrombus following myocardial
infarction

Downloaded from https://www.cambridge.org/core. University of Edinburgh, on 19 Aug 2019 at 13:21:27, subject to the Cambridge Core terms of
use, available at https://www.cambridge.org/core/terms. https://doi.org/10.1017/9781108566100.005
110 Chapter 2a: Anatomy Questions

Question 48
The following question relates to the structure of the right ventricle.
a. It forms the anterior surface of the heart
b. It pumps blood to the pulmonary arteries via the pulmonary valve
c. The pulmonary valve is formed by three semi-lunar cusps
d. The interventricular septum is formed as a muscular extension of the wall of the right
ventricle
e. The pulmonary valve is not affected by disease

Question 49
The following question relates to the structure of the left ventricle.
a. It forms the entirety of the apex of the heart
b. It pumps blood to the systemic circulation via the aortic valve
c. The aortic valve has the same structure as the pulmonary valve
d. The ascending aorta arises from its anterior part before arching posteriorly
e. At each cusp, there is a sinus, which opens to a coronary artery

Question 50
The following question is regarding the surface anatomy of the heart.
a. The contraction of the apex can be felt in the mid-clavicular line
b. The left border of the heart runs from the second left costal cartilage (para-sternally) to
the fifth left intercostal space, mid-clavicular line
c. The superior border runs from the second costal cartilage on the right to the second
costal cartilage on the left
d. The inferior border runs from the fifth costal cartilage on the right, to the fifth intercostal
space, mid-clavicular line on the left
e. The right border runs from the third to sixth costal cartilages on the right

Question 51
The following question concerns the anatomical location and the positions for auscultation
of the valves of the heart.
a. The mitral valve can be auscultated over its anatomical position, posterior to the fourth
left costal cartilage
b. The aortic valve is usually auscultated over its anatomical position, posterior to the third
left intercostal space.
c. In aortic stenosis, a referred pulse can be auscultated over the left carotid artery
d. The tricuspid valve can be auscultated over its anatomical position
e. The pulmonary valve can be auscultated at the level of the fifth left costal cartilage,
despite its anatomical position being at the level of the third left costal cartilage

Downloaded from https://www.cambridge.org/core. University of Edinburgh, on 19 Aug 2019 at 13:21:27, subject to the Cambridge Core terms of
use, available at https://www.cambridge.org/core/terms. https://doi.org/10.1017/9781108566100.005
Chapter 2a: Anatomy Questions 111

Question 52
The following question concerns the origin and propagation of the electrical impulses
generated in the heart to produce a mechanical effect.
a. The sinoatrial (SA) node is at the left atrium in the heart
b. The initial electrical impulse is generated in the SA node
c. The atrioventricular (AV) node is at the interatrial septum
d. A function of the AV node is to transmit the signal from the SA node to the ventricles
with a delay
e. From the AV node the impulse travels via the left and right bundles of His to the left and
right ventricles, respectively

Question 53
The following question relates the ECG to cardiac electrical activity.
a. The P-wave represents atrial contraction
b. The QRS complex represents ventricular contraction
c. The ST segment represents depolarization of the atria
d. The T-wave represents depolarization of the ventricles
e. The ST segment is isoelectric

Question 54
The following question concerns the arterial supply of the heart.
a. The right and left coronary arteries arise from the right and left coronary sinuses,
respectively
b. The coronary arteries do not supply the pericardium
c. The right coronary artery arises from the ascending aorta proximal to the aortic valve,
whereas the left coronary artery arises from the ascending aorta, distal to the aortic valve
d. The right coronary artery runs in the atrioventricular groove
e. The left coronary artery runs in the coronary groove

Question 55
The following question concerns the arterial supply to the heart.
a. The right coronary artery supplies the SA node in approximately 20% of cases
b. The right coronary artery supplies the marginal artery, which supplies the apex
c. The posterior interventricular artery anastomoses with the anterior interventicular and
circumflex arteries, arising from the left coronary artery
d. The left coronary artery supplies the left atrium via the marginal artery
e. The left coronary artery supplies the left ventricle via the anterior interventricular,
circumflex and marginal arteries

Question 56
The following question concerns the venous drainage of the heart.
a. The heart has two venous systems
b. The coronary sinus drains into the left atrium

Downloaded from https://www.cambridge.org/core. University of Edinburgh, on 19 Aug 2019 at 13:21:27, subject to the Cambridge Core terms of
use, available at https://www.cambridge.org/core/terms. https://doi.org/10.1017/9781108566100.005
112 Chapter 2a: Anatomy Questions

c. The coronary sinus runs in the posterior aspect of the atrioventricular groove
d. The great cardiac veins drain into the coronary sinus
e. The middle and small cardiac veins drain into the anterior cardiac vein

Question 57
The following question concerns the structure and relations of the pericardium.
a. There are two layers to the serous pericardium
b. The fibrous pericardium is fused to the diaphragm
c. The serous pericardium is composed of the parietal and visceral layers
d. The visceral layer of the serous pericardium is also known as the epicardium, the outer
layer of the heart
e. The pericardial space is between the fibrous and serous pericardium

Question 58
The following question concerns the blood and nerve supply to the pericardium.
a. The main arterial supply comes via the internal thoracic artery
b. The main venous drainage is via the internal thoracic veins and azygos venous system
c. The vagus nerve supplies the pericardium
d. The parasympathetic trunk supplies the pericardium
e. The phrenic nerve (T1–T4) supplies the pericardium

Question 59
Regarding the kidney and ureter:
a. The kidney lies in the retroperitoneum, surrounded by Gerota’s fascia
b. The ureter runs along the anterior surface of the paraspinal muscles in line with the
transverse spinous processes
c. In the renal pelvis, the renal artery is the most anterior structure
d. The blood supply to the ureter is via the blood supply from the bladder
e. A transplanted kidney receives its blood supply from the native renal artery and vein

Question 60
Regarding the kidney and ureter:
a. The renal vessels are normally found at level L1/L2
b. Inferior to both kidneys is the adrenal gland
c. Each adrenal gland is enclosed within Gerota’s fascia
d. Urine produced in the kidney drains from the minor calyces into the major calyces and
then into the renal pelvis
e. The right adrenal gland is more triangular, compared to the semilunar left gland

Question 61
With respect to the liver and gall bladder:
a. 70% of the blood supply to the liver is via the hepatic artery from the coeliac trunk
b. The portal vein drains into the inferior vena cava

Downloaded from https://www.cambridge.org/core. University of Edinburgh, on 19 Aug 2019 at 13:21:27, subject to the Cambridge Core terms of
use, available at https://www.cambridge.org/core/terms. https://doi.org/10.1017/9781108566100.005
Chapter 2a: Anatomy Questions 113

c. Calot’s triangle is formed by the lower edge of the liver, the common hepatic duct and
the cystic duct
d. The following can be found at the porta hepatis: portal vein, hepatic artery, common
hepatic duct, lymph nodes and autonomic nerve fibres
e. The portal triad is found at the lateral edge of the greater omentum

Question 62
With respect to the lobes of the liver and the peritoneal reflections in relation to the liver:
a. The two lobes of the liver are functionally independent of each other
b. The left lobe also contains the caudate and quadrate lobes
c. The left lobe is divided from the caudate and quadrate lobes by the ligamentum teres and
ligamentum venosum
d. The lesser omentum forms the hepatogastric and hepatosplenic ligaments
e. The free edge of the lesser omentum encloses the portal triad (hepatic artery, hepatic
vein, bile duct) and lymph vessels, lymph nodes and the hepatic plexus

Question 63
With respect to the pancreas:
a. The head of the pancreas is in close relation to the lateral aspect of the fifth part of the
duodenum
b. The blood supply to the head of the pancreas is via an arcade formed by the gastro-
duodenal and superior mesenteric arteries
c. The blood supply to the tail of the pancreas is via the splenic artery
d. The inferior and superior mesenteric veins converge posterior to the head of the
pancreas to form the portal vein
e. The pancreatic duct inserts into the third part of the duodenum

Question 64
The following question concerns the clinical aspects of pancreatic morbidity.
a. Pancreatic cancer often metastasizes to the liver via haematological means, due to the
splenic vein forming the portal vein with the superior mesenteric vein
b. Head of pancreas tumours can cause jaundice by obstructing the hepatic duct, leading to
retention of bile salts
c. Pancreatic injury can occur with acceleration/deceleration injuries, leading to complete
transection of the pancreatic head from the tail
d. Transection of the pancreas often leads to hyperinsulinaemia and a hypoglycaemic state
e. Transection of the pancreas can also lead to release of the exocrine enzymes and
autodigestion

Question 65
With regards to the arterial blood supply to the gastrointestinal tract:
a. The coeliac artery supplies the foregut
b. The superior mesenteric artery supplies the mid-gut
c. The superior mesenteric artery supplies the colon up to the hepatic flexure

Downloaded from https://www.cambridge.org/core. University of Edinburgh, on 19 Aug 2019 at 13:21:27, subject to the Cambridge Core terms of
use, available at https://www.cambridge.org/core/terms. https://doi.org/10.1017/9781108566100.005
114 Chapter 2a: Anatomy Questions

d. The inferior mesenteric artery supplies the upper rectum


e. The internal iliac artery supplies the lower rectum

Question 66
With regards to the structure of the jejunum and ileum:
a. The small bowel mesentery is attached to the posterior abdominal wall from the left side
of L2 to the right sacroiliac joint
b. The blood supply is entirely from the superior mesenteric artery
c. The entire small bowel drains into the superior mesenteric vein
d. A one-way valve between the jejunum and ileum identifies the division between the two
e. The small bowel is split equally between the jejunum and ileum

Question 67
The following question is regarding the muscles and fascia of the anterior abdominal wall
and their nerve supply.
a. The abdomen is covered by a superficial and a deep fascia
b. The three flat muscles (external and internal oblique, and transversus abdominis) form
the rectus sheath anteriorly
c. The rectus sheath contains the rectus abdominis, pyramidalis, superior and inferior
epigastric vessels, lymphatics and the ventral rami of T7–T12
d. The nerve supply of the anterior abdominal wall arises from the ventral rami of T7–L1
e. The cutaneous nerves run in a plane formed by the internal and external oblique muscles

Question 68
Which of the following questions are true?
a. The anterior divisions of T7–T11 form the intercostal nerves, which run within the
transversus abdominis plane
b. The anterior branch of T12 joins to form the iliohypogastric nerve
c. The L1 nerve root contributes to the formation of the ilioinguinal nerve
d. The triangle of Petit is formed by the borders of the lattismus dorsi, internal oblique and
anterior superior iliac spine (ASIS)
e. A right-sided TAP block can be used to reliably provide analgesia for an open
cholecystectomy

Question 69
The following question is regarding the surface anatomy of the abdomen.
a. The subcostal plane, at level L1, forms an imaginary line that joins the inferior borders of
the tenth costal cartilages and is at the level of the pylorus
b. The transtubercular plane, at level L5, forms an imaginary line which joins the tubercles
of the iliac crest
c. The median plane divides the body into right and left
d. The rectus abdominis lies between the linea alba and the linea semilunaris bilaterally
e. The superficial inguinal ring is situated approximately 1 cm inferolaterally to the pubic
tubercle

Downloaded from https://www.cambridge.org/core. University of Edinburgh, on 19 Aug 2019 at 13:21:27, subject to the Cambridge Core terms of
use, available at https://www.cambridge.org/core/terms. https://doi.org/10.1017/9781108566100.005
Chapter 2a: Anatomy Questions 115

Question 70
The following question is regarding the inguinal canal.
a. It runs parallel to and along the lateral half of the inguinal ligament
b. It contains the ilioinguinal nerve
c. The deep inguinal ring is found at the mid-inguinal point
d. The medial insertion of the inguinal ligament is the pubic symphisis
e. A direct inguinal hernia protrudes medial to the inferior epigastric artery

Question 71
The following question lists common abdominal pathology and sites where the pain may be
sensed.
a. A splenic bleed may refer pain to the shoulder
b. Appendicitis may refer pain to the umbilicus
c. Abdominal distension following laparoscopic surgery may refer pain to the shoulder
d. Renal colic may be referred to the scrotum
e. Rectal pain may be referred to the lower back

Question 72
Performing a coeliac plexus block involves:
a. Needle is inserted between the third and fourth lumbar vertebrae
b. The needle is positioned 5–7 cm away from the mid-line
c. Advancing the needle towards T12 causes blockade of the splanchnic nerves
d. The transabdominal approach is performed under ultrasound guidance
e. Orthostatic hypotension is a recognized complication

Question 73
Recognized complications of a coeliac plexus block include:
a. Hypertension
b. Diarrhoea
c. Impotence
d. Pleurisy
e. Paraplegia

Question 74
The autonomic nervous system comprises the sympathetic and parasympathetic pathways,
which usually have opposing effects on the organs they supply. With regards to the
sympathetic nervous system:
a. Preganglionic fibres arise from segments T1–L5 in the posterior roots of the spinal
nerves
b. Acetylcholine is the neurotransmitter released at the pre- and postganglionic neurones
c. The paired sympathetic trunks comprise ganglia from which myelinated nerve fibres
pass to somatic and visceral structures
d. The stellate ganglion lies on the first rib and gives rise to C7, C8 and T1 spinal nerves

Downloaded from https://www.cambridge.org/core. University of Edinburgh, on 19 Aug 2019 at 13:21:27, subject to the Cambridge Core terms of
use, available at https://www.cambridge.org/core/terms. https://doi.org/10.1017/9781108566100.005
116 Chapter 2a: Anatomy Questions

e. The coeliac plexus receives branches of nerves that originated in the thoracic and lumbar
sympathetic trunks

Question 75
With regards to the parasympathetic nervous system:
a. It comprises a cranial outflow that includes cranial nerves III, VII, IX and X, and a sacral
outflow from S2–S4
b. Preganglionic nerves are short and myelinated compared to the long, non-myelinated
fibres of the postganglionic neurones
c. Acetylcholine is released at the pre- and postganglionic synapses
d. The vagus nerve supplies parasympathetic innervation to the thorax and abdomen via
the inferior and superior vagal trunks
e. The sacral outflow supplies nerves that aid in micturition by contracting bladder
sphincters

Question 76
There are 12 pairs of cranial nerves. Knowledge of their anatomy and innervation can aid in
the diagnosis of intracranial pathologies.
a. The olfactory nerve reaches the olfactory bulb after passing through the cribiform plate
in the mastoid
b. The optic nerve decussates in the optic chiasm, which is located in the anterior cranial
fossa
c. A lesion posterior to the optic chiasm will lead to a contralateral homonymous
hemianopia
d. The oculomotor nerve contains parasympathetic fibres that supply the ciliary muscles
and sphincter pupillae
e. Raised intracranial pressure causes compression of the oculomotor nerve against the
tentorium cerebrum, leading to signs of palsy

Question 77
With regards to cranial nerves:
a. The trochlear nerve runs lateral to the oculomotor nerve in the posterior cranial fossa
b. The abducens nerve originates in the midbrain
c. The motor component of the trigeminal nerve arises from the trigeminal ganglion to
supply the muscles of mastication
d. The mandibular division of the trigeminal nerve carries the motor component, but does
not have a sensory innervation
e. Herpes zoster is known to affect the trigeminal nerve, although involvement is often
isolated to one division of the nerve

Question 78
With regards to cranial nerves:
a. After leaving the skull at the stylomastoid foramen, the facial nerve divides into five
branches: temporal, zygomatic, maxillary, buccal and cervical

Downloaded from https://www.cambridge.org/core. University of Edinburgh, on 19 Aug 2019 at 13:21:27, subject to the Cambridge Core terms of
use, available at https://www.cambridge.org/core/terms. https://doi.org/10.1017/9781108566100.005
Chapter 2a: Anatomy Questions 117

b. Within the skull, the greater petrosal nerve supplies parasympathetic innervation to the
lacrimal gland
c. The chorda tympani pass above the stylomastoid foramen and emerge from the base of
the temporal bone to supply the tongue
d. The two components of cranial nerve VIII, the vestibular nerve and the cochlear nerve,
anastamose at the internal auditory meatus
e. The glossopharyngeal nerve supplies sensory fibres to the posterior third of the tongue

Question 79
With regards to the cranial nerves:
a. The vagus nerve leaves the skull via the foramen magnum
b. The vagus nerve is enclosed within the carotid sheath and lies lateral to the internal
jugular vein and internal carotid artery
c. In the thorax, the vagus nerve joins with its contralateral counterpart to form the
anterior and posterior vagal trunks
d. The recurrent laryngeal nerves are branches of the vagus nerve
e. The accessory nerve supplies motor fibres to the sternocleidomastoid and trapezius
muscles

Question 80
Branches of the facial nerve, CN VII, include:
a. Ophthalmic
b. Mandibular
c. Maxillary
d. Zygomatic
e. Buccal

Question 81
Parasympathetic nerve fibres are present in the following cranial nerves:
a. Oculomotor nerve
b. Trochlear nerve
c. Abducens nerve
d. Facial nerve
e. Vagus nerve

Question 82
Knowledge of the anatomy of the epidural space is paramount in performing safe epidural
blockade. Variation in anatomy can explain the difficulty observed in some patients.
a. Structures contained within the epidural space include the extradural venous plexus,
spinal arteries, spinal nerves and the dural sac
b. The epidural veins are valveless and form a continuous network with the intracranial
veins above and pelvic veins below
c. The anterior and posterior longitudinal ligaments extend from the occiput to the sacrum
in connecting the vertebral bodies

Downloaded from https://www.cambridge.org/core. University of Edinburgh, on 19 Aug 2019 at 13:21:27, subject to the Cambridge Core terms of
use, available at https://www.cambridge.org/core/terms. https://doi.org/10.1017/9781108566100.005
118 Chapter 2a: Anatomy Questions

d. The interspinous ligament connects the vertebral laminae, whereas the supraspinous
ligament connects the spines of the vertebrae
e. Folds within the dura mater in the epidural space can result in patchy or unilateral
analgesia following an epidural injection

Question 83
Caudal epidurals are often inserted for analgesia in children, as well as in adults with chronic
back pain. The site of a caudal injection lies in the sacral hiatus.
a. The sacral bone is formed following the fusion of S1–S5
b. The sacral canal is approximately 5–7 cm and contains the dural sac, venous blood
supply and sacral nerves
c. The sacral hiatus is bordered on either side by the sacral cornua, which is continuous
with the fourth sacral spine.
d. The sacral hiatus contains the S5 nerve and the coccygeal nerves, which are overlaid with
the sacrococcygeal membrane
e. The venous plexus lies more posteriorly than anteriorly therefore it may be likely to get
a bloody tap

Question 84
The lumbar plexus gives off nerves supplying the lower limb. Many of these nerves can be
blocked to provide regional anaesthesia.
a. The plexus is formed by the anterior rami from the L1–L4 nerves
b. The nerves run behind the psoas muscle, anterior to the transverse processes
c. The largest nerve is the femoral nerve and originates from L1–L3
d. The obturator nerve supplies the abductor muscles of the thigh
e. The lateral cutaneous nerve enters the thigh lateral to the anterior superior iliac spine

Question 85
The sacral plexus contains branches supplying the hip, pelvis and lower limb.
a. The sacral plexus contains branches from the lumbosacral trunk and S1–S4
b. The sciatic nerve originates from L4–S3 and lies between the ischial tuberosity and the
greater trochanter
c. The sciatic nerve travels in the posterior aspect of the leg and divides into the tibial and
common peroneal nerve at the popliteal fossa
d. The sacral plexus supplies pelvic splanchnic nerves, which innervate the external anal
and vesical sphincters
e. The pudendal nerve carries parasympathetic fibres to the penis/clitoris

Question 86
The skull is made up of a number of bones housing multiple foramina through which nerves
and blood vessels enter and exit the brain.
a. The bregma is the junction at which the coronal suture meets the sagittal suture
b. The frontal bone contains the supraorbital and infraorbital foramina
c. The zygomatic arch is formed by the zygomatic and temporal bones

Downloaded from https://www.cambridge.org/core. University of Edinburgh, on 19 Aug 2019 at 13:21:27, subject to the Cambridge Core terms of
use, available at https://www.cambridge.org/core/terms. https://doi.org/10.1017/9781108566100.005
Chapter 2a: Anatomy Questions 119

d. The external acoustic meatus lies underneath the anterior zygomatic arch
e. Within the temporal fossa lies the pterion, which houses the middle meningeal artery

Question 87
Regarding the base of skull:
a. The mandibular nerve travels through the foramen ovale
b. The middle meningeal artery exits the brain through the foramen spinosum
c. Between the occipital and sphenoid bones lies the petrous bone, which houses the
carotid canal
d. The internal jugular vein enters through the jugular foramen and is accompanied by the
glossopharyngeal and accessory nerves
e. The stylomastoid foramen lies posteriorly to the mastoid process and allows passage of
the facial nerve

Question 88
The brain is subdivided into three main sections – the forebrain, midbrain and hindbrain.
a. The cerebrum, which is made up of two cerebral hemispheres, contains four distinct
lobes
b. The central sulcus divides the sensory and motor areas – the precentral gyrus is the
motor component and the postcentral gyrus is sensory
c. The auditory area is located within the temporal lobe, which lies superiorly to the lateral
sulcus
d. The motor speech area is located within the temporal lobe and lies inferiorly to the
lateral sulcus
e. The occipital lobe contains the visual area, which lies in the parieto-occipital sulcus

Question 89
Regarding the brain:
a. The thalamus is located within the diencephalon on either side of the third ventricle
b. The cerebral peduncles in the midbrain are subdivided superiorly into the crus cerebri
and inferiorly into the tegmentum
c. The medulla oblongata is situated in the brainstem
d. In each half of the medulla, there are two pyramids that receive the majority of the
ascending motor fibres
e. The cerebellum lies below the tentorium cerebelli behind the pons and medulla

Question 90
Regarding the brain:
a. The three cerebellar peduncles connect the cerebellum to the midbrain, pons and
medulla
b. Cerebrospinal fluid (CSF) is secreted by the choroid plexus at a rate of 150 ml.h–1
c. The two lateral ventricles carry CSF to the third ventricle via the intraventricular
foramen – also known as the foramen of Magendie

Downloaded from https://www.cambridge.org/core. University of Edinburgh, on 19 Aug 2019 at 13:21:27, subject to the Cambridge Core terms of
use, available at https://www.cambridge.org/core/terms. https://doi.org/10.1017/9781108566100.005
120 Chapter 2a: Anatomy Questions

d. The cerebral aqueduct that runs in the foramen of Luschka connects the third and fourth
ventricles
e. CSF is reabsorbed by the arachnoid villi

Question 91
The arterial blood supply to the brain is crucial and requires a complex network of blood
vessels.
a. The circle of Willis is an anastomosis of two internal carotid arteries and two basilar
arteries
b. The internal carotid artery enters the brain and bifurcates to form the anterior and
middle cerebral arteries
c. The posterior communicating artery connects the internal carotid artery with the
posterior cerebral artery
d. An occlusion in the anterior cerebral artery will compromise motor function in the legs
e. The middle cerebral artery provides the blood supply to all of the lateral areas of the
cerebral cortex

Question 92
Regarding the arterial blood supply to the brain:
a. The two vertebral arteries anastomose at the level of the pons to form the one basilar
artery
b. The vertebral arteries supply branches to form the meningeal, spinal and medullary
arteries
c. The basilar artery gives branches to the cerebellum via the anterior and posterior
cerebellar arteries
d. Venous drainage of the brain occurs via the venous sinuses located in the arachnoid
mater
e. Any pathology within the arachnoid mater can lead to hydrocephalus due to the
blockage in the reabsorption of CSF

Question 93
With regards to the spinal cord:
a. The grey matter surrounding the central canal is named such due to the presence of
nerve cell bodies
b. Injury to the posterior white column will result in impaired proprioception and fine
touch sense on the contralateral side
c. Testing a spinal block with ethyl chloride spray is used as an alternative to testing pain
sensation, both of which are transmitted in the lateral spinothalamic tract
d. The pyramidal tracts decussate in the medulla before descending down the spinal cord
and exiting at segmental levels
e. Paraplegia secondary to spinal cord injury is a contraindication to the use of suxa-
methonium in the first 24 hours following injury

Downloaded from https://www.cambridge.org/core. University of Edinburgh, on 19 Aug 2019 at 13:21:27, subject to the Cambridge Core terms of
use, available at https://www.cambridge.org/core/terms. https://doi.org/10.1017/9781108566100.005
Chapter 2a: Anatomy Questions 121

Question 94
With regards to the spinal blood supply and the spinal meninges:
a. The artery radicularis magna contributes to the anterior spinal artery in supplying the
upper third of the spinal cord
b. Venous drainage of the spinal cord is via the anterior and superior spinal veins, which
drain into the azygos vein
c. The dura mater is the outermost layer of meninges and extends from the foramen
magnum cranially to L1–L2 caudally
d. The arachnoid villi are involved in the secretion and reabsorption of CSF within the
brain
e. Fusion of the layers of the pia mater within the ventricles forms the choroid plexus

Question 95
With regards to spinal nerves:
a. There are 31 pairs of spinal nerves – 8 cervical, 12 thoracic, 5 lumbar, 5 sacral and 1
coccygeal
b. The posterior root of the spinal nerve makes up the sensory component and the anterior
root is the motor component
c. The anterior nerve root contains the sympathetic nerve fibres
d. The cauda equina is made up of lumbar, sacral and coccygeal nerves
e. There is no meningeal covering over the spinal nerves

Question 96
Complications associated with a high spinal block include:
a. Hypotension
b. Upper limb paralysis
c. Tachycardia
d. Tachypnoea
e. Circumoral tingling

Question 97
With regards to the brachial plexus:
a. It is composed in the order of roots, trunks, cords, divisions and nerves
b. The roots originate from the ventral rami of C6–T1
c. The trunks are composed of superior, inferior and lateral components
d. The cords of the plexus are named by their relationship to the subclavian artery
e. The anterior divisions supply the flexor muscles and the posterior divisions supply the
extensor muscles of the forearm

Question 98
With regards to the branches of the brachial plexus:
a. The median nerve is supplied by the lateral and medial cords
b. The ulnar nerve receives fibres primarily from C8–T1

Downloaded from https://www.cambridge.org/core. University of Edinburgh, on 19 Aug 2019 at 13:21:27, subject to the Cambridge Core terms of
use, available at https://www.cambridge.org/core/terms. https://doi.org/10.1017/9781108566100.005
122 Chapter 2a: Anatomy Questions

c. The radial nerve receives fibres from the anterior division of roots C5–T1
d. The long thoracic nerve receives fibres directly from the dorsal roots of C5–C7
e. The musculocutaneous nerve is supplied by the lateral cord

Question 99
The following describes the course of the nerves leaving the brachial plexus.
a. The musculocutaneous nerve descends between biceps brachii and brachialis
b. The axillary nerve accompanies the posterior circumflex humeral artery
b. The ulnar nerve runs posterior to the medial epicondyle
d. The radial nerve traverses between the long and medial heads of the triceps
e. The median nerve runs lateral to the axillary artery

Question 100
Regarding the distribution of the nerves in the arm:
a. The musculocutaneous nerve carries motor fibres in the upper arm and forearm
b. The axillary nerve supplies the deltoid, shoulder joint, teres minor and overlying skin
c. The ulnar nerve supplies the flexor carpi ulnaris and flexor digitorum profundus
d. The radial nerve supplies the flexors of the upper arm and forearm
e. The median nerve supplies all the flexor muscles of the forearm

Question 101
Regarding reflexes of the upper limb:
a. Upper limb reflexes are spinal reflexes
b. They are mediated by a monosynaptic reflex arc
c. The biceps reflex is supplied by C5–C6
d. The brachioradialis reflex is supplied by C6–C7
e. The triceps reflex is supplied by C7–C8

Question 102
Regarding the dermatomes of the arm:
a. The anterior aspect of the arm is supplied by dermatomes C4–T2
b. C4 supplies the medial aspect of the deltoid
c. The middle finger is innervated by C8
d. The thumb is innervated by C6
e. The little finger is innervated by T1

Question 103
The cutaneous innervation to the upper limb is supplied by the following nerves:
a. The radial nerve supplies the medial aspect of the thumb
b. The median nerve supplies the tip of the little finger
c. The ulnar nerve supplies the tip of the index finger
d. The axillary nerve supplies the lateral aspect of the upper arm
e. The intercostobrachial nerve supplies the axilla

Downloaded from https://www.cambridge.org/core. University of Edinburgh, on 19 Aug 2019 at 13:21:27, subject to the Cambridge Core terms of
use, available at https://www.cambridge.org/core/terms. https://doi.org/10.1017/9781108566100.005
Chapter 2a: Anatomy Questions 123

Question 104
Regarding the arterial supply to the upper limb:
a. The axillary artery originates from the subclavian artery at the medial border of the
first rib
b. The axillary artery is divided into three as it passes under the pectoralis major
c. The brachial artery begins at the inferior border of the teres major
d. The brachial artery divides into the radial and ulnar arteries deep to the bicipital
aponeurosis
e. The main component of the superficial palmar arch is the radial artery

Question 105
The arterial supply of the upper limb is divided into the following components:
a. The superior thoracic artery is the first branch of the axillary artery
b. The posterior and anterior circumflex humeral arteries form an anastomosis around the
head of the humerus
c. The brachial artery accompanies the median nerve in the forearm
d. The deep brachial artery and the superior and inferior ulnar collateral arteries form an
anastomosis around the elbow joint
e. In the forearm, the ulnar artery runs medial to the ulnar nerve

Question 106
The venous drainage of the upper limb is divided into the deep and superficial systems:
a. The axillary vein lies medial to the axillary artery
b. The axillary vein is formed by the brachial vein at the inferior border of the teres minor
c. The venae comitantes are the brachial veins draining into the axillary vein
d. The basilic vein originates at the wrist
e. The median cubital vein connects the cephalic vein to the basilic vein

Question 107
The branches of the venous system of the upper limb are as follows:
a. The cephalic vein originates at the wrist
b. The cephalic vein drains into the basilic vein in the upper arm
c. The dorsal venous arches in the hand form the cephalic and brachial veins
d. The brachial vein lies superficial to the brachial artery at the antecubital fossa
e. The cephalic vein runs along the lateral aspect of the upper arm

Question 108
Regarding lymphatic drainage of the arm:
a. Lymphatic drainage in the arm follows the arterial supply
b. There are five groups of axillary lymph nodes
c. Following axillary node clearance, patients may develop primary lymphadenopathy in
the associated arm

Downloaded from https://www.cambridge.org/core. University of Edinburgh, on 19 Aug 2019 at 13:21:27, subject to the Cambridge Core terms of
use, available at https://www.cambridge.org/core/terms. https://doi.org/10.1017/9781108566100.005
124 Chapter 2a: Anatomy Questions

d. Unilateral axillary lymphadenopathy may be caused by tuberculosis


e. Lymphoma usually presents with unilateral axillary lymphadenopathy

Question 109
The following question concerns the borders and structures in the axilla.
a. The axillary artery, vein, lymphatics and parts of the brachial plexus pass through the
axilla as they travel between the arm and the neck
b. The base of the axilla is formed by the thoraco-axial ligament
c. The axillary nerve, artery and vein travel through the axilla as a neurovascular bundle
d. The medial wall of the axilla is partly composed of serratus anterior muscle
e. The anterior wall of the axilla is formed by the pectoralis major and minor

Question 110
Nerve injuries of the brachial plexus and upper limb can manifest in several ways:
a. Injury to the deep part of the brachial plexus can lead to the characteristic waiter’s tip
position
b. Ulnar nerve injuries characteristically cause a ‘claw hand’ deformity
c. A proximal ulnar nerve injury leads to a greater deformity in the hand
d. Radial nerve injuries lead to wrist drop with extension of the digits at the metacarpo-
phalangeal joints
e. Motor dysfunction is seen prior to sensory dysfunction in carpal tunnel syndrome

Question 111
Regarding the muscles of the upper arm:
a. The proximal insertion of the short and long heads of biceps brachii is at the scapula
b. The distal insertion of the biceps brachii is the tuberosity of the ulna
c. Biceps brachii supinates the forearm
d. The proximal insertion of the long, lateral and medial heads of the triceps brachii is at
the humerus
e. The distal insertion of the triceps brachii is the olecranon of the ulna

Question 112
Regarding small muscles of the hand and forerarm:
a. The flexor muscles of the forearm are divided into three groups
b. The intrinsic muscles of the hand are divided into three groups
c. The thenar muscles are responsible for opposition of the thumb
d. The lumbricals flex and extend the digits at the metacarpophalangeal joints
e. The dorsal and palmar interossei abduct the digits

Question 113
The following structures are contained within the antecubital fossa:
a. Median nerve
b. Brachial artery

Downloaded from https://www.cambridge.org/core. University of Edinburgh, on 19 Aug 2019 at 13:21:27, subject to the Cambridge Core terms of
use, available at https://www.cambridge.org/core/terms. https://doi.org/10.1017/9781108566100.005
Chapter 2a: Anatomy Questions 125

c. Basilic vein
d. Biceps tendon
e. Ulnar nerve

Question 114
With regards to a supraclavicular block:
a. The block is achieved at the level of the cords
b. It can reliably be used for surgery of the lower forearm
c. The intercostobrachial nerve is not part of the brachial plexus
d. The subclavian artery lies posteriorly to the brachial plexus
e. The incidence of pneumothorax is 15–20%

Question 115
With regards to an infraclavicular block:
a. It provides anaesthesia for upper arm surgery
b. It can be used as anaesthesia for embolectomy of the brachial artery
c. Continuous anaesthesia can be provided by inserting a catheter
d. It carries a 10% risk of pneumothorax
e. It carries a 10% risk of phrenic nerve palsy

Question 116
With regards to an interscalene block:
a. It is appropriate for proximal arm surgery
b. It may not adequately anaesthetize the C8/T1 nerve roots
c. The block needle is placed in the interscalene groove between the anterior and posterior
scalenus muscles
d. A deltoid muscle motor response suggests adequate placement of the needle
e. Horner’s syndrome is the most frequently occurring complication

Question 117
With regards to an axillary nerve block:
a. It provides reliable anaesthesia to the forearm, wrist and hand
b. The needle is inserted above the axillary artery
c. The musculocutaneous nerve is commonly missed at first injection
d. Tourniquet pain is felt as a consequence of missing blocking of the intercostobrachial
nerve
e. The main complication is haematoma formation due to puncture of the artery

Question 118
Injury to the median nerve above the elbow can present with the following signs:
a. Inability to pronate the forearm
b. Inability to perform flexion at the wrist
c. Loss of thumb adduction

Downloaded from https://www.cambridge.org/core. University of Edinburgh, on 19 Aug 2019 at 13:21:27, subject to the Cambridge Core terms of
use, available at https://www.cambridge.org/core/terms. https://doi.org/10.1017/9781108566100.005
126 Chapter 2a: Anatomy Questions

d. Loss of sensation over the thenar eminence


e. Paraesthesia over the dorsum of the hand

Question 119
Injury to the radial nerve can present with the following signs:
a. Claw hand
b. Wrist drop
c. Trigger finger
d. Numbness over the dorsum of the hand
e. Paraesthesia at the lateral aspect of the hand

Question 120
Injury to the ulnar nerve at the wrist can present with the following signs:
a. Claw hand
b. Hyperextension at the wrist
c. Loss of sensation over the dorsum of the hand
d. Trigger finger
e. Paraesthesia over the palmar surface of the hand

Question 121
The lower limb fascia is composed of superficial and deep layers.
a. The superficial fascia is continuous with the inguinal ligament
b. The superficial fascia consists mainly of loose connective tissue and fat
c. The deep fascia is consistent with Scarpa’s fascia in the abdomen
d. The fascia lata is continuous with the crural fascia
e. There are normally no defects in the deep fascia

Question 122
The lower limb fascia divides the lower limb into various compartments, which form
functional muscular groups.
a. The thigh is divided into three groups, anterior, lateral and posterior
b. The fascia lata forms the intermuscular septa and attaches onto the gluteal tuberosity of
the femur
c. The lower leg is divided into three compartments: anterior, posterior and lateral
d. The posterior compartment of the leg is further divided into deep and superficial
e. The crural fascia attaches to the tibia to form the intermuscular septa

Question 123
The following question concerns the various muscles groups in the thigh.
a. The anterior thigh contains the iliopsoas, tensor fascia latae, pectineus, sartorius and the
quadriceps femoris
b. The lateral thigh contains the abductor longus, abductor brevis, abductor magnus,
gracilis and obturator externus

Downloaded from https://www.cambridge.org/core. University of Edinburgh, on 19 Aug 2019 at 13:21:27, subject to the Cambridge Core terms of
use, available at https://www.cambridge.org/core/terms. https://doi.org/10.1017/9781108566100.005
Chapter 2a: Anatomy Questions 127

c. The posterior compartment contains the semitendinosus, semimembranosus and biceps


femoris, also known as the hamstrings
d. The large gluteal muscles are composed of maximus, medius and minimus glutei
e. The deeper gluteal muscles internally rotate the thigh

Question 124
The leg is divided into three compartments, anterior, lateral and posterior. The following
question concerns the muscles in these compartments.
a. The anterior compartment contains the tibialis anterior, extensor hallucis longus,
extensor digitorum longus and fibularis
b. The muscles of the lateral compartment are responsible for eversion of the foot
c. The posterior compartment is divided into two compartments by the transverse septum
d. The superficial posterior compartment contains the gastrocnemius, soleus and plantaris,
which plantar flex the ankle and flex the knee
e. The deep compartment contains the small muscles that flex the toes

Question 125
The femoral triangle is a triangular space in the thigh.
a. The superior boundary is the inguinal ligament
b. The medial and lateral boundaries are the adductor brevis and sartorius respectively
c. The roof of the femoral triangle is formed by the fascia lata
d. The femoral sheath is a continuation of the transversalis fascia
e. The femoral sheath encloses the femoral artery, vein and nerve within the femoral
triangle

Question 126
The nerve supply to the thigh arises from the lumbar (L1–L4) and sacral (S1–S3) plexuses.
This question concerns the distribution of the nerves in the thigh.
a. The femoral nerve (L2–L4) supplies muscles in the posterior thigh and hip and knee
joints
b. The lateral femoral cutaneous nerve (L2–L3) supplies the lateral thigh
c. The posterior femoral cutaneous nerve (S2–S3) supplies the posterior thigh
d. The obturator nerve (L3–L5) generally supplies the medial thigh muscles
e. The ilioinguinal nerve (L1) supplies the skin over the femoral triangle

Question 127
The nerve supply to the leg arises from the femoral and sciatic nerves.
a. The common fibular and common peroneal nerves are the same nerve
b. The femoral nerve does not supply any muscles distal to the knee
c. The femoral nerve supplies skin over the posterolateral aspect of the leg via the sural
nerve
d. The sciatic nerve supplies all the muscles in the posterior compartment of the leg via the
tibial nerve

Downloaded from https://www.cambridge.org/core. University of Edinburgh, on 19 Aug 2019 at 13:21:27, subject to the Cambridge Core terms of
use, available at https://www.cambridge.org/core/terms. https://doi.org/10.1017/9781108566100.005
128 Chapter 2a: Anatomy Questions

e. The common peroneal nerve divides to give the superficial and deep peroneal nerves and
the lateral sural cutaneous nerve

Question 128
The following question concerns the arterial supply to the thigh.
a. The entire arterial supply of the lower limb originates from the femoral artery
b. The obturator artery supplies the adductors
c. The femoral artery supplies the anterior compartment of the thigh
d. The deep femoral artery supplies the posterior compartment of the thigh
e. The lateral circumflex femoral artery is a division of the superficial femoral artery

Question 129
The leg is supplied by the superficial femoral artery, as it becomes the popliteal artery.
a. The popliteal artery passes through the popliteal fossa and ends at the lower border of
the popliteus, by dividing into tibial and fibular arteries
b. The popliteal artery supplies the knee
c. The tibial artery divides to form the anterior and posterior tibial arteries
d. The dorsalis pedis is a continuation of the anterior tibial artery
e. The fibular artery is a continuation of the posterior tibial artery

Question 130
The lower limb is drained via superficial and deep venous systems.
a. The deep venous system is composed of the femoral and popliteal veins
b. The superficial venous system is composed of the great and lesser femoral veins
c. The two systems connect at the saphenous opening in the femoral triangle and the
popliteal fossa only
d. The great saphenous vein runs posterior to the medial malleolus
e. The lesser saphenous vein runs posterior to the lateral malleolus

Question 131
The popliteal fossa is an area, posterior to the knee, that contains the main vessels and
nerves supplying the leg and foot. It is of particular interest to anaesthetists with respect to
providing safer regional anaesthesia for ankle and foot surgery.
a. The superior boundaries of the popliteal fossa are the biceps femoris medially and the
semimembranosus and semitendinosus laterally
b. The inferior boundary of the popliteal fossa is the gastrocnemius
c. The popliteal artery travels anterior to the popliteal vein
d. The tibial nerve runs along the superiolateral border of the popliteal fossa
e. The small saphenous vein drains into the femoral vein

Downloaded from https://www.cambridge.org/core. University of Edinburgh, on 19 Aug 2019 at 13:21:27, subject to the Cambridge Core terms of
use, available at https://www.cambridge.org/core/terms. https://doi.org/10.1017/9781108566100.005
Chapter 2a: Anatomy Questions 129

Question 132
Regarding the anatomy of the ankle.
a. The ankle is formed by the tibia and the talus
b. The ankle is strengthened by fibrous ligaments. The strong medial ligaments consist of
three parts
c. The lateral ligament is better known as the deltoid ligament
d. The blood supply to the ankle is derived from the dorsalis pedis
e. The nerve supply is derived from the tibial and deep fibular nerves

Question 133
The arches of the foot allow for shock absorption and propulsion. They are formed by the
bones of the foot and maintained via the plantar ligaments and aponeurosis. This question
concerns the function of the arches and the bones of the foot which form them.
a. The foot has two arches, longitudinal and transverse
b. The longitudinal arch has two components – lateral and medial
c. The transverse arch has three components – anterior, median and posterior
d. The lateral longitudinal arch is composed of calcaneus, cuboid and metatarsals
e. The medial arch is composed of the calcaneus, navicular and metatarsals

Question 134
There are seven different joints in the foot. This question concerns the different joints and
their respective movements.
a. The subtalar joint is the articulation between the talus and the tibia
b. The subtalar joint is involved in plantar- and dorsiflexion at the ankle
c. The metatarsophalangeal joint is involved in abduction and adduction
d. There is no appreciable movement between the intermetatarsal joints
e. The interphalangeal joints are involved in abduction and adduction

Question 135
There are a large number of small muscles in the foot.
a. The muscles of the foot are divided into four muscular layers
b. The fine control to each toe that these muscles allow is important in balance
c. The muscles provide three layers through which neurovascular bundles pass to supply
the muscles and joints of the foot
d. The long muscles of the foot arise from the leg and help in flexion and extension of the
toes
e. The muscles are also divided into three longitudinal compartments by the deep fascia

Question 136
The following question concerns the arterial supply to the foot.
a. The dorsalis pedis artery is a continuation of the anterior tibial artey
b. The dorsalis pedis artery eventually divides to form the superficial plantar arch
c. The main arterial supply to the foot arises from the posterior tibial artery

Downloaded from https://www.cambridge.org/core. University of Edinburgh, on 19 Aug 2019 at 13:21:27, subject to the Cambridge Core terms of
use, available at https://www.cambridge.org/core/terms. https://doi.org/10.1017/9781108566100.005
130 Chapter 2a: Anatomy Questions

d. The posterior tibial artery divides to form the deep plantar arch
e. The dorsalis pedis and posterior tibial arteries supply the toes via the dorsal and plantar
digital arteries, respectively

Question 137
The following question concerns the nerve to the foot.
a. The tibial nerve supplies all the muscles of the foot and no cutaneous sensation
b. The common peroneal nerve supplies all the cutaneous sensation to the foot
c. The superficial fibular nerve, a branch of the common peroneal nerve, supplies most of
the dorsum of the foot
d. The sural nerve, a branch of the common peroneal nerve, supplies the lateral aspect of
the foot and the fifth digit
e. The saphenous nerve, a branch of the common peroneal nerve, supplies the medial side
of the foot

Question 138
Regarding the reflexes of the lower limb.
a. The knee jerk reflex tests spinal roots L2–L4
b. A tap at the patella causes an extension of the leg
c. The ankle reflex tests spinal roots S1, S2
d. When the foot is dorsiflexed, a tap over the Achilles tendon causes a voluntary plantar
flexion of the foot
e. A grading of ‘2’ to a stretch response is considered normal

Question 139
Regarding the lumbar and sacral dermatomes.
a. Anal continence is maintained by dermatomes S3 and S4
b. L3–L5 supply the anterior aspect of the knee
c. L4 supplies the medial malleolus
d. S1–S3 supply the posterior aspect of the knee
e. S2 and L4 supply the anterior and posterior aspects of the hallux, respectively

Question 140
With regards to the popliteal fossa:
a. The lateral border is formed by the biceps femoris
b. The medial border is formed by the semimembranosus
c. The lower border is formed by the heads of the gastrocnemius
d. The femoral nerve divides within the fossa into the tibial and the common peroneal
nerves
e. The popliteal artery runs anteriorly to the tibial nerve

Downloaded from https://www.cambridge.org/core. University of Edinburgh, on 19 Aug 2019 at 13:21:27, subject to the Cambridge Core terms of
use, available at https://www.cambridge.org/core/terms. https://doi.org/10.1017/9781108566100.005
Chapter 2a: Anatomy Questions 131

Question 141
A complete popliteal nerve block will cause loss of:
a. Sensation to the lateral aspect of the lower leg
b. Sensation to the medial aspect of the lower leg
c. Plantar flexion of the foot
d. Dorsiflexion of the foot
e. Motor function in the great toe

Question 142
Nerves blocked during a complete ankle block include:
a. Superficial peroneal nerve
b. Deep peroneal nerve
c. Tibial nerve
d. Sural nerve
e. Sciatic nerve

Question 143
A 3-in-1 block of the lower limb targets the:
a. Femoral nerve
b. Sciatic nerve
c. Saphenous nerve
d. Lateral cutaneous nerve of the thigh
e. Obturator nerve

Question 144
Blockade of the following nerves is achieved during a lumbar plexus block:
a. Femoral nerve
b. Lateral cutaneous nerve of the thigh
c. Obturator nerve
d. Ilioinguinal nerve
e. Genitofemoral nerve

Question 145
Anatomical changes associated with pregnancy include:
a. Flaring of the ribs is a direct consequence of the enlarging uterus
b. Displacement of the diaphragm leads to a decrease in the volume of the thoracic cavity
c. An increase in barrier pressure leads to heartburn in 50–80% of pregnant women
d. Compression of the inferior vena cava diverts blood through the epidural veins
e. Pressure within the epidural space can rise up to 60 cmH2O

Downloaded from https://www.cambridge.org/core. University of Edinburgh, on 19 Aug 2019 at 13:21:27, subject to the Cambridge Core terms of
use, available at https://www.cambridge.org/core/terms. https://doi.org/10.1017/9781108566100.005
132 Chapter 2a: Anatomy Questions

Question 146
Regarding the vascular supply to the uterus and placenta:
a. The uterine artery is a branch of the posterior division of the internal iliac artery
b. Uterine blood flow to the placenta is through the spiral arteries into the intervillous
spaces
c. There are close to 50 spiral arteries supplying the placenta
d. At term, uterine blood flow can exceed 1000 ml.min–1
e. The uterine veins drain into the external iliac vein

Question 147
Regarding the fetal circulation:
a. Comprises two umbilical arteries and one umbilical vein
b. Blood from the superior vena cava does not enter the left atrium
c. The foramen ovale directs approximately 85% of oxygenated blood into the left atrium
d. The left side of the heart supplies oxygenated blood to the lower half of the body
e. Complete closure of the ductus arteriosus occurs in the first three days after birth

Question 148
Regarding the paediatric airway and respiration:
a. The cricoid is the narrowest part of the airway
b. The larynx sits more posteriorly than in the adult
c. The diaphragm is the primary muscle of ventilation.
d. There are higher numbers of Type II muscle fibres present in the diaphragm
e. The larynx sits at the level of C2–C4

Question 149
Failing intravenous access in children, an interosseous needle can be placed in the:
a. Humerus
b. Sternum
c. Femur
d. Tibia
e. Fibula

Question 150
Common formulae used in paediatric calculations include:
a. Weight = (Age + 4) × 2
b. Tube size = Age/4 + 4.5
c. Oral tube length = Age/2 + 12
d. Tidal volume = 6–8 ml.kg–1
e. Cardiac output = 200 ml.kg–1.min–1

Downloaded from https://www.cambridge.org/core. University of Edinburgh, on 19 Aug 2019 at 13:21:27, subject to the Cambridge Core terms of
use, available at https://www.cambridge.org/core/terms. https://doi.org/10.1017/9781108566100.005
Chapter
Anatomy Answers

2b
Question 1: TFTTT
The internal jugular vein begins at the jugular foramen at the base of the skull. It continues
as part of the sigmoid venous sinus before it meets the subclavian vein and terminates
ultimately in the brachiocephalic vein. Upon exiting the base of skull, the internal jugular
vein lies lateral to the internal carotid artery and then to the common carotid artery. These
two structures, along with the vagus, are enclosed within the fascial compartment termed
the carotid sheath. The carotid sheath travels in the anterior triangle of the neck, which is
formed by the sternocleidomastoid posteriorly, the mandible superiorly and the mid-line of
the neck anteriorly. There are several tributaries of the internal jugular vein, including the
facial and lingual veins. Included also are the superior and middle thyroid veins, which are
particularly relevant during thyroid surgery as they must be identified and divided in order
to prevent bleeding from the internal jugular.

Question 2: FTFFT
The subclavian vein is a continuation of the axillary vein originating at the lateral
border of the first rib and ending medial to the scalenus anterior, where it meets the
internal jugular vein. Its only tributary, the external jugular vein, meets the subclavian
at the medial third of the clavicle. The left subclavian drains into the left brachioce-
phalic, where it also receives drainage from the thoracic duct. The right subclavian
drains into the right brachiocephalic along with the right lymph duct. The course taken
around the manubrium by the left brachiocephalic makes it on average around 6 cm
long compared to 3 cm on the right side.
Cannulation of the subclavian is best achieved on a supine patient with the needle
inserted at the mid-point of the clavicle aiming towards the sternoclavicular joint.
The internal jugular can be identified by lying the patient slightly cephalad and palpating
the mid-point between the mastoid process and the sternal notch level with the cricoid
cartilage.

Question 3: FTTTT
The cervical plexus is formed from the anterior rami of the first four cervical vertebrae.
The lower branches of C3 and C4 fuse with C5 to form the phrenic nerve, which supplies the
diaphragm, parietal pleura, pericardium and the upper border of the peritoneum.
The phrenic nerve descends down the neck traversing the scalenus anterior and travelling
behind the internal jugular vein. Due to its course, pathology within the pleura or sub-
diaphragmatic peritoneum can lead to pain perceived in the shoulder tip region. Superficial
133
Downloaded from https://www.cambridge.org/core. University of Edinburgh, on 19 Aug 2019 at 13:21:27, subject to the Cambridge Core terms of
use, available at https://www.cambridge.org/core/terms. https://doi.org/10.1017/9781108566100.006
134 Chapter 2b: Anatomy Answers

branches from the cervical plexus include the lesser occipital nerve, greater auricular nerve
and supraclavicular nerves, which largely supply structures in the back of the head. Deeper
branches of the plexus supply the sternocleidomastoid, trapezius, scalenus medius, serratus
anterior and rhomboids.
Cervical plexus blocks conducted for neck surgery provide regional anaesthesia over the
neck, shoulder and upper pectoral region. Care should be taken to avoid injection into
a phrenic or vagus nerve. Blockade of the sympathetic chain in the cervical region can lead
to Horner’s syndrome.

Question 4: FTTTF
The pharynx is muscular tube originating at the base of skull extending down to the level of
the C6 vertebra. It is divided into three components: the nasopharynx, oropharynx and
laryngopharynx. The walls of the pharynx are covered in four layers: mucosal, fibrous,
muscular and fascial. The mucosal coat is continuous with the nose where it consists of
ciliated, columnar epithelium. The rest is stratified squamous epithelium. The muscular
layer contains the pharyngeal constrictors: superior, middle and inferior. The muscles have
two functional parts, between which pharyngeal pouches can occur. The pharyngeal mus-
cles involved in swallowing are supplied largely by the vagus and accessory nerves.
The glossopharyngeal nerve supplies only the stylopharyngeus muscle.

Question 5: TTFTT
The nasopharynx extends from the base of the skull down through the pharyngeal isthmus,
where it connects to the oropharynx. The Eustachian tube enters the nasopharynx just
below the two posterior nasal openings, which are termed the nasal choanae.
The adenoids, which are also termed the nasopharyngeal tonsils, are small collections of
lymphoid tissue that lie on the roof of the nasopharynx.
The oropharynx then continues from the nasopharynx superiorly down to the upper
border of the epiglottis. The fauces within the orophayrnx contain collections of lymphoid
tissue, which are the palatine tonsils.
The laryngopharynx begins at the tip of the epiglottis down to the lower border of the
cricoid cartilage. The piriform fossae are recesses formed secondary to the bulgings of the
larynx into the laryngopharynx. As such, they are well placed to house tiny items of food
that have been inadvertently inhaled.

Question 6: TTTTT
There are nine cartilages in total, three are paired and three exist singly.
The single cartilages are:
i. Thyroid cartilage: this is the largest and is made up of two plates which join to form the
thyroid notch (Adam’s apple).
ii. Cricoid cartilage: this is shaped like a signet ring. The lateral surfaces consist of facets
which join with the inferior cornu of the thyroid cartilage. The upper border joins with
the arytenoid cartilages.
iii. Epiglottis: the inferior border extends posteriorly to attach to the thyroid cartilage.
The anterior surface is attached to the hyoid bone. The valleculae are two depressions

Downloaded from https://www.cambridge.org/core. University of Edinburgh, on 19 Aug 2019 at 13:21:27, subject to the Cambridge Core terms of
use, available at https://www.cambridge.org/core/terms. https://doi.org/10.1017/9781108566100.006
Chapter 2b: Anatomy Answers 135

that fall either side of the glossoepiglottic folds, which connect the epiglottis to the
posterior tongue.
The paired cartilages are:
i. Arytenoid cartilages: these are shaped like a three-sided pyramid. The anterior
processes attach to the vocal ligaments to form the vocal cords. The muscular
process projects laterally and is attached to the posterior and lateral cricoarytenoid
muscles.
ii. Corniculate and cuneiform cartilages: these are small cartilages that lie on the posterior
aryepiglottic folds.

Question 7: TTFFT
The ligaments within the larynx connect the cartilages and are generally named as
a combination of the cartilages that they conjoin.
• The aryepiglottic fold (may also be referred to as ligament or membrane) runs from the
side of the epiglottis down to the arytenoid cartilage. The inferior border of the fold runs
free and is covered by a loose membrane called the vestibular fold. This is known as the
false vocal cord.
• The cricothyroid ligament runs from the cricoid cartilage to the thyroid cartilage and is
the insertion point for an emergency needle cricothyroidotomy.
• The thyrohyoid membrane connects the thyroid cartilage to the hyoid bone. Through
this membrane run the superior laryngeal artery and the internal laryngeal nerve.
• The conus elasticus is a triangular shaped ligament that attaches to the cricoid cartilage
inferiorly. The upper border attaches to the arytenoid cartilage and is thickened to form
the vocal ligament. This forms the true vocal cord.

Question 8: FFTTT
Muscles of the larynx can be divided into the extrinsic and intrinsic muscles.
The extrinsic muscles and their respective functions are:
a. Sternothyroid: depresses the larynx
b. Thryohyoid: elevates the larynx
c. Inferior constrictor: constricts the pharynx.
The intrinsic muscles and their respective functions are:
a. Cricothyroid (paired): increases the diameter of the glottis
b. Posterior cricoayrtenoid (paired): abducts the vocal cord
c. Lateral cricoarytenoid (paired): adducts the vocal cord
d. Aryepiglottic (paired): minor constriction of laryngeal inlet
e. Thyroarytenoid (paired): relaxation of the vocal cord
f. Transverse arytenoids (unpaired): constriction of the glottis.

Question 9: FTFFT
The arterial supply to the larynx is from the superior and inferior laryngeal arteries, which
are respectively the branches of the superior and inferior thyroid arteries.

Downloaded from https://www.cambridge.org/core. University of Edinburgh, on 19 Aug 2019 at 13:21:27, subject to the Cambridge Core terms of
use, available at https://www.cambridge.org/core/terms. https://doi.org/10.1017/9781108566100.006
136 Chapter 2b: Anatomy Answers

Venous drainage is via the superior and inferior laryngeal veins, which ultimately drain
into the internal jugular vein.
The nerve supply is divided into two; above the vocal cords, innervation is via the internal
laryngeal nerve. Below the vocal cords, the muscles are innervated by the recurrent laryngeal
nerve. An exception to this is the cricothyroid muscle, which is supplied by the superior
laryngeal nerve.
The superior laryngeal artery and the internal laryngeal nerve both traverse the thyro-
hyoid membrane.

Question 10: TFTTT


The orbit is pyramidal in shape and houses the globe which is located anteriorly in
a superolateral position. The orbit has three posterior openings:
i. The optic canal: optic nerve and ophthalmic artery
ii. The superior orbital fissure: superior and inferior ophthalmic veins
iii. The inferior orbital fissure.
The globe itself consists of three layers:
i. Outer fibrous layer, sclera: covers the entire globe except the cornea
ii. Vascular layer: contains the choroid, iris and ciliary body
iii. Inner retina: contains photoreceptors. There are approximately 120 million rods and
7 million cones.

Question 11: TTTFT


Before an image can be processed by the brain it needs to be directed appropriately onto the
retina. The pupil constricts and dilates relative to light intensity and is controlled by the
action of the smooth muscle fibres in the iris. The lens is crucial in providing accommoda-
tion of the image and is altered by tension within the suspensory ligaments. These are
controlled by the ciliary muscles.
Images are then projected onto the retina, where they are converted into electrical
potentials. These are transmitted onto ganglion cells and reach the optic disc via the
axons. The optic nerves from each eye converge at the optic chiasm. Images from the
nasal half decussate, whereas images from the temporal half remain ipsilateral. Fibres are
then carried to the lateral geniculate nuclei and on to the visual cortex. The superior collicli
also intercept some fibres to allow for control of eye movement.

Question 12: FFTTF


CN III, the oculomotor nerve, carries parasympathetic fibres from the Edinger–Westphal
nucleus to cause pupillary constriction.
CN IV, the trochlear nerve, supplies the superior oblique muscle and CN VI, the
abducens nerve, supplies the lateral rectus muscle. All the other extraocular muscles are
supplied by the oculomotor nerve.
The sensory component of the corneal reflex is mediated by the ophthalmic division of
the trigeminal nerve, CN V. The temporal and zygomatic branches of the facial nerve, CN
VII, control the action of the ‘blink’. Other branches of the facial nerve include the buccal,
mandibular and cervical.

Downloaded from https://www.cambridge.org/core. University of Edinburgh, on 19 Aug 2019 at 13:21:27, subject to the Cambridge Core terms of
use, available at https://www.cambridge.org/core/terms. https://doi.org/10.1017/9781108566100.006
Chapter 2b: Anatomy Answers 137

Question 13: TTFFT


The oral cavity of the mouth contains the hard and soft palates and the tongue. It is
separated from the vestibule of the mouth anteriorly by the teeth. The tongue itself is
a muscular structure and is controlled by the actions of several intrinsic and extrinsic
muscles. The innervation to these muscles is via the glossopharyngeal nerve, CN IX, except
for the palatoglossus, which is supplied by CN X, the vagus nerve.
The tongue is superficially divided into an anterior two-thirds and a posterior third.
The lingual nerve, which arises from the mandibular branch of the trigeminal nerve,
supplies the anterior two-thirds of the tongue, passes through the chorda tympani and
joins the facial nerve to reach the nucleus of the tractus solitarius. The posterior third of the
tongue is supplied by the glossopharyngeal nerve.

Question 14: TTFFF


The nose is made up of two nasal cavities, which are separated by the nasal septum. Each
cavity is made up of a roof, a floor, a medial wall and a lateral wall.
The roof of the cavity consists of cartilages and bones and also the cribriform plate of the
ethmoid bone. The floor is formed by the palatine bone and the palatine process of the
maxilla.
The nasal septum makes up the medial wall and is mainly cartilaginous. The lateral wall
contains three conchae, which help to increase the surface area: superior, middle and
inferior. It receives openings from the paranasal sinuses and the nasolacrimal duct.
Nasal lining is primarily respiratory epithelium, except for the area over and surrounding
the superior conchae, which is yellow, olfactory epithelium.
Arterial blood supply to the nose is from the ophthalmic and maxillary arteries. The facial
vein drains blood away from the nose into the pterygoid venous plexus.

Question 15: TTTFF


The thyroid gland consists of two lobes that are connected by a central isthmus. The gland is
located at the region of the second and third tracheal rings with the isthmus overlying the
trachea. The thyroid lobes contain follicles or acini and it is in the colloid within these that
the thyroglobulin and iodine are stored. The parafollicular cells that are located on the
outside of the acini are responsible for the production of calcitonin.
The thyroid gland is a vascular-rich organ and receives its blood supply via the superior
and inferior thyroid arteries. Venous drainage is via the superior, middle and inferior
thyroid veins.

Question 16: FTFTF


The four parathyroid glands are located in both the two upper and two lower poles of the
thyroid gland. They consist of two cell types: the primary chief cells are responsible for the
production and storage of the parathyroid hormone (PTH). PTH is a polypeptide hormone
that is released into the bloodstream to raise levels of serum calcium. Calcitriol, which is
formed by vitamin D, also aids in the increase of serum calcium levels. Calcitonin, released
from the parafollicular C cells of the thyroid gland, causes a reduction in the level of serum
calcium.

Downloaded from https://www.cambridge.org/core. University of Edinburgh, on 19 Aug 2019 at 13:21:27, subject to the Cambridge Core terms of
use, available at https://www.cambridge.org/core/terms. https://doi.org/10.1017/9781108566100.006
138 Chapter 2b: Anatomy Answers

Question 17: FTTTF


The carotid sheath contains the:
• Internal carotid artery
• Internal jugular vein
• Vagus nerve
• Deep cervical lymph nodes.

Question 18: TFFTT


The relations of the internal jugular vein are:
• Anterolateral to the carotid artery (through its most part, but this can vary depending on
the level of the neck)
• Anterior to the vagus nerve
• Posterior to the sympathetic chain
• Medial to the hypoglossal and glossopharyngeal nerves
• On the left side, the internal jugular vein lies anterior to the thoracic duct.

Question 19: TTFFF


Complications associated with a deep cervical plexus block:
• Phrenic nerve palsy
• Haematoma
• Intrathecal injection: there may be a dural sleeve surrounding the cervical plexus into
which the injection could be placed. This would invariably lead to signs and symptoms of
a high spinal injection
• Horner’s syndrome and recurrent laryngeal nerve blockade are commonly seen with
superficial cervical plexus block.

Question 20: TTTFF


Deep or superficial cervical plexus blocks can be undertaken to provide anaesthesia for
carotid endarterectomy. Nerves within the cervical plexus include:
• Lesser occipital nerve
• Greater auricular nerve
• Transverse cervical nerve
• Supraclavicular nerve.
Often, retraction pain is felt over the submandibular region and therefore branches of the
trigeminal nerve also need to be blocked.

Question 21: TTTFF


The pterygopalatine fossa is often infiltrated with local anaesthetic to provide a maxillary
nerve block for oral and maxillofacial surgery.
Structures contained within the fossa include the maxillary nerve, which is a branch of the
trigeminal nerve. The pterygopalatine ganglion is also suspended within the fossa.

Downloaded from https://www.cambridge.org/core. University of Edinburgh, on 19 Aug 2019 at 13:21:27, subject to the Cambridge Core terms of
use, available at https://www.cambridge.org/core/terms. https://doi.org/10.1017/9781108566100.006
Chapter 2b: Anatomy Answers 139

The greater petrosal and deep petrosal nerves contain parasympathetic nerve fibres and
form the nerve of the pterygoid canal.
Local anaesthetic will block the maxillary nerve and provide loss of sensation to the
maxillary sinus, upper molars, canines and incisor teeth, the cheek and the gums.

Question 22: TTFTT


A sub-Tenon’s block is delivered by injection using a blunt needle into the sub-Tenon’s
space. Topical anaesthesia is applied onto the conjunctiva along with a few drops of aqueous
iodine. An incision is then made on the inferomedial aspect of the conjunctiva and the
needle is advanced along the posterior aspect of the sclera. Major complications are rare due
to the use of a blunt needle and relative distance away from the optic sheath. Subconjuctival
haemorrhage and subconjuctival swelling are seen in roughly 40–50% of patients.

Question 23: TTFTT


Complications associated with a partial thyroidectomy include:
• Bleeding leading to haematoma formation. This can cause airway obstruction, which will
manifest as stridor. Injury to the recurrent laryngeal nerve can also cause unilateral cord
paralysis, which can lead to hoarseness of voice.
• Hypoparathyroidism is more common in a complete thyroidectomy and can cause
hypocalcaemia.
• Thyroid storm is a rare, but severe, complication of hyperthyroidism. In the acute setting
it manifests as tachycardia, often associated with atrial fibrillation, fever and agitation
that may lead to myocardial infarction. In the longer term it causes diarrhoea and
vomiting, heart failure and eventually death. It may be caused by uncontrolled/untreated
hyperthyroidism or in the acute setting either by intercurrent infection in patients
suffering with mild hyperthyroidism or manipulation of a hyperthyroid gland, such as
during surgery. The treatment is twofold. Firstly the acute complications of thyroid
storm require immediate medical management in parallel with drugs to reduce thyroid
hormone secretion. Thus β-blockers, nitrites, paracetamol and manual cooling are
employed to treat the atrial fibrillation, hypertension and pyrexia, and antithyroid drugs
(e.g. propylthiouracil) are employed to reduce thyroid hormone production. Secondly,
the primary cause of the thyroid crisis requires management.

Question 24: FTFFT


The presence of a goitre can have anatomical manifestations that can affect anaesthetic
management.
Compression effects of the goitre on the airway can lead to tracheomalacia, although this
is potentially recognized as a postoperative complication.
Pressure on the superior vena cava can lead to oedema in the face and the airway, and also
engorgement of the nasopharyngeal veins. This can lead to problems with endotracheal as
well as awake nasal fibre-optic intubation.
A decreased venous return will also affect placement of a central venous catheter in the
internal jugular veins.

Downloaded from https://www.cambridge.org/core. University of Edinburgh, on 19 Aug 2019 at 13:21:27, subject to the Cambridge Core terms of
use, available at https://www.cambridge.org/core/terms. https://doi.org/10.1017/9781108566100.006
140 Chapter 2b: Anatomy Answers

Vocal cord palsy can result due to compression of the recurrent laryngeal nerve.
Unilateral involvement causing cord adduction will lead to a hoarse voice and bilateral
involvement will lead to stridor.

Question 25: FTTFF


Recurrent laryngeal nerve injury is a recognized complication of thyroid surgery.
Due to the anatomical orientation of the right recurrent laryngeal nerve, it is more prone
to injury than the left. Unilateral injury presents with ipsilateral vocal cord paralysis.
The cord is immobile and suspended in a slight abduction away from the mid-line.
Unilateral injury presents with hoarseness and a reversible injury may take up to six months
to improve. Corrective surgery should only be performed after this period unless there are
life-threatening circumstances.
Bilateral nerve injury is more likely in a total thyroidectomy and will lead to a partial
airway obstruction presenting as stridor or respiratory distress. As it is often only detected
post extubation, treatment consists of emergency reintubation or emergency tracheostomy.

Question 26: TFFTT


The trachea runs from the lower border of the larynx, at the level of C6, and divides into left
and right main bronchi at the level of the sternal angle, approximately T5. Each trachea has
15–20 C-shaped cartilaginous rings that are connected by the trachealis muscle anteriorly,
leaving a membranous wall posteriorly, which can collapse when the oesophagus expands.
The trachea is lined by pseudostratified ciliated columnar epithelium and goblet cells, which
produce mucus.

Question 27: TTFFF


The trachea lies median in the neck, but deviates slightly to the right towards its lower end.
It lies anterior to the oesophagus. Laterally, are the right and left lobes of the thyroid gland,
the parathyroid glands (normally two on each side) and the carotid sheath (containing the
common carotid artery, the internal jugular vein and the vagus nerve).
Other anterior relations to the trachea include the thyroid ima artery and the inferior
thyroid veins. The thyroid ima artery is only present in a small number of people, arises
from the brachiocephalic artery and supplies the thyroid isthmus. It can be damaged during
thyroid surgery or percutaneous tracheostomy insertion. The inferior thyroid veins drain
the lower lobes of the thyroid artery and drain into the brachiocephalic veins.
The arch of the aorta runs anteriorly to the trachea, but then superior to the left main
bronchus, once the trachea has bifurcated.

Question 28: FTFFT


A tracheostomy is normally made between tracheal rings 2 and 4. A transverse incision can
be made, dissecting through skin and superficial fat and dividing the infrahyoid muscles
(strap muscles) longitudinally in the mid-line. This is retracted laterally. At this point, the
operator must ensure they are in the correct position, avoiding the thyroid isthmus, the
thyroid ima artery and the inferior thyroid plexus of veins, all of which can be in the mid-

Downloaded from https://www.cambridge.org/core. University of Edinburgh, on 19 Aug 2019 at 13:21:27, subject to the Cambridge Core terms of
use, available at https://www.cambridge.org/core/terms. https://doi.org/10.1017/9781108566100.006
Chapter 2b: Anatomy Answers 141

line and cause bleeding. In children the thymus may also be present anteriorly. Also, in
children, the trachea is small and soft, and care must be taken in dividing it, to ensure the
oesophagus is not damaged posteriorly.

Question 29: FFFFF


Dual antiplatelet therapy is associated with a higher risk of bleeding from the procedure, but
is not an absolute contraindication.
Open surgical and percutaneous tracheostomy insertions are widely practised. Each has
their own benefits, however certain situations dictate the need for surgical insertion over the
percutaneous method.
Absolute contraindications to percutaneous insertion include:
• Age below 8 years old
• Abnormal neck anatomy
• Tumour
• Thyromegaly
• High innominate artery
Relative contraindications include:
• Obesity
• Prolonged bleeding time
• Platelet count <50 000 μl–1
• Infection at the wound site
• PEEP requirements >20 cmH2O

Question 30: TTFFT


Insertion of a tracheostomy can lead to a number of complications.
Early complications include:
• Bleeding
• Tracheal cartilage fracture
• Posterior tracheal wall injury
• Pneumothorax
• Infection
Late complications include:
• Inadvertant decannulation
• Infection
• Recurrent laryngeal nerve injury leading to vocal cord paralysis
• Tracheal stenosis
• Tracheomalacia
• Tracheoarterial fistula
• Persistent tracheal stoma

Question 31: TFTFT


The three layers of the muscles in the intercostal space are the external intercostal, inner
intercostal and the innermost intercostal (superficial to deep).

Downloaded from https://www.cambridge.org/core. University of Edinburgh, on 19 Aug 2019 at 13:21:27, subject to the Cambridge Core terms of
use, available at https://www.cambridge.org/core/terms. https://doi.org/10.1017/9781108566100.006
142 Chapter 2b: Anatomy Answers

The neurovascular bundle contains the intercostal vein, artery and ventral rami of
T1– T11 (superior to inferior) and lies in the space between the inner and innermost
intercostal muscles, in the intercostal groove of the rib above.
A collateral neurovascular bundle is also present in the space between the inner and
innermost intercostal muscles, in the intercostal space, which lies superior to the rib below.
Therefore care must be taken of this bundle when performing a pleural tap.

Question 32: TTTFF


The ventral rami of T1–T11 form the intercostal nerve. However, the dorsal and ventral
rami of T1–T12 spinal nerve roots form their respective dermatomes, via multiple cuta-
neous branches given off by the nerves as they pass anteriorly along the costal groove.
A group of muscles also supplied by the intercostal nerves are the myotome.
Rami communicantes connect the intercostal nerves to the sympathetic trunk, which
sends fibres with the nerve.

Question 33: FTTFF


The right main bronchus is shorter, wider and more vertical than the left, making foreign
bodies more likely to lodge in it than the left. The right main bronchus passes directly to the
root of the lung, accompanied by the pulmonary artery.
The left main bronchus is longer, narrower and more angulated than the right. It passes
inferior to the arch of the aorta and anterior to the oesophagus and aorta, before entering
the root of the lung, accompanied by the pulmonary artery.

Question 34: TTTTF


The trachea divides into two main bronchi, left and right. Each main bronchus is accom-
panied by pulmonary and bronchial vessels, lymph vessels and autonomic and sympathetic
nerve fibres into the root of the lung.
The left main bronchus then divides into two secondary bronchi, which supply two lobes,
superior and inferior. The right main bronchus divides into three secondary bronchi, which
supply three lobes, superior, inferior and middle. Each secondary bronchus divides into
tertiary bronchi that supply each bronchopulmonary segment. There are nine bronchopul-
monary segments on each side.
The oblique fissure divides the superior and inferior lobes on the left, and divides the
middle and inferior lobes on the right. The horizontal fissure divides the superior and
middle lobes on the left.

Question 35: TFTTT


Each lung has three surfaces: costal, mediastinal and diaphragmatic. The costal surface is
adjacent to the sternum and ribs, the mediastinal surface is adjacent to the mediastinum and
the vertebrae, and the diaphragmatic surface is adjacent to the diaphragm. The root of the
lung is on the mediastinal surface, and is the area of continuity of the parietal and visceral
layers of pleura. It contains the right and left main bronchi, pulmonary and bronchial
vessels, nerves and lymph vessels.

Downloaded from https://www.cambridge.org/core. University of Edinburgh, on 19 Aug 2019 at 13:21:27, subject to the Cambridge Core terms of
use, available at https://www.cambridge.org/core/terms. https://doi.org/10.1017/9781108566100.006
Chapter 2b: Anatomy Answers 143

Question 36: TTFTF


The apex of the lung is the superior aspect of the lung, covered by cervical pleura, and
extends through the superior thoracic aperture.
Each lung has three borders: anterior, inferior and posterior (there is no superior
border). The anterior border separates the costal and mediastinal surfaces anteriorly.
The left side forms the cardiac notch. The posterior border separates the costal and
mediastinal surfaces posteriorly. The inferior border encircles the diaphragmatic sur-
face of the lung.

Question 37: FTFTF


The lung, like the liver, has two sources of blood supply. The first is provided by a single
pulmonary artery, which carries deoxygenated blood from the right ventricle. This blood is
drained by two pulmonary veins, which carry oxygenated blood to the left atrium. Each
pulmonary artery gives off a superior branch prior to entering the hilum. Within the lung,
each artery divides into lobar branches.
The bronchial arteries supply the lung with the oxygen and nutrition it requires. These
generally arise directly from the thoracic aorta. The drainage of this blood is by the
bronchial and pulmonary veins.
The lungs also have two lymphatic plexuses, superficial and deep, each one divided into
left and right. The superficial plexus lies deep to the visceral pleura and drains into the
bronchopulmonary lymph nodes. The deep plexus lies in the submucosa of the bronchi and
drains into the pulmonary lymph nodes. These lymph plexuses drain either into the right or
left bronchomediastinal lymph plexuses, which, in turn, drain into the lymphatic duct on
the right and the thoracic duct on the left.

Question 38: TFTTF


The pulmonary plexus is located at the root of the lungs and supplies the lung, the
pulmonary arteries and veins, and the visceral pleura. The nerves of the lungs and visceral
pleura are derived from the pulmonary plexus. They contain sympathetic and parasympa-
thetic fibres. The sympathetic fibres arise from the sympathetic trunk and the parasympa-
thetic fibres arise from the vagus nerve (CN X). Parasympathetic innervation leads to
bronchoconstriction, vasodilatation and increased secretion of the glands of the bronchial
tree. Fibres of CN X are also found in the bronchial mucosa and muscles, interalveolar
connective tissue, pulmonary arteries and pulmonary veins. Sympathetic innervation leads
to bronchodilatation, vasoconstriction and reduced secretion of the glands of the bronchial
tree. It is in this context that antimuscarinic agents are useful for bronchospasm.
The intercostal and phrenic nerves supply sensory innervation to the parietal pleura.

Question 39: TFFFT


See explanation for Question 38.

Downloaded from https://www.cambridge.org/core. University of Edinburgh, on 19 Aug 2019 at 13:21:27, subject to the Cambridge Core terms of
use, available at https://www.cambridge.org/core/terms. https://doi.org/10.1017/9781108566100.006
144 Chapter 2b: Anatomy Answers

Question 40: TTFTT


The pleurae consist of two layers: visceral and parietal. Between them is a potential space
called the pleural cavity. The pleural cavity consists of a thin layer of pleural fluid, which
lubricates the movements of the lungs during respiration. The visceral pleura invests the
lungs and the parietal pleura lines the thoracic wall and diaphragm. The parietal pleura is
anatomically divided into costal pleura, mediastinal pleura, diaphragmatic pleura and
cervical pleura, depending on the region of the thoracic cavity they are covering.
The visceral and parietal pleura become continuous at the root of the lung and form the
pulmonary ligament.

Question 41: FFFFF


The main function of the diaphragm is inspiration, when it descends into the abdomen,
increasing the volume of the thorax, therefore reducing the intrathoracic pressure. It is
formed by a central tendinous portion, and an outer muscular portion. The central tendi-
nous portion has three main foramina within it to allow structures to pass between the
abdomen and thorax, but the diaphragm itself is not continuous with these structures.
The diaphragm is, however, lined by parietal peritoneum, which also lines these structures,
and it is this that is continuous with these structures.

Question 42: TTTTT


The outer part of the diaphragm, the muscular portion, forms the attachments of the
diaphragm to the body. The attachments can be divided into sternal, costal and lumbar.
The sternal part is attached to the xiphoid process and forms two small foramina either
side of it. The costal part is attached to the lower surface of the ribs and costal cartilages of
ribs 6–12 and the posterior part of the diaphragm is attached to the lumbar vertebrae via
the crura, a ligamentous portion of the diaphragm. The right crus is longer and inserts
into L1–L3, compared to the left crus which inserts into L1 and L2. The two crura are
connected via the median arcuate ligament.

Question 43: TFTFT


There are three main foramina within the diaphragm:
1. The aortic hiatus, formed posterior to the diaphragm. It is formed by the median arcuate
ligament anteriorly, the left and right crus of the diaphragm laterally and the anterior
surface of T12 posteriorly. The thoracic duct, sympathetic trunk, subcostal nerve and
azygous duct also pass through the aortic hiatus.
2. The vena caval foramen, formed within the central tendinous section of the diaphragm
at level of T8. It allows passage of the vena cava, right phrenic nerve and lymph vessels.
3. The oesophageal hiatus, formed by the overlapping flaps of the muscular part of the
diaphragm posteriorly. With it, the vagus and left gastric vessels also pass.
There are also two small foramina lateral to the sternum, which allow passage of the
internal thoracic artery.

Downloaded from https://www.cambridge.org/core. University of Edinburgh, on 19 Aug 2019 at 13:21:27, subject to the Cambridge Core terms of
use, available at https://www.cambridge.org/core/terms. https://doi.org/10.1017/9781108566100.006
Chapter 2b: Anatomy Answers 145

Question 44: TFTTF


The arterial supplies to the superior and inferior surfaces of the diaphragm are from the
respective phrenic arteries, via the aorta. The superior surface also has a supply from the
musculophrenic and pericardiophrenic arteries via the internal thoracic arteries.
The venous drainage of the superior surface of the diaphragm is via the musculophrenic
and pericardiophrenic veins to the internal thoracic veins. The venous drainage of the
inferior surface of the diaphragm is via the inferior phrenic veins, which on the right drain
into the IVC and on the left drain into the left suprarenal vein.
The lymph drainage of the superior surface of the diaphragm is to the diaphragmatic
lymph nodes, to the phrenic, parasternal and posterior mediastinal lymph nodes.
The lymph drainage to the inferior surface of the diaphragm is to the superior lumbar
lymph nodes. Lymph drainage between the superior and inferior surfaces of the diaphragm
communicates freely, which of course has huge ramifications in the spread of infection and
tumour between the abdomen and thorax.

Question 45: TTFTF


Unlike the vascular supply of the diaphragm, the innervation of the diaphragm is the same
on both the superior and inferior surfaces. The motor supply is entirely from the phrenic
nerve, C3–C5. The sensory supply is divided between the central part, which is supplied by
the phrenic nerve, C3–C5, and the peripheral part, which is supplied by the intercostal
(T5–T11) and subcostal nerves (T12) in direct relation to that part of the diaphragm.
Because of this split in sensory innervation, referred pain can be felt in two areas.
The most common place for referred pain is at the shoulder (dermatome C5), after irritation
of the central portion of the diaphragm, supplied by C3–C5. This occurs following laparo-
scopic intra-abdominal surgery. Alternatively, referred pain can also be felt over the
respective dermatomes T5–T12, depending on which part of the peripheral diaphragm is
being irritated.

Question 46: FTFTT


The right atrium forms the entire right border of the heart, as seen on chest X-ray. It receives
the inferior and superior vena cava and the coronary sinus, which drains venous blood from
the cardiac veins. There are two anatomical parts to the internal wall of the right atrium
divided by the crista terminalis; a smooth walled section posteriorly, and a rough, more
muscular section anteriorly. The smooth-walled part corresponds to the foramen ovale,
found in the fetal heart. As the right system is of lower pressure, the walls of the right side of
the heart are thinner, in comparison to the left side. The tricuspid valve lies between the
right atrium and the right ventricle.

Question 47: FFFTT


The left atrium forms the base and superior borders of the heart, as viewed on a chest X-ray.
It receives oxygenated blood via the pulmonary vein and pumps it to the left ventricle via the
tricuspid mitral valve. The mitral valve is attached to the internal wall of the atrium via
muscular extensions, called the papillary muscles, and tendinous extensions called chordae

Downloaded from https://www.cambridge.org/core. University of Edinburgh, on 19 Aug 2019 at 13:21:27, subject to the Cambridge Core terms of
use, available at https://www.cambridge.org/core/terms. https://doi.org/10.1017/9781108566100.006
146 Chapter 2b: Anatomy Answers

tendinae. The internal wall of the atrium is generally smooth, except for the papillary
muscles.

Question 48: TTTFF


The right ventricle forms most of the anterior and inferior (diaphragmatic) surfaces of the
heart.
The right ventricle receives blood from the right atrium, via the right atrioventricular/
tricuspid valve and pumps blood via the pulmonary valve and pulmonary artery to the
lungs. The pulmonary valve is also a tricuspid valve (formed by three cusps), but the
formation of the cusps is very different to that of the right atrioventricular valve.
The pulmonary valve is formed by three semi-lunar cusps and the tricuspid valve is formed
by three ‘U-shaped’ cusps, attached to the internal wall of the heart via muscular projections,
papillary muscles and tendinous insertions, chordae tendineae.
The interventricular septum is composed of a thin membranous section, which forms
attachments superiorly and posteriorly, and a muscular part, which forms the centrally
located division between the ventricles.

Question 49: TTTTF


The left ventricle forms the apex, the left surface and the diaphragmatic surfaces. It pumps
blood to the systemic circulation via the ascending aorta, which exits superoanteriorly from
it. The aortic valve sits at the entry to the aorta. It has three semi-lunar cusps, right, left and
posterior. Two aortic sinuses supply the coronary circulation, exiting from the right and left
aortic sinuses. There is no aortic sinus from the posterior cusp.

Question 50: TTFFT


When describing the surface anatomy of the heart, five points need to be addressed:
1. The apex (left ventricle) is commonly palpated at the fifth intercostal space in the mid-
clavicular line, on the left. In hypertrophic diseases of the heart, this is often displaced
more inferolaterally.
2. The left border runs from the second left costal cartilage to the fifth left intercostal space,
mid-clavicular line (apex).
3. The right border runs almost vertically, from the third to sixth costal cartilages.
4. The superior border runs from the third costal cartilage on the right to the second costal
cartilage on the left.
5. The inferior border runs from the sixth costal cartilage on the right to the apex.

Question 51: FTFTF


The mitral valve is posterior to the fourth left costal cartilage, but is auscultated at the apex.
The pulmonary valve is at the level of the third left costal cartilage and can be auscultated
at this point.
The tricuspid valve is posterior to the left side of the fifth intercostal space and can be
auscultated at this point.

Downloaded from https://www.cambridge.org/core. University of Edinburgh, on 19 Aug 2019 at 13:21:27, subject to the Cambridge Core terms of
use, available at https://www.cambridge.org/core/terms. https://doi.org/10.1017/9781108566100.006
Chapter 2b: Anatomy Answers 147

The aortic valve is indeed posterior to the left side of the third intercostal space, but is
auscultated at the right parasternal third intercostal space. A referred pulse from the aortic
valve can be auscultated at the right carotid in aortic stenosis.

Question 52: FTTTT


For an explanation, please see Question 53.

Question 53: TTFFT


The sinoatrial (SA) node is found in the right atrium and is known as the pacemaker of the
heart as it generates the initial electrical impulse. From here, the impulse travels to the AV
node, which after a delay transmits the electrical impulse via the bundle of His to the left and
right ventricles. A delay is required to prevent simultaneous contraction of the atria and
ventricles. This delay forms the P-R interval on the ECG. The Purkinje fibers stimulate each
muscle fiber individually to contract.
Parts of the ECG and how they relate to cardiac contraction:
• P-wave: atrial contraction
• P-R interval: delay caused by the AV node in transmitting the electrical impulse from
atria to ventricles
• QRS complex: ventricular contraction
• ST segment: depolarization of the ventricles
• T-wave: repolarization of the ventricles

Question 54: TFFTF


For an explanation, see Question 55.

Question 55: FTTFT


The left and right coronary arteries arise from the ascending aorta, distal to the aortic valve,
via the left and right coronary sinuses, respectively.
The right coronary artery runs in the coronary groove (also known as the atrioventricular
groove) and divides into:
• The SA nodal artery (in 60% of cases) to supply the SA node
• The right marginal artery, supplying the right ventricle and the apex
• The posterior interventricular artery, which supplies both ventricles
• The AV nodal artery, which supplies the AV node
The left mainstem artery runs for 1–2 cm in the anterior atrioventricular groove and
divides into:
• The SA nodal artery (in 40% of cases) to supply the SA node
• The left anterior descending artery (LAD) to supply the right and left ventricles
• The circumflex artery, which supplies the left atrium and ventricle
• The left marginal artery, which supplies the left ventricle
The posterior and anterior interventricular arteries form an arch of arteries and the left
circumflex also anastomoses with the main right coronary artery.

Downloaded from https://www.cambridge.org/core. University of Edinburgh, on 19 Aug 2019 at 13:21:27, subject to the Cambridge Core terms of
use, available at https://www.cambridge.org/core/terms. https://doi.org/10.1017/9781108566100.006
148 Chapter 2b: Anatomy Answers

Question 56: TFTFF


The venous drainage of the heart is composed of two systems: veins that drain into the
coronary sinus and the small veins (venae cordis minimae and anterior cardiac veins),
which drain directly into the right atrium.
The coronary sinus runs in the posterior aspect of the atrioventricular groove and receives
tributaries from the great, middle and small cardiac veins; it drains into the right atrium.

Question 57: TTTTF


The pericardium is composed of two layers, the outer fibrous pericardium and the inner
serous pericardium.
The outer fibrous pericardium is fused to the tunica adventitia of the great vessels and the
diaphragm, and attached to the posterior surface of the sternum via the sternopericardial
ligament.
The inner serous pericardium is composed of an outer parietal layer and inner
visceral layer (also known as the epicardium). The potential space between the parietal
and visceral layers is known as the pericardial space and contains fluid which helps
reduce friction.

Question 58: TTTFF


The arterial supply to the pericardium is mainly via the internal thoracic artery (via the
pericardiophrenic and musculophrenic arteries), but also directly via the thoracic aorta and
coronary arteries.
The venous drainage is via the pericardiophrenic vein to the internal thoracic vein and the
azygous veins.
The nerve supply to the pericardium is via the vagus nerve (CN X, parasympathetic
supply), sympathetic trunks and the phrenic nerves (C3–C5).

Question 59: TTFFF


The kidneys lie in the flanks, at the level of T12–L3, in the retroperitoneum. They are
surrounded by perirenal fat and a renal fascia, also called Gerota’s fascia. Each kidney is
supplied by a renal artery and renal vein, which enter/leave the renal hilum at L1/ L2 (N.B.
anatomical variations are common, in approximately 30% of people). The renal vein is the
most anterior structure, followed by the artery and then the ureter. The left renal vein
stretches anteriorly across the aorta and enters the inferior vena cava. The left kidney is
closely related to the stomach, spleen, jejunum, pancreas and left colon. The right kidney is
closely related to the liver, duodenum and right colon.
The ureters leave the renal pelvis and travel in the retroperitonem anterior to the
transverse processes of the lumbar vertebrae before crossing the external iliac artery, distal
to the bifurcation of the common iliac artery, after which they enter the urinary bladder.
The ureters receive their blood supply as small branches of the renal artery.
A transplanted kidney is normally placed in the pelvis, retroperitoneally, receives blood
from the common or external iliac artery and drains into the common or external iliac vein.

Downloaded from https://www.cambridge.org/core. University of Edinburgh, on 19 Aug 2019 at 13:21:27, subject to the Cambridge Core terms of
use, available at https://www.cambridge.org/core/terms. https://doi.org/10.1017/9781108566100.006
Chapter 2b: Anatomy Answers 149

Question 60: TFTTT


For an explanation, see Question 59.

Question 61: FFTTF


The liver is divided into two independent lobes (right and left), according to blood supply and
venous and biliary drainage. Most (70%) of the blood supply to the liver is via the portal vein
(deoxygenated blood) and the remainder via the hepatic artery (oxygenated blood). Both
vessels enter the liver at the porta hepatis, and, with the common hepatic duct, form the portal
triad. The lesser omentum encloses the portal triad at its lateral edge. Autonomic nervous
fibres and lymphatics can also be found at the porta. Venous drainage of the liver is via the
hepatic veins, which drain directly into the inferior vena cava, within the substance of the liver.
Calot’s triangle (cystohepatic triangle) is formed by the lower edge of the liver, the
common hepatic duct and the cystic duct, and contains the cystic artery, which supplies
the gall bladder.

Question 62: TTTFF


There are two functionally independent lobes of the liver – left and right. Despite the left
lobe being smaller than the right, it also contains the caudate and quadrate lobes (liver
appendages). Both lobes have separate right and left vessels and biliary trees. They are
divided by an imaginary line between the gall bladder and the IVC. The left lobe is separated
from the caudate and quadrate lobes via the line formed by the ligamenta teres and venosum
on the inferior surface.
Three peritoneal reflections cover the anterior and superior surface of the liver: coronary
ligament, left triangular ligament and ligamentum teres. The lesser omentum runs from the
inferior surface of the liver to the lesser edge of the stomach and the first 2 cm of the
duodenum, forming the hepatogastric and hepatoduodenal ligaments, respectively. The left
lateral free edge of the lesser omentum encloses the portal triad (hepatic artery, portal vein,
bile duct) and lymph and nerve plexuses.

Question 63: FTTFF


The pancreas sits in the retroperitoneum, and stretches horizontally at level L2, across the
abdomen from the ‘C’ of the duodenum (the head) to the hilum of the spleen (the tail).
The pancreas lies on a bed formed by the duodenum, inferior vena cava, aorta, left renal
vessels, left kidney and spleen (from right to left). Posterior to the pancreas is the superior
mesenteric artery and the portal vein, formed by the splenic and superior mesenteric veins.
Anterior to the pancreas is the peritoneum.
The gastroduodenal artery (branch of common hepatic artery) gives branches to form an
arcade at the head of the pancreas with branches from the superior mesenteric artery.
The body and tail are supplied by multiple branches of the splenic artery (branch of the
coeliac trunk), which runs along the superior border of the pancreas to the spleen.
The pancreatic duct originates in the tail of the pancreas and runs in the body of the
pancreas to the head, where it drains into the second part of the duodenum with the
common bile duct at the sphincter of Oddi (hepatopancreatic sphincter). The pancreatic
duct receives a tributary, the accessory duct, which drains the head of the pancreas.

Downloaded from https://www.cambridge.org/core. University of Edinburgh, on 19 Aug 2019 at 13:21:27, subject to the Cambridge Core terms of
use, available at https://www.cambridge.org/core/terms. https://doi.org/10.1017/9781108566100.006
150 Chapter 2b: Anatomy Answers

The duodenum is largely fixed to the retroperitoneum and forms a C-shape, circling the
head of the pancreas. Despite its course running in the retroperitoneum, it starts at the
gastroduodenal junction, an intraperitoneal structure. The duodenum is divided into four
parts:
1. Superior, lying on the superior border of the head of the pancreas, forming the
gastroduodenal junction and starting intraperitoneally before travelling deep to meet
the head of the pancreas
2. Descending, lying lateral to the head of the pancreas and receiving the pancreatic and
bile ducts
3. Horizontal, lying at the inferior border of the head of the pancreas
4. Ascending, largely lying independent of the pancreas and attaching to the jejunum at the
duodenojejunal junction.

Question 64: TFTFT


There are three main pathologies afflicting the pancreas, leading to severe morbidity.
1. Cancer. Often discovered late due to the insidious onset and often after it has
metastasized. Pancreatic cancer has great propensity to spread via the haematogenous
route because of its venous drainage. The splenic vein drains the pancreas and forms the
portal vein with the superior mesenteric vein, which supplies 70% of the blood supply to
the liver. It is therefore very common for pancreatic tumours to metastasize to the liver.
They also commonly metastasize to bone, lungs and brain. Pancreatic tumours are often
only noted when the patient presents with painless jaundice caused by the extrahepatic
obstruction of the common bile duct, leading to retention of bile salts. Pain is a late
presentation of pancreatic cancer.
2. Pancreatic injury is common with acceleration/deceleration injuries, due to the
transverse placement of the pancreas over the lumbar vertebrae, which act as an anvil,
transecting the pancreas at the neck. When this occurs, the pancreatic duct is commonly
also divided, leading to release of the exocrine enzymes, autodigestion and pancreatitis.
The endocrine function of the pancreas is rarely affected, unless the pancreatitis is severe
enough to cause almost complete necrosis of the pancreas.
3. Pancreatitis. The most common causes in the Western world are gallstones and alcohol.
It leads to an inflammatory response of the pancreas, release of amylase and lipase and in
severe cases, autodigestion, necrosis and mortality.

Question 65: TTFTT


The coeliac artery originates from the aorta at L1. It supplies the entire foregut: the lower
part of the oesophagus, stomach, duodenum (proximal to the ampulla of Vater in
the second part of the duodenum), liver, biliary tree and pancreas.
The superior mesenteric artery originates from the aorta at L2. It supplies the mid-gut:
the duodenum distal to the ampulla of Vater, jejunum, ileum, right colon, appendix and
transverse colon (proximal to splenic flexure).
The inferior mesenteric artery originates from the aorta at L4. It supplies the hindgut:
descending colon (distal to the splenic flexure), sigmoid colon and upper third of the
rectum.
The internal iliac artery supplies the middle and lower thirds of the rectum.

Downloaded from https://www.cambridge.org/core. University of Edinburgh, on 19 Aug 2019 at 13:21:27, subject to the Cambridge Core terms of
use, available at https://www.cambridge.org/core/terms. https://doi.org/10.1017/9781108566100.006
Chapter 2b: Anatomy Answers 151

Question 66: TTTFF


The jejunum begins at the duodenojejunal flexure (DJ flexure) and runs in continuity
with the ileum to terminate at the ileocaecal valve. The small bowel is attached to the
posterior abdominal wall via the small bowel mesentery, which runs in an oblique line
from the left side of L2 to the right sacroiliac joint. There is no anatomical structure to
delineate the ileum and jejunum. It is approximately 6 m long, the jejunum making up
approximately 40%.
The arterial supply to the small bowel is entirely from the superior mesenteric artery,
which runs in the mesentery and divides to give multiple branches that form arcades.
Venous drainage is via the superior mesenteric vein, to the portal vein.

Question 67: FTTTF


The abdomen is covered by a superficial fascia only. This can be divided into two layers:
a fatty layer (Camper’s fascia) and a membranous layer (Scarpa’s fascia). The three flat
muscles form the lateral coverings of the abdominal wall. External oblique fibres pass
inferiomedially, the internal oblique fibres run superolaterally (at right angles to the
external oblique) and the transverse oblique runs horizontally. Anteriorly, the fibres of
these muscles fuse to form the rectus sheath. The rectus sheath contains the muscles rectus
abdominis and pyramidalis, superior and inferior epigastric vessels, lymphatics and the
ventral rami of T7–T12. In the mid-line, the rectus sheath forms the linea alba.
The anterior abdominal wall is supplied by the ventral rami of T7–L1, superior to
inferior. T10 innervates the dermatome of the umbilicus. The nerves run in a plane formed
by internal oblique and transversus abdominis – the transversus abdominis plane (TAP).

Question 68: TTTFF


The anterior abdominal wall is innervated by the ventral rami of T7–L1. The anterior
divisions of T7–T11 form the intercostal nerves and traverse the abdominal space between
the internal oblique and transversus abdominis muscles. The T12–L1 nerve roots form part
of the lumbar plexus and contribute to the formation of the iliohypogastric and ilioinguinal
nerves. These nerves supply sensation over the gluteal and hypogastric regions and also over
the anterior aspect of the thigh.
Surface anatomical landmarks used to place a TAP block in the absence of ultrasound
guidance include the ‘double pop’ technique in the triangle of Petit. This is bound anteriorly
by the external oblique, posteriorly by the latissimus dorsi and inferiorly by the iliac crest.
The superior tip is formed at the lower costal margin. There has been much debate as to the
efficacy of TAP blocks in providing a reliable source of analgesia. Most would argue that
spread of ~20 ml local anaesthetic deposited unilaterally will range from at least T10–L1.
Bilateral injections can therefore be used for lower-abdominal surgery performed in the
mid-line. Care should be taken as to the concentration of local anaesthetic used, so as not to
exceed maximum local anaesthetic doses.

Question 69: FTTTF


The abdomen is divided into a number of segments, by imaginary lines, to aid in the
description of anatomy.

Downloaded from https://www.cambridge.org/core. University of Edinburgh, on 19 Aug 2019 at 13:21:27, subject to the Cambridge Core terms of
use, available at https://www.cambridge.org/core/terms. https://doi.org/10.1017/9781108566100.006
152 Chapter 2b: Anatomy Answers

Horizontal planes are:


• L1, the transpyloric plane; approximately cuts through the pylorus of the stomach
• L3, the subcostal plane; passes through the lower border of the tenth costal cartilage
• L5, the transtubercular plane; passes through the iliac tubercles
Vertical planes are:
• Median plane; divides the body equally in two, through the xiphoid process, umbilicus
and pubic symphisis. It is the surface marking of the linea alba, in the mid-line and is the
insertion of the rectus sheath from one side to the other. The lateral line of the rectus
sheath is known as the linea semilunaris.
• Mid-clavicular plane; from the mid-point of the clavicle to the mid-inguinal point (an
imaginary line between the anterior superior iliac spine and the pubic symphysis – not to
be confused with the mid-point of the inguinal ligament).
The inguinal ligament runs from the anterior superior iliac spine to the pubic tubercle.
A point 1 cm superior to the mid-point of the inguinal ligament identifies the deep inguinal
ring and the superficial inguinal ring can be found 1 cm superolateral to the pubic tubercle.

Question 70: FTFFF


The inguinal ligament inserts into the anterior superior iliac spine laterally and the pubic
tubercle medially. The inguinal canal runs slightly superior to and parallel to the medial half
of the ligament. The deep inguinal ring is found superior to the mid-point of the inguinal
ligament and the superficial inguinal ring is found superolateral to the pubic tubercle.
In males the canal contains the spermatic cord, in females it contains the round ligament of
the uterus and in both sexes it contains the ilioinguinal nerve.
The contents of the spermatic cord include three arteries, three nerves, and three other
structures: the vas deferens, pampiniform plexus and lymph vessels.
The walls of the inguinal canal are formed as follows:
• Anterior: external oblique aponeurosis
• Posterior: transversalis fascia
• Floor: the inguinal ligament (and lacunar ligament medially)
• Roof: internal oblique and transversus abdominis
A direct inguinal hernia protrudes ‘directly’ through the abdominal wall, medial to the
inferior epigastric vessels and does not enter the inguinal canal. An indirect inguinal hernia
enters the deep inguinal ring, travels along the inguinal canal and exits the superficial
inguinal ring. When reduced, it protrudes lateral to the inferior epigastric vessels.
An ilioinguinal nerve block can be used to provide analgesia and even anaesthesia to allow
a hernia repair to be done without general anaesthetic. A landmark- or ultrasound-guided
technique is used to infiltrate local anaesthetic into the fascial layers, which allows the
ilioinguinal and iliohypogastric nerves to be blocked. A ’double pop’ techinique is used by
palpating the ASIS (anterior superior iliac spine) and advancing 2 cm medially and 2 cm
superiorly. A ‘fan’ subcutaneous infiltration allows superficial subcostal nerves to be
covered.

Downloaded from https://www.cambridge.org/core. University of Edinburgh, on 19 Aug 2019 at 13:21:27, subject to the Cambridge Core terms of
use, available at https://www.cambridge.org/core/terms. https://doi.org/10.1017/9781108566100.006
Chapter 2b: Anatomy Answers 153

Question 71: TTTTT


Visceral pain arises as a result of pathology from the intra-abdominal viscera. It is often
poorly localized and radiates to the part of the body supplied by the same dermatome as the
nerve that supplies the organ in question. Therefore, heart pain is often referred to the left
arm. Examples of similar phenomena in the abdomen are:
• Diaphragmatic pain/irritation is referred to the shoulder tips
• Liver/gall bladder pain is referred to the RUQ, back and can also be referred to the right
shoulder
• Small bowel pain is referred to the umbilicus
• Pancreatic pain is referred to the lumbar back
• Renal and ureteric pain is referred to the scrotum
• Gastric/duodenal pain is referred to the epigastrium
• Large bowel pain is referred to the LIF

Question 72: FTTTT


A coeliac plexus block can be performed to aid analgesia in patients with acute and chronic
pancreatitis. It has also been suggested in the chronic pain management of abdominal
malignancy of the pancreas, colon and stomach.
The coeliac plexus is made up of the coeliac ganglia, which lie on either side of the L1
vertebrae. The splanchnic nerves run from T10 down to T12.
The posterior approach to the block is performed under X-ray guidance; however, in
patients who are unable to tolerate this, the transabdominal method can be used. This
involves the use of CT and ultrasound guidance.

Question 73: FTTTT


Complications secondary to block insertion include:
• Orthostatic hypotension
• Bleeding
• Pleurisy
• Abscess/cyst formation
• Paraplegia
• Bowel dysfunction

Question 74: FFFTT


The sympathetic nervous system comprises preganglionic neurones which originate from
T1–L2/3. The nerve fibres leave the spinal cord via the anterior root and synapse in the
autonomic ganglia of the paired sympathetic trunks. Preganglionic nerve fibres are short,
myelinated fibres and release acetylcholine as their neurotransmitter. Postganglionic nerve
fibres are long, non-myelinated and release noradrenaline. Exceptions to these are the
sympathetic supplies to the sweat glands, skin and adrenal glands.
There are three cervical ganglia of which the inferior cervical and the first thoracic ganglia
are fused to form the stellate ganglion. There are 11–13 thoracic ganglia that form the
thoracic sympathetic trunk. Branches from here give rise to the pulmonary and cardiac

Downloaded from https://www.cambridge.org/core. University of Edinburgh, on 19 Aug 2019 at 13:21:27, subject to the Cambridge Core terms of
use, available at https://www.cambridge.org/core/terms. https://doi.org/10.1017/9781108566100.006
154 Chapter 2b: Anatomy Answers

plexuses as well as contributing to the coeliac plexus through the greater splanchnic nerve.
The lumbar sympathetic trunk consists of four ganglia, branches of which pass to the coeliac
and hypogastric plexuses.

Question 75: TFTFF


The parasympathetic nervous system (PSNS) innervates organs in an opposing manner to
the sympathetic nervous system. The preganglionic neurones are longer and synapse close
to the effector organ. Acetylcholine is released at both the pre- and postganglionic synapses.
The PSNS is composed of the cranial and sacral outflow. Cranial nerve III (CN III) supplies
the ciliary ganglion to innervate the eye. CN VII supplies the sphenopalatine and subman-
dibular ganglia to innervate the lacrimal, nasal, pharyngeal and salivary glands. CN IX
supplies the otic ganglion, which innervates the parotid gland. The anterior and posterior
vagal trunks supply organs within the thorax and abdomen up to the splenic flexure. The sacral
outflow is composed of S2–S4, which supply the pelvic viscera. The parasympathetic supply
allows for relaxation of the bladder and bowel sphincters.

Question 76: FFTTF


Explanations regarding cranial nerves (CN) I–XII will be spread across questions 75–78.
CN I: Olfactory nerve. Fibres from the nasal mucosa pass through the cribriform plate in
the ethmoid bone to reach the olfactory bulb.
CN II: Optic nerve. Fibres from the optic nerve travel in the optic canal to the middle
cranial fossa, where they decussate in the optic chiasm, anterior and superior to the
pituitary gland. The ophthalmic artery is contained within the optic canal. Lesions
anterior to the optic chiasm lead to a unilateral monocular blindness. Lesions at the optic
chiasm result in a bitemporal hemianopia and lesions posterior to the chiasm lead to
a contralateral homonymous hemianopia.
CN III: Oculomotor nerve. The nerve travels from the posterior cranial fossa close to the
tentorium cerebelli into the middle cranial fossa ascending lateral to the cavernous sinus.
It then divides into the superior and inferior divisions near the superior orbital fissure.
The parasympathetic fibres supply sphincter pupillae and the ciliary muscle.

Question 77: TFTFT


CN IV: Trochlear nerve. Travels close to CN III in the posterior and middle cranial fossae
and enters the orbit above the superior orbital fissure. It supplies the superior oblique
muscle.
CN VI: Abducens nerve. Originates in the pons and enters the orbit through the superior
orbital fissure to supply the lateral rectus muscle.
CN V: Trigeminal nerve. The nerve arises in the pons and travels to the trigeminal
ganglion, which is situated in Meckel’s cave in the floor of the petrous bone. This is also
known as the Gasserion ganglion. From the ganglion, the nerve gives out three branches,
hence its name: (i) ophthalmic, (ii) maxillary, (iii) mandibular. Each of the three branches
has further subdivisions, and each of the three branches supplies sensory innervation to
a discrete area over the face and scalp. Motor fibres are carried in the mandibular branch
only, and supply the muscles of mastication. The mandibular branch leaves the middle

Downloaded from https://www.cambridge.org/core. University of Edinburgh, on 19 Aug 2019 at 13:21:27, subject to the Cambridge Core terms of
use, available at https://www.cambridge.org/core/terms. https://doi.org/10.1017/9781108566100.006
Chapter 2b: Anatomy Answers 155

cranial fossa via the foramen ovale. One of the largest subdivisions of the mandibular
branch is the lingual nerve, which supplies sensation to the anterior two-thirds of the
tongue.
Herpes zoster can affect the trigeminal nerve, usually isolated to the cutaneous innerva-
tion of a single division. Herpes zoster ophthalmicus (or ophthalmic shingles) is a poten-
tially sight-threatening emergency.

Question 78: FTTTT


CN VII: Facial nerve. The facial nerve leaves the pons and emerges from the skull at the
stylomastoid foramen, where it divides into five branches: temporal, zygomatic, buccal,
mandibular and cervical. Within the skull, the greater petrosal nerve leaves the facial
nerve at the facial ganglion. The chorda tympani pass through the middle ear to join the
lingual nerve and parasympathetic fibres supply the submandibular gland.
CN VIII: Vestibulocochlear nerve. The vestibular nerve arises in the cerebellum to supply
the semi-circular canals. The cochlear nerve arises from the hearing area in the cerebrum
to supply the organ of Corti in the cochlea. The two components join in the internal
auditory meatus before entering the medulla.
CN IX: Glossopharyngeal nerve. The nerve passes through the jugular foramen with the
accessory and vagus nerves. It supplies sensory innervation to the posterior third of the
tongue. Parasympathetic fibres supply the parotid gland.

Question 79: FFTTT


CN X: Vagus nerve. The nerve leaves the medulla accompanied by CN IX and XI, and the
internal jugular vein. It travels through the foramen magnum. Within the carotid sheath
the vagus nerve descends posterior to the common carotid artery and lateral to the
internal jugular vein. After dividing into the anterior and posterior branches in the
thorax, it supplies the alimentary tract up to the splenic flexure.
CN XI: Accessory nerve. The cranial root supplies the muscles of the pharynx, larynx and
oesophagus. The spinal root supplies fibres to the sternocleidomastoid and trapezius
muscles. It ascends via the foramen magnum and emerges from the skull through the
jugular foramen.
CN XII: Hypoglossal nerve. Exits the skull through the hypoglossal canal and travels
posterior to the internal carotid artery and vagus nerve. It then passes forward over the
external carotid artery and the hypoglossus muscle to supply the intrinsic and extrinsic
muscles of the tongue.

Question 80: FTFTT


Branches of the facial nerve, CN VII:
• Temporal
• Zygomatic
• Buccal
• Mandibular
• Cervical

Downloaded from https://www.cambridge.org/core. University of Edinburgh, on 19 Aug 2019 at 13:21:27, subject to the Cambridge Core terms of
use, available at https://www.cambridge.org/core/terms. https://doi.org/10.1017/9781108566100.006
156 Chapter 2b: Anatomy Answers

Question 81: TFFTT


Parasympathetic nerve fibres are present in cranial nerves III, oculomotor, VII, facial, IX,
glossopharyngeal and X, vagus. The parasympathetic system has a craniosacral outflow, in
contrast to the thoracolumbar outflow of the sympathetic nervous system.

Question 82: TTTFT


The spinal vertebral bodies are connected by the anterior and posterior longitudinal
ligaments. Ligamentum flavum connects the laminae of adjacent vertebrae. Posterior to
this lies the interspinous ligament, which connects the shafts of the spines, behind which is
the supraspinous ligament, which connects the tips of the spines. Between the posterior
longitudinal ligament and the ligamentum flavum lies the spinal canal containing the spinal
cord. The epidural space is the area in between the dura and the walls of the canal. It is
a negative-pressure space, greatest in the thoracic region. The venous drainage within the
epidural space is valveless and prone to distension in the presence of an obstructed inferior
vena cava. The arterial system is smaller in comparison.
Anatomical variations may result in an epidural injection into the folds within the dura
mater, resulting in patchy analgesia, whereas injection into a potential subdural space may
result in abnormally prolonged blockade.

Question 83: TFTTF


The sacrum is a fusion of the five sacral vertebrae. The sacral canal is formed anteriorly by
the sacral vertebrae and posteriorly by the sacral laminae. Its length can be 10–15 cm and it
contains the dural sac with the cauda equina, CSF and meninges, the sacral nerves, coccygeal
nerves and venous blood supply. The internal vertebral venous plexus lies more anteriorly in
the sacral canal. At the base of the sacrum lies the sacral hiatus, which is an anatomical
failure in the fusion of the laminar arches of S5. The sacral spines of S4 form the cornu,
which encompasses the sacral hiatus. The sacral nerves from S5 and the coccygeal nerves
transverse the sacral hiatus, which is covered by a layer of sacrococcygeal membrane.

Question 84: TTFFF


The lumbar plexus is formed by the anterior rami of L1–L4 nerves. The plexus lies in front of
the transverse processes and behind the psoas muscle. The lumbar plexus gives rise to seven
main nerves, the largest of which is the femoral nerve.
The other nerves include:
• The ilioinguinal and iliohypogastric nerves arise from the L1 nerve root to supply the
abdominal muscles and the inguinal region.
• The genitofemoral nerve arises from L1–L2 nerve roots and supplies the skin of the
genitalia and femoral triangle.
• The lateral cutaneous nerve of the thigh arises from L2–L3 and enters the thigh medial to
the anterior superior iliac spine. It supplies the skin on the lateral aspect of the thigh.
• The femoral nerve arises from L2–L4 and passes close to the femoral artery to supply the
extensor muscles of the knee.
• The obturator nerve arises from L2–L4 and supplies the adductor muscles of the thigh.
• The lumbosacral trunk contributes to the sacral plexus from L4–L5.

Downloaded from https://www.cambridge.org/core. University of Edinburgh, on 19 Aug 2019 at 13:21:27, subject to the Cambridge Core terms of
use, available at https://www.cambridge.org/core/terms. https://doi.org/10.1017/9781108566100.006
Chapter 2b: Anatomy Answers 157

Question 85: TTTFT


The sacral plexus is formed from the anterior rami of S1–S3 and the lumbosacral trunk.
The largest nerve to originate from the plexus is the sciatic nerve, which travels in the
posterior aspect of the leg, supplying the hamstring muscles. Just above the level of the
popliteal fossa, the sciatic nerve divides into the tibial and common peroneal nerves.
The splanchnic nerves arise from S2–S4 and supply the internal anal and vesical sphincters.
The pudendal nerve arises from S2–S4 and supplies the perineal muscles, gluteus maximus
and also carries the parasympathetic fibres to the penis and clitoris.

Question 86: TFTFT


The skull has three sutures: coronal, sagittal and lamboid. The coronal suture separates the
frontal bone from the two parietal bones. The sagittal suture separates the two parietal bones
and the lamboid suture separates the parietal bones from the occipital bone. The junction at
which the coronal suture meets the sagittal suture is the bregma and is often referred to as
the anterior fontanelle in infants in whom the ossification is yet incomplete. The junction at
which the lamboid and sagittal sutures meet is called the lambda.
The frontal bone contains the supraorbital foramen and the superior orbital fissure.
The inferior orbital fissure and the infraorbital foramen are contained within the maxillary
and zygomatic bones. The zygomatic arch is formed by the zygomatic and temporal bones,
and lies above the external acoustic meatus. The temporal fossae are large depressions on
either side of the skull that lie deep to the zygomatic arches. Within the fossa lies the pterion.
This is a region of the skull where the frontal, temporal, parietal and sphenoid bones join
together, and as such is a weak point. The middle meningeal artery lies beneath the pterion,
and a head injury over this area may subsequently lead to an extradural haematoma.

Question 87: TTTTF


On the inferior aspect of the skull lie foramina that allow passage of the cranial nerves and
blood vessels. The foramen ovale lies behind the lateral pterygoid process and allows passage
of the mandibular nerve. Behind this lies the foramen spinosum through which passes the
middle meningeal artery. The carotid canal lies in the petrous bone in between the occipital
and sphenoid bones. The jugular foramen lies posteriorly to the carotid canal and contains
the internal jugular vein, as well as the glossopharyngeal, vagus and accessory nerves.
The facial nerve travels through the stylomastoid foramen located behind the styloid
process. The foramen magnum, which contains the medulla oblongata, is located in the
occipital bone.

Question 88: TTFFF


The brain is divided into three main sections: forebrain, midbrain and hindbrain.
The forebrain contains the two cerebral hemispheres separated by the longitudinal fissure
and divided into four lobes: frontal, parietal, temporal and occipital. The central sulcus
separates the frontal lobe from the parietal lobes. Anterior to the central sulcus lies the
precentral gyrus, which serves to provide motor function. The postcentral gyrus provides
sensory innervation. The temporal lobe lies below the lateral sulcus and contains the

Downloaded from https://www.cambridge.org/core. University of Edinburgh, on 19 Aug 2019 at 13:21:27, subject to the Cambridge Core terms of
use, available at https://www.cambridge.org/core/terms. https://doi.org/10.1017/9781108566100.006
158 Chapter 2b: Anatomy Answers

auditory area, whereas Broca’s area, concerned with motor speech, lies just above the lateral
sulcus. The calacrine sulcus in the occipital lobe contains the visual area.
Also in the forebrain is the diencephalon, which lies in between the two cerebral hemi-
spheres and contains the thalamus and hypothalamus.

Question 89: TFTFT


The midbrain connects the forebrain and the hindbrain. It consists of two cerebral pedun-
cles, which are divided into an anterior part, crus cerebri, and a posterior part, tegmentum.
Between the two lies the substantia nigra. Also within the midbrain is the cerebral aqueduct,
which connects the third and fourth ventricles.
The hindbrain (brainstem) contains the pons, medulla and the cerebellum. It also con-
tains the fourth ventricle.
The pons sits above the medulla on the anterior surface of the cerebellum. The medulla
oblongata connects the pons to the spinal cord and receives the majority of the descending
motor fibres from the precentral gyrus. The medulla contains a longitudinal fissure that
separates the two pyramids. Motor fibres decussate in the pyramids before descending further
into the spinal cord. The cerebellum in the posterior cranial fossa lies behind the pons and
medulla, just below the tentorium cerebelli. The two halves of the cerebellum are connected
by the vermis. The two hemispheres are divided into the superior, middle and inferior
cerebellar peduncles, which connect with the midbrain, pons and medulla respectively.

Question 90: TFFFT


Cerebrospinal fluid (CSF) is produced in the choroid plexus. The total volume of CSF is
approximately 150 ml with roughly 550 ml produced during a day. The two lateral ventricles
drain CSF into the third ventricle via the interventricular foramen, which is also known as
the foreman of Monro. CSF enters the fourth ventricle via the foramen of Sylvius and then
into the subarachnoid space through the foramen of Magendie, which lies medially, and the
foramen of Luschka, which lies laterally. CSF is reabsorbed into the venous sinuses, which
lie in the arachnoid villi.

Question 91: FTTTT


The circle of Willis forms the network of arteries supplying the brain. It is formed through
a union of two internal carotid arteries and two vertebral arteries. The internal carotid then
divides into the anterior and middle cerebral arteries. The middle cerebral artery is the
largest branch and supplies all of the lateral cortex. The two anterior cerebral arteries are
connected by the anterior communicating artery and supply the medial cortex. They also
supply the leg area in the motor precentral gyrus.
The vertebral arteries branch from the subclavian artery and join to form the basilar
artery. The two posterior cerebral arteries come off the basilar artery to supply the visual
cortex and the midbrain.
The venous drainage of the brain is via the venous sinuses located in the arachnoid villi.

Question 92: TTFTT


See Question 91 for explanation.

Downloaded from https://www.cambridge.org/core. University of Edinburgh, on 19 Aug 2019 at 13:21:27, subject to the Cambridge Core terms of
use, available at https://www.cambridge.org/core/terms. https://doi.org/10.1017/9781108566100.006
Chapter 2b: Anatomy Answers 159

Question 93: TFTTF


The spinal cord begins as part of an extension to the brainstem and extends down to the
conus medullaris. It contains an H-shaped section surrounding the central canal, which is
termed the grey matter: so called due to the presence of nerve cell bodies. The white matter is
divided into the posterior, anterior and lateral columns and contains ascending and
descending tracts connecting peripheral nerves to the brain.
Ascending tracts:
• Dorsal column: fine touch and proprioception. Fibres decussate in the medulla.
• Lateral spinothalamic tract: pain and temperature. Fibres decussate in the anterior white
commissure.
• Spinocerebellar tracts: Proprioception. Fibres do not decussate in the spinal cord.
Descending tracts:
• Pyramidal tract: Axons from the motor cortex descend and decussate in the medulla; 80%
form the lateral corticospinal tract and the rest form the anterior corticospinal tract.
• Reticulospinal, tectospinal, rubrospinal and vestibulospinal tracts arise from the pons,
medulla and midbrain.
Following a spinal cord injury, muscle denervation leads to the formation of extra
junctional receptors at the motor end plates. Administration of suxamethonium will lead
to an exaggerated response, secondary to the extra receptors, and lead to an unpredictable
rise in serum potassium levels, which can lead to arrhythmias and/or cardiac arrest.
The time period for this is generally noted to be after the first 24 hours of injury up to 18
months.

Question 94: FFFFT


Blood supply to the spinal cord is via the anterior and posterior spinal arteries. Radicular
arteries contribute to the anterior arterial supply. The largest of these is the radicularis magna,
which supplies the lower two-thirds of the spinal cord. The anterior and posterior spinal veins
drain blood into the azygos, lumbar and sacral veins, depending on their level of insertion.
The spinal meninges are the dura, arachnoid and pia mater. The dura originates at the
foramen magnum and ends at the end of the dural sac, typically at the level of S2 although it
can be at L5.
Within the brain the arachnoid mater contains of numerous villi that are involved in
venous drainage. CSF is reabsorbed into the arachnoid villi. The pia mater within the brain
is the innermost layer and extends into the sulci. Within the ventricles, the layers of the pia
mater fuse to form the choroid plexus. CSF is secreted from the choroid plexus.

Question 95: TTTTT


There are 31 pairs of spinal nerves divided into eight cervical, twelve thoracic, five lumbar,
five sacral and one coccygeal. The first spinal nerve arises between the occipital bone and the
atlas. Thereafter the nerves take the names of the vertebrae above. Spinal nerves are made up
of an anterior and posterior nerve root. The posterior root contains afferent sensory fibres,
whereas the anterior root has efferent motor fibres. The anterior root also receives fibres via
the rami communicantes, which attach to the sympathetic ganglion.

Downloaded from https://www.cambridge.org/core. University of Edinburgh, on 19 Aug 2019 at 13:21:27, subject to the Cambridge Core terms of
use, available at https://www.cambridge.org/core/terms. https://doi.org/10.1017/9781108566100.006
160 Chapter 2b: Anatomy Answers

As the posterior and anterior roots of the spinal nerve emerges from the intervertebral
foramina, the dura mater blends with the neurolemmal sheath of the nerve. There is no pia
mater covering the spinal nerves.

Question 96: TTFFF


A high spinal block is generally reported as a spinal block above the level of T1 with
compromising side effects.
These include:
• Hypotension
• Bradycardia
• Upper limb paraesthesia/paralysis, particularly in the hands
• Respiratory compromise leading to apnoea
• Loss of consciousness
• Cardiac arrest
Circumoral tingling is a sign of neurological irritability, secondary to local anaesthetic
toxicity.

Question 97: FFFFT


The brachial plexus is composed of roots, trunks, divisions, cords and branches (nerves).
The roots are formed from the ventral (anterior) rami of C5–T1. There may also be small
contributions from C4 and T2.
The roots of C5–C6 give rise to the superior trunk, C7 forms the middle trunk and C8–T1
form the inferior trunk.
Each trunk divides into an anterior and posterior division. The anterior divisions supply
the flexor muscles and the posterior divisions supply the extensor muscles of the arm.
The anterior divisions of the superior and middle trunk form the lateral cord.
The posterior divisions of all three trunks form the posterior cord. The anterior division
of the inferior trunk forms the medial cord. The cords are named according to their
relationship with the axillary artery.
The cords divide further to form the branches (nerves).

Question 98: TTFFT


The branches are the terminal nerves of the brachial plexus.
The lateral cord forms the:
• Lateral pectoral nerve
• Musculocutaneous nerve
• Lateral root to the median nerve
The posterior cord forms the:
• Axillary nerve
• Radial nerve
• Upper and lower subscapular nerves
• Thoracodorsal nerve

Downloaded from https://www.cambridge.org/core. University of Edinburgh, on 19 Aug 2019 at 13:21:27, subject to the Cambridge Core terms of
use, available at https://www.cambridge.org/core/terms. https://doi.org/10.1017/9781108566100.006
Chapter 2b: Anatomy Answers 161

The medial cord forms the:


• Medial root to the median nerve
• Ulnar nerve
• Medial cutaneous nerves of the arm and forearm
The long thoracic nerve is formed directly from the ventral rami of the roots C5–C7.

Question 99: TTTTT


The musculocutaneous nerve enters deep to the coracobrachialis and descends between the
biceps brachii and brachialis. It then descends down the lateral aspect of the forearm.
The axillary nerve accompanies the posterior circumflex humeral artery through the
quadrangular space before winding around the neck of the humerus.
The ulnar nerve runs down the medial aspect of the arm posterior to the medial
epicondyle before entering the forearm.
The radial nerve runs in the radial groove with the deep brachial artery between the long
and medial heads of the triceps.
The median nerve runs in the mid-line of the arm, anterior to the elbow joint down to the
wrist.
NB: The median and ulnar nerves do not give off branches above the elbow.

Question 100: FTTFF


The musculocutaneous nerve has motor innervation to the coracobrachialis, biceps brachii
and brachialis. The sensory innervation is supplied via the lateral cutaneous nerve of the
forearm to the lateral aspect of the forearm.
The axillary nerve has motor innervation to the teres minor, deltoid and shoulder joint.
It has sensory innervation to the skin overlying the deltoid.
The ulnar nerve has motor innervation to the flexor carpi ulnaris in the forearm and the
medial half of the flexor digitorum profundus. It also supplies most of the intrinsic muscles
of the hand. It has sensory innervation to the skin overlying the medial one and a half digits
The median nerve has motor innervation to the flexor muscles of the forearm except the
flexor carpi ulnaris and half of the flexor digitorum profundus. It also supplies the lateral
two lumbricals, opponens pollicis, abductor pollicis brevis and flexor pollicis brevis (LOAF).
It completes the sensory innervation to the palm of the hand.

Question 101: TTTFF


All the upper limb reflexes (biceps, triceps and brachioradialis) are spinal reflexes and there-
fore involuntary. They are examples of stretch (or myotatic) reflexes, in which muscle
contraction is stimulated by stretching within that muscle. They are mediated by
a monosynaptic reflex pathway. The biceps reflex is mediated by the cord segments C5–C6
(musculocutaneous nerve) and elicits the biceps jerk. The brachioradialis reflex is mediated by
cord segments C5–C6 (musculocutaneous nerve) and elicits a supinator jerk. The triceps
reflex is mediated by cord segments C6–C7 (radial nerve) and elicits a triceps jerk.

Question 102: TTFTF


Dermatomes of the arm range from C4–T2 (Figure 2.102.1):

Downloaded from https://www.cambridge.org/core. University of Edinburgh, on 19 Aug 2019 at 13:21:27, subject to the Cambridge Core terms of
use, available at https://www.cambridge.org/core/terms. https://doi.org/10.1017/9781108566100.006
Supraclavicular
C3–C4

Axillary (superior
lateral cutaneous)
Radial, dorsal
C5–C6
antebrachial cutaneous
C5–C6 Intercostobrachial T2
Branches of radial
C5–C6
Medial brachial
cutaneous
Lateral antebrachial
T1–2
cutaneous
(musculocutaneous) Lateral antebrachial
C5–C6 cutaneous
Medial antebrachial
(musculocutaneous)
cutaneous
C5–C6
C8–T1
Radial
Radial superficial
superficial Ulnar C6–C8
C6–C8 C8–T1

Median
C5–C8

use, available at https://www.cambridge.org/core/terms. https://doi.org/10.1017/9781108566100.006


Median
C5–C8

Figure 2.102.1 Innervation of the upper limb.

Downloaded from https://www.cambridge.org/core. University of Edinburgh, on 19 Aug 2019 at 13:21:27, subject to the Cambridge Core terms of
Chapter 2b: Anatomy Answers 163

• C4 covers the medial aspect of the deltoid.


• C5 supplies the anterior aspect, lateral to the median line of the upper arm and forearm.
• C6 supplies the lateral aspect of the arm, including the thumb.
• C7 innervates the medial, posterior aspect of the upper arm, forearm and hand, including
the index and middle fingers anteriorly.
• C8 innervates the medial aspect of the arm, including the ring and little fingers.
• T1 innervates the medial aspect of the anterior surface of the arm.
• T2 innervates the axilla.

Question 103: FFFTT


The radial nerve supplies the lateral aspect of the elbow and the lateral half of the dorsum of
the hand, excluding the finger tips.
The ulnar nerve supplies the medial aspect of the hand, including the little finger and the
medial half of the ring finger. The rest of the hand is supplied by the median nerve.
The axillary nerve supplies the lateral aspect of the upper arm via the upper lateral
brachial cutaneous nerve.
The intercostobrachial nerve supplies the axilla.

Question 104: FFTTF


The axillary artery originates from the subclavian artery at the lateral border of the first rib.
It passes deep to the pectoralis minor, which divides the artery into three parts: first, second
and third. It continues to form the brachial artery, which originates at the inferior border of
the teres major. The brachial artery runs superficially and medially in the upper arm before
it divides to form the radial and ulnar arteries deep to the bicipital aponeurosis (antecubital
fossa).
The radial artery runs laterally and ends to form the deep palmar arch with the deep
branch of the ulnar artery. The ulnar artery runs medially and ends by forming the super-
ficial palmar arch with the deep branch of the radial artery.

Question 105: TFFTF


This is a tough question. If you got these correct, you’re above the rest. If you got some
wrong, do not be disillusioned, here is the explanation.
The axillary artery has six main branches:
• Superior thoracic artery
• Thoracoacromial artery
• Lateral thoracic artery
• Subscapular artery
• Anterior circumflex humeral artery
• Posterior circumflex humeral artery
It forms the brachial artery at the inferior border of the teres major. The brachial artery is
the main blood supply to the arm. It accompanies the median nerve in the upper arm, which
runs anterior to it and gives off three main branches:
• Deep brachial artery
• Superior ulnar collateral artery

Downloaded from https://www.cambridge.org/core. University of Edinburgh, on 19 Aug 2019 at 13:21:27, subject to the Cambridge Core terms of
use, available at https://www.cambridge.org/core/terms. https://doi.org/10.1017/9781108566100.006
164 Chapter 2b: Anatomy Answers

• Inferior ulnar collateral artery


The ulnar collateral arteries form the arterial anastomosis around the elbow.
The brachial artery divides to form the radial and ulnar arteries at the bicipital aponeuro-
sis. The ulnar artery runs medially in the forearm and lateral to the ulnar nerve. The radial
artery runs medially to the superficial branch of the radial nerve.

Question 106: TFTTT


The venous drainage of the upper limb can be divided into deep veins and superficial veins.
The venous drainage of the deeper structures is via the brachial veins (an example of venae
comitantes – veins which run alongside arteries). Superficial drainage is predominantly via
two large veins: the cephalic vein (lateral) and basilica vein (medial). The cephalic and
basilic veins are connected by the median cubital vein in the antecubital fossa. The axillary
vein is formed by the union of the basilic vein and brachial veins at the inferior border of the
teres major, and lies medial to the axillary artery. The superficial and deep drainage systems
are connected by perforator veins.

Question 107: TFFFT


The cephalic and basilic veins originate from the dorsal venous arches in the hand.
The cephalic vein runs superficially along the lateral border of the arm and drains into
the axillary vein at the deltopectoral triangle. The basilic vein runs superficially along the
medial aspect of the forearm. At the mid-upper arm, it transcends deep before joining the
deep brachial veins to form the axillary vein. The basilic vein lies superficial to the brachial
artery in the antecubital fossa. The median cubital vein connects the cephalic and basilic
veins in the antecubital fossa.

Question 108: FTFTF


Lymphatic drainage in the upper limb follows the venous system. There are five groups of
axillary lymph nodes. (The pectoral, subscapular, lateral and central groups lie deep to the
pectoralis minor. The apical group lies superior to the pectoralis minor. This degree of
knowledge is probably not needed for the FRCA!) Patients do develop lymphadenopathy
following axillary lymph node clearance, but it is secondary, not primary. Lymphoma may
present with axillary lymphadenopathy, but is usually bilateral.

Question 109: TFFTT


The axilla is a pyramidal space between the upper limb and the thorax.
• The apex is a point between the first rib, clavicle and superior edge of subscapularis muscle
• The lateral wall is the intertubercular groove of the humerus
• The medial wall is formed by the first four ribs and the muscles overlying them (serratus
anterior and intercostal muscles)
• The floor/base is formed by skin
• The anterior wall is formed by the pectoralis major and minor
• The posterior wall is formed by the scapula and subscapularis, teres major and latissimus
dorsi muscles.

Downloaded from https://www.cambridge.org/core. University of Edinburgh, on 19 Aug 2019 at 13:21:27, subject to the Cambridge Core terms of
use, available at https://www.cambridge.org/core/terms. https://doi.org/10.1017/9781108566100.006
Chapter 2b: Anatomy Answers 165

The axilla contains the axillary artery and vein, the brachial plexus, fat, lymph nodes and
sympathetic plexus. The artery, vein and cords of the brachial plexus are enclosed in a fascial
covering: the axillary sheath.

Question 110: FTFFF


The superficial aspect of the brachial plexus is commonly injured when there is wide
separation of the neck from the shoulder as can occur in motorcycle injuries or during
parturition. This characteristically leads to the ‘waiter’s tip position’ where the arm hangs by
the side. Compression of the ulnar nerve can occur at varying points along the arm. It can
lead to partial or complete denervation of the medial two lumbricals resulting in a claw hand
appearance. A more proximal nerve injury leads to denervation of the flexor digitorum
profundus, which leads to a decreased appearance of the deformity. This is termed the ulnar
paradox, as a more debilitating condition causes a less deformed appearance. Radial nerve
injury causes wrist drop and weakness of finger extension. Carpal tunnel syndrome is
caused by compression of the median nerve and results in loss of sensory function prior
to motor function.

Question 111: TFTFT


The biceps brachii has two proximal heads. The short head inserts at the tip of the coracoid
process of the scapula and the long head inserts at the supraglenoid tubercle of the scapula.
The distal insertion is at the tuberosity of the radius and fascia of the forearm via the
bicipital aponeurosis. It is supplied by the musculocutaneous nerve (C5–C6) and flexes and
supinates the forearm.
The triceps brachii has three proximal heads. The long head inserts at the infraglenoid
tubercle of the scapula. The lateral and medial heads insert at the posterior surface of the
humerus. The distal insertion is at the olecranon of the ulna. It is supplied by the radial
nerve (C6–C8) and extends the forearm.

Question 112: FFTFF


The flexor compartment of the forearm is divided into two groups, superficial and deep.
1. The superficial group contains:
• Pronator teres
• Flexor carpi radialis
• Palmaris longus
• Flexor carpi ulnaris
• Flexor digitorum superficialis
2. The deep group contains:
• Flexor digitorum profundis
• Flexor pollicis longus
• Pronator quadratus
The extensor compartment of the forearm is divided into three groups:

Downloaded from https://www.cambridge.org/core. University of Edinburgh, on 19 Aug 2019 at 13:21:27, subject to the Cambridge Core terms of
use, available at https://www.cambridge.org/core/terms. https://doi.org/10.1017/9781108566100.006
166 Chapter 2b: Anatomy Answers

1. Muscles that act on the thumb:


• Abductor pollicis longus
• Extensor pollicis longus
• Extensor pollicis brevis
2. Muscles that act on the digits:
• Extensor digitorum
• Extensor indicis
• Extensor digiti minimi
3. Muscles that act at the wrist joint:
• Extensor carpi radialis longus
• Extensor carpi radialis brevis
• Extensor carpi ulnaris
The intrinsic muscles of the hand are divided into four groups:
1. Thenar muscles (opposition of the thumb):
• Abductor pollicis brevis
• Flexor pollicis brevis
• Opponens pollicis
2. Adductor pollicis (adduction of the thumb)
3. Hypothenar muscles (move the little finger):
• Opponens digiti minimi
• Flexor digiti minimi
• Abductor digiti minimi
4. Short muscles of the hand:
• Lumbricals (flex digits at MCP joint and extend at interphalangeal joint)
• Interossei (adduct the digits)
A handy mnemonic for the above is All For One And One For All:
A: Abductor pollicis brevis
F: Flexor pollicis brevis
O: Opponens pollicis brevis
A: Adductor pollicis
O: Opponens digiti minimi
F: Flexor digiti minimi
A: Abductor digiti minimi

Question 113: TTFTF


The antecubital fossa is a downward-pointing, triangular-shaped area on the flexor surface
of the elbow. It has three borders:
• Superior: an imaginary horizontal line connecting the humeral epicondyles
• Medial/ulnar: lateral border of pronator teres

Downloaded from https://www.cambridge.org/core. University of Edinburgh, on 19 Aug 2019 at 13:21:27, subject to the Cambridge Core terms of
use, available at https://www.cambridge.org/core/terms. https://doi.org/10.1017/9781108566100.006
Chapter 2b: Anatomy Answers 167

• Lateral/radial: medial border of brachioradialis


The roof is composed of deep fascia, superficial to which lie the median cubital vein and
medial cutaneous nerve of the forearm. Lateral to the antecubital fossa lie the cephalic vein
and the lateral cutaneous nerve of the forearm. Medially to the antecubital fossa lies the
basilic vein. The structures located within the antecubital fossa are the median nerve,
brachial artery, tendon of biceps and the radial nerve.

Question 114: FFTFF


A supraclavicular nerve block is achieved at the level of the trunks and divisions. It provides
anaesthesia for the arm and forearm, but not for shoulder surgery. However, in procedures
requiring the use of an upper arm tourniquet, a separate injection will need to be made to
block the intercostobrachial nerve, which is not part of the brachial plexus. Using
a landmark technique with a nerve stimulator, the incidence of pneumothorax is approxi-
mately 5%. With an ultrasound technique, the incidence is lower, as there should be direct
visualization of the needle tip.

Question 115: FTTFF


Infraclavicular blocks are more reliable in providing anaesthesia for forearm, wrist and
hand surgery. They can be used as a mode to provide continuous infusion by insertion of
a catheter. However, there is a high risk of phrenic nerve palsy at approximately 25%.
The risk of pneumothorax is approximately 1%.

Question 116: TTFTT


The interscalene block is used to provide anaesthesia for shoulder and proximal arm
surgery. It does not reliably block the C8–T1 nerve roots and therefore cannot be used for
hand surgery. The injection is placed between the anterior and middle scalenus muscles.
Common complications associated with interscalene block include:
• Horner’s syndrome – 50–75%
• Hoarseness – 30%
• Phrenic nerve palsy – 10%

Question 117: TTTFT


The axillary nerve block provides anaesthesia for forearm, wrist and hand surgery.
The axillary sheath contains the lateral, medial and posterior cords, and it is just beyond
this where the axillary block is performed. The musculocutaneous nerve is not part of the
sheath and therefore is often missed at injection. Inadequate anaesthesia of this will lead to
pain over the radial aspect of the arm and tourniquet pain. Due to the close proximity of the
cords around the axillary artery, inadvertent vascular puncture leading to haematoma is the
most common complication.

Question 118: TTFTF


Median nerve injury above the elbow will have different signs and symptoms to that of
injury more distally on the arm.

Downloaded from https://www.cambridge.org/core. University of Edinburgh, on 19 Aug 2019 at 13:21:27, subject to the Cambridge Core terms of
use, available at https://www.cambridge.org/core/terms. https://doi.org/10.1017/9781108566100.006
168 Chapter 2b: Anatomy Answers

Motor signs are:


• Loss of pronation of the forearm
• Loss of flexion at the wrist
• Loss of thumb abduction and opposition
Sensory signs:
• Loss of sensation over the thenar eminence and over the lateral three and half digits

Question 119: FTFTF


Radial nerve injury can cause the following signs and symptoms:
• Motor signs:
• Loss of extension in the forearm, wrist and fingers
• Difficulty in supinating the forearm
• Sensory signs:
• Loss of sensation over the dorsum of the hand particularly the lateral aspect

Question 120: TFFFF


Injury to the ulnar nerve at the wrist can cause:
• Motor signs:
• Claw hand
• Inability to flex the fourth and fifth digits
• Sensory signs:
• Loss of sensation over the hypothenar eminence
• Paraesthesia in the medial one and a half digits
Injury to the ulnar nerve more proximally in the arm leads to a less pronounced claw
hand due to innervation from the flexor digitorum profundus. This is known as the ulnar
paradox.

Question 121: FTFTF


The superficial fascial layer is mainly composed of loose connective tissue and fat, and is
continuous with Scarpa’s fascia in the abdomen.
The deep layer (called the fascia lata in the thigh and the crural fascia in the leg) attaches
to the inguinal ligament, pubic arch, body of pubis and pubic tubercle superiorly, the iliac
crest laterally and the sacrum, coccyx, sacrotuberous ligament, ischial tuberosity and iliac
crest posteriorly. Laterally, it also forms a tendinous tract, the iliotibial tract, which connects
the iliac tubercle to the lateral condyle of the tibia. Distally, the fascia lata attaches to the
knee and is continuous with the crural fascia. Medially, the saphenous opening allows for
passage of the long saphenous vein to drain into the femoral vein.
The crural fascia is a continuation of the fascia lata, distal to the knee, and forms the superior
and inferior extensor retinacular at the ankle. It allows for the proximal attachments of muscles.

Downloaded from https://www.cambridge.org/core. University of Edinburgh, on 19 Aug 2019 at 13:21:27, subject to the Cambridge Core terms of
use, available at https://www.cambridge.org/core/terms. https://doi.org/10.1017/9781108566100.006
Chapter 2b: Anatomy Answers 169

Question 122: FFTTF


The thigh is divided by the intermuscular septa, from the deep aspect of the fascia lata, into
anterior (hip flexors and leg extensors), posterior (hip extensors and leg flexors) and medial
(adductors) compartments by attaching onto the linea aspera of the femur.
The leg is divided into three compartments by the crural fascia attaching onto the fibula:
the anterior compartment (leg extensor and foot flexor), the lateral compartment and the
posterior compartment (leg flexor and foot extensor). The posterior compartment is further
divided into superficial and deep compartments.

Question 123: TFTTF


There are three groups of thigh muscles, plus the gluteal muscles.
The gluteal muscles are divided into two (the larger, more superficial muscles and the
deeper smaller muscles). The larger muscles are the maximus, medius and minimus, which
extend and abduct the thigh. The smaller, deeper muscles (piriformis, obturator internus,
gemelli and quadratus femoris) externally rotate the thigh.
The anterior thigh contains the iliopsoas, tensor fascia latae, pectineus, sartorius and
quadriceps femoris, and flex the hip, abduct, externally rotate the hip and extend the knee.
The medial thigh contains the adductor longus, adductor brevis, adductor magnus,
gracilis and obturator externus and adduct, flex and internally rotate the thigh.
The posterior thigh contains semitendinosus, semimembranosus and biceps femoris and
extend the thigh and flex the leg.

Question 124: TTTTT


There are three compartments in the leg, anterior, lateral and posterior; the last is divided
into superior and deep.
The anterior compartment contains the tibialis anterior, extensor hallucis longus, exten-
sor digitorum longus and fibularis. It dorsiflexes and inverts the foot and extends the toes.
The fibularis also everts the foot.
The lateral compartment contains the fibularis longus and brevis, which evert the foot.
The posterior compartment is divided into two compartments by the transverse septum.
The superficial posterior compartment contains the gastrocnemius, soleus and plantaris,
which plantar flex the ankle and flex the knee. The common insertion is via the Achilles
tendon. The deep compartment contains the popliteus, flexor hallucis longus, flexor digi-
torum longus and tibialis posterior. The muscles in this compartment flex the toes and
ankles, and also invert the foot.

Question 125: TFTTF


The femoral triangle is a triangular space in the superiomedial thigh that allows passage of
the femoral canal, vein, artery and nerve (medial to lateral) and lymph nodes. The triangle is
bounded by:
• Inguinal ligament (superior)
• Adductor longus (medial)
• Sartorius (lateral)
• Iliopsoas, pectineus, adductor longus (floor)

Downloaded from https://www.cambridge.org/core. University of Edinburgh, on 19 Aug 2019 at 13:21:27, subject to the Cambridge Core terms of
use, available at https://www.cambridge.org/core/terms. https://doi.org/10.1017/9781108566100.006
170 Chapter 2b: Anatomy Answers

• Fascia lata (roof).


The femoral sheath (a distal continuation of the transversalis fascia) bounds the artery
and vein, but not the nerve, and distally becomes continuous with loose connective tissue of
the thigh. The femoral canal allows expansion of the femoral vein, but is clinically impor-
tant, as it is the site a femoral hernia can be palpable.

Question 126: FTTFT


The motor and cutaneous nerve supply to the thigh arises from the femoral (L1–L4) and
sacral (S1–S3) plexuses.
Motor supply:
• Femoral nerve (L2–L4): muscles of the anterior thigh and the hip and knee joints.
• Sciatic nerve (L4–L3): muscles of the posterior thigh and the hip and knee joints.
• Obturator nerve (L3–L4): muscles of the medial thigh.
Cutaneous supply:
• Genital branch of the genitofemoral nerve (L1–L2): scrotum/labia majora.
• Ilioinguinal (L1) and femoral branches of the genitofemoral nerve (L1–L2): skin over the
femoral triangle. N.B. This is not THE femoral nerve.
• Femoral nerve (L2–L4): anteromedial thigh.
• Posterior femoral cutaneous nerve (S2–S3): buttock and posterior thigh.
• Lateral femoral cutaneous nerve (L2–L3): anterolateral thigh.
• Medial femoral cutaneous nerve (L2, branch of femoral nerve): anteromedial thigh.
N.B. The sciatic and obturator nerves do not have sensory divisions in the thigh.

Question 127: TTFTT


In human anatomy, the ‘leg’ usually refers to the lower limb below the knee. The limb above
the knee is termed the ‘thigh’. All motor and sensory supply to the leg is supplied by the
femoral and sciatic nerves.
The femoral nerve:
• Does not supply any muscles distal to the knee
• Supplies cutaneous innervation to the medial aspect of the leg and foot via the saphenous
nerve
The sciatic nerve:
• Supplies the innervation of all muscles in the leg and foot and the rest of the cutaneous
innervation
• Divides to give off the:
• tibial nerve (which supplies all muscles in the posterior compartment of the leg)
• common peroneal nerve (also known as common fibular nerve)
• The common peroneal nerve is therefore the cutaneous branch of the sciatic nerve, as
well as supplying the anterior muscles of the leg (via the superficial and deep peroneal
nerves)

Downloaded from https://www.cambridge.org/core. University of Edinburgh, on 19 Aug 2019 at 13:21:27, subject to the Cambridge Core terms of
use, available at https://www.cambridge.org/core/terms. https://doi.org/10.1017/9781108566100.006
Chapter 2b: Anatomy Answers 171

Question 128: FTTTF


The arterial supply of the lower limb arises from the femoral artery (via the external iliac
artery) and the obturator artery (via the internal iliac).
The femoral artery is formed from the external iliac artery, passes inferior to the inguinal
ligament and enters the femoral triangle. At the distal point of the femoral triangle the artery
divides to give off the superficial and deep femoral arteries. The superficial femoral artery
supplies the anterior compartment of the thigh and terminates at the adductor hiatus.
The deep femoral artery passes inferiorly, supplying the posterior compartment of the thigh
and the gluteal region, and the hip and knee joints via the lateral and medial circumflex
femoral arteries.
The obturator artery, a branch of the internal iliac artery, supplies the medial compart-
ment of the thigh, including the adductors.

Question 129: FTFTT


The entire supply of the leg arises from the superficial femoral artery as its continuation, the
popliteal artery, which is formed at the adductor hiatus. It passes through the popliteal fossa
and at the lower border of the popliteus divides into the anterior and posterior tibial arteries.
The anterior tibial artery supplies the anterior compartment in the leg before descending into
the foot to form the dorsalis pedis. The posterior tibial artery supplies the posterior and lateral
compartments of the leg before forming the plantar arches of the foot. It gives off the peroneal
artery on its descent, which also supplies the posterior and lateral compartments of the leg.

Question 130: TFFFT


There are two venous systems that drain the lower limb: the deep and superficial venous
systems.
The deep system runs deep to the deep fascia and continues to form the external iliac vein.
It is formed in the foot and mirrors the arterial supply in the leg, to form the popliteal vein in
the popliteal fossa and the femoral vein proximal to this. The femoral vein travels deep to the
sartorius, accompanied by the femoral artery. It becomes the external iliac vein as it travels
deep to the inguinal ligament.
The superficial system runs superficially to the deep fascia. The great saphenous vein is
formed in the foot, by the dorsal venous arch, travels anterior to the medial malleolus,
caudally passing approximately one hand’s breadth posterior to the patella and draining into
the femoral vein at the saphenous opening. The lesser saphenous vein travels posterior to the
lateral malleolus, arching posteriorly to drain into the popliteal vein at the popliteal fossa.
The superficial and deep venous systems connect at these two points, but also at a number of
points along the length of both systems, via perforators connecting the two systems.
Lymph drainage of the lower limb follows the venous drainage via popliteal and super-
ficial and deep inguinal lymph nodes.

Question 131: FTTFF


The popliteal fossa is a diamond-shaped fossa, posterior to the knee.
The boundaries of the popliteal fossa are:
• Superior: biceps femoris (laterally) and semimembranosus and semitendinosus (medially)

Downloaded from https://www.cambridge.org/core. University of Edinburgh, on 19 Aug 2019 at 13:21:27, subject to the Cambridge Core terms of
use, available at https://www.cambridge.org/core/terms. https://doi.org/10.1017/9781108566100.006
172 Chapter 2b: Anatomy Answers

• Inferior: gastrocnemius
• Posterior (roof): deep fascia and skin
• Anterior (floor): posterior surface of femur
The popliteal fossa contains:
• Popliteal artery (and branches)
• Popliteal vein (and branches) centrally located
• Tibial nerve (posteriolaterally)
• Common fibular nerve, along superolateral border of fossa
• Small saphenous vein, which drains into the popliteal vein

Question 132: FFFFT


The ankle is a hinge-joint, formed by the distal tibia, fibula and talus. It is therefore able to
dorsi- and plantar flex, but also rotate, abduct and adduct. Its blood supply arises from the
malleolar branches of the fibular, and the anterior and posterior tibial arteries. Its nerve
supply is via the tibial and deep fibular nerves. The joint capsule is thin and weak and is
therefore supported by strong medial and lateral fibrous ligaments. The medial ligament,
the deltoid, attaches onto the distal tibia and forms a fan-shape by inserting into the talus,
calcaneus and navicular. There are three individual lateral ligaments: the anterior and
posterior talofibular ligaments and the calcaneofibular ligament.

Question 133: TTFTF


The foot has two sets of arches, perpendicular to each other, which form an ‘H’ shape,
allowing an even spread of forces throughout the foot.
The longitudinal arch is composed of medial and lateral arches. The medial arch is
composed of the calcaneus, talus, navicular, three cuneiforms and three metatarsal bones.
The lateral arch is composed of the calcaneus, cuboid and lateral two metatarsals.
The transverse arch is formed by the cuboid, cuneiforms and bases of the metatarsals.

Question 134: FFTTF


1. Sabtalar joint: between the talus and the calcaneus (not a component of the ankle joint)
and is involved with inversion and eversion of the foot
2. Talocalcaneonavicular joint: between the talus, calcaneus and navicular and is involved
with rotation
3. Calcaneocuboid joint: between the calcaneus and the cuboid, and also involved with
inversion and eversion
4. Tarsometatarsal joints: between the tarsal and its respective metatarsal, involved in
gliding over each other
5. Intermetatarsal joints: between the respective metatarsals, with little movement at the
joints
6. Metatarsophalangeal joints: between the heads of the metatarsals and their respective
phalanges, involved with flexion, extension, abduction, adduction and rotation of the toes
7. Interphalangeal joints: between the phalanges and involved with flexion and extension
of the toes

Downloaded from https://www.cambridge.org/core. University of Edinburgh, on 19 Aug 2019 at 13:21:27, subject to the Cambridge Core terms of
use, available at https://www.cambridge.org/core/terms. https://doi.org/10.1017/9781108566100.006
Chapter 2b: Anatomy Answers 173

Question 135: TFFFT


The muscles of the foot are divided into four muscular layers from superficial to deep and
three longitudinal compartments: medial, central and lateral. The muscles maintain the
arches of the foot, but individually have little fine control over the movement of the toes.
They act as groups to move the toes in groups, only the first and fifth digits have any
significant movement individually.
The four layers of muscles provide two planes, between the first and second layers, and
the third and fourth layers, through which the pedal vessels and nerves pass.

Question 136: TFTFT


The anterior tibial artery continues to form the dorsalis pedis (DP) artery, which is formed
at the mid-point of the anterior intermalleolar line. It runs to the first interosseus space,
where it divides to form the deep plantar arch and gives off the first dorsal interosseous
artery. The deep plantar arch joins the lateral plantar arch to form a circular blood supply to
the foot.
The lateral plantar arch derives from the posterior tibial artery, which supplies the
majority of the blood to the foot. It runs posterior to the medial malleolus and divides to
form the medial and lateral plantar arteries, which supply the digits via the plantar digital
arteries.

Question 137: FFTFF


The motor supply to the foot is entirely from the tibial nerve, except for the extensor
digitorum brevis, which is supplied by the deep peroneal nerve.
The sensory supply to the foot arises from:
• Femoral nerve, via the saphenous nerve to the medial side of the foot
• Common fibular nerve, via:
• Superficial fibular nerve to all skin on dorsum of foot except lateral aspect and first
web space
• Deep fibular nerve to first web space
• Tibial nerve, via:
• Medial plantar nerve to medial side of the sole of the foot, including medial three and
a half digits
• Lateral plantar nerve to lateral side of the sole of the foot, including lateral one and
a half digits
• Sural nerve (combination of tibial and common peroneal nerves) to lateral side of foot
• Calcaneal branches (combination of tibial and sural nerves) to the heel of the foot

Question 138: TTTFT


There are two spinal reflexes that can be tested in the lower limb.
1. The knee jerk (patellar reflex): assesses spinal roots L2–L4 and the anterior thigh
muscles, in their ability to extend the lower limb at the knee when it is flexed.

Downloaded from https://www.cambridge.org/core. University of Edinburgh, on 19 Aug 2019 at 13:21:27, subject to the Cambridge Core terms of
use, available at https://www.cambridge.org/core/terms. https://doi.org/10.1017/9781108566100.006
174 Chapter 2b: Anatomy Answers

2. The ankle reflex: assesses spinal roots S1 and S2 and the posterior leg muscles, in their
ability to plantar flex the foot at the ankle when it is dorsiflexed.
The grading system for assessing spinal reflexes is from 0–4:

0 – no response, always abnormal


1 – minimal response, occasionally abnormal
2 – brisk response, normal
3 – very brisk response, occasionally abnormal
4 – clonus, always abnormal

Question 139: TTTTT


Dermatomes L1–S5 supply the lower limb, originating at the inguinal ligament and ending
at the anus.
To test for the specific dermatomes, the following areas can be assessed:
• L1, inguinal ligament
• L2, anterior medial thigh
• L3, medial epicondyle of the femur
• L4, medial malleolus
• L5, over dorsalis pedis
• S1, lateral aspect of calcaneus
• S2, popliteal fossa
• S3, ischial tuberosity
• S4 and S5, perineum

Question 140: TTTFT


The popliteal fossa is bordered as follows:
• Medial border: semimembranosus and semitendinosus
• Lateral border: biceps femoris
• Lower border: heads of gastrocnemius
• Superior cover: fascia lata
Structures present within the fossa include:
• Popliteal artery and vein, which lie anteromedially to the common peroneal nerve
• Sciatic nerve divides within the fossa into the tibial and the common peroneal nerves.

Question 141: TFTTT


A popliteal nerve block will affect the sciatic nerve in the popliteal fossa. When administered
proximally enough it will block the sciatic nerve prior to its subdivision into the tibial and
common peroneal nerves. This will therefore affect all motor and sensory innervation to the
lower leg, ankle and foot, except for a medial strip on the lower leg. This is innervated by the
saphenous nerve, which is a branch of the femoral nerve and therefore will need blocking
independently.
If the block is administered too distally and does not target the tibial nerve, plantar flexion
of the foot will be preserved.

Downloaded from https://www.cambridge.org/core. University of Edinburgh, on 19 Aug 2019 at 13:21:27, subject to the Cambridge Core terms of
use, available at https://www.cambridge.org/core/terms. https://doi.org/10.1017/9781108566100.006
Chapter 2b: Anatomy Answers 175

Question 142: TTTTF


An ankle block covers the following nerves:
• Deep nerves: posterior tibial and deep peroneal
• Superficial nerves: superficial peroneal, sural and saphenous
All nerves except the saphenous nerve are branches of the sciatic nerve. The saphenous
nerve descends from the femoral nerve.

Question 143: TFFTT


A 3-in-1 block of the lower limb targets the:
• Femoral nerve
• Lateral cutaneous nerve of the thigh
• Obturator nerve
It is useful in providing anaesthesia/analgesia for anterior thigh and knee surgery.

Question 144: TTTFF


A lumbar plexus block will target the following nerves:
• Femoral nerve
• Lateral cutaneous nerve of the thigh
• Obturator nerve
A lumbar plexus block provides a similar anaesthetic block to the 3-in-1 block. The 3-in-1
block is arguably somewhat safer, but less effective. The lumbar plexus block is more
successful, but carries a higher risk of adverse events.

Question 145: FFFTT


Flaring of the ribs increases the circumference of the thoracic cage by 5–7 cm. This is an
anatomical change associated with pregnancy that occurs within the first trimester before
there is marked enlargement of the uterus. Elevation of the diaphragm therefore does not
decrease the internal volume of the thoracic cage, as this is accommodated within the
enlarged circumference of the thoracic cavity.
Barrier pressure is the difference between the lower oesophageal sphincter (LOS) pressure
and the intragastric pressure. During pregnancy there is an increase in the intragastric
pressure and a decrease in the LOS pressure. This leads to an overall reduction in the barrier
pressure. A result of this is gastro-oesophageal reflux, which can lead to symptoms of
heartburn in approximately 55–80% of women.
Engorgement of the epidural veins occurs in pregnancy due to the diversion of blood
through the vertebral plexuses. This reduces the epidural volume and causes pressure within
the epidural space to rise to positive values. During the second and third stages of labour, the
pressure changes can range from 20–60 cmH2O.

Downloaded from https://www.cambridge.org/core. University of Edinburgh, on 19 Aug 2019 at 13:21:27, subject to the Cambridge Core terms of
use, available at https://www.cambridge.org/core/terms. https://doi.org/10.1017/9781108566100.006
176 Chapter 2b: Anatomy Answers

Question 146: FTFFF


The uterine artery is the major source of blood supply to the uterus and is a branch of the
anterior division of the internal iliac artery. Venous drainage occurs in the uterine venous
plexus, which drains into the uterine veins and on to the internal iliac vein.
Placental blood supply is governed largely by the uterine blood vessels. By 20 weeks’
gestation there are approximately 120 spiral arteries that enter the intervillous spaces.
At term, the blood flow to the placenta can range from 500–800 ml.min–1.
Uterine blood flow can become compromised due to changes in the systemic blood
pressure secondary to aorta–caval compression.

Question 147: TTFFF


A single umbilical vein directs oxygenated blood (80% saturation) from the placenta into the
inferior vena cava (IVC) of the fetal circulation. The ductus venosus shunts half of the
oxygenated blood into the IVC away from the liver. Two-thirds of the blood entering the
right atrium from the IVC enters the left atrium through the patent foramen ovale.
The remaining blood enters the right ventricle and mixes with blood from the superior
vena cava (25% saturation) before entering the pulmonary artery.
Oxygenated blood from the left atrium (65% saturation) passes into the left ventricle and
travels along the ascending aorta to supply the brain and upper body. The ductus arteriosus
connects the pulmonary vasculature to the descending aorta, which supplies the lower body
with approximately 55% oxygen saturation.
The internal iliac arteries return deoxygenated blood in the umbilical arteries back to the
placenta.
At birth, changes in pulmonary and systemic vascular resistance lead to closure of the
ductus arteriosus. There is functional closure at 24 hours in most newborns. Histological
changes occur by three weeks. Increase in left-sided heart pressures leads to closure of the
foramen ovale.

Question 148: TFTFT


With regards to the airway and respiration there are a few anatomical variations that should
be considered:
• The larynx sits higher and more anteriorly than in adults and the cricoid is the narrowest
part of the airway.
• Respiratory effort is primarily controlled by the diaphragm, where there are a majority of
Type I muscle fibres.

Question 149: FFTTF


Intraosseous injection sites in children include:
• Proximal tibia
• Distal tibia
• Distal femur
Additionally, in adults, the following sites have been used:
• Proximal humerus

Downloaded from https://www.cambridge.org/core. University of Edinburgh, on 19 Aug 2019 at 13:21:27, subject to the Cambridge Core terms of
use, available at https://www.cambridge.org/core/terms. https://doi.org/10.1017/9781108566100.006
Chapter 2b: Anatomy Answers 177

• Distal radius or ulnar


• Sternum
• Iliac crest
It is important to avoid the epiphyseal growth plates in younger children. Also, be aware
of siting IO needles distal to fracture sites.

Question 150: TTTTT


The formulae are often used as a guide for estimation. Certain parameters may be modified
from time to time and therefore it is best practice to check up-to-date guidelines.

Downloaded from https://www.cambridge.org/core. University of Edinburgh, on 19 Aug 2019 at 13:21:27, subject to the Cambridge Core terms of
use, available at https://www.cambridge.org/core/terms. https://doi.org/10.1017/9781108566100.006
Chapter
Pharmacology Questions

3a
Question 1
The following agents are examples of racemic mixtures:
a. Isoflurane
b. Enflurane
c. Sevoflurane
d. Atropine
e. Levobupivacaine

Question 2
Regarding optical isomers and their properties:
a. S(+)-ketamine produces less intense emergence phenomena than R(–)-ketamine
b. R(–)-ketamine is three times more potent than S(+)-ketamine
c. Levobupivacaine requires a higher plasma concentration to produce myocardial
depression compared to bupivicaine
d. Levobupivacaine is more likely to precipitate excitatory central nervous system effects
compared to bupivicaine
e. S-ropivacaine is more lipid soluble than bupivacaine and this results in reduced pene-
tration of Aβ nerve fibres

Question 3
Regarding drug delivery to cells:
a. The degree of ionization determines the duration of drug action
b. Alfentanil has a more rapid onset of action compared to fentanyl as a result of its lipid
solubility
c. Aspirin has a pKa of 3, meaning that it is wholly unionized at physiological pH
d. Addition of 8.4% sodium bicarbonate to 2% lidocaine raises its pH and therefore
increases the speed of onset
e. Bupivacaine readily crosses the placenta

Question 4
Dose–response curves and log10 dose–response curves are important tools in establishing
the effective range of drug doses both in vitro and in vivo.
a. Dose is on the y-axis and response is on the x-axis
178
Downloaded from https://www.cambridge.org/core. University of Edinburgh, on 19 Aug 2019 at 13:21:27, subject to the Cambridge Core terms of
use, available at https://www.cambridge.org/core/terms. https://doi.org/10.1017/9781108566100.007
Chapter 3a: Pharmacology Questions 179

b. On a log10 dose–response curve, if drug A is more potent than drug B, then the curve for
drug A will lie to the left of the curve for drug B
c. On a log10 dose–response curve, if drug A is less efficacious than drug B, then the curve
for drug B will plateau higher than the curve for drug A
d. If A is an antagonist of B increasing the concentration of A will move the log10 dose–
response curve of B to the right
e. The action of a competitive antagonist cannot be overcome by increasing the dose of the
agonist

Question 5
The definition of an agonist is an agent that can bind to a receptor and elicit a biological
response. Antagonists are drugs that decrease the actions of an endogenous ligand or
another drug. The following are drug–receptor interactions:
a. Phenylephrine is an antagonist at α-adrenergic receptors
b. Doxazosin is an agonist at α-adrenergic receptors
c. Ondansetron is an antagonist at 5-HT2 receptors
d. Ketamine is a competitive agonist at the NMDA receptor
e. Dexmedetomidine is a selective α1-adrenergic receptor agonist

Question 6
Dose–response curves can be used to determine ED50 (effective dose 50%), as well as the
toxic effect in the form of the LD50 (lethal dose 50%). The ratio of LD50:ED50 is known as the
therapeutic index and can be utilized to estimate drug safety.
a. For greater safety, a drug should have a low therapeutic index
b. Warfarin has a high therapeutic index
c. Penicillin has a low therapeutic index
d. Ibuprofen has a high therapeutic index
e. The units of the therapeutic index are usually mg.ml–1

Question 7
Drug–receptor interactions may result in a range of responses:
a. A partial agonist occupies less than half of receptor sites, which results in a submaximal
response
b. An antagonist has no effect on the state of the receptor in the absence of an agonist or
inverse agonist
c. There is a linear relationship between efficacy and affinity
d. A partial agonist can have antagonist activity
e. Buprenorphine acts as an inverse agonist at the μ-opioid receptor

Question 8
Concerning G-protein-coupled receptor pharmacology:
a. When activated, guanylyl triphosphate (GTP) binds the β-subunit of the trimeric
G-protein

Downloaded from https://www.cambridge.org/core. University of Edinburgh, on 19 Aug 2019 at 13:21:27, subject to the Cambridge Core terms of
use, available at https://www.cambridge.org/core/terms. https://doi.org/10.1017/9781108566100.007
180 Chapter 3a: Pharmacology Questions

b. Opiates act via stimulation of the Gi subtype of G-protein-coupled receptor (GPCR)


c. Activation of the Gq subtype of GPCR results in the activation of protein kinase C
d. α1-Adrenoreceptors are of the Gs subtype
e. α2-Adrenoreceptors are of the Gi subtype

Question 9
Concerning response to repeated drug doses:
a. Tachyphylaxis refers to the phenomenon of rapid loss of response to repeated doses of
a drug
b. Tolerance occurs when loss of response occurs over a longer period of administration
c. Desensitization occurs when incrementally larger doses of drug are required to produce
the same response
d. Desensitization occurs in chronic opiate abuse
e. Ephedrine displays tachyphylaxis

Question 10
Concerning acetylcholinesterase inhibitors:
a. Neostigmine prolongs depolarizing neuromuscular blockade
b. Edrophonium forms a carbamylated complex with acetylcholinesterase
c. Edrophonium has use in the treatment of myasthenia gravis
d. Acetylcholinesterase inhibitors have a treatment role in Alzheimer’s disease
e. Dicobalt ededate is a suitable antidote for organophosphate poisoning

Question 11
Concerning antiarrhythmics and their effect on the action potential:
a. Lidocaine prolongs the refractory period of cardiac muscle
b. Flecainide has no effect on the refractory period of cardiac muscle
c. Verapamil’s primary effect is on the L-type slow voltage calcium channels of the SA and
AV nodes
d. Amiodarone decreases the repolarization rate of the cardiac membrane
e. Amiodarone is a potassium channel activator

Question 12
Regarding mechanisms of drug action:
a. cAMP formed under the regulation of G proteins is broken down by the action of
phosphodiesterases
b. G-protein receptors consist of four subunits – two α and two β
c. The GABAA receptor has a pentameric structure
d. Thyroid hormones act via receptors that are part of the cell membrane
e. Nitrous oxide does not act on the NMDA receptor

Downloaded from https://www.cambridge.org/core. University of Edinburgh, on 19 Aug 2019 at 13:21:27, subject to the Cambridge Core terms of
use, available at https://www.cambridge.org/core/terms. https://doi.org/10.1017/9781108566100.007
Chapter 3a: Pharmacology Questions 181

Question 13
Regarding adverse drug reactions:
a. Type A reactions are unpredictable and usually involve the immune system
b. Type B drug reactions encompass both anaphylactic and anaphylactoid reactions
c. An idiosyncratic drug reaction is a predictable drug reaction that is dose dependent
d. An anaphylactoid reaction is mediated by IgE antibodies
e. Adverse drug reactions should be reported to the Medicines and Healthcare Products
Regulatory Agency (MHRA)

Question 14
The following agents are cytochrome P450 enzyme inhibitors:
a. Cyclosporin
b. Amiodarone
c. Grapefruit juice
d. Carbamazepine
e. Chronic alcohol

Question 15
The following drug combinations can potentially cause serious adverse reactions:
a. Moclobemide and ephedrine
b. Ramipril and diclofenac
c. Warfarin and orange juice
d. Bisoprolol and verapamil
e. Levodopa and metoclopramide in a patient with Parkinson’s disease

Question 16
The following statements are true regarding drug action/interactions:
a. A synergistic reaction occurs when the net effect of several drugs is the sum of the
individual actions of each drug
b. Reversal of heparin with protamine is a physicochemical interaction
c. The increase in acetylcholine after neostigmine administration is an indirect pharma-
codynamic interaction
d. Sodium bicarbonate will make the urine more alkaline and enhance the excretion of
weak acids
e. β-Blockers may decrease the time to onset of fasciculation following the administration
of suxamethonium

Question 17
The following drugs display zero-order kinetics:
a. Aspirin
b. Warfarin
c. Theophylline

Downloaded from https://www.cambridge.org/core. University of Edinburgh, on 19 Aug 2019 at 13:21:27, subject to the Cambridge Core terms of
use, available at https://www.cambridge.org/core/terms. https://doi.org/10.1017/9781108566100.007
182 Chapter 3a: Pharmacology Questions

d. Alfentanil
e. Phenytoin

Question 18
Which of these drugs readily penetrate the blood–brain barrier?
a. Atropine
b. Glycopyrronium
c. Benzylpenicillin
d. Thiopentone
e. Vecuronium

Question 19
Which of the following statements related to drug handling by the body are true?
a. The elderly population have a reduced volume of distribution (VD)
b. The presence of portocaval shunts in hepatic impairment reduces bioavailability
c. Plasma protein binding tends to be higher in the neonate than in the adult
d. Patients with renal impairment may have an increased VD and may require a higher
loading dose of drug
e. With regards to lidocaine use in the neonate, the proportion of free drug will be lower
than in the adult

Question 20
The rapid onset of action of thiopentone when administered intravenously is as a result of:
a. Its pKa of 7.6
b. Cerebral blood flow
c. Redistribution to muscle and adipose tissue
d. Extensive hepatic metabolism
e. Its degree of lipophilicity

Question 21
Bioavailability:
a. Is greater by the enteral route than the sublingual route
b. Is indicated by the area under the plasma concentration–time curve
c. May be affected by coeliac disease if drugs are given orally
d. Is low if a drug undergoes minimal first pass metabolism
e. Is 100% if a drug is given intravenously

Question 22
The following drugs cross the placenta in significant quantities:
a. Isoflurane
b. Diclofenac
c. Thiopentone

Downloaded from https://www.cambridge.org/core. University of Edinburgh, on 19 Aug 2019 at 13:21:27, subject to the Cambridge Core terms of
use, available at https://www.cambridge.org/core/terms. https://doi.org/10.1017/9781108566100.007
Chapter 3a: Pharmacology Questions 183

d. Morphine
e. Co-amoxiclav

Question 23
The following drugs are predominantly metabolized by the liver:
a. Propofol
b. Cisatracurium
c. Esmolol
d. Mivacurium
e. Lisinopril

Question 24
The following drugs are correctly paired with a recognized mechanism by which they enter
cells:
a. Thiopentone and passive diffusion
b. Fluconazole and active transport
c. Iron and pinocytosis
d. Glucose and passive diffusion
e. Penicillin G and active transport

Question 25
The following drugs are efficiently removed from the plasma by haemofiltration:
a. Atenolol
b. Aspirin
c. Enoxaparin
d. Digoxin
e. Warfarin

Question 26
Regarding drug elimination:
a. Clearance refers to the amount of drug removed from the body per unit time
b. Elimination of most drugs follows zero-order kinetics
c. Elimination is often related to renal function
d. Rate of elimination is influenced by volume of distribution
e. A time constant is longer than a half-life

Question 27
Concerning multicompartmental pharmacokinetic models:
a. Peripheral compartments represent less vascular structures
b. A drug can be eliminated from any compartment
c. Catenary models link a central compartment to peripheral compartments

Downloaded from https://www.cambridge.org/core. University of Edinburgh, on 19 Aug 2019 at 13:21:27, subject to the Cambridge Core terms of
use, available at https://www.cambridge.org/core/terms. https://doi.org/10.1017/9781108566100.007
184 Chapter 3a: Pharmacology Questions

d. In three-compartment models, equilibration between central and peripheral compart-


ments occurs at different speeds
e. In a tri-exponential decay curve, the sum of the three tangents’ y intercepts is equal to
concentration at the t = 0 (Co) value

Question 28
Concerning the context-sensitive half-life:
a. It is defined as the time for the plasma concentration of a drug to fall by 50% subsequent
to cessation of an infusion after plasma loading
b. Remifentanil displays a context-insensitive half-life
c. Context sensitive half-life predicts time to waking subsequent to termination of infusion
of a hypnotic agent
d. Alfentanil demonstrates context sensitivity after prolonged infusion times
e. Fentanyl demonstrates context sensitivity after prolonged infusion times

Question 29
Concerning total intravenous anaesthesia:
a. Plasma concentrations are not assessed during anaesthesia
b. A dedicated cannula for anaesthesia is mandatory
c. It is indicated in patients with a history of malignant hyperpyrexia
d. The Schnider pharmacokinetic model is more appropriate for use in elderly patients
undergoing propofol target-controlled infusion
e. No adjunctive analgesia is required in anaesthesia with remifentanil and propofol

Question 30
Regarding prolonged depolarizing neuromuscular blockade (suxamethonium apnoea):
a. 96% of the population is homozygous for the Eu gene
b. Those with the genotype Ea:Ea may have a resultant block, which is prolonged by up to
10 minutes
c. The dibucaine number refers to the direct activity of plasma cholinesterase
d. Those with a homozygous normal genotype and phenotype have a dibucaine number
of 20
e. The alleles responsible for altered plasma cholinesterase activity have been identified on
chromosome 3

Question 31
Regarding genetic differences in drug handling:
a. Malignant hyperpyrexia (MH) has been associated with defects in the ryanodine
receptor on chromosome 17
b. A diagnosis of MH is based on the response of biopsied muscle to 2% halothane and
caffeine
c. Trigger agents for MH include etomidate and ephedrine

Downloaded from https://www.cambridge.org/core. University of Edinburgh, on 19 Aug 2019 at 13:21:27, subject to the Cambridge Core terms of
use, available at https://www.cambridge.org/core/terms. https://doi.org/10.1017/9781108566100.007
Chapter 3a: Pharmacology Questions 185

d. Drugs that may be affected by acetylator status include hydralazine and isoniazid
e. Up to 90% of oriental populations are ‘fast acetylators’

Question 32
Thiopentone:
a. Is a methylated oxybarbiturate
b. Is more protein bound than pentobarbitone
c. Has a pH of 10.5 in solution
d. Potentiates the α-subunit of the GABAA receptor
e. May produce an isoelectric EEG

Question 33
Propofol:
a. Has a calorie load of 1 kcal.ml–1
b. Undergoes sulfuronidation within the liver
c. Commonly precipitates a reflex tachycardia when given at induction doses
d. May cause greenish discolouration of the hair
e. Has a terminal elimination half-life of 5–12 hours

Question 34
Ketamine:
a. May be given rectally
b. Potentiates the neurotransmitter glutamate at the NMDA receptor
c. Shows agonist action at OP3 receptors
d. Increases cerebral blood flow
e. Is metabolized to norketamine by cytochrome P450 enzymes

Question 35
Etomidate:
a. Contains 35% v/v ethylene glycol in solution to improve stability
b. Commonly results in histamine release on injection
c. Is a hydroxylated imidazole derivative
d. Inhibits the 17α-hydroxylase enzyme
e. May precipitate a porphyric crisis

Question 36
Regarding thiopentone:
a. Rapid emergence from a single bolus dose is due to rapid metabolism
b. It may cause a reduction in urine output as a result of increased ADH release
c. Anaphylactic reactions are seen in approximately 1:7500 administrations
d. It produces a reduction in cerebral metabolic oxygen requirement (CMRO2)
e. Solubility is dependent on tautomerism

Downloaded from https://www.cambridge.org/core. University of Edinburgh, on 19 Aug 2019 at 13:21:27, subject to the Cambridge Core terms of
use, available at https://www.cambridge.org/core/terms. https://doi.org/10.1017/9781108566100.007
186 Chapter 3a: Pharmacology Questions

Question 37
Regarding propofol:
a. Volume of distribution is approximately 4 l.kg–1
b. Epileptiform movements are seen in up to 10% of patients
c. It is 80% bound to plasma albumin
d. Offset is more rapid than thiopentone following an initial induction dose
e. Pain on injection can be reduced by the addition of 1% lidocaine to the syringe

Question 38
Benzodiazepine metabolism involves the following reactions:
a. Hydroxylation
b. Acetylation
c. Dealkylation
d. Glucuronidation
e. Oxidation

Question 39
Regarding benzodiazepines:
a. They have low oral bioavailability
b. They display high protein binding
c. Their half-life may be prolonged due to genetic variability
d. They have a small volume of distribution
e. Midazolam has relatively low clearance compared to other benzodiazepines

Question 40
Metabolism of benzodiazepines occurs in the liver and there may be some active metabo-
lites. In considering metabolism and excretion:
a. Urinary excretion for benzodiazepines is in the order of 10%
b. Benzodiazepines are effectively removed by dialysis
c. Chlordiazepoxide has active metabolites following transformation in the liver
d. Diazepam has an elimination half-life of over 24 hours
e. Temazepam has no active metabolites

Question 41
Isoflurane:
a. Has a molecular weight of 200
b. Has an oil:gas partition coefficient of 225
c. Causes greater myocardial depression than halothane
d. Causes dose-dependent uterine relaxation
e. Is 2% metabolized

Downloaded from https://www.cambridge.org/core. University of Edinburgh, on 19 Aug 2019 at 13:21:27, subject to the Cambridge Core terms of
use, available at https://www.cambridge.org/core/terms. https://doi.org/10.1017/9781108566100.007
Chapter 3a: Pharmacology Questions 187

Question 42
Concerning nitrous oxide:
a. The equilibration of fractional alveolar to fractional inspired concentration (FA/Fi) is
faster with desflurane compared to nitrous oxide
b. It is produced by the heating of ammonium sulfate
c. It reduces uterine muscle tone
d. It causes megaloblastic anaemia with prolonged use
e. Its use is contraindicated in patients with pneumothorax

Question 43
Concerning minimum alveolar concentration (MAC):
a. MAC is decreased in hyperthyroidism
b. MAC is decreased in pregnancy
c. There is an inverse relationship to the oil:gas partition coefficient of agents
d. The MAC of nitrous oxide is higher than that of halothane
e. It is defined as a percentage of 1 atmosphere

Question 44
Concerning sevoflurane:
a. Has a boiling point of 58 °C
b. Has a saturated vapour pressure of 32 kPa at 20 °C
c. Has an oil:gas partition coefficient of 97
d. Approximately 3–5% is metabolized
e. Compound A, produced from sevoflurane’s reaction with moist soda lime, is nephro-
toxic in humans

Question 45
Concerning inhalational anaesthetic potency:
a. There is inverse proportionality between oil:gas partition coefficients and potency
b. MAC is directly proportional to potency
c. Halothane is more potent than sevoflurane
d. Enflurane is more potent than methoxyflurane
e. Desflurane’s quick onset and offset is a reflection of its low potency

Question 46
Concerning halothane:
a. It is unstable in light
b. It has a saturated vapour pressure of 32 kPa at 20 °C
c. It is irritant to breathe
d. It increases myocardial sensitivity to catecholamines
e. It is 2% metabolized

Downloaded from https://www.cambridge.org/core. University of Edinburgh, on 19 Aug 2019 at 13:21:27, subject to the Cambridge Core terms of
use, available at https://www.cambridge.org/core/terms. https://doi.org/10.1017/9781108566100.007
188 Chapter 3a: Pharmacology Questions

Question 47
Regarding the mechanism of action of general anaesthetics:
a. The proposed mechanism is said to be consistent with the Meyer–Briggs rule
b. The rule suggests the anaesthetic agent acts by dissolving in the protein bilayer of cells
c. GABAA and glycine are inhibitory receptors
d. Glycine receptors belong to the family of ligand-gated ion channels
e. Hydrophobic anaesthetics are less potent

Question 48
Local anaesthetics are weak bases and poorly soluble. Amongst other factors, the pKa
correlates to the speed of onset of a particular agent. The following are other factors that
affect speed of onset of peripheral nerve blocks:
a. Degree of local anaesthetic ionization
b. Lipid solubility of local anaesthetic
c. Proximity of anaesthetic to the target nerve
d. Protein binding of the anaesthetic agent
e. Type of block

Question 49
The following are the maximum recommended doses of each agent:
a. Bupivacaine with epinephrine 5 mg.kg–1
b. Bupivacaine without epinephrine 3 mg.kg–1
c. Lidocaine with epinephrine 7 mg.kg–1
d. Levobupivacaine without epinephrine 1 mg.kg–1
e. Ropivacaine without epinephrine 3 mg.kg–1

Question 50
Metabolism of local anaesthetics depends on chemical structure.
a. Amide local anaesthetics are rapidly metabolized by plasma cholinesterase
b. Lidocaine has a low extraction ratio
c. Ester local anaesthetics are rapidly metabolized in cerebrospinal fluid
d. Metabolites from ester hydrolysis are excreted in the urine
e. Metabolism of bupivacaine is independent of hepatic function

Question 51
Local anaesthetics are available in various strengths and preparations:
a. Glucose in a concentration of 8 g.ml–1 is added to bupivacaine to increase the baricity of
the solution for use in spinal anaesthesia
b. The preservative methyl parahydroxybenzoate is often added to multidose vials
c. The addition of bicarbonate reduces the amount of the unionized form of the local
anaesthetic

Downloaded from https://www.cambridge.org/core. University of Edinburgh, on 19 Aug 2019 at 13:21:27, subject to the Cambridge Core terms of
use, available at https://www.cambridge.org/core/terms. https://doi.org/10.1017/9781108566100.007
Chapter 3a: Pharmacology Questions 189

d. Lidocaine and amethocaine are mixed in equal parts to form a eutectic mixture of local
anaesthetic cream
e. The preparation of bupivacaine as a hydrosulfite salt allows it to be dissolved in water,
forming an acidic solution

Question 52
Local anaesthetics alter nerve function by their action on sodium channels.
a. Local anaesthetics can also have an effect on calcium channels
b. Sodium channels in the resting state have a high affinity for local anaesthetic
c. The sodium channel is composed of a single polypeptide chain with four repeating units
d. Local anaesthetics bind directly to the extracellular portion of the sodium channel to
exert the effect
e. The binding of bupivacaine to the sodium channel is irreversible

Question 53
The following questions concern intralipid and its role in the treatment of local anaesthetic
toxicity.
a. Intralipid can be used in the treatment of tricyclic antidepressant overdose
b. The initial loading dose when treating local anaesthetic toxicity is 15 ml.kg–1
c. Propofol can be used as a substitute where Intralipid is not available
d. CPR may have to continue for over a hour after the administration of intralipid
e. It should only be used in cardiac arrest situations

Question 54
Concerning intralipid and its role in parenteral nutrition:
a. Soya bean oil is the major constituent
b. Selenium-induced neurotoxicity can occur with long-term use
c. Hepatomegaly and jaundice are recognized delayed complications
d. It should be used with caution in cases of pancreatitis with associated hyperlipidaemia
e. Thrombocythemia is a recognized complication

Question 55
Regarding patient-controlled epidural analgesia for labour:
a. It is associated with improved maternal satisfaction compared to continuous epidural
infusion
b. Intermittent boluses alone provide superior analgesia to intermittent boluses combined
with a background infusion
c. Increases the likelihood of caesarean section
d. May be associated with more motor block than continuous epidural infusion
e. Can cause maternal fever

Downloaded from https://www.cambridge.org/core. University of Edinburgh, on 19 Aug 2019 at 13:21:27, subject to the Cambridge Core terms of
use, available at https://www.cambridge.org/core/terms. https://doi.org/10.1017/9781108566100.007
190 Chapter 3a: Pharmacology Questions

Question 56
Regarding paracetamol:
a. It has consistently better absorption by the rectal route when compared to the oral route
b. It is extensively bound to plasma proteins at normal doses
c. Metabolism is by conjugation in the liver predominantly with glucuronide or phosphate
d. By one hour after administration, the plasma levels for oral and intravenous routes are
similar
e. At normal doses, 85% of the drug is metabolized by CYP2E1 to produce N-acetyl-
p-benzoquinoneimine

Question 57
Concerning paracetamol:
a. Overdose causes around 50% of fulminant hepatic failure cases in the UK
b. Ingestion of alcohol induces CYP2E1, thus increasing the risk of hepatotoxicity
c. Treatment of overdose consists of early replacement of glutathione either orally or
intravenously
d. There is a reduction in mortality if N-acetylcysteine is given up to 72 hours after
overdose
e. N-acetylcysteine is toxic to the fetus and therefore cannot be used in pregnancy

Question 58
Aspirin:
a. Is mainly absorbed in the small bowel
b. Increases oxygen consumption
c. Irreversibly inhibits cyclo-oxygenase (COX) in the endothelium
d. May cause hypoventilation, coma and pyrexia in overdose
e. Excretion may be enhanced by acidification of urine

Question 59
NSAIDs:
a. Inhibit leukotriene production
b. Promote sodium retention
c. Can cause thrombocytopenia
d. Can cause delayed healing of fractures
e. Reduce opioid consumption in the perioperative period

Question 60
Regarding NSAIDs:
a. Meloxicam has an increased side-effect profile compared to other NSAIDs
b. Ibuprofen can be used in doses of up to 30 mg.kg–1 in children
c. Indomethacin promotes closure of the ductus arteriosus in infants

Downloaded from https://www.cambridge.org/core. University of Edinburgh, on 19 Aug 2019 at 13:21:27, subject to the Cambridge Core terms of
use, available at https://www.cambridge.org/core/terms. https://doi.org/10.1017/9781108566100.007
Chapter 3a: Pharmacology Questions 191

d. They are absolutely contraindicated in asthma


e. Celecoxib should not be used in patients with a sulfonamide allergy

Question 61
With reference to specific COX-2 inhibitors:
a. They affect platelet function, especially in high doses
b. They have an analgesic efficacy that is lower than comparable non-specific NSAIDs
c. Cardiac side effects may be increased
d. The incidence of gastrointestinal side effects appears to be reduced
e. Concurrent aspirin therapy is safe

Question 62
Tramadol is a synthetic opioid and is an aminocyclohexanol derivative of codeine.
a. It has a structural resemblance to venlafaxine
b. Metabolism of tramadol has no pharmacogenetic variability
c. Tramadol has no action at the NMDA receptor
d. The main metabolite of tramadol is active at the μ-opioid receptor
e. Tramadol produces less respiratory depression than other opioids

Question 63
Like morphine, codeine is a naturally occurring derivative of an opium alkaloid.
The following are documented pharmacological characteristics of codeine:
a. It is poorly absorbed after oral administration
b. The major metabolite is codeine-3-glucuronide
c. Morphine is a metabolite of codeine
d. Codeine metabolism is affected by CYP2D6 polymorphism
e. The reduction in bowel peristalsis is mediated via kappa receptors

Question 64
Morphine:
a. Is metabolized to a pharmacologically active compound
b. Reduces minute ventilation predominantly via reducing tidal volumes
c. Has a longer half-life than naloxone
d. Is licensed for intrathecal use
e. Is an opiate

Question 65
Remifentanil:
a. Exhibits tachyphylaxis
b. Has a predictable dose–response relationship
c. Displays a context-sensitive half-life

Downloaded from https://www.cambridge.org/core. University of Edinburgh, on 19 Aug 2019 at 13:21:27, subject to the Cambridge Core terms of
use, available at https://www.cambridge.org/core/terms. https://doi.org/10.1017/9781108566100.007
192 Chapter 3a: Pharmacology Questions

d. Used in combination with propofol (TIVA) reduces the risk of awareness when com-
pared to general anaesthesia with volatile agents
e. Can cause chest wall rigidity

Question 66
Diamorphine:
a. Is a synthetic opioid formed by joining two molecules of morphine via an ester bond
b. Has a half-life of 2 to 3 minutes
c. Is associated with pruritus when used intrathecally
d. Is given in similar doses whether via intravenous or epidural administration
e. Causes nausea and vomiting due to smooth muscle contraction in the gastrointestinal
tract

Question 67
When comparing fentanyl to alfentanil:
a. Fentanyl is more potent than alfentanil
b. Alfentanil has a higher lipid solubility than fentanyl
c. The majority of fentanyl is ionized at pH 7.4
d. Clearance of alfentanil exceeds that of fentanyl
e. Alfentanil has a smaller volume of distribution than fentanyl

Question 68
Concerning the safety features of patient-controlled analgesia (PCA) devices:
a. If morphine PCA devices are being used in patients, observation of respiratory rate is
mandatory
b. Pumps are designed to detect disconnection from the patient
c. If intravenous PCA devices are not being used via dedicated cannulae, use of a Y
connector with a one-way valve is mandatory
d. PCAs may not be used concurrently with epidural infusions
e. Programmable limits to bolus doses and lock-out times reduce the risk of overdose

Question 69
Concerning drugs used in PCA:
a. Morphine has the greatest efficacy of all the opioid drugs used in PCA
b. Tramadol is widely accepted as a suitable PCA drug
c. Background infusions of morphine with additional patient-controlled boluses provide
superior pain control
d. Ketamine is a recognized non-opiate drug that can be delivered via PCA in addition to
morphine
e. The routine addition of antiemetics in PCA syringes is mandatory

Downloaded from https://www.cambridge.org/core. University of Edinburgh, on 19 Aug 2019 at 13:21:27, subject to the Cambridge Core terms of
use, available at https://www.cambridge.org/core/terms. https://doi.org/10.1017/9781108566100.007
Chapter 3a: Pharmacology Questions 193

Question 70
Cisatracurium:
a. Is derived from the three cis-cis isomers of atracurium
b. Is eliminated predominantly via Hoffman degradation
c. Is associated with less histamine release than atracurium
d. May be preferred over atracurium in epileptic patients as it is not metabolized to
laudanosine
e. Differs from atracurium in that it is more potent, but slower to provide maximal muscle
relaxation (in equipotent doses)

Question 71
Rocuronium:
a. Is predominantly excreted in the urine (assuming normal renal function)
b. Can raise arterial blood pressure and heart rate
c. Should be avoided in asthmatics
d. Will usually provide intubating conditions within 60 s when given at a dose of
0.6 mg.kg–1
e. Becomes unstable if left at room temperature for more than 24 hours

Question 72
Atracurium:
a. Improves compliance with mechanical ventilation and reduces mortality in ARDS
b. Muscle relaxation may be reversed 2 minutes after an intubating dose by the adminis-
tration of 16 mg.kg–1 of sugammadex
c. Is less associated with bronchospasm than rapacuronium
d. Consists of 16 isomers
e. Is approximately 99% protein bound in the plasma

Question 73
Regarding suxamethonium:
a. It is an antagonist at the nicotinic acetylcholine receptor
b. It should be given in an increased dose for patients with Duchenne muscular dystrophy
c. Phase 2 block behaves like a non-depolarizing neuromuscular block
d. It is metabolized by acetylcholinesterase
e. It can induce bradycardia via its action on muscarinic receptors

Question 74
Concerning suxamethonium:
a. It is formed by combining two acetylcholine molecules
b. It is a trigger for malignant hyperpyrexia
c. Risk of postoperative muscle pain is directly proportional to muscle mass

Downloaded from https://www.cambridge.org/core. University of Edinburgh, on 19 Aug 2019 at 13:21:27, subject to the Cambridge Core terms of
use, available at https://www.cambridge.org/core/terms. https://doi.org/10.1017/9781108566100.007
194 Chapter 3a: Pharmacology Questions

d. Patients with myasthenia gravis are likely to require a reduced dose of suxamethonium
e. Risk of anaphylaxis is around 1 in 2000

Question 75
Mivacurium:
a. Is characterized by its rapid onset and offset of action
b. Is preferred to suxamethonium for short-term muscle relaxation in patients with known
suxamethonium apnoea
c. Undergoes urinary excretion
d. Is potentiated by volatile anaesthetics
e. Is safe for use in pregnancy

Question 76
Regarding sugammadex:
a. It is a specific reversal agent for rocuronium
b. It is a modified tertiary amine
c. It can reverse profound neuromuscular blockade from rocuronium within 4 minutes
when given at a dose of 4 mg.kg–1
d. The dose should be reduced for patients with severe renal impairment
e. For obese patients, the dose should be based on actual body weight, not ideal body
weight

Question 77
Neostigmine:
a. May be used in the diagnosis and treatment of myasthenia gravis
b. Is a medium-acting anticholinesterase
c. When given IV to reduce neuromuscular block at the end of surgery, is commonly
associated with a tachycardia
d. Increases smooth muscle contractility
e. Can cause seizures if given at a dose of more than 50 μg.kg–1

Question 78
Atropine:
a. Is a competitive antagonist at all muscarinic receptors
b. Is a parasympathomimetic drug
c. Is given at a standard dose of 3 mg during resuscitation from cardiac arrest with non-
shockable rhythms
d. Can cause bradycardia
e. Is useful in the treatment of a cholinergic crisis

Downloaded from https://www.cambridge.org/core. University of Edinburgh, on 19 Aug 2019 at 13:21:27, subject to the Cambridge Core terms of
use, available at https://www.cambridge.org/core/terms. https://doi.org/10.1017/9781108566100.007
Chapter 3a: Pharmacology Questions 195

Question 79
Glycopyrrolate:
a. Is a more potent antisialogogue than atropine
b. Is completely ionized at physiological pH
c. Is associated with amnesia, confusion and excitation in elderly patients
d. Is a recognized treatment for hyperhydrosis
e. Is a tertiary ammonium compound

Question 80
Regarding hyoscine:
a. It can reduce the risk of postoperative nausea and vomiting when given preoperatively
via transdermal patch
b. Hyoscine butylbromide crosses the blood–brain barrier
c. It is more sedating than atropine
d. It causes mydriasis
e. It is one of the ingredients of Omnopon ®
Question 81
Regarding phosphodiesterase inhibitors:
a. They cause peripheral vasoconstriction
b. They have a positive inotropic action
c. They increase the intracellular levels of cAMP
d. Milrinone is a selective PDE III inhibitor
e. They act by blocking the Na+/K+ pump

Question 82
Dopexamine:
a. Is structurally similar to dobutamine
b. Increases splanchnic blood flow
c. Has a natriuretic and diuretic effect
d. Has significant α-adrenergic agonist action
e. Causes bronchodilation

Question 83
The following have positive inotropic activity:
a. Thyroxine
b. Digoxin
c. Aminophylline
d. Methoxamine
e. Glucagon

Downloaded from https://www.cambridge.org/core. University of Edinburgh, on 19 Aug 2019 at 13:21:27, subject to the Cambridge Core terms of
use, available at https://www.cambridge.org/core/terms. https://doi.org/10.1017/9781108566100.007
196 Chapter 3a: Pharmacology Questions

Question 84
Levosimendan:
a. Is indicated in the treatment of chronic heart failure
b. Increases the sensitivity of myocytes to calcium
c. Closes ATP-sensitive K+ channels to cause vascular smooth muscle contraction
d. Binds to tropomyosin
e. Is contraindicated in ventricular outflow tract obstruction

Question 85
The following statements are correct:
a. Adrenaline may increase minimum alveolar concentration and increase peripheral pain
threshold
b. Dopamine reduces pulmonary vascular resistance
c. Isoprenaline may cause hyperglycaemia
d. Dobutamine may be used as an alternative to exercise in cardiac stress testing
e. Inotropes increase the force of myocardial contractility

Question 86
Metaraminol is a vasopressor agent with direct and indirect effects on the sympathetic
system.
a. It usually results in a negative inotropic effect
b. It has no action on β-adrenergic receptors
c. It usually causes an increase in cardiac output
d. It cannot be administered via the intramuscular route due to intense local ischaemia
e. It causes a marked decrease in glycogenolysis

Question 87
Vasopressin is a potent vasoconstrictor with the following properties:
a. Action via α-adrenergic receptors
b. Causes a significant increase in pulmonary vascular resistance
c. Has no direct cardiac effects
d. Has a significant role in maintenance of arteriolar tone in health
e. Causes greater constriction of renal afferent arterioles compared to efferent arterioles

Question 88
Phenylephrine has been proven to be an effective choice of vasopressor to mitigate the
hypotension seen during spinal anaesthesia for caesarean section. It has the following
characteristics:
a. Structurally similar to epinephrine
b. Significant effects on β-adrenergic receptors
c. It is metabolized by monoamine oxidase (MAO) and catechol-О-methyl transferase
(COMT)

Downloaded from https://www.cambridge.org/core. University of Edinburgh, on 19 Aug 2019 at 13:21:27, subject to the Cambridge Core terms of
use, available at https://www.cambridge.org/core/terms. https://doi.org/10.1017/9781108566100.007
Chapter 3a: Pharmacology Questions 197

d. Exhibits tachyphylaxis on repeated dosing


e. Administered intravenously it has a duration of action of about 6–8 minutes

Question 89
Concerning vasodilators:
a. Hydralazine causes arterial and venous dilatation to the same extent
b. Sodium nitroprusside predominantly causes venodilatation
c. One of the metabolites of sodium nitroprusside, thiocyanate, is non-toxic
d. Sodium nitroprusside toxicity is more likely in patients with vitamin B12 deficiency
e. Vitamin B12 is an accepted treatment for sodium nitroprusside toxicity

Question 90
Concerning nitrates:
a. Glyceryl trinitrate (GTN) should be used with caution in patients with severe hypoxia
b. GTN acts by activating adenylate cyclase
c. GTN has a high oral bioavailability
d. Isosorbide mononitrate has a high oral bioavailability
e. GTN can occasionally precipitate methaemaglobinaemia

Question 91
The following drugs decrease pulmonary vascular resistance (PVR).
a. Epoprostenol
b. Sodium nitroprusside
c. Noradrenaline
d. Isoprenaline
e. Sevoflurane

Question 92
Thiazide diuretics have the following side effects:
a. Hypokalaemic hypochloraemic acidosis
b. Hypercalcaemia
c. Hypouricaemia
d. Precipitate pancreatitis
e. Hyperglycaemia

Question 93
Indications for ACE inhibitors include:
a. Acute myocardial infarction with left ventricular dysfunction
b. Essential hypertension
c. Pregnancy-induced hypertension
d. Severe heart failure
e. Diabetic nephropathy

Downloaded from https://www.cambridge.org/core. University of Edinburgh, on 19 Aug 2019 at 13:21:27, subject to the Cambridge Core terms of
use, available at https://www.cambridge.org/core/terms. https://doi.org/10.1017/9781108566100.007
198 Chapter 3a: Pharmacology Questions

Question 94
Nifedipine:
a. Is a dihydropyridine
b. Undergoes extensive first pass metabolism
c. Causes a reflex increase in heart rate
d. Has no effect on coronary blood flow
e. Increases the MAC of volatile anaesthetics

Question 95
Regarding atenolol:
a. It is safe in diabetics
b. It is a non selective β-blocker
c. It is safe in asthmatics
d. It is a negative inotrope
e. The dose should be reduced in renal failure

Question 96
The following antiarrhythmics are correctly matched to the Vaughan-Williams classification:
a. Class IV – flecainide
b. Class Ia – procainamide
c. Class III – amiodarone
d. Class Ib – quinidine
e. Class Ic – lidocaine

Question 97
Adenosine:
a. Has a duration of action of 2 minutes
b. Is a purine nucleoside
c. Is a yellow solution
d. Is contraindicated in second-degree heart block
e. Should be used with caution in heart transplant patients

Question 98
Side effects of amiodarone include:
a. Irreversible corneal microdeposits
b. Photosensitivity
c. Pneumonitis
d. Peripheral neuropathy
e. Raised serum transaminases

Downloaded from https://www.cambridge.org/core. University of Edinburgh, on 19 Aug 2019 at 13:21:27, subject to the Cambridge Core terms of
use, available at https://www.cambridge.org/core/terms. https://doi.org/10.1017/9781108566100.007
Chapter 3a: Pharmacology Questions 199

Question 99
Side effects of furosemide include:
a. Hypocalcaemia
b. Hyperglycaemia
c. Metabolic acidosis
d. Hypomagnesaemia
e. Hypotension

Question 100
The following mechanisms are thought to explain the actions of the following types of
diuretic:
a. Bendroflumethiazide and inhibition of sodium–chloride symport channels in the distal
convoluted tubule
b. Furosemide and inhibition of sodium–chloride symport channels in the loop of Henle
c. Amiloride and competitive antagonism of aldosterone
d. Mannitol and increased renal plasma flow
e. Metolazone and inhibition of carbonic anhydrase

Question 101
Regarding bendroflumethiazide:
a. It is safe to use in pregnancy-induced hypertension
b. It is the first-line antihypertensive in patients of African or Caribbean family origin
c. Is available in oral and intravenous preparations
d. Is ineffective when GFR falls below 30 ml.min–1.1.73 m–2
e. May be given with amiloride as co-amilozide to reduce the risk of hypokalaemia

Question 102
Spironolactone:
a. Has a steroidal structure
b. Is a non-competitive inhibitor of aldosterone
c. Causes gynaecomastia in males
d. Is hepatically metabolized and excreted mainly by the kidneys
e. Can be useful in an Addisonian crisis

Question 103
Warfarin:
a. Prevents the formation of reduced vitamin K
b. Is 95% protein bound
c. Metabolism is reduced by erythromycin
d. Can be reversed by prothrombin complex concentrates
e. Metabolism is enhanced by amiodarone

Downloaded from https://www.cambridge.org/core. University of Edinburgh, on 19 Aug 2019 at 13:21:27, subject to the Cambridge Core terms of
use, available at https://www.cambridge.org/core/terms. https://doi.org/10.1017/9781108566100.007
200 Chapter 3a: Pharmacology Questions

Question 104
The following statements regarding heparin are true:
a. Low molecular weight heparin has a shorter half-life than unfractionated heparin
b. Unfractionated heparin administration is associated with a lower incidence of heparin-
induced thrombocytopenia
c. 10 mg of protamine reverses 100 units of heparin
d. Heparin binds to antithrombin and inhibits the action of thrombin
e. Heparin occurs naturally in the liver and mast cell granules

Question 105
Regarding drugs affecting coagulation:
a. Lepirudin is an indirect thrombin inhibitor
b. Alteplase is a recombinant tissue-type plasminogen activator
c. Aprotinin has been withdrawn due to a risk of cardiac arrhythmias
d. Tranexamic acid enhances the conversion of plasminogen to active plasmin
e. Rapid administration of protamine may cause acute hypotension, bradycardia and
flushing

Question 106
Central neuraxial blockade is contraindicated in the following:
a. 12 hours following therapeutic LMWH administration
b. Regular aspirin administration 75 mg once daily
c. Regular clopidogrel administration 75 mg once daily
d. Warfarin stopped 3 days ago, INR 1.7 on day of surgery
e. 12 hours following fondaparinux administration

Question 107
Clopidogrel:
a. Reversibly prevents ADP binding to its platelet receptor
b. Prevents activation of the glycoprotein IIb/IIIa complex
c. Has fewer gastrointestinal side effects than aspirin
d. Needs to be stopped 7 days prior to surgery
e. Is a coronary vasodilator

Question 108
Dipyridamole:
a. Inhibits adenosine uptake by platelets
b. Activates platelet phospodiesterase
c. Results in lower levels of cAMP in platelets
d. At low dose potentiates the activity of prostacyclin
e. Is a monotherapy agent for the prevention of stroke

Downloaded from https://www.cambridge.org/core. University of Edinburgh, on 19 Aug 2019 at 13:21:27, subject to the Cambridge Core terms of
use, available at https://www.cambridge.org/core/terms. https://doi.org/10.1017/9781108566100.007
Chapter 3a: Pharmacology Questions 201

Question 109
Concerning antiplatelet agents:
a. Glycoprotein IIb/IIIa inhibitors block platelet adhesion
b. Glycoprotein IIb/IIIa inhibitors are used in conjunction with unfractionated heparin
c. Glycoprotein IIb/IIIa inhibitors are primarily indicated in the treatment of ST elevation
myocardial infarction
d. Epoprostenol is used in the anticoagulation of haemofiltration circuits
e. Dextrans specifically inhibit von Willebrand’s factor to cause their anticoagulant effect

Question 110
Regarding tranexamic acid:
a. Potentiates the action of plasmin
b. Is one of the WHO Essential Medicines
c. Is more efficacious than aminocaproic acid
d. Should be given as early as possible if used in major trauma
e. Is contraindicated in patients with thromboembolic disease

Question 111
Aprotinin:
a. Has a bioavailability of approximately 80%
b. Affects the extrinsic pathway of the clotting cascade
c. Is extracted from bovine tissue
d. Is proven to be safer than tranexamic acid when used during CABG
e. Is both metabolized and excreted by the kidney

Question 112
Regarding the thromboelastogram (TEG):
a. It allows a dynamic assessment of the coagulation cascade
b. The maximal amplitude (MA) can be used to estimate platelet function
c. Is a form of near patient testing
d. Can be used to diagnose platelet dysfunction from von Willebrand’s disease or from
platelet inhibitors, e.g. aspirin, clopidogrel
e. Modern versions can distinguish between abnormalities in the intrinsic or extrinsic
pathways

Question 113
Regarding the content of intravenous fluids, the following statements are correct:
a. 500 ml of 20% mannitol contains approximately 100 g of mannitol
b. ®
500 ml of Gelofusine contains 77 mmol of sodium ions
c.
d.
®
1000 ml of 6% Volulyte contains 137 mmol sodium ions, 4 mmol potassium ions
1000 ml of 0.9% saline contains 154 mmol of sodium and chloride ions
e. 1000 ml of 8.4% bicarbonate contains 1000 mmol of sodium and bicarbonate ions

Downloaded from https://www.cambridge.org/core. University of Edinburgh, on 19 Aug 2019 at 13:21:27, subject to the Cambridge Core terms of
use, available at https://www.cambridge.org/core/terms. https://doi.org/10.1017/9781108566100.007
202 Chapter 3a: Pharmacology Questions

Question 114
With respect to an infusion of 0.9% saline to an adult patient in the perioperative period:
a. Infusing a 1000 ml bag will distribute 250 ml to the plasma volume and 750 ml into the
interstitial fluid compartment
b. It will result in the development of hyperchloraemic alkalosis
c. It will increase in GFR
d. It will be an infusion of a solution with the same pH as the extracellular fluid of the
patient
e. It can cause hypokalaemia

Question 115
The following statements regarding blood products are true:
a. Citrate supports red cell metabolism
b. Platelets are stored at 4 °C
c. Cryoprecipitate contains factor VIII and fibrinogen
d. The usual dose of FFP is 15 ml.kg–1
e. Packed red cells have an increased amount of 2,3-DPG

Question 116
Complications resulting from blood product transfusion include:
a. Hyperkalaemia
b. Hypercalcaemia
c. Metabolic alkalosis
d. Acute respiratory distress syndrome
e. Coagulopathy

Question 117
The following fluids are the correct management for the clinical situation described:
a. Diabetic ketoacidosis and 0.9% saline
b. Acute upper GI massive haemorrhage and 5% dextrose
c. Major sepsis and hydroxyethyl starches
d. Neurogenic diabetes insipidus and 5% dextrose
e. Severe vomiting and diarrhoea and 0.9% saline

Question 118
The following statements are correct:
a. Sodium chloride 0.18%/4% glucose contains 50 mmol.l–1 of sodium ions
b. Human albumin 4.5% is isotonic
c. Colloids distribute throughout the total body water
d. 1000 ml of Hartmann’s contains 29 mmol of lactate
e. A 1 litre bag of infused 5% dextrose leaves approximately 85 ml in the intravascular fluid
compartment

Downloaded from https://www.cambridge.org/core. University of Edinburgh, on 19 Aug 2019 at 13:21:27, subject to the Cambridge Core terms of
use, available at https://www.cambridge.org/core/terms. https://doi.org/10.1017/9781108566100.007
Chapter 3a: Pharmacology Questions 203

Question 119
Concerning phenytoin:
a. It has a membrane stabilizing effect
b. It is a Class 1c antiarrhythmic agent
c. It is a cytochrome P450 enzyme inhibitor
d. It has good oral bioavailability
e. Skin eruptions and gingival hyperplasia are signs of toxicity

Question 120
Concerning antiepileptic drugs:
a. Levetiracetam is active at GABA channels
b. Levetiracetam has a role in the treatment of myoclonic seizures
c. Sodium valproate is mainly active at GABA channels
d. Carbamazepine has a role in the management of trigeminal neuralgia
e. Carbamazepine is a cytochrome P450 enzyme inhibitor

Question 121
Concerning drugs affecting intraocular pressure:
a. Prostaglandin analogues increase the outflow of fluid from the uveal–scleral tract to
decrease intraocular pressure
b. Acetazolomide reduces intraocular pressure by an osmotic effect
c. Timolol acts via a similar mechanism to prostaglandins
d. Suxamethonium increases intraocular pressure
e. Ketamine decreases intraocular pressure

Question 122
Regarding magnesium sulfate:
a. It is effective in the treatment of polymorphic ventricular tachycardia
b. It is the first-line antihypertensive treatment for severe pre-eclampsia
c. Common side effects of treatment include hyporeflexia and muscle weakness
d. Toxic levels can be reversed by the administration of calcium
e. It shortens the duration of action of neuromuscular blocking agents

Question 123
With reference to the magnesium ion:
a. It is one of the most abundant extracellular cations in the body
b. It promotes thrombin-induced platelet aggregation
c. It exerts renal vasodilator and diuretic effects
d. Deficiency may be seen in up to 60% of critically ill patients
e. It is a powerful bronchodilator

Downloaded from https://www.cambridge.org/core. University of Edinburgh, on 19 Aug 2019 at 13:21:27, subject to the Cambridge Core terms of
use, available at https://www.cambridge.org/core/terms. https://doi.org/10.1017/9781108566100.007
204 Chapter 3a: Pharmacology Questions

Question 124
Salbutamol:
a. Increases intracellular cAMP concentrations
b. May cause hyperglycaemia
c. Also has a tocolytic action
d. Is not a cause of lactic acidosis
e. Can cross the placenta

Question 125
The following statements regarding the management of acute severe asthma are true:
a. Steroids reduce mortality
b. Combining ipratropium with salbutamol results in significantly greater bronchodilation
than salbutamol alone
c. Heliox is an effective treatment
d. Aminophylline loading dose is 15 mg.kg–1
e. Magnesium sulfate dose is 8 mmol

Question 126
Prednisolone has the following pharmacodynamic and pharmacokinetic properties:
a. Oral bioavailability 80–100%
b. Is reversibly bound to albumin in the plasma
c. Decreases the number of β-adrenoreceptors
d. Decreases potassium excretion
e. Stimulates gluconeogenesis

Question 127
Carbocisteine is a mucolytic agent often prescribed to patients with chronic obstructive
pulmonary disease (COPD). The following statements are correct:
a. Carbocisteine is poorly absorbed orally
b. Carbocisteine has antioxidant properties
c. Penetration of lung tissue by carbocisteine is poor
d. It has no action on gastric mucus levels
e. N-acetylcysteine is a mucolytic agent

Question 128
Concerning prokinetics:
a. Erythromycin reduces the effectiveness of warfarin
b. Erythromycin causes prolongation of the QT interval
c. Metoclopramide has direct effects on gastric smooth muscle
d. Metoclopramide has no cholinergic effect
e. Metoclopramide can precipitate oculogyric crisis

Downloaded from https://www.cambridge.org/core. University of Edinburgh, on 19 Aug 2019 at 13:21:27, subject to the Cambridge Core terms of
use, available at https://www.cambridge.org/core/terms. https://doi.org/10.1017/9781108566100.007
Chapter 3a: Pharmacology Questions 205

Question 129
Concerning antacids:
a. Ranitidine acts as a cytochrome P450 enzyme inhibitor
b. Ranitidine is an H1 antagonist
c. Proton pump inhibitors (PPIs) are more effective than histamine antagonists in gastric
acid suppression
d. PPIs inhibit the gastric parietal luminal H+/K+ ATPase enzyme
e. PPIs are prodrugs

Question 130
Concerning antiemetics:
a. Metoclopramide acts both centrally and peripherally
b. Motion sickness is best treated by using drugs acting on the chemoreceptor trigger zone
c. Rapid administration of cyclizine causes tachycardia
d. Dexamethasone has potent antidopaminergic activity
e. Cannabinoids have a role in the treatment of chemotherapy-mediated emesis

Question 131
Concerning laxatives:
a. Lactulose directly stimulates bowel peristalsis
b. Lactulose has a role in the secondary prevention of hepatic encephalopathy
c. Sodium docusate is a stimulant laxative
d. Phosphate enemas should be used with caution in renal impairment
e. Ispaghula is an ideal agent to use in patients with diverticular disease

Question 132
Concerning antisialogogues:
a. Hyoscine hydrobromide is useful in the treatment of spasmodic gut pain
b. Atropine has greater sedative properties than glycopyrronium bromide
c. Glycopyrronium bromide is administered together with neostigmine to reduce its
nicotinic side effects when reversing neuromuscular blockade
d. Atropine terminates salivary gland secretion to a greater extent than glycopyrronium
bromide
e. Glycopyrronium bromide is less likely than atropine to cause blurred vision

Question 133
Regarding uterotonic drugs:
a. Syntometrine is contraindicated in asthmatics
b. Since 2004, the National Institute of Health and Care Excellence recommend that all
women are given oxytocin 5 IU following delivery to improve uterine contraction and
reduce blood loss

Downloaded from https://www.cambridge.org/core. University of Edinburgh, on 19 Aug 2019 at 13:21:27, subject to the Cambridge Core terms of
use, available at https://www.cambridge.org/core/terms. https://doi.org/10.1017/9781108566100.007
206 Chapter 3a: Pharmacology Questions

c. A Syntocinon infusion of 1 IU.h–1 is recommended following caesarean delivery for all


®
multiparous patients
d. Carboprost can be used before or after delivery to reduce risk of postpartum
haemorrhage
e. Atosiban is more effective than carboprost at reducing postpartum haemorrhage

Question 134
The following drugs are recognized tocolytics:
a. Ritodrine
b. Magnesium sulfate
c. Terbutaline
d. Gylceryl trinitrate
e. Slow-release nifedipine

Question 135
The following statements with regards to antifungal agents are true:
a. Amphotericin is nephrotoxic
b. Azoles all work by stimulating ergosterol synthesis
c. Posaconazole has the broadest spectrum of activity with fewest drug interactions
d. Fluconazole has poor activity against yeast infections
e. Hepatic dysfunction is associated with all of the azoles

Question 136
The following antibiotics are bacteriostatic:
a. Erythromycin
b. Clindamycin
c. Glycopeptides
d. Ciprofloxacin
e. Aminoglycosides

Question 137
With regards to antibiotic resistance:
a. Gene transfer is the main mechanism by which antimicrobials acquire resistance
b. Mutation is the main mechanism by which antimicrobials acquire resistance
c. Organisms with intrinsic resistance often have low virulence
d. The use of broad-spectrum antibiotics generates resistance
e. Antibiotic misuse of cephalosporins and quinolones has led to more virulent strains of
Clostridium difficile

Question 138
The following statements about antibiotics are true:
a. Ganciclovir is the treatment for CMV in the immunocompromised host

Downloaded from https://www.cambridge.org/core. University of Edinburgh, on 19 Aug 2019 at 13:21:27, subject to the Cambridge Core terms of
use, available at https://www.cambridge.org/core/terms. https://doi.org/10.1017/9781108566100.007
Chapter 3a: Pharmacology Questions 207

b. Metronidazole inhibits protein synthesis


c. Vancomycin does not cause histamine release
d. Aminoglycosides decrease non-depolarizing muscle relaxant duration of action
e. Meropenem is a β-lactam antibiotic

Question 139
Insulin:
a. Is rapidly metabolized by glutathione insulin transhydrogenase
b. Causes fat deposition in adipose tissue
c. Has some antihyperglycaemic action when administered orally
d. Is used in the management of hypokalaemia
e. Is removed by haemodialysis

Question 140
The following statements are correct:
a. Sulfonylureas are effective in insulin-dependent diabetics
b. Metformin lowers plasma cholesterol and triglyceride levels
c. Action of glibenclamide may be potentiated by atenolol
d. Metformin can delay glucose uptake from the gut
e. Use of metformin may be complicated by lactic acidosis

Question 141
Regarding newer hypoglycaemic agents:
a. Rosiglitazone acts at the PPAR γ1 and γ2 receptors
b. Thiazolidinediones (glitazones) increase levels of TNFα
c. Sitagliptin works by blocking the action of incretin
d. Sitagliptin has agonist action at the DPP4 receptor
e. Pioglitazone can be used safely in patients with congestive cardiac failure

Question 142
Thyroxine is a thyroid hormone that is also administered exogenously in patients with
hypothyroidism. It exerts a series of physiological effects including the following:
a. Increases stroke volume
b. Decreases sweating
c. Increases neonatal lung maturation
d. Increases non-HDL cholesterol levels
e. Alteration in spermatogenesis

Question 143
Carbimazole is a prodrug used in the treatment of hyperthyroidism. Recognized complica-
tions of such therapy include:
a. Maculopapular rash

Downloaded from https://www.cambridge.org/core. University of Edinburgh, on 19 Aug 2019 at 13:21:27, subject to the Cambridge Core terms of
use, available at https://www.cambridge.org/core/terms. https://doi.org/10.1017/9781108566100.007
208 Chapter 3a: Pharmacology Questions

b. Myopathy
c. Agranulocytosis
d. Cholestatic hepatitis
e. Type III hypersensitivity reaction

Question 144
There is a range of oral corticosteroids available and these have equivalent anti-inflammatory
doses. Prednisolone 5 mg is equivalent to:
a. Dexamethasone 4 mg
b. Hydrocortisone 20 mg
c. Methylprednisolone 1 g
d. Triamcinolone 4 mg
e. Betamethasone 750 μg

Question 145
Endogenous and exogenous glucocorticoids have multisystemic effects and act via intracel-
lular receptors to influence target gene transcription. The following effects are documented
effects of exogenous glucocorticoids:
a. Suppression of appetite
b. Excess glucocorticoids stimulate skeletal growth in children
c. Decrease in libido
d. Prolongation of rapid eye movement sleep
e. Decrease in peripheral conversion of thyroxine (T4) to triiodothyronine (T3)

Question 146
Central nervous system (CNS) stimulants have a wide range of clinical uses and are also
important as drugs of abuse. Psychomotor stimulants produce euphoria and excitement.
The following are pharmacological characteristics of these agents:
a. Caffeine competitively inhibits phosphodiesterase
b. Caffeine does not cross the placenta
c. Nicotine does not cross the blood–brain barrier
d. Amphetamines are metabolized by catechol-O-methyltransferase
e. Methylphenidate which can be used to treat attention deficit hyperactivity disorder is
a reversible dopamine reuptake inhibitor

Question 147
Doxapram is an analeptic or respiratory stimulant. It has the following characteristics:
a. Increases the depth but not the rate of respiration
b. The major metabolite is 2-ketodoxapram
c. Causes an increase in cardiac output
d. Volume of distribution is around 1.5 l.kg–1
e. The intravenous preparation contains 0.9% benzyl acetate

Downloaded from https://www.cambridge.org/core. University of Edinburgh, on 19 Aug 2019 at 13:21:27, subject to the Cambridge Core terms of
use, available at https://www.cambridge.org/core/terms. https://doi.org/10.1017/9781108566100.007
Chapter 3a: Pharmacology Questions 209

Question 148
When considering effects of nicotine and tobacco smoking in the perioperative setting:
a. There is evidence that expired carbon monoxide (CO) concentration, which is an
indicator of recent smoking, is correlated with the frequency of ST depression during
general anaesthesia
b. The incidence of postoperative nausea and vomiting is higher in smokers
c. Nicotine reduces postoperative acute pain
d. The sympathomimetic effects of nicotine on the heart last for longer than 6 hours
e. Short-term abstinence from smoking decreases the incidence of postoperative pul-
monary complications

Question 149
The consumption of alcohol both acutely and chronically leads to several important effects
that need to be considered in relation to the provision of general anaesthesia:
a. Ethanol depresses the responses mediated by the γ amino-butyric acid a type (GABAA)
receptor
b. Wernicke’s encephalopathy is a neurological disorder in chronic alcoholics caused by
vitamin B12 deficiency
c. Clonidine should be avoided in chronic alcoholics
d. There is a significant decrease in delayed hypersensitivity that contributes to increased
infection rates postoperatively
e. Plasma testosterone levels are usually elevated in those who are chronically dependent
on alcohol

Question 150
MDMA (3,4-methylenedioxy-N-methylamphetamine) (Ecstasy) and cocaine are widely
abused central nervous stimulants. There are several important pharmacological effects
that need to be considered by the anaesthetist in the perioperative setting:
a. If a vasopressor agent is required in a patient who has used cocaine recently then
phenylephrine should be avoided
b. The use of MDMA can result in excessive production of arginine vasopressin
c. Cocaine use is associated with delayed gastric emptying
d. Hypernatraemia is a common complication of MDMA use
e. β-Blockers are relatively contraindicated in cocaine-induced hypertension

Downloaded from https://www.cambridge.org/core. University of Edinburgh, on 19 Aug 2019 at 13:21:27, subject to the Cambridge Core terms of
use, available at https://www.cambridge.org/core/terms. https://doi.org/10.1017/9781108566100.007
Chapter
Pharmacology Answers

3b
Question 1: TTFTF
Racemic mixtures are mixtures of different enantiomers in equal proportions. While the
mixture may contain equal amounts of the two isomers, the contribution to activity
(pharmacodynamic and pharmacokinetic) may be very different. One isomer may be
completely inactive or, at worst, be responsible for toxicity and undesirable side effects.
All of the volatile agents, with the exception of sevoflurane, are racemic mixtures. Other
examples include racemic bupivacaine, ketamine, atropine and racemic epinephrine.
Levobupivacaine, by virtue of the fact that it is one of the optical isomers of bupivacaine,
is enantiopure (i.e. not a mix of two isomers). This selection of the more desirable moiety
from the racemic mixture improves the safety profile of the drug, by using the isomer with
fewer side effects and less chance of toxicity.

Question 2: TFTFF
S(+)-ketamine has several advantages over racemic ketamine. It produces less intense
(although no less frequent) emergence phenomena. It also has greater affinity for the
NMDA receptor than R(–)-ketamine, meaning that it is three times as potent an analgesic.
S(+)-ketamine is thought to produce less direct cardiac depression; therefore the risk of
cardiac ischaemia is lower. Recovery is more rapid with S(+)-ketamine.
The single advantage of levobupivacaine (the S-enantiomer) over bupivacaine and other
local anaesthetics is its potential for reduced toxicity. It has two useful properties: firstly, the
dose required to produce myocardial depression is higher for levobupivacaine and, sec-
ondly, excitatory CNS effects or convulsions occur at lower doses with bupivacaine than
levobupivacaine.
Ropivacaine is prepared as the pure S-enantiomer (the R-enantiomer is less potent and more
toxic). The main differences between it and bupivacaine lie in its pure formula, improved side
effect profile and lower lipid solubility. This lower lipid solubility may result in reduced
penetration of the large myelinated Aβ motor fibres, so that these are initially spared.

Question 3: FFFTF
Since only the unbound fraction of a drug in the plasma is free to cross the cell
membrane, it is in fact the degree of protein binding that determines the duration of
drug action (the higher the degree of protein binding, the greater the duration of
action). The lipophilic cell membrane will only allow the passage of the uncharged
fraction of a drug. The degree of drug ionization depends on the molecular structure of

210
Downloaded from https://www.cambridge.org/core. University of Edinburgh, on 19 Aug 2019 at 13:21:27, subject to the Cambridge Core terms of
use, available at https://www.cambridge.org/core/terms. https://doi.org/10.1017/9781108566100.008
Chapter 3b: Pharmacology Answers 211

the drug and the pH of the solution in which it is dissolved. Ionization therefore
determines the speed of onset of the drug action.
Although lipid solubility reflects the ability of a drug to cross the cell membrane, it does
not necessarily equal rapid onset of action. Alfentanil is almost seven times less lipid-soluble
than fentanyl yet its onset is much more rapid. This is due to various factors, including the
fact that it has a smaller volume of distribution and a lower pKa than fentanyl (meaning that
at physiological pH a greater fraction of alfentanil is unionized). Aspirin is almost wholly
ionized at physiological pH as it is an acid. However, in the acidic environment of the
stomach, pH is closer to the pKa of aspirin and the drug becomes essentially unionized, thus
increasing its rate of absorption. Applying the same principles to the addition of sodium
bicarbonate to lidocaine, by raising the pH of the solution, this increases the proportion of
unionized local anaesthetic, enabling it to penetrate nerve membranes more readily. Thus,
speed of onset is increased. The degree of protein binding will affect placental transfer.
Bupivacaine is more highly bound than lidocaine, so less crosses the placenta. If the fetus
becomes acidotic there will be an increase in the ionized fraction and local anaesthetic will
accumulate in the fetus (ion trapping).

Question 4: FTTTF
Dose is plotted on the x-axis and the response on the y-axis. The log10 dose–response curve
can be used to determine the potency of a drug. In general, the more potent a drug, the further
to the left it will lie on a dose–response curve and indeed the steeper the curve will be. The
ED50 can be determined from the log10 dose–response curve and used to define potency. This
is the dose of the drug that produces 50% of the maximal response. Lower efficacy of a drug is
exhibited when the curve does not reach the same plateau as another drug in comparison.
The effect of an antagonist on the dose–response curve of an agonist is to shift it to the
right. There are two types of antagonist, competitive and non-competitive. The action of a
competitive antagonist can be overcome by increasing the dose of the agonist. However, in
the case of non-competitive antagonism this is not possible due to the fact that the binding
sites are different.

Question 5: FFFFF
Phenylephrine is a direct-acting sympathomimetic α1 receptor agonist. It causes an increase
in blood pressure due to an increase in the systemic vascular resistance with an associated
reflex bradycardia.
Doxazosin is an α-adrenergic blocking agent that is used in the treatment of essential
hypertension.
The antiemetic drug ondansetron is highly selective as an antagonist at the 5HT3
receptor. It has actions centrally and peripherally.
Ketamine is a non-competitive antagonist at the NMDA-type glutamate receptor. It
causes a dose-dependent depression of the CNS, resulting in a dissociative state character-
ized by analgesia and amnesia.
Dexmedetomidine is a specific α2-adrenergic receptor agonist that may be used for the
short-term sedation of ventilated patients on an intensive care unit. As a single agent it may
provide effective sedation, analgesia and anxiolysis with a stable respiratory rate.

Downloaded from https://www.cambridge.org/core. University of Edinburgh, on 19 Aug 2019 at 13:21:27, subject to the Cambridge Core terms of
use, available at https://www.cambridge.org/core/terms. https://doi.org/10.1017/9781108566100.008
212 Chapter 3b: Pharmacology Answers

Question 6: FFFTF
The therapeutic window is the range of doses of a drug that leads to a therapeutic response
in the absence of toxic effects. This can be quantified by the use of the therapeutic index,
which is a ratio of LD50:ED50, hence it has no units. For greater safety the drug should have a
high therapeutic index.
Warfarin, vancomycin and digoxin are examples of drugs with a low therapeutic index.
Penicillin and ibuprofen have a high therapeutic index.

Question 7: FTFTF
The efficacy of a partial agonist is greater than zero but less than the full agonist, despite full
receptor occupancy. Antagonists require the presence of an agonist or partial agonist to
exert their effect. The fact that an agent may have affinity does not mean that it will have
efficacy. Once binding has occurred then the ability to bind and the size of response are
associated, but this is not a linear relationship. Partial agonists and full agonists bind to the
same site on the receptor and thus can reduce the effect of the full agonist. This means that
partial agonists can act as competitive antagonists. Buprenorphine is a partial agonist at the
μ-opioid receptor and as such is used in opioid addiction programmes.

Question 8: FTTFT
G-protein-coupled receptors (GPCRs) are intracellular trimeric proteins associated with a
transmembrane receptor. The trimer consists of α, β and γ subunits. Extracellular receptor
binding causes activation of the GPCR. The α subunit of the trimer is activated and
substitutes GDP for GTP. This causes splitting of the trimer and activation of downstream
cell signalling cascades. The physiological effects depend on the ligand binding the receptor
and the receptor type. Receptor variation arises via variances in the α subtype. Gi GPCR
activation results in the deactivation of adenylyl cyclase and a reduction in cAMP levels.
Opiates are agonists at the Gi GPCR. α1-Adrenoreceptors are of the Gq subtype and binding
of an agonist results in the activation of protein kinase C. α2-Adrenoreceptors are of the Gi
subtype.

Question 9: TFFFT
The differences between tachyphylaxis, desensitization and tolerance are subtle but impor-
tant to recognize. Ephedrine is an example of a drug that, with repeated doses, displays
tachyphylaxis. The rapid loss of response to repeated doses of ephedrine is attributed to
depletion of noradrenaline stores at sympathetic nerve terminals, which ephedrine indir-
ectly stimulates. Tolerance is the phenomenon associated with progressively larger doses of
drug being needed to produce the same biological effect. Theoretically, the maximal
biological response is still possible. Opiate abuse is an example of this. In contrast, desensi-
tization and tachyphylaxis cause a reduction in maximal biological effect. With tachyphy-
laxis the mechanism is due to the depletion of a messenger intermediate. Desensitization
results in qualitative or quantitative deficiency in receptors.

Downloaded from https://www.cambridge.org/core. University of Edinburgh, on 19 Aug 2019 at 13:21:27, subject to the Cambridge Core terms of
use, available at https://www.cambridge.org/core/terms. https://doi.org/10.1017/9781108566100.008
Chapter 3b: Pharmacology Answers 213

Question 10: TFFTF


Acetylcholinesterase inhibitors have direct everyday relevance to the anaesthetist, therefore
detailed pharmacological knowledge is expected. Edrophonium reversibly binds to the
anionic and esteratic sites of acetylcholinesterase, rendering the enzyme incapable of
catalyzing the breakdown of acetylcholine. The relative instability of the complex accounts
for the short duration of action. It therefore has a diagnostic rather than a treatment role in
the management of myasthenia gravis. Neostigmine binds to the anionic site of acetylcho-
linesterase and results in the formation of a carbamylated enzyme complex. It is an analogue
of the naturally occurring compound physostigmine. Both of these agents are known as
‘carbamates’, and are related to carbamic acid. The carbamylated complex has greater
stability than the acetylcholinesterase–edrophonium complex. Therefore the duration of
action of carbamates is longer. Neostigmine is used to reverse non-depolarizing neuromus-
cular blockade. Paradoxically, it can prolong the duration of action of suxamethonium due
to its additional inhibition of plasma cholinesterase. Neostigmine is also used in the
treatment of myasthenia gravis.
One of several hypotheses for the aetiology of Alzheimer’s disease is depletion of
acetylcholine as a central neurotransmitter. Rivastigmine and galantamine are other carba-
mates used in the treatment of this disease. Organophosphates irreversibly phosphorylate
acetylcholinesterase, and victims of organophosphate poisoning may die from the subse-
quent cholinergic effects. Treatment includes administration of pralidoxime, which hydro-
lyzes the phosphorylated enzyme and promotes recovery. Dicobalt edetate is the treatment
for cyanide poisoning.

Question 11: FTTTF


Lidocaine is a Class 1b antiarrhythmic agent. By blocking fast sodium channels it reduces
the rate of rise of membrane depolarization and increases the membrane depolarization
threshold (prolongation of phase 0). The refractory period is reduced owing to faster
repolarization (phase 3). Flecainide (Class 1c) similarly blocks sodium channels resulting
in a prolonged phase 0. However, it has no effect on the refractory period. Verapamil is a
calcium channel blocker with a predilection for the pacemaker ion channels. It has less effect
on vascular smooth muscle, although it does cause a degree of coronary artery vasodilata-
tion. By blocking slow L calcium channels in the SA and AV nodes, the degree of sponta-
neous membrane depolarization is reduced, therefore leading to reduced automacity.
Amiodarone is primarily a Class 3 agent and a potassium channel blocker. The result is a
slower repolarization rate (phase 3) and consequently a longer refractory period and action
potential.

Question 12: TFTFF


G-proteins are cell membrane proteins that mediate the intracellular response to coupled
membrane receptors. They consist of three subunits: α, β and γ. The GABAA receptor is a
member of the pentameric family of receptors and has five subunits, which span the cell
membrane. Another example from this family is the nicotinic acetylcholine receptor at the
neuromuscular junction. There are at least four different types of receptors that co-ordinate
biochemical events within the cell:

Downloaded from https://www.cambridge.org/core. University of Edinburgh, on 19 Aug 2019 at 13:21:27, subject to the Cambridge Core terms of
use, available at https://www.cambridge.org/core/terms. https://doi.org/10.1017/9781108566100.008
214 Chapter 3b: Pharmacology Answers

Type 1: Ligand gated ion channel. Examples: nicotinic acetylcholine receptor and GABAA.
Type 2: G-protein coupled receptor. Examples: muscarinic acetylcholine receptor and
opioid receptors.
Type 3: Enzymes e.g. tyrosine kinase. Example: insulin receptor.
Type 4: Intracellular receptors (which act via gene transcription). Examples: steroid and
thyroid hormones.

Question 13: FTFFT


An adverse drug reaction may be defined as ‘the occurrence of any drug effect that is not of
therapeutic, diagnostic or prophylactic benefit to the patient’. They may be subdivided into
type A and B reactions. Type A reactions are predictable and dose dependent, e.g. hypoten-
sion following propofol administration. Type B reactions are also known as idiosyncratic
drug reactions. These are less common than Type A reactions. They are unpredictable, dose
independent and unrelated to the known pharmacological properties of a drug. These
reactions usually involve the immune system. Anaphylaxis (IgE mediated) and anaphylac-
toid (non-IgE mediated) reactions are examples of Type B drug reactions. All unexpected or
life-threatening adverse drug reactions should be reported to the MHRA.

Question 14: TTTFF


Cytochrome P450 is found in the smooth endoplasmic reticulum of hepatocytes. Inhibitors
and inducers of this enzyme system cause drug interactions; inhibition will result in
reduced/delayed metabolism of other cytochrome-metabolized drugs, whereas inducers
will shorten the effect of other cytochrome-metabolized drugs. Cytochrome P450 inducers
in common use include phenytoin, carbamazepine, rifampicin, chronic alcohol, barbitu-
rates and cigarette smoking. Cytochrome P450 inhibitors in common use include erythro-
mycin, metronidazole, omeprazole, amiodarone, grapefruit juice and cyclosporin.

Question 15: TTFTT


Moclobemide is a reversible monoamine oxide (MAO) inhibitor used to treat depression. It
selectively inhibits MAO-A, resulting in a reduced breakdown of serotonin, noradrenaline
and dopamine. Selective MAO-A inhibitors are thought to be safer than the older, non-
selective MAO inhibitors. These were associated with potential hypertensive crises when
taken in conjunction with tyramine-containing foods (the ‘cheese reaction’). Tyramine is
also metabolized by MAO, and tyramine accumulation can cause hypertension via potential
displacement of noradrenaline at nerve terminals. Selective MAO-A inhibitors allow MAO-
B to continue to function and clear dietary tyramine. However, a potential hypertensive
crisis may still be precipitated by co-administration of indirectly acting sympathomimetic
agents, e.g. ephedrine and metaraminol. ACE inhibitors and non-steroidal drugs may
cause acute kidney injury, especially in the elderly and during the perioperative period.
Warfarin and grapefruit juice may increase the risk of bleeding via cytochrome P450
inhibition. β-Blockers and the non-dihydropyridine calcium channel blockers (verapamil
and diltiazem) can cause profound bradycardia, conduction defects and depression of
myocardial contractility. Metoclopramide is a dopamine antagonist and as such may
cause extrapyramidal side effects and worsen the symptoms of Parkinson’s disease.

Downloaded from https://www.cambridge.org/core. University of Edinburgh, on 19 Aug 2019 at 13:21:27, subject to the Cambridge Core terms of
use, available at https://www.cambridge.org/core/terms. https://doi.org/10.1017/9781108566100.008
Chapter 3b: Pharmacology Answers 215

Question 16: FTTTF


Drug interactions are a common cause of morbidity and mortality. In the same way as
classification of the properties of an individual drug, interactions may be classified into
physicochemical, pharmacokinetic and pharmacodynamic interactions. Physicochemical
interactions result from chemical or physical incompatibility, e.g. the activity of the acidic
heparin is terminated by the strongly basic protamine. Pharmacokinetic interactions occur
due to the effects of co-administered drugs on absorption, distribution, metabolism and
elimination. For example, β-blockers reduce cardiac output, which may reduce the dis-
tribution of other drugs to their site of action. Pharmacodynamic interactions occur as a
result of competition for the binding site of an enzyme or receptor. Competition may be
direct – substance A and B compete for the same site on a receptor (e.g. flumazenil and
benzodiazepines) – or indirect – in the presence of substance A on one site of a receptor,
substance B is unable to bind to another site on a receptor (e.g. neostigmine and acetylcho-
line). Synergism occurs where the net effect of two or more drugs is more than the sum of the
individual actions. For example, in the Oxford league table of analgesic efficiency, the NNT
to cause a 50% reduction in pain is 16.7 with 60 mg of codeine, 3.8 with 1 g of paracetamol,
but only 2.2 with a combination of 60 mg codeine and 1 g paracetamol. The isobolograms
seen with remifentanil and propofol, when given as total intravenous anaesthesia, are
another example of synergism.

Question 17: TTTFT


Most processes involving drugs within the body can be described by first-order kinetics.
This means that plasma levels of the drug are proportional to the amount of drug present
(i.e. an exponential function – the rate of change of drug A is proportional to the concen-
tration of A). The majority of metabolic processes are first-order as there is a relative excess
of enzyme compared to substrate, so enzyme activity is not rate-limiting. In zero-order
kinetics, the rate of change of plasma drug concentration is constant rather than being
dependent on the concentration of drug present. It is otherwise known as saturation
kinetics, indicating that enzyme activity cannot be increased by increasing substrate con-
centration. A good example is the metabolism of ethanol. Humans metabolize ethanol at a
constant rate, regardless of how much we have ingested. This is because the enzyme alcohol
dehydrogenase requires a co-factor for its reaction, which is only present in small amounts.
There is no steady state in zero-order kinetics. If rate of drug delivery exceeds excretion,
plasma levels will continue to rise to toxic levels. Drugs that undergo zero-order kinetics
may therefore have a narrow therapeutic window; a small increase in dose may cause a large
increase in plasma levels, making toxicity more likely. Phenytoin obeys first-order kinetics
at low dose, but zero-order at higher therapeutic doses.

Question 18: TFFTF


The blood–brain barrier (BBB) is an anatomical and functional barrier of tight junctions
between the circulation and the central nervous system. In health, this barrier is tightly
controlled, and the predominant method of molecular transfer is by active transport and
facilitated diffusion. Only lipid-soluble, low molecular weight drugs can cross by simple
diffusion, whilst large, polar molecules cannot.

Downloaded from https://www.cambridge.org/core. University of Edinburgh, on 19 Aug 2019 at 13:21:27, subject to the Cambridge Core terms of
use, available at https://www.cambridge.org/core/terms. https://doi.org/10.1017/9781108566100.008
216 Chapter 3b: Pharmacology Answers

Whilst atropine readily crosses the BBB (it is an uncharged, tertiary amine), glyco-
pyrronium does not, due to its quaternary, charged nitrogen. This means it is far less
likely to produce the centrally mediated confusion or sedation seen with atropine use.
In health, penicillin poorly penetrates the BBB. However, in conditions such as menin-
gitis, the BBB becomes inflamed and compromised. This allows greater permeability for
drugs such as benzylpenicillin and hence allows them to have a more therapeutic
action. Thiopentone is highly lipid-soluble, therefore crosses the BBB easily.
Vecuronium is a large, polar muscle relaxant, which explains why it cannot penetrate
the BBB.

Question 19: FFFTT


A number of pharmacokinetic differences are seen in the elderly population. Older people
have a relative reduction in muscle mass with a resulting increase in the proportion of
adipose tissue; this increases VD.
Hepatic impairment alters many aspects of drug handling. Protein synthesis is decreased,
resulting in reduced protein binding and increased free drug. Phase I and II reactions are
affected, which reduces the metabolism of drugs. Ascites increases VD and portocaval shunts
increase bioavailability by reducing hepatic clearance of drugs. Although it would be
expected that it is necessary to reduce the dose of renally excreted drugs in those with
renal impairment, this may not always be the case. In renal failure, patients may have
significant fluid retention and hence the VD is often increased. The implication of this is that
patients may require a higher loading dose of the drug to achieve desired plasma concen-
trations initially.
In the neonate, the pH of blood tends to be lower than in the adult. Therefore, the acid–
base value will affect the relative proportions of ionized and unionized drug available. Since
lidocaine has a pKa of 7.7, a slightly higher proportion will be ionized in the more acidic
neonatal blood, hence the amount of free unionized drug will be lower.

Question 20: TTFFT


The rapid onset of thiopentone is due to the high blood flow to the brain (hence increased
drug delivery to the desired site of action), the high degree of lipophilicity (therefore making
it able to cross the blood–brain barrier with ease) and finally its low degree of ionization at
physiological pH. Only the unionized fraction of thiopentone crosses the blood–brain
barrier; it has a pKa of 7.6, meaning that 61% is unionized at pH 7.4. The relatively brief
duration of anaesthesia following a bolus of thiopentone is due to redistribution to muscle
and fat; this has no impact on the onset of action. Likewise, hepatic metabolism will affect
the duration of action and the plasma concentration rather than the speed of onset.

Question 21: FTTFT


Bioavailability can be defined as the proportion of a dose of a specified drug preparation
entering the systemic circulation after administration by a specified route and may be
calculated by the area under a plasma concentration–time curve. By virtue of this definition,
if a drug is given intravenously, 100% of it will reach the systemic circulation. The
sublingual, nasal and buccal routes all have rapid onset and, by avoiding the portal tract,

Downloaded from https://www.cambridge.org/core. University of Edinburgh, on 19 Aug 2019 at 13:21:27, subject to the Cambridge Core terms of
use, available at https://www.cambridge.org/core/terms. https://doi.org/10.1017/9781108566100.008
Chapter 3b: Pharmacology Answers 217

have higher bioavailability than the enteral route. The presence of congenital or acquired
malabsorption syndromes, such as coeliac disease, will affect absorption. Drugs absorbed
from the gut pass via the portal tract to the liver, where they may be subjected to first pass
metabolism, thus reducing the amount reaching the systemic circulation. Therefore, if a
drug undergoes minimal first pass metabolism, more drug reaches the circulation and
bioavailability is higher. First pass metabolism may be increased or decreased through the
induction or inhibition of hepatic enzymes.

Question 22: TTTTT


When considering drug transfer from mother to fetus across the placental barrier, the usual
factors affecting drug transfer across lipid layers still apply. Low molecular weight, lipo-
philic, uncharged drugs are transferred with greater ease than larger, charged molecules.
Anaesthetists should consider placental transfer whenever giving anaesthetic or non-anaes-
thetic drugs to pregnant women. This may be in the immediate antenatal/labour/operative
delivery setting, or when looking after women in earlier stages of pregnancy who may
require anaesthetic intervention. All volatile agents and induction agents cross the placenta,
and caesarean section under general anaesthesia is associated with poorer neonatal condi-
tion. Often, it is difficult to determine whether poor Apgar scores are a consequence of fetal
exposure to anaesthetic agents, or due to whatever process necessitated emergency delivery.
However, it is desirable to avoid fetal exposure to sedating anaesthetic agents. Morphine
crosses the placenta and should be avoided antenatally. Babies born to women with opioid
dependence may display features of neonatal abstinence syndrome and require specialist
care for the management of opioid withdrawal. Diclofenac should be avoided if possible in
pregnancy, and especially in the third trimester, where it is associated with closure of the
fetal ductus arteriosus or neonatal pulmonary hypertension. NICE recommend that all
women undergoing caesarean section are offered prophylactic antibiotics prior to knife-to-
skin. Although the British National Formulary states that co-amoxiclav is not known to be
harmful in pregnancy, the 2011 NICE guidelines for caesarean section suggest it is avoided
due to an association with neonatal necrotizing enterocolitis.

Question 23: TFFFF


Propofol, like other IV induction agents, is predominantly metabolized via conjugation by
the cytochrome P450 system in the liver. However, the rate of clearance of propofol exceeds
the rate of hepatic blood flow, suggesting that propofol undergoes some degree of extra-
hepatic metabolism. Various enzymes from the cytochrome system have been identified in
other tissues, including the central nervous system.
Cisatracurium, like its parent compound atracurium, does undergo hepatic metabolism,
however the majority of the drug undergoes Hofmann elimination, a pH- and temperature-
dependent degradation of the drug.
®
Esmolol is marketed as Breviblock , and has a very short duration of action. This is due to
rapid metabolism via ester hydrolysis by cholinesterase enzymes found in red blood cells.
Mivacurium, like suxamethonium, is metabolized by butyrylcholinesterase (also known as
plasma cholinesterase and pseudocholinesterase, to differentiate it from acetylcholinester-
ase). Therefore, patients with a history of any form of suxamethonium apnoea should not be
given mivacurium. Several ACE inhibitors are prodrugs and metabolized to the active

Downloaded from https://www.cambridge.org/core. University of Edinburgh, on 19 Aug 2019 at 13:21:27, subject to the Cambridge Core terms of
use, available at https://www.cambridge.org/core/terms. https://doi.org/10.1017/9781108566100.008
218 Chapter 3b: Pharmacology Answers

compound in vivo. For example, ramipril is converted to ramprilat by liver estaerases.


Lisinopril, however, is not a prodrug, does not undergo hepatic metabolism, and is excreted
unchanged in the urine. Mechanisms of metabolism are important to know, particularly for
patients with significant organ dysfunction. Drugs that display organ-independent meta-
bolism may be used with a greater safety margin in patients with failing livers or kidneys.
Bear in mind, however, that many drugs are hepatically metabolized and renally excreted;
those drugs that are metabolized by extrahepatic systems may still have pharmacologically
active metabolites which can accumulate in renal disease.

Question 24: TFTFT


Many drugs are required to cross a variety of cell membranes during their distribution from
site of administration to site of action. This process may be passive or active. In general,
small, unionized, lipid-soluble drugs cross membranes via passive diffusion, which requires
no energy, and occurs as a consequence of concentration gradients on either side of the
membrane. Conversely, large, polar drugs are less able to cross lipid bilayers, and require
specific transport proteins and energy in the form of ATP to facilitate transport. The
majority of drugs of relevance to anaesthesia, including most intravenous induction agents
and opioids, are transported by passive diffusion.
Larger drugs may require a transmembrane carrier protein to take them into cells in a
non-energy-dependent manner. This process, known as facilitated diffusion, is also depen-
dent on concentration gradients and is subject to saturation and inhibition. Examples:
glucose (via the GLUT4 receptor), cephalexin, azoles. Active transport involves the move-
ment of compounds, via transmembrane channels, against their concentration gradients.
This is an energy-dependent process, driven by ATP. Examples include the movement of
sodium and potassium via Na+/K+ ATPase, and the chemotherapeutic agent 5-fluorouracil.
The predominant mechanism by which penicillin is cleared from the plasma is active
transport into the nephron, a process that can be inhibited by probenecid. Very large or
toxic molecules can enter cells via an invagination of the cell membrane. The molecule may
also be bound to a transport protein. An example is iron. This process is called pinocytosis
(for liquids) or phagocytosis (for solids).

Question 25: TTFFF


There are several factors to consider when administering drugs to patients receiving renal
replacement therapy. Firstly, patients with renal failure and a reduced glomerular filtration
rate will accumulate drugs that are normally metabolized by the kidney. Active metabolites
that are normally renally excreted will also accumulate in these patients. Consequently,
doses of drugs and/or intervals between doses should be altered. The situation becomes
more complex when renal replacement therapy is commenced, as some drugs will be cleared
from the plasma by this process, particularly water-soluble substances. Specialist guidance
should be sought in such patients. Another important topic is the removal of toxins, or toxic
levels of drugs, via haemofiltration. Aspirin is a lipid-soluble drug that is usually rapidly
absorbed from the small intestine, metabolized by the liver and excreted by the kidneys.
When ingested in excess, it can cause severe metabolic, acid–base and neurological dis-
turbances. Overdose may be managed by administration of activated charcoal (shortly after
ingestion), administration of sodium bicarbonate (to alkalinize the urine and enhance

Downloaded from https://www.cambridge.org/core. University of Edinburgh, on 19 Aug 2019 at 13:21:27, subject to the Cambridge Core terms of
use, available at https://www.cambridge.org/core/terms. https://doi.org/10.1017/9781108566100.008
Chapter 3b: Pharmacology Answers 219

excretion) or via haemodialysis (in severe cases). Although aspirin is lipid soluble in the
acidic environment of the stomach, it becomes more ionized and therefore more water
soluble in the less acidic environment of the plasma. Water solubility favours removal via
haemofiltration. Although charcoal haemoperfusion is a more efficient system for removing
aspirin from plasma, haemofiltration is preferred in clinical practice. The increasing water
solubility in plasma permits some removal of the drug, but the system also permits
manipulation of volume and acid–base status. Low molecular weight heparins (LMWH),
e.g. enoxaparin are often used in patients receiving continuous renal replacement therapy
on the intensive care unit. This may be for thromboprophylaxis, as well as reducing the risk
of clots forming within the filter. LMWHs are not removed by haemofiltration and have the
potential to accumulate with an increased risk of bleeding. Dose reduction should be
considered and the effect may be monitored by measurement of anti-Xa levels. Atenolol is
water soluble and renally excreted. The dose should be reduced in patients with renal
impairment. Atenolol is cleared by haemofiltration. Therefore, patients on atenolol should
receive their dose after a dialysis session. Massive β-blocker overdose is rare. It may be
managed with glucagon, but haemodialysis can be used to remove renally cleared β-blocker
overdoses that are refractory to pharmacological therapy. Factors that impair drugs being
cleared by haemofiltration are: large size, large volume of distribution (e.g. digoxin) and
high protein binding (e.g. warfarin).

Question 26: FFTTT


Most drugs are eliminated from the body by the kidneys. Consequently, impaired renal
function leads to decreased clearance of most drugs. This is not always the case, however, as
some drugs are metabolized and eliminated by other organ systems, or display entirely
organ-dependent metabolism. For most drugs, elimination usually follows first-order
kinetics; the rate of elimination is directly proportional to the serum drug concentration.
However, the proportion of drug eliminated per unit time remains constant, e.g. there is
1000 mg of drug X in the plasma, after 10 minutes, 500 mg remain, after 20 minutes, 250 mg
remain. Although the absolute amount of drug X eliminated in the second 10 minutes is half
that of the first 10 minutes, there is a consistently a 50% reduction in the amount of drug X
eliminated. This consistent proportion is known as the elimination rate constant (Kel).
Clearance (Cl) of a drug refers to the rate at which the drug is removed from the plasma. It is
a theoretical value, defined as the volume of plasma from which the drug is entirely removed
per unit time and expressed in ml.min–1. Therefore, as clearance of a drug increases, the
half-life will fall. Most drugs are cleared only from the plasma. Therefore, those drugs with a
small volume of distribution (VD), with the majority of drug remaining in the plasma, will
be cleared at a faster rate than those with a higher VD. Those with a higher Kel will also be
cleared at a faster rate: Cl = Kel × VD
The half-life (t½) of a drug refers to the time taken for the amount of drug in the plasma to
fall by 50%. The time constant (tau or τ) is 1/Kel and represents the time taken for the
amount of drug in the plasma to fall to 36.7% of its starting value. Therefore, the time
constant is longer than the half life, and τ = t1/2 /0.693.

Downloaded from https://www.cambridge.org/core. University of Edinburgh, on 19 Aug 2019 at 13:21:27, subject to the Cambridge Core terms of
use, available at https://www.cambridge.org/core/terms. https://doi.org/10.1017/9781108566100.008
220 Chapter 3b: Pharmacology Answers

Question 27: TFFTT


Multicompartmental pharmacokinetic models attempt to represent more physiological
drug distribution and elimination. Mammillary models depict a well-perfused central
compartment (for instance plasma) to which a drug is introduced and eliminated.
Terminal elimination can only take place from this compartment. Peripheral compartments
are connected to the central compartment. They are not anatomically distinct, but represent
less-vascular structures in the body. Distribution of a drug from central to peripheral
compartments will vary depending on the degree of perfusion. A three-compartment
model represents a central compartment linked to two peripheral compartments of inter-
mediate and poor perfusion. Therefore rates of equilibration will vary. Catenary models, in
contrast, depict compartments linked adjacently rather than peripherally and centrally. The
sum of y intercepts of a tri-exponential decay curve (representing three-compartment
model drug elimination) is equal to the concentration 0 value.

Question 28: FTFFT


Context sensitivity is defined as the time taken for the plasma concentration of a drug to fall
by 50% subsequent to the cessation of an infusion. This is after steady state plasma
concentration has been achieved. Remifentanil is unique among infusion drugs by display-
ing context insensitivity. Regardless of the duration of infusion, when a remifentanil
infusion is terminated it will be eliminated within 3–5 minutes. This is because it is
metabolized by non-specific plasma esterases that are in abundant supply. Context-sensitive
half-life has no bearing on predicting waking time. The decrement time to waking may be
greater than 50% when waking occurs. Interestingly, context insensitivity is displayed with
alfentanil infusions of greater than 2 hours. Fentanyl displays a long context-sensitive half-
life, rendering it unsuitable for use in infusions.

Question 29: TTTTF


Total intravenous anaesthesia (TIVA) is a viable alternative to the use of volatile agents. It is
certainly indicated in patients with a history of malignant hyperpyrexia, where volatile
agents are contraindicated. In the interests of avoiding awareness under anaesthesia, a
dedicated cannula should be used and regularly inspected for patency. Plasma concentra-
tions of agent cannot be measured during TIVA, unlike end tidal volatile agent. Therefore
complex algorithms are required to calculate desired plasma or effect-site concentration of
agent. They require parameters such as age, weight, height and gender. Special infusion
pumps programmed with three-compartment pharmacokinetic models control infusion
rates to maintain steady state concentrations associated with anaesthesia. The Schnider
pharmacokinetic model, unlike the Marsh model, calculates the lean body mass of patients.
This in turn restricts the estimated volume of the central compartment, preventing the
overdosing of patients. It may therefore be more suitable for use in elderly patients. It is
important to use a long-acting opiate towards the end of TIVA for satisfactory postoperative
analgesia. Indeed, stopping remifentanil after prolonged periods of infusion in the absence
of adjunctive analgesia may result in rebound hyperalgesia.

Downloaded from https://www.cambridge.org/core. University of Edinburgh, on 19 Aug 2019 at 13:21:27, subject to the Cambridge Core terms of
use, available at https://www.cambridge.org/core/terms. https://doi.org/10.1017/9781108566100.008
Chapter 3b: Pharmacology Answers 221

Question 30: TFFFT


Plasma cholinesterase activity may be reduced by genetic variants or acquired conditions,
leading to prolonged neuromuscular block. Four alleles – usual (normal), atypical (dibu-
caine-resistant), silent (absent) and fluoride-resistant – have been identified at a single locus
of chromosome 3 and make up the 10 genotypes.
Most (96%) of the population is homozygous for the normal Eu gene and rapidly
metabolize suxamethonium. Up to 4% may be heterozygotes, resulting in a mildly pro-
longed block of up to 10 minutes. A very small percentage (Ea:Ea, Es:Ea and Es:Es) have
little or no cholinesterase activity, meaning blocks could last for many hours, requiring a
period of sedation and ventilation until it wears off. Alternatively, administration of fresh
frozen plasma provides a source of plasma cholinesterase that reverses the block.
Dibucaine is an amide local anaesthetic that inhibits normal plasma cholinesterase, but
inhibits variant forms less effectively. At a standard concentration (10−5 mol.l–1), dibucaine
inhibits the Eu:Eu form by 80% and the Ea:Ea form by only 20%. The dibucaine number
refers to the percentage inhibition of plasma cholinesterase and indicates the genotype of an
individual, but gives no information regarding the quantity of enzyme in the plasma.

Question 31: FTFTT


Malignant hyperpyrexia (MH) is a rare (1:200 000) autosomal dominant condition asso-
ciated with a defect in the ryanodine (RYR1) receptor on chromosome 19. Trigger agents
(which are the volatile anaesthetic agents and suxamethonium) cause excessive calcium
release from the sarcoplasmic reticulum, which activates muscle contraction. Etomidate is
not associated with this reaction. Although ephedrine causes catecholamine release and an
increase in intracellular calcium causing vasoconstriction, it is not related to the calcium
release involved in MH.
The UK MH investigation unit is in Leeds, and a diagnosis is based on the response of
biopsied muscle to 2% halothane and caffeine. Patients are labelled as ‘susceptible’ (positive
to both), ‘equivocal’ (positive to either halothane or caffeine) or ‘non-susceptible’ (negative
to both).
Acetylation is a phase 2 metabolic pathway in the liver. Drugs metabolized by this route
include hydralazine and isoniazid. There are variants in isoenzymes that are genetically
determined that acetylate at either a fast or a slow rate. The acetylator status of the
individual causes variation in the pharmacokinetic and pharmacodynamic profiles of
certain drugs. There does appear to be racial variation, with 50% of Europeans/North
Americans being ‘slow’ acetylators – this means a deficiency in N-acetyltransferase, which
is likely to result in accumulation of drugs that undergo acetylation. Of the oriental
population, 90% are ‘fast’ acetylators, meaning that drugs requiring acetylation will be
rapidly metabolized and are unlikely to have an effect.
Other examples of relevant pharmacogenetics include the metabolism of codeine. 5–15%
of codeine undergoes O-demethylation to morphine, which is dependent on the CYP2D6
isoenzyme. This exhibits genetic polymorphism, meaning that poor metabolizers get little
pain relief (9% of the UK population and 30% in the Hong Kong Chinese).

Downloaded from https://www.cambridge.org/core. University of Edinburgh, on 19 Aug 2019 at 13:21:27, subject to the Cambridge Core terms of
use, available at https://www.cambridge.org/core/terms. https://doi.org/10.1017/9781108566100.008
222 Chapter 3b: Pharmacology Answers

Question 32: FTTFT


Thiopentone is a thiobarbiturate and is the sulfur analogue of pentobarbitone, unlike
methohexitone, which is a methylated oxybarbiturate. It is approximately 80% bound to
plasma proteins, with high lipid solubility and is completely metabolized within the liver.
Pentobarbitone is only 40% protein bound and is excreted unchanged in the urine.
Thiopentone is formulated as a sodium salt and is stored under nitrogen to prevent
formation of the insoluble undissociated acid. Sodium thiopentone is a weak acid and forms
an alkaline solution when dissolved in water, with a pH of 10.5. The 2.5% solution is stable
for many days and the high pH, in theory, should ensure that it is bacteriostatic.
Barbiturates increase the duration of GABA-dependent Cl– channels opening in the
central nervous system. Increased Cl– conductance leads to hyperpolarization and neuronal
inhibition. Thiopentone appears to potentiate only the β-subunit of the GABAA receptor
and also affects central Na+ and K+ channels.
It may be used in the treatment of status epilepticus and causes significant reduction in
cerebral oxygen requirement. At high plasma concentrations, an isoelectric EEG is to be
expected.

Question 33: FFFTT


Propofol (2,6 diisopropylphenol) is highly lipid soluble and is presented as a 1% or 2% lipid–
water emulsion containing soya bean oil and purified egg phosphatide. Due to its lipid
content, propofol has a calorie load of 1 cal.ml–1. This should be borne in mind when
administered as a prolonged infusion, as fat overload syndrome, with hyperlipidaemia and
fatty infiltration of the liver, heart, kidney and lungs, has occurred. This also explains the
caution required when considering paediatric patients.
Metabolism is mostly hepatic, with approximately 40% undergoing conjugation to a
glucuronide and 60% metabolized to a quinol; all metabolites are inactive and excreted in
the urine.
Propofol causes a fall in systemic vascular resistance and a subsequent fall in blood
pressure. A reflex tachycardia is rare and propofol is usually associated with a brady-
cardia, particularly with co-administration of fentanyl or alfentanil. Sympathetic activ-
ity is reduced.
The terminal elimination half-life is thought to be between 5 and 12 hours, however this
may change when infused for long periods, when the context-sensitive half-life increases.
This may reflect the slow release of propofol from fat.
Propofol may turn urine and hair green as a result of the phenol content.

Question 34: TFFTT


Ketamine is a phencyclidine derivative that forms an acidic solution (pH 3.5–5.5) in water.
Although generally given intravenously, it has been used via the oral and rectal routes for
sedation and also intrathecally or via the epidural route for prolonged analgesia. However,
its use here has been limited by undesirable side effects.
Ketamine antagonizes glutamate at the NMDA receptors within the central nervous
system and, in contrast with most other anaesthetic agents, it has no activity at the GABA
receptors. Interaction with opioid receptors is thought to be complex, but ketamine seems to

Downloaded from https://www.cambridge.org/core. University of Edinburgh, on 19 Aug 2019 at 13:21:27, subject to the Cambridge Core terms of
use, available at https://www.cambridge.org/core/terms. https://doi.org/10.1017/9781108566100.008
Chapter 3b: Pharmacology Answers 223

be antanalgesic, i.e. antagonistic at the OP3 (mu) receptors, while displaying agonist activity
at OP1 and OP2 receptors.
It produces a state of dissociated anaesthesia with intense analgesia and amnesia. Vivid
dreams, delirium and hallucinations may follow its use. Cerebral blood flow, oxygen
consumption and intracranial pressure (ICP) are all increased and this should be considered
when dealing with patients with head injuries or raised ICP.
Following administration, ketamine (which is only 25% protein bound) is demethylated
to the active metabolite norketamine by P450 enzymes in the liver. Norketamine (which is
30% as potent) is further metabolized to inactive glucuronides, which are then excreted in
the urine.

Question 35: FFFTT


Etomidate is a carboxylated imidazole derivative and an ester. It is prepared as a 0.2%
solution at a pH of 4.1 and traditionally contains 35% v/v propylene glycol to improve
stability and reduce pain on injection. However, a lipid emulsion of equivalent strength is
now available.
Despite some unpleasant side effects, such as excitatory movements, nausea, vomiting and
the aforementioned pain on injections, etomidate is one of the least likely agents to cause
histamine release. Hypersensitivity reactions are also much less common. Thiopentone is
most likely to cause anaphylaxis, with an incidence of approximately 1:15 000. In contrast,
etomidate is well known to be a trigger for an acute porphyric crisis.
Etomidate suppresses adrenocortical function by inhibition of the enzymes 11β-hydro-
xylase and 17α-hydroxylase, resulting in inhibition of cortisol and aldosterone synthesis. It
was associated with an increased mortality when used for sedation in critically ill patients.
Although single doses can affect adrenocortical function, this is not thought to be significant
in otherwise healthy subjects.

Question 36: FTFTT


Thiopentone is a barbiturate and these compounds are not readily soluble in water at
neutral pH. Their solubility depends on transformation from the keto to the enol form
(tautomerism). Tautomerism refers to the dynamic interchange between two forms of a
molecular structure, which is often precipitated by a change in the physical environment.
Thiopentone is a highly lipid-soluble, highly protein-bound drug that is completely
metabolized within the liver. It tends to produce anaesthesia at induction doses in one
arm–brain circulation time. Its high lipid solubility means that rapid emergence is due to
rapid initial distribution into tissues, not metabolism.
Although thiopentone causes a reduction in systemic vascular resistance and stroke
volume, and hence a decrease in cardiac output, it does not reduce urine output by this
method alone (i.e. reduced renal blood flow). The resultant central nervous system depres-
sion leads to an increase in ADH production, which in turn conserves water and reduces
urine output.
Thiopentone is one of the more likely drugs to cause a severe anaphylactic reaction; these
are seen in approximately 1:15 000 administrations. Although this is by no means common,
it is the highest incidence amongst all of the intravenous induction agents.

Downloaded from https://www.cambridge.org/core. University of Edinburgh, on 19 Aug 2019 at 13:21:27, subject to the Cambridge Core terms of
use, available at https://www.cambridge.org/core/terms. https://doi.org/10.1017/9781108566100.008
224 Chapter 3b: Pharmacology Answers

Question 37: TFFTT


Propofol is 98% protein-bound to albumin and yet has the largest volume of distribution of
all the induction agents at 4 l.kg–1. Following a bolus dose, its duration of action is short due
to the rapid decrease in plasma levels as distribution to vessel-rich tissues occurs. Although
metabolism is mostly hepatic, clearance of propofol (30–60 ml.kg–1.min–1) exceeds hepatic
blood flow, suggesting some extrahepatic metabolism. As a result of this high clearance,
plasma levels fall more rapidly than those of thiopentone (clearance 3.5 ml.kg–1min–1).
Excitatory effects have been seen with propofol in up to 10% of patients; these are not
thought to be epileptiform and do not represent true cortical seizure activity. Movements
are typically dystonic with choreiform elements and opisthotonus. In fact, propofol can be
used to control status epilepticus.
Despite the change in preparation to a lipid emulsion, patients still report pain on
injection of propofol. Lidocaine 1% is regularly added to propofol in paediatric practice
and does seem to reduce the discomfort. There are some studies that have looked at
pretreatment with lidocaine rather than an admixture and this may be superior.

Question 38: TTTTT


There are two primary pathways of benzodiazepine biotransformation, involving hepatic
microsomal oxidation (N-dealkylation or aliphatic hydroxylation) and glucuronide con-
jugation. The hydroxylated metabolites are pharmacologically active, and many of them
have long half-lives, some longer than the parent compound. Metabolites of diazepam
include nordiazepam, temazepam and oxazepam. Subsequent conjugation of the micro-
somal metabolites by glucuronyl transferases results in glucuronides that are excreted in the
urine.

Question 39: FTTFF


Benzodiazepines have a high oral bioavailability and high protein binding (70–95%).
Diazepam and subsequent metabolites have half-lives of between 40 and over 100 hours.
This may be prolonged up to 200 hours in those who have a genetically slower metabolism.
The volume of distribution is large, ranging from around 0.8 to 2.5 l.kg–1 across the class.
Midazolam has a relatively high clearance of around 6–10 ml.kg–1.min–1 compared to
diazepam, which has a clearance of around 0.8–1.8 ml.kg–1min–1 and lorazepam, with a
clearance of 0.2–0.5 ml.kg–1.min–1.

Question 40: FFTTT


Urinary excretion for all benzodiazepines ranges from around 20% to 80%. They are not
readily removed by dialysis and therefore this option is not considered effective in isolated
benzodiazepine overdose.
Chlordiazepoxide does have active metabolites, including oxazepam, desmethyl chlor-
diazepoxide and desmethyl diazepam. Lorazepam, oxazepam and temazepam do not have
active metabolites and therefore may be considered as a better choice for elderly patients.
The elimination half-life of diazepam is long and considered to be greater than 24 hours.

Downloaded from https://www.cambridge.org/core. University of Edinburgh, on 19 Aug 2019 at 13:21:27, subject to the Cambridge Core terms of
use, available at https://www.cambridge.org/core/terms. https://doi.org/10.1017/9781108566100.008
Chapter 3b: Pharmacology Answers 225

Question 41: FFFTF


Isoflurane is a halogenated ether with a molecular weight of 184.5, boiling point of 49 °C and
saturated vapour pressure of 33 kPa at 20 °C. It has a blood:gas partition coefficient of 1.4
and an oil:gas partition coefficient of 97. Halothane and enflurane cause a greater degree of
myocardial depression than isoflurane, though all three cause hypotension. Dose-depen-
dent uterine relaxation may exacerbate uterine atony, thus worsening obstetric haemor-
rhage. Isoflurane is less than 0.2% metabolized.

Question 42: FFFTT


Desflurane has a blood:gas partition coefficient of 0.45. It is less soluble in blood than
nitrous oxide, which has a blood:gas partition coefficient of 0.47. One could therefore
assume that desflurane would reach FA/Fi equilibration more rapidly. However, nitrous
oxide is typically administered in far higher inspired concentrations of around 50–60%.
This has a concentrating effect on alveolar nitrous oxide. When nitrous oxide is absorbed
into the blood, alveolar volume diminishes thus concentrating it and any other inhalational
agents in the gas mixture. These phenomena are referred to as the concentration and second
gas effects respectively. Nitrous oxide is produced by the heating of ammonium nitrate to
240 °C. It has no effect on uterine muscle tone. With prolonged use it inhibits the enzyme
methionine synthetase, leading to a megaloblastic anaemia. Nitrous oxide occupies and
expands air-filled spaces. This could lead to large increases in the size of pneumothoraces.
Its use is therefore contraindicated in these patients.

Question 43: FTTTT


MAC is defined as the concentration of inhalational agent preventing movement of 50% of
subjects in response to a standard surgical stimulus. This is at normal barometric pressure,
with the patient inspiring oxygen and no other inhalational agent or analgesics. It is
increased in hyperthyroidism and decreased in pregnancy. The oil:gas partition coefficient
is analogous to potency. Therefore the higher the oil:gas partition coefficient, the lower the
fractional concentration of agent (and hence MAC) required to induce anaesthesia. Nitrous
oxide is far less potent than halothane. Its MAC is consequently far higher, 105% versus
0.76%. It is defined as a percentage of 1 atmosphere.

Question 44: TFFTF


Sevoflurane is an inhalational anaesthetic agent with useful properties. These include its use in
gaseous induction of anaesthesia owing to its lack of respiratory irritation. It has rapid onset
and offset therefore making it an ideal agent in day case surgery and difficult airway situations.
It is a clear colourless liquid with a molecular weight of 200, a boiling point of 58 °C and a
saturated vapour pressure of 21 kPa at 20 °C. It owes its rapid onset and offset to a blood:gas
partition coefficient of 0.69. The oil:gas partition coefficient of 53 correlates with moderate
potency. It is 3–5% metabolized. The toxicity of compound A has only been demonstrated in
rats at high temperatures under conditions of prolonged low-flow anaesthesia. The toxicity of
compound A has not been demonstrated in humans, even with low-flow anaesthesia.

Downloaded from https://www.cambridge.org/core. University of Edinburgh, on 19 Aug 2019 at 13:21:27, subject to the Cambridge Core terms of
use, available at https://www.cambridge.org/core/terms. https://doi.org/10.1017/9781108566100.008
226 Chapter 3b: Pharmacology Answers

Question 45: FFTFF


The potency of inhalational anaesthetic agents is directly related to their lipid solubility and
thus the oil:gas partition coefficient (which is a reflection of lipid solubility). Minimal
alveolar concentration is inversely proportional to the oil:gas partition coefficient (and
thus potency). Halothane has a oil:gas partition coefficient of 225 and is more potent than
sevoflurane, which has an oil:gas partition coefficient of 53. The oil:gas partition coefficient
of methoxyflurane is 970 versus 98 for enflurane. It is important to understand that
anaesthetic potency has no relationship with speed of onset and offset. This is governed
by the blood:gas partition coefficient, which is a measure of solubility. The lower the blood:
gas coefficient the more rapid the onset and offset.

Question 46: TTFTF


Halothane is a potent volatile anaesthetic (oil:gas partition coefficient = 225). It has a
molecular weight of 197, a boiling point of 50 °C with a saturated vapour pressure of
32 kPa at 20 °C. It is non-flammable, but is slightly unstable in light and is therefore
formulated with thymol as a preservative. It was a popular inhalational induction agent
due to its smooth and rapid action, along with its non-irritant respiratory properties. Its use
in the UK has been discontinued for a number of reasons. It is the most extensively
hepatically metabolized volatile agent at 20%. Associated with this is an increased hepatitis
risk, particularly with repeated use. Furthermore, nodal rhythms and bradycardias can
result. It also sensitizes the myocardium to catecholamines.

Question 47: FFTTF


The mechanism of action of general anaesthetics is still not fully understood. The Meyer–
Overton rule was proposed at the turn of the twentieth century. This stated that the
hydrophobicity of an agent is directly related to its potency as an anaesthetic. Thus
hydrophobic anaesthetics are more potent and may act by dissolving in the lipid bilayer
of cells. However, one of the several flaws in the rule is that not all hydrophobic agents have
anaesthetic properties, the long-chain hydrocarbons for example. GABAA and glycine
receptors (a ligand-gated ion channel) are indeed inhibitory receptors and the potentiation
of these by certain anaesthetic agents is another proposed mechanism. Excitatory receptors
(nicotinic acetylcholine, 5HT3, and NMDA) are inhibited by anaesthetics. Thus to induce
anaesthesia there has to be enhancement of inhibition, inhibition of excitation or a combi-
nation of the two.

Question 48: TTTFT


It is the relationship between pKa and tissue pH that will relate to the ionization of the drug.
The pKa is the pH at which the drug exists as a 50:50 mixture of ionized and non-ionized
molecules. The lower the pKa of a drug in physiological solution, the more drug is available
in the neutral form. It is this neutral form that is able to cross the cell membrane to block the
sodium channels, thus preventing conduction. The pKa of lidocaine is 7.9 and for bupiva-
caine it is 8.1.
Lipid solubility of local anaesthetics is a significant determining factor of drug potency.
The more lipid soluble an agent, the greater the potency.

Downloaded from https://www.cambridge.org/core. University of Edinburgh, on 19 Aug 2019 at 13:21:27, subject to the Cambridge Core terms of
use, available at https://www.cambridge.org/core/terms. https://doi.org/10.1017/9781108566100.008
Chapter 3b: Pharmacology Answers 227

There is avid binding of local anaesthetics to circulating plasma proteins, which will
effectively inactivate the drug. The affinity of the agent for protein molecules has been
correlated with the duration of anaesthetic effect, not speed of onset. The protein binding of
lidocaine is 65% and bupivacaine 96%.
The type and location of block will both impact on the speed of onset.

Question 49: FTTFT


In theory, the maximum dose of lidocaine can be increased from 3 mg.kg–1 to 7 mg.kg–1 by
the addition of a vasoconstrictor such as epinephrine. It should be noted, however, that the
maximum safe dosage in the individual can vary significantly depending on clinical status,
site of injection and concomitant use of other agents.

Question 50: FFFTF


Ester local anaesthetics undergo rapid metabolism by pseudocholinesterase enzymes. The
rapid hydrolysis in plasma results in water-soluble metabolites, which are excreted in urine.
No metabolism occurs in cerebrospinal fluid and therefore intrathecally placed ester
anaesthetics must be absorbed into the vascular system prior to metabolism. The extraction
ratio of lidocaine is high and therefore is reliant on sufficient hepatic blood flow. Amide
local anaesthetics are primarily metabolized by microsomal P450 enzymes in the liver.
Bupivacaine, as an amide, is metabolized in the liver. It is conjugated to glucuronic acid with
only 5% excreted unchanged in the urine. Hepatic failure generally has to be severe before
metabolism is significantly affected.

Question 51: FTFFF


Typically, the addition of 80 mg.ml–1 of glucose increases the baricity of bupivacaine
in order to control spread within cerebrospinal fluid. The preservative methyl
para-hydroxybenzoate is often added to local anaesthetic and in very rare instances may
provoke an allergic reaction. The addition of bicarbonate to local anaesthetic should
increase the amount of the unionized form by increasing local pH. As a result this should
increase the speed of onset of anaesthesia.
Lidocaine 2.5% and prilocaine 2.5% can be mixed to form a eutectic mixture of local
anaesthetics (EMLA). In the form of a cream it can be used to provide superficial anaesthesia
for skin interventions. Bupivacaine is prepared as the hydrochloride salt in order to dissolve
the drug in water, forming an acidic solution.

Question 52: TFTFF


The sodium channel is composed of a single polypeptide chain with four repeating units.
The S4 region has multiple charged amino acids that give rise to the voltage-dependent
nature of the channel. Sodium channels in any of the conformations (open, closed, resting
or inactivated) can bind local anaesthetic, but in the resting state, the affinity is very low. The
local anaesthetic–sodium channel interaction is a reversible one.
It is necessary for the local anaesthetic to cross the cell membrane in order for it to
interact with the sodium channel. This results in cessation of the depolarization process by
preventing sodium ions entering the cell. In sufficient concentration, local anaesthetics will

Downloaded from https://www.cambridge.org/core. University of Edinburgh, on 19 Aug 2019 at 13:21:27, subject to the Cambridge Core terms of
use, available at https://www.cambridge.org/core/terms. https://doi.org/10.1017/9781108566100.008
228 Chapter 3b: Pharmacology Answers

interact with other receptors, including potassium channels, calcium channels and nicotinic
acetylcholine receptors.

Question 53: TFFTF


Intralipid has been shown to be effective in the treatment of overdoses of lipophillic drugs,
including tricyclic antidepressants and β-blockers. The initial loading dose is 1.5 ml.kg–1.
The infusion dose thereafter is 15 ml.kg–1.h–1. Propofol, although presented in lipid emul-
sion, is not an acceptable substitute in the treatment of local anaesthetic toxicity because of
its cardio-depressant effects. Intralipid can be used in periarrest situations, but if used in
cardiac arrest, CPR may have to be prolonged.

Question 54: TFTFF


Intralipid constituents are as follows: 20% soya bean oil, 1.2% egg yolk phosphatides, 2.25%
glycerine and water. The soya bean and phosphatide are rich sources of essential fatty acids.
Aluminium rather than selenium can accumulate with prolonged use, particularly where
renal function is impaired. This can result in neurotoxicity and bone marrow toxicity.
Essential fatty acids in intralipid are metabolized hepatically. Central lobular cholestasis due
to fat overload can result in hepatomegaly, along with deranged liver function. The use of
Intralipid is contraindicated in cases of pancreatitis with hyperlipidaemia. This is due to the
greater risk of fat overload. Thrombocytopenia is a recognized complication of prolonged use.

Question 55: TFFFT


There are many regimes in clinical use for patient-controlled epidural analgesia (PCEA) in
labour. Factors that appear to provide improved analgesia and maternal satisfaction are the
use of high-volume boluses of low-dose local anaesthesia, the addition of a low-rate back-
ground infusion and the addition of fentanyl to the local anaesthetic solution. Technology
will likely allow improvement in performance, with automated mandatory boluses and
feedback loops currently being evaluated.
There is a suggestion that PCEA is associated with a lower total volume of anaesthetic
given, compared with continuous infusion alone, which may explain the reductions in
motor block observed in several studies.
Women requesting epidural analgesia should be warned that there is a risk of delaying the
second stage of labour and an increased likelihood of requiring instrumental delivery. There
is no evidence that epidurals increase the likelihood of caesarean delivery.
Consent should be gained prior to siting an epidural. The Obstetric Anaesthetists
Association publishes an information leaflet for mothers, ‘Pain Relief in Labour’, which
lists the risks of epidural analgesia and anaesthesia. This includes maternal fever, which may
be associated with fetal distress.

Question 56: FFFTF


Paracetamol in the oral form is absorbed readily from the gastrointestinal tract with peak
plasma levels between 1 and 3 hours. It is variably absorbed following rectal administration.
When administered in normal doses there is minimal plasma protein binding of around 10–
20%. The VD is 0.5 to 1.3 l.kg–1. Paracetamol is primarily conjugated by hepatic transferases

Downloaded from https://www.cambridge.org/core. University of Edinburgh, on 19 Aug 2019 at 13:21:27, subject to the Cambridge Core terms of
use, available at https://www.cambridge.org/core/terms. https://doi.org/10.1017/9781108566100.008
Chapter 3b: Pharmacology Answers 229

largely with glucuronic acid and to a lesser degree sulfuric acid. The intravenous preparation
achieves peak plasma levels twice those of the oral preparation by 15 minutes, however by
one hour levels are similar. Around 3% of the therapeutic dose is oxidized by the cyto-
chrome P450 isoforms CYP2A6, 1A2, 3A4 and 2E1. The resultant metabolites include the
highly reactive N-acetyl-p-benzoquinoneimine.

Question 57: TTTTF


Paracetamol is responsible for around 50% of cases of acute liver failure in the UK. The
frequency of severe paracetamol hepatotoxicity has fallen since sales were restricted
in 1998. Alcohol abuse can predispose to paracetamol hepatotoxicity. It causes induc-
tion of CYP2E1, leading to a greater production of the toxic metabolite N-acetyl-p-
benzoquinoneimine. A confirmed overdose should be treated early with an agent to
replace the depleted glutathione stores. This can be in the form of oral methionine and
oral or intravenous N-acetylcysteine. Even if it is given up to 72 hours after overdose
there is some evidence that N-acetylcysteine reduces mortality. Consistent with the low
molecular weight, N-acetylcysteine readily crosses the placenta, but there are minimal
effects on the fetus. Therefore it should be administered early in order to reduce the
risk to both mother and fetus.

Question 58: TTFFF


Although aspirin is a weak acid with a pKa of 3 (meaning that it is present in the unionized
form in the stomach), the alkaline mucosal cells mean than salicylate ions may become
trapped and therefore prevented from reaching the systemic circulation. Due to its large
total surface area, the small bowel absorbs more of the drug.
It has significant metabolic effects, particularly in overdose. Aspirin uncouples oxidative
phosphorylation, thus increasing oxygen consumption and carbon dioxide production.
Hence, although an aspirin overdose may cause pyrexia, the rise in carbon dioxide results
in hyperventilation, with coma only seen in very rare cases. The excretion of aspirin is
enhanced by forced alkaline diuresis and the drug may be removed by haemofiltration/
haemodialysis.
At low dose, aspirin selectively inhibits platelet COX, while preserving vessel wall
COX. This means that while aspirin inhibits TXA2-induced vasoconstriction and
platelet aggregation, it leaves vessel wall synthesis of prostaglandins unchanged and
therefore, dilated.

Question 59: FTFTT


NSAIDs inhibit the cyclo-oxygenase (COX) enzymes 1 and 2, thereby preventing the
production of both prostaglandins and thromboxanes from membrane phospholipids. By
inhibiting COX, more arachidonic acid is converted to leukotrienes, which are known to
cause bronchospasm.
Adverse features of NSAIDs include impairment of renal function from a reduction in
renal blood flow and glomerular filtration rate. They may therefore promote sodium and
fluid retention and hyperkalaemia.

Downloaded from https://www.cambridge.org/core. University of Edinburgh, on 19 Aug 2019 at 13:21:27, subject to the Cambridge Core terms of
use, available at https://www.cambridge.org/core/terms. https://doi.org/10.1017/9781108566100.008
230 Chapter 3b: Pharmacology Answers

Platelet numbers are not decreased, although platelet function may be impaired. The
reduced production of cyclic endoperoxidases and thromboxane A2 prevents platelet
aggregation and vasoconstriction and, therefore, inhibits the haemostatic process.
NSAIDs may inhibit osteoblast function and therefore this can lead to delayed bone
healing.
These drugs are widely used to treat mild to moderate pain, and to reduce opioid
consumption in the perioperative period. They are more effective in the treatment of
somatic pain.

Question 60: FFTFT


Meloxicam has limited preferential selectivity for COX-2 and has a reduced gastrointestinal
side-effect profile when compared to other NSAIDs. Its renal side effects appear to be
equivalent.
The recommended dosing of ibuprofen in children is up to a maximum of 20 mg.kg–1. It
has the lowest incidence of side effects of the most commonly used NSAIDs.
Indomethacin is a potent anti-inflammatory agent with less analgesic effect. It is used to
promote closure of a patent ductus arteriosus in the neonate by inhibiting prostaglandin
synthesis.
Acute severe asthma may be precipitated in up to 4–20% of asthmatics when given
NSAIDs. These individuals may have a history of chronic rhinitis or nasal polyps.
Therefore, the majority of people with asthma will be able to tolerate NSAIDs.
Celecoxib has a sulfonamide structure and therefore should not be used in patients with a
sulfonamide allergy or porphyria.

Question 61: FFTTF


Unlike non-specific NSAIDs, drugs in this group do not affect platelet function (even at high
doses) as this process is thought to be mediated by inhibition of the COX-1 isoenzyme. In
terms of analgesic efficacy, celecoxib and rofecoxib have demonstrated analgesic efficacy
equivalent to non-specific NSAIDs.
Although in theory the idea of eliminating the deleterious effects associated with COX-1
inhibition seems highly beneficial, the reality is that only the incidence of gastrointestinal
side effects appears to be reduced (CLASS trial and VIGOR study). Cardiac side effects may
be increased; by decreasing vasodilatory and antiaggregatory prostaglandin production,
COX-2 inhibitors may lead to increased prothrombotic activity. The available data from
VIGOR and CLASS raise a cautionary flag about the risk of increased cardiovascular events
in those patients on COX-2 inhibitors.
Recent recommendations suggest that these drugs should only be used in patients who
are at increased risk from COX-1 inhibitors and who are not on aspirin, as concurrent
administration of COX-2 inhibitors and aspirin increases their side-effect profile.

Question 62: TFFTT


Tramadol has structural similarities to codeine and venlafaxine. It is dependent on cyto-
chrome CYP2D6 for metabolism and therefore effects may be variable. It is a weak μ-opioid
agonist and blocks both noradrenaline and serotonin uptake.

Downloaded from https://www.cambridge.org/core. University of Edinburgh, on 19 Aug 2019 at 13:21:27, subject to the Cambridge Core terms of
use, available at https://www.cambridge.org/core/terms. https://doi.org/10.1017/9781108566100.008
Chapter 3b: Pharmacology Answers 231

Tramadol and the main metabolite M1 (О-desmethyl-tramadol) also block NMDA


receptors. M1 has greater affinity than the parent drug for the μ-opioid receptor.
Tramadol causes less respiratory depression than the other opioids. The action on serotonin
uptake means that use in conjunction with selective serotonin reuptake inhibitors can result
in serotonin syndrome.

Question 63: FFTTF


Codeine is well absorbed following oral administration with a bioavailability somewhat
similar to that of oral morphine (average 40–50%). Glucuronidation to codeine-6-glucur-
onide is the major metabolic pathway, constituting around 80% of metabolites.
Morphine or morphine metabolites constitute approximately 4–6%. The metabolism to
morphine is dependent on О-demethylation and is subject to variation resulting from
genetic polymorphisms in the cytochrome P450 enzyme system (CYP2D6). Reduction in
gut motility by both morphine and codeine is by action on μ-receptors.

Question 64: TFTTT


Naturally occurring narcotic alkaloids found in the poppy are known as opiates. Examples
include morphine and codeine. Synthetic or semi-synthetic compounds with morphine-like
pharmacological actions are grouped under the heading of opioid analgesic drugs.
Morphine is widely used in anaesthesia, critical care and pain medicine for the relief of
acute and chronic pain. It may also be used in the management of myocardial ischaemia,
pulmonary oedema and palliative care.
It is often administered by oral, intramuscular or intravenous routes. It is metabolized to
morphine-3-glucuronide and morphine-6-glucuronide. Morphine-6-glucuronide is more
potent than the parent molecule and will accumulate in patients with renal impairment.
There are several unpleasant side effects of morphine that should be considered. As
with other opioids, morphine can cause nausea, vomiting, constipation, confusion and
respiratory depression. The reduction in minute ventilation with morphine is predomi-
nantly due to a reduction in respiratory rate. Tidal volume reduction also occurs, as well
as impairment of the usual increases in ventilation associated with hypercarbia and
hypoxia.
Morphine has a terminal half-life of approximately 3 hours. Treatment of opioid
overdose commonly involves administration of naloxone, which is an antagonist at all
opioid receptor types. However, the pharmacodynamic effects of naloxone are shorter
than those of morphine; not only does it have a reduced terminal half-life compared to
morphine (60–90 min), it is also removed from the central nervous system much more
rapidly via redistribution.
Intrathecal morphine has been shown to provide effective and sustained pain relief
following many types of surgery. It may be used as a sole intrathecal agent or in conjunction
with local anaesthetic agents. The principal concern with intrathecal morphine is respira-
tory depression, particularly with doses greater than 0.5 to 1.0 mg. A preservative-free
solution must be used.

Downloaded from https://www.cambridge.org/core. University of Edinburgh, on 19 Aug 2019 at 13:21:27, subject to the Cambridge Core terms of
use, available at https://www.cambridge.org/core/terms. https://doi.org/10.1017/9781108566100.008
232 Chapter 3b: Pharmacology Answers

Question 65: TFFFT


Remifentanil is an ultra-short-acting opioid which is a pure μ-agonist. It is given intrave-
nously by infusion, with or without a preceding bolus. It may be used for sedation (for
painful procedures, or in critical care) or as part of a general anaesthetic in conjunction with
propofol (TIVA) or volatile agents. The opioid effects of remifentanil permit a reduction in
the amount of anaesthetic agent required, although there is no absolute proportion by which
anaesthesia may safely be reduced. Remifentanil is not an anaesthetic drug and care must be
taken to avoid awareness when reducing volatile or propofol delivery rates.
There is a wide range of response to remifentanil, with wide variability between patients in
terms of required plasma concentrations. A reasonable starting rate would be 0.2 μg.kg–1.min–1,
although requirements may be anything from 0.1 to 2.0 μg.kg–1.min–1. There are also reports of
rapid development of tachyphylaxis with remifentanil (declining effect with same dose),
although the mechanism of this is unclear.
One of the useful features of remifentanil is the almost complete lack of context-sensitive
half-life. Unlike thiopentone or fentanyl, the offset of remifentanil is almost entirely
independent of the duration of infusion (the ‘context’, in this case). The drug is rapidly
metabolized by non-specific plasma and tissue esterases to an inactive metabolite, regardless
of the duration of infusion, and independent of hepatic and renal function. If used as part of
a balanced general anaesthetic, the rapid offset of action means that pre-emptive analgesia
will be required before stopping the infusion.
In common with other potent piperidine opioids (fentanyl, alfentanil, sufentanil), remi-
fentanil may cause chest wall muscle rigidity, with a consequent risk of respiratory
insufficiency.

Question 66: FTTTF


Diamorphine is the common name for diacetylmorphine, which consists of two acetyl
groups joined to a morphine molecule via ester bonds. Diamorphine is a prodrug, and is
rapidly deacetylated to 3-acetyl-morphine, 6-acetyl-morphine and finally morphine. The
half life of the drug is 2 to 3 minutes. The presence of the acetyl groups increases the lipid
solubility of the drug. Therefore, when given intravenously, diamorphine is able to enter the
central nervous system and exert its effects more rapidly than morphine. The potency of
diamorphine appears to be 2–3 times that of morphine.
Diamorphine is frequently associated with pruritus, particularly when used intrathecally,
although the mechanism for this is not well understood. There is likely to be an element of
histamine release, and chlorphenamine is an effective treatment in some patients. However,
others gain more benefit from subcutaneous naloxone, suggesting that the pruritus may be a
direct μ-receptor effect.
The equipotent intrathecal dose of diamorphine is approximately 10% of that of the
epidural dose (a concentration gradient is not required to enable the drug to cross the dura).
Intravenous and epidural doses should be roughly equivalent.
Diamorphine has a marked inhibitory effect on the smooth muscle of the gastrointestinal
tract, and is associated with nausea and vomiting as a consequence of delayed gastric transit
and impaired gastric emptying.

Downloaded from https://www.cambridge.org/core. University of Edinburgh, on 19 Aug 2019 at 13:21:27, subject to the Cambridge Core terms of
use, available at https://www.cambridge.org/core/terms. https://doi.org/10.1017/9781108566100.008
Chapter 3b: Pharmacology Answers 233

Question 67: TFTFT


Fentanyl and alfentanil are both commonly used, rapid-acting opioids, often given at the
induction of anaesthesia. They have contrasting pharmacological properties.
Fentanyl is very lipid soluble. This allows the drug to readily cross the blood–brain barrier
and it is an effective analgesic in very small doses of 1–2 μg.kg–1. The high fat solubility also
means fentanyl has a large volume of distribution (Vd) of 4 l.kg–1. As well as CNS uptake, the
drug is rapidly and extensively taken up by fat, muscle and other tissues. This redistribution
into other tissues is responsible for the offset of the drug, and means that the terminal half-
life is prolonged; much of the total quantity of the drug in vivo is not within the plasma at
any one time and therefore unable to be cleared.
Alfentanil is known to have a more rapid onset than fentanyl, suggesting it must enter the
CNS faster following IV bolus injection. In fact, alfentanil is poorly lipid soluble. However,
this ensures that it has a much smaller Vd of 0.5 l.kg–1 when compared to fentanyl, and will
have much higher initial plasma concentrations when given in equipotent doses. Combined
with a low degree of ionization of approximately 10%, this enables alfentanil to enter the
CNS very rapidly.
Plasma clearance of alfentanil is much slower than that of fentanyl, but the high lipid
solubility and Vd of fentanyl means that its terminal half-life is in fact longer than that of
alfentanil.
The values in Table 3.67.1 are approximate and serve to highlight the pharmacological
differences between the drugs, rather than to be a reference.

Table 3.67.1 Approximate characteristics

Fentanyl Alfentanil

Lipid solubility High Low


–1
Volume of distribution (l.kg ) 4.0 0.5
% Ionized at pH 7.4 90 10
Dose at induction 1–2 μg.kg–1 10–50 μg.kg–1
Clearance (ml.min–1) 1500 250
Terminal t½ (min) 200 100

Question 68: TTTFT


PCA is widely used for postoperative pain management. In addition to the pharmacological
advantages, PCA may offer psychological benefit by empowering patients to self-manage
their pain.
When used intravenously, an initial bolus dose is usually given to establish an effective
plasma level. Intermittent demand boluses (with or without a background infusion) main-
tain therapeutic steady state levels with greater reliability than other routes.
Programmable bolus doses and lock-out times are agent specific to prevent the risk of
overdose. In adult practice, it is common to have standard concentrations of opioid solution
e.g. morphine 1 mg.ml–1, and alter bolus doses, lock out times and background infusion

Downloaded from https://www.cambridge.org/core. University of Edinburgh, on 19 Aug 2019 at 13:21:27, subject to the Cambridge Core terms of
use, available at https://www.cambridge.org/core/terms. https://doi.org/10.1017/9781108566100.008
234 Chapter 3b: Pharmacology Answers

rates to provide effective analgesia within safe limits. Conversely, in paediatric practice,
bolus doses, lock out times and infusion rates are more commonly fixed, and the concen-
tration of opioid adjusted according to the weight of the child.
Pumps usually have built-in alarms and locks. They commonly have a high pressure
alarm to detect an obstruction in the giving set or cannula, or an extravasation. A low-
pressure alarm is designed to detect disconnection.
The absence of one-way valves on a Y connector would allow dangerously unpredictable
boluses of analgesic to be administered when another infusion is running concurrently with
a PCA. Respiratory rate is a sensitive sign of opiate overdose and is important to monitor. It
is acceptable to run PCA and epidurals simultaneously, provided the epidural has no opiate
in it.

Question 69: FFFTF


While there is variance in potency between opiates, they are all similarly efficacious.
Tramadol can produce similar levels of analgesia to more potent opioids, but dysphoria,
nausea and vomiting complicate its use.
There is a lack of evidence to suggest that running a background infusion of opiate with
PCA on top provides superior pain control. As expected, the complications of respiratory
depression and hypoxaemia are more common with this practice.
There is evidence that concurrent use of ketamine in PCA for patients with severe
postoperative pain is effective and opiate sparing particularly in opiate-tolerant patients.
The routine addition of antiemetics to PCA is still controversial. The rationale behind
PCA use is patient demand. Therefore with every demand potentially unnecessary anti-
emetic is being delivered.

Question 70: FTTFT


Cisatatracurium is a non-depolarizing neuromuscular blocker in the benzylisoquinoli-
nium class. It is derived from a single cis-isomer of the parent molecule atracurium, of
which it comprises approximately 15%. It was manufactured in an attempt to produce a
‘cleaner’ drug without associated histamine release, and there are no such side effects
with cisatracurium. The majority (up to 80%) of the drug is eliminated via Hofmann
degradation, which is temperature and pH dependent (occurring more rapidly at higher
temperatures and higher pH). Like atracurium, laudanosine is a recognized metabolite.
This tertiary amine is a known CNS stimulant, but no association with epilieptiform
activity has been demonstrated. Cisatracurium is more potent than atracurium – a
common intubating dose would be 0.15 mg.kg–1. However, onset of muscle relaxation is
slower than atracurium, in equipotent doses (e.g. 2 × ED95). Duration of action is
similar for both drugs.

Question 71: FTFFF


Rocuronium (Rapid Onset curonium) is a non-depolarizing neuromuscular blocker in the
aminosteroid class. It undergoes almost exclusive hepatobiliary excretion. The ED95 is
approximately 0.3 mg.kg–1. Multiples of this dose are used clinically. Most commonly, a
dose of 0.6 mg.kg–1 will give intubating conditions in 90–120 s, and provide intermediate-

Downloaded from https://www.cambridge.org/core. University of Edinburgh, on 19 Aug 2019 at 13:21:27, subject to the Cambridge Core terms of
use, available at https://www.cambridge.org/core/terms. https://doi.org/10.1017/9781108566100.008
Chapter 3b: Pharmacology Answers 235

duration muscle relaxation; 0.3 mg.kg–1 can be used for short-term (10–15 min) muscle
relaxation, but has a longer onset of action. Rapid onset (45–60 s) can be achieved with 0.9–
1.2 mg.kg–1, and this dose may be used as part of a ‘modified’ RSI, where there is a reason to
avoid suxamethonium.
Like the long-acting aminosteroid pancuronium, rocuronium is a mild vagolytic, so may
increase heart rate and blood pressure, although this effect is less pronounced than with
pancuronium.
Rocuronium has been implicated in several cases of anaphylaxis, and, with suxametho-
nium, is responsible for the majority of cases of anaphylaxis to muscle relaxants. Several
studies have estimated the incidence of anaphylaxis to all muscle relaxants to be between
1:6500 and 1:20 000. Bronchospasm is a recognized feature of anaphylaxis, but this does not
mean rocuronium is contraindicated in asthmatics. Indeed, it may be preferred to atracur-
ium, which is more associated with histamine release.
Although refrigeration of rocuronium will prolong the shelf-life of the drug, it remains
active for up to one month when stored at room temperature.

Question 72: FFTFF


Atracurium may be encountered in the management of ARDS as a means to reduce patients
‘fighting the ventilator’ to promote more efficient mechanical ventilation or lessen the
requirement for heavy sedation. However, it is not recommended for routine use in
ARDS and has not been shown to reduce mortality. There is some evidence that it may be
of benefit in severe ARDS, with very low PaO2:FiO2 ratios, but it is also associated with
increased risks of complications such as pneumothorax and critical care neuropathy.
Atracurium is a benzylisoquinolinium and therefore unaffected by sugammadex.
Atracurium is well recognized to cause transient histamine release. However, this is
usually short-lived and of little clinical significance. Some advocate avoiding atracurium
in asthmatic patients due to fears of inducing bronchospasm. Rapacuronium (RAPid Acting
curonium) was an aminosteroid muscle relaxant withdrawn from the market in 2001 due to
risk of fatal bronchospasm.
Atracurium consists of 10 stereoisomers and is approximately 80% protein-bound in the
plasma.

Question 73: FFTFT


Suxamethonium resembles acetylcholine (ACh) and acts as an agonist at nicotinic
acetylcholine receptors at the neuromuscular junction (NMJ). However, its action at
the receptor is prolonged compared to ACh, leading to prolonged depolarization of the
muscle – seen clinically as fasciculations.
One reason for its prolonged action is its delayed metabolism when compared to ACh,
which occurs via plasma butylcholinesterase (pseudocholinesterase), rather than via
acetylcholinesterase in the NMJ.
As may be deduced from its structure, the action of suxamethonium is not limited to
nicotinic receptors. Muscarinic actions include inducing bradycardia via action at the SA
node.
In the presence of large, repeated or infusion doses, the initial (phase 1) block may
proceed to a phase 2 block. The exact mechanism for this is unclear. However, it is

Downloaded from https://www.cambridge.org/core. University of Edinburgh, on 19 Aug 2019 at 13:21:27, subject to the Cambridge Core terms of
use, available at https://www.cambridge.org/core/terms. https://doi.org/10.1017/9781108566100.008
236 Chapter 3b: Pharmacology Answers

characterized by an inability for ACh to cause a new depolarization, even when the
resting membrane potential has been restored. This resistance to ACh then presents like
an NDNMB, i.e. fade on train-of-four stimulation and an ability to overcome the block
with anticholinesterases.
Administration of suxamethonium is associated with a transient rise in plasma potassium
levels due to widespread passage of potassium out of cells, down its concentration gradient,
into the ECF, via open ACh receptor channels. In certain conditions, there is increased
expression of extrajunctional ACh receptors, resulting in an exaggerated hyperkalaemic
effect. This phenomenon is seen following burns and denervation injuries.
In Duchenne muscular dystrophy, increased potassium release may also occur following
suxamethonium administration due to excess muscle damage and rhabdomyolysis. There
have been cases of suxamethonium-induced cardiac arrest in these patients.

Question 74: TTFFF


Suxamethonium is formulated by combining two ACh molecules via their acetyl groups.
Among its many side effects, it is a known trigger for MH and one of the most common
culprits for anaphylaxis in anaesthesia. Several studies have estimated the incidence of
anaphylaxis to all muscle relaxants to be between 1:6500 and 1:20 000.
More commonly, it is a cause of postoperative muscle pain, most likely as a consequence
of widespread fasciculation. Counterintuitively, this does not appear to be directly related to
muscle mass. The patient group most affected has been reported to be young, early
ambulating females.
The action of suxamethonium in patients with myasthenia gravis (MG) is complex. Since
it acts in a similar manner to acetylcholine at poorly functioning ACh receptors, it would be
expected that these patients are resistant to suxamethonium, as they are to acetylcholine.
Several texts describe a doubling of the ED95 in patients with MG, or at least a decrease in
effect seen in subtherapeutic doses when compared to patients without MG.

Question 75: FFTTT


Mivacurium is a short-acting benzylisoquinolinium non-depolarizing neuromuscular
blocker. A dose of 0.2 mg.kg–1 would be expected to provide intubating conditions in
2 minutes (in the presence of volatile anaesthesia, which potentiates its effect) and have a
clinical duration of action of 20 minutes; 0.1 mg.kg–1 would be expected to last for
13 minutes.
In common with suxamethonium, it is metabolized by plasma cholinesterase (pseudo-
cholinesterase), which means it should be avoided in patients with known mutations of the
plasma cholinesterase gene, and may be expected to have a prolonged duration of action in
situations where the activity of this enzyme is reduced (e.g. pregnancy). Its inactive
metabolites are primarily excreted in the urine.

Question 76: FFTFT


Sugammadex is a modified cyclodextrin that may be used to reverse the effects of rocur-
onium and vecuronium by encapsulating circulating drug in the plasma and preventing it
from binding at the neuromuscular junction. Although the drug has less affinity for

Downloaded from https://www.cambridge.org/core. University of Edinburgh, on 19 Aug 2019 at 13:21:27, subject to the Cambridge Core terms of
use, available at https://www.cambridge.org/core/terms. https://doi.org/10.1017/9781108566100.008
Chapter 3b: Pharmacology Answers 237

vecuronium than rocuronium, the higher potency of vecuronium results in fewer molecules
of the drug circulating in the plasma, enabling sugammadex to still exert a clinically useful
effect.
Sugammadex is given according to patient weight, and based on absolute weight, not
ideal or lean body mass. Routine reversal from a moderate block (reappearance of T2 on
TOF testing) may be achieved within 2 minutes with a dose of 2 mg.kg–1. In this respect, its
use could be considered analogous to neostigmine, although the haemodynamic effects seen
with the anticholinesterase would be avoided.
Sugammadex has a more novel use in the reversal of deep blocks. Complete reversal of a
deep neuromuscular block (post-tetanic count of >2) may be achieved within 3 minutes at a
dose of 4 mg.kg–1. It may also be used in emergency scenarios such as failed intubation,
where a dose of 16 mg.kg–1 sugammadex can reverse an ‘immediate’ neuromuscular block
(i.e. 3 minutes after administration of 1.2 mg.kg–1 rocuronium) within 1.5 minutes.
No dose reduction is required in patients with mild to moderate renal impairment, but
there are not enough safety data to recommend the use of sugammadex in patients with
severe renal impairment.

Question 77: FTFTF


Neostigmine is a medium-acting inhibitor (1–2 hours) of acetylcholinesterase, and is classed
as an anticholinesterase and parasympathomimetic drug. It acts by binding to the esteratic
binding site of the enzyme, where it is slowly metabolized, and prevents binding and
breakdown of acetylcholine during this period. Consequently, it allows increased amounts
of acetylcholine to act at both nicotinic and muscarinic receptors.
Clinically, it may be used to reverse the effects of neuromuscular blockade and may be
given orally in the treatment of myasthenia gravis (MG). Myasthenia is usually diagnosed
with the much shorter-acting (10 seconds) anticholinesterase edrophonium (Tensilon),
which will produce a transient improvement in muscle power in patients with MG.
Neostigmine’s action on muscarinic receptors can result in bradycardia, bronchospasm
and smooth muscle contractility. It is therefore relatively contraindicated in asthmatics,
pyrexial patients (where sweating may be inhibited) and may induce postoperative nausea,
vomiting and diarrhoea. This effect can be exploited in the treatment of pseudo-obstruction
of the bowel secondary to paralytic ileus.
Neostigmine is commonly given in combination with glycopyrrolate to ameliorate the
bradycardic response, which in fact can promote a relative tachycardia. Neostigmine does
not cross the blood–brain barrier and in overdose can precipitate a cholinergic crisis.

Question 78: TFFTT


Atropine is a core emergency anaesthetic drug, useful in the emergency treatment of
bradycardia. It acts as a competitive antagonist at all five subtypes of muscarinic receptor
(M1–M5), reducing the effects of acetylcholine in the parasympathetic nervous system. As
such, it may variously be described as an anticholinergic, a parasympatholytic, or a vagolytic
drug, depending on the clinical situation.
It is used in the emergency treatment of bradycardia. The Resuscitation Council UK
recommend an initial dose of 0.5 mg in their bradycardia treatment algorithm. They
previously recommended 3 mg for the treatment of cardiac arrest from non-shockable

Downloaded from https://www.cambridge.org/core. University of Edinburgh, on 19 Aug 2019 at 13:21:27, subject to the Cambridge Core terms of
use, available at https://www.cambridge.org/core/terms. https://doi.org/10.1017/9781108566100.008
238 Chapter 3b: Pharmacology Answers

rhythms, but since 2010 this guidance has been removed due to lack of evidence of
benefit.
Paradoxically, atropine may also cause bradycardia, particularly in low doses. The
reasons for this are unclear, but may relate to blockade of presynaptic muscarinic receptors
(blocking negative feedback of ACh release), blockade of potential sympathetic muscarinic
receptors or actions in the central nervous system.
Cholinergic crises arise when there is an excess of acetylcholine flooding and saturating
acetylcholine receptors. This may arise from relative overdosing of anticholinesterase, for
example in the treatment of myasthenia gravis or reversal of neuromuscular blockade, or in
organophosphate poisoning. Anticholinergic drugs such as atropine are useful for such
crises, as they block the acetylcholine receptors and mitigate some of the effects of the excess
neurotransmitter.

Question 79: TTFTF


Glycopyrrolate (glycopyrroium) is a charged quaternary ammonium compound that is fully
ionized at physiological pH. It therefore is unable to cross lipid membranes, and will not
readily cross the placenta or blood–brain barrier. It has minimal effects on the central
nervous system, unlike atropine, which is associated with amnesia, confusion and excitation
in elderly patients.
It has less pronounced haemodynamic effects than atropine, but is a more potent
antisialogogue, hence its use as a premedication for drying of secretions prior to fibreoptic
intubation. It is also used as a means to avoid unwanted bradycardia (paediatric intubation,
following neostigmine etc.).
The usual arrangement of the sympathetic nervous system is that preganglionic fibres
release acetylcholine, which acts on nicotinic receptors, and that postganglionic receptors
release catecholamines. An exception to this are the sweat glands, which are activated by
acetylcholine acting on muscarinic receptors. Hence, glycopyrrolate may usefully be
employed in the treatment of hyperhydrosis.

Question 80: TFTTF


Hyoscine (as the hydrobromide) is a lipid-soluble drug, which is able to enter the central
nervous system, where it has sedating and antiemetic properties. A ‘scopolamine patch’ is a
recognized treatment for PONV, and is usually sited preoperatively. Hyoscine is also sold
over the counter as a preventative treatment for motion sickness.
As the butylbromide, hyoscine is not lipid soluble, but is a useful antispasmodic in the
treatment of cramping abdominal pain from smooth muscle spasm (Buscopan ). ®
Of the anticholinergics, hyoscine is particularly associated with mydriasis, and is used
topically by ophthalmologists prior to diagnostic procedures.
Omnopon is a trade name for paperveretum, formerly popular in the UK as a premedica-
tion prior to anaesthesia. It is actually a synthetic combination of morphine, papaverine and
codeine. Although hyoscine may be considered to have some useful premedicant properties
(drying of secretions, sedation, amnesia), it is not one of the ingredients of Omnopon.

Downloaded from https://www.cambridge.org/core. University of Edinburgh, on 19 Aug 2019 at 13:21:27, subject to the Cambridge Core terms of
use, available at https://www.cambridge.org/core/terms. https://doi.org/10.1017/9781108566100.008
Chapter 3b: Pharmacology Answers 239

Question 81: FTTTF


Phosphodiesterase inhibitors are a heterogeneous group of drugs that are either selective or
non-selective in their action on the five phosphodiesterase (PDE) isoenzymes. In particular,
they exert a positive inotropic effect by increasing intracellular cAMP levels and possibly
cGMP levels (PDE enzymes hydrolyze cAMP). By effectively increasing cAMP levels within
the myocardium, they increase the slow Ca2+ inward current during the cardiac action
potential. This produces an increase in Ca2+ release from intracellular stores, and hence
results in a positive inotropic effect.
Enoximone and milrinone are selective inhibitors of the PDE III isoenzyme found
specifically in the myocardium, smooth muscle and platelets. This in theory improves
cardiac output by increasing stroke volume, but without significant tachycardia or increase
in myocardial oxygen demand. They also reduce systemic vascular resistance by vasodilata-
tion; this is also thought to be due to interference with Ca2+ flux into vascular smooth
muscle.
PDE inhibitors do not have any action on the Na+/K+ pump.

Question 82: FTTFT


Dopexamine is a synthetic dopamine analogue and is an agonist at dopaminergic D1- and
D2-receptors, leading to relaxation of vascular smooth muscle in the renal, mesenteric,
cerebral and coronary arterial beds (D1-effects) and stimulation of sympathetic prejunc-
tional receptors, thereby decreasing noradrenaline release (D2-effect). The drug also
inhibits uptake of noradrenaline and has potent β2-adrenergic activity. This will result
in bronchodilation. It has no α-adrenergic activity.
Splanchnic blood flow may increase due to mesenteric vasodilatation. Dopexamine
reduces renal vascular resistance, leading to an increase in renal plasma flow and resultant
diuresis and natriuresis.

Question 83: TTTFT


Thyroxine (T4) and triiodothyronine (T3) have positive inotropic and chronotropic effects
via intracellular mechanisms. These effects may be mediated by an increase in the number of
myocardial β-adrenergic receptors.
Digoxin is a cardiac glycoside and acts both directly and indirectly. Its direct action is
exerted by binding to and inhibiting the action of Na+/K+ ATPase within the sarcolemma.
The resultant increase in intracellular sodium ion concentration causes displacement of
bound intracellular calcium ions. The positive inotropic action is a consequence of the
increased availability of the calcium ions. Digoxin increases the force of cardiac contraction,
automaticity and contractility.
Aminophylline is a non-selective phosphodiesterase (PDE) inhibitor. By inhibiting PDE
enzymes, intracellular levels of cAMP and possibly cGMP are increased. This rise in cAMP
mobilizes intracellular calcium stores, which results in a mild positive inotropic and
chronotropic effect.
Methoxamine is a selective α1-adrenergic agonist. It has similar effects to phenylephrine
and is no longer in production. An intravenous bolus produces a rapid rise in systemic

Downloaded from https://www.cambridge.org/core. University of Edinburgh, on 19 Aug 2019 at 13:21:27, subject to the Cambridge Core terms of
use, available at https://www.cambridge.org/core/terms. https://doi.org/10.1017/9781108566100.008
240 Chapter 3b: Pharmacology Answers

vascular resistance and, therefore, blood pressure. However, it has no inotropic effects as it
has no action at β-adrenergic receptors.
The activation of glucagon receptors, via G-protein mediated mechanisms, stimulates
adenylate cyclase and increases intracellular cAMP, hence exerting a positive inotropic
effect.

Question 84: FTFFT


Levosimendan is a calcium sensitizer – it increases the sensitivity of the heart to calcium,
thus increasing cardiac contractility without a rise in intracellular calcium. It exerts its
positive inotropic effect by binding to cardiac troponin C in a calcium-dependent manner.
It also has a vasodilatory effect, by opening ATP-sensitive potassium channels in vascular
smooth muscle to cause smooth muscle relaxation. The combined inodilator actions result
in an increased force of contraction, decreased preload and decreased afterload.
It is indicated for inotropic support in acutely decompensated congestive cardiac failure
and has been subject to a high number of Phase III clinical trials, one of which (SURVIVE)
did a head-to-head comparison between levosimendan and dobutamine. As yet, it does not
appear to have benefit in those patients with chronic heart failure.
The use of levosimendan is contraindicated in patients with: moderate to severe renal
impairment, severe hepatic impairment, severe ventricular filling or outflow tract obstruc-
tion, severe hypotension and tachycardia and/or history of torsades de pointes.

Question 85: TFTTF


Adrenaline may increase minimum alveolar concentration and increase pain threshold via
its effects at the α2-adrenergic receptors. These receptors are widespread throughout the
nervous system and when stimulated, cause sedation, analgesia and attenuation of sym-
pathetically mediated responses.
Dopamine attenuates the response of the carotid body to hypoxaemia. Pulmonary
vascular resistance is increased.
Isoprenaline is a highly potent synthetic catecholamine with actions at β1 and β2-
adrenoceptors. These β-effects lead to raised blood glucose and free fatty acids.
Dobutamine is a direct-acting synthetic catecholamine derivative of isoprenaline.
β1 effects predominate; therefore its main actions are to increase contractility, heart rate
and myocardial oxygen requirement. Blood pressure is usually increased. These properties
mean that it is a suitable pharmacological substitute in those who are unable to undergo
exercise testing, as it reproduces the physiological response to physical exercise.
Inotropes are simply agents that affect myocardial contractility and may do so either
positively or negatively. So, although drugs that increase the force of contraction are
commonly referred to as ‘inotropes’, drugs such as β-blockers are also inotropes; they are
negatively inotropic as they decrease myocardial contractility. This may seem to be a minor
point, but the correct use of terminology is very important when sitting the FRCA exams.

Question 86: FFFFF


Metaraminol has direct and indirect effects on the sympathetic system. It is used as a
treatment for hypotension during general or central neuraxial anaesthesia.

Downloaded from https://www.cambridge.org/core. University of Edinburgh, on 19 Aug 2019 at 13:21:27, subject to the Cambridge Core terms of
use, available at https://www.cambridge.org/core/terms. https://doi.org/10.1017/9781108566100.008
Chapter 3b: Pharmacology Answers 241

There are actions on both α- and β-adrenergic receptors, although the α-effect predomi-
nates. It has a positive inotropic effect. There is a sustained increase in systolic and diastolic
blood pressure with an associated reflex bradycardia. The increase in systemic vascular
resistance often causes a fall in cardiac output. It is commonly administered as either a bolus
or as a titrated intravenous infusion.
Although when given intramuscularly it does carry the risk of local ischaemia, this is still a
well-recognized route. It inhibits insulin release and stimulates glycogenolysis resulting in
the potential for elevated plasma glucose levels.

Question 87: FFTFF


Vasopressin is produced in the paraventricular nucleus of the hypothalamus and trans-
ported by neuronal axons to the posterior pituitary. V1a, V1b and V2 are G-protein-coupled
receptors that are widely distributed. V1a occur predominantly on vascular smooth muscle.
There is no vasoconstrictor effect on the pulmonary vasculature and indeed it may actually
cause vasodilation.
Vasopressin has no action on adrenergic receptors and no direct cardiac effects. It is
thought that the role in maintaining vascular tone in health is relatively minor, though there
is increased release and action in shock states such as haemorrhage. There is a greater
distribution of vasopressin receptors on the renal efferent arterioles compared to the
afferent arterioles. The result of this differential distribution can lead to an increased
glomerular filtration rate and increased urine output.

Question 88: TFFFT


Phenylephrine is almost identical to epinephrine, though it lacks a hydroxyl group at
position 4 on the benzene ring. It is a pure α1-adrenergic agonist at clinical concentrations,
though it may have some minor β-action at extremely high plasma levels. It is metabolized
by MAO but not COMT.
When administered intravenously, it has a duration of action of around 6–8 minutes.
There is no tachyphylaxis with repeated dosing. Phenylephrine was once felt to be contra-
indicated in obstetric patients due to concerns regarding decreases in uterine blood flow.
However, studies indicate fetal pH may be higher when using a phenylephrine infusion
compared to ephedrine alone.

Question 89: FFFTF


Hydralazine is an agent used in the treatment of both acute and chronic hypertension. It has
a role in the management of pre-eclampsia. Its mode of action is not entirely clear, but
causes a greater degree of arterial dilatation than venodilatation.
Sodium nitroprusside (SNP) is a rapidly acting agent used in the acute treatment of
hypertension. Its use is complicated by the risk of cyanide toxicity. It acts by releasing nitric
oxide, which acts in the vascular endothelium to bring about vasodilatation. It causes
venous and arterial vessel dilatation. Despite glyceryl trinitrate (GTN) working via a similar
mechanism, it causes venous dilatation to a greater extent. Toxic CN– ions are released from
erythrocytes when SNP reacts with oxyhaemoglobin. They are converted to thiocyanate
(SCN) in the liver and kidney. SCN is far less toxic than CN but can accumulate in renal

Downloaded from https://www.cambridge.org/core. University of Edinburgh, on 19 Aug 2019 at 13:21:27, subject to the Cambridge Core terms of
use, available at https://www.cambridge.org/core/terms. https://doi.org/10.1017/9781108566100.008
242 Chapter 3b: Pharmacology Answers

impairment causing toxicity. CN can react with vitamin B12 to produce cyanocobalamin,
which is non-toxic. Therefore CN toxicity can occur in patients with vitamin B12 deficiency.
While B12 may have a role in prophylaxis of toxicity, it is not ideal for the acute treatment of
toxicity. Nitrates, dicobolt edetate and SCN are suitable alternative treatments.

Question 90: TFFTT


Glyceryl trinitrate (GTN) acts on the vascular endothelium via release of nitric oxide. This
in turn activates guanylate cyclase. This raises cyclic GMP concentrations, impairing
calcium influx, which promotes vasodilation. GTN use attenuates hypoxic pulmonary
vasoconstriction. This could in theory exacerbate shunt, causing further hypoxia.
GTN is extensively cleared by first pass metabolism. It is therefore presented in infusion
and sublingual form to overcome its poor oral bioavailability. In contrast, isosorbide
mononitrate has near 100% bioavailability. When ferrous haemoglobin (Fe2+) is oxidized
to ferric (Fe3+) by nitrate, it can convert oxyhaemoglobin to methaemoglobin.

Question 91: TTFTT


PVR is the resistance in the pulmonary circulation and is normally 20–120 dynes.s.cm–5
MPAP  LAP
PVR ¼
CO

where MPAP = mean pulmonary artery pressure, LAP = left atrial pressure and CO =
cardiac output.
Epoprostenol (Flolan) is a potent vasodilator. As well as inhibiting platelet aggregation, it
reduces PVR and pulmonary artery pressure.
Sodium nitroprusside is a prodrug that vasodilates arteries and veins to control hyper-
tension. It is converted to nitrites in vascular smooth muscle, which then react with
hydrogen ions to produce nitric oxide, thus reducing systemic vascular resistance and PVR.
Noradrenaline stimulates the sympathetic nervous system via α-adrenergic receptors,
increasing PVR.
Isoprenaline is a non-selective β-agonist that causes peripheral and pulmonary
vasodilation.
Inhaled anaesthetic agents have a minimal pulmonary vasodilator effect.

Question 92: FTFTT


Thiazides are commonly used for the treatment of hypertension and mild heart failure.
The mechanism of action is:
1. They act on the distal tubule by inhibiting sodium and chloride reabsorption and by
increasing water excretion.
2. The increased sodium load at the distal tubule is exchanged with hydrogen ions and
potassium ions leading to a hypokalaemic hypochloraemic alkalosis.
3. The reduction in plasma volume and systemic vascular resistance produces the
antihypertensive effect.
Thiazides have the following side effects:

Downloaded from https://www.cambridge.org/core. University of Edinburgh, on 19 Aug 2019 at 13:21:27, subject to the Cambridge Core terms of
use, available at https://www.cambridge.org/core/terms. https://doi.org/10.1017/9781108566100.008
Chapter 3b: Pharmacology Answers 243

• Hypokalaemia
• Hypercalcaemia
• Hyperuricaemia
• Hyponatraemia
• Hypomagnesaemia
• Hyperglycaemia
• Hypercholesterolaemia
• Aplastic anaemia
• Can precipitate pancreatitis

Question 93: TTFTT


ACE inhibitors inhibit the conversion of angiotensin I to angiotensin II, a known potent
vasoconstrictor (approximately five times more potent than noradrenaline). Post myocar-
dial infarction, the prognosis is improved if patients are treated with an ACE inhibitor in the
context of left ventricular dysfunction. All grades of heart failure can be treated with ACE
inhibitors.
Contraindications include pregnancy (craniofacial abnormalities, patent ductus arterio-
sus) and bilateral renal artery stenosis.

Question 94: TFTFF


Nifedipine is a class II calcium channel antagonist, a dihydropyridine derivative. It
blocks the entry of calcium through L-type channels, which is responsible for the
plateau phase of the cardiac action potential. It is used in the treatment of angina,
hypertension and Raynaud’s disease. The oral absorption is 95%, with 60% oral
bioavailability. Nifedipine is highly protein-bound and is metabolized in the liver
with no active metabolites. Nifedipine reduces mean arterial pressure with a reflex
increase in heart rate. Cardiac output is increased, as well as coronary blood flow.
Calcium antagonists may also reduce the MAC of volatile agents and potentially pro-
long the duration of neuromuscular blocking agents.

Question 95: FFFTT


Atenolol is a selective β1-antagonist. It is highly water soluble with a 50% bioavailability. It
has low protein binding and is excreted largely unchanged. Side effects include bradycardia
and possible heart block, bronchoconstriction, uterine contraction and hypoglycaemia.
Therefore, atenolol should be avoided in asthma, congestive cardiac failure and peripheral
vascular disease. Care is also needed in diabetic patients. As atenolol is only 3% protein-
bound and 95% is excreted unchanged in the urine, a reduced dosage is required in severe
renal impairment.

Downloaded from https://www.cambridge.org/core. University of Edinburgh, on 19 Aug 2019 at 13:21:27, subject to the Cambridge Core terms of
use, available at https://www.cambridge.org/core/terms. https://doi.org/10.1017/9781108566100.008
244 Chapter 3b: Pharmacology Answers

Question 96: FTTFF


Table 3.96.1 The correct Vaughan-Williams classification

Ia Sodium channel blockade Quinidine, procainamide


Ib Sodium channel blockade Lidocaine, phenytoin
Ic Sodium channel blockade Flecainide, propafenone
II β-blockade Propranolol, esmolol
III Potassium channel blockade Amiodarone, bretylium
IV Calcium channel blockade Verapamil, diltiazem

Question 97: FTFTT


Adenosine is a purine nucleoside and is used for the rapid reversion to sinus rhythm from
paroxysmal supraventricular tachycardias. It is presented as a colourless solution in vials
containing 3 mg.ml–1. The duration of action is 10–12 seconds.
Contraindications to the use of adenosine are: asthma, second/third-degree heart block,
sick sinus syndrome and long QT syndrome.
Additionally, adenosine should be used with caution in patients with cardiac transplant
(where it may cause prolonged asystole – a reduced dose is recommended) and Wolf–
Parkinson–White syndrome and associated atrial flutter/fibrillation (where it can precipi-
tate ventricular fibrillation).

Question 98: FTTTT


Almost all patients on continuous amiodarone therapy develop yellow–brown microcrys-
tals in the cornea, which may cause visual halos or blurring of vision. However, these
deposits do regress following discontinuation of therapy.
Most skin reactions are usually tingling, burning and erythema of sun-exposed skin, but
severe phototoxic reactions with blistering may be seen.
Amiodarone may cause pulmonary fibrosis and is also associated with pneumonitis,
pleuritis and pneumonia. Sometimes these conditions can be fatal. Presenting features
include dyspnea of non-cardiac origin and weight loss, fatigue and fever.
Peripheral neuropathy can be caused by amiodarone, which may be severe, although is
usually reversible on drug withdrawal.
Liver function tests need to be carried out on a regular basis and the amiodarone dose
may need to be reduced or withdrawn if necessary.

Question 99: FTFTT


The functional unit of the kidney is the nephron. These act in unison to filter blood,
reabsorb ions, and excrete the remaining fluid as urine.
Broadly, the nephron has four functional areas. The glomerulus receives and filters
arterial blood to form an ultrafiltrate. This filtrate then passes into the proximal convoluted

Downloaded from https://www.cambridge.org/core. University of Edinburgh, on 19 Aug 2019 at 13:21:27, subject to the Cambridge Core terms of
use, available at https://www.cambridge.org/core/terms. https://doi.org/10.1017/9781108566100.008
Chapter 3b: Pharmacology Answers 245

tubule, where the majority of reabsorption of water and solutes occurs. Next, the loop of
Henle uses pumps and active transport to create and maintain a hypertonic medulla. Finally,
the filtrate passes through the collecting ducts where a variable amount of water is reab-
sorbed into the hypertonic medulla. The remaining filtrate leaves the kidney and enters the
bladder as urine.
The descending limb of the loop of Henle is permeable to water, but contains no ion
channels, so is impermeable to sodium, potassium, etc. Therefore, water flows out of the
tubules and into the medulla down its concentration gradient. The ascending limb receives
filtrate with a high solute concentration. Once in the ascending limb, the walls of the tubules
become impermeable to water (due to the presence of tight gap junctions), but permeable to
ions via the action of various transport proteins.
Firstly, sodium flows into the cells of the tubular wall down its own concentration
gradient (across the luminal wall). The transmembrane protein responsible for this active
transport is the Na-K-2Cl co-transporter (NKCC), which transports one sodium, one
potassium and two chloride molecules, to maintain electrical neutrality.
This is not an energy-dependent process as ions are moving along concentration gradi-
ents. However, this process is still referred to as secondary active transport, as it relies on
another, primary active transport process to generate this concentration gradient. In the
case of the loop of Henle, this is the action of Na+/K+ ATPase on the basolateral membrane
of the tubular cells, which continues to pump sodium out of the cells into the interstitium,
and potassium into cells, against their concentration gradients.
Potassium is also believed to flow back into the tubular fluid, via passive transport, down
its concentration gradient. This potassium ‘leak’ maintains an electrical gradient between
tubular fluid and luminal wall cells, which promotes the reabsorption of other cations,
specifically calcium and magnesium.
Furosemide works by blocking the NKCC. Therefore, the secondary active transport of
sodium and potassium out of the tubular filtrate is impaired. Consequently, more sodium
and potassium are passed through the nephrons and excreted in the urine. As sodium is not
able to be reabsorbed and exert its effect on maintaining blood pressure, the drug achieves
its primary aim of reducing blood pressure. However, it does this at the risk of causing
hyponatraemia and hypokalaemia.
The impact on the electrochemical gradient (via the block of potassium transport) can
affect the reuptake of magnesium and calcium. Furosemide is also associated, therefore,
with hypomagnesaemia. Calcium, however, can be reabsorbed via active transport in the
distal convoluted tubule, between the loop of Henle and the collecting duct. Consequently,
furosemide is not particularly associated with hypocalcaemia.
Hypokalaemia is associated with hyperglycaemia so furosemide, like thiazide diuretics,
should be used with caution in diabetics.
Additionally, furosemide’s impairment of chloride reuptake can result in a chloride loss
and a hypochloraemic metabolic alkalosis.
Interestingly, the side effects of furosemide are mimicked in Bartter’s syndrome, an
autosomal recessive disorder where there is a mutation of the NKCC in the loop of Henle.
Sufferers display a metabolic alkalosis and low blood pressure.
Another side effect of furosemide is ototoxicity and deafness, although the mechanism of
action of this is poorly understood.

Downloaded from https://www.cambridge.org/core. University of Edinburgh, on 19 Aug 2019 at 13:21:27, subject to the Cambridge Core terms of
use, available at https://www.cambridge.org/core/terms. https://doi.org/10.1017/9781108566100.008
246 Chapter 3b: Pharmacology Answers

Question 100: TFFTF


Diuretics act by increasing the volume of urine excreted, thereby lowering total body water.
They are commonly used in hypertension, heart failure, liver failure and renal disorders.
They work via several mechanisms:
Loop diuretics: Inhibition of Na-K-2Cl co-transporter in the loop of Henle, e.g.
furosemide, bumetanide.
Thiazides: Inhibition of Na-Cl symporter in the distal convoluted tubule, e.g.
bendroflumethiazide, hydrochlorothiazide, metolazone.
Osmotic diuretics: Expand the plasma volume by exerting an osmotic effect. This
increases renal blood flow and thus glomerular filtration rate. The drugs are filtered
at the glomerulus, but not reabsorbed from the proximal convoluted tubule, resulting
in retention of water in the tubule, and reduction in reabsorption, e.g. mannitol,
glucose.
Carbonic anhydrase inhibitors: Usually, filtered bicarbonate combines with secreted
hydrogen ions in the renal tubule to form carbonic acid. Carbonic anhydrase then
catalyzes the breakdown of carbonic acid into CO2 and water. Inhibition of carbonic
anhydrase by acetazolamide causes a build-up of carbonic acid and a reduction in the
ability to combine bicarbonate and hydrogen ions. Therefore, there is an increase in
bicarbonate excretion. The remaining hydrogen ions are reabsorbed along with chloride
ions, at the expense of sodium reabsorption. Therefore, acetazolamide causes increased
sodium excretion and hyperchloraemic metabolic acidosis. This effect makes it useful in
the treatment of respiratory alkalosis (e.g. in mountain sickness). Its diuretic effect is
weak.
Potassium-sparing diuretics: Either competitive antagonism of aldosterone (e.g.
spironolactone, potassium canreonate), or blockade of epithelial sodium channels in the
collecting duct (e.g. amiloride, triamterene). These channels are also physiologically
blocked by atrial natriuretic peptide, which also promotes diuresis.

Question 101: FFFTF


Bendroflumethiazide is a thiazide diuretic that is only available as an oral preparation. It is
contraindicated in pregnancy due to neonatal bone marrow depression and other side
effects. It is safe to be used during breast-feeding.
NICE, in conjunction with the British Antihypertensive Society, recommend that
patients over 55 and those of African or Caribbean family origin (of any age) are treated
with a calcium channel blocker as the first-line antihypertensive. Those under 55 should be
started on an ACE inhibitor. Diuretics are usually second-line treatments for hypertension,
except when there is evidence of heart failure.
When GFR falls below 30 ml.min–1.1.73 m–2 thiazides become ineffective, with the
exception of metolazone.
Co-amilozide is a combination of amiloride and hydrochlorothiazide.

Downloaded from https://www.cambridge.org/core. University of Edinburgh, on 19 Aug 2019 at 13:21:27, subject to the Cambridge Core terms of
use, available at https://www.cambridge.org/core/terms. https://doi.org/10.1017/9781108566100.008
Chapter 3b: Pharmacology Answers 247

Question 102: TFTTF


Spironolactone is a steroidal compound that acts an antagonist at intracellular mineralo-
corticoid receptors and a glucocorticoid antagonist via inhibition of 11β-hydroxylase. It is
also antiandrogenic via direct blockade of the receptor and via inhibition of 17α-
hydroxylase.
As a competitive aldosterone antagonist, it is useful as a potassium-sparing diuretic. It is
particularly useful in treating oedema due to heart failure and liver disease. It is an
antihypertensive, and can also be used in the treatment of primary or secondary hyper-
aldosteronsim (e.g. Conn’s disease).
In clinically relevant doses, it does not suppress glucocorticoid activity.
It may also be used to treat conditions related to testosterone activity, e.g. hirsutism and
alopecia in females, and male pattern baldness in men. Its antiandrogenic activity can result
in gynaecomastia, testicular atrophy and erectile dysfunction.
It is hepatically metabolized. A minimal amount is excreted by the biliary system, with the
majority being renally excreted.

Question 103: TTTTF


Warfarin is a vitamin K antagonist and inhibits the enzymes involved in the regeneration of
the reduced form of vitamin K. The lack of vitamin K then inhibits the production of
vitamin K-dependent clotting factors II, VII, IX and X.
Warfarin is given orally with complete absorption from the gut. It is 95% protein-bound
and undergoes hepatic metabolism to form products that are excreted by the kidneys.
Protein-bound drugs such as NSAIDs will displace warfarin, while drugs that inhibit
hepatic enzymes (P450), such as erythromycin, cimetidine, alcohol, amiodarone and
metronidazole will reduce the rate of metabolism and potentiate the effect of warfarin.
Drugs such as barbiturates, carbamazepine and the oral contraceptive pill may decrease the
activity of warfarin.
Prothrombin complex concentrate (e.g. Beriplex) contains factors II, VII, XI and X and
for this reason can be used to reverse the anticoagulant effects of warfarin. It has been used
in major haemorrhage with hypovolaemic shock, acute intracerebral bleeds and patients
requiring emergency surgery.

Question 104: FFFTT


Heparins are a family of mucopolysaccharides subdivided into the unfractionated form
(UFH) or the fractionated low molecular weight heparins (LMWH).
UFH inhibits coagulation by potentiating the action of antithrombin III. This inactivates
thrombin and inhibits factors II, IX, X, XI and XII. Heparin activity is monitored by
measuring the APTT.
LMWH is more effective at inhibiting factor Xa. The activity of LMWH is not usually
monitored; however, an anti-Xa assay is available if required in high-risk patients.
LMWH has a longer half-life and greater bioavailability than UFH, allowing once daily
subcutaneous administration and more predictable anticoagulant response. It also has a
lower incidence of heparin-induced thrombocytopenia.

Downloaded from https://www.cambridge.org/core. University of Edinburgh, on 19 Aug 2019 at 13:21:27, subject to the Cambridge Core terms of
use, available at https://www.cambridge.org/core/terms. https://doi.org/10.1017/9781108566100.008
248 Chapter 3b: Pharmacology Answers

Protamine is a basic protein that combines with the strong acid heparin to neutralize its
anticoagulant effects. It is more effective for UFH and is given intravenously, the standard
dosing being 1 mg to reverse 100 units of heparin.

Question 105: FTFFT


Lepirudin is a direct thrombin inhibitor. It has a short half-life and is given intravenously. It
has a narrow therapeutic index and requires APTT monitoring. Patients who have had an
allergic reaction to heparin may be given lepirudin.
Alteplase is a fibrinolytic and is used for thrombolysis for acute stroke, massive pulmon-
ary embolism and acute myocardial infarction if primary percutaneous intervention is not
available.
Aprotinin was temporarily withdrawn internationally in 2007/2008 following a trial that
suggested it was associated with renal damage and increased risk of strokes. However, many
countries have now lifted the suspension.
Tranexamic acid is an antifibrinolytic that is used in massive haemorrhage associated
with major trauma, and massive obstetric haemorrhage. It inhibits the conversion of
plasminogen to active plasmin, and therefore helps to control bleeding due to excessive
fibrinolysis.
Protamine is a basic protein made from fish sperm that may cause cardiovascular
instability if administered too rapidly. Severe allergic reactions may also occur.

Question 106: TFTTT


With such a variety of anticoagulants coming into use in clinical practice, anaesthetists need
to be aware of current advances. The consequences of haematoma following neuraxial
blockade can be catastrophic.
The AAGBI produced guidance in 2011 ‘Regional anaesthesia in patients with abnorm-
alities in coagulation’. This should be used together with consideration of each individual’s
circumstances. It states the following acceptable intervals between drug administration and
regional block:
Therapeutic LMWH: 24 hours
Prophylactic LMWH: 12 hours
Warfarin: less than or equal to INR 1.4
Clopidogrel: 7 days
Fondaparinux: >36 hours
Low-dose aspirin: no additional precautions

Question 107: FTTTF


An appreciation of platelet physiology is necessary to understand the pharmacology of
antiplatelet agents. Platelet adhesion and aggregation is the prelude to thrombus formation
in the clotting cascade. Damaged vessel endothelium promotes platelet adhesion via von
Willebrand’s factor (itself a component of factor VIII) and fibrinogen. Platelet aggregation
is mediated via the platelet glycoprotein IIb/IIIa receptor (this receptor is itself activated by
ADP). The process of platelet aggregation is enhanced and amplified via the release of

Downloaded from https://www.cambridge.org/core. University of Edinburgh, on 19 Aug 2019 at 13:21:27, subject to the Cambridge Core terms of
use, available at https://www.cambridge.org/core/terms. https://doi.org/10.1017/9781108566100.008
Chapter 3b: Pharmacology Answers 249

platelet thromboxane A2. Platelet aggregation is limited and controlled by the release of
prostacyclin (PGI2) from adjacent undamaged vessel endothelium.
Clopidogrel is a very potent agent that irreversibly inhibits ADP binding to platelet
receptors. This therefore prevents the activation of the glycoprotein IIb/IIIa receptor. It
does have fewer gastrointestinal side effects than aspirin. Owing to its irreversible action on
platelets, it needs to be stopped seven days prior to surgery. Dipyridamole is a coronary
artery vasodilator.

Question 108: TFFTF


Dipyridamole inhibits platelet adhesion through the inhibition of adenosine uptake. It also
inhibits adenosine deaminase, further increasing extracellular concentrations of adenosine.
At low doses it potentiates prostacyclins. Furthermore, it inhibits platelet phosphodiesterase
resulting in higher platelet cAMP concentration. High cAMP levels attenuate the effect of
ADP on glycoprotein IIb/IIIa receptor activation.
Dipyridamole in combination with aspirin has been proven to be effective in the second-
ary prevention of TIA and stroke in the ESPRIT trial. It is therefore recommended for use in
combination with aspirin.

Question 109: FTFTT


Glycoprotein IIb/IIIa inhibitors are short-acting drugs given intravenously for treat-
ment of unstable angina and non-ST elevation myocardial infarction (tirofiban and
eptifibatine). They are also used to minimize thrombotic complications following
percutaneous coronary intervention. Oral forms are associated with higher mortality
and are not available.
Platelet adhesion to damaged vessel endothelium is mediated via von Willebrand’s factor.
In contrast, glycoprotein IIb/IIIa inhibitors block platelet aggregation, which is a process
downstream of platelet adhesion. They are used with unfractionated heparin. This is
because of their short half-life and also because, as a sole agent, they have no effect on
platelet adhesion or the secretion of prothrombotic factors from platelets.
Epoprostenol is prostacyclin PGI2, which limits platelet aggregation and is used to prime
extracorporeal haemofiltration circuits.
The use of dextrans is declining, but they do have antiplatelet activity that is mediated via
inhibition of von Willebrand’s factor.

Question 110: FTTTT


Tranexamic acid is a synthetic analogue of lysine. It is commonly used in the treatment of
heavy bleeding, both non-life-threatening (e.g. orally for menorrhagia) and life-threatening
(e.g. intravenously for major trauma and massive obstetric haemorrhage). It was added to
the WHO essential medicines list in 2011.
It inhibits the conversion of plasminogen to plasmin. Plasmin is required for
degradation of fibrin clots (fibrinolysis) thus tranexamic acid is an antifibrinolytic
agent. Aminocaproic acid is another amino-acid antifibrinolytic, but is less efficacious
than tranexamic acid.

Downloaded from https://www.cambridge.org/core. University of Edinburgh, on 19 Aug 2019 at 13:21:27, subject to the Cambridge Core terms of
use, available at https://www.cambridge.org/core/terms. https://doi.org/10.1017/9781108566100.008
250 Chapter 3b: Pharmacology Answers

The CRASH-2 study showed a statistically significant reduction in mortality following


major trauma when patients were treated with tranexamic acid within 8 hours of injury.
Further analysis suggested that mortality was reduced if the drug was given within 3 hours;
later than this was associated with an increase in deaths from bleeding. It is unclear whether
the beneficial effects of tranexamic acid are due to antifibrinolysis per se, or via inhibition of
the proinflammatory effects of plasmin.
Although no demonstrable increase in vaso-occlusive events was seen following tranexa-
mic acid administration in the CRASH-2 study population, thromboembolic disease still
remains a contraindication for its routine use (e.g. menorrhagia).

Question 111: FFTFT


Aprotinin (Trasylol ) is a serine protease inhibitor. It reduces fibrinolysis via the inhibition
®
of the formation of plasmin from plasminogen. In this respect, its mechanism of action is
similar to tranexamic acid and aminocaproic acid. It also inhibits kallikreins, which are
required for the activation of factor XII in the intrinsic pathway of the clotting cascade. It is
derived from bovine lung tissue.
It is given intravenously, therefore its bioavailability must be 100%. Following adminis-
tration and redistribution, it is both metabolized and excreted by the kidney. Therefore, the
clearance of the drug may be decreased, and the elimination half-life increased in those
patients with renal failure.
A common use for aprotinin is to reduce bleeding during and after coronary artery
bypass grafting. It has been demonstrated to reduce the requirement for blood transfusion
and returns to theatre for bleeding. There is concern that, as it is an antifibrinolytic, it may
be associated with an increased risk of graft thrombosis.
The BART study (Blood conservation using Antifibrinolytics Randomized Trial) com-
pared the use of aprotinin and other antifibrinolytics (e.g. tranexamic acid, aminocaproic
acid) during cardiac surgery. Aprotinin was temporarily withdrawn after the study showed
an increase in mortality associated with its use, particularly related to renal and cardiovas-
cular toxicity. This suspension has now been lifted.
Aprotinin appears to be more efficacious at reducing bleeding than other antifibrinoly-
tics, but is more expensive, and associated with more allergic reactions. It may be reserved
for cases where there is a higher risk of bleeding e.g. redo sternotomy.

Question 112: TTTFT


Thromboelastography is a near-patient testing device which allows rapid assessment of
coagulation. Several machines exist of varying sophistication. They all rely on measuring the
increasing strength of a blood clot via a pin placed into a small cuvette of blood. Either the
pin rotates in the cuvette, or the cuvette rotates around the pin. Either way, blood will begin
to clot around the pin, and the movement of the pin through the blood will be increasingly
resisted as the clot evolves. This increased resistance is what is measured and translated to a
graphical display of coagulation, and the shape of this graphical representation can be used
to assess different aspects of coagulation.
Several parameters are displayed, and these are given different names by different
manufacturers. The features of importance are the time taken from the beginning of the
test until coagulation is first detected (dependent on clotting factors), the subsequent

Downloaded from https://www.cambridge.org/core. University of Edinburgh, on 19 Aug 2019 at 13:21:27, subject to the Cambridge Core terms of
use, available at https://www.cambridge.org/core/terms. https://doi.org/10.1017/9781108566100.008
Chapter 3b: Pharmacology Answers 251

rate of clot formation (dependent on clotting factors), the maximum clot firmness
(dependent predominantly on platelets) and the pattern of clot dissolution (dependent
on fibrinolysis)
®
Modern versions (e.g. ROTEM ) are more robust. They use various reagents to provide
simultaneous representation of the intrinsic and extrinsic pathways, to exclude the effect of
heparin, and assess potential deficiencies in fibrinogen.
No thromboelastrography/thromboelastometry devices are yet able to distinguish
between decreased platelet number and decreased platelet function.

Question 113: TTTTT


Each 1 ml of any 1% solution contains 10 mg of that substance. Hence, each ml of 20%
mannitol contains 200 mg. In a 500 ml bag there will be 100 g of mannitol.
®
Gelofusine (gelatin), 500 ml contains 77 mmol sodium and 60 mmol chloride ions. Its
average molecular weight is 30 000 daltons with a plasma half-life of approximately 4 hours.
There is a risk of anaphylaxis of 1 in 10 000.
®
6% Volulyte (hydroxyethyl starch), 1000 ml contains 137 mmol sodium, 4 mmol
potassium, 1.5 mmol magnesium, 110 mmol chloride and 34 mmol acetate.
Starches have higher molecular weights than gelatins so they will remain in the intravas-
cular fluid for longer (plasma half-life 12–16 hours), but with greater side effects. They can
affect the coagulation system (increasing the risk of bleeding), the renal system (causing
renal tubular obstruction) and accumulate in the reticuloendothelial system (causing
pruritus). The risk of anaphylaxis is approximately 1 in 20 000.
There are concerns over the risks of coagulopathy and acute kidney injury with hydroxy-
ethyl starches.

Question 114: TFFFT


1000 ml of 0.9% saline has the following contents: sodium 154 mmol, chloride 154 mmol,
osmolality 308 mosmol.kg–1, pH 5.
Following infusion of 1000 ml of normal saline, the distribution ratio will be the same as
the plasma and interstitial fluid, which is 1:3. Hence, 250 ml will remain in the intravascular
space and 750 ml in the interstitial compartment. This is why blood loss of 1000 ml requires
three to four times the volume of IV fluid replacement to restore normal intravascular
volume.
Infusing high quantities of normal saline can cause a hyperchloraemic acidosis. This can
lead to renal vasoconstriction and reduced GFR.
As normal saline does not contain any potassium, hypokalaemia may present, which may
precipitate arrhythmias.

Question 115: FFTTF


Packed red cells are stored in an anticoagulant preservative solution, either citrate phos-
phate dextrose (CPD) or saline adenine glucose mannitol (SAGM).
The citrate binds calcium and acts as an anticoagulant, and the dextrose and adenine
support red cell metabolism.
Blood products are stored at the following temperatures:

Downloaded from https://www.cambridge.org/core. University of Edinburgh, on 19 Aug 2019 at 13:21:27, subject to the Cambridge Core terms of
use, available at https://www.cambridge.org/core/terms. https://doi.org/10.1017/9781108566100.008
252 Chapter 3b: Pharmacology Answers

Packed red cells 4 °C


FFP –20 °C
Cryoprecipitate –20 °C
Platelets room temperature
Albumin room temperature
A unit of packed cells will remain usable for 35 days, although there are metabolic changes
during this period. 2,3-DPG levels decrease, shifting the oxygen dissociation curve to the left
and decreasing oxygen delivery to the tissues. Additionally, cell membrane integrity weak-
ens, leading to potassium leakage and increasing potassium concentration.

Question 116: TFFTT


Complications can be classified into immunological, infective, metabolic and others:
• Immunological:
• Haemolytic – usually due to ABO incompatibility. This presents as hypotension,
tachycardia and pulmonary oedema
• Acute transfusion reactions – due to minor antigen/antibody incompatibility, e.g.
transfusion-related acute lung injury (TRALI)
• Infective:
• Viral (hepatitis, HIV, CMV), bacterial, prion disease, malaria
• Metabolic:
• Acidosis, hyperkalaemia, hypocalcaemia, hypothermia
• Other:
• Pulmonary oedema

Question 117: TFFTT


In diabetic ketoacidosis, normal saline is the best initial fluid to use as these patients are
depleted in both sodium and water due to the osmotic diuresis caused by the
hyperglycaemia.
A massive acute upper GI bleed requires blood transfusion.
Controversies still surround the colloid/crystalloid debate, however it is now accepted
that colloids such as starches are less favoured for patients with severe sepsis due to
complications such as acute kidney injury and coagulopathy. Hartmann’s solution would
be more suitable.
Diminished secretion of ADH in neurogenic diabetes insipidus leads to polyuria, as the
kidneys are unable to concentrate the urine. Hence, mostly water is lost. 5% dextrose is
therefore the best fluid replacement.
Severe vomiting and diarrhoea leads to the loss of fluid and electrolytes, and high sodium
and chloride losses. Normal saline would be initially appropriate in this case. Potassium
chloride would also need to be added to the fluid regime.

Downloaded from https://www.cambridge.org/core. University of Edinburgh, on 19 Aug 2019 at 13:21:27, subject to the Cambridge Core terms of
use, available at https://www.cambridge.org/core/terms. https://doi.org/10.1017/9781108566100.008
Chapter 3b: Pharmacology Answers 253

Question 118: FTFTT


Sodium chloride 0.18%/4% dextrose contains 31 mmol.l–1 of sodium and 31 mmol.l–1 of
chloride ions. The glucose content is 40 g.
Human albumin 4.5% is a colloid-containing protein derived from human plasma. It is
stored at room temperature. The 4.5% concentration is isotonic. More concentrated solu-
tions (20%) are available. Hypersensitivity reactions can occur.
Colloids are largely restricted to the intravascular compartment due to their high
molecular weight.
One litre of 5% dextrose is distributed to all the fluid compartments in proportion to their
total body water.
The ratio of ICF:ECF is about 2:1 therefore ICF = 660 ml, ECF = 340 ml.
In the ECF, the ratio of interstitial fluid volume (ISF) to intravascular fluid volume (IVF)
is approximately 3:1, hence ECF = 340 ml (ISF = 255 ml and IVF = 85 ml).

Question 119: TFFFF


Phenytoin is a drug used to treat grand mal seizures. It also has a role in the treatment of
chronic pain and as a Class 1b antiarrhythmic agent. It owes its cardiac and neurological
effects to its action as a membrane-stabilizing agent.
It is a P450 enzyme inducer and thus should be used with care in patients with poly-
pharmacy. It is variably absorbed orally and is 90% protein-bound. Because it displays
saturation kinetics over concentrations of 10 mg.l–1, levels need to be monitored.
Skin and gum signs are idiosyncratic, whereas confusion, tremor, ataxia and nystagmus
may indicate toxicity.

Question 120: FTFTF


Levetiracetam is a newer antiepileptic agent that is well tolerated in the majority of patients.
It can be used in generalized, partial and myoclonic seizure prophylaxis. Its mode of action
is unclear, but it is known to prevent presynaptic calcium channel activation, attenuating
nerve impulse conduction.
While sodium valproate has activity at GABA channels, its main mode of action is via
sodium channel blockade.
Carbamazepine has a range of therapeutic indications outside of epilepsy treatment.
These include its use as a mood stabilizer and in the treatment of trigeminal neuralgia
and other chronic pain disorders. It is a P450 enzyme inducer.

Question 121: TTFTF


Prostaglandin analogues do indeed increase the outflow of fluid from the uveal–scleral tract
to decrease intraocular pressure.
Acetazolomide decreases the renal absorption of bicarbonate, causing a relative metabolic
acidosis. This is exploited therapeutically to treat the respiratory alkalosis associated with
altitude sickness. In the eye, reduced bicarbonate reduces the influx of sodium and hence
water, which would have followed sodium by osmosis.
By blocking β-receptors in the eye, timolol reduces aqueous humour production.

Downloaded from https://www.cambridge.org/core. University of Edinburgh, on 19 Aug 2019 at 13:21:27, subject to the Cambridge Core terms of
use, available at https://www.cambridge.org/core/terms. https://doi.org/10.1017/9781108566100.008
254 Chapter 3b: Pharmacology Answers

Both suxamethonium and ketamine increase intraocular pressure and should be used
with caution in cases of penetrating trauma to the globe.

Question 122: TFFTF


Magnesium sulfate acts a membrane stabilizer and slows the rate of sinoatrial (SA) node
impulse formation and prolongs SA conduction time, the PR interval and atrioventricular
nodal effective refractory period. Hence, it has antiarrhythmic properties. The mechanism
of action of magnesium in polymorphic VT is thought to be as a result of shortening of the
action potential through myocardial K+ channels.
Although magnesium will lower blood pressure as a result of peripheral vasodilatation, it
is not used in severe pre-eclampsia for this purpose, but for the prevention and treatment of
eclamptic seizures. Drugs such as labetalol, nifedipine and hydralazine are more commonly
used to lower blood pressure.
Hyporeflexia and muscle weakness tend to only occur with toxic levels of magnesium and
this is rare. Common side effects include flushing, nausea, headache and dizziness.
Magnesium is a natural calcium antagonist and therefore its toxic effects can be reversed
by the administration of calcium.
Magnesium produces a dose-dependent presynaptic inhibition of acetylcholine release at
the neuromuscular junction. It enhances the effects of other CNS depressants and neuro-
muscular blocking agents and therefore dose reductions of these drugs of up to 50% should
be considered in the presence of magnesium sulfate treatment or hypermagnesemia.

Question 123: FFTTT


Magnesium is one of the most abundant intracellular cations in the human body (~39% is
within cells, ~1% remains extracellular with the rest mainly contained within the skeleton)
and is an essential co-factor in over 300 enzyme systems. It is also essential for the
production of ATP, DNA, RNA and protein function.
It is absorbed from the GI tract and more than 50% of dietary magnesium is excreted in
the urine. Magnesium deficiency is seen in malnourished patients and 25–60% of critically
ill patients will have low levels. Hypomagnesaemia is a contributory factor in prolonged
respiratory weaning from a ventilator.
Magnesium has a wide range of therapeutic benefits; it is a highly effective bronchodilator
and is part of the British Thoracic Society guidelines in the treatment of asthma. It exerts a
renal vasodilator and diuretic effect, and decreases uterine tone and contractility.
Magnesium prolongs the clotting time of whole blood, decreases thromboxane B2 synthesis
and inhibits thrombin-induced platelet aggregation.

Question 124: TTTFT


Salbutamol is a synthetic sympathomimetic agent with actions mainly at β2-adrenoceptors
but also has minor β1-agonistic effects. Salbutamol stimulates adenyl cyclase, which
increases intracellular cAMP causing bronchodilatation and uterine relaxation. It also
inhibits the release of histamine.
Its β-effects lead to raised blood glucose and free fatty acids.

Downloaded from https://www.cambridge.org/core. University of Edinburgh, on 19 Aug 2019 at 13:21:27, subject to the Cambridge Core terms of
use, available at https://www.cambridge.org/core/terms. https://doi.org/10.1017/9781108566100.008
Chapter 3b: Pharmacology Answers 255

Salbutamol is a known cause of lactic acidosis. The mechanism is thought to be due to


increased glycogenolysis and lipolysis due to β2-stimulation (type B lactic acidosis) rather
than impaired oxygen delivery to the tissues (type A lactic acidosis).
Uncomplicated preterm labour may be treated with salbutamol as it relaxes the gravid
uterus. Approximately 10% of the delivered dose may cross the placenta leading to a fetal
tachycardia.

Question 125: TTFFT


The earlier steroids are given in an acute asthma attack, the better the outcome. They reduce
mortality, relapses, subsequent hospital admission and requirement for β2-agonist therapy.
Anticholinergic agents such as ipratropium are not necessary in mild exacerbations of
asthma, however their combination with a β2-agonist causes greater bronchodilation,
leading to faster recovery and shorter duration.
Heliox (mixture of helium/oxygen) is not recommended for use in acute asthma outside a
clinical trial setting.
Some patients with near fatal asthma and a poor response to initial therapy may gain
benefit from IV aminophylline. The loading dose is 5 mg.kg–1 over 20 minutes, unless on
maintenance oral therapy, then infusion of 0.5–0.7 mg.kg–1.h.
Magnesium sulfate dose for acute asthma is 2 g (8 mmols) over 20 minutes.

Question 126: TTFFT


Prednisolone is a synthetic glucocorticosteroid.
Pharmacodynamic effects:
• CVS: Positive effect on myocardial contractility. Causes vasoconstriction by increasing
the number of α1-adrenoreceptors and β-adrenoreceptors.
• CNS: increases excitability.
• GI: Increases likelihood of peptic ulceration.
• GU: Weak mineralocorticoid effects and produces sodium retention and increased
potassium excretion.
• Metabolic/other: Stimulates gluconeogenesis. Inhibits neutrophil and macrophage
recruitment.
Pharmacokinetic effects:
• Absorption: Oral/rectal 80–100% bioavailability.
• Distribution: Reversibly bound in the plasma to albumin.
• Metabolism: Occurs in the liver by hydroxylation.
• Excretion: 11–14% excreted unchanged.

Question 127: FTFFT


Carbocisteine is a mucoactive drug that has proven anti-inflammatory and antioxidant
properties. This has been demonstrated in both animal and human studies. Oral absorption
is good and it can affect gastric mucus. It should therefore be avoided in those with active
gastric ulceration. It achieves very good penetration of lung tissue and bronchial secretions.

Downloaded from https://www.cambridge.org/core. University of Edinburgh, on 19 Aug 2019 at 13:21:27, subject to the Cambridge Core terms of
use, available at https://www.cambridge.org/core/terms. https://doi.org/10.1017/9781108566100.008
256 Chapter 3b: Pharmacology Answers

N-acetylcysteine has anti-inflammatory and antioxidant properties and can also be given
via the inhaled route. It has been shown to reduce the frequency of COPD exacerbations and
hospital admissions.

Question 128: FTTFT


Erythromycin is a macrolide antibiotic used in lower doses as a prokinetic. It is a direct
motilin agonist. It causes prolongation of the QT interval, particularly in the critically ill.
This can precipitate dysrhythmias such as torsades de pointe.
It is a CYP3A4 inhibitor, so co-administration with warfarin can enhance the anti-
coagulant effect.
Metoclopramide is a centrally and peripherally acting D2-antagonist. Its central actions
account for its antiemetic effect and its neuroleptic effects. As a consequence it can lead to
dyskinesias, oculogyric crisis being an extreme manifestation. Peripherally it enhances
gastric motility through cholinergic and D2-effects. It also has direct action on gastric
smooth muscle.

Question 129: FFTTT


Ranitidine is an H2-antagonist. It is effective in intravenous and oral from with a bioavai-
labilty of 80% orally. Unlike cimetedine, which is also an H2-blocker, ranitidine has no effect
on the cytochrome P450 enzyme system.
The luminal H+/K+ ATPase pump represents the terminal phase of gastric acid secretion.
In blocking this, PPIs can almost completely stop acid secretion. This therefore renders
them more potent than H2 antagonists.

Question 130: TFTFT


When given in high doses, metoclopramide has central 5-HT antagonism in addition to its
antidopaminergic action. It also has direct action on gastric smooth muscle.
Vestibular afferents to the vomiting centre are responsible for causing emesis rather than
via the chemoreceptor trigger zone. The muscarinic antagonist hyoscine is therefore an
ideal agent for the prevention of motion sickness.
Rapid administration of cyclizine causes tachycardia.
The mode of action of dexamethasone has yet to be elucidated.

Question 131: FTTTT


Lactulose is an osmotically active laxative agent. It is broken to active acidic compounds by
natural gut flora. These compounds are osmotically active, causing greater hydration of
stool. The acidic conditions generated intraluminally indirectly increase gut peristalsis.
Lactulose helps to reduce ammonia load and therefore improves the symptoms of hepatic
encephalopathy.
Docusate is a direct stimulant of peristalsis.
Phosphate enemas can theoretically cause hyperphosphataemia in renal failure.
Bulk-forming laxatives including ispaghula are ideal in patients with diverticular disease.

Downloaded from https://www.cambridge.org/core. University of Edinburgh, on 19 Aug 2019 at 13:21:27, subject to the Cambridge Core terms of
use, available at https://www.cambridge.org/core/terms. https://doi.org/10.1017/9781108566100.008
Chapter 3b: Pharmacology Answers 257

Question 132: FTFFT


Hyoscine hydrobromide is a tertiary ammonium compound and antisialogogue that, unlike
atropine, has less effect on heart rate. Hyoscine butylbromide is its quaternary ammonium
compound, which has a specific role as an antispasmodic.
Atropine is a tertiary amine that crosses the blood–brain barrier, thus rendering patients
susceptible to central side effects such as confusion and sedation.
Conversely, glycopyrronium is a quaternary amine and is unable to penetrate the blood–
brain barrier. There are consequently fewer central side effects such as confusion and
blurred vision. It is has slower onset and is longer-lasting than atropine. It is also used to
offset the muscarinic-induced bradycardia caused by acetylcholinesterase inhibitors in the
reversal of neuromuscular blockade.

Question 133: FTFTF


Several types of uterotonic drug exist, reflecting the variety of receptors present in the
uterine smooth muscle. Since 2004, the National Institute of Health and Care Excellence
recommend that all women are given oxytocin 5 IU at delivery to improve uterine contrac-
tion and reduce the risk of postpartum haemorrhage. It is available in the synthetic form
®
Syntocinon . Larger doses should be avoided due to the risk of haemodynamic compromise
®
and cardiovascular collapse. Additionally, Syntocinon should be given as a slow IV bolus
to reduce the severity of these side effects. There is some evidence that lower IV doses may
be as effective.
®
Postoperative Syntocinon infusions are commonly prescribed for observed or predicted
uterine atony. Ongoing bleeding, increasing parity and twin pregnancies would be common
indications. The usual rate is 10 IU.h–1.
Ergometrine is a second line uterotonic, used alone or in combination with oxytocin
(Syntometrine ). The usual dose is 500 μg IM. It can cause a rise in blood pressure, chest
®
pain and arrhythmias. It is contraindicated in patients with pre-eclampsia, eclampsia, severe
hypertension or cardiac disease, and should be used with caution in patients with mild
hypertension.
Carboprost is a synthetic analogue of prostaglandin F2α. It is marketed as Hemabate and ®
used to treat postpartum bleeding not controlled by other methods. As a stimulant of
smooth muscle contraction, it commonly causes diarrhoea and should be used with caution
in asthmatics. Misoprostol is a synthetic analogue of prostaglandin E1, which is used as a
pessary to induce labour, or may be given rectally following caesarean delivery to reduce risk
of postpartum haemorrhage. Prostaglandins may be used in early pregnancy to induce
abortion. Atosiban is an oxytocin receptor antagonist, and therefore a tocolytic.

Question 134: TTTTT


Tocolytics are usually implemented by obstetricians to stop premature labour, although
anaesthetists may become involved in their use during emergency tocolysis for intrauterine
fetal resuscitation or emergency uterine relaxation in theatre.
Many agents can be used to relax the uterus. β2-agonists such as ritodrine, terbutaline and
salbutamol are all effective, but their use is declining due to unpleasant CVS side effects.

Downloaded from https://www.cambridge.org/core. University of Edinburgh, on 19 Aug 2019 at 13:21:27, subject to the Cambridge Core terms of
use, available at https://www.cambridge.org/core/terms. https://doi.org/10.1017/9781108566100.008
258 Chapter 3b: Pharmacology Answers

Ritodrine is no longer manufactured. For emergency uterine relaxation intraoperatively,


sublingual GTN or subcutaneous terbutaline remain in use as they are easy to administer.
There is little evidence to show that tocolysis improves neonatal outcomes when used for
premature labour. Additionally, there is little evidence to support the use of one particular
agent. The Royal College of Obstetricians and Gynaecologists suggest avoiding ritodrine
and using either nifedipine or atosiban. Nifedipine is cheaper and simpler to administer.
Use of a slower-release formulation may reduce the risk of maternal hypotension.
Other tocolytic agents include magnesium sulfate and the NSAIDs indomethacin and
ketoralac (which act as prostaglandin synthetase inhibitors).

Question 135: TFTFT


There are two main classes of antifungal agents: polyenes and azoles.
Polyenes exhibit a similar toxicity to mammalian and fungal cells. The main agent is
amphotericin B, which has a wide spectrum of activity. It is used for serious systemic
infections, covering Aspergillus, Candida and Cryptococcus.
Amphotericin reacts with ergosterol which is part of the fungal cell membrane. It creates
pores within this membrane causing cell lysis and death. It is nephrotoxic and may cause
gastrointestinal disturbance, convulsions, anaphylaxis and deranged liver function tests.
Azoles are divided further into imidazoles (ketoconazole) and triazoles (fluconazole,
posaconazole). They differ with regards to spectrum of activity, pharmacokinetics and
toxicities. Their mechanism of action involves inhibiting the biosynthesis of ergosterol,
increasing cellular permeability and cell death.
Fluconazole is available in both oral and IV preparations. It has limited activity against
yeast infections but distributes well into the central nervous system.
Posaconazole is only available as an oral suspension and effective absorption requires oral
intake. It has the broadest spectrum of activity with the fewest drug interactions.
All azoles are associated with hepatic dysfunction, ranging from mild elevation of the
transaminases to fulminant hepatic failure.

Question 136: TTFFF


Table 3.136.1

Bactericidal Bacteriostatic

Glycopeptides (vancomycin, teicoplanin) Trimethoprim


Penicillins Erythromycin
Aminoglycosides (gentamicin) Clindamycin
Carbapenems (meropenem) Tetracycline
Monobactams (aztreonam)
Rifampicin

Downloaded from https://www.cambridge.org/core. University of Edinburgh, on 19 Aug 2019 at 13:21:27, subject to the Cambridge Core terms of
use, available at https://www.cambridge.org/core/terms. https://doi.org/10.1017/9781108566100.008
Chapter 3b: Pharmacology Answers 259

Question 137: TFTTT


In a few cases, bacteria have natural resistance to antibiotics, i.e. they have intrinsic
resistance. These organisms tend to have low virulence, but may become a problem in
vulnerable patients e.g. Pseudomonas. The main mechanism of resistance is the alteration of
the genome in the micro-organism. It can be acquired, either by genetic mutation or more
commonly by gene transfer.
Genetic material can be transferred in the following ways:
• Transformation: DNA released from bacteria is taken up and incorporated into the
genome of another bacterium.
• Transduction: Bacteriophages are viruses that infect bacteria. They can exchange or
transfer DNA which may encode resistance.
• Conjugation: Involves the direct contact of bacteria and exchange of genetic material,
most often plasmids.
• Transposons: Small segments of DNA that encode resistance.

Question 138: TFFFT


Ganciclovir is an antiviral agent used for the treatment and prevention of CMV infection.
Metronidazole inhibits DNA synthesis and is active against anaerobes and protozoa.
Vancomycin is a glycopeptide and inhibits cell wall synthesis. Histamine release may
cause hypotension, tachycardia and widespread rash known as ‘red man syndrome’. This is
more common with rapid administration; infusion should not exceed 10 mg.min–1.
Aminoglycosides inhibit protein synthesis and prolong the action of non-depolarizing
muscle relaxants.
β-Lactam antibiotics inhibit the growth of sensitive bacteria by inactivating enzymes in
the bacterial cell wall. Examples include: penicillins, cephalosporins, carbapenems,
monobactams.
The production of β-lactamase from bacterial cells is a form of resistance to some
β-lactam antibiotics. It hydrolyzes the β-lactam ring and renders the antibiotic ineffective.

Question 139: TTFFF


Human insulin is produced commercially by recombinant DNA techniques. It is rapidly
metabolized in the liver, muscle and kidneys by glutathione insulin transhydrogenase.
Insulin is an anabolic polypeptide hormone whose main actions are to stimulate carbo-
hydrate metabolism, protein synthesis and lipogenesis. It causes fat deposition in adipose
tissue by increasing the hepatic synthesis of fatty acids; these are used within the liver to
form triglycerides, which are then released into the blood. Insulin also stimulates lipopro-
tein lipase in adipose tissue, which splits triglycerides into fatty acids. This enables them to
be taken up and stored.
Insulin may be administered intravenously, subcutaneously or intramuscularly; it is
inactive when given orally, as it is destroyed by proteases within the GI tract.
It increases the permeability of many cells to potassium, magnesium and phosphate ions.
The effect on potassium is clinically important. Insulin activates Na+/K+ ATPases, causing a
flux of potassium into the cell. Therefore, it is used in the acute management of
hyperkalaemia.

Downloaded from https://www.cambridge.org/core. University of Edinburgh, on 19 Aug 2019 at 13:21:27, subject to the Cambridge Core terms of
use, available at https://www.cambridge.org/core/terms. https://doi.org/10.1017/9781108566100.008
260 Chapter 3b: Pharmacology Answers

Insulin is not significantly removed by haemodialysis, probably because of its large


molecular weight.

Question 140: FTTTT


Sulfonylureas are used to control Type II or non-insulin-dependent diabetes. These drugs
work by displacing insulin from β-cells in the islets of Langerhans. For this reason, they are
ineffective in those requiring treatment with insulin, who by definition have no functioning
β-cells. β-blockers potentiate the effect of sulfonylureas such as glibenclamide, either as a
result of displacement from plasma proteins or inhibition of their hepatic metabolism.
Metformin is the only currently available biguanide in the UK. Its mechanism of action
may include delayed uptake of glucose from the gut, increased peripheral insulin sensitivity
(increasing peripheral glucose utilization) and inhibition of hepatic and renal gluconeogen-
esis. It does lower plasma cholesterol, triglycerides and low-density lipoproteins. Metformin
inhibits lactate metabolism and its use may rarely be complicated by profound lactic
acidosis, especially if taken by alcohol abusers or those with renal impairment/severe
dehydration.

Question 141: TFTFF


Thiazolidinediones, including rosiglitazone, were introduced in the late 1990s as a treatment
for Type 2 diabetes mellitus. They act by activating peroxisome proliferator-activated recep-
tors (PPARs), a group of nuclear receptors with greatest affinity for PPAR-γ1 and γ2. This
causes upregulation in a great number of genes, which result in decreased insulin resistance,
decreased leptin levels (increased appetite) and reduction of certain types of lipids. They also
cause a decrease in a number of proinflammatory cytokines, including IL-6 and TNFα.
The main side effect of all thiazolidinediones is water retention, leading to oedema. This
occurs in up to 5% of individuals, and may lead to decompensation of potentially previously
unrecognized heart failure. These drugs should be prescribed with extreme caution in this
situation and especially in patients with decreased ventricular function (NYHA grade III or
IV heart failure).
Sitagliptin is a newer oral hypoglycaemic agent that inhibits the dipeptidyl peptidase 4
(DPP4) receptor. This enzyme-inhibiting drug is used in combination with other hypogly-
caemic agents or alone in the treatment of Type 2 diabetes mellitus. These drugs were
developed from the saliva of the Gila monster and competitively inhibit DPP4. This enzyme
breaks down incretins (GI hormones) released in response to a meal. By preventing the
action of incretins, they are able to increase insulin secretion and suppress the release of
glucagon by the pancreas.

Question 142: TFTFT


Thyroxine (T4) is essential for the regulation of metabolism and development of the nervous
system. It exerts its effects by conversion to triiodothronine (T3), which binds to nuclear
receptors, subsequently modulating gene transcription and protein synthesis. It increases
basal metabolic rate and heat production, inducing sweating. It increases heart rate and
stroke volume.
Both thyroxine and cortisol increase lung maturation.

Downloaded from https://www.cambridge.org/core. University of Edinburgh, on 19 Aug 2019 at 13:21:27, subject to the Cambridge Core terms of
use, available at https://www.cambridge.org/core/terms. https://doi.org/10.1017/9781108566100.008
Chapter 3b: Pharmacology Answers 261

Thyroxine has been shown to reduce non-HDL cholesterol in patients with subclinical
hypothyroidism and primary hypothyroidism. A persistently low level of thyroxine disturbs
Sertoli cell function, resulting in alterations in adult spermatogenesis.

Question 143: TTTTT


As a prodrug, carbimazole undergoes metabolism to the active metabolite thiamazole,
which inhibits iodination of tyrosyl residues in thyroglobulin. It also acts to inhibit the
coupling of iodotyrosines. These actions ultimately result in the inhibition of thyroid
hormone production.
Potential side effects include the development of a maculopapular rash, which does not
necessarily require treatment to be discontinued. Cases of myopathy related to carbimazole
have been reported and patients who complain of myalgia whilst on carbimazole should
have creatine phosphokinase levels monitored. Bone marrow depression leading to neu-
tropenia, leucopenia and agranulocytosis is a clearly recognized risk. There are documented
cases of fatal agranulocytosis. In cases of significantly abnormal liver function tests, hepa-
titis, cholestatic hepatitis or cholestatic jaundice, carbimazole should be withdrawn.
Angioedema and type III hypersensitivity reactions such as cutaneous vasculitis have also
been recorded.

Question 144: FTFTT


Prednisolone 5 mg is equivalent to betamethasone 750 μg, dexamethasone 750 μg, hydro-
cortisone 20 mg, methylprednisolone 4 mg and triamcinolone 4 mg. However, this does not
take account of any mineralocorticoid action. Such action may be significant with hydro-
cortisone, less so with dexamethasone and negligible with prednisolone.

Question 145: FFTFT


Glucocorticoids have direct effects on brain metabolism and readily cross the blood–brain
barrier. They decrease cerebral oedema and are used therapeutically for increased intracra-
nial pressure. They have a propensity to cause insomnia with a reduction in rapid eye
movement (REM) sleep. The effect of appetite stimulation can be advantageous in oncology
patients. There may be a general increase in irritability and emotional instability, with a
decrease in libido.
There is a direct inhibitory effect of glucocorticoids on the epiphyses and thus potential
for overall growth inhibition. There is a decrease in peripheral conversion of thyroxine (T4)
to triiodothyronine (T3). Thyroid-stimulating hormone (TSH) is also suppressed by over-
riding thyrotropin-releasing hormone (TRH) induction.

Question 146: TFFFT


Caffeine competitively inhibits phosphodiesterase, which is the enzyme that degrades cyclic
AMP. Caffeine also exerts its physiological effects in part through antagonism of central
adenosine receptors. Oral absorption of the methylxanthines is good. Caffeine crosses the
placenta and is also excreted in breast milk.
Nicotine has high lipid solubility and thus easily traverses the blood–brain barrier.

Downloaded from https://www.cambridge.org/core. University of Edinburgh, on 19 Aug 2019 at 13:21:27, subject to the Cambridge Core terms of
use, available at https://www.cambridge.org/core/terms. https://doi.org/10.1017/9781108566100.008
262 Chapter 3b: Pharmacology Answers

The amphetamines differ from catecholamines as they lack the catechol structure (–OH
groups at positions 3 and 4 of the phenyl ring). This means they cannot be metabolized by
catechol-O-methyltransferase (COMT).
Children with attention deficit hyperactivity disorder (ADHD) appear to have weak
dopamine signals. This is thought to result in altered reward responses and thus a reduced
attention span. Methylphenidate acts by blocking dopamine transport in the synaptic cleft
and thus enhancing dopamine activity.

Question 147: FTTTF


Doxapram is a peripherally and centrally acting respiratory stimulant. It may be used in the
treatment of postoperative respiratory depression, acute respiratory failure and neonatal
apnoea. It increases the depth and rate of respiration. It can also increase cardiac output and
may also result in cardiac dysrythmias.
Concurrent administration with monoamine oxidase inhibitors can result in a significant
hypertensive response.
The volume of distribution is 1.5 l.kg–1. The onset of action is around 1 minute after
intravenous administration.
It is metabolized in the liver, with 2-ketodoxapram being the major metabolite. It is
prepared in 0.9% benzyl alcohol as a preservative, which is particularly important when
used in neonates, as it can result in metabolic acidosis and an increased incidence of
kernicterus.

Question 148: TFTFF


Those patients who smoke less than 12 hours prior to a general anaesthetic have an
increased risk of cardiovascular events due to increased myocardial workload and a
decreased oxygen delivery, secondary to carbon monoxide. This can also lead to coronary
vasoconstriction and excessive catecholamine release. The sympathomimetic effect of
nicotine on the heart is transient, lasting only 20–30 minutes.
There does appear to be an association between expired carbon monoxide and the
frequency of ST depression during general anaesthesia.
The incidence of postoperative nausea and vomiting is lower in smokers.
The mechanism of nicotine-induced analgesia appears to involve several pathways
including activation of spinal cord descending inhibitory pain pathways and local release
of noradrenaline, with activation of α2-adrenergic receptors.
Return of normal pulmonary immune function takes at least 6 weeks. Therefore short-
term abstinence from smoking will not decrease the incidence of postoperative pulmonary
complications.

Question 149: FFFTF


Ethanol enhances the responses mediated by the GABAA receptor, as well as at serotonin
and glycine receptors. There is also antagonism of the excitatory neurotransmitter gluta-
mate at the N-methyl-D-aspartate (NMDA) receptor.

Downloaded from https://www.cambridge.org/core. University of Edinburgh, on 19 Aug 2019 at 13:21:27, subject to the Cambridge Core terms of
use, available at https://www.cambridge.org/core/terms. https://doi.org/10.1017/9781108566100.008
Chapter 3b: Pharmacology Answers 263

Wernicke’s encephalopathy is a serious neurological condition that may be seen in


chronic alcohol users and is caused by thiamine (B1) deficiency. It is characterized by the
triad of acute mental confusion, ataxia and ophthalmoplegia.
α2-Agonists such as clonidine and dexmedetomidine may be useful to control autonomic
instability during anaesthesia.
Acute and chronic alcohol use has been shown to result in depressive effects on innate and
adaptive immunity. This includes a significant decrease in delayed hypersensitivity.
There are multiple endocrine and metabolic effects seen in long-term alcohol use. These
include a decrease in plasma testosterone levels, hypothyroidism, hypoglycaemia and
decreased gluconeogenesis.

Question 150: FTTFT


Recreational cocaine use is increasingly recognized as a cause of cardiovascular morbidity
and mortality. If a vasopressor is required, then a direct-acting agent such as metaraminol or
phenylephrine would be preferred over ephedrine, which may result in an exaggerated
response due to levels of circulating catecholamines.
In the case of hypertension, the use of β-blockers to mitigate this may result in unopposed
α-activity, leading to further increases in blood pressure and coronary vasoconstriction.
The use of cocaine is associated with delayed gastric emptying.
MDMA is a potent serotonergic agent that leads to a substantial release of arginine
vasopressin or antidiuretic hormone (ADH). When it is combined with excessive exertion
and water intake, release of ADH can result in clinically significant hyponatraemia. This
may result in cerebral oedema, seizures and ultimately death.

Downloaded from https://www.cambridge.org/core. University of Edinburgh, on 19 Aug 2019 at 13:21:27, subject to the Cambridge Core terms of
use, available at https://www.cambridge.org/core/terms. https://doi.org/10.1017/9781108566100.008
Chapter
Physics Questions

4a
Question 1
Which of the following statements regarding SI units are correct?
a. SI stands for ‘Standard Indices’
b. There are seven SI base units
c. SI base units include current, temperature and luminance
d. Frequency is a base SI unit and is measured in hertz (Hz)
e. Force is a derived SI unit and is measured in newtons (kg.m.s−2)

Question 2
Which of the following SI definitions are correct?
a. The mole is the amount of substance of a system which contains as many elementary
entities as there are atoms in 0.012 kg of carbon-14
b. The metre is the length of the path travelled by light in vacuum during a time interval of
1/299 792 458 of a second
c. A watt is the energy expended when the point of application of a force of 1 newton moves
1 metre in the direction of the force
d. A joule is a measurement of the rate of energy expenditure (power). 1 joule is 1 watt per
second
e. A pascal is the pressure of 1 newton per square metre

Question 3
The following are exact equivalents to 1 atmosphere of pressure:
a. 1.013 bar
b. 101.33 Pa
c. 760 mmHg
d. 760 Torr
e. 9 898 043 cmH2O

Question 4
Regarding heat and temperature:
a. A kelvin is defined as 1/273.16 of the thermodynamic temperature of the triple point of
water

264
Downloaded from https://www.cambridge.org/core. University of Edinburgh, on 19 Aug 2019 at 13:21:27, subject to the Cambridge Core terms of
use, available at https://www.cambridge.org/core/terms. https://doi.org/10.1017/9781108566100.009
Chapter 4a: Physics Questions 265

b. The triple point of water is the temperature and pressure at which its three different
phases are in equilibrium
c. The triple point of water occurs at 100 oC
d. Temperature (K) is equal to temperature (°C) + 273.16
e. Heat capacity is defined as the quantity of heat required to raise the temperature of an
object by 1/273.16 K

Question 5
Regarding techniques of temperature measurement:
a. All four of these are techniques used in hospital practice: the bimetallic strip, the
resistance thermometer, the thermistor and the thermocouple
b. The bimetallic strip utilizes the generation of a small voltage at the junction of two
dissimilar metals, the magnitude of which is dependent on the temperature at the
junction
c. In a resistance thermometer, the electrical resistance increases exponentially with
temperature
d. In a thermistor, the electrical resistance decreases exponentially with temperature
e. The thermocouple utilizes the Seebeck effect to measure temperature

Question 6
Consider the following statements regarding heat production and loss:
a. In man, body temperature is normally regulated to 34 ± 0.5 oC
b. Children up to the age of 12 utilize brown fat for thermogenesis
c. Radiation typically contributes the majority of heat loss
d. Respiration typically contributes 30% of heat loss
e. Convection typically contributes 20% of heat loss

Question 7
Which of the following statements regarding latent heat are correct?
a. The change in heat required to change a vapour to a liquid is known as the latent heat of
vaporization
b. The latent heat of vaporization of a gas is zero at its critical temperature
c. When in use, the pressure gauge reading on a nitrous oxide cylinder will always be
accurate
d. The critical temperature of oxygen is –119 oC
e. The critical temperature of nitrous oxide is –36.5 oC

Question 8
Which of the following statements regarding gas laws are correct:
a. Boyle’s law describes that at a constant temperature, the volume of gaseous oxygen in a
cylinder varies inversely with gauge pressure
b. Charles’ law states that at constant pressure, the volume of a fixed mass of gas is directly
proportional to its temperature

Downloaded from https://www.cambridge.org/core. University of Edinburgh, on 19 Aug 2019 at 13:21:27, subject to the Cambridge Core terms of
use, available at https://www.cambridge.org/core/terms. https://doi.org/10.1017/9781108566100.009
266 Chapter 4a: Physics Questions

c. Gay-Lussac’s law states that at a constant volume, the absolute pressure of a fixed mass of
gas is directly proportional to its temperature
d. The standard temperature used is 273.15 K
e. The standard pressure used is 760 mmHg

Question 9
An ‘ideal gas’ is a theoretical concept. Which of the following are true regarding ideal gases?
a. Collisions never result in loss of kinetic energy
b. Van der Waals’ forces exist between molecules
c. The gas molecules are bound by Newton’s laws
d. The molecules are in constant, random motion
e. Carbon dioxide is often regarded as an ideal gas

Question 10
Which of the following statements regarding gases are true:
a. Avogadro’s hypothesis states that at a constant temperature and pressure, equal volumes
of ideal gases contain the same number of molecules
b. Avogadro’s number is 2.18 × 108
c. One mole of any gas at standard temperature and pressure will occupy 22.4 litres
d. The critical temperature is defined as the temperature above which a gas cannot be
liquefied by any pressure
e. The critical pressure is the pressure required to liquefy a vapour at its critical
temperature

Question 11
Linear regression is a term commonly used in statistical analyses of data. Which of the
following statements regarding linear regression are true?
a. The linear regression coefficient is the gradient of the plotted data
b. Analysis yields an intercept that defines the position of the line on the y-axis
c. Analysis yields a correlation coefficient that is an indication of the ‘goodness of fit’ of the
line to the data
d. Linear regression applies a technique of minimizing squared differences
e. Both the dependent and the independent variables must be continuous

Question 12
Randomization of two treatments in a clinical trial means that:
a. Results are treated in a random order
b. Treatment arms are allocated by reference to a series of random numbers
c. Results can be analyzed by Student’s t-test
d. Selection bias is reduced
e. An independent person allocates the treatment arms

Downloaded from https://www.cambridge.org/core. University of Edinburgh, on 19 Aug 2019 at 13:21:27, subject to the Cambridge Core terms of
use, available at https://www.cambridge.org/core/terms. https://doi.org/10.1017/9781108566100.009
Chapter 4a: Physics Questions 267

Question 13
In a study of 600 cancer patients, the distribution of time from diagnosis to death was
positively skewed, with a peak at two years and a median survival of three years. The
following statements are true:
a. The mean survival rate is greater than three years
b. The mean survival gives the best guide to the average length of life
c. In the first three years after diagnosis 300 patients died
d. The mean survival was less than the median survival
e. Fewer patients died at the mean time of death than the median

Question 14
Type I errors in statistics:
a. Are false positives
b. Are represented by the P value
c. Are related to sample size
d. Are reduced by double blinding
e. Are defined by incorrect rejection of the null hypothesis

Question 15
Type II errors in statistics:
a. Are occurrences of a negative test result when the actual result is positive
b. Are true negative errors
c. Are defined by incorrect acceptance of the null hypothesis
d. Are reduced if the power of a clinical trial is more than 80%
e. Are β-errors

Question 16
If data are normally distributed:
a. The mean, median and mode will be the same
b. Approximately 96% of the sample data will lie within two standard deviations from the
mean
c. It demonstrates Gaussian distribution
d. It cannot be analyzed by a Student’s t-test
e. It cannot be analyzed by non-parametric tests

Question 17
Regarding non-parametric tests:
a. An example is the Mann–Whitney rank sum test
b. Imply that the variable cannot be accurately measured
c. Can be used on small samples
d. Are generally mathematically more complex than parametric tests
e. Can be applied to samples that are not normally distributed

Downloaded from https://www.cambridge.org/core. University of Edinburgh, on 19 Aug 2019 at 13:21:27, subject to the Cambridge Core terms of
use, available at https://www.cambridge.org/core/terms. https://doi.org/10.1017/9781108566100.009
268 Chapter 4a: Physics Questions

Question 18
The chi-squared test:
a. Requires the null hypothesis to be applied
b. Measures the overall difference between the observed and expected frequencies
c. Is used to prove that one treatment is better than another
d. Applies only to continuous variables
e. Can have a result of less than zero

Question 19
Regarding categorical data:
a. The data are qualitative
b. Interval data is an example
c. Ordinal data is an example
d. Discrete quantitative data is an example
e. Nominal data is the most useful form of categorical data because it can be easily
statistically manipulated

Question 20
Regarding a forest plot:
a. Extreme outliers are represented by dots
b. Visually demonstrates the range and interquartile range
c. Can only be used in normally distributed continuous data
d. Usually uses the odds ratio or relative risk on the x-axis
e. Meta-analysis results are shown as a diamond

Question 21
The following are examples of negative exponential processes:
a. Xt = X0e–Y
b. Alcohol clearance
c. The inflation of the lungs with a Manley ventilator
d. Cardiac output measurement using a PA catheter
e. Maintenance of anaesthesia using propofol via a target-controlled infusion

Question 22
1 joule is equal to:
a. 1 N.m
b. 1 kg.m2.s–2
c. 100 000 dyne.m
d. 1 Pa.m3
e. 1 kg.m–1.s–2

Downloaded from https://www.cambridge.org/core. University of Edinburgh, on 19 Aug 2019 at 13:21:27, subject to the Cambridge Core terms of
use, available at https://www.cambridge.org/core/terms. https://doi.org/10.1017/9781108566100.009
Chapter 4a: Physics Questions 269

Question 23
Which of the following statements regarding laminar and turbulent flow are correct?
a. In laminar flow, the flow rate is inversely proportional to the fluid viscosity
b. In laminar flow, the flow rate is directly proportional to the fourth power of the radius
c. Turbulent flow will always be present if the Reynolds number is greater than 1000
d. Turbulent flow will always be present if the Reynolds number is greater than 2000
e. In turbulent flow, the flow rate is inversely proportional to the fluid density

Question 24
Regarding flowmeters:
a. Oscillatory flowmeters utilize the Coanda effect
b. The pneumotachograph is an example of a constant orifice, variable pressure flowmeter
c. The rotameter is an example of a variable pressure, variable orifice flowmeter
d. A Wright respirometer measures flow
e. A Wright respirometer tends to underestimate at low volumes

Question 25
Which of the following laws are correctly defined?
a. Charles’ law: at a constant pressure, the volume of a gas is directly proportional to its
absolute temperature
b. Henry’s law: at a constant temperature, the amount of gas dissolved in a solvent is
proportional to its partial pressure above the solvent
c. Dalton’s law: the pressure exerted by a fixed amount of gas in a mixture of gases is equal
to the pressure it would exert alone
d. Murphy’s law: the osmolality of a solution is equal to the number of osmoles per
kilogram of solvent
e. Beer’s law: the intensity of transmitted light decreases exponentially as distance travelled
through the substance increases

Question 26
Regarding antistatic footwear worn in theatre:
a. The recommended impedance should be between 75 kΩ and 10 MΩ when new
b. The impedance should be high enough to prevent dissipation of electrostatic charge
c. The impedance should be low enough to protect against electric shock
d. They are a mandatory component of the ‘British Standard EN60601’
e. Their impedance remains constant even when wet

Question 27
Regarding Class I electrical equipment:
a. It has no symbol
b. It requires at least two fuses
c. It is double-insulated

Downloaded from https://www.cambridge.org/core. University of Edinburgh, on 19 Aug 2019 at 13:21:27, subject to the Cambridge Core terms of
use, available at https://www.cambridge.org/core/terms. https://doi.org/10.1017/9781108566100.009
270 Chapter 4a: Physics Questions

d. It is the safest for use in theatre


e. Any conducting part accessible to the user should be connected to an earth wire

Question 28
Class II electrical equipment:
a. Is usually double-insulated
b. Has two squares, one inside the other, as its symbol
c. Does not require an earth wire
d. Can draw a maximum current of 24 mA
e. Can have a single layer of insulation if it is reinforced

Question 29
Regarding microshock:
a. The severity increases as the frequency of the current increases
b. The risk is high at mains frequency
c. Direct current is more likely to cause microshock than alternating current
d. It can only be caused by faulty equipment
e. Oesophageal temperature probes are a possible source of microshock

Question 30
Regarding leakage current standards:
a. Type C equipment may be connected directly to the heart
b. Type B equipment has the symbol of a square box with a stickman inside
c. Type B equipment has a maximum permitted leakage current of 500 μA
d. They are set using equipment with a single fault
e. Type CF equipment has a maximum permitted leakage current of 100 μA

Question 31
The Penaz technique:
a. Gives intermittent measurement of blood pressure
b. Approximates arterial blood pressure by non-invasive means
c. Uses photoplethysmography
d. Is uncomfortable in awake patients due to compression of the finger
e. Uses an infrared beam

Question 32
Arterial cannulae:
a. Should only be flushed with small syringes (less than 5 ml)
b. Should be continuously flushed at 10 ml.h–1
c. Should have parallel sides
d. Are nationally recommended to be inserted using full asepsis including hat, mask, gown
and sterile gloves
e. Can be inserted in the ulnar artery

Downloaded from https://www.cambridge.org/core. University of Edinburgh, on 19 Aug 2019 at 13:21:27, subject to the Cambridge Core terms of
use, available at https://www.cambridge.org/core/terms. https://doi.org/10.1017/9781108566100.009
Chapter 4a: Physics Questions 271

Question 33
Regarding blood pressure measurement:
a. A fibreoptic catheter tip transducer can be used
b. A Von Recklinghausen oscillotonometer requires a double cuff
c. With non-invasive methods the reading may be inaccurate in dysrhythmias
d. Radial artery compression can provide a blood pressure waveform
e. Ultrasound Doppler can be used to detect changes in the frequency of vibration of the
carotid arterial wall to give a blood pressure reading

Question 34
Regarding fluid manometers:
a. Mercury can be used to measure pressures up to atmospheric pressure
b. The manometer tube top must not be open
c. They can be used to measure gauge pressures
d. They can be designed to have a sloped tube
e. The effect of surface tension on the liquid inside the manometer can affect the readings

Question 35
Bourdon gauges:
a. Contain liquid that cause a tube to uncoil and move a pointer over a scale on a dial
b. Can be used to measure gauge pressures
c. Cannot be used for measuring pressures above 100 kPa
d. Can be used to measure absolute pressures
e. Cannot act as a differential gauge pressure measurement device

Question 36
The following will increase the damping of an arterial trace:
a. Blood clots
b. Air bubbles
c. A long arterial cannula
d. Use of a tourniquet on the measured limb
e. An arterial cannula that is too stiff

Question 37
Moderate damping of an arterial trace can make the following measurements inaccurate:
a. Systolic blood pressure
b. Diastolic blood pressure
c. Mean arterial pressure
d. Heart rate
e. Stroke volume

Downloaded from https://www.cambridge.org/core. University of Edinburgh, on 19 Aug 2019 at 13:21:27, subject to the Cambridge Core terms of
use, available at https://www.cambridge.org/core/terms. https://doi.org/10.1017/9781108566100.009
272 Chapter 4a: Physics Questions

Question 38
Concerning intra-arterial blood pressure monitoring:
a. The brachial artery must not be cannulated
b. The limb that is cannulated should be kept level with the transducer
c. Normal saline should be used in the transducer set in order to minimize the chance of
microshock
d. The system is designed to keep the resonant frequency above 40 Hz
e. Accuracy of strain gauge-type transducers is significantly altered by changes in ambient
temperature

Question 39
Regarding automated non-invasive blood pressure measurement systems:
a. They are associated with a 1 in 100 chance of permanent nerve damage distal to the point
of application
b. They give reliable results in atrial fibrillation
c. They detect mean arterial pressure most accurately
d. They always require two cuffs
e. Oscillations maximally increase in magnitude at systolic pressure

Question 40
Regarding damping:
a. Optimal damping means the system responds rapidly to a change in signal, but allows a
small amount of overshoot
b. Optimal damping has a coefficient of 0.47
c. Underdamping might allow resonance within the system, which has the advantage of
more faithful reproduction of the intended waveform
d. Critical damping represents the best compromise between response speed and accuracy
e. Critical damping has a coefficient of 1.0

Question 41
Clinical signs of adequate return of neuromuscular function following non-depolarizing
neuromuscular blockade include:
a. Sustained head lift for at least 3 seconds
b. Generation of a vital capacity breath of at least 4 ml.kg–1
c. Generation of a vital capacity breath of at least 10 ml.kg–1
d. Tidal volume adequate to maintain oxygen saturation above 96%
e. Normal respiratory volumes can be generated with only 20% functional diaphragm
muscle receptors

Question 42
There are multiple methods of cardiac output monitoring. Regarding PiCCO as a method,
which of the following are true?

Downloaded from https://www.cambridge.org/core. University of Edinburgh, on 19 Aug 2019 at 13:21:27, subject to the Cambridge Core terms of
use, available at https://www.cambridge.org/core/terms. https://doi.org/10.1017/9781108566100.009
Chapter 4a: Physics Questions 273

a. PiCCO stands for ‘power induced contour cardiac output’


b. It uses the principle of the Doppler effect
c. It requires a central line
d. It uses a standard arterial cannula
e. It can also give an estimate of intrathoracic blood volume

Question 43
Concerning train-of-four stimulation:
a. The TOF ratio compares the ratio of the strengths of the first and fourth twitches
b. Four twitches of 0.5 Hz each are applied over 2 seconds
c. On administration of non-depolarizing neuromuscular blocking agent, fade occurs
before the disappearance of the twitches
d. On recovery, the fourth twitch appears first
e. For upper abdominal surgery, at least two twitches must be absent to achieve adequate
surgical condition

Question 44
A clinically useful nerve stimulator for the assessment of neuromuscular block must have
the following characteristics:
a. It should be able to generate current pulses up to 60 microamps
b. It should be powered by a mains transformer to guarantee optimal function
c. The supramaximal twitch current should be determined when neuromuscular block is
fully developed
d. It has leads marked with blue and yellow insulation
e. The pulse width should be variable between 0.01 and 0.1 seconds

Question 45
Which of the following concerning transthoracic impedance (also known as transthoracic
electrical bioimpedance) as a method of cardiac output measurement are true?
a. It is an invasive method of cardiac output measurement
b. It measures the electrical resistance of the thorax
c. It uses a low-amplitude, high-frequency alternating current
d. It can provide a continuous measurement of cardiac output
e. It is unaffected by arrhythmias

Question 46
The following are characteristics of laminar flow:
a. Eddies and turbulence can occur
b. Flow rate is greatest at the centre of the flow stream, being twice the flow rate at the side
of the tube
c. A pressure difference is essential for fluid to flow

Downloaded from https://www.cambridge.org/core. University of Edinburgh, on 19 Aug 2019 at 13:21:27, subject to the Cambridge Core terms of
use, available at https://www.cambridge.org/core/terms. https://doi.org/10.1017/9781108566100.009
274 Chapter 4a: Physics Questions

d. It is impossible to quantify the relationship between pressure and flow without knowing
the viscosity and density of the fluid
e. Resistance of the tube is a product of the pressure and flow

Question 47
With regards to turbulent flow in a tube of liquid;
a. Flow is inversely proportional to both the square root of the pressure and the square of
the tube radius
b. Flow is inversely proportional to the square root of the tube length
c. Flow is inversely proportional to the square root of the fluid density
d. Fluid viscosity becomes increasingly important in determining resistance as the density
of the fluid increases
e. On transition from laminar to turbulent flow, resistance to flow increases, but less
energy is required to generate the same flow

Question 48
Concerning Wright’s respirometer:
a. The device measures gas flow directly by mechanical rotation of vanes
b. It is exquisitely accurate for continuous flow
c. It is best suited for paediatric practice where flow rates are reduced
d. Plotting accuracy against flow rate will yield a U-shaped curve
e. It can be used to measure tidal volumes in anaesthesia

Question 49
The pneumotachograph:
a. Is a constant-pressure, constant-orifice device
b. Measures volumes, and hence flow can be calculated (volume of gas per unit time)
c. The gauze screen causes small eddy currents
d. The pressure difference across the gauze screen is proportional to flow
e. Some modern pneumotachograph devices employ a variable orifice to maintain laminar
flow as fluid flow rates change

Question 50
Rotameters are commonly seen on anaesthetic machines. Which of the following are true of
these devices?
a. They are variable-pressure, variable-orifice devices
b. They measure gas flow directly
c. The glass tube is always tapered
d. The pressure across the bobbin changes with flow rate
e. Calibration is dependent on both the viscosity and the density of the fluid

Downloaded from https://www.cambridge.org/core. University of Edinburgh, on 19 Aug 2019 at 13:21:27, subject to the Cambridge Core terms of
use, available at https://www.cambridge.org/core/terms. https://doi.org/10.1017/9781108566100.009
Chapter 4a: Physics Questions 275

Question 51
Electronic signal analysis is common in a range of anaesthetic monitoring systems. Which
of the following are true?
a. Electronic noise only occurs in amplifiers
b. High-pass filters reject signals below a specified frequency
c. Low-pass filters reject signals below a specified frequency
d. Notch filters can be used to reject a specific frequency, for example, diathermy
interference
e. SNR = 10 log10(signal amplitude/noise amplitude)

Question 52
Regarding a thermodilution technique using a Swan–Ganz technique for cardiac output
measurement, which of the following are true?
a. Requires a specialized arterial cannula
b. Requires the catheter to be correctly placed in the pulmonary artery
c. Cardiac output calculations are based on the Stewart–Hamilton equation
d. Cardiac output is directly proportional to the area under the temperature–time curve
produced
e. May be inaccurate in the presence of tricuspid regurgitation

Question 53
The design of monitoring systems can greatly influence their use and usefulness.
Considering the case of a direct arterial line pressure monitor, which of the following are
true?
a. For Fourier analyisis to be clinically useful, analysis up to the fortieth harmonic is
required
b. The natural frequency of oscillation is different from the resonant frequency
c. A harmonic is a multiple of the fundamental frequency
d. Critical damping occurs when the damping factor, D, is 0.64
e. If the system is underdamped, there will be a falsely low systolic and falsely low diastolic
pressure

Question 54
Concerning exponential functions:
a. The rate of change of quantity of a substance is directly proportional to the quantity of
substance at that time
b. Nitrogen washout during preoxygenation is an example of an exponential decay curve
c. Lung volume during passive expiration is an example of an exponential decay curve
d. Both drug wash-in and drug wash-out curves are examples of exponential curves
e. Lung volume during pressure-controlled ventilation is not an example of exponential
growth curve

Downloaded from https://www.cambridge.org/core. University of Edinburgh, on 19 Aug 2019 at 13:21:27, subject to the Cambridge Core terms of
use, available at https://www.cambridge.org/core/terms. https://doi.org/10.1017/9781108566100.009
276 Chapter 4a: Physics Questions

Question 55
Concerning defibrillator waveforms:
a. A monophasic waveform is an example of a dampened sinusoidal wave
b. In a biphasic waveform, current flows in alternating directions, forming a complete
biphasic waveform in approximately 10 ms
c. The advantage of a monophasic waveform is that lower energy can be used, reducing the
risk of burns and myocardial damage
d. Both monophasic and biphasic defibrillators deliver an AC shock
e. Because biphasic defibrillators deliver an AC shock there is less myocardial damage and
the shock is less arrhythmogenic

Question 56
Concerning defibrillator function:
a. A step-up transformer is often used to generate a potential difference of 5000 kV
b. A capacitor is designed to generate a potential difference across the myocardium
c. Capacitors have a low reactance to AC and a high resistance to DC
d. Inductors have a low reactance to AC and a high resistance to DC
e. A modern biphasic defibrillator delivers a current of 3 A for 3 ms

Question 57
Concerning capacitance, current and energy delivery of defibrillators:
a. The energy stored in a fully charged 100 μF capacitor with a 2 kV potential difference
across it will be 100 J
b. Thoracic impedance is generally in the μΩ range
c. Internal defibrillators (monophasic type) use 360 J energy to defibrillate
d. Delivered (quoted) energy is less than stored charge due to some loss within the inductor
e. During defibrillation of pulseless VF, a synchronized shock must be delivered to prevent
R on T phenomenon

Question 58
Concerning pacing:
a. There is no risk of microshock in transvenous pacing as the potential difference required
is less than 4 V
b. Mobitz type 1 second-degree heart block is always an emergency indication for tem-
porary pacing
c. The pulse duration of transvenous pacing is shorter than in transcutaneous pacing
d. All permanent pacemakers operate on a demand mode principal
e. Modern pacemaker function can be safely changed externally by using magnets

Downloaded from https://www.cambridge.org/core. University of Edinburgh, on 19 Aug 2019 at 13:21:27, subject to the Cambridge Core terms of
use, available at https://www.cambridge.org/core/terms. https://doi.org/10.1017/9781108566100.009
Chapter 4a: Physics Questions 277

Question 59
Regarding the definitions of pulmonary function tests:
a. The peak expiratory flow rate is the maximal flow rate that can be developed by forced
expiratory effort following full inspiration
b. Maximal mid-expiratory flow rate is measured at 50% exhalation during a forced
exhalation from TLC
c. FEV1 is the volume exhaled after 1 second and is usually around 50–60% of the FVC in
healthy individuals
d. In obstructive airways disease both FEV1 and FVC are reduced, with a reduced FEV1/
FVC ratio
e. In restrictive lung disease both FEV1 and FVC are reduced, with a normal or increased
FEV1/FVC ratio

Question 60
Concerning volumes:
a. A normal total lung capacity is 50 ml.kg–1
b. A normal residual volume is 15–20 ml.kg–1
c. A normal tidal volume has evolved to be 1–2 ml.kg–1 to reduce the effect of volutrauma
d. A normal inspiratory capacity is 50 ml.kg–1
e. A normal vital capacity breath is approximately 65 ml.kg–1

Question 61
Concerning some common medical conditions:
a. In asthma the residual volume is usually normal
b. In small airways disease, the residual volume is usually decreased
c. In pulmonary fibrosis, the residual volume is usually normal
d. In respiratory muscle disease, the residual volume is usually normal
e. In emphysema, the residual volume is usually increased

Question 62
In the measurement of lung volume:
a. In whole body plethysmography, it is the decrease in box pressure with inspiration that is
measured
b. Whole body plethysmography cannot be used for airway resistance measurement
c. Dilution of helium in a closed circuit spirometer allows measurement of alveolar volume
d. The FRC, using the helium dilution technique, is derived from the law of mass
conservation
e. Measurement of FRC by dilution will only measure the volume of communicating gases

Question 63
FRC is increased by:
a. Standing position

Downloaded from https://www.cambridge.org/core. University of Edinburgh, on 19 Aug 2019 at 13:21:27, subject to the Cambridge Core terms of
use, available at https://www.cambridge.org/core/terms. https://doi.org/10.1017/9781108566100.009
278 Chapter 4a: Physics Questions

b. General anaesthesia
c. Asthma
d. COPD
e. PEEP

Question 64
Heat loss during surgery in an anaesthetized patient can be divided into phases. Regarding
these phases:
a. There are five phases
b. The redistribution phase involves vasodilatation, with transfer of heat from the
peripheral to the core compartment
c. The redistribution phase involves vasoconstriction, with transfer of heat from the core to
the peripheral compartment
d. The plateau phase involves vasoconstriction of the peripheral compartment
e. Regional blockade affects the plateau phase

Question 65
Accurate core body temperature measurement is an important aspect of clinical monitor-
ing. Which of the following are reliable sites for bedside core body temperature
measurement?
a. Rectum
b. Oropharynx
c. Nasopharynx
d. Upper oesophagus
e. Tympanic membrane

Question 66
There are many different ways to measure temperature in anaesthetic practice. Which of the
following concerning mercury thermometers are true?
a. They have a fast response time
b. They are used to measure high temperatures
c. They rely on the principle of liquid expansion
d. They require a power supply
e. They are expensive, therefore limiting their use

Question 67
pH is an important physiological indicator measured by arterial blood analyzers. Regarding
the use of the pH electrodes in arterial blood gas analyzers:
a. It consists of a reference electrode of silver/silver chloride
b. It consists of a pH electrode of silver/silver chloride
c. It can be used to measure H+ concentration in blood, mucus, urine and cerebrospinal
fluid

Downloaded from https://www.cambridge.org/core. University of Edinburgh, on 19 Aug 2019 at 13:21:27, subject to the Cambridge Core terms of
use, available at https://www.cambridge.org/core/terms. https://doi.org/10.1017/9781108566100.009
Chapter 4a: Physics Questions 279

d. It produces an electrical potential of 60 V per unit change in measured pH


e. It is temperature independent

Question 68
Regarding calibration and maintenance of the pH electrode in an arterial blood gas analyzer:
a. Calibration reduces errors caused by drift
b. The pH electrode system is calibrated using three buffer solutions
c. Calibration buffer solutions consist of two solutions of fixed pH
d. The pH electrode system must be maintained at 37 °C
e. Electrodes do not require cleaning

Question 69
The Severinghaus electrode is one of the electrodes used within arterial blood gas analyzers.
Regarding the Severinghaus electrode:
a. It is a modified pH electrode
b. It relies on hydrogen ion diffusion
c. The glass electrode consists of hydrogen-sensitive glass
d. The reference electrode consists of silver/silver chloride
e. It has a slow response time

Question 70
Capnography is an essential component of patient monitoring during general anaesthesia.
As such, end tidal carbon dioxide is one of the minimum recommended standards of
monitoring during anaesthesia. Carbon dioxide concentration may be measured using
which of the following methods?
a. Gas chromatography
b. Piezoelectric resonance
c. Infrared spectrophotometry
d. Mass spectroscopy
e. Raman scattering

Question 71
There are different types of analyzers used in the monitoring of end tidal carbon dioxide. In
comparison to main-stream carbon dioxide analyzers, side-stream carbon dioxide
analyzers:
a. Have a faster response time
b. Do not require calibration, whereas main-stream analyzers do
c. Can also be used for other gas and vapour analysis
d. Consist of a tube with a 1.2 mm internal diameter
e. Consist of a tube made of Teflon ®

Downloaded from https://www.cambridge.org/core. University of Edinburgh, on 19 Aug 2019 at 13:21:27, subject to the Cambridge Core terms of
use, available at https://www.cambridge.org/core/terms. https://doi.org/10.1017/9781108566100.009
280 Chapter 4a: Physics Questions

Question 72
The principles of magnetism and electromagnetism are important in many anaesthetic
devices. Which of the following statements are true?
a. In a permanent ferromagnet, atoms remain permanently aligned such that a magnetic
field is produced around the object without the movement of charge within the object
b. The wire within an electromagnet must only be made of copper
c. A current flowing through a copper wire always produces an electromagnetic field
perpendicular to the flow of current
d. The internal workings of a dynamo are the same as an electric motor
e. All ferromagnets must have both north and south poles

Question 73
Which of the following statements are true regarding transformers used in British substa-
tions and electrical devices?
a. Transformers are often used to change the voltage and current in one circuit to another,
without the loss of power
b. British homes utilize an AC frequency of 50 Hz as this has the best safety profile for
humans
c. The ratio of coils between the primary and secondary coils in the transformer determines
the current ratio in the secondary circuit
d. The peak voltage out of a British AC mains circuit is 240 V
e. Hospitals commonly have separate mains supplies of 415 V

Question 74
Electrical circuits are composed of electrical components. Which of the following state-
ments concerning electrical components are true?
a. A capacitor stores electrical charge
b. An inductor generates an electromotive force
c. A diode is a semiconductor
d. A diode changes one form of energy into another
e. An amplifier modifies the input to output ratio

Question 75
Domestic electrical devices are connected to the power outlet in the UK by a standardized
plug. Which of the following statements are correct?
a. There are four wires
b. The live wire is coloured blue
c. The earth wire is coloured with green and yellow stripes
d. If the live wire is touched electrocution can result
e. Wires can be connected to the pins of the plug in any orientation

Downloaded from https://www.cambridge.org/core. University of Edinburgh, on 19 Aug 2019 at 13:21:27, subject to the Cambridge Core terms of
use, available at https://www.cambridge.org/core/terms. https://doi.org/10.1017/9781108566100.009
Chapter 4a: Physics Questions 281

Question 76
Direct and alternating currents are types of electricity supply to an electrical circuit. Which
of the following describes direct current, rather than alternating current?
a. Describes the flow of electrons in a sinusoidal manner
b. Thermocouples are a source of direct current
c. UK mains supply is a source of direct current
d. Batteries are a source of direct current
e. Direct current is used to charge the plates of a capacitor in a defibrillator

Question 77
Ohm’s law describes an important relationship between crucial concepts in electricity.
Which of the following statements concerning Ohm’s law are true?
a. The equation can be written as I = V/R
b. It was discovered by Georg Simon Ohm
c. It defines the relationship between current, potential difference and temperature
d. States that current is directly proportional to potential difference, and inversely pro-
portional to resistance
e. Defines the relationship between temperature, electric charge and flow

Question 78
Scavenging systems are used in theatres to reduce pollution from waste anaesthetic gases.
Regarding methods of scavenging, an active scavenging system:
a. Is a high-pressure, high-volume system
b. Is a low-pressure, high-volume system
c. Is a high-pressure, low-volume system
d. Requires generation of a vacuum
e. Is powered by the patient’s expiratory flow

Question 79
Passive scavenging systems include the Cardiff Aldasorber, a canister containing activated
charcoal particles. Disadvantages of the Cardiff Aldasorber include:
a. Ability to only absorb isoflurane
b. It does not absorb nitrous oxide
c. Heating the canister leads to release of scavenged gases
d. The only indication of exhaustion of the device is increasing weight of the canister
e. It is not portable

Question 80
Circle breathing systems utilizing carbon dioxide absorption, unidirectional and pressure-
limiting relief valves are commonly used in modern anaesthetic practice. Which of the
following are true regarding this system?
a. There must be a unidirectional valve between the reservoir and the patient

Downloaded from https://www.cambridge.org/core. University of Edinburgh, on 19 Aug 2019 at 13:21:27, subject to the Cambridge Core terms of
use, available at https://www.cambridge.org/core/terms. https://doi.org/10.1017/9781108566100.009
282 Chapter 4a: Physics Questions

b. A unidirectional valve is required only on the expiratory side of the circle


c. Fresh gas must enter the system between the patient and the expiratory valve
d. The adjustable pressure-limiting valve should be placed between the patient and the
inspiratory unidirectional valve
e. Plenum vapourizers cannot be used inside the circle system

Question 81
Soda lime absorbs carbon dioxide in an exothermic reaction and is utilized in many
breathing circuits. Which of the following regarding this process are true?
a. Soda lime is capable of absorbing up to 25 litres of carbon dioxide per 100 g
b. Colour change is an indicator of soda lime exhaustion
c. Intermittent use allows soda lime to ‘regenerate’
d. White to violet is the usual colour change
e. Soda lime dust is harmless and, when inhaled, it is simply exhaled again

Question 82
The following are true of the Mapleson E and F breathing systems:
a. The type F system is a modified type E system
b. Controlled ventilation is not possible with a type E system
c. They are used mainly for adult anaesthesia due to their high resistance
d. The volume of the expiratory limb should be slightly greater than the patient’s tidal
volume
e. A flow rate of 1.5 times minute ventilation is considered adequate for spontaneous
ventilation

Question 83
Regarding the Mapleson D system used for spontaneous ventilation:
a. During the exhalation phase, dead space gas is the first to be voided
b. Fresh gas flows of two to three times the minute ventilation are required to prevent
rebreathing
c. It functions as a T-piece-type breathing system
d. To prevent rebreathing, the fresh gas flow must be high enough to eliminate exhaled
gases that have undergone gaseous exchange
e. Normocapnia can be achieved at fresh gas flow equalling minute ventilation

Question 84
Concerning the Mapleson A breathing systems:
a. If it is used for controlled ventilation at a flow rate of 9 l.min–1, it is associated with raised
inspiratory CO2
b. The coaxial version of this system is known as a Bain circuit
c. When it is used for spontaneous ventilation, respiratory rate is a key determinant of
rebreathing likelihood

Downloaded from https://www.cambridge.org/core. University of Edinburgh, on 19 Aug 2019 at 13:21:27, subject to the Cambridge Core terms of
use, available at https://www.cambridge.org/core/terms. https://doi.org/10.1017/9781108566100.009
Chapter 4a: Physics Questions 283

d. It is the most efficient of the Mapleson classified circuits for spontaneous ventilation
e. Large tidal volumes may result in the presence of CO2 in the reservoir bag

Question 85
The magnets in use in modern magnetic resonance imaging (MRI) units:
a. Have a variable magnetic field that can be changed, depending on scan type, between 0.2
and 4.0 Tesla
b. Generate a magnetic field which causes neutrons in the body to align in the direction of
the field
c. May be either permanent magnets or electromagnets
d. Use superconducting magnets that have their coils immersed in liquid oxygen in order
to keep them below 4.2 kelvin
e. Produce a magnetic field whereby the field strength falls away linearly with distance from
the scanner

Question 86
Regarding magnetic resonance imaging (MRI):
a. With the application of a pulsed external magnetic field in a direction parallel to the
main field, nuclei precess and produce a rotating magnetic field
b. The largest magnetic resonance signal is produced by nitrogen due to the element’s high
natural abundance in the body
c. Phosphorus generates a weak magnetic resonance signal in the body so stronger magnets
are needed to produce a better quality image
d. It does not involve ionizing radiation
e. A conductive copper mesh enclosing the entire scan room prevents interference from
external electromagnetic sources

Question 87
Regarding X-rays and radiation:
a. A becquerel (Bq) is one nuclear disintegration per second
b. X-rays are produced when an electron beam leaving the anode is accelerated to strike the
cathode, often made of tungsten
c. An X-ray beam passing through a patient’s body is exponentially attenuated
d. In modern plain X-rays, photographic film is used to capture images that are then
digitized for viewing on computer screens
e. Chromium-51 is the radioactive isotope most commonly used in imaging techniques
because it can be easily attached to chemical compounds suitable for injecting into
patients

Question 88
Regarding computerized tomography (CT) imaging:
a. It generally requires higher doses of radiation than standard X-ray imaging of the same
area

Downloaded from https://www.cambridge.org/core. University of Edinburgh, on 19 Aug 2019 at 13:21:27, subject to the Cambridge Core terms of
use, available at https://www.cambridge.org/core/terms. https://doi.org/10.1017/9781108566100.009
284 Chapter 4a: Physics Questions

b. It can sometimes use ‘real-time’ image display


c. It records an image for ‘slices’ in the body by rotating an emitter–detector set around the
body
d. Slice thickness and spacing can be varied according to clinical requests and information
required – typically 8–10 mm for thoracic or abdominal scans, and 2–5 mm for the skull,
orbits and spine
e. Involving intravenous contrast requires a rapid flow rate of 2–6 ml.s–1 and therefore a
cannula of at least 1 mm diameter (20G) should be used

Question 89
Concerning positron emission tomography (PET)
a. It is a complex form of three-dimensional nuclear imaging that gives dynamic infor-
mation on organ function
b. The PET process involves single γ-rays of 511 keV
c. PET uses radioisotopes with short half-lives that can be incorporated into compounds
(e.g. glucose) normally used by the body
d. The most commonly used radionuclide is fluorine-18 in the form of fluorodeoxyglu-
cose (FDG)
e. The principle of PET involves detection of pairs of γ-photons emitted from the same
location in the body, but having travelled in opposite directions

Question 90
Regarding work:
a. One joule of work is done when a force of one newton moves its point of application one
metre in the direction of the force
b. When a force is constant, the work done can be defined as the product of force and the
area over which that force is acting
c. Mechanical work is a form of energy
d. The law of conservation of energy states that the change in total energy of an isolated
system is equal to the heat supplied to the system minus the amount of work performed
by the system
e. In the body, mechanical work performed is calculated by the shortening of the muscle
multiplied by the mean force exerted

Question 91
Regarding power:
a. Power is the product of force and pressure
b. During breathing, power depends on the type of airflow in the airways, as pressure
gradients are greater when flow is laminar rather than turbulent
c. The term ‘horsepower’ can be used to describe the power delivered by a machine
d. Power is the rate at which work is done and is measured in watts
e. The watt (W) is an SI base unit

Downloaded from https://www.cambridge.org/core. University of Edinburgh, on 19 Aug 2019 at 13:21:27, subject to the Cambridge Core terms of
use, available at https://www.cambridge.org/core/terms. https://doi.org/10.1017/9781108566100.009
Chapter 4a: Physics Questions 285

Question 92
Regarding energy:
a. Energy can be neither created nor destroyed, only converted from one form to another
b. The SI base unit of energy is the joule
c. During inspiration, a proportion of mechanical energy is converted to and stored as heat
energy, which is subsequently used for the work of expiration
d. Expiration requires energy to overcome friction and airways resistance
e. The difference between the inspiratory and expiratory pathways (hysteresis) generated
during breathing represents heat dissipation

Question 93
Which of the following statements regarding energy are true?
a. Kinetic energy is equal to the mass of an object multiplied by the square of the speed of
light
b. Electrical energy is related to the storage of electrical charge
c. Thermal energy will always flow from a substance containing a larger amount of energy
to a substance with a lesser amount
d. Elastic energy is the potential energy stored in the chemical bonds of a stretchable
material
e. Sound energy can be transferred by mechanical vibration of air molecules in a wave-like
manner

Question 94
Which of the following statements regarding work and power are true?
a. The SI unit of work done over a period time is the watt
b. Expending 3000 J in 0.5 minutes equates to a power of 100 W.s–1
c. The respiratory muscles in healthy spontaneously breathing adults are approximately
80% efficient; 20% of the energy utilized is spent as heat
d. A patient has a mean arterial blood pressure of 90 mmHg, a pulmonary venous pressure
of 0 mmHg, and a cardiac output of 5 l.min–1. The power exerted by a theoretically 100%
efficient left ventricle would be 45 W
e. Theoretically, the work done by an 80 kg man carrying two 10 kg bags over 10 m on a flat
surface with frictionless shoes is 1 kJ

Question 95
Regarding heat capacity, latent heat and temperature:
a. The difference between specific heat capacity and heat capacity relates to the mass of
substance involved
b. Heat is a form of kinetic energy that can be transferred from a colder substance to a
hotter substance
c. Temperature is the property of a substance that determines whether an object will
receive heat or give heat to another object

Downloaded from https://www.cambridge.org/core. University of Edinburgh, on 19 Aug 2019 at 13:21:27, subject to the Cambridge Core terms of
use, available at https://www.cambridge.org/core/terms. https://doi.org/10.1017/9781108566100.009
286 Chapter 4a: Physics Questions

d. An example of latent heat of fusion is the heat associated with a solid dissolving into a
liquid at a constant temperature
e. A clinical example of latent heat is in the use of ethyl chloride spray for topical
anaesthesia

Question 96
Regarding temperature and heat capacity:
a. Heat capacity is the amount of heat required to raise the temperature of an object by one
degree Celsius
b. The specific heat capacity of blood is 3.6 kJ.K–1
c. The specific heat capacity of water is 1 kJ.K–1
d. The SI unit of temperature is the kelvin, defined as the thermodynamic temperature of
the triple point of water
e. In a body there is a temperature gradient between the core and the shell. The difference
between the two temperatures is known as the Sharma gap

Question 97
Regarding laser safety:
a. Specific health and safety legislation exists in the UK that covers the use of lasers
b. Spectacle wearers are not protected from the laser beam
c. A high oxygen concentration should be used in the anaesthetic gas mixture
d. Only a laser protection supervisor can use a laser in an operating theatre
e. Reflected laser beams are low energy and not dangerous to theatre staff

Question 98
Regarding lasers:
a. Laser is an acronym for light amplification by stimulated emission of radiation
b. Helium–neon lasers generate light in the ultraviolet part of the spectrum
c. An argon laser emits green light
d. If an airway fire occurs the surgeon should flood the field with saline
e. The beam produced is an incoherent beam of light

Question 99
Laser light is:
a. Monochromatic
b. Divergent from the point of origin
c. Composed of photons
d. Coherent
e. Collimated

Downloaded from https://www.cambridge.org/core. University of Edinburgh, on 19 Aug 2019 at 13:21:27, subject to the Cambridge Core terms of
use, available at https://www.cambridge.org/core/terms. https://doi.org/10.1017/9781108566100.009
Chapter 4a: Physics Questions 287

Question 100
Absolute humidity:
a. Decreases as a gas is heated
b. Will vary with the temperature of the air
c. Is 34 g.m–3 in the alveoli, when fully saturated at 37 °C
d. Is the mass of water vapour present in a given volume of air
e. Has the units grams per cubic metre (g.m–3) or kilograms per cubic metre (kg.m–3)

Question 101
Regarding humidifiers:
a. Water overload is a danger if using ultrasonic nebulizers
b. Ideal droplet size in a nebulizer humidifier is between 1 μm and 5 μm
c. A heat and moisture exchanger is more efficient than a heated water bath
d. The Bernoulli nebulizer is less efficient than a heat and moisture exchanger
e. Heat and moisture exchangers are active humidifiers

Question 102
Relative humidity:
a. Is a measure of the amount of water vapour present in air
b. Varies with temperature
c. Is measured with a Regnault’s hygrometer
d. Is expressed as a percentage
e. Is decreased when heating a gas

Question 103
Paramagnetic analyzers:
a. Can measure oxygen concentration in a mixture of gases
b. Only measure gases that are diamagnetic
c. May be used to measure gas concentration in a liquid
d. Use the null hypothesis method
e. Can be used to measure nitrous oxide levels

Question 104
Gas chromatography can be used for measuring concentrations of:
a. Sevoflurane
b. Barbiturates
c. Benzodiazepines
d. Steroids
e. Catecholamines

Downloaded from https://www.cambridge.org/core. University of Edinburgh, on 19 Aug 2019 at 13:21:27, subject to the Cambridge Core terms of
use, available at https://www.cambridge.org/core/terms. https://doi.org/10.1017/9781108566100.009
288 Chapter 4a: Physics Questions

Question 105
Infrared spectrophotometry:
a. Can be used to measure carbon dioxide
b. Has a short response time of <100 ms
c. Cannot be used for end tidal analysis
d. Can, in principle, detect any molecule composed of two or more dissimilar atoms
e. Can detect oxygen

Question 106
Mass spectrometry:
a. Does not alter the molecular structure of the analyte
b. Cannot be used to provide real-time results
c. Equipment requires safety shielding because radioactive material is used to ionize
samples
d. Deflects charged ions using a magnetic field
e. Separates the components of gas mixtures according to their mass and charge

Question 107
When measuring gas concentrations:
a. Infrared spectrophotometry can be used to measure oxygen concentration
b. The piezoelectric technique can be used to distinguish between volatile anaesthetic
agents
c. The Van Slyke apparatus can measure carbon dioxide concentrations in blood samples
d. The Clark electrode can be used to measure oxygen concentration in a mixture of gases
e. Paramagnetic analysis can measure nitrogen concentrations

Question 108
The Raman effect:
a. Relies on the change in velocity of light shone through a gas mixture due to Raman
scattering
b. Uses an argon laser
c. Can be used to measure concentration of sevoflurane
d. Can be used to measure concentration of helium
e. Uses light of a wavelength 720 nm

Question 109
A mass spectrometer:
a. Is able to distinguish between carbon dioxide and nitrous oxide
b. Involves passing charged particles through a quadrupole
c. Has a fast enough response time to allow breath by breath analysis
d. Allows sampled gases to be injected back into the circuit after analysis
e. Produces particles that are positively charged

Downloaded from https://www.cambridge.org/core. University of Edinburgh, on 19 Aug 2019 at 13:21:27, subject to the Cambridge Core terms of
use, available at https://www.cambridge.org/core/terms. https://doi.org/10.1017/9781108566100.009
Chapter 4a: Physics Questions 289

Question 110
When analyzing a mixture of anaesthetic gases to determine individual concentrations:
a. Photoacoustic measurement uses two quartz electrodes, one coated in oil to absorb
halogenated vapours and the other oil-free to act as a reference electrode
b. The piezoelectric effect uses sound measured at different wavelengths
c. The Rayleigh refractometer is the gold standard for calibrating instruments delivering
anaesthetic vapours
d. Refractometry uses the fact that light travels slower in a vacuum and the degree of
deceleration depends on the concentration of matter in the path
e. Mass spectrometry distinguishes carbon dioxide from nitrous oxide by fragmentation
products produced

Question 111
The Clark electrode has:
a. A silver/silver chloride anode
b. A platinum cathode
c. An electrolyte solution containing sodium hydroxide
d. An anode where oxygen combines with electrons and water to give hydroxyl ions
e. An anode where hydroxyl ions are produced

Question 112
Regarding transcutaneous oxygen measurement:
a. It is highly accurate
b. It has a slow response time
c. The skin locally is heated to 43 °C
d. Uses an atypical polarographic electrode
e. Measures capillary oxygen tensions

Question 113
Regarding oxygen concentration analysis:
a. Continuous intra-arterial oxygen is best measured using principles of the paramagnetic
property of oxygen
b. Paramagnetic analysis is often used in modern anaesthetic machines
c. The Raman effect is ineffective
d. Infrared absorption spectroscopy is only used in approximately 50% of modern
machines
e. Mass spectrometry cannot be used, as oxygen does not contain two dissimilar atoms

Question 114
A pulmonary artery catheter (PAC) can be inserted for use during cardiac output measure-
ment. Regarding a pulmonary artery catheter:
a. Standard adult lines are 50 cm in length

Downloaded from https://www.cambridge.org/core. University of Edinburgh, on 19 Aug 2019 at 13:21:27, subject to the Cambridge Core terms of
use, available at https://www.cambridge.org/core/terms. https://doi.org/10.1017/9781108566100.009
290 Chapter 4a: Physics Questions

b. A thermistor lies at the distal tip of the catheter


c. 1.5 ml of air is required to inflate the balloon tip
d. Newer PACs do not require the injection of cold saline
e. The proximal lumen point may lie outside the heart

Question 115
Fuel cells used for oxygen concentration measurement:
a. Have a lead anode
b. Have a platinum cathode
c. Have a cathode that uses the same reaction as the Clark electrode
d. Require a thermistor for temperature compensation
e. Can be used to measure oxygen in gas mixtures

Question 116
Regarding pulse oximeters:
a. They are reliably calibrated down to 50% saturation only
b. They tend to give falsely low readings in patients with carbon monoxide toxicity
c. They are useful as they provide an indication of the adequacy of a patient’s minute
ventilation
d. Deoxyhaemoglobin absorbs more red light than oxyhaemoglobin
e. They cannot be used at altitudes >3500 m

Question 117
Regarding pulse oximetry:
a. It acts as a pulsed monitor of ventilatory function
b. It can be used as a substitute for arterial blood gas analysis
c. It is used in many modified early warning score charts
d. The pulse waveform gives valuable extra information in addition to the oxygen
saturation
e. It can become unreliable if the patient is hypovolaemic

Question 118
Pulse oximetry:
a. Relies on the Beer–Lambert law
b. Uses a diode that flashes 50 times a second
c. Traditionally use LEDs emitting light of two wavelengths, which are 660 nm and 940 nm
d. Requires two photodetectors, and allows compensation for ambient light
e. Uses the difference in light absorption of oxyhaemoglobin and carboxyhaemoglobin

Question 119
Bunsen’s solubility coefficient:
a. Does not need to be corrected to standard temperature and pressure (STP)

Downloaded from https://www.cambridge.org/core. University of Edinburgh, on 19 Aug 2019 at 13:21:27, subject to the Cambridge Core terms of
use, available at https://www.cambridge.org/core/terms. https://doi.org/10.1017/9781108566100.009
Chapter 4a: Physics Questions 291

b. Is the volume of gas that can be dissolved in a unit volume of liquid


c. Requires the partial pressure of the gas above the liquid to be at one standard atmo-
spheric pressure
d. Changes with temperature
e. Is the volume of liquid that dissolves in one unit volume of gas at the temperature
concerned

Question 120
Regarding colligative properties of a solution:
a. They depend on the mass or size of the particles and not the number of molecules present
b. The boiling point of a solution is raised if more solute is added
c. The saturated vapour pressure of a solution is lowered if more solute is added
d. The freezing point of a solvent is lowered if more solute is added
e. As more solute is added the osmotic pressure exerted by the solution is increased

Question 121
Fick’s law of diffusion outlines the variables that affect the diffusion of gas across a
membrane. Fick’s law of diffusion states that the rate of gas diffusion is:
a. Directly proportional to the surface area
b. Inversely proportional to the membrane thickness
c. Directly proportional to the concentration gradient
d. Inversely proportional to the square root of its density
e. Inversely proportional to the diffusion constant

Question 122
The movement of water across a membrane from an area of low solute concentration to an
area of high solute concentration is called osmosis. Which of the following statements is
true?
a. When dissolved in 22.4 l of solvent, one osmole exerts an osmotic pressure of approx.
101 kPa (1 atm).
b. The number of osmotically active particles present per litre of solution (mmol.l–1) is
described as osmolarity
c. Osmolality is the number of osmotically active particles present per kg of solvent
(mmol.kg–1)
d. Osmotic pressure is the pressure exerted within a sealed system of solution in response
to the presence of osmotically active particles on one side of a semi-permeable
membrane
e. One osmole is the osmotic pressure of an amount of solute which, when dissolved, forms
a mole of particles: 6.023 × 1023

Question 123
Which of the following are correct regarding the physical processes involved when a
substance is kept in a closed system or sealed environment?

Downloaded from https://www.cambridge.org/core. University of Edinburgh, on 19 Aug 2019 at 13:21:27, subject to the Cambridge Core terms of
use, available at https://www.cambridge.org/core/terms. https://doi.org/10.1017/9781108566100.009
292 Chapter 4a: Physics Questions

a. The substance in its gaseous state and above its critical temperature is classed as a vapour
b. Kinetic energy of molecules increases with temperature
c. Saturated vapour pressure (SVP) is not affected by changes in temperature
d. SVP is independent of ambient pressure
e. Heat energy is absorbed from the environment during the transition from the liquid to
the gaseous phase

Question 124
Temperature and pressure are important variables in determining the physical state in
which a substance exists. Which of the following are correct?
a. Above its critical pressure a substance is in its gaseous form
b. If a substance in its vapour phase is compressed sufficiently, it will always revert back to
its liquid form
c. A gas is defined as a substance at its critical temperature
d. Each liquid at its critical temperature has its own saturated vapour pressure, and this is
defined as critical pressure
e. At room temperature, the physical state of oxygen within a cylinder is constant,
regardless of pressure

Question 125
The physical properties of desflurane make use in a conventional plenum or temperature-
compensated vaporizer unsafe. Which of the following are true about desflurane?
a. Desflurane is close to its boiling point at room temperature
b. When placed in a plenum vaporizer within normal operational conditions, the SVP of
desflurane is stable
c. Desflurane vapour is directly injected into the vapour chamber
d. Desflurane is actively heated and pressurized to 29 °C and 3 atm
e. The amount of desflurane injected during use of the vaporizer is independent of fresh
gas flow

Question 126
The ability to provide an accurate concentration of volatile agent in the anaesthetic breath-
ing system is a key feature of modern plenum or temperature-compensated vaporizers.
Consider the following statements regarding how this is achieved.
a. Heat sinks of copper can be utilized to reduce the heat capacity of the vaporizer
b. Bimetallic strips are often incorporated, reducing the amount of gas entering the vapour
chamber as the temperature drops
c. Plenum vaporizers are designed to have a low heat capacity
d. A series of wicks and baffles can help to keep a constant rate of vaporization within the
vaporization chamber
e. Each vaporizer is designed to be used with multiple types of anaesthetic volatile agent

Downloaded from https://www.cambridge.org/core. University of Edinburgh, on 19 Aug 2019 at 13:21:27, subject to the Cambridge Core terms of
use, available at https://www.cambridge.org/core/terms. https://doi.org/10.1017/9781108566100.009
Chapter 4a: Physics Questions 293

Question 127
Consider the impact of increasing altitude and subsequent lower barometric pressure on the
conduct of general anaesthesia using volatile agents, relative to sea level:
a. Percentage output of isoflurane vapour is decreased when using a plenum vaporizer
b. Modern, temperature-compensated plenum vaporizers overcompensate for increases in
altitude
c. Partial pressure of anaesthetic vapour is the underlying physiological principal govern-
ing maintenance of anaesthesia
d. Dalton’s law of partial pressure can only be applied at sea level
e. Desflurane vaporizers require a higher dialled value for the same clinical effect

Question 128
Although some isotopes used in medical practice occur naturally, many are produced
artificially by a nuclear reactor, or cyclotron. Which of the following statements is true?
a. Carbon-14 contains eight protons and six neutrons in each carbon nucleus
b. Isotopes are variants of particular chemical elements which differ in proton number
c. Each isotope of a substance can be either stable or radioactive
d. A ‘daughter nuclide’ is produced following decay of an isotope
e. γ-Radiation is not part of the electromagnetic spectrum

Question 129
Anaesthetists encounter ionizing radiation in many aspects of their daily practice. Consider
the following, relating to its safe use:
a. The use of radiation is governed by the Ionising Radiation Regulation 1999 and Ionising
Radiation (Medical Exposure) Regulations 2000 Acts
b. Radiation dosage can be quantified using both gray (Gy) and sievert (Sv)
c. A sievert quantifies the biological effect of low-dose ionizing radiation
d. Biological damage is inversely related to dose and exposure
e. UK average annual background radiation dose is 0.2 mSv

Question 130
Regarding critical temperature:
a. Each gas has its own specific critical temperature
b. The critical temperature of carbon dioxide is 36.5 °C
c. The critical temperature of nitrous oxide in a cylinder at 52 bar is approximately half that
of the same gas at pipeline pressure
d. The pseudocritical temperature of Entonox is –30 °C ®
e. A hydrogen thermometer is used in international temperature scale measurements
because hydrogen gas closely resembles the ideal gas

Question 131
The gas laws describe the relationship between pressure, volume and temperature. Which of
the following statements regarding the gas laws are true?

Downloaded from https://www.cambridge.org/core. University of Edinburgh, on 19 Aug 2019 at 13:21:27, subject to the Cambridge Core terms of
use, available at https://www.cambridge.org/core/terms. https://doi.org/10.1017/9781108566100.009
294 Chapter 4a: Physics Questions

a. According to Boyle’s Law, halving the volume (V) of a container will halve the absolute
pressure (P) of the same mass of gas contained in that container
b. At any one particular temperature, if an equilibrium exists between the rate of molecules
transferring between a liquid and its vapour, the vapour above the liquid is said to be at
its saturated vapour pressure (SVP)
c. The ideal gas law is a combination of Boyles, Charles’, Gay-Lussac’s and Avogadro’s laws
and states that PV = nRT
d. An adiabatic change occurs when the state of a system is altered by exchanging heat with
its surroundings
e. Isotherms can be used to describe the relationship between temperature and volume for
a substance at different pressures

Question 132
Transducers are used in pressure measurement. Which of the following are true regarding
transducers?
a. The purpose of a transducer is to convert one form of energy into heat energy
b. An invasive pressure recording uses a strain gauge variable-resistance transducer
c. With changes in arterial pressure the movement of the saline column of an arterial line
results in a change in resistance and current flow through the transducer
d. The resonance and damping in the measuring system have no effect on the mean
pressure measurement
e. The transduced signal can be displayed as a waveform

Question 133
Regarding fluid flow and pressure:
a. In a Venturi device, a marked fall in pressure occurs across a constriction in a tube,
where the cross-section gradually reduces and then increases
b. Laplace’s law states that the pressure difference across the wall of a tube is equal to the
tension divided by the radius
c. According to Laplace’s law the pressure difference across the wall of a sphere is equal to
the square of the tension divided by the radius
d. In turbulent flow, the flow through a tube is proportional to the pressure difference
between the ends of a tube
e. In laminar flow, the resistance of a tube is a constant and can be defined as the ratio of
pressure to flow

Question 134
Regarding pressure in cylinders and the inter-relationship between different pressure
measurements:
a. Gauge pressure is the difference between absolute and atmospheric pressure
b. Bourdon gauges contain liquid within the coiled tubing
c. A full oxygen cylinder has an absolute pressure of 137 bar
d. Arterial and venous blood pressure readings are gauge pressures
e. ®
An Entonox valve is an example of a two-stage demand valve

Downloaded from https://www.cambridge.org/core. University of Edinburgh, on 19 Aug 2019 at 13:21:27, subject to the Cambridge Core terms of
use, available at https://www.cambridge.org/core/terms. https://doi.org/10.1017/9781108566100.009
Chapter 4a: Physics Questions 295

Question 135
Regarding ECG and EEG potentials:
a. Fourier analysis is the process of analyzing complex signal wave patterns and converting
them to simpler sine wave patterns
b. The size of the ECG signal detected is about 90 mV
c. The larger the cardiac muscle mass that waves of depolarization pass through, the larger
the potential detected at the skin surface
d. EEG potentials are much larger than ECG potentials
e. In an EEG the frequency of slow (delta) waves is about 1 Hz

Question 136
Regarding detection and display of biological signals:
a. Commonly available ECG electrodes consist of a plastic film with a thin silver electrode
in contact with silver chloride on its surface
b. Polarization occurs when the electrode potential is altered, and is a problem of the skin–
electrode junction
c. Cathode ray tubes (CRTs) can be used to display biological signals
d. Galvanometers are well suited to the display of EMG signals
e. Differential signal amplifiers eliminate interference that is common to both input
terminals

Question 137
Regarding EEG potentials, EMG potentials and nerve stimulators used in assessing neuro-
muscular block:
a. The range of frequencies in an EEG is about 0.5–100 Hz
b. The EMG gives a pattern of sharp spikes rather than the pattern seen in an ECG because
skeletal muscle potentials have a much shorter duration than cardiac potentials
c. Apparatus used in delivering an electrical stimulus, for example in nerve stimulators,
produces a square-wave electrical pulse
d. The positive electrode of a nerve stimulator should be placed over the nerve and the
negative electrode should be positioned a few centimetres proximal to this
e. Both the nerve stimulator and transcutaneous electrical nerve stimulation (TENS)
machine use currents of up to 60 mA

Question 138
Regarding the use of gas chromatography:
a. It is a common technique used in most UK hospitals for gas and vapour analysis
b. The sample of substance to be analyzed is usually injected slowly into the gas
chromatograph
c. Gas chromatography cannot analyze very low concentrations of substances
d. Gas chromatography allows continuous analysis of substances
e. Clinically it can be used to measure concentrations of volatile anaesthetic agents in the
theatre environment

Downloaded from https://www.cambridge.org/core. University of Edinburgh, on 19 Aug 2019 at 13:21:27, subject to the Cambridge Core terms of
use, available at https://www.cambridge.org/core/terms. https://doi.org/10.1017/9781108566100.009
296 Chapter 4a: Physics Questions

Question 139
Regarding the principles behind the gas chromatograph:
a. The speed at which the gas mixture to be analyzed passes through the column depends
on the solubility of the components within the two phases
b. Within reason, a longer length of column means that there will be better separation of
individual components of the gas mixture
c. The column is heated along its length to increase solubility of the individual components
d. Oxygen can be used as the carrier gas in the mobile phase
e. The retention time is the time between the initial injection of a gas sample and its
appearance at the detector

Question 140
The pin-index system is used as a safety feature on gas cylinders. Which of the following pin-
index positions are correct?
a. Oxygen 2 and 7
b. Oxygen 2 and 6
c. Carbon dioxide 1 and 6
d. Entonox 7
e. Air 1 and 5

Question 141
Nitrous oxide is commonly stored on an anaesthetic machine in cylinders. Regarding
nitrous oxide cylinders, which of the following are true?
a. Cylinders have a blue body with blue and white quartered shoulders
b. Gas in cylinders obeys Boyle’s law
c. Nitrous oxide is stored in both the liquid and vapour phases
d. Cylinder pressure at room temperature is 4400 kPa
e. The cylinder pressure gauge always reflects the contents of the cylinder

Question 142
®
Entonox can be stored in cylinders. Regarding the storage of Entonox in a cylinder: ®
a. ®
Entonox is stored at a pressure of 137 000 Pa
b. Cylinders have a blue body with blue and white quartered shoulders
c. There is a risk of the Poynting effect below –5.5 °C
d. There may be a risk of delivering a hypoxic mixture below –5.5 °C
e. Cylinders should not be agitated before use if they have been stored for more than
2 hours

Question 143
Pipelines are used as one means of providing a gas supply to anaesthetic machines.
Regarding the piped gas supply to an anaesthetic machine:
a. Piped gases are supplied at 400 kPa

Downloaded from https://www.cambridge.org/core. University of Edinburgh, on 19 Aug 2019 at 13:21:27, subject to the Cambridge Core terms of
use, available at https://www.cambridge.org/core/terms. https://doi.org/10.1017/9781108566100.009
Chapter 4a: Physics Questions 297

b. Pipes are colour coded


c. Pipes are connected to the anaesthetic machine via a non-interchangeable screw thread
d. Pipes are connected to wall outlets via Schrader sockets
e. Pipes are made of copper alloy

Question 144
A vacuum-insulated evaporator is used for storage of oxygen. Which of the following
concerning a vacuum-insulated evaporator are true?
a. It consists of a double-walled copper alloy tank
b. Perlite is used in the vacuum between the double walls of the tank
c. Oxygen is stored as a liquid
d. Oxygen is stored above its critical temperature
e. Oxygen enters the pipeline supply at 7 bar

Question 145
The Doppler effect is a physical principle originally described by Johann Christian Doppler.
Which of the following statements about the Doppler effect are true?
a. Because of the Doppler effect, as the source of a sound approaches a subject the
wavelength becomes shorter and there will be a decrease in frequency of the sound
b. Collagen exhibits piezoelectric properties
c. When using colour Doppler to study vessels red indicates arterial blood and blue
indicates venous blood
d. Doppler frequency shift is proportional to velocity of flow
e. The Doppler equation relates the transmitted Doppler frequency, the speed of blood
flow, the cosine of the blood flow to beam angle and the speed of sound in air

Question 146
Sound and its properties have many uses in medical imaging. Regarding sound, which of the
following statements are true?
a. Sound can be described as a pressure wave
b. When sound waves reach the interface between air and skin the full signal is transmitted
through the skin into the body
c. 1 Hz is one cycle per second
d. The speed of sound in gases is temperature-independent
e. Some sound energy is reflected every time a wave changes media

Question 147
Ultrasound is a non-invasive imaging technique commonly used in many aspects of medical
practice. Which of the following statements regarding ultrasound are true?
a. The human ear cannot detect ultrasound
b. When ultrasound passes through body tissues, some of the sound energy is absorbed by
the tissues
c. The use of gel on an ultrasound probe helps to increase attenuation of the signal

Downloaded from https://www.cambridge.org/core. University of Edinburgh, on 19 Aug 2019 at 13:21:27, subject to the Cambridge Core terms of
use, available at https://www.cambridge.org/core/terms. https://doi.org/10.1017/9781108566100.009
298 Chapter 4a: Physics Questions

d. Tissue cavitation is a potential complication of the use of ultrasound


e. Piezoelectric crystals in the ultrasound probe head act as a transducer

Question 148
Ultrasound imaging uses the transmission and reflection of ultrasonic sound waves in
tissues. Which of the following statements are correct?
a. The nominal frequency range of human hearing is from around 20 to around
20 000 KHz
b. Ultrasound imaging is based on the ‘pulse-echo’ Doppler system
c. Better resolution of an ultrasound image can be obtained using higher-frequency
ultrasound
d. The frequency of medical ultrasound is 2.5 to 15 MHz
e. Deeper structures such as the liver are imaged at a lower frequency, because this allows
greater penetration of the ultrasound energy

Question 149
Ultrasound and the Doppler effect have many uses in anaesthesia, as well as medicine and
surgery. Which of the following statements concerning their use are correct?
a. In ultrasound imaging, bone is represented by a collection of black dots and fluid by
white dots
b. 2-Dimensional imaging ultrasound is recommended by NICE to aid central venous
cannulation
c. Oesophageal Doppler allows measurement of blood flow in the ascending thoracic aorta
d. The depth of a structure equals the velocity of the ultrasound wave divided by the time
between emission and detection of said wave
e. The velocity of sound in tissue is assumed constant at 540 m.s–1

Question 150
Measurement of cardiac output can be achieved using an oesophageal Doppler probe. An
oesophageal Doppler probe:
a. Has a diameter of 6 mm
b. Is best placed in the upper oesophagus, at the level of T5–T6
c. Cannot be safely used in the presence of diathermy
d. Measures blood flow in the descending aorta
e. Uses ultrasound waves

Downloaded from https://www.cambridge.org/core. University of Edinburgh, on 19 Aug 2019 at 13:21:27, subject to the Cambridge Core terms of
use, available at https://www.cambridge.org/core/terms. https://doi.org/10.1017/9781108566100.009
Chapter
Physics Answers

4b
Question 1: FTFFT
SI stands for Système Internationale (d’Unités). There are seven base units (from which all
other units can be derived).
A neat way of remembering this mnemonic is ‘SM2ACK2’:
Second (s)
Metre (m)
Mole (mol)
Ampère (A)
Candela (cd) – a measure of luminous intensity
Kilogram (kg)
Kelvin (K) (note – not degrees)
They make up the fundamental buildings blocks of all other units of measurement. For
example, the unit of frequency is the hertz; but hertz it is not a base unit. It is a derived unit
and is defined as 1 s–1. Another example is that of force. Statement e gives the correct
derivation of the newton. Other derived units include pressure, power, volume, electrical
potential, energy, charge and resistance. This list is not exhaustive.

Question 2: FTFFT
Beware of subtle changes in the wording. There are some trick questions here, testing your
knowledge. The mole is the number of atoms of carbon-12, not 14 in 12 g (0.012 kg).
The metre was previously defined as the distance occupied by 1 650 763.73 wavelengths of
the standard radiation of krypton-86, but now this subdivision of the speed of light is used
(since 1983). Statement c gives the definition of a joule. 1 watt is 1 joule per second
(N.m.s−2). The definition of 1 pascal is the pressure of 1 newton per square metre.

Question 3: TFTTF
1 Atmosphere (atm) = 1.013 bar
= 101.3 kPa
= 760 mmHg
= 760 Torr
= 1033.227 cmH2O
= 14.7 PSI

299
Downloaded from https://www.cambridge.org/core. University of Edinburgh, on 19 Aug 2019 at 13:21:26, subject to the Cambridge Core terms of
use, available at https://www.cambridge.org/core/terms. https://doi.org/10.1017/9781108566100.010
300 Chapter 4b: Physics Answers

This simple MCQ is a primary exam favourite. It tests understanding of the many different
units of pressure measurement. Unfortunately, in clinical practice we still haven’t got round
to standardizing our different gauges.

Question 4: TTFFF
This question is worded to ensure candidates truly understand the concept of the
kelvin. The first two statements are the correct definitions. The triple point occurs at
273.16 K which is 0.01 °C ; °C is a derived unit. The freezing point of water, which is
0 °C, is 273.15 K.
Heat capacity is the amount of heat (a form of energy, measured in J) required to raise the
temperature of an object by 1 K, so its unit would be J.K–1. (Specific heat capacity is the
amount of energy required to raise the temperature of 1 kg of a specific object by 1 K.)

Question 5: TFFTT
There are many techniques used for measurement of temperature. These are indeed four of
them. The bimetallic strip is arranged in a coil, and as temperature changes, the degree of
uncoiling is shown by a pointer. The description in statement b is the basis for the
thermocouple. In a platinum resistance wire, resistance increases proportionally with
temperature, not exponentially. The thermistor is a small semiconductor bead and resis-
tance does decrease exponentially. The Seebeck effect is described in statement b.

Question 6: FFTFF
Body temperature is regulated to around 37 °C. The standard deviation is about 0.2 °C,
although there is variability throughout the day of up to 0.7 °C. Brown fat thermogenesis is
a feature of the neonate, not the child. The specific percentages of heat loss from different
modalities are variable. However, anaesthetists should know which modalities are more
problematic. Roughly speaking, radiation does account for about 40–50% of heat loss,
respiration (along with evaporation from the skin) accounts for about 20% of the loss,
and convection can account for up to 30–40% of heat loss. Conduction is relatively
insignificant in air, but increases markedly in water.

Question 7: TTFTF
Latent heat is the energy required to change the state of a substance without changing its
temperature. Somewhat confusingly, the reaction can be in either direction (liquid to
vapour or vapour to liquid) so statement a is true. Latent heat falls as temperature rises
and is zero at the critical temperature, as no further heat is required to convert the liquid to
a gas, as it cannot exist in the liquid phase at that temperature. Nitrous oxide cylinders
contain liquid nitrous oxide with vapour above. As nitrous oxide vapour is removed from
a cylinder, heat will be required to vaporize more of the remaining liquid nitrous oxide. This
is taken from the surrounding fluid and cylinder walls, making them cold (adiabatic cool-
ing). Water vapour in the surrounding air may condense or even freeze on the cold walls of
the cylinder. The drop in temperature of the remaining liquid will reduce the resultant
vapour pressure (as the saturated vapour pressure is also related to temperature) and so the
gauge will under-read until the cylinder is switched off and thermal equilibrium is restored.

Downloaded from https://www.cambridge.org/core. University of Edinburgh, on 19 Aug 2019 at 13:21:26, subject to the Cambridge Core terms of
use, available at https://www.cambridge.org/core/terms. https://doi.org/10.1017/9781108566100.010
Chapter 4b: Physics Answers 301

The critical temperature of nitrous oxide is +36.5 oC. This means it is inhaled as a vapour
(below its critical temperature) but exhaled as a gas (above its critical temperature),
a popular viva question!

Question 8: FTTTT
Boyle’s law states that volume is inversely proportional to absolute pressure, not gauge
pressure. Statement c gives the third gas law, Gay-Lussac’s law. However, Gay-Lussac has
been attributed to both the second (Charles’) and third laws. Strictly speaking, the third law
was first described by Amontons 100 years before Gay-Lussac. Standard temperature and
pressure (STP) are described as 273.15 K (note not ‘degrees’ K) and 101.325 kPa. These
equate to 0 oC and 760 mmHg, but the correct SI units should ideally be used.

Question 9: TFTTT
The molecules of an ideal gas must be identical and not interact with each other (so no forces
exist between them). However, there is no true ideal gas in existence. All molecules are
subject to the Newtonian laws of mechanics. Nitrogen, oxygen, hydrogen, noble gases and
carbon dioxide are generally regarded to behave like ideal gases within accepted tolerances.
The movement of molecules within an ideal gas is at random.

Question 10: TFTTT


Statement a is Avogadro’s hypothesis. Statement b should be 6.022 × 1023. The critical
temperature is the highest temperature at which a gas can still be liquefied by pressure alone.
This is the critical pressure. Questions originating from definitions of critical temperature
and pressure are common. At the critical temperature, a gas can be in a gaseous form or
a vapour. Above this temperature, it can only exist as a gas.

Question 11: TTFTT


Regression analysis determines the magnitude of change of one variable produced by the
other variable. It requires the formulation of a mathematical model to explain the data and
results in an equation in the form
y = mx + c
y being the dependent variable, x the independent variable, m being the gradient of the line
demonstrating the regression coefficient, and c being the intercept which defines the
position of the line. Whilst the correlation coefficient is an indicator of the ‘goodness of
fit’ this is yielded from correlation analysis, not linear regression analysis. It utilizes the least
squares method to find the regression line that has the most explanatory power and least
error. Although the data do not need to be normally distributed to perform linear regression
analysis, both variables must be continuous data.

Question 12: FTFTF


Randomization refers only to the method of selecting individuals to each of the study
groups. It is an important way to reduce selection bias. There are a number of ways in
which randomization can be performed, including using random number tables, a random

Downloaded from https://www.cambridge.org/core. University of Edinburgh, on 19 Aug 2019 at 13:21:26, subject to the Cambridge Core terms of
use, available at https://www.cambridge.org/core/terms. https://doi.org/10.1017/9781108566100.010
302 Chapter 4b: Physics Answers

number generator or by simply tossing a coin. Using randomization, any individual in


a population has the same chance of being allocated to each treatment group as any other
individual in the sample population.

Question 13: TFTFT


In normally distributed data, the mean, median and mode are the same. In skewed data this
is not the case. If the data are positively skewed, the peak (or the mode) lies to the left, with
the tail sloping off to the right towards the more positive values (Figure 4.13.1). The mean
lies towards the tail of the curve and the median value sits between the mode and the mean.
This type of data can sometimes be normalized by logarithmic transformation of the data.
The graph below demonstrates the positive skew of the data mentioned in the question.
Median survival gives a better indication of length of life after diagnosis than the mean does.
The median is the middle value of a data series and thus if the median survival was three
years then 50% of the population (300 patients) would be deceased at three years. Because
the data are positively skewed the majority of the population lies to the left.

Mode Median Mean


y axis variable

1 2 3 4 5 6
x axis variable

Figure 4.13.1 Skewed data: mode, median and mean.

Question 14: TTFFT


A Type 1 error is also known as an α error or a false positive (rejection of the null
hypothesis). The P value represents the probability that a Type 1 error will be made.
Type II (β or false negative) errors are reduced by increasing the sample size, not Type
I errors. Blinding will not affect the likelihood of there being a difference between
groups.

Downloaded from https://www.cambridge.org/core. University of Edinburgh, on 19 Aug 2019 at 13:21:26, subject to the Cambridge Core terms of
use, available at https://www.cambridge.org/core/terms. https://doi.org/10.1017/9781108566100.010
Chapter 4b: Physics Answers 303

Question 15: TFTTT


Type II (β) errors are false negatives and would lead to an incorrect acceptance of the null
hypothesis. Power is 1 – (the Type 2 error rate).

Question 16: TFTFF


Statement a is one of the definitions of a normal (or Gaussian) distribution. The mean (the
average of all the values), the median (the value with 50% of the data points on each side of
it) and the mode (the most frequently occurring value) are all the same. ‘Normality’ is
usually quoted as the 95% confidence interval and this would be represented by the mean ±
1.96 standard deviations. Student’s (unpaired) t-test is the classic way to compare two
normally distributed sets of data. Normally distributed continuous data are parametric,
but they can be analyzed using non-parametric tests. The converse is not true, however.

Question 17: TFTFT


Mann–Whitney is a non-parametric test for ordinal data. Chi-squared is used for nominal
data. Accurately measured but non-normally distributed data are non-parametric. These
tests need not be more complicated.

Question 18: TTFFF


Statements a and b essentially describe the chi-squared test. Continuous variables would use
the t-test family. It cannot be less than zero.

Question 19: TFTFF


Categorical (or nominal or attributive) data are those where items are grouped accord-
ing to their attributes into categories – hair colour, for example. There is no ranking
and no continuity between groups. Ordinal data need to be managed carefully as they
have numerical associations, but are not continuous (e.g. ASA; you can’t be ASA
grade 2.5!)

Question 20: FFFTT


The forest plot is used in documenting meta-analyses. Individual trials are shown as
a square with an area proportional to the weight given to that trial in the meta-analysis.
From each side of the square extends a line that represents the confidence interval.
The combination of all the trials is represented as a diamond.

Question 21: TFTTF


Statement a is the basic negative exponential definition. Alcohol observes zero-order
kinetics (a fixed amount is removed per unit time, regardless of the initial amount present).
The Manley is a time-cycled pressure-generating ventilator. This is regarded as
a physiological build-up negative exponential because the rate of increase in y is decreasing
exponentially with time. In a PA catheter, the decay in concentration of indicator (tem-
perature) is governed by an exponential process. Using TCI for propofol involves a three-

Downloaded from https://www.cambridge.org/core. University of Edinburgh, on 19 Aug 2019 at 13:21:26, subject to the Cambridge Core terms of
use, available at https://www.cambridge.org/core/terms. https://doi.org/10.1017/9781108566100.010
304 Chapter 4b: Physics Answers

compartment model of concentration and so does not demonstrate a true exponential


decay.

Question 22: TTTTF


At its most basic definition, the joule is the work done when a force of 1 newton is applied over
1 metre (1 Nm). As a newton is the force required to accelerate a mass of 1 kg by
1 m.s−2, a joule is also 1 kg.m2.s−2; 1 dyne is the force required to accelerate 1 g by 1 cm. s−2
and so 100 000 dyne is equivalent to 1 N; 1 pascal (Pa) is 1 N.m−2, which makes statement d true
as well. Statements e and b cannot both be correct. This is a simple case of learning the units.

Question 23: TTFFF


Laminar flow is governed by the Hagen–Poiseuille equation:
Q = πPr4/8ηl
where Q = flow, P = pressure, r = radius of tube, η = viscosity, l = length of tube.
Reynolds number gives a likelihood of flow being turbulent. This is defined as
NR = vρd/η
where v = velocity of flow, ρ = density of fluid, d = diameter of the tube, η = viscosity of fluid.
If this number is greater than 2000, flow is likely to be turbulent, but not guaranteed,
especially if local changes in the tube cause variability in flow. In turbulent conditions, flow
is inversely proportional to the square root of the density.

Question 24: TTFFT


In the oscillatory flowmeter, the flow is directed in alternating directions by the Coanda
effect. This oscillation is proportional to the velocity of the fluid. In the pneumotachograph,
flow across a gauze screen in the stream causes a drop in pressure. Pressure is measured on
either side of the screen and the difference is related to the velocity of flow. In the rotameter
the bobbin is supported by the gas flow; as the upward pressure balances the downward
force of gravity on the bobbin, pressure must be constant. The orifice (around the bobbin) is
variable. The Wright respirometer measures volume, not flow. As there is a degree of drag in
the gearing inside the device, low volumes may not be accurately measured.

Table 4.24.1

Equipment that measures flow Equipment that measures volume

Rotameter Benedict Roth spirometer


Pneumotachograph Vitalograph
Wright’s peak-flow meter Wright’s respirometer
Mass flowmeter Dry gas meter

Downloaded from https://www.cambridge.org/core. University of Edinburgh, on 19 Aug 2019 at 13:21:26, subject to the Cambridge Core terms of
use, available at https://www.cambridge.org/core/terms. https://doi.org/10.1017/9781108566100.010
Chapter 4b: Physics Answers 305

Question 25: TTTFF


Statements a–c are the correct definitions. The definition of osmolality is correct, but
Murphy did not describe it. His law states that ‘if anything can go wrong, it will!’ Beer’s
law states that the absorption by a given thickness of solution of a given concentration
is the same as that of twice the thickness of solution of half the concentration.
Lambert’s law states that each layer of equal thickness absorbs an equal fraction of
the radiation passing through.

Question 26: TFFFF


The lower the impedance, the more current will flow in the event of touching a live
connection. It needs therefore to be low enough to allow dissipation of electrostatic charges
but high enough to protect against electric shock. British Standard EN60601 gives require-
ments for medical electrical equipment, not footwear. Water will reduce impedance.

Question 27: TTFFT


Class I equipment has no symbol. There must at least be a fuse in both the live and neutral
circuits. In the UK, the plug also has a fuse in it. Double insulated equipment is Class II.
Class I is the least safe. The floating circuit is the safest for use in theatres. Any part of the
equipment (especially the casing, if conducting) should be earthed for protection in the
event of a short circuit.

Question 28: TTTFT


Double-insulated equipment is known as Class II equipment. The symbol is as given.
As there are two layers of protection between the user and any live parts, an earth wire is
not required. Class III equipment must run on a supply of no more than 24 V. The likely
current flow caused by a short circuit in a system using 240 V mains supply with only skin
impedance (about 10 kΩ) is 24 mA. Reinforced insulation is an acceptable alternative to
double insulation.

Question 29: FTTFT


Severity increases with decreasing frequency. Mains frequency (50 Hz) is already quite low
and so its lethality is high. It is used at this frequency because of its transfer efficiency.
Diathermy by contrast is at much higher frequency (0.5–1 MHz or higher). At this fre-
quency, the penetration of the current is low and so lethality is lower. The severity of
microshock increases as the frequency of the current decreases, so the risk is greatest at low
frequencies, such as mains frequency and with direct current. It is possible for normally
functioning equipment to induce currents in, for example, pacing leads and cause micro-
shock. An oesophageal temperature probe may, rarely, cause microshock.

Question 30: FFTTF


No type C equipment is described. Type CF equipment may be connected to the heart.
The square box and stick man symbol is for type BF equipment. Type B has no box.
The maximum for Class I type B equipment is 500 μA. It is 100 μA for Class II type B. All

Downloaded from https://www.cambridge.org/core. University of Edinburgh, on 19 Aug 2019 at 13:21:26, subject to the Cambridge Core terms of
use, available at https://www.cambridge.org/core/terms. https://doi.org/10.1017/9781108566100.010
306 Chapter 4b: Physics Answers

these standards relate to single faults. Type CF equipment has standards of 50 μA for Class
I and 10 μA for Class II.

Question 31: FTTTT


The Penaz technique (also known as finapres) gives a continuous measurement of blood
pressure. It is a non-invasive technique using an infrared photoplethysmograph (like the
‘sats probe’) to measure pulse volume and a servo-controlled cuff around the finger (like
a small non-invasive blood pressure cuff) to vary pressure to keep the volume constant.
The pressure applied is related to arterial pressure. The flow is not interrupted (although it is
restricted) during this technique and so it can be used for continuous monitoring.

Question 32: FFTFT


Small syringes can develop very high pressures and cause damage either to arterial walls or
the pressure transducer. The recommended smallest size for flushing is 5 ml. Continuous
flush should be at no more than 4 ml.h–1. The ideal cannula is short, with parallel sides and
®
coated with a non-stick material like Teflon . Good asepsis is required, but full surgical
precautions are not currently mandated. Any artery can be used, but ideally one with
collateral cover from another should be used so that a clot in the cannulated artery does
not cause distal ischaemia. Either the radial artery in the wrist or the dorsalis pedis artery in
the foot are the most common sites, but the ulnar and posterior tibial are also options. It is
recommended to avoid brachial, femoral and similar arteries if possible.

Question 33: TTTTT


Fibreoptic transducers are available, but are expensive and not routinely used. They are
useful, for example, in MRI scanners, as they are not affected by the magnetic field.
The original oscillotonometer used two cuffs. Modern non-invasive methods use the same
cuff for occlusion and sensing. Accurate blood pressure readings rely on a constant volume
pulse. In dysrhythmias, there can be a wide beat-to-beat variation, which will cause
inaccuracies. Radial artery compression techniques are available, but not widely used in
clinical situations. Ultrasound Doppler can measure frequency changes and calculate
pressure. Many of these techniques are prone to user intervariability and calibration
problems.

Question 34: TFTTT


Mercury manometers can theoretically measure any pressure, but are limited by the
practicalities of size. As atmospheric pressure is 760 mmHg, a 760 mm column of mercury
will be required. Any higher pressures than this become unwieldy. To measure gauge
pressure, the top must be open (usually sealed with an air-permeable membrane to prevent
leakage of mercury). A sealed manometer with a vacuum above the mercury will measure
absolute pressure. A standard manometer will measure gauge pressure. The tube can be
sloped, but the measurement must be taken in a vertical plane (or the scale adapted to reflect
this). Surface tension will affect the result. Water manometers will over-read and mercury
ones will under-read due to this effect. This effect is reduced by using a wide-bore tube.

Downloaded from https://www.cambridge.org/core. University of Edinburgh, on 19 Aug 2019 at 13:21:26, subject to the Cambridge Core terms of
use, available at https://www.cambridge.org/core/terms. https://doi.org/10.1017/9781108566100.010
Chapter 4b: Physics Answers 307

Question 35: FTFTF


Aneroid gauges do not use liquid; the name comes from the Greek for ‘not wet’ or ‘without
liquid’. Sealed aneroid gauges measure absolute pressure. One with an open interior
measures gauge (or differential) pressure. They are ideal for measuring high pressures.

Question 36: TTFTF


Anything that reduces the transmission of the pressure along the system will cause damping.
Blood clots may completely occlude the system. Air bubbles will affect the resonant
frequency and increase damping. A long cannula will not affect damping directly, but
increases the risk of blood clots. Anything that interrupts flow may affect damping; three-
way taps are an example of this. A stiff cannula will reduce damping, not increase it.

Question 37: TTFFT


Damping reduces the amplitude of oscillation. Mean pressure will therefore be unaffected,
but as systolic and diastolic pressures represent the maximum and minimum amplitude,
these will tend toward the mean as damping increases. Heart rate (as the frequency of
oscillation) is unaffected. Stroke volume is calculated from the amplitude and so will be
affected by damping.

Question 38: FFFTF


It is not ideal to cannulate the brachial artery, as a clot in this vessel will cause forearm
ischaemia. This does not mean this artery must not be used. The transducer should be kept
at the level of the heart, not the level of the limb. Normal saline is used in the arterial line, but
the risk of microshock is increased by using saline, as it conducts electricity. Standard
transducers have a resonant frequency of about 100 Hz but it is only as the frequency falls
below 40 Hz that significant resonance is seen. Strain gauges are relatively insensitive to
temperature variation.

Question 39: FFFFF


Any cuff can cause nerve damage, either directly or by interrupting the blood supply to the
nerve, but it is not as high as 1 in 100. Atrial fibrillation will cause significant inaccuracy in
the measurements made. This is a contentious point, but most texts claim that the systolic
pressure is the most accurate measurement (the first presence of oscillation). Mean is next
most accurate (maximum amplitude of oscillation). While the original oscillotonometer
required two cuffs (one to occlude, one to sense), modern devices use the same cuff for both.

Question 40: TFFFT


Statement a is a definition of optimal damping. This usually occurs with a coefficient of
0.6–0.7. Underdamping does lead to resonance, but the signal will overshoot, not under-
shoot. Underdamping might allow resonance, but this does not give better reproduction of
the intended waveform. Optimal damping allows a rapid response (at the expense of a small
amount of overshoot). Critical damping (a coefficient of 1.0) will lead to no overshoot, but
a slow response.

Downloaded from https://www.cambridge.org/core. University of Edinburgh, on 19 Aug 2019 at 13:21:26, subject to the Cambridge Core terms of
use, available at https://www.cambridge.org/core/terms. https://doi.org/10.1017/9781108566100.010
308 Chapter 4b: Physics Answers

Question 41: FFTFT


Adequate return of neuromuscular function may be derived clinically. Signs include
sustained head lift for at least 5 seconds, generating a VC of at least 10 ml.kg–1 or generation
of an inspiratory pressure of at least –25 cmH2O. Tidal volume is not a reliable guide to
recovery, as normal volumes can be generated with only 20% functional diaphragm muscle
receptor activity. In addition, oxygen saturations can be maintained with minimal ventila-
tory effort. The carbon dioxide levels will increase, however.

Question 42: FFTFT


PiCCO is based on the principle of pulse contour analysis. PiCCO stands for Pulse Induced
Contour Cardiac Output. PiCCO requires a standard central line with a thermistor on the
distal lumen and a special arterial cannula with a thermodilution sensor. The arterial
cannula is sited in the femoral, brachial or axillary artery.

Question 43: TFTFF


In TOF, the ratio of the fourth to the first twitch is measured. Four twitches of 2 Hz each are
applied over 2 seconds. One should leave a gap of 10 seconds between each TOF. As the
muscle relaxant is administered, fade is noticed first, followed by the disappearance of the
third then the second and finally by the first twitch. On recovery, the first twitch is the first to
appear, then the second, followed by the third and fourth; reversal of the neuromuscular
block is easier if the second twitch is visible. For upper abdominal surgery, at least three
twitches must be absent to achieve adequate surgical conditions.

Question 44: FFFFF


In order to reliably detect muscular twitches as neuromuscular blocking drugs are being
cleared from the junction, a nerve stimulator must be able to deliver a current that is greater
than the current required to produce a maximal twitch. This is known as a supramaximal
twitch current. In general, with well-applied electrodes, the maximum twitch current is
around 60 mA (not microamps, as in the question) so a reliable stimulator must produce
more than this. Most sources recommend about 80 mA.
Mains power might, in the event of a fault, be transmitted directly to the patient through
the highly conductive electrodes. This could be life-threatening so stimulators are battery
powered. It is also far more convenient for batteries to be used, as the stimulator is often
moved or used in locations where mains power would be inconvenient. There are some
exceptions to this. Some anaesthesia workstations have neuromuscular monitoring modules
that are mains powered. The danger of mains conduction is ameliorated using optoisolators
to ensure that no conductive path is available between the mains power and the patient.
If the block has fully developed then there will be no twitch at any current. This is the basis
of the monitoring. The supramaximal twitch must be tested before the block is initiated.
The leads are generally coloured red and black for positive and negative, respectively.
This allows identification of the leads so that the negative can be placed distally, which
minimizes current required.
Most neuromuscular blocker monitors supply twitches of 0.2 s. It is not normal practice
to use variable-width twitches and this function is not supplied in stimulators for this

Downloaded from https://www.cambridge.org/core. University of Edinburgh, on 19 Aug 2019 at 13:21:26, subject to the Cambridge Core terms of
use, available at https://www.cambridge.org/core/terms. https://doi.org/10.1017/9781108566100.010
Chapter 4b: Physics Answers 309

purpose. They often offer single twitches, trains of four (four twitches delivered in 2 s) and
tetanic sequences for measuring different types of block and for use with different agents.

Question 45: FTTTF


Cardiac output monitors can be classified as non-invasive, invasive, and semi-invasive or
minimally invasive. Transthoracic impedance is a non-invasive technique. Other non-
invasive techniques include radionuclide scans, magnetic resonance imaging and transthor-
acic Doppler.
Transthoracic impedance applies a high-frequency, low-magnitude current, which is
detected using six externally placed electrodes. Thoracic impedance is indirectly propor-
tional to changes in blood volume, reflecting changes in the cardiac cycle.
This technique has a number of disadvantages, including its inaccuracy in the presence of
arrhythmias.

Question 46: FFTTF


In laminar flow, fluid moves in a steady manner, flow rate is greatest at the centre of the flow
stream (twice the average flow rate), a pressure difference must exist for fluid to flow, flow is
directly proportional to this pressure difference and resistance of the tube is calculated by
the ratio of pressure to flow. A number of variables must be known to quantify the
relationship between pressure and flow. Both the Hagen–Pouseille equation and
Reynolds’ estimation are important.

Question 47: FTTFF


In turbulent flow, flow is characterized by swirls and eddies; the transition of laminar to
turbulent flow can occur at constrictions. Fluid velocity varies across the cross-sectional area
of the tube. Flow is proportional to the square root of pressure, resistance is no longer
constant (the relationship between pressure and flow is no longer linear), density of the fluid
is the important determinant in turbulent flow.

Question 48: FFFFT


Wright’s respirometer is a compact and light respirometer used to measure tidal volume
and minute volume. For clinical use, the respirometer reads accurately (±5–10%) within the
range of 4–24 l.min–1. A minimum flow of 2 l.min–1 is required for the respirometer to
function accurately. A paediatric version exists with a capability of accurate tidal volume
measurements of between 15 and 200 ml.

Question 49: FFFTF


The pneumotachograph measures gas flow, sensing the change in pressure across
a fixed resistance through which gas flow is laminar. Density and viscosity of the gas
can alter the accuracy; compensation is by continuous gas composition analysis via
a sampling tube.

Downloaded from https://www.cambridge.org/core. University of Edinburgh, on 19 Aug 2019 at 13:21:26, subject to the Cambridge Core terms of
use, available at https://www.cambridge.org/core/terms. https://doi.org/10.1017/9781108566100.010
310 Chapter 4b: Physics Answers

Question 50: FTTFT


Flowmeters measure the flow rate of a gas passing though them and are calibrated for each
gas. They have an accuracy of ±2.5%. Both laminar and turbulent flows are encountered,
making both the viscosity and density of the gas relevant. The pressure across the bobbin is
constant.

Question 51: FTFFT


Noise describes the effect of external interference as the output signal is generated. Noise
can be from many sources, not just electrical. High pass filters allow passage of higher
frequencies, hence the name. Notch filters are used in ECG machines to ignore the effects of
mains electricity. SNR (signal-to-noise ratio) is given by this equation.

Question 52: FTTFT


Thermodilution requires a pulmonary artery flotation catheter; 5–10 ml of cold saline or 5%
dextrose is injected via the proximal port. Changes in blood temperature are detected by the
thermistor at the tip of the pulmonary artery flotation catheter. A thermodilution curve
plotting temperature against time is produced. Cardiac output is inversely proportional to
the area under this curve. Bedside computer programs calculate cardiac output using the
Stewart–Hamilton equation. Measurements are affected by changes in intrathoracic pres-
sure during the respiratory circle, so injections are performed at the end of expiration.

Question 53: FFTFF


In Fourier analysis, the tenth harmonic is usually cited as being clinically necessary.
Resonance is the tendency of a system to oscillate at maximum amplitude at certain
frequencies. Resonant (or natural) frequency is determined by the mass and stiffness of
the system. Critical damping is the best compromise for the system. By convention, in
a critically damped system D = 1. In invasive arterial monitoring, if the system is under-
damped, the output changes quickly in response to the step-up input, overshooting and
oscillating around the true value.

Question 54: TTTTF


An exponential function describes a situation in which the rate of change of quantity of
a substance is directly proportional to the quantity of substance at that time. The quantity of
the substance remaining approaches, but never touches, the x-axis, until it reaches a steady
state known as an asymptote. The absolute amount of substance eliminated per unit time
varies, but the proportion of substance eliminated per unit time is constant.

Question 55: TTFFF


A defibrillator applies an electrical current to the heart in order to convert ventricular
fibrillation to sinus rhythm. Generally speaking, biphasic defibrillators use 30–40% less
energy than monophasic ones. Both mono- and biphasic defibrillators deliver a DC
shock.

Downloaded from https://www.cambridge.org/core. University of Edinburgh, on 19 Aug 2019 at 13:21:26, subject to the Cambridge Core terms of
use, available at https://www.cambridge.org/core/terms. https://doi.org/10.1017/9781108566100.010
Chapter 4b: Physics Answers 311

Question 56: FFTFF


In a defibrillator, a step-up transformer is used to generate a potential difference of 5–8 kV.
The capacitor stores the required charge before discharging across the patient’s heart.
The inductor has a low resistance to DC. A defibrillator typically delivers a current of
50 A into a resistance of 50 Ω.

Question 57: FFFTF


Remember, E = ½ CV2, thus 200 J. Thoracic impedance is in the region of 50–150 Ω. This is
reduced after the first shock, by conductive gels, front to back defibrillation and application
of firm paddle pressure. During defibrillation of pulseless VT or VF, a non-synchronized
shock can be administered.

Question 58: FFTFF


Classically, pacemakers maintain an adequate heart rate in the presence of acquired
symptomatic atrioventricular block. There is a risk of microshock during transvenous
pacing. Modern pacemakers combine a pacemaker and defibrillator in a single implantable
device, and are externally programmable.

Question 59: TFFFT


Spirometry is the timed measurement of dynamic lung volumes during forced expiration
and inspiration. Measurement of maximum expiratory flow rate over the middle 50% of the
vital capacity (FEF 25–75%) is a sensitive index of small airway function. FEV1/FVC <80%
constitutes an obstructive ventilatory defect, FEV1/FVC >80% constitutes a restrictive
ventilatory defect.

Question 60: FTFTT


Table 4.60.1 Approximate given values

Total lung capacity 80 ml.kg–1


Vital capacity 65 ml.kg–1
Inspiratory capacity 50 ml.kg–1
Functional residual capacity 30 ml.kg–1
Inspiratory reserve volume 40–45 ml.kg–1
Tidal volume 7 ml.kg–1
Expiratory reserve volume 15 ml.kg–1
Residual volume 15–20 ml.kg–1

Downloaded from https://www.cambridge.org/core. University of Edinburgh, on 19 Aug 2019 at 13:21:26, subject to the Cambridge Core terms of
use, available at https://www.cambridge.org/core/terms. https://doi.org/10.1017/9781108566100.010
312 Chapter 4b: Physics Answers

Question 61: FFFFT


In obstructive airway diseases the chest is often hyperexpanded with an increased residual
volume. In increased age, supine posture, anaesthesia, muscle relaxants, laparoscopic
surgery, pulmonary fibrosis, pulmonary oedema, obesity, pregnancy and respiratory muscle
diseases, the residual volume is decreased.

Question 62: FFTTT


In body plethysmography the subject sits in an air-tight chamber. The subject inhales or
exhales to a particular volume, then a shutter drops across their breathing tube. The subject
makes respiratory effort against the closed shutter, causing their chest volume to expand and
decompressing the air in their lungs. The increase in their chest volume slightly reduces the
box volumes and this slightly increases the pressure in the box. Using Boyle’s law, FRC can
be deduced.
Both nitrogen wash-out and helium wash-in methods only measure communicating gas.
This is a disadvantage compared to total body plethysmography.

Question 63: TFTTT


FRC is dependent on the balance of the tendency of the lungs to recoil and the thoracic cage
to expand.

Question 64: FFFTT


There are three (not five) phases of heat loss during surgery. Redistribution is characterized
by vasodilatation, with transfer of heat from the core to the periphery. The linear phase is
subdivided into heat loss by radiation, convection, conduction, evaporation and respiration.
It is this phase at which most heat loss prevention techniques are directed. The final, plateau,
phase is characterized by peripheral vasoconstriction to limit further heat loss. It is impaired
by sympathetic blockade, either as a result of disease processes like diabetes or iatrogenically
with, for example, neuraxial blockade.

Question 65: TFTFT


Rectal temperatures tend to be reliably higher than core (by 0.5–1%) due to local heat
generation by gut flora. Oropharyngeal temperatures are too unreliable to be used practically
and probes are easy to displace. When placed accurately (just behind the soft palate), naso-
pharyngeal temperatures are reliable. The upper oesophagus is cooled by inspired gases and is
unreliable. Probes should be placed in the lower 25%. The tympanic membrane provides an
easy and reliable measurement, although it is used infrequently during anaesthesia.

Question 66: FTTFF


Mercury thermometers are liquid expansion devices. Their response time is slow (measured
in minutes) and they have a relatively narrow temperature range. Special devices are
required for low and high temperatures. They do not require a power supply and are
cheap, although concerns around the safety of mercury-containing devices and the

Downloaded from https://www.cambridge.org/core. University of Edinburgh, on 19 Aug 2019 at 13:21:26, subject to the Cambridge Core terms of
use, available at https://www.cambridge.org/core/terms. https://doi.org/10.1017/9781108566100.010
Chapter 4b: Physics Answers 313

contamination due to breakage now limit their use (mercury-in-glass sphygmomanometers


are similarly now only used occasionally).

Question 67: FTFFF


The reference electrode can be an Ag/AgCl or Hg/HgCl (Calamel) electrode. H+ concentra-
tions can be measured in blood, urine and CSF, but not mucus. The potential output is
60 mV per pH unit. Measurements are done at 37 °C as the electrode is temperature
sensitive.

Question 68: TFTTF


Regular calibration reduces drift-induced errors. Two buffer solutions are used, each with
a fixed concentration of hydrogen ion decided by international agreement. A correction
factor is used if the sample is not at 37 °C as all measurements are done at this temperature.
Electrodes must be regularly cleaned to prevent build-up of protein. This is done auto-
matically after every sample and also at regular servicing.

Question 69: TFTTT


The Severinghaus electrode uses H+-sensitive glass and so is a modified pH electrode. It is
CO2, not H+, that diffuses across the membrane and combines with water to form H+ and
bicarbonate ions. The reference is Ag/AgCl. As the system relies on diffusion of CO2 across
a membrane, the response time is slow, usually 2–3 minutes.

Question 70: TFTTT


Piezoelectric resonance has been used for anaesthetic agent monitoring, but not for CO2
monitoring.

Question 71: FFFTT


Side-stream analyzers are slower as the sample has some distance to travel to the
analyzer. A delay of less than 3.8 s is considered acceptable. It is affected by the length of
the tubing, the calibre of the tubing (usually 1.2 mm) and the sampling rate (frequently
150–200 ml.min–1 but less in paediatric monitors). The majority of monitors require
®
calibration. The tube is Teflon coated to render it impermeable to CO2 and to prevent
the plastic reacting with anaesthetic vapours. Whilst the tubing and hardware can be used
for analysis of other gases, a carbon dioxide analyzer only measures carbon dioxide.

Question 72: TFTTT


Fleming stated that when a current flows in a wire and an external magnetic field is applied
across that flow, the wire experiences a force perpendicular to both the flow and the field.
A dynamo and electric motor are exactly the same components. In an electric motor,
electricity is provided to turn the shaft of the motor. In a dynamo, the shaft of the motor
(dynamo) is manually turned to produce electricity.

Downloaded from https://www.cambridge.org/core. University of Edinburgh, on 19 Aug 2019 at 13:21:26, subject to the Cambridge Core terms of
use, available at https://www.cambridge.org/core/terms. https://doi.org/10.1017/9781108566100.010
314 Chapter 4b: Physics Answers

Question 73: TFTFT


A frequency of 50–60 Hz is a highly dangerous for humans, but it is cheap and easy to
provide and so is used almost universally for AC power transmission; 240 V is the root mean
square (RMS) voltage of UK AC power. The peak voltage of the sine wave is about
340 V. High-voltage lines reduce current required for the same power transmission and
therefore the power lost to heat. Hospitals frequently receive higher-voltage supplies and
then step this down in local transformers.

Question 74: TFTFT


In an ideal capacitor the charge held is denoted by Q in the equation:
Q = CV
where C is the capacitance, measured in farads, and V is the potential difference, measured
in volts.
A simple inductor is composed of a coil of wire wound around a core, often composed of
a material called ferrite. Any electric current flowing through a wire generates a magnetic
field in proportion to the current. The change in the magnetic field generates an electric field
in the opposite direction to the change that caused it. This is known as Lenz’s law. The effect
is that any change in current is opposed by the coil.
Diodes are generally made of silicon, but other semiconductors are used for specific
purposes.
A transistor is a semiconductor component that uses a small input voltage between the
‘base’ lead and the ‘emitter’ lead to switch or regulate a current between the ‘collector’ lead
and the ‘emitter’ lead. There are many types of transistor optimized for different purposes.
Transistors do not change one form of energy into another.
Amplifiers typically receive an input and deliver an output of the same form as the input,
but with a greater amplitude. A familiar example of this is a home audio amplifier.

Question 75: FFTTF


In UK domestic supplies there are three wires. These are the ‘live’, which is insulated with
brown plastic, the ‘neutral’, which is insulated with blue plastic, and the ‘earth’, which is
insulated with green and yellow striped plastic.
If the live wire is touched then electrocution is possible. If a person touches the live wire in
such a way as to complete a circuit between the live wire and ‘earth’, which can mean
something as simple as being stood on the ground, then electricity will flow through that
person according to the resistance of their body.
The commonly seen British three-pin mains plug is described in ‘British Standard 1363,
13A Plugs, Socket Outlets, Adaptors and Connection Units’. This standard covers the design
of the plugs and sockets and the correct method for wiring them. It is extremely important
to connect appliances correctly, including the wiring of the plug.

Question 76: FTFTT


Direct current is a steady flow of current in one direction round a circuit. Alternating
current flows first in one direction then the opposite and cycles between these two states in
a standardized waveform. In the UK the waveform is sinusoidal with a frequency of 50 Hz

Downloaded from https://www.cambridge.org/core. University of Edinburgh, on 19 Aug 2019 at 13:21:26, subject to the Cambridge Core terms of
use, available at https://www.cambridge.org/core/terms. https://doi.org/10.1017/9781108566100.010
Chapter 4b: Physics Answers 315

and a root mean square amplitude of 230 V. Batteries, in contrast, provide a source of direct
current the voltage of which depends on the chemistry of the cells in the battery and the
number and arrangement of cells connected together.
Thermocouples exploit the Seebeck effect. This is the production of electric current at
a junction between two types of metal. The current produced at the junction varies with the
temperature of the junction and the devices can be accurately calibrated to measure
temperature.
Nowadays most defibrillator units are biphasic. This technique involves the application of
a positive current followed by a negative current over a time period of around 10 ms.
The timing and charge waveforms are dependent on the manufacturer of the device but, in
all cases, the capacitors that store the charge prior to its delivery are charged by DC.

Question 77: TTFTF


In 1827, Georg Simon Ohm (1789–1854) published a book called ‘Die Galvanische Kette,
Mathematische Bearbeitet’, which translates roughly to ‘A Mathematical Investigation of
Galvanic Circuits’.
Mathematically Ohm’s law can be expressed as:
V = IR, I = V/R or R = V/I
Ohm’s law does not relate to the temperature of the system. It has no terms representing
charge or temperature. Current flow is represented by the letter I, measured in amperes,
potential difference by V, measured in volts, and resistance by R, measured in ohms.

Question 78: FTFFF


Scavenging systems are covered by British Standard BS EN ISO 80601–2-13:2012 ‘Medical
Electrical Equipment: Particular Requirements for Basic Safety and Essential Performance
of an Anaesthetic Workstation’, which is under review at the time of this publication.
The scavenging system must be able to remove sufficient gas to prevent the release of
anaesthetic gas mixtures even at high input gas flows and during patient exhalation, which
can reach high flow rates in some situations. The standard details both an extraction flow
rate and an induced flow rate. The extraction rate is the rate of gas extraction from the outlet
of the breathing system.
The standard referenced above dictates that the extraction flow rate should be between 50
and 75 l.min–1 and the induced flow rate should be below 0.05 l.min–1.
The system does not require the generation of a vacuum. Low pressure is provided by
a pump system typically providing negative pressure to a cluster of theatres.

Question 79: FTTFF


Activated charcoal has an extremely high surface-area-to-volume ratio. When anaesthetic
agents are passed through a cannister containing activated charcoal they are adsorbed (held
as a thin film on the surface of the charcoal grains – this is different to absorption, which is
dispersal throughout a medium rather than adherence to its surface) and retained while the
majority of the gas is allowed to pass through.

Downloaded from https://www.cambridge.org/core. University of Edinburgh, on 19 Aug 2019 at 13:21:26, subject to the Cambridge Core terms of
use, available at https://www.cambridge.org/core/terms. https://doi.org/10.1017/9781108566100.010
316 Chapter 4b: Physics Answers

Activated charcoal is particularly useful in adsorbing organic and non-polar molecules,


so it is ideally suited to use in scavenging all of the commonly used anaesthetic gases.
Unfortunately, it does not adsorb nitrous oxide, which passes through unimpeded.
The Cardiff Aldosorber is quoted as needing replacement after 12 hours of use or on
gaining 50 g in weight, whichever occurs sooner.
The carbon used in the units shows no indication of exhaustion other than the increase in
weight. It may be possible to measure the difference between input and output of anaesthetic
gases, but weighing the units is the preferred method.
On the application of heat to a used unit, adsorbed gases will be released into the
atmosphere. In the past this has been utilized to ‘regenerate’ used units, but the practice is
not recommended. The gases are also released if the unit is disposed of by incineration.
The units are relatively large, but not unduly heavy. They can be readily transported
provided that sufficient volume is available to store them.

Question 80: TFFFT


In a circle system, unidirectional valves are required on both inspiratory and expiratory arms
in order to maintain flow in one single direction. Fresh gas should not enter between the
patient and the expiratory valve, nor should the APL valve be between the patient and the
inspiratory unidirectional valve. If any of these exist, it is possible to have rebreathing.
Placing modern vaporizers within the circuit would add far too much resistance to breathing.

Question 81: TFTTF


Under optimum conditions (every granule of soda lime absorbs its maximum capacity for
CO2), 25 l.100 g–1 is achievable, but this is rarely the case in practical use. The indicator only
shows the state of the outside of the granule. There may still be absorptive capacity inside.
After a period of non-use of only a few hours, granules may turn from violet back to white
due to a chemical reaction inside the granule. This may give a falsely reassuring assessment
of absorptive capacity. Soda lime in this state will rapidly turn violet again when exposed to
CO2. White to violet is now the approved colour change, although some anaesthetists will
remember that the colour changes used to be different. Soda lime is highly efficient at
removing CO2. The design of the canister and the size and shape of the granules affect how
much of the soda lime is in contact with the gas travelling through it. The canister should
have a length-to-diameter ratio of 1.5:1 and the granules should be 4–8 mesh in size to
maximize absorptive capacity, while minimizing resistance to breathing. Soda lime is
corrosive and causes a range of nasty respiratory problems.

Question 82: TFFTF


The Jackson–Rees modification was the addition of a bag to the end of a Mapleson E
breathing circuit. The bag is open at both ends. Controlled ventilation with the Mapleson
E can be achieved by intermittently occluding the end of the tubing. These systems are low
resistance and are mainly used in paediatric practice. If the volume of the expiratory limb is
smaller than the patient’s tidal volume, rebreathing of expired gases may occur. For
controlled ventilation, 1.5–2 times minute volume is appropriate; 2.5–3 times minute
volume is required for spontaneous ventilation.

Downloaded from https://www.cambridge.org/core. University of Edinburgh, on 19 Aug 2019 at 13:21:26, subject to the Cambridge Core terms of
use, available at https://www.cambridge.org/core/terms. https://doi.org/10.1017/9781108566100.010
Chapter 4b: Physics Answers 317

Question 83: TTTTF


The first gas leaving the system is (unused) dead space gas, which is inefficient.
The recommended flow in spontaneous ventilation is 150–200 ml.kg–1.min–1.
During spontaneous ventilation, the Mapleson D behaves just like a T-piece
(Mapleson E). During the expiratory pause, the tubing between the patient and the
APL valve is purged of expired gas. Normocapnia can be achieved at relatively low
flows (70–100 ml.kg–1.min–1) in controlled ventilation, but about twice this is
required when spontaneously ventilating.

Question 84: FFTTT


The fresh gas flow required to prevent rebreathing in the Mapleson A circuit is
70–85 ml.kg–1.min–1, which equates to 5–6 l.min–1. The coaxial A is called a Lack circuit.
The Bain is a coaxial Mapleson D. At high respiratory rates, the end-expiratory pause is
shortened and efficiency falls as it is during the expiratory pause that the tubing between the
reservoir bag and the APL valve is purged. The Mapleson A is highly efficient for sponta-
neous ventilation as, at lower flows, the expired dead space gas is also reused. At high tidal
volumes, expired gas can flow back down the tubing and contaminate the reservoir bag.

Question 85: FFTFF


The magnetic field strength is fixed for each MRI machine, although most current machines
use between 0.2 and 4 Tesla. The magnets in an MRI scanner can be permanent or electro-
magnets. The size of permanent magnet required to generate the higher flux densities is too
great (and too heavy) for these to be practical, so electromagnets are used instead. Most MRI
scanners do not use ferromagnetic cores in their electromagnets. These ‘air core’ magnets
need very high currents through the windings to generate the same flux density and the
resistance in the wires is such that heat generation causes an unacceptable loss of power.
In order to overcome this, the magnet is cooled with liquid helium (not oxygen) below 4.2 K.
At these sorts of temperatures, the windings become superconductors (virtually resistance-
free) and so generate much less heat and power loss. The field around a scanner causes
nuclei (not just neutrons) to align and decreases exponentially with distance.

Question 86: FFTTT


The pulsed magnetic field is applied perpendicular, not parallel, to the main field, inducing
precession. Hydrogen (not nitrogen) produces a strong signal and is abundant mostly due to
being a component of water. Phosphorus does indeed generate a weaker magnetic reso-
nance signal, therefore a stronger magnet is needed. In fact, modern steel hospitals often act
like a Faraday cage reducing mobile phone signals inside.

Question 87: TFTFF


The electron beam in an X-ray device is generated at the cathode and strikes a tungsten anode to
produce X-rays. Technetium-99m is most commonly used for injected isotope imaging because
it can be easily attached to chemical compounds. It also has a short half-life, thus reducing
radiation dose to the patient. Chromium-51 is used to calculate red cell and plasma volumes,

Downloaded from https://www.cambridge.org/core. University of Edinburgh, on 19 Aug 2019 at 13:21:26, subject to the Cambridge Core terms of
use, available at https://www.cambridge.org/core/terms. https://doi.org/10.1017/9781108566100.010
318 Chapter 4b: Physics Answers

but not necessarily imaging. In modern plain X-rays, a phosphor screen (not photographic film)
is used to capture images, which are then digitized for viewing on computer screens.

Question 88: TTTTT


CT scanners provide varying doses of radiation. In terms of whole-body dose, a chest X-ray
scan might be 0.02 mSv (millisieverts). A head CT is 1–2 mSv, a chest CT is 5–7 mSv and an
abdomen CT is 8 mSv. The fastest scanners can produce a real-time image.

Question 89: TFTTT


PET scanning involves positive β-decay where a proton (frequently but not exclusively
fluorine-18 in FDG) decays into a neutron and a positron (like an electron but with
a positive charge). When this then interacts with an electron, an annihilation event occurs,
resulting in the destruction of both particles and the production of two γ-photons at
511 keV, being emitted in approximately opposite directions. Both of these photons need
to be detected to differentiate from other γ-emissions.

Question 90: TFFFT


The joule is a derived unit of energy. Its most basic derivation from the SI base units is:
J = kg.m2.s–2 . It can also be derived: J = N.m as stated in the question, J = Pa.m3, J = W.s
and J = C.V.
Division of force by area yields pressure, not work.
Work is the rate of expenditure of energy, not the energy itself. If an amount of energy is
supplied to a system in the form of heat then that energy is available for release as work.
Muscular work can be calculated by multiplying the force applied by the muscle by the
distance over which the force has acted.

Question 91: FFTTF


Power is the rate of energy expenditure. It is expressed in watts and is calculated as W = J.s–1.
Work is the amount of energy expended in joules so if 1 joule of work is done per second, the
power will be 1 watt. The watt is not one of the seven SI base units.
The unit ‘horsepower’ is an imperial unit and was used historically use to describe the
power output of machines. One horsepower is approximately equal to 750 W.
The work of breathing, and therefore the power of breathing, does depend on the type of
airflow in the airways, but laminar flow is more efficient than turbulent flow and so requires
less expenditure of energy and, therefore, less power.

Question 92: TTFTT


That energy can be neither created nor destroyed is the law of conservation of energy.
Energy can change form and be expended to do work, but the total energy in a closed system
always remains the same.
The joule is one of the seven base SI units.
Inspiration is the expenditure of energy by the inspiratory muscles to increase the size of
the thorax, reducing the pressure within it. Ambient air then flows along its pressure

Downloaded from https://www.cambridge.org/core. University of Edinburgh, on 19 Aug 2019 at 13:21:26, subject to the Cambridge Core terms of
use, available at https://www.cambridge.org/core/terms. https://doi.org/10.1017/9781108566100.010
Chapter 4b: Physics Answers 319

gradient. This is analogous to a force being moved through a distance and is the expenditure
of energy, or work.
Some of the muscular energy is stored in the elastic tissue of the lungs, but some is lost
and a proportion of this loss is heat. Heat is not ‘stored’ in this action, but is carried away by
the blood and air, and released elsewhere. The energy stored in the elastic tissue of the lung
parenchyma is used in passive expiration. Active expiration, using muscular effort, can be
used in higher rates of respiration.
The difference between the inspiratory and expiratory limbs of the pressure–volume loop
is a manifestation of hysteresis. The area of the loop represents the work of breathing.
The energy used in doing the work is lost, at least partially as heat. Some of the energy is used
in the ‘thoracic pump’ and is transferred to the circulation, for example.

Question 93: FFFTT


Kinetic energy is calculated by multiplying the mass of an object by the square of its own
velocity.
Electrical energy can be stored in a magnetic field, but this is not necessarily the case.
Most batteries, for example, are not intrinsically magnetic at any charge state.
Thermal energy is not the same as temperature. A litre of water at 95 °C contains far more
thermal energy than a steel sewing needle at 400 °C. If the hot needle is dropped into the far
cooler water, energy will flow from the needle to the water even though the water contains
far more energy than the needle.
It is extremely important to understand the difference between heat and temperature.
Elastic energy is stored in any material that can be deformed and will tend to return
towards its original shape. A spring, for example, can be compressed or expanded from its
rest shape and will store energy. If the material is stretched beyond its elastic limit then the
amount of energy returned may be reduced.
Sounds are propagated through matter in the form of compression waves. Air is mani-
festly capable of transferring sound energy – we use this property when speaking to each
other.

Question 94: TFFFF


The watt is derived: W = J.s−1 and represents power or work done per unit time. When using
the unit, written W, it is important to note that the time is already included so watts
per second is meaningless.
The joule is the SI base unit for energy; 1 joule of energy is expended as work when a force
of 1 N moves through 1 m.
Efficiency is the ratio of the total amount of work done to the amount of useful work
output. In all processes some energy is lost as heat, so no process is 100% efficient. Human
muscles have been estimated to be around 18–20% efficient in trained athletes. It is very
clear that muscles are inefficient because shivering is used as a method of maintaining body
temperature.
If a human has a cardiac output of approximately 5 l.min–1 and a blood pressure of about
90 mmHg, we can convert mmHg to kPa by dividing by 7.6: 90/7.6 = 11.84 kPa.
Next we can calculate the blood flow per second by dividing 5 l by 60:
5/60 = 0.083 l.s–1 = 8.3 × 10−4 m3.s–1

Downloaded from https://www.cambridge.org/core. University of Edinburgh, on 19 Aug 2019 at 13:21:26, subject to the Cambridge Core terms of
use, available at https://www.cambridge.org/core/terms. https://doi.org/10.1017/9781108566100.010
320 Chapter 4b: Physics Answers

Now, since pressure = force/area and flow = area × length/time, if we multiply the pressure
by the flow, we end up with force × length/time.
Since power is work done (force × length) in a unit time and we have converted to SI
units, we can simply multiply the two to find the power: 11840 (11.84 kPa) × 0.000084 =
0.995 W.
So the power of a human heart is approximately 1 W. Obviously the heart is not 100%
efficient. It is around 10% efficient, so the total power input for the heart, at rest, is
around 10 W.
The work done in lifting an object is calculated:
Mass × height lifted × gravity
Gravity has a value of 9.8 m.s–2 so 100 × 20 × 9.8 = 19600 J.

Question 95: TTTFF


Heat cannot be created or destroyed. Heat is energy and is subject to the law of conservation
of energy.
Temperature is a description of the average kinetic energy of the molecules in a substance.
It is possible for, example, for 1 l of water to be colder than 1 g of iron, even though it might
contain significantly more heat energy. This is due to the higher specific heat capacity of
water (it requires more heat to raise the temperature of 1 kg of water by 1 °C than it does to
raise the temperature of 1 g of iron by 1 °C) and the fact that there is 1000 times as much
water as there is iron.
If the temperature of a given object increases then there must be more heat per molecule
on average and, therefore, more heat in total.
If heat is added to a solid substance, such as water ice, its temperature will increase until it
reaches the melting point of the substance. At this point, further heat will cause more
melting until the whole amount is liquid. Then the temperature will begin to rise again until
the boiling point is reached. Here, further heat will cause evaporation until the whole
amount is converted to vapour. Then the temperature will begin to rise again. The energy
used in melting is the latent heat of fusion and the energy used in evaporation is the latent
heat of vapourization. The heat can be reclaimed if the substance is condensed and then
frozen.

Question 96: FFFFF


The specific heat capacity of a substance is the amount of heat energy required to raise the
temperature of 1 kg of the substance by 1 °C. In part a, no mass was specified for the object.
The specific heat of water is 4.2 kJ.kg–1 and that of blood, approximately 3.5 kJ.kg–1.
The difference is because of the large quantity of material other than water in blood.
The material that is not water has a much lower specific heat capacity and therefore, on
average, the specific heat capacity of blood is lower than that of water.
The SI unit of temperature is the kelvin. One kelvin is equal in magnitude to one degree
Celsius and the two are often used almost interchangeably when discussing changes in
temperature.

Downloaded from https://www.cambridge.org/core. University of Edinburgh, on 19 Aug 2019 at 13:21:26, subject to the Cambridge Core terms of
use, available at https://www.cambridge.org/core/terms. https://doi.org/10.1017/9781108566100.010
Chapter 4b: Physics Answers 321

Whereas a degree Celsius is defined with respect to the freezing and boiling points of
water, the kelvin is defined as 1/273.16 of the triple point of water, which is exactly 0.01 °C.
This means that:
• the degree Celsius is exactly equal to the kelvin
• zero kelvin rests at thermodynamic absolute zero
• water freezes at 273.15 K
The Sharma gap is purely fictional – Dr Sharma gave us the idea of this question! In general,
in a human body, there will be a gradient between the core and the periphery, it is known as
the core–periphery difference and may be useful in monitoring patients, for example under
anaesthesia or in critical care.

Question 97: TTFFF


There is a variety of legislation in the UK, one of which is the British Standard BS EN
60825–1:2007. Normal spectacles provide no protection from most lasers.
The recommendation is for an oxygen concentration of less than 30% where possible to
reduce the risk of fire. Nitrous oxide should be avoided. Laser protection supervisors
provide a higher level of knowledge and training, but use of lasers is not restricted to
these personnel. Anyone using a laser should have a basic safety awareness training and
there should be a trained laser safety officer in the clinic where a laser is being used. Laser
light is dangerous even after multiple reflections and so protection is required for all staff in
the room where these devices are being used. Also, matte, non-reflecting materials should be
used where possible.

Question 98: TFTTF


Helium–neon lasers can produce light in multiple spectra, especially infrared, red and
green, but not UV. The argon laser is most efficient in the green spectrum, but can also
emit blue light. One of the features of laser light is that it is coherent.

Question 99: TFTTT


Laser light is classically monochromatic (one wavelength), non-divergent (the beam
remains a constant width over considerable distances), coherent (the waves of light are in
phase and so do not cancel each other out, rather they amplify each other) and collimated
(another word for non-divergent). Photons are particles of light (remember the wave and
particle theories of electromagnetic radiation).

Question 100: FFFTT


Absolute humidity is the mass of water in a given unit of gas. This does not change with
temperature. Relative humidity is this same quantity expressed as a proportion of the total
amount of water that the gas can hold. As the amount a gas can hold increases with
temperature, relative humidity falls as a gas is heated. In the alveoli, fully saturated air has
an absolute humidity of 43 g.m–3; 34 g.m–3 is the figure for the upper trachea at 34 °C.
Relative humidity is derived from the Regnault hygrometer, but it actually measures dew
point (the temperature at which relative humidity is 100%).

Downloaded from https://www.cambridge.org/core. University of Edinburgh, on 19 Aug 2019 at 13:21:26, subject to the Cambridge Core terms of
use, available at https://www.cambridge.org/core/terms. https://doi.org/10.1017/9781108566100.010
322 Chapter 4b: Physics Answers

Question 101: TTFFF


Water overload is a risk with ultrasonic nebulizers. Droplets of greater than 5 μm will
be deposited in the trachea and bronchi. Those less than 1 μm may impair gas
exchange. Heat and moisture exchangers (HMEs) are passive systems, less efficient
than water baths.

Question 102: FTTTT


See answer to Question 100.

Question 103: TFFFF


Paramagnetic analyzers are gas analyzers and use the fact that oxygen is paramagnetic (it is
attracted to a magnetic field). Diamagnetic gases are repelled and not measured. Nitric oxide
is also paramagnetic (but not nitrous oxide). Some use a null-deflection method. This is
different to the null hypothesis (which is a statistical concept)!

Question 104: TTTTT


Gas chromatography can be used for measuring concentrations of all of these compounds.

Question 105: TTFTF


Infrared spectroscopy requires the presence of two dissimilar atoms in the gas (CO2, N2O).
It has a rapid response time and is frequently used for end tidal analysis. Note, side-stream
analyzers do have a delay in measurement, but it is associated with the length of the tubing
and not the actual analyzer. As oxygen has similar atoms, IR spectroscopy cannot be used to
measure it.

Question 106: FFTTT


Mass spectrometers can measure tiny samples. The analyte is ionized and is often broken
into fragments. Modern spectrometers have response times of approx. 100 ms. They are,
however, large and expensive. Mass spectrometry uses the principle of deflecting compo-
nents of a gas mixture in magnetic field according to their mass and charge. Samples are
ionized by an electron beam, which is produced by a heated filament. Radioactivity is not
inherent in the functioning of mass spectrometry.

Question 107: FFTTF


Infrared spectroscopy requires dissimilar atoms and so oxygen cannot be measured.
The piezoelectric effect is seen where a crystal is made to oscillate with a specific frequency
by an alternating potential. Dissolved anaesthetic agents in a coating on the crystal change
the resonant frequency according to their concentration, but is does not differentiate
between agents. The van Slyke apparatus can measure both oxygen and carbon dioxide.
Polarographic (Clark) electrodes are usually used for measurement of oxygen in liquids, but
they can also be used in gas mixtures. Paramagnetic analyzers can measure oxygen (and
nitric oxide, but not nitrogen or nitrous oxide).

Downloaded from https://www.cambridge.org/core. University of Edinburgh, on 19 Aug 2019 at 13:21:26, subject to the Cambridge Core terms of
use, available at https://www.cambridge.org/core/terms. https://doi.org/10.1017/9781108566100.010
Chapter 4b: Physics Answers 323

Question 108: FTTTF


The Raman effect uses the fact that when light is shone through a gas, some of the energy
from that light is absorbed by the gas and changes the frequency with which the bonds
between the molecules vibrate (the wavelength usually decreases as energy is absorbed).
Different lasers will produce light at different frequencies. Different filters in the detectors
can be used to detect different agents, including sevoflurane and helium.

Question 109: TTTFT


The mass spectrometer detects chemicals according to mass. Although CO2 and N2O
have the same molecular mass (44), some molecules will break up and it is this pattern
of ‘cracking’ that will allow individual agents to be identified. Not all mass spectro-
meters use a quadrupole. This particular technique allows ions of specific masses to be
removed from the stream for analysis. Sampled gases may well be chemically altered
(the cracking mentioned earlier) and so they should not be returned. The sample is
ionized, and both positively and negatively charged particles will result. In anaesthesia,
mass spectrometers with a rapid response time of less than 100 ms have been used for
breath analysis of expired air. However, the cost and size of the mass spectrometer
make it impractical for clinical use.

Question 110: FFTFT


The definitions in a and b are the opposite. Photoacoustics uses the sounds produced from
a substance when light is shone on it. The piezoelectric effect uses the change in frequency of
vibration of a crystal. Refractometry (also called interferometry) is the standard way of
checking the performance of vaporizers. It uses the principle that as light passes through
a gas, it is slowed and refracted proportionally to the concentration of that gas. Carbon
dioxide and nitrous oxide have the same molecular mass (44) and so it is these fragmenta-
tion products that allow them to be differentiated. See the answer for Question 109 for
further explanation of ‘cracking’.

Question 111: TTFFF


The Clark (or polarographic) electrode has a silver/silver chloride anode and a platinum
cathode. The electrolyte solution is potassium chloride, however other solutions have also
been used, such as potassium chloride, potassium bromide, sodium chloride and sodium
hydroxide. Potassium hydroxide is used in the fuel cell. Electrons, water and oxygen
combine at the cathode (not anode) to form hydroxyl ions.

Question 112: FTTTT


Transcutaneous oxygen measurement uses a modified polarographic electrode. It is less
accurate than blood gas analysis and its response time is about a minute. The skin is heated
to 43–44 °C to dilate the blood vessels. It does measure capillary oxygen tensions, but these
lag behind arterial tensions, causing further inaccuracy.

Downloaded from https://www.cambridge.org/core. University of Edinburgh, on 19 Aug 2019 at 13:21:26, subject to the Cambridge Core terms of
use, available at https://www.cambridge.org/core/terms. https://doi.org/10.1017/9781108566100.010
324 Chapter 4b: Physics Answers

Question 113: FTFFF


Paramagnetic analysis is a common way of measuring oxygen in gases. It is used in most
modern anaesthetic machines, but cannot be used to measure intra-arterial oxygen con-
centration directly. The Raman effect (change in wavelength of vibration of molecules when
light is shone on them) can be used. IR absorption requires two dissimilar atoms in
a molecule, so oxygen cannot be measured this way. Mass spectrometry can be used and
does not require dissimilar atoms.

Question 114: FFTTT


In spite of having many disadvantages, the pulmonary artery catheter is still considered
as the gold standard method of cardiac output monitoring; 50 cm would be far too short,
and the standard adult line is 110 cm. A thermistor usually lies about 3.7 cm from the tip,
not at the tip itself.

Question 115: TFTTT


The fuel cell has a lead anode and a gold mesh cathode (Clark electrode uses platinum) with
a potassium hydroxide solution as an electrolyte. The same oxygen + electrons + water
becoming hydroxyl ions occurs in both devices. Temperature compensation is required.
It can measure oxygen in gaseous mixtures.

Question 116: FFFTF


Pulse oximetry is reliable to about 70%. Below this level, values are extrapolated. Carbon
monoxide toxicity tends to give falsely high readings. Methaemoglobinaemia gives falsely
low readings. They only give information on oxygenation. Quality of ventilation is best seen
in capnography. A well preoxygenated patient may have 5 minutes of apnoea before
a change in oxygen saturations is detected. The relative absorbance of light at different
wavelengths is the principle behind the device. At the 660 nm (visible red) point, deoxy-
haemoglobin absorbs more light. At the 940 nm (infrared) point, this is reversed. It can be
used at any altitude, but it must be borne in mind that ‘normal’ values at altitude are much
lower.

Question 117: FFTTT


Pulse oximetry only provides information on the adequacy of oxygenation. It gives no
information on the quality of ventilation, for example. It is not a substitute for arterial
analysis, although it is often used to guide clinicians on when analysis is required. It is
almost universally used in early warning score charts as it is an easily obtained and
reproducible measurement. A fall is frequently significant, but a lack of fall can be falsely
reassuring. If provided (and not all oximeters provide a graphic display), the shape and
quality of the waveform will give valuable extra information. Hypovolaemia will cause
a smaller signal, which is less reliable. Also the patient is likely to have peripheral vasocon-
striction which may result in an abnormally low value.

Downloaded from https://www.cambridge.org/core. University of Edinburgh, on 19 Aug 2019 at 13:21:26, subject to the Cambridge Core terms of
use, available at https://www.cambridge.org/core/terms. https://doi.org/10.1017/9781108566100.010
Chapter 4b: Physics Answers 325

Question 118: TFTFF


Beer’s law states that the absorption by a given thickness of a substance of given concentra-
tion is the same as that of twice the thickness with half the concentration. Lambert’s law
states that each layer of the same thickness absorbs the same proportion. These laws
underpin all spectrophotometric devices. The diodes in the oximeter flash several hundred
times a second. The standard wavelengths used are 660 nm and 940 nm, but more modern
devices may also use other wavelengths. One detector only is required as long as it can detect
all the wavelengths used. When both diodes are off, ambient light can be measured and
compensation applied. The difference in absorption is between oxy- and deoxyhaemoglo-
bin. Carboxyhaemoglobin is a source of error and tends to make the device over-read.

Question 119: FTTFF


The Bunsen solubility coefficient is the volume of gas (not liquid), corrected to STP, which
dissolves in one unit volume of the liquid (not gas) at the temperature concerned, where the
partial pressure of the gas above the liquid is 1 standard atmosphere. The solubility will
change with temperature, not the coefficient.

Question 120: FTTTT


The colligative properties of a solution are those that depend on the osmolarity of that
solution. These are:
• Increase in osmotic pressure
• Depression (not elevation) of freezing point
• Reduction in vapour pressure (Raoult’s law)
• Elevation (not depression) of boiling point
Osmolarity is related to the number of osmotically active particles, not their size or mass.
Some physical properties of a solution are altered by its osmolarity; these are known as
colligative properties. The presence of solute particles within a solvent causes a reduction in
saturated vapour pressure (SVP) and freezing point; 1 mole of solute added to 1 kg of water
causes a reduction in freezing point of 1.86 °C. Boiling point and osmotic pressure are
increased.

Question 121: TTTFF


Graham’s law states that the rate of diffusion of a gas is inversely proportional to the square
root of its molecular weight. Fick’s law of diffusion states that gas transfer across
a membrane is directly proportional to the concentration gradient, surface area and diffu-
sion constant, but inversely proportional to membrane thickness.

Question 122: TTTTT


These are standard definitions.

Downloaded from https://www.cambridge.org/core. University of Edinburgh, on 19 Aug 2019 at 13:21:26, subject to the Cambridge Core terms of
use, available at https://www.cambridge.org/core/terms. https://doi.org/10.1017/9781108566100.010
326 Chapter 4b: Physics Answers

Question 123: FTFTT


When liquids change to gas they require heat energy, this process is termed the latent heat of
vaporization; heat energy is utilized as kinetic energy to aid the change of phase. Saturated
vapour pressure therefore is affected by changes in temperature due to the relationship
between heat and kinetic energy; the higher the temperature the greater the SVP and kinetic
energy. Ambient pressure, however, causes no such change. A vapour occurs when
a gaseous substance is below its critical temperature, above this point it can only be a gas.

Question 124: FTFTT


Critical temperature can be defined by the temperature above which a vapour cannot be
liquefied; above this temperature the substance is a gas, below it, a vapour. The pressure
required to liquefy a vapour at its critical temperature is its critical pressure. A vapour
therefore can always theoretically be compressed to its liquid state. Oxygen has a critical
temperature of –119 °C, therefore at room temperature it will always exist as a gas.

Question 125: TFFFF


Desflurane has a boiling point of 23 °C, therefore it is highly volatile at room temperature.
Because of this, small changes in ambient temperature result in clinically significant
fluctuations in its saturated vapour pressure – rendering plenum vaporizers unsuitable for
use with this agent. A TEC 6 desflurane vaporizer is electronically controlled, actively
heating to 39 °C and 2 atm of pressure. Desflurane is injected into the fresh-gas flow,
calibrated according to flow rates.

Question 126: FFFFF


Modern plenum or temperature-compensated vaporizers are designed to be agent specific
and thus incorporate an agent-specific filling system. Historically, vaporizers suffered from
reduced output with prolonged anaesthetic times due to the effects of latent heat of
vaporization. Thus, modern types are designed to have a high heat capacity and incorporate
heat sinks, often copper. Bimetallic strips are present to increase the flow to the vapour
chamber as temperature decreases. Wicks and baffles are included because they increase the
surface area and ensure full vapour saturation, not a constant rate of vaporization.

Question 127: FTTFT


The clinical effect of volatile agents is determined by the agent partial pressure within
tissues, which is directly related to the agent saturated vapour pressure (SVP). Dalton’s law
and SVP are unaffected by ambient barometric pressure. Modern plenum vaporizers are
temperature compensated, and agent specific. Output setting (often via a dial) represents
agent partial pressure, not percentage. Because of this feature they automatically compen-
sate for changes in barometric pressure associated with altitude. Experimental data suggest
that they in fact slightly overcompensate.
An electronic desflurane vaporizer generates a vapour pressure of 2 atm inside.
The number on the desflurane vaporizer dial directly reflects the percentage that will be
delivered, regardless of ambient pressure. At high altitude, what is delivered to the patient is

Downloaded from https://www.cambridge.org/core. University of Edinburgh, on 19 Aug 2019 at 13:21:26, subject to the Cambridge Core terms of
use, available at https://www.cambridge.org/core/terms. https://doi.org/10.1017/9781108566100.010
Chapter 4b: Physics Answers 327

a percentage of a reduced ambient pressure; therefore, the partial pressure of desflurane in


the alveoli will be much less that it would be at sea level.

Question 128: FFTTF


Isotopes are atoms of a single element with the same number of protons, but differing
numbers of neutrons. Carbon-12 and carbon-14 are isotopes, one with six neutrons and one
with eight. Technetium-99m is a nuclear isomer of technetium-99, commonly used in
diagnostic radiology. Following decay of an isotope, a daughter nuclide is produced. All
types of electromagnetic radiation are included on the electromagnetic spectrum. γ-Rays,
X-rays, and the higher ultraviolet part of the electromagnetic spectrum are ionizing. They
possess enough energy to remove tightly bound electrons from the orbit of an atom, causing
atoms to become charged or ‘ionized’.

Question 129: TTTFF


Radiation is quantified by two derived SI units: sievert (Sv) and gray (Gy). One gray is the
deposit of one joule of radiation energy in one kilogram of tissue. The sievert serves a dual
purpose: representing dose and serving to quantifying biological effect, commonly referred
to as dose equivalent radiation. Levels of biological damage are associated with total dose
exposure. High doses result in cell death, lower doses in cell damage and carcinogenesis.
A chest X-ray is equivalent to 0.02 mSv, a CT abdomen/chest 8–10 mSv. The misuse of
radiation could lead to prosecution under The Health and Safety Act. The UK average
annual natural background radiation is 2.2 mSv or 2.7 mSv if artificial factors are taken into
consideration.

Question 130: TFFFT


Critical temperature is defined as the temperature above which a substance cannot be
liquefied, no matter how much pressure is applied. It applies only to single gases; when
there is a mixture of gases such as Entonox (oxygen and nitrous oxide) the term ‘pseudo-
®
critical’ temperature is used. The pseudocritical temperature of Entonox depends on its ®
pressure. It is –5.5 °C at 117 bar (highest), –7 °C in a cylinder at 137 bar and –30°C in
a pipeline at 4 bar. It is true that hydrogen obeys the gas laws most closely and is therefore
the gas used for measurements in the international temperature scale. The filling ratio is
a description of how much gas is used to fill a cylinder and is defined as the mass of
substance (gas) in a cylinder divided by the mass of water the cylinder could contain. It is
higher in temperate climates than tropical climates, due to the risk of ambient temperature
exceeding the critical temperature, thereby causing the gas pressure in the cylinder to rise
and risking cylinder explosion.

Question 131: FTTFF


The gas laws consist of three primary laws: Boyle’s law, Charles’ law and the third perfect gas
law, combining together to make up the ideal gas law. The constant in each of the laws can
be remembered by the following: Boyle = boil = temperature, (Prince) Charles = P for
pressure, leaving the third law constant, which must be V, volume. For a gas to change state,
heat energy must be added or taken away, however a change of state can also occur without

Downloaded from https://www.cambridge.org/core. University of Edinburgh, on 19 Aug 2019 at 13:21:26, subject to the Cambridge Core terms of
use, available at https://www.cambridge.org/core/terms. https://doi.org/10.1017/9781108566100.010
328 Chapter 4b: Physics Answers

a gas exchanging heat energy with its environment: an adiabatic change. An isotherm is
a curved line relating pressure to volume for a substance at different temperatures.

Question 132: FTTFT


Transducers convert one form of energy into another form of energy, not just specifically
heat. In CVP or arterial pressure monitoring transducers convert mechanical energy
(pressure of the blood against a strain gauge) into electrical energy, allowing
a conventional, useable figure to be displayed on a monitor. Resonance and damping act
to alter both the displayed systolic and diastolic figures as well as the mean blood pressure,
although to a lesser extent. An oscilloscope is used to detect changes in a signal over time
and is commonly used to display and analyze waveforms. Before the digital revolution
oscilloscopes used cathode ray tubes to display a signal, but these have largely been super-
seded by digital versions.

Question 133: TTFFT


A fall in pressure at the narrowest point of a Venturi tube is known as the Bernoulli effect and it
is used in many different devices, including the Venturi oxygen mask. Due to the drop in
pressure (potential energy) there is an increase in fluid velocity (kinetic energy) because the total
energy of the system must remain constant. This allows fluid or gas to be entrained at the
constriction. In the Venturi mask, air is entrained, causing dilution of the oxygen to give
a clinically useful concentration reaching the patient. Laplace’s law allows the calculation of
pressure from the tension and radius of a hollow distensible structure. It states that transmural
pressure is equal to the wall tension divided by the radius of the structure. Fluid flow can be
described as laminar or turbulent. Flow is more likely to be turbulent if the velocity increases,
such as at a constriction in a tube. Reynold’s number can be calculated from the velocity,
diameter, density and viscosity of a fluid, and is used to give a figure to show if laminar or
turbulent flow is most likely to be present. In laminar flow, the flow is proportional to the
pressure gradient. However, in turbulent flow, the flow is proportional to the square root of the
pressure gradient.

Question 134: FFFTT


Pressure can be described as the force of an object applied over a surface and is measured in
pascals (Pa). Absolute pressure is the sum of gauge plus atmospheric pressures. Generally,
atmospheric pressure is ignored; hence a full oxygen cylinder has a gauge pressure of 137 bar
but an absolute pressure of 138 bar (as atmospheric pressure is 1 bar). A Bourdon gauge
contains no fluid, hence the term ‘aneroid’ gauge, because it measures gauge pressure
relative to atmospheric pressure. Anaesthetic machines contain many types of valve,
including pressure relief, inspiratory and expiratory, and pressure-reducing, to modify
®
pressure within the machine. Entonox utilizes a demand valve that contains a pressure-
reducing valve, preventing continual flow from the cylinder and adding an important safety
aspect as the patient controls administration.

Downloaded from https://www.cambridge.org/core. University of Edinburgh, on 19 Aug 2019 at 13:21:26, subject to the Cambridge Core terms of
use, available at https://www.cambridge.org/core/terms. https://doi.org/10.1017/9781108566100.010
Chapter 4b: Physics Answers 329

Question 135: TFTFT


Biological signals can be detected by appropriate electrodes and recoded as an ECG, EMG or
EEG. Fourier analysis allows a complex wave pattern such as an ECG to be broken down to
a collection of pure sine waves. The size of the ECG signal detected is 1–2 mV, rather than
90 mV, which is the resting membrane potential of a cardiac myocyte. Conversely, EEG
potentials are much smaller at only 50 μV, whereas EMG potentials can range between
100 μV and multiple mV.

Question 136: TTTFT


Choice of electrode is important to enable accurate recording of a biological signal, as well as
reducing polarization and interference. A galvanometer is an analogue meter utilizing
a pointer needle, which is deflected in response to an electric current flowing through its
magnetic field. Unlike digital meters it is a useful device for observing trends, however it is
difficult to display rapidly changing signals like EMG potentials. As biological signals tend
to be very small, amplifiers are used to augment a signal, as well as eliminating any artefact
or interference. Common-mode rejection is the term used to describe the process of how
differential signal amplifiers work, whereby interference common to both electrodes is
eliminated, thus producing a clear signal.

Question 137: FTTFT


The range of frequencies in an EEG is narrower than in an ECG, which is between 0.5 and
100 Hz. Nerve stimulators are used to assess transmission across the neuromuscular
junction. When using peripheral nerve stimulators the negative black electrode is posi-
tioned over the nerve (‘negative to nerve’) and the positive red electrode a few centimetres
away from this. A monophasic square-waveform shape avoids repetitive nerve firing.

Question 138: FFFFT


Gas chromatography is indeed a technique used for gas analysis, however it is very slow and
much quicker techniques can be used for rapid analysis in hospitals, including spectrometry
and the fuel cell. The sample to be analyzed is rapidly injected via the inlet into a stream of
gas: the mobile phase. The constituents of the mixture pass through a column, the stationary
phase, at a speed depending on their relative solubility in the two phases. Eventually all
constituents will arrive at the detector, giving rise to peaks, allowing calculation of the
amount of each component present in the mixture. As gas chromatography does not allow
continuous analysis it can be used for one-off measurements of the components in the
theatre environment.

Question 139: TTFFT


As gas solubility varies with temperature it is important that the temperature of the column,
usually a glass tube containing calcium carbonate or silica-alumina particles, is kept con-
stant. The mobile phase consists of a stream of inert carrier gas such as helium or argon,
which is used to carry the sample gas. The retention time varies for each individual gas and
although it allows calculation of component quantity it does not allow identification; the

Downloaded from https://www.cambridge.org/core. University of Edinburgh, on 19 Aug 2019 at 13:21:26, subject to the Cambridge Core terms of
use, available at https://www.cambridge.org/core/terms. https://doi.org/10.1017/9781108566100.010
330 Chapter 4b: Physics Answers

types of substance must be preknown. Liquids may also be analyzed by chromatography,


although it is important that a suitable detector is used, for example the flame ionization
detector.

Question 140: FFTTT


MCQs on the topic of the pin-index system are very common. This international system is
a safety feature of the anaesthetic machine, preventing the accidental connection of the
wrong cylinder to the yoke. The following gases have the corresponding pin-index
positions:
Air: 1 and 5
Oxygen: 2 and 5
Nitrous oxide: 3 and 5
Carbon dioxide: 1 and 6
Entonox : 7 ®
Question 141: FFTTF
Questions on nitrous oxide cylinders are interesting, if not for the many pitfalls one may fail
to spot! Firstly, nitrous oxide cylinders are French blue (both body and shoulder). Unlike an
oxygen cylinder, nitrous oxide cylinders do not obey Boyle’s law. Nitrous oxide cylinders
contain both liquid and vapour because the critical temperature of nitrous oxide is 36.5 °C.
As vapour is used, it is replaced by vapour from the liquid below, therefore the pressure
exerted by the vapour phase does not change. Hence, the cylinder pressure remains at
4400 kPa until all of the liquid is used and only vapour remains. Therefore, the cylinder
pressure gauge does not reflect the contents of the cylinder.

Question 142: FTTTF


®
The pressure within an Entonox cylinder is 137 bar or 137 kPa. Despite the pascal (Pa)
being the SI unit of pressure, most pressure measurements are given in bar or kPa.
The pseudocritical temperature of Entonox is –7 °C, such that below this temperature,
®
as per the definition of pseudocritical temperature, there is separation of gas mixtures.
The Poynting effect describes the situation whereby oxygen and nitrous oxide separate
within an Entonox cylinder below –5.5 °C. Within the cylinder itself, there is a hypoxic
®
liquid mixture at the bottom and a high-concentration oxygen–gas mixture above it.
Cylinders should be stored horizontally to prevent separation of the gas mixture by
increasing the area for diffusion. Agitating unattached Entonox cylinders poses minimal ®
risk (other than manual handling), and it could be argued that thorough mixing of the
contents is a good thing.

Question 143: TTTTT


Unfortunately, this question just requires pure recollection of factual knowledge which
although seems obscure, pays dividends during transfers where you can enlighten all those
around you as to the importance of Schrader sockets!

Downloaded from https://www.cambridge.org/core. University of Edinburgh, on 19 Aug 2019 at 13:21:26, subject to the Cambridge Core terms of
use, available at https://www.cambridge.org/core/terms. https://doi.org/10.1017/9781108566100.010
Chapter 4b: Physics Answers 331

Question 144: FTTFF


A vacuum-insulated evaporator is a double-walled steel tank. Perlite is used as insulation
within the two layers of steel. The critical temperature of oxygen is –118 °C. Liquid oxygen
within a vacuum-insulated evaporator is stored at –150 °C to –170 °C. The pipeline pressure
of oxygen is 4 bar or 400 kPa.

Question 145: FTFTF


The Doppler effect is an increase in the frequency of a signal when the source signal moves
towards the observer, because the wavelength of the sound becomes shorter. The converse is
also true as there is a decrease in the frequency of a signal when the source signal moves
away, due to the lengthening of the sound waves. For example, an ambulance siren’s pitch
gets higher as it approaches you and gets lower as it drives away. Although the frequency of
this signal has not actually changed, it is the relationship between frequency and wavelength
that means our ears detect the shorter wavelength as an increase in frequency and therefore
a higher-pitched sound. Doppler principles, including Doppler shift, can be used to measure
velocities and flow rates in medicine, such as in cardiac echo, using the Doppler equation.
Colour can be superimposed on an ultrasound image to show direction of flow; red
illustrates blood moving towards the probe and blue is blood moving away.
The ultrasound probe comprises tiny piezoelectric crystals that act as transducers convert-
ing electrical energy to mechanical energy and back again. Many biological substances
exhibit piezoelectric properties, including collagen, enamel and DNA.
2ft V cos θ
fD ¼
c

where fD = Doppler shift frequency, ft = frequency of transmitted beam, V = velocity of flow,


θ = angle of incidence between ultrasound beam and the direction of flow, c = speed of
sound in tissue.

Question 146: TFTFT


When any type of wave reaches an interface between two different substances, a proportion
of the wave continues to be transmitted through the substance, but part of the wave is
reflected back. This variability depends on the properties of the material through which the
wave is travelling. In the use of sound waves such as ultrasound, this is due to a difference in
density between the two substances. Sound travels faster in liquids than air, and faster still in
solids. In an ideal gas temperature is the only variable that affects sound speed.

Question 147: TTFTT


The range of frequencies heard by humans is typically between 20 and 20000 Hz.
Ultrasound refers to frequencies greater than 20 000 Hz and is therefore above the range
of human hearing. Medical ultrasound utilizes piezoelectric crystals in the head of the probe
to transmit ultrasonic waves through tissues. As different tissues have different densities,

Downloaded from https://www.cambridge.org/core. University of Edinburgh, on 19 Aug 2019 at 13:21:26, subject to the Cambridge Core terms of
use, available at https://www.cambridge.org/core/terms. https://doi.org/10.1017/9781108566100.010
332 Chapter 4b: Physics Answers

part of the wave is transmitted through that tissue and part is reflected back to the probe.
Here, it is transduced or converted into an electrical signal for analysis. Gel is used on the
ultrasound probe to reduce the difference in densities of the substance that the ultrasound
wave has to travel through, therefore enhancing wave transmission and signal quality by
reducing attenuation. When body tissues are exposed to ultrasound, some of the energy is
absorbed. At lower power levels, this is generally of no consequence but at higher power
levels the pressure in the ultrasound wave can fall to zero at the bottom of the waveform,
thus creating bubbles in tissues which rapidly collapse or cause cavitation. Cavitation is
generally an undesirable effect, however it is becoming popular in the beauty industry as an
alternative to liposuction.

Question 148: FTTTT


Human hearing range varies between individuals and at different stages of life but is
generally between 0.02 and 20 kHz (remember that 1 Hz is equal to 0.001 kHz). Medical
ultrasound is typically very high frequency sound and ranges between 2.5 and 15 MHz.
Ultrasound is based on the pulse-echo system. An ultrasound ‘pulse’ wave is transmitted
through tissues and ‘echoed’ back to the transducer when it hits a tissue of different density
and is displayed on screen as a greyscale. In tissues with no internal interfaces, e.g. from the
wave travelling through fluid, no echoes will be returned and therefore they will appear
black on the display. High-frequency ultrasound probes are used to image superficial body
tissues because they offer high resolution, but limited depth of penetration. Deep tissues are
imaged using low-frequency probes.

Question 149: FTFFF


Different tissue densities determine the proportion of an ultrasound wave that is either
reflected or transmitted by a tissue. The velocity of sound in tissue depends on the tissue
density, but is generally taken as 1540 m.s–1, the same velocity as sound in water. When an
ultrasound wave is reflected back to the surface and transduced to allow on-screen display
a greyscale is used to demonstrate different tissue types. The more solid a structure the
whiter it appears, thus bone will appear white and fluid black. NICE recommends that
‘2-D imaging ultrasound guidance should be the preferred method when inserting a central
venous catheter into the internal jugular vein and should be considered in most clinical
situations where CVC insertion is necessary’. Oesophageal Doppler is a minimally invasive
alternative to a pulmonary artery catheter. It is used in cardiac output monitoring and
measures blood velocity in the descending thoracic aorta. The equation distance = velocity ×
time can be used to derive the depth of tissue. The depth is half of the distance the beam has
travelled.

Question 150: TFFTT


The use of cardiac output monitoring is becoming common practice in anaesthesia and
intensive care, given that the trend in their values forms one of the cornerstones of goal-
directed fluid therapy.

Downloaded from https://www.cambridge.org/core. University of Edinburgh, on 19 Aug 2019 at 13:21:26, subject to the Cambridge Core terms of
use, available at https://www.cambridge.org/core/terms. https://doi.org/10.1017/9781108566100.010
Chapter 4b: Physics Answers 333

The best position for the probe is at the level of T5–T6, this corresponds to the distal
oesophagus rather than the upper oesophagus. Although frequently suffering from inter-
ference in the presence of diathermy, the probe can still be used safely. However, the use of
oesophageal Doppler is contraindicated in the presence of thoracic aortic aneurysms, severe
facial trauma and oesophageal varices. The probe uses a high frequency of 4 MHz to
measure blood flow.

Downloaded from https://www.cambridge.org/core. University of Edinburgh, on 19 Aug 2019 at 13:21:26, subject to the Cambridge Core terms of
use, available at https://www.cambridge.org/core/terms. https://doi.org/10.1017/9781108566100.010
Chapter
Clinical Anaesthesia Questions

5a
Question 1
A 35-year-old male presents with a perforated duodenal ulcer, requiring an emergency
laparotomy. He tells you that his father had a reaction to anaesthesia with high tempera-
tures. With regards to malignant hyperpyrexia (MH):
a. The patient’s risk of having MH is 25%
b. A mutation on chromosome 3 is responsible
c. Abnormal ryanodine receptors prevent calcium release into the cytoplasm
d. Presentation may be delayed
e. Previous history of uneventful anaesthesia is reassuring

Question 2
You are on call for emergency theatres and preassess a 6-year-old child coming to theatre for
a suspected appendicitis. You plan a rapid sequence induction. What are essential compo-
nents of a rapid sequence induction?
a. Cricoid pressure
b. Predetermined dose of induction agents
c. Cuffed endotracheal tube
d. Capnography trace
e. Suction

Question 3
NAP3 investigated the perioperative complications of central neuraxial blockade (CNB).
Which of the following are true?
a. Staphylococcus aureus is an organism commonly associated with epidural abscesses
b. Major complications are more commonly seen following spinal anaesthetics than with
epidurals
c. Back pain is a more common symptom of vertebral canal haematoma than leg weakness
d. The incidence of permanent injury following perioperative epidural in adults is less than
8 per 100 000
e. The incidence of paraplegia and death following CNB is approximately 1 per 100 000

334
Downloaded from https://www.cambridge.org/core. University of Edinburgh, on 19 Aug 2019 at 13:21:27, subject to the Cambridge Core terms of
use, available at https://www.cambridge.org/core/terms. https://doi.org/10.1017/9781108566100.011
Chapter 5a: Clinical Anaesthesia Questions 335

Question 4
The following are considered cyanotic congenital heart diseases:
a. Coarctation of the aorta
b. Tetralogy of Fallot
c. Double outlet right ventricle with pulmonary stenosis
d. Atrioventricular septal defect
e. Transposition of great arteries with large VSD

Question 5
Regarding metabolic equivalents (METs):
a. One MET represents the oxygen consumption of an adult at rest
b. One MET is equivalent to 2.5 ml.kg–1.min–1 oxygen consumption
c. Patients should be able to perform more than 5 METS to undertake major surgery
d. Patients should be able to climb at least one flight of stairs to undertake major surgery
e. MET values of activities range from 0.9 to 23 ml.kg–1.min–1

Question 6
A 42-year-old male is on the CEPOD list for an acute appendicectomy. Preassessment
reveals a history of myasthenia gravis (MG). The following statements regarding MG are
true:
a. In the performance of a rapid sequence induction the dose of suxamethonium remains
unchanged
b. The underlying pathophysiology is decreased release of acetylcholine from the pre-
synaptic nerve terminal
c. Myasthenic syndrome and myasthenia gravis are interchangeable terms
d. Neuromuscular monitoring will demonstrate improved post-tetanic count following
tetanic stimulation
e. Arterial line siting is recommended for general anaesthesia as autonomic dysfunction is
commonly seen

Question 7
A 72-year-old woman presents to your pain clinic with a 12-month history of pain along her
left chest wall following an episode of shingles. Her GP has given her a diagnosis of
postherpetic neuralgia (PHN). With regard to PHN:
a. It is rare in younger patients (<50 years)
b. It most commonly involves thoracic dermatomes
c. In 20% of patients the ophthalmic division of the trigeminal nerve is involved
d. It occurs in 5% following herpes zoster
e. It is more common in older men

Downloaded from https://www.cambridge.org/core. University of Edinburgh, on 19 Aug 2019 at 13:21:27, subject to the Cambridge Core terms of
use, available at https://www.cambridge.org/core/terms. https://doi.org/10.1017/9781108566100.011
336 Chapter 5a: Clinical Anaesthesia Questions

Question 8
The following are accepted strategies to increase the viability and number of transplantable
organs:
a. Hormone resuscitation
b. Fluid loading to achieve a central venous pressure of 8–12 mmHg
c. Ventilation with a high positive end expiratory pressure of 10–15 cmH2O
d. Vasopressin used at doses of 0.5–4 U.h−1 in septic patients
e. Noradrenaline as a first-line measure in treating hypotension

Question 9
A 30-year-old male is involved in a house fire. Which of the following are true with regards
to a total body surface area (TBSA) percentage burn?
a. Burns to the anterior surface of his chest, the whole of his right arm, his perineum, and
the anterior surface of his right leg would cause a 30% TBSA burn
b. Burns to both arms and his back would result in a 45% TBSA burn
c. Burns to his anterior chest, his back, and his perineum would cause a 37% TBSA burn
d. Burns to both legs and his back would cause a 54% TBSA burn
e. Burns to his head and perineum would cause a 15% TBSA burn

Question 10
You have been asked to anaesthetize a gentleman with a BMI of 60 for a gastric banding
procedure. Which of the following statements are true?
a. Laparoscopic techniques have a lower morbidity and mortality in the short term
compared to open techniques
b. A raised BMI in isolation is a predictor of difficult intubation
c. The incidence of obstructive sleep apnoea in bariatric patients approaches 80%
d. Peripheral nerve injuries are more common in this group of patients
e. Suxamethonium, if used, should be dosed on ideal rather than actual body weight

Question 11
Regarding paediatric day-case services:
a. Ex-premature neonates are not suitable in the first three months of life
b. Anaesthetists should have child protection training
c. Access to paediatricians is essential
d. A centre must have the ability to ventilate children postoperatively
e. Play specialists must be available on units

Question 12
Features suggestive of a difficult intubation include:
a. Ability to protrude mandibular incisors in front of maxillary incisors
b. The presence of a high-arched palate
c. Sternomental distance >12.5 cm

Downloaded from https://www.cambridge.org/core. University of Edinburgh, on 19 Aug 2019 at 13:21:27, subject to the Cambridge Core terms of
use, available at https://www.cambridge.org/core/terms. https://doi.org/10.1017/9781108566100.011
Chapter 5a: Clinical Anaesthesia Questions 337

d. Previous radiotherapy to the neck


e. Pierre Robin syndrome

Question 13
Pain after thoracic surgery can be considerable and forms one of the commonest causes of
postoperative chronic pain. Paravertebral nerve blocks may provide adequate analgesia for
thoracic surgery. Regarding placement of paravertebral blocks:
a. The most prominent cervical spinous process is at C7
b. The lower tip of the scapula is usually in line with T6
c. Total spinal anaesthesia is a recognized common complication
d. Paravertebral muscle pain may occur
e. The paravertebral space is defined by the vertebral body, the intervertebral disc and
intervertebral foramen medially, visceral pleura anterolaterally and the superior costo-
transverse ligament posteriorly

Question 14
A 79-year-old male suffered an out of hospital VF arrest. Following return of sponta-
neous circulation (ROSC), he had three coronary stents inserted in the cath lab. On his
admission to ICU, his ABP was 75/32 mmHg, HR 96 min–1 in sinus rhythm and CVP
12 cmH2O. The following are true regarding his management on the ICU:
a. Dobutamine is the first line of treatment as the patient is in cardiogenic shock
b. Noradrenaline is a first-line treatment if serum lactate is elevated
c. Adrenaline will be useful for its vasoconstrictive properties
d. An intra-aortic balloon pump (IABP) should be considered early in this patient
e. Dobutamine will help in case of diastolic dysfunction

Question 15
With acute fatty liver of pregnancy (AFLP):
a. The incidence is higher in multiple pregnancies
b. Maternal mortality is approximately 50%
c. Regional anaesthesia is contraindicated
d. It can occur concurrently with pre-eclampsia
e. The radiological appearance of the liver may be normal

Question 16
You are assisting in the preassessment clinic and have been asked to review the CPET results
of a patient for pneumonectomy for lung cancer. The following are true regarding CPET in
thoracic surgery patients:
a. All patients undergoing pneumonectomy should have a CPET
b. Patients with peak VO2 above 15 ml.kg–1.min–1 can have resection up to
pneumonectomy

Downloaded from https://www.cambridge.org/core. University of Edinburgh, on 19 Aug 2019 at 13:21:27, subject to the Cambridge Core terms of
use, available at https://www.cambridge.org/core/terms. https://doi.org/10.1017/9781108566100.011
338 Chapter 5a: Clinical Anaesthesia Questions

c. Predicted postoperative peak VO2 measurements of less than 10 ml.kg–1.min–1 are


associated with significantly increased risk of mortality and morbidity after lung
resection
d. The ability to climb five flights correlates with a VO2 max >20 ml.kg–1.min–1
e. The anaerobic threshold is approximately 80% of VO2 max in untrained individuals

Question 17
Sickle cell disease is a common disorder of haemoglobin. Regarding sickle cell disease:
a. It results from a mutation on chromosome 13
b. The pathological amino-acid substitution is valine for glutamic acid
c. An abnormal β-globin chain is produced
d. ®
A Sickledex test will differentiate between sickle cell disease and sickle cell trait
e. Sickle cell trait confers a degree of protection against Plasmodium falciparum malaria

Question 18
You have a 3-year-old child listed for tonsillectomy. The following are true regarding the
paediatric airway:
a. The larynx is situated at C5–C6
b. The epiglottis is large and U-shaped
c. The airway is widest at the cricoid ring
d. Respiration is predominantly diaphragmatic
e. The functional residual capacity is lower than the closing capacity

Question 19
A 14-year-old is undergoing an urgent laparoscopic appendicectomy. Intraoperatively,
persistent tachycardia is noted with a rising end tidal CO2. The temperature has risen
from 37.8 to 38.5 °C in 20 minutes. Malignant hyperpyrexia is suspected. You should
immediately:
a. Increase the set respiratory rate
b. Ask the surgeon to stop operating and deflate the abdomen
c. Call for senior help
d. Administer 2 mg.kg–1 dantrolene IV stat
e. Cover the patient in ice

Question 20
Which of the following factors in a patient’s history would concern you with regards to
increased risk of aspiration?
a. Alcohol
b. Trauma
c. Dehydration
d. Diabetes
e. Cleft palate

Downloaded from https://www.cambridge.org/core. University of Edinburgh, on 19 Aug 2019 at 13:21:27, subject to the Cambridge Core terms of
use, available at https://www.cambridge.org/core/terms. https://doi.org/10.1017/9781108566100.011
Chapter 5a: Clinical Anaesthesia Questions 339

Question 21
You are a covering a renal list with local supervision. The list includes a living donor renal
transplant. With regard to the donor in a living donor renal transplant:
a. The presence of a consultant surgeon and consultant anaesthetist is mandatory during
the donor nephrectomy
b. Hypertension is a contraindication to donating a kidney
c. Diabetes mellitus is a contraindication to donating a kidney
d. Routine antibiotic prophylaxis is required during the nephrectomy
e. Invasive monitoring is usually appropriate

Question 22
Prostaglandin infusion is a useful medical treatment in the following congenital heart
conditions:
a. Transposition of the great arteries
b. Tricuspid atresia
c. Patent ductus arteriosus
d. Pulmonary stenosis
e. Large VSD

Question 23
The following volumes of local anaesthetic boluses are appropriate in a 60 kg man:
a. 20 ml of 2% lignocaine in an epidural top-up
b. 15 ml of 0.5% levobupivacaine in a femoral block
c. 10 ml of hyperbaric prilocaine 2% in spinal anaesthesia for a hip replacement
d. 30 ml of lignocaine 1% with adrenaline as infiltration in a hernia wound
e. 10 ml of lignocaine 1% with adrenaline for a ring block

Question 24
A 13-year-old boy presents for elective correction of idiopathic scoliosis. Use of intraopera-
tive neurophysiological monitoring using somatosensory evoked potentials (SSEPs) and
motor evoked potentials (MEPs) is planned. SSEPs and MEPs are significantly affected by:
a. Nitrous oxide
b. Volatile anaesthetic agents
c. Opioids
d. Hypothermia
e. Hypotension

Question 25
You preassess a 42-year-old female for a laparoscopic cholecystectomy. She informs you
that she ‘woke up’ during her last anaesthetic and asks about the risks of this happening
again. According to the 5th National Audit Project on accidental awareness during general
anaesthesia:

Downloaded from https://www.cambridge.org/core. University of Edinburgh, on 19 Aug 2019 at 13:21:27, subject to the Cambridge Core terms of
use, available at https://www.cambridge.org/core/terms. https://doi.org/10.1017/9781108566100.011
340 Chapter 5a: Clinical Anaesthesia Questions

a. Estimated incidence of awareness under general anaesthesia is 1:1900


b. Cardiothoracic anaesthesia and caesarean sections have the highest incidence of
awareness
c. Females had a higher incidence of awareness
d. Awareness incidence is highest during induction of anaesthesia
e. Higher ASA grade was associated with a higher incidence of awareness

Question 26
A 60-year-old male is ventilated on the ICU with a presumed diagnosis of Guillain–Barré
syndrome (GBS). Which of the following are true?
a. The early administration of antibiotics improves outcome in these patients
b. Whilst plasmapheresis may reduce the severity of GBS, the duration of illness remains
unaffected
c. Autonomic disturbance is a common feature in patients with more than five days of
symptoms
d. Exposure to Epstein–Barr virus is a risk factor in the development of GBS
e. Ventilatory support is required in the majority of patients with GBS

Question 27
A 42-year-old male with acromegaly is on your trauma list for an open reduction and
internal fixation of a fractured ankle. Regarding acromegaly:
a. Diagnosis with acromegaly is likely to have been made in his second decade
b. Performance of an ECG is unlikely to be of value
c. Meticulous attention to patient positioning is required due to the increased risk of an
unstable cervical spine
d. He is at increased risk of exophthalmos
e. Regional anaesthesia is contraindicated in this case

Question 28
A 22-year-old female is listed for a diagnostic laparoscopy. The gynaecology registrar
casually mentions that she has a history of a form of myotonic dystrophy. The following
is true in the management of this patient:
a. Use of suxamethonium is contraindicated due to the risk of severe hyperkalaemia
b. Local anaesthetic injected directly into muscles will not prevent muscle contraction
c. Persistent episodes of skeletal muscle contraction may lead to hypertrophy of flexor
muscle groups
d. Dantrolene should be considered in the treatment of severe muscle contraction
e. The condition commonly presents in the second decade of life

Question 29
You are asked to anaesthetize a patient with known cerebral palsy for a baclofen pump
insertion as part of their management of spastic paraparesis. Regarding cerebral palsy (CP):
a. A rapid sequence induction should be performed with a reduced dose of rocuronium

Downloaded from https://www.cambridge.org/core. University of Edinburgh, on 19 Aug 2019 at 13:21:27, subject to the Cambridge Core terms of
use, available at https://www.cambridge.org/core/terms. https://doi.org/10.1017/9781108566100.011
Chapter 5a: Clinical Anaesthesia Questions 341

b. The increased incidence of gastro-oesophageal reflux is as a result of lower oesophageal


sphincter tone
c. It accounts for up to one-third of cases of motor impairment in childhood
d. Up to half of affected children have some form of epilepsy
e. Pulmonary function tests would demonstrate an obstructive pattern

Question 30
A 92-year-old female is listed for a wide local excision of an angiosarcoma of the left leg plus
a split skin graft. The patient has refused a regional anaesthetic and therefore requires
general anaesthesia. The following are true:
a. A MAC value of 0.6 for isoflurane would be appropriate
b. Intravenous opioids, but not benzodiazepines, exhibit an age-related increase of their
elimination half-life
c. Use of TIVA is a relative contraindication due to the risk of greater cardiovascular
instability
d. Higher free-drug concentrations are as a result of reduced protein binding
e. The lower blood-gas solubility coefficient of desflurane reduces the risk of postoperative
cognitive dysfunction

Question 31
Ultrasound is commonly used in modern anaesthetic practice. Which of the following
statements are true?
a. Linear probes typically operate at frequencies between 5 and 18 MHz
b. The lower the frequency, the better the resolution of detail
c. The higher the frequency, the lower the penetration
d. Curvilinear probes operate at high frequencies allowing for greater tissue penetration
e. Time gain compensation increases gain at selected tissue depths

Question 32
A 32-year-old female with a history of migraine presents with a sudden onset severe
headache. She has recently changed job and admits to feeling increasingly stressed.
On examination she has slight weakness in her non-dominant hand. The following are true:
a. Brief neurological deficit is common in migraine
b. Migraine is more common in females
c. Pizotifen is second line in the acute treatment of migraine
d. Oramorph should be given early in the treatment of migraine
e. Neuroimaging is not indicated in this patient

Question 33
The following potential injuries and/or clinical consequences are correctly associated with
the relevant description of positioning for neurosurgery:
a. Foot drop: lateral position
b. Brachial plexus injury: prone position

Downloaded from https://www.cambridge.org/core. University of Edinburgh, on 19 Aug 2019 at 13:21:27, subject to the Cambridge Core terms of
use, available at https://www.cambridge.org/core/terms. https://doi.org/10.1017/9781108566100.011
342 Chapter 5a: Clinical Anaesthesia Questions

c. Venous air embolism: sitting position


d. Ocular complications: lateral position
e. Ischaemic optic neuropathy: use of a horseshoe headrest

Question 34
The following are true with regards to the difference in body surface area of a child
compared to an adult:
a. The Lund–Browder chart divides the body into areas of 9% total body surface area
(TBSA)
b. According to the Lund–Browder chart the head of a 1-year-old child makes up 17%
TBSA compared to 7% in an adult
c. One thigh of a 1-year-old child is half the TBSA of that of an adult
d. The neck and buttocks make up the same TBSA regardless of age
e. The Lund–Browder chart does not take into account amputated body parts when
estimating a percentage burn

Question 35
Laparoscopy is frequently used for diagnostic and therapeutic procedures. Which of the
following statements concerning laparoscopic surgery are true?
a. Shoulder pain is common after surgery as diaphragmatic irritation causes referred pain
over the C5–C7 dermatomes
b. CO2 is used as an insufflation gas as it is safer than medical air
c. Brain tumours are relative contraindications to laparoscopy
d. Typical insufflation pressures are 35–40 mmHg
e. Laparoscopy mandates general anaesthesia

Question 36
A 50-year-old male with Type II diabetes presents for a day-case hernia repair. Regarding
diabetic medications in patients for day-case surgery:
a. Metformin should be stopped 48 hours before surgery
b. Sulfonylureas should be omitted on the morning of surgery
c. Glucagon-like-peptide 1 analogues should be omitted on the day of surgery
d. Gliptins should be omitted on the day of surgery
e. Metformin should be restarted three days after surgery

Question 37
Regarding cerebrospinal fluid (CSF) analysis:
a. Low glucose and high protein is consistent with tuberculous meningitis
b. Low glucose may be consistent with viral meningitis
c. Raised lymphocytes may be a feature of bacterial meningitis
d. Normal CSF glucose is equal to serum levels
e. Normal CSF pressure may be up to 30 cmH2O

Downloaded from https://www.cambridge.org/core. University of Edinburgh, on 19 Aug 2019 at 13:21:27, subject to the Cambridge Core terms of
use, available at https://www.cambridge.org/core/terms. https://doi.org/10.1017/9781108566100.011
Chapter 5a: Clinical Anaesthesia Questions 343

Question 38
The following are appropriate starvation times in a patient with presumed normal gastric
emptying undergoing elective surgery:
a. Carbonated drinks: four hours
b. Breast milk: two hours
c. Water: two hours
d. Tea without milk: four hours
e. Formula milk: six hours

Question 39
The following features are true when considering heart block:
a. In bifascicular block, a right bundle branch block and first-degree atrioventricular block
are present
b. In isolated left anterior fascicular block, the QRS width is greater than 0.12 seconds
c. Left posterior fascicular block is associated with right axis deviation
d. Left posterior fascicular block is more common than left anterior fascicular block
e. Left bundle branch block is present in trifascicular block

Question 40
Implantable pacemakers have revolutionized the management of chronic cardiac arrhyth-
mias. In the five letter classification system for pacemakers, e.g. VVI00:
a. The first letter is the chamber sensed
b. The second letter is chamber sensed
c. The third letter refers to programmability
d. The fourth letter refers to programmability
e. The fifth letter refers to programmability

Question 41
Amniotic fluid embolism:
a. Amniotic fluid substances observed in the blood is pathognomonic
b. Has a mortality between 20 and 40%
c. Can present as cardiac arrest
d. Age over 35 years is a risk factor
e. Can present as an isolated coagulopathy

Question 42
The most valid single test for post-thoracotomy respiratory complications is the predicted
postoperative FEV1 percentage (ppo FEV1%). Regarding the ppo FEV1%:
a. For lobectomy, the calculation uses the number of bronchopulmonary segments
removed compared with the total number in one lung
b. The percentage predicted volumes are corrected for age and sex, but not height

Downloaded from https://www.cambridge.org/core. University of Edinburgh, on 19 Aug 2019 at 13:21:27, subject to the Cambridge Core terms of
use, available at https://www.cambridge.org/core/terms. https://doi.org/10.1017/9781108566100.011
344 Chapter 5a: Clinical Anaesthesia Questions

c. Absolute limits for ppo FEV1 of 0.8 l is always used as the lower limit of acceptability for
resection
d. Patients with a ppo FEV1% >40% have minor postresection respiratory complications
e. Patients with ppo FEV1% <30% should never have lung resection

Question 43
There are many causes of preoperative anaemia. What are the causes of a microcytic
anaemia?
a. Renal failure
b. Bone marrow failure
c. Pregnancy
d. Phenytoin
e. Acute blood loss

Question 44
You are urgently called to delivery suite to assist in the resuscitation of a neonate. Physiology
of the neonate is characterized by:
a. Obligatory nose breathing
b. Increase in alveolar ventilation predominantly achieved by increase in tidal volume
c. Relatively fixed stroke volume
d. Low levels of vitamin-K-dependent clotting factors at birth
e. Normal maintenance fluid requirement of 80 ml.kg–1.h–1

Question 45
Regarding anaphylaxis:
a. It is an example of an IgE-mediated type IV hypersensitivity reaction
b. It only occurs in atopic individuals
c. A rash must be present to confirm diagnosis
d. Epipens deliver the same intramuscular dose of adrenaline as that recommended in the
2015 ALS algorithm
e. Anaphylactic reactions to neuromuscular blocking agents can only occur after previous
exposure to the drug

Question 46
During an elective orthopaedic operating list a patient is scheduled to have an achilles
tendon repair for which the patient has to be placed in the prone position. Complications
directly from prone positioning include:
a. Compartment syndrome
b. Hypotension
c. Visual loss
d. Aspiration
e. Neuropraxia

Downloaded from https://www.cambridge.org/core. University of Edinburgh, on 19 Aug 2019 at 13:21:27, subject to the Cambridge Core terms of
use, available at https://www.cambridge.org/core/terms. https://doi.org/10.1017/9781108566100.011
Chapter 5a: Clinical Anaesthesia Questions 345

Question 47
With regard to adult conscious sedation:
a. During moderate sedation patients respond normally to verbal stimulation
b. When a combination of a benzodiazepine and an opioid is to be used, it is appropriate to
administer the benzodiazepine first
c. The AAGBI recommend that capnography is used for all sedated patients
d. Oxygen should be administered to all sedated patients
e. The elimination half-life of midazolam is significantly increased in renal failure

Question 48
These factors help prevent respiratory distress syndrome of the neonate:
a. Cortisol
b. Thyroxine
c. Insulin
d. Hypothermia
e. β-Adrenergic drugs

Question 49
A 29-year-old patient is listed for an ORIF of the wrist. He asks about the possibility of
surgery under local anaesthetic and you discuss a brachial plexus block. The following are
true about the anatomy of the brachial plexus:
a. The ulnar nerve is a branch of the lateral cord
b. The median nerve receives supply from both the medial and lateral cord
c. The radial nerve has no sensory supply in the arm
d. The suprascapular nerve is a branch of the upper trunk
e. The musculocutaneous nerve is a branch of the posterior cord

Question 50
A 72-year-old male is listed for an elective AAA repair. You note in preassessment that the
patient has undergone cardiopulmonary exercise testing. The report highlights a borderline
anaerobic threshold. Regarding anaerobic threshold:
a. The anaerobic threshold will not vary with patient motivation
b. The peak VO2 will not vary with patient motivation
c. Varies greatly with age
d. An anaerobic threshold of at least 15 ml.kg–1.min–1 is required to safely undertake
significant surgery
e. Patients with anaerobic thresholds less than 11 ml.kg–1.min–1 may benefit from post-
operative critical care

Question 51
A 55-year-old female presents to your pain management clinic with a six-month history of
continuous pain in her wrist following a fractured distal radius. Her GP wonders if this is

Downloaded from https://www.cambridge.org/core. University of Edinburgh, on 19 Aug 2019 at 13:21:27, subject to the Cambridge Core terms of
use, available at https://www.cambridge.org/core/terms. https://doi.org/10.1017/9781108566100.011
346 Chapter 5a: Clinical Anaesthesia Questions

complex regional pain syndrome. Which of the following signs or symptoms form part of
the criteria for the diagnosis?
a. Tremor
b. Hypoaesthesia
c. Hyperalgesia
d. Oedema
e. Hypohydrosis

Question 52
Regarding the diagnosis and perioperative care of patients with epilepsy:
a. It is defined as one or more epileptic seizures not caused by any immediately identifiable
cause
b. In established epilepsy investigated using electroencephalography, epileptiform activity
is seen in up to 50% of cases
c. Intravenously injectable forms of both sodium valproate and phenytoin are available for
use perioperatively in poorly controlled epilepsy
d. In the management of status epilepticus, lorazepam at 0.1 mg.kg–1 is recommended as
a first-line benzodiazepine
e. Cerebral venous thrombosis is a recognized complication associated with status
epilepticus

Question 53
The following statements are true:
a. A burn unit has immediate access to operating theatres
b. A burn unit offers the highest level of critical care
c. A burn unit offers a separately staffed, discrete ward
d. A burn facility equates to a standard plastic surgical ward for the care of non-complex
burn injuries
e. A burn facility equates to a specialized burns/plastic surgical ward for the care of
moderate complexity burn injuries

Question 54
You have been asked to anaesthetize a patient for a resection of a small bowel tumour,
suspected to be carcinoid. Which of the following statements are true?
a. Autonomic symptoms are more common with GI tumours than pulmonary tumours
b. The patient is at high risk of mitral valve disease
c. Ingestion of blue cheese or chocolate may precipitate a carcinoid attack
d. Atracurium should be avoided
e. Noradrenaline infusion may cause hypotension

Question 55
With regard to the management of meningitis in children (over three months):
a. Early empirical antibiotic therapy is with ceftriaxone

Downloaded from https://www.cambridge.org/core. University of Edinburgh, on 19 Aug 2019 at 13:21:27, subject to the Cambridge Core terms of
use, available at https://www.cambridge.org/core/terms. https://doi.org/10.1017/9781108566100.011
Chapter 5a: Clinical Anaesthesia Questions 347

b. Neuroimaging is mandatory for all cases


c. Notification of meningitis to public health authorities is mandatory in all cases
d. Signs of raised ICP in children may include abnormal posturing, focal neurological
defects and doll’s eye movements
e. Large resuscitative fluid requirement is an indication for intubation

Question 56
Regarding airway assessment, the following form part of Wilson’s criteria:
a. Obesity
b. Restricted jaw movement
c. Previous difficult intubation
d. Thyromental distance <6 cm
e. Receding mandible

Question 57
Advantages of using ultrasound in the practice of regional anaesthesia when compared to a
landmark-based technique are:
a. Faster onset of sensory blockade
b. Improved success rates
c. Decrease in neurological complications
d. Decrease risk of inadvertent vascular puncture
e. Decrease risk of complete hemidiaphragmatic paresis while performing brachial plexus
blockade

Question 58
Whilst anaesthesia is now a very safe endeavour, informing patients regarding the peri-
operative risk posed by surgery is a vital part of informed consent. Several scoring systems
may be used for this. The Goldman Cardiac Risk Index includes:
a. MI within the last year
b. Age >70 years
c. Severe aortic stenosis
d. Creatinine >200 μmol.l–1
e. Active endocarditis

Question 59
A 60 kg previously fit and well adult patient was admitted to ICU with severe sepsis
secondary to pneumonia. He has been started on increasing doses of both noradrenaline
and adrenaline following fluid resuscitation. His CVP is 12 cmH2O and MAP is maintained
at 70 mmHg with a HR of 130 min–1. His serum lactate however has been doubled to
5 mmol.l–1 over the last 6 hours. He is passing about 35 ml of urine hourly. The following
measures may help reduce his rising serum lactate.
a. Start vasopressin infusion to achieve a MAP of 75 mmHg
b. Increase noradrenaline infusion rate to achieve a MAP of 75 mmHg

Downloaded from https://www.cambridge.org/core. University of Edinburgh, on 19 Aug 2019 at 13:21:27, subject to the Cambridge Core terms of
use, available at https://www.cambridge.org/core/terms. https://doi.org/10.1017/9781108566100.011
348 Chapter 5a: Clinical Anaesthesia Questions

c. Increase adrenaline infusion rate to achieve a MAP of 75 mmHg


d. Reduce adrenaline infusion rate and increase the noradrenaline if needed
e. Start renal-dose dopamine infusion to increase his urine output

Question 60
Cardiac disease in pregnancy:
a. Pre-existing hypertrophic cardiomyopathy has a worse prognosis than pre-existing
dilated cardiomyopathy
b. Is the most common overall cause of death
c. Peripartum cardiomyopathy can present up to six months post delivery
d. The largest cause of cardiac-related maternal death is myocardial infarction
e. Women with severe chest pain should have radiological investigations undertaken

Question 61
Thalassaemia is an inherited disorder of haemoglobin. Regarding thalassaemia:
a. Thalassaemias have an autosomal dominant pattern of inheritance
b. Cooley’s anaemia is the major clinical phenotype of α-thalassaemia
c. Bart’s hydrops fetalis syndrome is the major clinical phenotype of β-thalassaemia
d. Penicillin may also precipitate haemolysis
e. It is not associated with difficult laryngoscopy

Question 62
You are asked to assess a neonate prior to cardiac surgery. On examination you note the
child is blue. The following conditions cause cyanosis:
a. Tetralogy of Fallot
b. Transposition of the great vessels
c. Isolated pulmonary stenosis
d. Patent ductus arteriosus
e. Ventricular septal defect

Question 63
Complications and physiological changes associated with malignant hyperpyrexia include:
a. Hyperkalaemia
b. Myoglobinuria
c. Vasodilatation
d. Disseminated intravascular coagulation
e. Compartment syndrome

Question 64
During an elective urology case the surgeons ask for the patient to be placed in the Lloyd–
Davis position. What would be the potential complications?
a. Atelectasis

Downloaded from https://www.cambridge.org/core. University of Edinburgh, on 19 Aug 2019 at 13:21:27, subject to the Cambridge Core terms of
use, available at https://www.cambridge.org/core/terms. https://doi.org/10.1017/9781108566100.011
Chapter 5a: Clinical Anaesthesia Questions 349

b. Pulmonary oedema
c. Visual loss
d. Compartment syndrome
e. Alopecia

Question 65
When using low-frequency jet ventilation (LFJV) for airway surgery:
a. Gas is normally delivered intermittently at a rate of 20–30 breaths per minute
b. LFJV utilizes the Venturi effect
c. It is useful for patients who have upper airway obstruction
d. Adequacy of ventilation is best assessed using ETCO2
e. A MAC of 1.5–2.0 should be maintained

Question 66
Which of the following conditions are associated with a difficult airway?
a. Achondroplasia
b. Hurler’s syndrome
c. Hunter’s syndrome
d. Beckwith–Wiedemann syndrome
e. Goldenhar syndrome

Question 67
A male baby is born at 29 weeks’ gestation, three weeks after spontaneous rupture of
membranes. His breathing effort is poor, requiring assisted ventilation. After intuba-
tion, his umbilical artery blood gas sample shows: pH 7.18, paCO2 8.5 kPa, paO2
6.5 kPa, despite high FiO2 of 0.8. Which of the following is considered appropriate in
his management:
a. Echocardiogram
b. Increasing PEEP
c. Intravenous antibiotics
d. Surfactant
e. 100% O2 for 24 hours

Question 68
Regarding blood products:
a. Whole blood has a shelf-life of 35 days
b. Packed red cells may be stored for up to 30 days
c. Platelets can be stored for up to five days at 4 °C
d. A platelet transfusion requires ABO compatibility
e. A platelet transfusion requires Rh compatibility for all patients

Downloaded from https://www.cambridge.org/core. University of Edinburgh, on 19 Aug 2019 at 13:21:27, subject to the Cambridge Core terms of
use, available at https://www.cambridge.org/core/terms. https://doi.org/10.1017/9781108566100.011
350 Chapter 5a: Clinical Anaesthesia Questions

Question 69
Electroconvulsive therapy (ECT) is a recognized treatment modality in cases of severe or
medication-resistant depression. Regarding ECT:
a. An initial tachycardia and rise in blood pressure is common
b. Propofol is the preferred induction agent as it allows the longest seizure duration
c. Etomidate is contraindicated as an induction agent as it raises the seizure threshold
d. An increased dose of suxamethonium may be required in cachectic patients
e. Following ECT, myalgia is common

Question 70
A 72-year-old female comes to your pain management clinic via her GP. She complains of
12 months of right-sided facial pain below the eye. She describes the ‘stabbing’ pain as brief
but severe and that it can be provoked by a light touch to the skin. Which of the following
would be an appropriate first line in her management?
a. Acyclovir
b. 5% lidocaine plaster
c. Carbamazepine
d. Gabapentin
e. 0.075% capsaicin cream

Question 71
Regarding patients with myotonic dystrophy and their perioperative management:
a. Myotonic dystrophy is inherited in an autosomal dominant pattern
b. Prolonged muscle relaxation can be precipitated by the use of non-depolarizing muscle
relaxants
c. The use of regional anaesthesia may prevent prolonged myotonic muscle contraction
d. Reversal of non-depolarizing muscle relaxants using neostigmine is recommended
e. Troublesome muscle spasm may be alleviated by using phenytoin 3–5 mg.kg–1
intravenously

Question 72
A 25-year-old woman with a diagnosis of myasthenia gravis is listed for an appendicectomy
on the emergency list. Anaesthetic considerations include:
a. An increased dose of succinylcholine may be needed for rapid sequence induction
b. Only one-fifth of the normal dose of atracurium should be used for ongoing muscle
relaxation during the operation
c. Disease duration of greater than six years is a predictor of the need for postoperative
ventilation
d. Dose requirement of pyridostigmine of greater than 750 mg per day is a predictor of the
need for postoperative ventilation
e. Reversal of non-depolarizing muscle blockade with one-tenth of the normal dose of
neostigmine is recommended

Downloaded from https://www.cambridge.org/core. University of Edinburgh, on 19 Aug 2019 at 13:21:27, subject to the Cambridge Core terms of
use, available at https://www.cambridge.org/core/terms. https://doi.org/10.1017/9781108566100.011
Chapter 5a: Clinical Anaesthesia Questions 351

Question 73
Regarding the intracranial pressure waveform:
a. ‘C’ waves have an amplitude of <20 mmHg
b. ‘B’ waves have an amplitude of <50 mmHg
c. ‘Ramp’ waves are variations of ‘B’ waves
d. ‘A’ waves last 5–20 minutes
e. ‘A’ waves are most common in patients with intracranial tumours

Question 74
The moorLD12-BI can be employed in the assessment of burn wounds. Regarding the
moorLD12-BI:
a. It is a laser Doppler blood-flow imaging system
b. It can be used to treat partial thickness burns
c. It can predict wound healing in burns
d. It is used to guide fluid therapy in burns
e. It is non-invasive

Question 75
You have been asked to anaesthetize a patient for a laparoscopic cholecystectomy. His
medical history is remarkable for pulmonary hypertension (PH) and moderate COPD.
Concerning PH:
a. Milrinone and dobutamine are safe to use in PH
b. Central neuraxial anaesthesia is not recommended in PH
c. Maintaining a normal SVR is essential in safely anaesthetizing a patient with PH
d. Metaraminol is contraindicated due to hypersensitive pulmonary α-1 receptors
e. Ketamine may raise pulmonary vascular resistance

Question 76
Thyrotoxic storm may be treated using:
a. Propanolol
b. Hydrocortisone
c. Lugol’s solution via the Jod–Basedow mechanism
d. Amiodarone
e. Ibuprofen

Question 77
The effects of smoking include:
a. Reduction in postoperative nausea and vomiting
b. Increased airway reactivity
c. Hepatic enzyme inhibition
d. Shift of the oxyhaemoglobin dissociation curve to the right
e. Hypertension

Downloaded from https://www.cambridge.org/core. University of Edinburgh, on 19 Aug 2019 at 13:21:27, subject to the Cambridge Core terms of
use, available at https://www.cambridge.org/core/terms. https://doi.org/10.1017/9781108566100.011
352 Chapter 5a: Clinical Anaesthesia Questions

Question 78
In the patient undergoing elective surgery who has atrial fibrillation:
a. A slow ventricular response may be an indication for pacemaker insertion
b. Routine β-blockers should be omitted due to the risk of hypotension intraoperatively
c. Digoxin doses should be reduced by a quarter if converted from oral to intravenous
formulation
d. Patients receiving digoxin should have their levels measured on the day of surgery
e. All medications should be converted to intravenous on the day of surgery

Question 79
Regarding interscalene brachial plexus block:
a. It provides reliable anaesthesia and analgesia for procedures involving the proximal
humerus and shoulder joint
b. It effectively blocks proximal nerve roots and the distal cervical plexus
c. If using ultrasound, a high-frequency (10–13 MHz) probe is used to scan the neck for
plexus visualization
d. C5, C6 and C7 nerve roots are seen as hyperechoic round images in between the scalene
muscles
e. There is level 1a evidence to confirm phrenic nerve blockade is almost always seen with
high volume (20 ml or more) local anaesthetic injection

Question 80
The liver is the principle site of drug metabolism in the human body. Regarding drug
metabolism:
a. Phase 2 metabolism involves conjugation of drugs with hydrophobic groups
b. Cytochrome enzymes are found in the kidney
c. Oxidation, reduction, hydrolysis and hydration are part of phase 1 drug metabolism
d. Morphine-3-glucuronide is an active metabolite of morphine formed by liver drug
metabolism
e. Cytochrome P450 contains a haem–iron centre

Question 81
The following statements are correct regarding the pathophysiology of sepsis:
a. Multiple organ failure secondary to sepsis is the most common cause of death in ICU
b. Oxygen delivery is usually the main problem in severe sepsis
c. Nitric oxide production is increased during sepsis, whilst ATP production is reduced
d. Endogenous antioxidant systems become overwhelmed in sepsis
e. Antioxidants have been shown to improve survival in young septic patients

Question 82
In pregnant women with pre-existing spinal cord injury:
a. Pre-emptive epidural anaesthesia reduces the effects of autonomic dysreflexia

Downloaded from https://www.cambridge.org/core. University of Edinburgh, on 19 Aug 2019 at 13:21:27, subject to the Cambridge Core terms of
use, available at https://www.cambridge.org/core/terms. https://doi.org/10.1017/9781108566100.011
Chapter 5a: Clinical Anaesthesia Questions 353

b. Contractions may still be felt if a complete spinal cord lesion is between T5 and T10
c. Potential cardiovascular complications occurring during labour include hypotension,
bradycardia and cardiac arrest
d. Epidural catheters should be taken out as soon as possible after delivery
e. Suxamethonium should not be used

Question 83
With regards to the cardiac assessment of patients listed for lung resection:
a. Elective pulmunory resection is regarded as an ‘intermediate-risk’ procedure in terms of
perioperative cardiac ischaemia
b. The overall incidence of documented post-thoracotomy ischemia is 5%
c. Post-thoracotomy ischaemia peaks on the second to third day
d. Patients with intermediate clinical predictors of increased cardiac risk who have ade-
quate functional capacity do not need further cardiac investigations
e. CT coronary angiography is highly sensitive, but it is less specific

Question 84
Hypocalcaemia may be detected following thyroid surgery. Hypocalcaemia may present
with:
a. Chovstek’s sign on non-invasive blood pressure cuff inflation
b. Perioral tingling
c. Increased QT interval
d. Facial spasm upon tapping on the submandibular gland
e. Seizures

Question 85
Neonates compared with adults are resistant to suxamethonium because:
a. Pseudocholinesterase is 50% more active in neonates
b. Neonates have more motor end plates per kg
c. Suxamethonium is excreted by the neonate
d. Neonates have a larger extracellular volume per kg
e. Neonates have a greater proportion of haemoglobin F

Question 86
A 45-year-old female is undergoing laparoscopic cholecystectomy. She has been induced
with propofol and fentanyl, paralyzed with rocuronium and anaesthesia maintained with
isoflurane in an oxygen/air mix. She has also received co-amoxiclav at induction.
On insufflation of the abdomen the non-invasive blood pressure becomes 36/18 mmHg.
Causes include:
a. Pneumoperitoneum
b. Too small a blood pressure cuff
c. Hypovolaemia

Downloaded from https://www.cambridge.org/core. University of Edinburgh, on 19 Aug 2019 at 13:21:27, subject to the Cambridge Core terms of
use, available at https://www.cambridge.org/core/terms. https://doi.org/10.1017/9781108566100.011
354 Chapter 5a: Clinical Anaesthesia Questions

d. Anaphylaxis
e. Air embolism

Question 87
Whilst following up a patient postoperatively they describe explicit recall. Factors from their
intraoperative period that could have contributed include:
a. Emergency anaesthesia
b. Paralysis
c. Hypotension
d. Opioids
e. TIVA

Question 88
During a rapid sequence induction (RSI):
a. A force of 40 N should be applied to the cricoid cartilage
b. Up to four attempts should be should be made at initial intubation
c. In some circumstances a second dose of suxamethonium may be appropriate if initial
intubation is not successful
d. In an anaesthetized patient O2 consumption remains relatively constant at approxi-
mately 250 ml.min–1
e. Anaemia causes a significant decrease in the time taken for deoxygenation during an RSI

Question 89
A 72-year-old gentleman is listed for a TURP. Which of the following are true?
a. Glycine 1.5% is an isotonic irrigation fluid
b. Irrigation fluid is usually absorbed by the patient at a rate of 20 ml.min–1
c. The irrigation fluid should be kept at high pressure
d. Risk of TURP syndrome increases with prostates weighing over 50 mg
e. Preoperative hypertension increases the risk of TURP syndrome

Question 90
These factors help keep the ductus arteriosus open:
a. Acidosis
b. Low partial pressure of O2 in the blood passing through the ductus
c. Alkalosis
d. Pulmonary hypertension
e. Prostaglandin

Question 91
The following is true about local anaesthetic toxicity:
a. Sudden alteration in mental status with or without tonic–clonic convulsions
b. Sinus bradycardia can occur

Downloaded from https://www.cambridge.org/core. University of Edinburgh, on 19 Aug 2019 at 13:21:27, subject to the Cambridge Core terms of
use, available at https://www.cambridge.org/core/terms. https://doi.org/10.1017/9781108566100.011
Chapter 5a: Clinical Anaesthesia Questions 355

c. Ventricular arrhythmia is a common presentation


d. Intercostal block carries the highest rate of absorption of the LA drugs
e. The first line of action is to draw blood for analysis

Question 92
You have been asked to assist in the creation of an anaesthetic preassessment clinic at your
hospital. Cardiopulmonary exercise testing requires the following:
a. An adequately ventilated room with full resuscitation equipment and two members of
staff
b. A metabolic cart and treadmill
c. Gas flows and volumes quantified by a pressure differential pneumotachograph
d. A 12-lead ECG, non-invasive blood pressure and SpO2 monitors
e. Duration of exercise between 5 and 15 minutes

Question 93
You are performing a sciatic nerve block as part of your management of a total knee
replacement. You favour the inferior approach. Which of the following statements regard-
ing sciatic nerve blocks are true?
a. The inferior approach is also known as the Raj approach
b. Landmarks to the inferior approach include posterior superior iliac spine and the
greater trochanter
c. The inferior approach is unlikely to block the posterior cutaneous nerve of the thigh
d. As the sciatic nerve is large, adrenaline may be a useful adjunct
e. The sciatic nerve at the inferior approach is commonly 40–80 mm deep

Question 94
A 65-year-old man with a history of chronic low back pain is taking slow-release oral
morphine 70 mg twice daily. Which of the following patches would offer dose equivalence?
a. Buprenorphine 5 μg.h–1
b. Buprenorphine 10 μg.h–1
c. Buprenorphine 20 μg.h–1
d. Fentanyl 50 μg.h–1
e. Fentanyl 75 μg.h–1

Question 95
Considerations in managing patients with acromegaly presenting for pituitary surgery
include:
a. Up to 70% of patients will have significant obstructive sleep apnoea
b. There is an increased risk of nerve entrapment syndromes
c. Exophthalmos secondary to retro-orbital fat deposits is seen in one-third of patients
d. Cervical and supraclavicular fat pads make central venous cannulation more difficult
e. The onset of disease is likely to have been rapid

Downloaded from https://www.cambridge.org/core. University of Edinburgh, on 19 Aug 2019 at 13:21:27, subject to the Cambridge Core terms of
use, available at https://www.cambridge.org/core/terms. https://doi.org/10.1017/9781108566100.011
356 Chapter 5a: Clinical Anaesthesia Questions

Question 96
You are referred a 75-year-old man for assessment regarding the need for invasive ventila-
tion. He presents with a three-day-long history of diarrhoea and vomiting, an ascending
motor paralysis and areflexia. He now has respiratory compromise, thought to be due to
Guillain–Barré syndrome (GBS). Which of the following statements regarding GBS are
true?
a. The most likely pathogen responsible for this presentation is Campylobacter jejuni
b. Weakness that is more proximal than distal is characteristic
c. Wide fluctuations in blood pressure and pulse may ensue following intubation
d. Ventilatory support is required in approximately half of patients
e. A vital capacity of less than 50 ml.kg–1 is an indication for endotracheal intubation

Question 97
The following statements regarding chlorhexidine are true:
a. 2% Chlorhexidine gluconate in 70% isopropyl alcohol is recommended for all skin
preparation prior to invasive procedures
b. 1% Chlorhexidine cannot be used in surgery involving the inner or middle ear because it
is ototoxic
c. Chlorhexidine is not sporicidal
d. The activity of chlorhexidine is reduced in the presence of blood
e. Chlorhexidine is a cationic molecule

Question 98
Concerning the intraoperative management of one-lung ventilation during
oesophagectomy:
a. One-lung ventilation can be achieved with a single lumen ETT
b. Hypoxia may be treated by clamping the pulmonary artery of the dependent lung
c. Right-sided double-lumen tubes are most commonly used
d. Malpositioned tubes do not have any major implications
e. The tube is rotated through 180° after it passes through the vocal cords

Question 99
Concerning myxoedema coma:
a. Downregulation of sympathetic receptors makes patients very sensitive to vasopressors
and inotropes
b. Thyroid imaging is essential for diagnosis
c. Patients are typically hypernatraemic
d. Low contractility and reduced cardiac index lead to reflex tachycardia
e. Steroid administration and aggressive re-warming with active measures are important
resuscitative measures

Downloaded from https://www.cambridge.org/core. University of Edinburgh, on 19 Aug 2019 at 13:21:27, subject to the Cambridge Core terms of
use, available at https://www.cambridge.org/core/terms. https://doi.org/10.1017/9781108566100.011
Chapter 5a: Clinical Anaesthesia Questions 357

Question 100
You are asked to assess a patient preoperatively who is dialysis dependent. The following are
important to note:
a. The presence and location of fistulae
b. When the patient is usually dialyzed
c. Whether the patient is fluid restricted
d. The presence of peripheral neuropathy
e. Volume status of the patient

Question 101
Regarding regional anaesthesia in paediatric patients:
a. It is usually performed after general anaesthesia has been administered
b. The maximum allowable local anaesthetic dose should be based on a patient’s total body
weight
c. If using ultrasound-guided blocks, lower-frequency probes will be more suitable
d. Penile block performed using a landmark approach has a reported failure rate of 10–15%
e. Systemic absorption of local anaesthetic after ilioinguinal and iliohypogastric blocks
performed using ultrasound is higher than after landmark approach

Question 102
Over the last 50 years, liver transplantation has provided a pathway for survival in end-stage
liver failure. Regarding liver transplantation:
a. The commonest reason for liver transplantation worldwide is for fulminant liver failure
secondary to paracetamol overdose
b. Venovenous bypass can be used to enable liver removal from the patient
c. Reperfusion syndrome will typically cause the patient’s temperature to rise
d. During the anhepatic phase, hypercalaemia can be an issue
e. Hepatic artery thrombosis occurs in 20% of patients in the early postoperative phase and
may necessitate superurgent retransplantation

Question 103
The following are true regarding the diagnosis and definition of acute respiratory distress
syndrome (ARDS):
a. It can be classified into mild, moderate and severe according to the degree of hypoxaemia
b. The onset of ARDS must be acute: within seven days of the insulting event
c. The presence of bilateral lung opacities that are not explained by consolidation or
effusion
d. There is no need to exclude high pulmonary capillary wedge pressure
e. The need for PEEP >10 cmH2O is predictive of poor outcome

Downloaded from https://www.cambridge.org/core. University of Edinburgh, on 19 Aug 2019 at 13:21:27, subject to the Cambridge Core terms of
use, available at https://www.cambridge.org/core/terms. https://doi.org/10.1017/9781108566100.011
358 Chapter 5a: Clinical Anaesthesia Questions

Question 104
Regarding cardiac arrests in pregnant women:
a. They may occur during labour with no warning
b. They have an incidence of around 1:30 000 pregnancies
c. If the fetus is over 20 weeks’ gestation then it is recommended that perimortem
caesarean section be performed within 10 minutes of maternal arrest
d. They are increasing in frequency
e. If in a shockable rhythm require a lower-level electrical shock than in a non-pregnant
individual

Question 105
A patient with emphysema is listed for lung reduction surgery. The inclusion criteria for
lung volume reduction surgery (LVRS) include the following:
a. A predicted FEV1 between 15 and 35%
b. PAP <70 mmHg
c. paCO2 <8 kPa
d. Age <75 years
e. Prednisone requirement <20 mg.day–1

Question 106
Rheumatoid arthritis can affect the atlantoaxial joint, and may manifest as atlantoaxial
subluxation. Regarding the subtypes of atlantoaxial subluxation:
a. Anterior subluxation involves destruction of the odontoid peg
b. Anterior subluxation involves destruction of transverse and apical ligaments
c. Anterior subluxation is the rarest form of atlantoaxial subluxation secondary to rheu-
matoid arthritis
d. Vertical subluxation leads to compression of the cervicomedullary junction
e. Posterior subluxation is worsened by neck extension

Question 107
Inhalational induction in children with sevoflurane is faster because of their:
a. Increased MAC
b. Increased cardiac output per kg compared to adults
c. Increased minute ventilation per kg compared to adults
d. Increased tissue solubility
e. Small functional residual capacity

Question 108
The following may cause malignant hyperpyrexia:
a. Ketamine
b. Etomidate
c. Enflurane

Downloaded from https://www.cambridge.org/core. University of Edinburgh, on 19 Aug 2019 at 13:21:27, subject to the Cambridge Core terms of
use, available at https://www.cambridge.org/core/terms. https://doi.org/10.1017/9781108566100.011
Chapter 5a: Clinical Anaesthesia Questions 359

d. Nitrous oxide
e. Suxamethonium

Question 109
You have chosen to anaesthetize a patient using TIVA for a prolonged case and decide to
employ the BIS (bispectral) monitor. Regarding BIS monitoring:
a. It uses EEG
b. A higher reading is associated with deeper anaesthesia
c. The target reading is >60
d. The range is 0–100
e. Sensors can be placed anywhere on the face

Question 110
A 72-year-old patient is on your list for TURP. Regarding the glycine irrigation fluid
commonly used during a TURP procedure:
a. It is a 0.5% solution
b. It is suspended in 0.9% saline
c. It is metabolized in the liver by oxidation
d. One of its metabolites is ammonium
e. Glycine is an inhibitory neurotransmitter

Question 111
The following is true regarding congenital heart diseases:
a. The treatment of a patent ductus arteriosus is always medical
b. Coarctation of the aorta is the most common cardiac defect in Turner syndrome
c. Complete atrioventricular septal defect is the most common cardiac anomaly in Down’s
syndrome
d. Left to right shunts lead to cyanosis
e. Right to left shunts lead to symptoms of heart failure

Question 112
You are assisting with a liver transplant and the most recent thromboelastograph (TEG)
implies the patient requires fresh frozen plasma. Regarding fresh frozen plasma (FFP):
a. It contains all clotting factors, albumin and γ-globulin
b. It must be transfused to an ABO-compatible donor
c. The usual starting dose is 5 ml.kg–1
d. Transfusion of FFP for children born after 1995 is derived from unpaid American
donors
e. Cryoprecipitate is precipitated from FFP and contains high levels of factor VIII, fibri-
nogen and von Willebrand factor

Downloaded from https://www.cambridge.org/core. University of Edinburgh, on 19 Aug 2019 at 13:21:27, subject to the Cambridge Core terms of
use, available at https://www.cambridge.org/core/terms. https://doi.org/10.1017/9781108566100.011
360 Chapter 5a: Clinical Anaesthesia Questions

Question 113
You have been asked to perform a coeliac plexus block on a patient with pancreatic cancer.
Regarding coeliac plexus blocks:
a. The plexus lies posterior to the aorta
b. The plexus has contributions from the vagus and phrenic nerves
c. Paraplegia occurs in approximately 1:5000
d. Common complications include urinary retention and constipation
e. Damage to the L3 nerve root may occur

Question 114
Regarding anaesthesia for awake craniotomy:
a. Inability to lie flat is an absolute contraindication
b. The surgery can be performed using a regional technique as the sole technique, with the
patient awake throughout
c. The patient can be sedated or anaesthetized during intraoperative mapping of the
tumour
d. On each side of the scalp, nine nerves can be targeted for an anatomical scalp block
e. The greater auricular nerve is blocked just anterior to the auricle at the level of the
temporomandibular joint

Question 115
The following statements regarding thermal burns are true:
a. Full thickness burns are painful
b. Deep dermal burns have no hair follicles
c. Superficial burns are painful
d. Deep dermal burns are also known as second-degree burns
e. Areas of erythema should be counted in burns assessment

Question 116
You have been asked to anaesthetize a Jehovah’s witness for a laparotomy for a vascular
tumour. He has declined blood transfusion on religious grounds. Concerning other strate-
gies that may be employed intraoperatively:
a. Cell salvage is acceptable to all Jehovah’s witnesses
b. Consultant anaesthetic involvement may be desirable, but not essential in the manage-
ment of these cases
c. Anaesthetists have no obligation to anaesthetize a Jehovah’s witness for elective surgery
d. An advanced directive can be changed by the patient intraoperatively if undergoing
regional anaesthesia
e. Tranexamic acid may be useful intraoperatively

Downloaded from https://www.cambridge.org/core. University of Edinburgh, on 19 Aug 2019 at 13:21:27, subject to the Cambridge Core terms of
use, available at https://www.cambridge.org/core/terms. https://doi.org/10.1017/9781108566100.011
Chapter 5a: Clinical Anaesthesia Questions 361

Question 117
The following are required for day surgery to proceed safely:
a. The patient should have an adult at home for 24 hours following surgery
b. BMI under 35
c. Patients must be able to eat and drink within 36 hours
d. Age under 85 years
e. Patients with epilepsy are not suitable for day case surgery due to sleep–wake
disturbance

Question 118
When assessing a patient with renal disease for theatre it is important to anticipate that the
patient may have the following:
a. Delayed gastric emptying
b. Altered metabolism of atracurium
c. Thrombocytopenia
d. Hyperkalaemia
e. A decreased free fraction of protein-bound drugs

Question 119
Regarding epidural analgesia in abdominal surgery:
a. There is enough evidence to suggest there is a decrease in postoperative thromboembolic
complications
b. Effective epidural analgesia may decrease the incidence of chronic postsurgical pain
c. Has immunomodulatory effects and hence may reduce the risk of cancer recurrence
d. Postoperative cognitive dysfunction is higher in patients receiving epidural analgesia
e. Neurological complications are higher when performed in anaesthetized patients

Question 120
To enable surgery on the still heart in a bloodless field, patients undergoing cardiac surgery
are typically placed onto a cardiopulmonary bypass circuit. During cardiac surgery:
a. Cardioplegia is typically inserted into the descending aorta to perfuse the coronaries and
cause cardiac standstill
b. Cardioplegia solution is a hyperkalaemic solution
c. The aim for cardioplegia is for the heart to arrest in systole
d. Roller pumps can provide pulsatile flow
e. Prior to going onto cardiopulmonary bypass the ACT must be >300 s

Question 121
The following statements are true regarding nutritional support for critically ill patients:
a. Overfeeding can easily happen with both enteral and parenteral feeding
b. Total parenteral nutrition (TPN) can be associated with bacterial translocation
c. It is better to underfeed than overfeed

Downloaded from https://www.cambridge.org/core. University of Edinburgh, on 19 Aug 2019 at 13:21:27, subject to the Cambridge Core terms of
use, available at https://www.cambridge.org/core/terms. https://doi.org/10.1017/9781108566100.011
362 Chapter 5a: Clinical Anaesthesia Questions

d. Postpyloric feeding does not increase the incidence of diarrhoea


e. It is better to target lower caloric intake during an acute phase of illness than during
recovery

Question 122
With pregnancy-related diabetes:
a. A woman should be routinely screened for gestational diabetes if her BMI is greater
than 30
b. Pre-eclampsia is more prevalent
c. Metformin is contraindicated due to the risk of lactic acidosis developing
d. During labour, women with pre-existing diabetes should receive an insulin and dextrose
infusion
e. Epidural analgesia is recommended for diabetic mothers in labour due to an increased
risk of requiring obstetric intervention

Question 123
Which of the following enhance hypoxic pulmonary vasoconstriction?
a. Nitrous oxide
b. Thoracic epidural
c. Almitrine
d. Propofol
e. Opioids

Question 124
Patients have the right to refuse treatment. Regarding advance directives and Jehovah’s
Witnesses, which of the following are true?
a. It is a legally binding document
b. It can be made by anyone over the age of 14 years
c. It must be updated in order for the patient to change their wishes
d. It is only kept with the patient
e. It can be disregarded in an emergency situation

Question 125
You are anaesthetizing a child for a cleft palate repair. Regarding cleft lip and palate:
a. Cleft palate cannot occur without cleft lip
b. History of apnoea during feeds may indicate chronic airway obstruction
c. Bilateral cleft lip predicts difficult laryngoscopy
d. Difficulty with mask ventilation is common
e. Airway obstruction after cleft lip and palate repair may be treated with insertion of
a nasopharyngeal airway

Downloaded from https://www.cambridge.org/core. University of Edinburgh, on 19 Aug 2019 at 13:21:27, subject to the Cambridge Core terms of
use, available at https://www.cambridge.org/core/terms. https://doi.org/10.1017/9781108566100.011
Chapter 5a: Clinical Anaesthesia Questions 363

Question 126
With regards to suxamethonium apnoea:
a. E1f E1f homozygotes have plasma cholinesterase present
b. Diagnostic testing involves the use of benzylcholine
c. Dibucaine inhibits plasma cholinesterase
d. Fresh frozen plasma is routinely used in the management
e. Methotrexate inhibits plasma cholinesterase activity

Question 127
You are asked to review your patient in recovery who has a core temperature of 36 °C.
Regarding perioperative hypothermia:
a. The preoperative phase starts on arrival in the anaesthetic room
b. Loss of behavioural response is a contributor
c. The postoperative phase ends 12 hours after leaving the anaesthetic room
d. A higher ASA grade is associated with a higher risk of preoperative hypothermia
e. Fluid warmers are recommended when giving fluid volumes greater than 500 ml

Question 128
You are on ICU performing a broncheoalveolar lavage. A medical student asks you to
describe the bronchial anatomy. Which of the following are true?
a. The left main bronchus is shorter than the right main bronchus
b. The left main bronchus is wider than the right main bronchus
c. The left main bronchus is more oblique than the right main bronchus
d. The left lower lobe bronchus gives off the lingular bronchus
e. The middle lobe bronchus bifurcates into the posterior and lateral lobes

Question 129
Twenty-four hours after birth, a male neonate is found centrally cyanosed with no signs of
respiratory distress. His blood pressure is 90/60 mmHg with a pulse rate of 140 bpm. Chest
examination reveals a precordial heave and ejection systolic murmur in the second inter-
costal space. ECG shows signs of right ventricular hypertrophy and right axis deviation.
The options for treatment are:
a. Balloon atrial septostomy
b. Prostaglandin infusion
c. Indomethacin
d. Pulmonary artery banding
e. Modified Blalock–Taussig shunt

Question 130
You have been informed that your Trust is investigating a ‘Never Event’ in which you were
involved. According to NHS England, the following are ‘Never Events’:
a. Wrong site surgery

Downloaded from https://www.cambridge.org/core. University of Edinburgh, on 19 Aug 2019 at 13:21:27, subject to the Cambridge Core terms of
use, available at https://www.cambridge.org/core/terms. https://doi.org/10.1017/9781108566100.011
364 Chapter 5a: Clinical Anaesthesia Questions

b. Wrong site block for pain control


c. Wrong route of administration of medication
d. Feeding with misplaced naso- or orogastric tubes
e. Retained throat pack inserted by anaesthetist

Question 131
A 50-year-old male presents to your pain clinic for management of his chronic lower back
pain. A positive response to two diagnostic local anaesthetic blocks of the medial branches
in his lumbar spine makes him suitable for further treatment with radiofrequency denerva-
tion. With regard to radiofrequency (RF) denervation procedures in the lumbar spine:
a. RF procedures use high-frequency alternating current
b. A permanent thermal lesion of the nerve is produced by direct heating via the needle tip
c. Continuous RF is selective for sensory nerve fibres
d. Pulsed RF does not produce significant heating of tissues
e. Pulsed RF has a better side-effect profile

Question 132
Regarding the pharmacological management of Parkinson’s disease, the following drugs are
correctly matched to their parent group:
a. Anticholingergic agents: amantidine
b. Dopamine agonists: entacapone
c. Catechol-O-methyl transferase inhibitors: apomorphine
d. Monoamine oxidase B inhibitors: selegeline
e. Atypical agents: ropinorole

Question 133
With regards to fluid resuscitation in burns patients, the following statements are true:
a. The Parkland formula describes the amount of Ringer’s lactate to be given in the first 24
hours after a burn
b. The volume of fluid to be given according to the Parkland formula is 2 ml.kg–1 body
weight × % TBSA burn.
c. An 80 kg man who has sustained a 40% TBSA burn one hour prior to arriving in hospital
should have his resuscitation fluid started at 1600 ml.h–1 plus maintenance
d. A 60 kg man who has sustained a 30% TBSA burn four hours prior to arriving in hospital
should have his resuscitation fluid started at 1200 ml.h–1 plus maintenance
e. When using the Parkland formula to calculate resuscitation fluid volumes, half of the
fluid is given in the first eight hours after the burn

Question 134
Concerning intra-abdominal compartment syndrome:
a. Normal intra-abdominal pressure is zero or subatmospheric
b. Abdominal compartment syndrome is defined as an intra-abdominal pressure (IAP)
>15 mmHg

Downloaded from https://www.cambridge.org/core. University of Edinburgh, on 19 Aug 2019 at 13:21:27, subject to the Cambridge Core terms of
use, available at https://www.cambridge.org/core/terms. https://doi.org/10.1017/9781108566100.011
Chapter 5a: Clinical Anaesthesia Questions 365

c. Surgical decompression is indicated when IAP is above 25 mmHg


d. Abdominal compartment syndrome may decrease shunt fraction
e. Patients with suspected abdominal compartment syndrome should be aggressively
volume resuscitated

Question 135
Presenting features of Guillain–Barré syndrome include:
a. Back and/or limb pain
b. Cranial nerve involvement before the upper extremities
c. Tachycardia, alternating hyper- and hypotension and/or dysrhythmias
d. Respiratory failure necessitating ventilation in 80% of cases
e. Pupillary dilatation

Question 136
When assessing a patient with diabetes preoperatively, it is important to consider that they
are at risk of the following:
a. Difficult intubation
b. Peripheral neuropathy
c. Renal disease
d. Ischaemic heart disease
e. Autonomic dysfunction

Question 137
The following statements are correct regarding the management of status epilepticus:
a. Continuous EEG monitoring is required if the patient is sedated and paralyzed
b. Thiopentone infusion is better than propofol infusion in controlling seizure activity
c. If the patient is known to be on regular oral phenytoin, an IV loading dose should be
avoided until a serum phenytoin level has been checked
d. Midazolam is superior to lorazepam as a first-line therapy
e. Midazolam can be given intramuscularly

Question 138
The following are considered absolute contraindications for epidural analgesia in
pregnancy:
a. Patient refusal
b. Raised temperature
c. Hypovolaemic shock
d. Coagulopathy
e. Infection at site of insertion

Question 139
Duchenne muscular dystrophy is a disorder of muscle. Which of the following regarding
Duchenne muscular dystrophy are true?

Downloaded from https://www.cambridge.org/core. University of Edinburgh, on 19 Aug 2019 at 13:21:27, subject to the Cambridge Core terms of
use, available at https://www.cambridge.org/core/terms. https://doi.org/10.1017/9781108566100.011
366 Chapter 5a: Clinical Anaesthesia Questions

a. X-linked recessive pattern of inheritance


b. Affected skeletal and smooth muscle, with cardiac muscle sparing
c. Progressive wasting and weakness of distal muscles
d. Presentation may be with a waddling gait
e. Patients have a normal life expectancy

Question 140
A patient has been listed for appendicectomy on the CEPOD list. You note the history of
Pierre Robin syndrome. The following are true of Pierre Robin syndrome:
a. Describes a triad of micrognathia, glossoptosis and cleft palate
b. May be associated with cardiac and ocular problems
c. Often requires tracheostomy at birth
d. Intubation may become difficult with increasing age
e. Should not be given opioid analgesia postoperatively

Question 141
Regarding the causes of anaphylaxis under anaesthesia:
a. Rocuronium causes more reactions than suxamethonium
b. Propofol causes fewer reactions than etomidate
c. Neuromuscular blocking agents are attributable for 40% of reactions
d. Colloids cause less than 5% of reactions
e. Remifentanil causes more anaphylactic reactions than morphine

Question 142
You are involved in anaesthetizing a patient for an emergency AAA repair.
Which of the following could be related to the massive transfusion of blood products
intraoperatively?
a. Hypercalcaemia
b. Hyperkalaemia
c. Metabolic acidosis
d. Thrombocytopenia
e. Hypothermia

Question 143
Features of TURP syndrome include:
a. Hypotension
b. Hypertension
c. Bradycardia
d. Tachycardia
e. Seizures

Downloaded from https://www.cambridge.org/core. University of Edinburgh, on 19 Aug 2019 at 13:21:27, subject to the Cambridge Core terms of
use, available at https://www.cambridge.org/core/terms. https://doi.org/10.1017/9781108566100.011
Chapter 5a: Clinical Anaesthesia Questions 367

Question 144
The following are true regarding anaesthetic considerations in anaesthetizing a child with
congenital aganglionic megacolon (Hirschsrpung’s disease):
a. Treatment of uncomplicated cases requires emergency operation
b. It is unlikely to be associated with other congenital anomalies
c. There is a perioperative risk of septic shock
d. Electrolyte imbalance is a common presentation
e. Third-space loss may be a preoperative problem

Question 145
Regarding cardiopulmonary exercise testing (CPET):
a. Has a mortality rate of 2–4 in 100 000
b. Measurements include work rate in joules
c. VO2 is equal to cardiac output multiplied by arterial–mixed venous oxygen difference
d. Cardiac output and arterial–mixed oxygen differences increase linearly with VO2 in
most patients until a peak oxygen extraction ratio of 60% is reached
e. Readings are displayed graphically in a standardized format called the nine-panel plot

Question 146
A 55-year-old female is listed for a total abdominal hysterectomy. Your anaesthetic plan
includes the administration of a TAP block. The following are true:
a. Innervation of the anterolateral abdominal wall arises from the anterior rami of spinal
nerves T7 to L1
b. The anterior divisions of T7–T11 enter the abdominal wall between the internal oblique
and transversus abdominis muscles
c. The iliohypogastric nerve (L1) divides between the internal and external oblique muscles
giving off cutaneous branches
d. The posterior edge of the triangle of Petit is the latissimus dorsi muscle
e. Spread of local anaesthetic to the femoral nerve is not possible as there is not
a continuous plane to this nerve

Question 147
You are asked to assess and provide anaesthesia for a 36-year-old man who was involved in
a road traffic accident 24 hours ago. A CT scan has revealed a possible ligamentous injury of
the cervical spinal column and further investigation with magnetic resonance imaging is
needed. He will remain intubated, ventilated and sedated for the scan on account of his
concurrent aspiration pneumonia. Consider the following statements regarding anaesthetic
monitoring and equipment for this scan:
a. Equipment marked ‘MR conditional’ is suitable for use
b. When using a cardiac monitor appropriate for use with MRI, arrhythmias may be
difficult to detect
c. A standard pulse oximetry probe is suitable for use

Downloaded from https://www.cambridge.org/core. University of Edinburgh, on 19 Aug 2019 at 13:21:27, subject to the Cambridge Core terms of
use, available at https://www.cambridge.org/core/terms. https://doi.org/10.1017/9781108566100.011
368 Chapter 5a: Clinical Anaesthesia Questions

d. There may be a delay of up to 50 seconds in the capnography waveform


e. The ECG leads should be short and of low impedance

Question 148
Which of the following are warning signs of an airway burn?
a. Singed nasal hair
b. Carboniferous sputum
c. Hoarse voice
d. Carbon monoxide levels >1%
e. Burns occurring in an enclosed space

Question 149
Concerning the management of miscarriage:
a. A threatened miscarriage is defined as vaginal bleeding at <28 weeks’ gestation
b. Patients may present with sepsis of unknown origin
c. Serum β-hCG will halve every two days in miscarriage
d. Rhesus-positive women will require anti-D if there is significant bleeding
e. Uterine perforation is a potential complication of surgical management

Question 150
A 40-year-old man is rescued following a fall through ice into a cold lake. On arrival in the
ED, he is in cardiac arrest. Which of the following are true?
a. Cardiac dysrhythmias associated with hypothermia are often exacerbated by rewarming
b. During rewarming, fluid administration should be restricted to avoid pulmonary
oedema
c. Adrenaline should be given at double dose if the core temperature is between 30 and 35 °C
d. Rectal temperature monitoring is the gold standard
e. Cervical spine injuries occur in 35% of drowning victims

Question 151
When anaesthetizing a patient for direct current cardioversion:
a. The patient must be fully anticoagulated
b. A maximum of three attempts at DC cardioversion should be made
c. In a compromised patient, DC cardioversion is appropriate even when the rhythm
cannot be determined
d. A potassium of 2.8 mmol.l–1 is a contraindication to elective cardioversion
e. Successful cardioversion is more likely when the self-adhesive pads are placed in the
anteroposterior position

Question 152
The following are true regarding the use of inotropic support in the management of
neurogenic shock following a high thoracic spinal cord injury:

Downloaded from https://www.cambridge.org/core. University of Edinburgh, on 19 Aug 2019 at 13:21:27, subject to the Cambridge Core terms of
use, available at https://www.cambridge.org/core/terms. https://doi.org/10.1017/9781108566100.011
Chapter 5a: Clinical Anaesthesia Questions 369

a. Vasopressors may be initiated early to avoid volume overload


b. Phenylephrine can be safely used until central venous access is obtained
c. Noradrenaline should be the first-line agent once vasopressors are indicated
d. Associated bradycardia should respond to atropine
e. Adrenaline has no role in the management of neurogenic shock

Question 153
With regard to general anaesthesia for caesarean section:
a. Propofol is now the induction agent of choice
b. Suxamethonium crosses the placenta, unlike non-depolarizing agents
c. Opiates should be given early to reduce the incidence of maternal awareness
d. In a category 1 caesarean, surgery should be commenced as soon as the patient is asleep
e. Cricoid pressure should be at a force of 30 N when the patient is asleep

Question 154
Regarding pneumonectomy:
a. The average mortality is approximately 20%
b. The incidence of acute lung injury after pneumonectomy is 4%
c. Left pneumonectomy carries a higher risk than right pneumonectomy
d. Cardiac herniation is more common after right-sided pneumonectomy
e. Extrapleural pneumonectomy has a higher incidence of morbidity and mortality

Question 155
Sickle cell disease has a number of infectious consequences. Which of the following
organisms should be vaccinated against?
a. Pneumococcus
b. Mycobacterium tuberculosis
c. Salmonella
d. Haemophilus influenzae B
e. Neisseria meningitidis

Question 156
You are asked to attend the ED urgently to assess a suspected case of epiglottis in a 5-year-
old child. Which of the following suggest a diagnosis of epiglottitis rather than croup (acute
laryngotracheobronchitis)?
a. Rapid onset
b. Axillary temperature of 39 °C
c. Haemophilus influenzae type B
d. Barking cough
e. Signs of dysphagia, dysphonia, drooling and respiratory distress

Downloaded from https://www.cambridge.org/core. University of Edinburgh, on 19 Aug 2019 at 13:21:27, subject to the Cambridge Core terms of
use, available at https://www.cambridge.org/core/terms. https://doi.org/10.1017/9781108566100.011
370 Chapter 5a: Clinical Anaesthesia Questions

Question 157
The immediate management of suspected anaphylaxis in an adult under general anaesthe-
sia, spontaneously ventilating via a LMA, should include:
a. Intravenous adrenaline
b. Administration of a neuromuscular blocking agent and ventilation
c. 20 ml.kg–1 colloid bolus
d. Intravenous hydrocortisone 200 mg and IV chlorpheniramine 10 mg
e. Intravenous salbutamol

Question 158
During the postoperative ITU stay following a revision hip replacement a patient develops
a tachycardia. Which of the following features would suggest TRALI (transfusion-related
acute lung injury)?
a. Fever
b. Hypotension
c. Myalgia
d. Hypoxaemia
e. Headache

Question 159
A patient is listed for a temperomandibular joint replacement. Preoperative assessment
reveals limited mouth opening and the decision is taken for an awake fibreoptic intubation
(AFOI). Which of the following is true regarding anaesthesia to the larynx for AFOI?
a. Topical anaesthesia using a ‘spray as-you go’ technique with 4% lidocaine can provide
effective anaesthesia
b. Topical anaesthesia using a ‘spray as-you go’ technique with 2% lidocaine can provide
effective anaesthesia
c. Nebulized 4% lidocaine for 15 minutes can provide effective anaesthesia
d. Superior laryngeal nerve block is adequate as a solo technique to provide adequate
anaesthesia for intubation
e. Translaryngeal block with 2% lidocaine can provide effective analgesia

Question 160
The following can improve the absorption of local anaesthetics used in regional blockade:
a. Bicarbonate
b. Adrenaline
c. Clonidine
d. Local infection
e. High pH

Question 161
With regard to the screening of donated blood, which tests does the World Health
Organization regard as a minimum?

Downloaded from https://www.cambridge.org/core. University of Edinburgh, on 19 Aug 2019 at 13:21:27, subject to the Cambridge Core terms of
use, available at https://www.cambridge.org/core/terms. https://doi.org/10.1017/9781108566100.011
Chapter 5a: Clinical Anaesthesia Questions 371

a. Hepatitis B surface antigen


b. Antibody to hepatitis C
c. Antibody to HIV
d. Antibodies to malaria
e. Serology for syphilis

Question 162
A 44-year-old patient presents to your pain clinic with a two-year history of left radicular leg
pain radiating to the foot. He has had previous spinal surgery, but is now not considered
a surgical candidate and has not responded to antineuropathic medications. You discuss
spinal cord stimulation as a possible management strategy. With regard to spinal cord
stimulation (SCS), which of the following are true?
a. It is more likely to be effective for axial pain rather than limb pain
b. A background of significant depression would be considered a contraindication
c. Future neuraxial block will be safe with antibiotic cover
d. Future surgical procedures should utilize bipolar diathermy only
e. The presence of a cardiac pacemaker is an absolute contraindication to SCS

Question 163
The following are beneficial in the recognition of venous air embolism:
a. End tidal CO2
b. An oesophageal stethoscope
c. Transcranial Doppler
d. A central venous line
e. Transoesophageal echocardiography

Question 164
People with stable neurological symptoms from acute non-disabling stroke or TIA who
have symptomatic carotid stenosis of 70–99% should:
a. Receive best medical treatment (control of blood pressure, antiplatelet agents,
cholesterol-lowering through diet and drugs, lifestyle advice)
b. Be assessed and referred for carotid endarterectomy within one week of onset of stroke
or TIA symptoms
c. Have a cerebral angiogram within ten days of onset of stroke or TIA symptoms
d. Undergo surgery within a maximum of two weeks of onset of stroke or TIA symptoms
e. Be given regional anaesthesia for surgery as this allows earlier detection of cerebral
ischaemia, earlier shunting and reduces the risk of further strokes compared to general
anaesthesia

Question 165
You have been asked to anaesthetize a patient for a total hip replacement. You decide upon
a spinal as your mode of anaesthesia. Concerning intrathecal opioids:
a. Fentanyl should not be used as it may cause delayed respiratory depression

Downloaded from https://www.cambridge.org/core. University of Edinburgh, on 19 Aug 2019 at 13:21:27, subject to the Cambridge Core terms of
use, available at https://www.cambridge.org/core/terms. https://doi.org/10.1017/9781108566100.011
372 Chapter 5a: Clinical Anaesthesia Questions

b. Intrathecal opiates will bind to G-protein-linked receptors in laminae V and VI


c. Intrathecal opioids decrease the release of GABA
d. Potency of intrathecal opioids increases with hydrophobicity
e. Fentanyl has a larger volume of distribution in the spinal cord than morphine

Question 166
Regarding use of the day-case unit:
a. Emergency cases are not suitable for day case
b. Spinal anaesthesia may be suitable for selected day-case patients
c. Follow-up should be by agreed protocols
d. Acutely confused elderly people must be admitted to a suitable hospital
e. All day-case units should participate in audit and quality improvement

Question 167
Factors suggestive that usual preoperative starvation times are inadequate include:
a. Pain
b. Pregnancy
c. Administration of opiates
d. Consumption of alcohol
e. Administration of glycopyrrolate

Question 168
Regarding regional anaesthesia for trauma:
a. It is not indicated if there is a risk of acute compartment syndrome
b. It can be performed in patients with pre-existing nerve injury after individual risk–
benefit assessment
c. If patients are on prophylactic dose of rivaroxaban, central neuraxial blocks can be
performed within 18 hours of the last dose
d. An inability to elicit motor response to peripheral nerve stimulation confirms the needle
is not in contact with the targeted nerve
e. Pain is a sensitive indicator for diagnosing acute compartment syndrome

Question 169
The following clinical features are criteria for diagnosing ICU-acquired weakness:
a. Weakness developing after critical illness
b. Flaccid weakness affecting distal more than proximal muscles
c. Dependence on mechanical ventilation
d. Presence of autonomic dysfunction
e. Involvement of two or more cranial nerves

Question 170
The physiological changes associated with pregnancy have the following clinical
implications:

Downloaded from https://www.cambridge.org/core. University of Edinburgh, on 19 Aug 2019 at 13:21:27, subject to the Cambridge Core terms of
use, available at https://www.cambridge.org/core/terms. https://doi.org/10.1017/9781108566100.011
Chapter 5a: Clinical Anaesthesia Questions 373

a. An increase in the rate of difficult intubation to around 1:150


b. Prolonged action of suxamethonium due to a reduced action of plasma cholinesterase by
around 25%
c. CSF volume is reduced and therefore a lower dose of local anaesthetic is required for
subarachnoid block
d. Care should be taken performing epidurals as CSF pressure can reach up to 70 mmHg
during contractions
e. At term, oxygen consumption is at least 50% higher than prepregnancy levels

Question 171
Elective thyroid surgery should be delayed until the patient is rendered euthyroid.
Regarding carbimazole:
a. It is a prodrug
b. It inhibits thyroid peroxidase
c. It renders a hyperthyroid patient euthyroid in 48–72 hours
d. It is safe to use in pregnancy
e. It should be given on the day of surgery

Question 172
A 5-year-old presents to the ED with suspected epiglottitis. The management of children
with epiglottitis should include the following:
a. Direct inspection of the epiglottitis using a tongue depressor
b. Immediate lateral neck X-ray to aid diagnosis
c. Early intravenous steroids
d. Early intravenous access
e. Inhalational induction with an oxygen/sevoflurane mix

Question 173
Regarding local anaesthetic toxicity:
a. Local anaesthetics inhibit mitochondrial energy production
b. There is a triphasic response
c. Intralipid removes local anaesthetic from the plasma phase
d. It is more likely in patients with carnitine deficiency
e. Only occurs at time of infiltration

Question 174
Regarding blood transfusions, what would prompt the request for irradiated blood?
a. Intrauterine infusions
b. DiGeorge syndrome
c. Donation from first-degree relative
d. Hodgkin’s disease
e. Neonatal exchange transfusion

Downloaded from https://www.cambridge.org/core. University of Edinburgh, on 19 Aug 2019 at 13:21:27, subject to the Cambridge Core terms of
use, available at https://www.cambridge.org/core/terms. https://doi.org/10.1017/9781108566100.011
374 Chapter 5a: Clinical Anaesthesia Questions

Question 175
A patient is having an elective surgical procedure under general anaesthetic using an LMA.
On incision the patient coughs and there is evidence of gastric secretions within the lumen
of the LMA. The patient is quickly intubated but you are concerned about the risk of
aspiration. The following are true regarding aspiration:
a. Occurs in approximately 1 in 2000 elective general anaesthetics
b. Most commonly affects the right middle and lower lobes when the patient is in the
supine position
c. Results in subsequent aspiration pneumonia in approximately 20% of cases
d. The aspiration pneumonia is most commonly due to a Gram-positive organism
e. Patients should be treated empirically with broad spectrum antibiotics

Question 176
Eisenmenger’s syndrome is characterized by the following:
a. Left to right shunt
b. Central cyanosis
c. Mean pulmonary artery pressure is greater than 25 mmHg at rest
d. Good prognosis after correcting the cardiac defect
e. Could be iatrogenic after corrective cardiac surgery

Question 177
Regarding transfusion compatibility:
a. Blood group O is considered a ‘universal recipient’
b. Blood group AB is considered a ‘universal donor’
c. Group B+ can be given to a B– recipient
d. Group B– can be given to a B+ recipient
e. In an emergency Group O+ blood can be transfused

Question 178
A 35-year-old female is listed for a day-case laparoscopic cholecystectomy. She takes
methadone 150 mg daily, but has no other significant past medical history. Her liver
function is normal.
a. Her methadone should be converted to an equivalent IV morphine dose
b. Preoperative ECG is not indicated
c. Preoperative electrolytes are necessary
d. Methadone has poor oral bioavailability
e. Ondansetron is an appropriate antiemetic

Question 179
Regarding the potential causes of Parkinsonism, the following mechanisms can be
implicated:
a. Genetic predisposition

Downloaded from https://www.cambridge.org/core. University of Edinburgh, on 19 Aug 2019 at 13:21:27, subject to the Cambridge Core terms of
use, available at https://www.cambridge.org/core/terms. https://doi.org/10.1017/9781108566100.011
Chapter 5a: Clinical Anaesthesia Questions 375

b. Prochlorperazine induced
c. Normal pressure hydrocephalus
d. Acquired immunodeficiency virus
e. Multi-infarct disease

Question 180
Sub-Tenon’s blocks should be avoided in patients with the following:
a. Conjunctivitis
b. Ocular pemphigoid
c. Ocular Steven Johnson’s syndrome
d. Scleral banding
e. Patients who have had previous sub-Tenon’s block

Question 181
A 50-year-old woman presents for a hysterectomy for menorrhagia and tells you she
previously was a difficult intubation. You decide to perform the operation under combined
spinal–epidural anaesthesia (CSE). Concerning CSE:
a. Both mid-line and paramedian techniques can be used
b. Failure of the spinal component is more common with ‘needle-through-needle’
techniques
c. The rate of post-dural-puncture headache is approximately five times higher with CSE
than epidural anaesthesia alone
d. CSE poses a higher risk of infection than either spinal or epidural anaesthesia in
isolation
e. In obstetrics, CSEs have been shown to improve analgesia and muscle relaxation in
comparison to epidurals alone

Question 182
Regarding discharge following day surgery, patients should be advised:
a. Not to drive for 24 hours or until pain from surgery permits safe car control
b. Not to consume alcohol for 24 hours
c. What problems to be aware of following surgery
d. Whom to contact for help should a problem arise
e. To commence oral analgesics once a local anaesthetic block has clearly receded

Question 183
Regarding the preoperative management of diabetes:
a. It is imperative to place patients with diabetes first on the theatre list
b. Patients with Type I diabetes must be converted to a variable rate intravenous insulin
infusion preoperatively
c. Blood glucose should be measured in the hour prior to anaesthetizing the patient
d. No patient with Type II diabetes requires a variable rate intravenous insulin infusion
e. Long-acting insulin must be stopped preoperatively

Downloaded from https://www.cambridge.org/core. University of Edinburgh, on 19 Aug 2019 at 13:21:27, subject to the Cambridge Core terms of
use, available at https://www.cambridge.org/core/terms. https://doi.org/10.1017/9781108566100.011
376 Chapter 5a: Clinical Anaesthesia Questions

Question 184
One-lung ventilation typically involves the use of double lumen tubes. Regarding double
lumen tubes:
a. Double lumen tubes sizes are given in French gauge
b. French gauge is the same as wire gauge
c. A 39Ch tube has an external diameter of approximately 20 mm
d. The Carlens double lumen tube was designed to sit in the left main bronchus
e. Right-sided tubes are generally preferred to enable ventilation of the right upper lobe

Question 185
The following interventions are considered current best practice in the management of
septic shock:
a. Start vasopressors if BP is not responding to initial fluid resuscitation and CVP is above
8 mmHg
b. Start low-dose corticosteroids after performing ACTH stimulation test (short synacthen
test)
c. Maintain peak airway pressure less than 40 cmH20 if the patient is mechanically
ventilated
d. Start activated protein C if the APACHE score is higher than 25 and in the absence of
contraindications
e. Insert central line early and aim for a central venous oxygen saturation (ScvO2)
above 70%

Question 186
Direct oral anticoagulants (DOACs) are being increasingly used in patients who require
anticoagulation. The American Society of Regional Anesthesia recently updated their
guidelines regarding the cessation of DOACs before neuraxial anaesthesia (April 2018).
With regard to these guidelines, which of the following are true?
a. Apixaban should be omitted 48 hours before neuraxial block
b. Rivaroxaban should be omitted 72 hours before neuraxial block
c. Renal function should be assessed prior to omitting dabigatran
d. No more than 3.1% of the drug should remain in the system prior to intrathecal block
e. There should be an interval of 5 half lives between stopping DOACs and intrathecal
block

Question 187
The following are true regarding revised cardiac risk index scoring (BTS Guidelines 2011):
a. All thoracic surgery patients get 1 point in the high-risk surgery group
b. Treated hypertension is considered a risk factor
c. Preoperative serum creatinine >177 mmol.l–1 is a risk factor
d. All diabetic patients are at increased risk of postoperative cardiac complications
e. With three risk factors, the predicted incidence of cardiac complications is 11%

Downloaded from https://www.cambridge.org/core. University of Edinburgh, on 19 Aug 2019 at 13:21:27, subject to the Cambridge Core terms of
use, available at https://www.cambridge.org/core/terms. https://doi.org/10.1017/9781108566100.011
Chapter 5a: Clinical Anaesthesia Questions 377

Question 188
A macrocytic anaemia may be caused by vitamin B12 deficiency. What are the causes of
a vitamin B12 deficiency?
a. Metformin
b. Acute blood loss
c. Intrinsic factor deficiency
d. Chronic tapeworm infestation
e. Pregnancy

Question 189
The following are true of laryngotracheobronchitis (croup):
a. Commonly affects children of school age
b. The onset is usually abrupt
c. Symptoms include a barking cough, hoarseness and stridor
d. Management includes oxygen, hydration, steroids and nebulized adrenaline
e. Antibiotics should be administered early

Question 190
A primigravida with a BMI of 40 undergoes spinal anaesthesia for a category 1 caesarean
section. She starts desaturating, has a sensory level to cold of C4 and weak grip strength.
The following are risk factors for high spinal:
a. Using plain bupivacaine
b. The volume of local anaesthetic injected
c. Using a fine gauge spinal needle
d. Obesity
e. Pregnancy

Question 191
When assessing the electrolyte concentration of intravenous fluids, which of the following
have a sodium concentration higher than plasma concentration?
a. 0.9% sodium chloride
b. Hartmann’s solution
c. 5% dextrose
d. 8.4% sodium bicarbonate
e. 4% dextrose/0.18% sodium chloride

Question 192
You are required to perform anaesthesia on a patient with an existing tracheostomy.
Regarding tracheostomies:
a. After initial insertion, the tracheostomy should not be changed for at least two weeks to
allow a tract to form
b. Tracheostomies are described in terms of their outer diameter

Downloaded from https://www.cambridge.org/core. University of Edinburgh, on 19 Aug 2019 at 13:21:27, subject to the Cambridge Core terms of
use, available at https://www.cambridge.org/core/terms. https://doi.org/10.1017/9781108566100.011
378 Chapter 5a: Clinical Anaesthesia Questions

c. In fenestrated tracheostomies the fenestration lies below the level of the cuff
d. Percutaneous tracheostomies are associated with a higher stoma infection rate than
surgical tracheostomies
e. The procedure-related mortality with percutaneous tracheostomy is approximately 2%

Question 193
The following are common features of the hypoplastic left heart syndrome:
a. Stenosis of the aortic valve
b. Atresia of the mitral valve
c. Patent foramen ovale
d. Ventricular septal defect
e. Patent ductus arteriosus

Question 194
Regarding blood transfusion administration:
a. Minimum patient identifiers are last name, first name and date of birth
b. All components must be given through an administration set with a 170–200 μm
integral mesh filter
c. Acute transfusion reactions present within one hour of transfusion
d. Symptoms of an acute transfusion reaction include: fever (>2 °C rise or >39 °C), rigors,
myalgia, nausea or vomiting and/or loin pain
e. TRALI is most associated with packed red cell transfusions

Question 195
A 72-year-old female with postherpetic neuralgia (PHN) affecting the left chest wall for 12
months attends your pain-management clinic. In the treatment of PHN, which of the
following statements are true?
a. Antiviral drugs reduce acute pain and severity but have no effect on PHN severity
b. The use of antidepressants is not supported by evidence
c. Gabapentin and pregabalin have a similar efficacy
d. Lidocaine gel has been shown to be useful
e. Opioids are relatively contraindicated

Question 196
Consider whether the following statements regarding myasthenic syndrome are true or
false:
a. There is decreased release of acetylcholine from the presynaptic nerve terminal
b. Usually affects distal limb muscles to a greater extent than proximal muscles
c. Classically, motor function improves with exercise
d. Patients show increased sensitivity to both depolarizing and non-depolarizing muscle
relaxants
e. Motor power is improved only slightly with neostigmine

Downloaded from https://www.cambridge.org/core. University of Edinburgh, on 19 Aug 2019 at 13:21:27, subject to the Cambridge Core terms of
use, available at https://www.cambridge.org/core/terms. https://doi.org/10.1017/9781108566100.011
Chapter 5a: Clinical Anaesthesia Questions 379

Question 197
The following are recognized symptoms of cerebral hyperperfusion syndrome:
a. Headache
b. Intracerebral haemorrhage
c. Facial nerve palsy
d. Hyperalgesia
e. Seizures

Question 198
The following adults require referral to a specialized burns centre:
a. 30% burn
b. 15% burn with inhalational injury
c. Significant burn to hands or feet
d. Patients who are pregnant
e. Any non-blanching circumferential burn

Question 199
The following nerve blocks may be appropriate for a woman undergoing a hysterectomy
with a Pfannenstiel incision:
a. Rectus sheath block
b. Iliohypogastric nerve block
c. Transversus abdominis plane block
d. Lumbar plexus block
e. Ilioinguinal block

Question 200
A patient with long-standing myasthenia gravis (MG) is admitted to hospital with pyelo-
nephritis. After two days in hospital, she is referred to the critical care team with respiratory
failure.
a. Her respiratory failure may be due to the use of ciprofloxacin to treat her infection
b. Her respiratory failure may be due to use of gentamicin to treat her infection
c. Failure of power to improve following administration of edrophonium proves she
cannot have MG
d. If intubation is required, suxamethonium should be used at a reduced dose
e. If intubation is required, rocuronium should be used at a reduced dose

Question 201
Regarding medications used in the management of diabetes mellitus:
a. Pioglitazone increases hepatic sensitivity to insulin
b. Liraglutide is a dipeptidylpeptidase-4 inhibitor
c. Sitagliptin is given by subcutaneous injection

Downloaded from https://www.cambridge.org/core. University of Edinburgh, on 19 Aug 2019 at 13:21:27, subject to the Cambridge Core terms of
use, available at https://www.cambridge.org/core/terms. https://doi.org/10.1017/9781108566100.011
380 Chapter 5a: Clinical Anaesthesia Questions

d. Metglitinides increase insulin secretion


e. Acarbose increases insulin secretion

Question 202
Pugh’s modification to Child’s score estimates mortality in liver disease patients undergoing
surgery. The score is made up of:
a. Grade of encephalopathy
b. Creatinine
c. INR
d. Serum bilirubin
e. ALT

Question 203
Which of the following is true regarding the different types of renal replacement therapy?
a. Dialysis is more effective than filtration in removing middle-sized molecules
b. Both haemodialysis and haemofiltration systems incorporate a semipermeable
membrane
c. Haemofiltration follows Fick’s law
d. Disequilibrium syndrome is more likely to occur with haemofiltration
e. Haemodialysis is more analogous to the renal glomerulus when compared to
haemofiltration

Question 204
The following are means of providing anaesthesia for an emergency (category 1) caesarean
section:
a. General anaesthesia
b. Spinal anaesthesia
c. Epidural anaesthesia
d. Local anaesthetic infiltration
e. Combined spinal and epidural

Question 205
Regarding arrhythmias in the postoperative period in thoracic surgery the following are
true:
a. The most common arrhythmia is SVT
b. They are more common after lobectomies
c. Diltiazem is the most useful drug for post-thoracotomy arrhythmia prophylaxis
d. Digoxin prevents arrhythmias after pneumonectomy or other intrathoracic procedures
e. Decrease in pulmonary vascular bed resistance is thought to be a cause for arrhythmias

Question 206
A morbidly obese patient presents for an elective laparoscopic cholecystectomy. Which of
the following regarding perioperative management are true?

Downloaded from https://www.cambridge.org/core. University of Edinburgh, on 19 Aug 2019 at 13:21:27, subject to the Cambridge Core terms of
use, available at https://www.cambridge.org/core/terms. https://doi.org/10.1017/9781108566100.011
Chapter 5a: Clinical Anaesthesia Questions 381

a. Ramping involves a position where the incisors are level with the sternum
b. Higher PEEPs have been shown to be detrimental
c. Use short-acting opioids
d. Depth of anaesthesia monitors are not reliable
e. An RSI is mandatory in all patients with a BMI >45

Question 207
With regards to cerebral palsy (CP):
a. Two-thirds of patients have impaired cognitive function
b. 10% of patients have either focal or generalized forms of epilepsy
c. Gaseous inhalational induction of anaesthesia is contraindicated because of the risk of
gastro-oesophageal reflux
d. Succinylcholine may be safely used in CP patients
e. Regional analgesic techniques can reduce painful muscle spasm

Question 208
A 84-year-old female is undergoing cemented hemiarthroplasty for fractured neck of femur
under spinal anaesthesia with sedation. She suddenly desaturates at the time of prosthesis
insertion. You suspect bone cement implantation syndrome (BCIS). Consider the following:
a. Ischaemic heart disease does not increase the risk of BCIS
b. Features of BCIS include hypoxia, hypertension and cardiac arrhythmias
c. Pulmonary vascular resistance would be expected to be >200 dyn.s.cm–5 in BCIS
d. Desaturation may be short lived
e. Management should include cardiothoracic review for urgent embolectomy

Question 209
You are anaesthetizing an 82-year-old female for a revision hip replacement. You are using
a cell saver system. Complications of cell salvage include:
a. Amniotic fluid embolism
b. Fluid overload
c. Haemolysis
d. Coagulopathy
e. Thromboembolism

Question 210
You have been asked to anaesthetize a two-year-old child with Down’s syndrome for an
inguinoscrotal hernia repair. He appears otherwise fit and well and has no cardiovascular or
renal disease. Which of the following statements are true?
a. There is an increased risk of atlantoaxial dislocation during intubation
b. Caudal anaesthesia is an appropriate analgesic technique
c. Down’s syndrome is an absolute indication for a preoperative ECG
d. There is an increased incidence of obstructive sleep apnoea amongst Down’s syndrome
patients

Downloaded from https://www.cambridge.org/core. University of Edinburgh, on 19 Aug 2019 at 13:21:27, subject to the Cambridge Core terms of
use, available at https://www.cambridge.org/core/terms. https://doi.org/10.1017/9781108566100.011
382 Chapter 5a: Clinical Anaesthesia Questions

e. There is an increased incidence of Hirschsprung’s disease amongst Down’s syndrome


patients

Question 211
The postnatal circulation in hypoplastic left heart syndrome depends on three major factors:
a. Adequacy of interatrial communication
b. FiO2 supplied to the neonate after birth
c. Patency of the ductus arteriosus
d. Level of pulmonary vascular resistance
e. Gestational age

Question 212
A 62-year-old male is listed for a radical prostatectomy. He is a Jehovah’s Witness, but has
consented to the use of cell salvage. Regarding cell salvage:
a. Operative indications include anticipated blood loss over 1000 ml or 20% estimated
blood volume
b. The suction tip used should be 2 mm and high vacuum pressure applied to maximize
collection
c. It is contraindicated for use in resection of malignant tissue
d. It is approved for use in obstetric practice
e. An anticoagulant is added to the salvaged blood in the reservoir to prevent coagulation

Question 213
The advantages of sedation for potentially painful procedures include:
a. It requires less detailed preprocedural assessment than for general anaesthesia
b. It reduces anxiety
c. No risk of the complications associated with general anaesthesia
d. It reduces pain
e. It provides amnesia

Question 214
You are managing a 55-year-old female with a diagnosis of complex regional pain syndrome
affecting the arm. Which of the following treatments should be considered?
a. Intravenous regional sympathetic blocks, e.g. guanithidine
b. Intravenous bisphosphonate, e.g. pamidronate
c. Tricyclic antidepressants
d. Spinal cord stimulation
e. Cognitive behavioural therapy

Question 215
Indications for intubation and ventilation of patients with brain injury include:
a. Bilaterally fractured mandible

Downloaded from https://www.cambridge.org/core. University of Edinburgh, on 19 Aug 2019 at 13:21:27, subject to the Cambridge Core terms of
use, available at https://www.cambridge.org/core/terms. https://doi.org/10.1017/9781108566100.011
Chapter 5a: Clinical Anaesthesia Questions 383

b. Glasgow coma score of 8 or less


c. Hypoxaemia (paO2 <13 kPa on oxygen)
d. A drop of GCS from 13 (E4/V4/M5) to 11 (E4/V4/M3)
e. Spontaneous hyperventilation causing paCO2 <4.0 kPa

Question 216
The following children require referral to a specialized burn centre:
a. 15% TBSA full-thickness burn
b. Any circumferential burn
c. 15% TBSA if less than one year old
d. Any burn to face
e. Any chemical or electrical burn

Question 217
A 50-year-old woman presents to you for a laparoscopic salpingo-oophorectomy. She tells
you that she has had postoperative nausea and vomiting (PONV) after previous general
anaesthetics. The following are risk factors for developing PONV:
a. Gynaecological surgery
b. Propofol TIVA
c. Age >60
d. No IV fluids given intraoperatively
e. Handling of bowel

Question 218
NAP6 is a national study looking at the incidence of perioperative anaphylactic events.
Regarding anaphylaxis:
a. It may present with isolated refractory hypotension under anaesthesia
b. Ranitidine or cimetidine may be useful for their H2 receptor blockade
c. Mast cell tryptase should be taken as soon as possible, at one to two hours and at seven
days
d. Aminophylline or magnesium may be needed
e. Metaraminol may be needed to supplement an adrenaline infusion

Question 219
When assessing a smoker preoperatively, the following advice is appropriate:
a. The risk of wound infection is reduced if cigarette smoking is stopped four weeks
preoperatively
b. Cigarette smoking should be avoided for at least several hours preoperatively
c. Stopping smoking for several hours preoperatively reduces the effects on the oxyhae-
moglobin dissociation curve
d. Sputum production is likely to be increased in the postoperative period
e. There is an increased risk of postoperative lower respiratory tract infection

Downloaded from https://www.cambridge.org/core. University of Edinburgh, on 19 Aug 2019 at 13:21:27, subject to the Cambridge Core terms of
use, available at https://www.cambridge.org/core/terms. https://doi.org/10.1017/9781108566100.011
384 Chapter 5a: Clinical Anaesthesia Questions

Question 220
Regarding the perioperative management of diabetes:
a. All Type I diabetics must receive a variable rate intravenous insulin infusion
b. Preoperative high carbohydrate drinks should be avoided in patients who receive insulin
for their diabetes
c. Long-acting insulin such as glargine and determir should be discontinued during the
perioperative period
d. 0.45% saline with 5% glucose and potassium chloride is recommended as the substrate
fluid in patients receiving a variable rate intravenous insulin infusion
e. Patients not receiving a variable rate intravenous insulin infusion should receive 0.9%
saline as their substrate fluid

Question 221
The following are true when comparing percutaneous with surgical tracheostomy:
a. A surgically placed tracheostomy tube is less likely to get dislodged
b. A recently sited tracheostomy tube that has become dislodged is easier to replace if it is
a percutaneous rather than a surgical stoma
c. Percutaneous tracheostomy is associated with less infection rate.
d. Bleeding is likely to be less following percutaneous tracheostomy
e. Cervical spine injury is a contraindication for both techniques

Question 222
Regarding neuraxial adjuvants in obstetric anaesthesia:
a. Intrathecal opioids primarily work upon the dorsal horn of the spinal cord
b. Clonidine can safely be added to epidural infusions for labour analgesia
c. Adding sodium bicarbonate to lidocaine prolongs its duration of action
d. Clonidine increases the motor blockade by acting upon α2-receptors on the dorsal horn
of the spinal cord
e. Given intrathecally, highly lipophilic opioids have a lower risk of delayed respiratory
depression than less lipophilic opioids

Question 223
The following factors are correlated with increased risk of desaturation during one-lung
ventilation:
a. High percentage of ventilation or perfusion to the operative lung on preoperative VQ
scan
b. Poor paO2 during two-lung ventilation, particularly in the lateral position
intraoperatively
c. Left-sided thoracotomy
d. Normal preoperative spirometry (FEV1 or FVC)
e. Supine position during one-lung ventilation

Downloaded from https://www.cambridge.org/core. University of Edinburgh, on 19 Aug 2019 at 13:21:27, subject to the Cambridge Core terms of
use, available at https://www.cambridge.org/core/terms. https://doi.org/10.1017/9781108566100.011
Chapter 5a: Clinical Anaesthesia Questions 385

Question 224
Which of the following are true regarding the association between acromegaly and
a potentially difficult airway?
a. Maxillary enlargement
b. Tonsillar enlargement
c. Macroglossia
d. Epiglottis enlargement
e. Reduced neck movement secondary to tissue fibrosis

Question 225
Regarding foreign body aspiration in children:
a. Most common in children aged one to three years
b. A rapid sequence induction must always be performed because an empty stomach
cannot be guaranteed
c. In partial airway obstruction, inspiratory X-ray film will reveal air trapping
d. Cyanosis, stridor and altered level of consciousness are ominous signs and may predict
impending respiratory arrest
e. Without a clear history of choking, the symptoms can be difficult to differentiate from
acute asthma

Question 226
During an ENT resection of a laryngeal tumour with laser, an airway fire occurs.
Which of the following are true?
a. Immediately administer 100% oxygen
b. Flood the surgical field with saline
c. Continue with current endotracheal tube due to risk of airway swelling and losing the
airway if tube change is attempted
d. There is a high risk of inhalational injury
e. Surgical tracheostomy is always required

Question 227
Whilst assessing a patient preoperatively they mention that for religious reasons they do not
want any naturally occurring fluids or drugs. Which of the following are naturally occurring
colloids?
a. Albumin solution
b. Gelatin
c. HES
d. Plasma
e. Dextrans

Downloaded from https://www.cambridge.org/core. University of Edinburgh, on 19 Aug 2019 at 13:21:27, subject to the Cambridge Core terms of
use, available at https://www.cambridge.org/core/terms. https://doi.org/10.1017/9781108566100.011
386 Chapter 5a: Clinical Anaesthesia Questions

Question 228
You are assisting a consultant in performing an awake fibreoptic intubation. The technique
involves a superior laryngeal nerve block. The result of the block will provide effective
anaesthesia to the following structures:
a. Base of the tongue
b. Anterior epiglottis
c. Aryepiglottic folds
d. Vocal cords
e. Trachea

Question 229
The following is true regarding intracranial haemorrhage in the neonate:
a. Intraventricular haemorrhage is the most common type in preterm babies
b. Epidural haemorrhage is the commonest in term babies
c. CT brain is the investigation of choice
d. Epilepsy is one of the long-term effects
e. Conservative and supportive treatment usually lead to excellent results

Question 230
Regarding pulmonary artery flotation catheters:
a. The thermodilution technique always produces an accurate cardiac output
measurement
b. The Fick principle is used for cardiac output measurement
c. It contains a thermocouple
d. The balloon at the catheter tip holds up to 3 ml volume
e. Normal pulmonary capillary wedge pressure is 10–15 mmHg

Question 231
In patients with back pain, the features, signs or symptoms that indicate serious spinal
pathology are known as ‘red flags’. Which of the following are red flags?
a. An absent ankle reflex
b. A positive Hoffman’s sign
c. Pain worse at night
d. Positive sciatic stretch test
e. Intravenous drug use

Question 232
Following brainstem death, the following pathophysiological changes are seen in more than
a quarter of patients:
a. Diabetes insipidus
b. Hypotension
c. Pulmonary oedema

Downloaded from https://www.cambridge.org/core. University of Edinburgh, on 19 Aug 2019 at 13:21:27, subject to the Cambridge Core terms of
use, available at https://www.cambridge.org/core/terms. https://doi.org/10.1017/9781108566100.011
Chapter 5a: Clinical Anaesthesia Questions 387

d. Cardiac arrhythmias
e. Metabolic acidosis

Question 233
The following statements regarding burn injuries are true:
a. A patient who has been intubated for a thermal injury should only have their endo-
tracheal tube cut after 24 hours
b. Carbon monoxide and cyanide prevent aerobic metabolism at the cellular level
c. The treatment for cyanide toxicity is 100% oxygen
d. The half-life of carbon monoxide is reduced to 40 minutes in 100% oxygen at
3 atmospheres
e. Nebulization of heparin and N-acetylcysteine in children with massive burn injury and
smoke inhalation injury reduces mortality

Question 234
A woman presents for a ureteroscopy as a day-case patient. During your preoperative
assessment, she tells you she is currently breast feeding. The following drugs may be
excreted in breast milk:
a. Morphine
b. Paracetamol
c. Thiopentone
d. Tramadol
e. Rocuronium

Question 235
A 50-year-old man is admitted to intensive care with suspected Guillain–Barré syndrome.
The management includes:
a. Intravenous immunoglobulin
b. Plasma exchange
c. Steroids
d. Analgesia
e. Broad-spectrum antibiotics

Question 236
A 68-year-old gentleman with known hypertension presents for an elective inguinal hernia
repair. On the ward his blood pressure is 175/100 mmHg, despite taking his prescribed
medication that morning. It does not improve following repeated measurements. He is
otherwise well and asymptomatic. The following are appropriate:
a. Refer the patient to cardiology
b. Administer intravenous antihypertensives
c. Proceed with the operation
d. Refer the patient back to his GP
e. Check the patient’s urea and electrolytes

Downloaded from https://www.cambridge.org/core. University of Edinburgh, on 19 Aug 2019 at 13:21:27, subject to the Cambridge Core terms of
use, available at https://www.cambridge.org/core/terms. https://doi.org/10.1017/9781108566100.011
388 Chapter 5a: Clinical Anaesthesia Questions

Question 237
The liver is one of the largest organs in the human body and undertakes multiple tasks.
Regarding liver function:
a. 70% of liver blood flow is via the hepatic artery
b. Portal tracts contain lymphatics
c. Plasma cholinesterase is produced by the liver
d. Vitamin E is stored in the liver
e. Plasma concentration of alanine aminotransferase increases in normal pregnancy

Question 238
The following parameters are appropriate when ventilating a patient with life-threatening
asthma:
a. Increased I:E ratio
b. Avoid PEEP
c. Low respiratory rate of 12–14 breaths per minute
d. Tidal volume of 4–8 ml.kg–1
e. Accepting SpO2 >92% and pH >7.2

Question 239
Regarding pregnant women with pre-existing neurological diseases:
a. The most common cause of seizures is epilepsy
b. Regional anaesthesia is contraindicated in patients with benign intracranial
hypertension
c. Myasthenia gravis increases in severity during pregnancy in the majority of patients
d. In multiple sclerosis there is an increased risk of postpartum relapse if an epidural has
been used
e. Epidural blocks may be patchy in patients with neurofibromatosis

Question 240
The following are appropriate doses for administration of preoperative sedation in children:
a. 0.1 mg.kg–1 oral midazolam
b. 0.5 mg.kg–1 IM ketamine
c. 0.5 mg.kg–1 IV midazolam
d. 1 mg.kg–1 oral ketamine
e. 3 μg.kg–1 oral clomidine

Question 241
Neuromuscular disorders have different pathophysiologies. Regarding myotonic dystrophy:
a. It has an autosomal dominant pattern of inheritance
b. It involves an abnormality of potassium channels
c. It involves an abnormality of sodium channels

Downloaded from https://www.cambridge.org/core. University of Edinburgh, on 19 Aug 2019 at 13:21:27, subject to the Cambridge Core terms of
use, available at https://www.cambridge.org/core/terms. https://doi.org/10.1017/9781108566100.011
Chapter 5a: Clinical Anaesthesia Questions 389

d. Hypothermia may precipitate myotonias


e. Aspiration may be a complication

Question 242
Regarding sepsis in children:
a. Neisseria meningitides, Haemophilus influenzae and Streptococcus pneumoniae are the
most common causes of sepsis in children over three months
b. The most useful haemodynamic and oxygenation variables for assessing the severity of
shock are BP, ScvO2 and SaO2
c. The appropriate initial resuscitation fluid and volume for a child in septic shock would
be 20 ml.kg–1 crystalloid
d. In severe septic shock the capillary refill time may be normal
e. Bradypnoea and apnoea are late, prearrest findings

Question 243
The following statements are true with regards to suxamethonium apnoea:
a. 80% of people are homozygous E1u:E1u (dibucaine number 80)
b. E1s:E1s is the rarest genetic variation
c. The dibucaine number for E1s:E1s is 0
d. E1s:E1s is more common in Asian patients
e. E1u:E1s has the same dibucaine number as E1u:E1u

Question 244
A patient is brought to A&E following an RTA. He requires cervical spine stabilization and
has a hard cervical collar in place. You elect to site a central venous catheter in the femoral
vein. With regards to the anatomy of the femoral vein:
a. It lies inferior to the inguinal ligament
b. It lies in a sheath with the femoral artery and nerve
c. It is medial to the artery
d. It is the continuation of the popliteal vein
e. It becomes the internal iliac vein

Question 245
Regarding the use of extracorporeal shock-wave lithotripsy (ESWL) to treat renal stones:
a. Calcium phosphate stones are the most common type of renal stone
b. ESWL is most suitable for stones <2 cm in diameter
c. ESWL cannot be used for stones located high in the ureter
d. A permanent pacemaker is a contraindication to ESWL
e. If an epidural is performed it is preferable to use a loss-of-resistance (LOR) to air
technique as opposed to LOR to saline

Downloaded from https://www.cambridge.org/core. University of Edinburgh, on 19 Aug 2019 at 13:21:27, subject to the Cambridge Core terms of
use, available at https://www.cambridge.org/core/terms. https://doi.org/10.1017/9781108566100.011
390 Chapter 5a: Clinical Anaesthesia Questions

Question 246
At 32/40 gestation a poor CTG trace has resulted in a category I caesarean section
performed under general anaesthesia. The male neonate remained floppy with no respira-
tory effort and required intubation and transfer to NICU. The following could be
a differential diagnosis to his presentation at birth:
a. Intracranial haemorrhage
b. Pulmonary hypoplasia
c. Extended effect of the maternal drugs from the general anaesthesia
d. Respiratory distress syndrome
e. Immaturity of the respiratory centre

Question 247
Electrical nerve stimulation remains an important part of regional anaesthetic practice.
Which of the following statements is true?
a. Rheobase is the minimum current in milliamps required to create a nerve impulse
b. Chronaxie is the minimum duration of stimulus, at twice the rheobase, that must be
applied to the nerve to initiate an impulse
c. Chronaxie varies depending on the type of nerve
d. The stimulating needle is connected to the cathode
e. The ECG pad is connected to the anode

Question 248
The following are true of pain measurement scales:
a. The numeric rating scale is a ten-point scale
b. The verbal rating scale is insensitive to small changes in pain
c. The visual analogue scale is sensitive to small changes in pain
d. The brief pain inventory is used to measure pain in the cognitively impaired
e. The CHEOPS method is inappropriate for children

Question 249
Concerning grading systems for subarachnoid haemorrhage (SAH):
a. A patient with a World Federation of Neurosurgeons grade 1 SAH presents with
a Glasgow coma score of 14–15 with no motor deficit
b. A patient with a World Federation of Neurosurgeons grade 5 SAH presents with
a Glasgow coma score of 3–6, with or without motor deficit
c. The Fisher scale classifies the severity of subarachnoid haemorrhage based on CT scan
appearances alone
d. SAH presenting with moderate to severe hemiparesis is classified as grade 3 in the Hess
and Hunt classification
e. SAH presenting with no neurological deficit other than cranial nerve palsy is classified as
grade 2 in the Hess and Hunt classification

Downloaded from https://www.cambridge.org/core. University of Edinburgh, on 19 Aug 2019 at 13:21:27, subject to the Cambridge Core terms of
use, available at https://www.cambridge.org/core/terms. https://doi.org/10.1017/9781108566100.011
Chapter 5a: Clinical Anaesthesia Questions 391

Question 250
The Department of Health issued ‘Saving Lives: High Impact Interventions Guidance and
Tools’ for:
a. Central venous catheter care
b. Peripheral venous catheter care
c. Taking blood cultures
d. Prevention of surgical site infections
e. Surgical drain care

Question 251
A multiparous woman presents for a sterilization. Concerning female sterilization:
a. It must be performed in the first week of her menstrual cycle
b. Has a lower failure rate than vasectomy
c. Has a lower failure rate than the Mirena IUD
d. Can only be carried out on multiparous women
e. Cannot be reversed

Question 252
A 75-year-old man with myasthenia gravis, COPD and angina undergoes an emergency
laparotomy for a sigmoid volvulus. Which of the following increase the risk of his need for
postoperative ventilation?
a. Co-existent COPD
b. Co-existent ischaemic heart disease
c. Abdominal surgery
d. Diagnosis more than six years ago
e. Pyridostigmine dose of 1000 mg.day–1

Question 253
According to the NICE hypertension guidelines 2011 the following are true:
a. The first-line agent in a patient over 55 years old is an ACE inhibitor
b. The first-line agent in a black person of African or Caribbean family history of any age is
a calcium-channel blocker
c. If an ACE inhibitor is not tolerated, an angiotensin receptor blocker should be
considered
d. Diuretics are considered third-line agents
e. If diuretic therapy is considered, thiazides are the treatment of choice

Question 254
A 64-year-old female has been booked for emergency surgery for a suspected necrotizing
fasciitis. She is known to be diabetic and hypertensive. Prior to induction she has a pulse of
110 min–1, a BP of 110/45 mmHg and an RR of 32 min–1. She has a large bore cannula and an
arterial line inserted by you. At induction, the patient’s BP drops to 60/40 mmHg.
An appropriate plan of action constitutes which of the following?

Downloaded from https://www.cambridge.org/core. University of Edinburgh, on 19 Aug 2019 at 13:21:27, subject to the Cambridge Core terms of
use, available at https://www.cambridge.org/core/terms. https://doi.org/10.1017/9781108566100.011
392 Chapter 5a: Clinical Anaesthesia Questions

a. Administer 500 ml of starch-based colloid


b. Insert a central line and start noradrenaline infusion
c. Postpone surgery until the patient is stabilized on the ICU
d. Start regular IV clindamycin after obtaining blood cultures
e. Start insulin infusion to keep blood glucose below 10 mmol.l−1

Question 255
In the pregnant woman undergoing non-obstetric surgery:
a. Anaesthetic agents are teratogenic
b. Ketamine should be avoided as it increases uterine tone
c. Laparoscopic surgery is considered to be no higher risk to the fetus than standard
laparotomy
d. Routine CTG monitoring should take place perioperatively
e. Volatile agents reduce fetal heart rate and its variability

Question 256
Rheumatoid arthritis is a systemic disease with many manifestations affecting anaesthesia.
What are the risk factors for development of rheumatoid arthritis?
a. Cigarette smoking
b. HLA-DR7 subtype
c. Food allergies
d. Exposure to heavy metals
e. Childhood rheumatic fever

Question 257
Regarding craniopharyngioma:
a. Craniopharyngiomas are the third most common benign tumour in children
b. Symptoms are caused by pressure on the pineal gland, hypothalamus and optic chiasm
c. Desmopressin is rarely used perioperatively
d. It can be associated with abnormalities in body temperature regulation
e. MRI is useful to determine the extent of the tumour and for planning surgery

Question 258
A 58-year-old male with end-stage renal failure is undergoing a total knee replacement. He
weighs 80 kg. He is being ventilated via an LMA and receives femoral and sciatic nerve
blocks for postoperative analgesia with a total volume of 30 ml 0.5% bupivacaine. He rapidly
becomes bradycardic then goes into ventricular fibrillation.
a. The patient should be intubated
b. Local anaesthetic toxicity is unlikely as a safe dose was used
c. Propofol infusion should be started
d. Atropine 3 mg should be administered
e. An incident form should be completed after the case

Downloaded from https://www.cambridge.org/core. University of Edinburgh, on 19 Aug 2019 at 13:21:27, subject to the Cambridge Core terms of
use, available at https://www.cambridge.org/core/terms. https://doi.org/10.1017/9781108566100.011
Chapter 5a: Clinical Anaesthesia Questions 393

Question 259
A cardiac ITU nurse has a student with her and is describing the CVP trace. She asks you to
explain why the CVP trace may be abnormal. Which of the following can lead to an
abnormal trace?
a. Atrial fibrillation
b. Complete heart block
c. Mitral stenosis
d. Tricuspid regurgitation
e. Left ventricular failure

Question 260
The benzodiazepine midazolam is a common sedative agent. Which of the following are
true?
a. It can cause pain on injection of the intravenous form
b. It is 89% ionized at physiological pH
c. It has an oral bioavailability of approximately 40%
d. It has active metabolites
e. It undergoes significant metabolism in the kidney

Question 261
Postoperative apnoea in neonates is characterized by the following:
a. Cessation of respiration for 15–20 seconds, which may be associated with desaturation
and bradycardia
b. More common less than 56–60 weeks postgestational age
c. Is less likely to happen if the neonate requires postoperative oxygen supplementation
d. Occurs up to 72 hours postoperatively
e. Episodes are usually self-limiting or require mild stimulation of the baby to encourage
respiration

Question 262
You are anaesthetizing a 48-year-old female for a total thyroidectomy. Considering intra-
operative management of thyroidectomy cases:
a. Neuromuscular monitoring is mandatory to maintain no more than three twitches for
a safe surgical plane
b. An arterial line is mandatory
c. A head-up position aids surgery
d. Depth of anaesthesia monitoring should be considered due to the raised risk of
awareness
e. Cardiovascular collapse in recovery should prompt urgent liothyronine infusion

Question 263
The following effects on respiratory function and clinical consequences are correctly
associated with the relevant spinal cord level:

Downloaded from https://www.cambridge.org/core. University of Edinburgh, on 19 Aug 2019 at 13:21:27, subject to the Cambridge Core terms of
use, available at https://www.cambridge.org/core/terms. https://doi.org/10.1017/9781108566100.011
394 Chapter 5a: Clinical Anaesthesia Questions

a. C1–C3: apnoea and immediate death unless ventilation applied


b. C3–C5: varied impairment of diaphragmatic contraction
c. C3–C5: expiration is entirely passive
d. C6–C8: problematic retention of secretions
e. C1–C8: a third of all patients will require intubation at some point

Question 264
The following are complications of sub-Tenon’s block:
a. Scleral perforation
b. Central spread of local anaesthetic
c. Retrobulbar haemorrhage
d. Subconjunctival haemorrhage
e. Chemosis

Question 265
Regarding congenital diaphragmatic hernia:
a. Elective caesarian section should be performed in all cases
b. Aggressive bag-valve-mask ventilation should be performed to recruit the immature
lung immediately after delivery
c. Oxygen saturation of >95% should be aimed for on initial resuscitation and ventilation
d. Surgical correction should be performed within six hours of delivery
e. 90% of defects are posterolateral

Question 266
Hypothermia may be associated with:
a. Thromboembolism
b. Hyperlactataemia
c. Acute pancreatitis
d. Immunosuppression
e. Improved outcome from traumatic brain injury

Question 267
The following are correct with reference to permanent pacemakers:
a. In three- or five-letter nomenclature codes, the first letter refers to the chamber sensed
b. In five-letter nomenclature, the fourth letter refers to the presence or absence of
antitachycardia functions
c. The code DDD refers to a response to both trigger and inhibit
d. If the fourth letter is a P, this refers to the ability to undergo rate modulation
e. The letter D always refers to both atrium and ventricle

Question 268
A 64-year-old male is admitted to ICU with acute pancreatitis requiring inotropic support.
After two days he develops severe hypoxaemia requiring invasive ventilation. His CXR

Downloaded from https://www.cambridge.org/core. University of Edinburgh, on 19 Aug 2019 at 13:21:27, subject to the Cambridge Core terms of
use, available at https://www.cambridge.org/core/terms. https://doi.org/10.1017/9781108566100.011
Chapter 5a: Clinical Anaesthesia Questions 395

shows bilateral opacities, however his chest is not productive. In spite of being on FiO2 of 1.0
and high PEEP for almost 24 hours, his pO2 continues to drop below 8.0 kPa. Which of the
following techniques can help to improve his oxygenation?
a. Starting furosemide infusion
b. Starting atracurium infusion
c. Prone positioning
d. Changing to high frequency oscillatory ventilation (HFOV)
e. Nitric oxide inhalation

Question 269
Regarding obesity in pregnancy:
a. It results in an increased risk of difficult intubation when compared to the general
population
b. It requires pelvic tilt to be greater than 15° to prevent aortocaval compression
c. It is associated with an increased incidence of postpartum haemorrhage
d. Nerve blocks for post caesarean section analgesia should be avoided due to an increased
incidence of local anaesthetic toxicity in the obese
e. Requires an increase in blood flow and therefore cardiac output of around 2–3 ml.min–1
per 100 g fat

Question 270
Cushing’s syndrome arises from raised levels of circulating glucocorticoids. With regards to
Cushing’s:
a. Hyponatraemia is a feature
b. Metyrapone may be used as a treatment option
c. Hypokalaemia is a feature
d. Entacapone may be used as a treatment option
e. Adrenalectomy may result in Sheehan’s syndrome

Question 271
Which of the following are the strongest predictors of a difficult intubation in an obese
patient?
a. Male gender
b. Females with large breasts
c. Short neck
d. Obstructive sleep apnoea
e. Central obesity

Question 272
An 82-year-old patient is attending preassessment clinic for an endovascular aortic aneur-
ysm repair (EVAR). Regarding anaesthesia for the elderly population:
a. An ECG is a mandatory investigation
b. Cardiopulmonary exercise tests are not possible in patients with severe arthritis

Downloaded from https://www.cambridge.org/core. University of Edinburgh, on 19 Aug 2019 at 13:21:27, subject to the Cambridge Core terms of
use, available at https://www.cambridge.org/core/terms. https://doi.org/10.1017/9781108566100.011
396 Chapter 5a: Clinical Anaesthesia Questions

c. Oxygenation is improved by adopting the sitting position


d. Silent aspiration is common
e. A patient with postoperative confusion is no more likely to develop it following
a subsequent general anaesthetic

Question 273
In a case of suspected local anaesthetic toxicity:
a. Lorazepam 0.1 mg.kg–1 can be given for seizure control
b. Up to three boluses of intralipid can be given four minutes apart
c. Intralipid infusion at 30 ml.kg–1.h–1 is acceptable
d. A maximum dose of 15 ml.kg–1 intralipid 20% can be given
e. Resuscitation efforts should be discontinued after 45 minutes in view of poor prognosis

Question 274
The nurse on the trauma ward contacts you regarding a patient you anaesthetized that day.
She is concerned with the condition of the patient. Which of the following could be
secondary to the use of intrathecal opioids?
a. Pruritus
b. Respiratory depression
c. Urinary incontinence
d. Visual disturbance
e. Sedation

Question 275
The following sedative agents have an action that is significantly increased in renal failure:
a. Propofol
b. Midazolam
c. Dexmedetomidine
d. Ketamine
e. Clonidine

Question 276
The following factors favour the development of kernicterus in the newborn:
a. Hypoalbuminaemia
b. High pH
c. Prematurity
d. Administration of salicylates
e. Antenatal administration of anti-D immunoglobulin to the mother

Question 277
You are performing a sciatic nerve block as part of your management of a total knee
replacement. You favour the posterior approach. Which of the following statements regard-
ing sciatic nerve blocks are true?

Downloaded from https://www.cambridge.org/core. University of Edinburgh, on 19 Aug 2019 at 13:21:27, subject to the Cambridge Core terms of
use, available at https://www.cambridge.org/core/terms. https://doi.org/10.1017/9781108566100.011
Chapter 5a: Clinical Anaesthesia Questions 397

a. The posterior approach is also known as the Labat approach


b. The posterior approach requires identification of the posterior superior iliac spine and
the greater trochanter
c. Nerve stimulation producing plantar flexion of the foot indicates peroneal nerve
stimulation
d. Nerve stimulation producing dorsiflexion of the foot indicates tibial nerve stimulation
e. The medial aspect of the ankle joint will not be covered by a sciatic nerve block

Question 278
Neurolytic agents are used to produce long-lasting pain relief through disabling or destroy-
ing nerves. Classically, these agents are used in the presence of malignant pain due to the
risk of significant morbidity. The following statements regarding neurolytic agents are true:
a. 50–100% alcohol injection can be associated with severe pain
b. 3% phenol injection will spare motor nerve fibres
c. Alcohol is hypobaric
d. Alcohol should not be used for coeliac plexus blocks
e. With phenol, position the patient painful side down

Question 279
Regarding the perioperative management of a patient with coronary stents:
a. Elective surgery should be postponed until the period of dual antiplatelet therapy has
been completed
b. Continuation of aspirin is a contraindication to central neuraxial blockade
c. In procedures with a low risk of bleeding, dual antiplatelet therapy can be continued
throughout the perioperative period
d. The risk of stent thrombosis is greater with drug-eluting stents when compared with
bare metal stents
e. Clopidogrel must be stopped preoperatively

Question 280
The following drugs are known to increase intraocular pressure:
a. Suxamethonium
b. Rocuronium
c. Ketamine
d. Ondansetron
e. Metoclopramide

Question 281
Gastroschisis is a congenital abnormality of the abdominal wall. Which of the following
statements are true?
a. Risk factors include low maternal age, maternal smoking and use of decongestants
b. It is more common in males than females
c. Delayed closure is the preferred management of gastroschisis

Downloaded from https://www.cambridge.org/core. University of Edinburgh, on 19 Aug 2019 at 13:21:27, subject to the Cambridge Core terms of
use, available at https://www.cambridge.org/core/terms. https://doi.org/10.1017/9781108566100.011
398 Chapter 5a: Clinical Anaesthesia Questions

d. Nitrous oxide is recommended as a carrier gas for maintenance of anaesthesia and


analgesia
e. Early feeding to stimulate normal bowel flora development is highly recommended

Question 282
A Type II diabetic patient is listed first in the morning for a laparoscopic hernia repair.
Which of the following are considered appropriate in the management of patients taking
oral hypoglycaemics?
a. Omit metformin on morning of surgery
b. Pioglitazone should be taken as normal
c. Meglitnides such as repaglinide and nateglinide should be omitted on the morning of
surgery
d. Glucagon-like peptide-1 inhibitors such as exenatide and liraglutide should be taken as
normal on the day of surgery
e. Sulfonylureas such as glibenclamide should be taken on the day of surgery

Question 283
The following interventions have a strong level of evidence regarding the prevention of
ventilator-associated pneumonia:
a. The use of chlorhexidine oral rinse
b. Good hand hygiene
c. Selective gut decontamination
d. Daily interruption of sedation (sedation holding)
e. Regular suctioning of subglottic secretions

Question 284
When preparing a patient for phaeochromocytoma surgery:
a. High doses of α-blockade should be used initially to prevent further organ damage
b. Labetalol may be used as a sole agent due to its combined α- and β-blockade
c. Serum blood glucose should be measured
d. Echocardiography should be considered
e. Postural drop in blood pressure preoperatively is indicative of excess α-blockage

Question 285
Concerning thromboembolic disease in pregnancy:
a. It is the leading cause of maternal mortality in the UK
b. Obstetric intervention is considered the most important risk factor
c. In patients with suspected pulmonary embolism, V/Q scans have a higher diagnostic rate
in pregnant women when compared to non-pregnant women
d. Women on therapeutic heparin need 12 hours post dose before they can undergo
regional anaesthesia
e. 60 mg SC daily is an appropriate thromboprophylactic dose of enoxaparin in a 92 kg
woman

Downloaded from https://www.cambridge.org/core. University of Edinburgh, on 19 Aug 2019 at 13:21:27, subject to the Cambridge Core terms of
use, available at https://www.cambridge.org/core/terms. https://doi.org/10.1017/9781108566100.011
Chapter 5a: Clinical Anaesthesia Questions 399

Question 286
A 32-year-old female was admitted to ICU following a paracetamol overdose two days
earlier. She was found to be jaundiced with a GCS of 9 and an INR of 3. The following
options are appropriate regarding further management:
a. N-acetylcysteine should be started regardless of the paracetamol level in blood
b. Intracranial pressure monitoring is indicated
c. Jugular venous bulb oxygen saturation is needed to guide further management
d. Hypernatraemia should be avoided
e. Prophylactic antifungal therapy is indicated

Question 287
Regarding the use of propofol to provide sedation:
a. It can be administered by an intermittent bolus technique
b. It can be administered as a target-controlled infusion
c. When co-administered with opiates the effect is additive
d. Co-administration of benzodiazepines is contraindicated
e. It has a rapid recovery profile

Question 288
With regards to skin flaps, the following statements are true:
a. A pedicle flap involves removing the whole neurovascular pedicle from the donor site
and transplanting to a new location by microvascular anastomosis
b. Blood flow in a successful free flap can be reduced to half the original flow for more than
a week postoperatively
c. Free flaps are denervated
d. Secondary ischaemia is more harmful to a flap than primary ischaemia
e. Pain can be a cause of flap failure

Question 289
With regards to performing a sub-Tenon’s block, the following statements are true:
a. Proxymetacaine 0.5% or oxybuprocaine 0.5% drops are given to provide topical anaes-
thesia to the eye before the block is performed
b. 10% povidone iodine rather than chlorhexidine is used to clean the eye
c. The fused conjunctiva and anterior Tenon’s capsule is picked up at an inferonasal point
5–10 mm from the limbus using non-toothed forceps
d. After making a small cut, the sub-Tenon’s space is accessed using the Westcott scissors
to create a thin channel in the sub-Tenon’s space between the insertion of the medial and
inferior rectus muscles
e. After injection of the local anaesthetic the eye should be massaged to encourage spread
of the local anaesthetic around the sub-Tenon’s space

Downloaded from https://www.cambridge.org/core. University of Edinburgh, on 19 Aug 2019 at 13:21:27, subject to the Cambridge Core terms of
use, available at https://www.cambridge.org/core/terms. https://doi.org/10.1017/9781108566100.011
400 Chapter 5a: Clinical Anaesthesia Questions

Question 290
A 27-year-old female asthmatic presents for diagnostic laparoscopy as a day case. She had
eczema as a child and her asthma has worsened since starting work as a research assistant in
a laboratory. She is on salbutamol inhaler when required, salmeterol inhaler twice daily and
beclomethasone 800 micrograms daily.
a. This patient has intrinsic asthma
b. She is on step 4 of the British Thoracic Society (BTS) management recommendations
c. She has approximately 1 in 50 risk of bronchospasm under anaesthesia
d. She should be induced with ketamine
e. Intraoperative bronchospasm should be managed with 4 g magnesium sulfate over
20 minutes

Question 291
The following are recognized advantages of enteral feeding over parenteral nutrition:
a. Less expensive
b. Reduces the risk of bacterial translocation
c. Less likely to cause diarrhoea
d. Avoids the risk of hyperglycaemia
e. Less likely to cause refeeding syndrome

Question 292
A 16-year-old smoker presents for excision of a large lipoma from the forearm. He declines
regional anaesthesia and you agree to general anaesthesia. Shortly after LMA insertion, his
breathing becomes noisy with a ‘see saw’ diaphragmatic pattern. You suspect laryngospasm.
Risk factors for laryngospasm include:
a. Deep plane of anaesthesia
b. Childhood obesity
c. Smoking
d. Upper respiratory tract infection
e. Secretions in the oropharynx

Question 293
Potential advantages of endovascular aneurysm repair over open surgical repair of abdom-
inal aortic aneurysms are:
a. Reduced blood loss
b. Reduced length of hospital stay
c. Reduced 30-day mortality
d. Reduced rates of complications
e. Reduced rates of reintervention

Downloaded from https://www.cambridge.org/core. University of Edinburgh, on 19 Aug 2019 at 13:21:27, subject to the Cambridge Core terms of
use, available at https://www.cambridge.org/core/terms. https://doi.org/10.1017/9781108566100.011
Chapter 5a: Clinical Anaesthesia Questions 401

Question 294
Regarding the preoperative assessment of a patient with ischaemic heart disease:
a. One metabolic equivalent (MET) represents oxygen consumption of 4 ml.kg–1.min–1
b. A Duke activity status of greater than 4 METs predicts increased morbidity
perioperatively
c. ‘Marked limitation of physical activity’ equates to grade II on the Canadian
Cardiovascular Society angina score
d. Elective surgery should be delayed for six months following myocardial infarction
e. An anaerobic threshold of less than 11 ml.kg–1.min–1 is suggestive of poorer peri-
operative outcomes

Question 295
Which of the following statements is true regarding the use of high-frequency oscillatory
ventilation (HFOV) in acute respiratory distress syndrome (ARDS)?
a. It is associated with improved outcome if initiated early in cases of severe ARDS
b. Hypercapnia is common, but should not be accepted beyond a paCO2 of 8.0 kPa
c. Hypotension is a recognized complication
d. Spontaneous breathing should be avoided during HFOV
e. Once FiO2 is less than 0.40, mechanical ventilation can be attempted

Question 296
The following statements are true regarding post-dural-puncture headaches:
a. More than 75% will be cured by a single epidural blood patch
b. They usually occur within the first 24 hours of dural puncture
c. Gutsche’s test may aid in the diagnosis
d. Synacthen has been trialed as a treatment
e. First-line treatment should include bed rest, IV fluids and caffeine

Question 297
Which of the following biochemical abnormalities are associated with Addison’s disease?
a. Hyperkalaemia
b. Hyponatraemia
c. Hypercalcaemia
d. Hypoglycaemia
e. Hypochloraemia

Question 298
Many elderly patients presenting for anaesthesia suffer from osteoporosis. Which of the
following statements regarding osteoporosis are true?
a. Osteoporosis is a disease of decreased bone density
b. Severe osteoporosis denotes low bone density in the presence of one or more fragility
fractures

Downloaded from https://www.cambridge.org/core. University of Edinburgh, on 19 Aug 2019 at 13:21:27, subject to the Cambridge Core terms of
use, available at https://www.cambridge.org/core/terms. https://doi.org/10.1017/9781108566100.011
402 Chapter 5a: Clinical Anaesthesia Questions

c. Low testosterone level is a risk factor for men


d. Bone density is reliably calculated by X-ray of the femur
e. The effect of biphosphonates is less marked in individuals with low bone density

Question 299
A 67-year-old female with a history of ischaemic heart disease is undergoing a umbilical
hernia repair. During the operation she becomes increasingly hypotensive to a blood
pressure of 65/40 mmHg and you notice a change in her cardiac rhythm and suspect
complete heart block. Which of the following are true?
a. Right coronary artery infarction is a likely cause
b. Adrenaline infusion would be beneficial
c. Boluses of atropine 500 μg up to 3 mg should be given
d. Glucagon is definitely indicated
e. Transcutaneous pacing should be started in theatre

Question 300
A 55-year-old woman is admitted to the ITU with suspected sepsis. A central venous
catheter is sited for administration of noradrenaline. Her nurse asks you to review the
patient because she has become increasingly hypotensive. Which of the following are
complications associated with central venous catheters that could explain this?
a. Thrombus
b. Catheter fracture
c. Pneumothorax
d. Tamponade
e. Chylothorax

Question 301
The following would be considered as a contraindication to conscious sedation in a child:
a. Previous apnoeic spells
b. Any history of epilepsy
c. Abdominal distension
d. Renal failure requiring dialysis
e. Large tongue

Question 302
Which of the following statements regarding sedation are correct?
a. Sedation is a state from which the patient can be roused
b. Conscious sedation refers to the maintenance of verbal contact throughout
c. Anaesthesia refers to any state where consciousness is lost
d. Sedation and anaesthesia form a spectrum with considerable overlap
e. During sedation airway reflexes should be maintained

Downloaded from https://www.cambridge.org/core. University of Edinburgh, on 19 Aug 2019 at 13:21:27, subject to the Cambridge Core terms of
use, available at https://www.cambridge.org/core/terms. https://doi.org/10.1017/9781108566100.011
Chapter 5a: Clinical Anaesthesia Questions 403

Question 303
The following factors are responsible for developing retinopathy of the premature:
a. Neonates younger than 30 weeks of gestation
b. Neonatal apnoea
c. Low carbon dioxide
d. High plasma pH
e. Bradycardia

Question 304
With regards to ophthalmic anaesthesia for cataract surgery the following statements are
true:
a. Retrobulbar block produces better pain control than peribulbar block
b. Peribulbar block produces better pain control than sub-Tenon’s block
c. The technique most likely to cause severe systemic complications is a retrobulbar block
d. Sub-Tenon’s block provides better pain control than topical anaesthesia alone
e. Sub-Tenon’s block produces better pain control than retrobulbar block

Question 305
A five-year-old boy is brought into your department from a house fire. He has jumped from
his bedroom window on the first floor and was brought straight in by paramedics without
any medical interventions. His BP is 85/47 mmHg, HR 130 bpm, SpO2 95% and he has full
thickness burns to his anterior chest and right arm, estimated at 27%. Which of the
following are true?
a. Ketamine is a reasonable induction agent for intubation
b. Immediate escharotomies are required
c. Cling film is an appropriate first-line dressing
d. Pulse oximetry is reliable in this situation
e. Dressing the burns is the immediate management priority

Question 306
Which of the following statements regarding exomphalos are correct?
a. Infants with exomphalos only have intestine present in the defect
b. There is no membranous covering of the defect
c. It is more common than gastroschisis
d. Infants with exomphalos should be delivered early
e. NG tubes should not be inserted once delivered

Question 307
Regarding anaphylaxis:
a. Asthmatics and those on β-blockers may have more severe reactions
b. Smokers appear to be at higher risk of reactions to antibiotics

Downloaded from https://www.cambridge.org/core. University of Edinburgh, on 19 Aug 2019 at 13:21:27, subject to the Cambridge Core terms of
use, available at https://www.cambridge.org/core/terms. https://doi.org/10.1017/9781108566100.011
404 Chapter 5a: Clinical Anaesthesia Questions

c. There are more anaphylactic reactions to neuromuscular blocking agents in Sweden


than Norway
d. A negative skin test to a neuromuscular blocker proves an agent is safe in future
e. There has been an increase in the incidence of anaphylaxis due to halogenated vapours in
the last 20 years

Question 308
You have reviewed a patient preoperatively for emergency surgery who has a pacemaker
in situ. Appropriate management includes:
a. Not proceeding with surgery until the pacemaker is deactivated
b. Ensuring an alternative method of pacing is available
c. Instructing the surgeons they must avoid monopolar diathermy
d. Instructing the surgeons they must avoid bipolar diathermy
e. Placing a magnet over the pacemaker

Question 309
The following is correct regarding the investigation of ICU-acquired weakness:
a. Creatine kinase level can help differentiating between critical illness polyneuropathy
(CIP) and critical illness myopathy (CIM)
b. Muscle biopsy can be abnormal in both CIP and CIM
c. Nerve biopsy may be normal in CIP
d. CSF may show elevated protein level in CIP
e. Nerve conduction velocity is usually normal in CIP

Question 310
In the pre-eclamptic patient:
a. Blood pressure control and seizure prevention are the primary aims of treatment
b. If IV therapy is required then hydralazine is the first choice antihypertensive in patients
with cardiac disease
c. Magnesium is used to prevent seizures with a loading dose of 1 mg and then 1 mg.h–1
infusion
d. If the magnesium plasma concentration rises above 7 mmol.l–1 then respiratory arrest is
likely
e. Magnesium toxicity is treated with intravenous calcium gluconate

Question 311
Which of the following are generally unacceptable products to receive for Jehovah’s
Witnesses?
a. Vaccinations
b. Autologous predonation
c. Platelets
d. Epidural blood patch
e. Whole blood

Downloaded from https://www.cambridge.org/core. University of Edinburgh, on 19 Aug 2019 at 13:21:27, subject to the Cambridge Core terms of
use, available at https://www.cambridge.org/core/terms. https://doi.org/10.1017/9781108566100.011
Chapter 5a: Clinical Anaesthesia Questions 405

Question 312
The prevalence of dementia is increasing, partly because of the ageing population. Which of
the following statements are true regarding dementia?
a. Dementia is a condition in which there is gradual loss of brain function, decline in
cognitive function and short-term memory loss
b. Long-term memory is not affected
c. Change in personality is a feature of dementia and is a sign of posterior fossa disease
d. Dementia presents with a sudden onset of difficulty with understanding and speech
e. Postoperative cognitive dysfunction is more common in patients with dementia

Question 313
You are on the labour ward when a woman self presents at 37 weeks, bleeding heavily with
known placenta praevia. Regarding massive haemorrhage and transfusion:
a. Massive transfusion is defined as the replacement of the patient’s total blood volume in
less than 24 hours
b. Full blood count and clotting should not be taken until resuscitation with blood
products has begun
c. A platelet count of >80 × 109 l–1 should be targeted
d. This patient should be taken immediately to theatre for caesarean section
e. Cell salvage should not be used because of risk of amniotic fluid embolism

Question 314
During your obstetric on-call you are asked to site an epidural in a labouring patient.
A student midwife asks about the anatomy of the epidural space. What structures might you
possibly encounter?
a. Spinal nerves
b. Venous plexus
c. Ligamentum flavum
d. Dural sac
e. Fat

Question 315
The following are correct regarding sedation in children:
a. IV access is necessary if a child is to be sedated for painless imaging with oral chloral
hydrate
b. A child should be adequately fasted before sedation with nitrous oxide
c. Ketamine 2 mg.kg–1 IV may be suitable sedation for a painful procedure in the ER
d. Chloral hydrate up to a dose of 100 mg.kg–1 would be suitable sedation for a 50 kg child
undergoing painless imaging
e. Midazolam IV at a starting dose of 25–50 μg.kg–1 is suitable sedation for an adolescent
undergoing a dental procedure

Downloaded from https://www.cambridge.org/core. University of Edinburgh, on 19 Aug 2019 at 13:21:27, subject to the Cambridge Core terms of
use, available at https://www.cambridge.org/core/terms. https://doi.org/10.1017/9781108566100.011
406 Chapter 5a: Clinical Anaesthesia Questions

Question 316
The following is true regarding glucose metabolism in the neonate:
a. The fetus cannot make glucose from glycogen
b. The liver storage of glycogen is sufficient for 10–12 hours of fasting
c. In the first few days of life, the brain can use ketone bodies as a source of energy without
harm
d. Blood sugar level less than 3.5 mmol.l–1 should be treated
e. Neonatal hypoglycaemia can result in long-term developmental defects

Question 317
Regarding transthoracic electrical bioimpedance:
a. It is a non-invasive method of measuring cardiac output
b. It measures electrical resistance of the thorax to a high-frequency, low-magnitude
current
c. The bioimpedance is directly proportional to the volume of thoracic fluid
d. It is ideal for intraoperative cardiac output measurement
e. It is accurate when used on awake patients

Question 318
The long-term use of strong opioids in the management of chronic non-cancer pain is
associated with the development of the following:
a. Diabetes
b. Erectile dysfunction
c. Hypertension
d. Reduced libido
e. Immune dysfunction

Question 319
Regarding acute spinal cord injury:
a. It is associated with head injury in up to two-thirds of cases
b. Autonomic hyper-reflexia typically occurs at an interval of 8–12 weeks post injury
c. The timeframe for paralytic ileus is up to 6–8 weeks post injury
d. The timeframe for flaccid paralysis is up to 2–3 weeks post injury
e. With central cord injury, lower limbs are affected to a greater extent than upper limbs

Question 320
You are anaesthetizing a 66-year-old female undergoing a partial maxillectomy and recon-
struction with radial forearm flap. Regarding flap surgery:
a. The ambient temperature in theatre should be maintained at 22–24 °C
b. Central venous pressure should be monitored
c. Tramadol can be used to treat postoperative shivering

Downloaded from https://www.cambridge.org/core. University of Edinburgh, on 19 Aug 2019 at 13:21:27, subject to the Cambridge Core terms of
use, available at https://www.cambridge.org/core/terms. https://doi.org/10.1017/9781108566100.011
Chapter 5a: Clinical Anaesthesia Questions 407

d. Norepinephrine should be used to maintain MAP >70 mmHg


e. The ideal difference between central and core temperature is <3 °C

Question 321
A 21-year-old male with Down’s syndrome presents for surgical extraction of wisdom teeth.
Concerning Down’s syndrome and anaesthesia:
a. Desflurane anaesthesia is contraindicated due to likelihood of cardiac disease
b. Nitrous oxide should be avoided in all Down’s syndrome patients
c. Adults with Down’s syndrome cannot provide valid consent for surgical procedures
d. The rate of malignant hyperthermia is the same as the normal population
e. Regional techniques are contraindicated

Question 322
Enoximone is a type 3 phosphodiesterase inhibitor. When administered to a patient with
left ventricular failure the following would be expected:
a. Increased left ventricular work index and myocardial oxygen consumption
b. Increased cardiac index
c. Increased pulmonary vascular resistance
d. Increased heart rate
e. Increased sensitivity to intracytosolic calcium

Question 323
The following factors need to be addressed prior to anaesthetizing a patient with atrial
fibrillation for elective surgery:
a. Potassium 2.0 mmol.l–1
b. The presence of a right bundle branch block on ECG
c. A ventricular rate of 40 without medication
d. Failure to administer the patient’s regular digoxin
e. The administration of aspirin 75 mg preoperatively

Question 324
The following are true regarding the use of renal replacement therapy (RRT) in critically ill
patients with acute kidney injury (AKI):
a. The higher the intensity of RRT, the lower the mortality
b. Anticoagulation is the most frequently encountered problem during RRT
c. There is a good evidence to support early initiation of RRT in patients with AKI
d. Phenytoin clearance is likely to be affected by the use of RRT
e. The predilution method for infusing replacement fluid may reduce the risk of clotting of
the haemofiltration machine

Downloaded from https://www.cambridge.org/core. University of Edinburgh, on 19 Aug 2019 at 13:21:27, subject to the Cambridge Core terms of
use, available at https://www.cambridge.org/core/terms. https://doi.org/10.1017/9781108566100.011
408 Chapter 5a: Clinical Anaesthesia Questions

Question 325
You anaesthetize a 55-year-old patient for an emergency appendicectomy. In the recovery
room the patient complains that one of his teeth is chipped. With regard to dental damage
under anaesthesia:
a. It is more likely during emergency intubations
b. It is rare on emergence from anaesthesia
c. The use of Guedel oropharyngeal airways as bite guards is advocated to reduce the risk of
dental damage on biting endotracheal tubes
d. The maxillary incisors are most commonly injured
e. The risk of dental damage is approximately 1:1000

Question 326
A 24-year-old woman is one hour post normal vaginal delivery. The obstetric team are
concerned about postpartum bleeding. She is alert, has some mild abdominal pain and
a small amount of ongoing vaginal bleeding. Her BP is 106/44 mmHg and HR 120 bpm.
In this situation:
a. The most likely cause of bleeding is retained placental tissue
b. Anaesthesia, if required, should not be commenced without invasive monitoring in
place
c. She may have lost up to 40% of her circulating volume
d. A spinal would be the most appropriate choice for anaesthesia
e. Uterotonics and fluid resuscitation are appropriate first-line treatment

Question 327
You are asked to anaesthetize a 15-year-old child for tonsillectomy. The family are Jehovah’s
Witnesses. With regards to consent for treatment of a child under the age of 16 years with
Jehovah’s Witness parents, which of the following are true?
a. A Gillick-competent child may give consent
b. A child may make an advance directive outlining their wishes
c. Children should not be involved in the decision-making process
d. A Specific Issue Order removes all parental authority
e. In a life-saving emergency situation where parents refuse blood transfusion, this refusal
is legally binding

Question 328
Electroconvulsive therapy (ECT) can be used in the management of a number of different
psychiatric illnesses. With regard to ECT, which of the following are relative
contraindications?
a. Pregnancy
b. Epilepsy
c. The presence of a permanent pacemaker
d. Severe osteoporosis
e. Deep venous thrombosis

Downloaded from https://www.cambridge.org/core. University of Edinburgh, on 19 Aug 2019 at 13:21:27, subject to the Cambridge Core terms of
use, available at https://www.cambridge.org/core/terms. https://doi.org/10.1017/9781108566100.011
Chapter 5a: Clinical Anaesthesia Questions 409

Question 329
You have been asked to review a patient in the HDU who is complaining of pain following
thoracotomy for an oesophagectomy. The following is true regarding analgesia for post-
thoracotomy pain:
a. The dorsal rami are not blocked by an intercostal nerve block and hence this technique
may be ineffective for posterolateral thoracotomies
b. Interpleural blocks are very effective
c. The intercostal nerves consistently lie in a plane deep to the internal intercostal muscle
d. Cryoanalgesia is shown to reduce the incidence of chronic post-thoracotomy pain
e. The thoracic paravertebral space has well-defined superior and inferior borders

Question 330
Regarding sedation, the following are true:
a. Immediate availability of resuscitation equipment is unnecessary
b. Single drugs are easier to titrate than concurrent administration of multiple medications
c. Preprocedural fasting is unnecessary
d. An explanation to the patient of what to expect is vital
e. Monitoring should be in accordance with the AAGBI minimal mandatory monitoring
requirements

Question 331
The following statements are true regarding the performance of percutaneous
tracheostomy:
a. The stoma is ideally placed between the second and third tracheal rings
b. Multiple graduated dilators are the most common technique currently used in the UK
c. Tracheal ring fracture is a common complication
d. Skin incision can be transverse or longitudinal
e. It can be safely performed using a laryngeal mask airway rather than endotracheal tube if
there is no risk of aspiration

Question 332
Osteoarthritis is the most common joint disease of the elderly and a major cause of
disability. Which of the following statements regarding osteoarthritis (OA) are true?
a. It is an inflammatory disease affecting the articular surface of one or more joints usually
due to ageing or repetitive joint trauma
b. The spine may be involved, causing nerve root compression
c. Bony enlargements, referred to as Heberden’s nodes, are seen at the distal interphalan-
geal joints of the fingers
d. Degenerative changes are most significant in the upper cervical area
e. Systemic steroids are only used when joint replacement therapy is contraindicated

Downloaded from https://www.cambridge.org/core. University of Edinburgh, on 19 Aug 2019 at 13:21:27, subject to the Cambridge Core terms of
use, available at https://www.cambridge.org/core/terms. https://doi.org/10.1017/9781108566100.011
410 Chapter 5a: Clinical Anaesthesia Questions

Question 333
During posterior fossa surgery in the sitting position, there are sudden acute changes to the
capnograph, with the end tidal CO2 halving and a marked reduction in blood pressure.
Venous air embolism is suspected. With regards to venous air embolism:
a. The air bubble causes left ventricular outflow tract obstruction
b. Paradoxical embolism occurs only in patients with patent foramen ovale
c. The patient may take longer than anticipated to recover from general anaesthesia
d. A Valsalva manoeuvre is contraindicated
e. The patient should immediately be placed in the left lateral decubitus position

Question 334
Whilst on call for ITU, the medical team contact you regarding a patient on the ward,
requiring placement of a central venous catheter (CVC). Indications for placement of
a CVC include:
a. Fluid resuscitation
b. Vasoactive drugs
c. Sedation
d. Fluid management
e. Mixed venous saturation measurement

Question 335
You are called urgently to see a patient on the ward who underwent an anterior resection
earlier that day. The nurses are concerned, as he has become agitated and confused. As you
arrive he has a seizure. He has an epidural running and you suspect local anaesthetic
toxicity. Your initial management plan would include:
a. Supporting the airway, administering oxygen and considering tracheal intubation and
ventilation
b. Administering lorazepam 4 mg IV
c. Removing the epidural catheter
d. Giving 1.5 ml.kg–1 of 10% lipid emulsion
e. Treating arrhythmias with lidocaine

Question 336
An 80-year-old man is undergoing a hip hemiarthroplasty after sustaining a fractured neck
of femur 24 hours earlier. His preoperative medications were: clopidogrel, aspirin, atenolol
and loop diuretics. In the anaesthetic room he is in atrial fibrillation, rate 120 bpm, blood
pressure 170/95 mmHg and oxygen saturations 85%. The following modes of anaesthesia
are appropriate:
a. Spinal anaesthesia with IV sedation
b. General anaesthesia with an endotracheal intubation using sevoflurane combined with
a femoral nerve block after the use of esmolol preoperatively
c. Femoral and sciatic nerve block with IV sedation
d. Combined spinal and epidural block

Downloaded from https://www.cambridge.org/core. University of Edinburgh, on 19 Aug 2019 at 13:21:27, subject to the Cambridge Core terms of
use, available at https://www.cambridge.org/core/terms. https://doi.org/10.1017/9781108566100.011
Chapter 5a: Clinical Anaesthesia Questions 411

e. Postpone surgery until his heart rate is controlled and his clopidogrel has stopped for
seven days.

Question 337
A 68-year-old female attends your pain management clinic with a 12-month history of
significant chest wall pain following mastectomy and reconstruction. There is no obvious
cause for her pain and chronic postsurgical pain (CPSP) has been given as the diagnosis.
The following are true:
a. CPSP is rare after mastectomy
b. Preoperative chemotherapy may have a protective effect
c. Older patients (>55 years) are more likely to develop CPSP
d. Severe pain immediately following mastectomy is a risk factor for CPSP
e. Perioperative gabapentin is associated with a reduction in CPSP

Question 338
The following people should be scheduled for elective surgery for repair of an abdominal
aortic aneurysm:
a. Patients with aneurysms larger than 9.0 cm in diameter
b. Patients with aneurysms larger than 4.5 cm in diameter that have increased by more
than 1 cm in the past year
c. Symptomatic aneurysms of 4.5–5.5 cm
d. Symptomatic aneurysms of less than 4.5 cm in diameter that have increased by more
than 1 cm in the past year
e. All of the above

Question 339
You have been asked to anaesthetize a 5 kg six-week-old baby for a pyloromyotomy
following a diagnostic ultrasound that revealed pyloric stenosis. Which of the following is
true regarding this condition?
a. As soon as the diagnosis is confirmed this becomes an urgent surgical case and should be
operated on within the hour
b. The common biochemical derangement is a hyperchloraemic metabolic alkalosis
c. Insertion of an NG tube will be difficult due to obstruction and should not be attempted
d. Pyloric stenosis is the commonest surgical condition presenting within the first six
months of life
e. Urinary chloride levels are useful in assessing degree of dehydration and filling

Question 340
In a patient with sepsis:
a. Indicators of sepsis may include mottled hands, raised bilirubin or raised procalcitonin
b. Venoconstriction by noradrenaline will compensate for hypovolaemia when resusci-
tating septic patients

Downloaded from https://www.cambridge.org/core. University of Edinburgh, on 19 Aug 2019 at 13:21:27, subject to the Cambridge Core terms of
use, available at https://www.cambridge.org/core/terms. https://doi.org/10.1017/9781108566100.011
412 Chapter 5a: Clinical Anaesthesia Questions

c. Low-dose dopamine infusion can be used to increase splanchnic flow and reduce acute
kidney injury
d. Dobutamine may be trialled in low cardiac output states
e. Target blood pressure should be a systolic at 90 mmHg

Question 341
With regards to third-degree heart block preoperatively:
a. Insertion of a temporary transvenous pacing wire may be appropriate
b. May indicate myocardial ischaemia
c. An isoprenaline infusion may improve heart rate
d. Transcutaneous pacing may be necessary
e. It may be transient

Question 342
Phase 1 metabolism is principally undertaken in the liver and involves cytochrome P450
enzymes. The following are cyctochrome P450 inducers:
a. Phenytoin
b. Rifampicin
c. Omeprazole
d. Grapefruit juice
e. Fluconazole

Question 343
Which of the following statements are true regarding the management of hyperglycaemia in
critically ill patients?
a. Tight glycaemic control is beneficial in severe sepsis when compared with conventional
control
b. Glucose meter readings from a capillary finger-stick are comparable to blood gas
analyzers
c. Acute drops in blood glucose are less tolerated in patients with well-established diabetes
mellitus
d. The latest evidence suggests targeting a blood glucose level below 8 mmol.l−1
e. Wide fluctuations in blood glucose are more hazardous than sustained hyperglycaemia

Question 344
In a pregnant woman with a mechanical heart valve:
a. Low molecular weight heparin is as effective as warfarin at preventing valve thrombosis
b. Unfractionated heparin crosses the placenta
c. There is a higher miscarriage and stillbirth rate with warfarin use when compared to
heparin
d. Both unfractionated and low molecular weight heparins can be used throughout preg-
nancy for thrombus prevention
e. Epidurals are a suitable form of labour analgesia

Downloaded from https://www.cambridge.org/core. University of Edinburgh, on 19 Aug 2019 at 13:21:27, subject to the Cambridge Core terms of
use, available at https://www.cambridge.org/core/terms. https://doi.org/10.1017/9781108566100.011
Chapter 5a: Clinical Anaesthesia Questions 413

Question 345
Clinical features associated with Parkinson’s disease include:
a. Myoclonus
b. Hypokinesia
c. Jerking
d. Postural instability
e. Intention tremor

Question 346
Concerning anaesthetic drugs and the elderly:
a. Reduced doses of neuromuscular drugs are required, owing to reduced muscle mass
b. MAC values of inhalational agents are reduced by 20–30%
c. β-receptor sensitivity is reduced, resulting in a reduction in response to exogenous
β-agonists
d. Intravenous and inhalational anaesthetic agents can suppress the cardiac and smooth
muscle contractility
e. Duration of action of opioids and benzodiazepines exhibits an age-related increase in the
elimination half-life

Question 347
With regards to upper respiratory tract infections:
a. There is increased risk of laryngospasm and bronchospasm during general anaesthesia
b. Haemophilus influenzae is the commonest cause of retropharyngeal abscess
c. Respiratory syncytial virus can cause laryngitis, tracheitis and bronchitis
d. In epiglottitis bacteraemia is unlikely
e. Croup requires ventilatory support in 20% of cases

Question 348
The ODP is helping you set up for a spinal anaesthetic and realizes there are no more spinal
needles in the room. He offers you the choice of needle. Which of the following are suitable
for spinal anaesthesia?
a. Quinke
b. Hustead
c. Sprotte
d. Whitacre
e. Weiss

Question 349
Regarding supraglottic airway devices:
a. The classic LMA can achieve a median pharyngeal seal of approximately 20 cmH20
b. The i-Gel has a relatively low oesophageal seal pressure
c. The ProSeal LMA has an anterior inflatable cuff

Downloaded from https://www.cambridge.org/core. University of Edinburgh, on 19 Aug 2019 at 13:21:27, subject to the Cambridge Core terms of
use, available at https://www.cambridge.org/core/terms. https://doi.org/10.1017/9781108566100.011
414 Chapter 5a: Clinical Anaesthesia Questions

d. When seated correctly, the tip of the ProSeal LMA sits over the oesophageal inlet
e. Paediatric sizes of the ProSeal LMA are available

Question 350
The following are true regarding hypocalcaemia of the neonate:
a. Infants of insulin-dependant diabetic mothers are at risk
b. Cow’s milk ingestion is not a risk
c. Neonatal seizures may be the first manifestation in neonates
d. Electrocardiographic evaluation is often not characteristic
e. Treatment intraoperatively should be prompted by hypotension

Question 351
With regards to ophthalmic needle blocks:
a. If the axial globe length is >26 mm, retrobulbar block is preferred to minimize the risk of
globe penetration
b. If the axial globe length is >26 mm there is a high risk of globe penetration if a sub-
Tenon block is performed
c. The axial globe length will usually be <24 mm in people with myopic eyes
d. A 25G 35 mm needle is used for peribulbar blocks
e. A medial canthal approach is safer than an inferotemporal approach in patients with
axial myopia

Question 352
Regarding necrotizing enterocolitis (NEC):
a. It is commonly seen without other congenital life-threatening complications
b. It classically presents within 48 hours of birth
c. Perinatal asphyxia, maternal cocaine use and umbilical artery cannulation are risk
factors for developing NEC
d. NEC can be caused by viruses as well as bacterial pathogens
e. NEC only occurs in preterm infants

Question 353
Concerning noradrenaline:
a. Coronary blood flow is increased
b. Fetal oxygenation may be reduced
c. Hepatic blood flow is increased
d. Monoamine oxidase inhibitors do not affect metabolism
e. It is recommended as the first-line vasopressor in sepsis

Downloaded from https://www.cambridge.org/core. University of Edinburgh, on 19 Aug 2019 at 13:21:27, subject to the Cambridge Core terms of
use, available at https://www.cambridge.org/core/terms. https://doi.org/10.1017/9781108566100.011
Chapter 5a: Clinical Anaesthesia Questions 415

Question 354
Important features of assessment of a patient with a pacemaker include:
a. Mode of function of pacemaker
b. Indication for insertion
c. 12-lead ECG
d. Electrolytes
e. Date of last battery and function check

Question 355
Intercostal chest drains (ICDs) enable drainage of air or fluid from the pleural space and are
often seen in patients on the ICU. Regarding the insertion and management of ICDs, the
following are acceptable practice:
a. The use of ultrasound to detect the presence of an anterior pneumothorax in the
emergency department
b. Placing an ICD in the second intercostal space at the mid-clavicular line
c. Clamping the drain after the rapid drainage of 1 l of pleural effusion
d. Clamping the drain to help detect small air leaks
e. Clamping the drain at the time of removal

Question 356
The following are true in pregnancy-related sepsis:
a. Group A Streptococcus has become the most significant pathogen
b. Due to the normal physiological changes of pregnancy an acidosis developing due to
sepsis can be well compensated for
c. The signs of systemic inflammatory response present early
d. An appropriate empirical treatment to commence in all septic patients would be
piperacillin–tazobactam 4.5 g eight-hourly and gentamicin 3–5 mg.kg–1 daily until any
organism is identified
e. High-dose corticosteroids may be required

Question 357
Pharmacological treatment of Parkinson’s disease is dependent on the stage and severity of
the disease. Regarding classes and mechanisms of action, which of the following are true?
a. Selegiline is a catechol-O-methyltransferase inhibitor
b. Selegiline is a monoamine oxidase-A inhibitor
c. Entacapone is a catechol-O-methyltransferase inhibitor
d. Apomorphine antagonizes the excitatory effects of cholinergic pathways
e. Bromocriptine has a mechanism of action that is not fully understood

Question 358
A surgical colleague confides in you that his elderly mother has ‘never been the same’
following surgery for a fractured neck of femur some months earlier. The following state-
ments regarding postoperative cognitive dysfunction (POCD) are true:

Downloaded from https://www.cambridge.org/core. University of Edinburgh, on 19 Aug 2019 at 13:21:27, subject to the Cambridge Core terms of
use, available at https://www.cambridge.org/core/terms. https://doi.org/10.1017/9781108566100.011
416 Chapter 5a: Clinical Anaesthesia Questions

a. The risk of POCD is higher with increasing age


b. It is associated with changes in the level of consciousness
c. Neuropsychological testing may be needed for recognition
d. Surgical stress response can contribute to development of POCD
e. Sevoflurane is associated with less POCD then isoflurane

Question 359
A 68-year-old man presents for elective total hip replacement. His past medical history
includes COPD, for which he takes regular inhalers. With regards to this case:
a. This man will have a smoking history
b. The operation should be postponed pending formal spirometry
c. The severity of his condition will be reflected in his inhaler doses and frequency
d. He should not be given oxygen postoperatively in case it obtunds his hypoxic drive
e. NSAIDs are absolutely contraindicated in this case

Question 360
You are anaesthetizing for the trauma list. The next patient has a neck of femur fracture and
you feel they would benefit from a regional anaesthetic. What would be absolute contra-
indications for regional anaesthesia?
a. Coagulopathy
b. Sepsis
c. Chronic back pain
d. Dementia
e. Refusal

Question 361
The following are physiological effects commonly associated with epidural blockade:
a. Increase in GIT peristalsis
b. Increase in vagal tone
c. Venodilatation and arterial vasodilatation
d. Reduced renal function
e. Loss of proprioception prior to motor blockade

Question 362
A 71-year-old male with a background of ischaemic heart disease and diabetes undergoes
a right below-knee amputation on your emergency list due to worsening sepsis. With regard
to limb amputation:
a. 30-day mortality following emergency amputation is as high as 10%
b. Unlike phantom pain, persistent stump pain is unusual
c. Phantom pain is more likely in those with a background of depression and anxiety
d. Prolonged perineural blockade in the perioperative period is associated with a reduced
incidence of phantom pain
e. There is no difference in the incidence of phantom pain between the sexes

Downloaded from https://www.cambridge.org/core. University of Edinburgh, on 19 Aug 2019 at 13:21:27, subject to the Cambridge Core terms of
use, available at https://www.cambridge.org/core/terms. https://doi.org/10.1017/9781108566100.011
Chapter 5a: Clinical Anaesthesia Questions 417

Question 363
The following patient factors are known to affect the success of a free tissue flap:
a. Advanced age
b. Hypothyroidism
c. Diabetes mellitits
d. Obesity
e. Smoking

Question 364
Concerning Down’s syndrome:
a. Trisomy 21 is always the cause
b. It is commonly associated with cleft palate
c. Patients may be difficult to intubate because of an enlarged tongue
d. There is a higher incidence of epilepsy in Down’s syndrome patients
e. There is a higher incidence of thyroid disease in Down’s syndrome patients

Question 365
A 57-year-old male is admitted to the ICU with probable pneumonia. He has a background
of hypertrophic obstructive cardiomyopathy (HOCM). In a patient with HOCM:
a. Enoximone may paradoxically decrease cardiac output
b. Adrenaline may paradoxically decrease cardiac output
c. In a shocked state, such a patient benefits from restrictive fluid administration
d. Maintaining organ perfusion may be best achieved with noradrenaline
e. Coronary perfusion can be enhanced with supranormal positive chronotropy

Question 366
The following are indications for insertion of a permanent pacemaker prior to elective
surgery:
a. Asymptomatic Mobitz type I atrioventricular block
b. Mobitz type II atrioventricular block with trifascicular block
c. Atrial fibrillation with slow ventricular rate
d. First-degree atrioventricular block
e. Asymptomatic third-degree atrioventricular block

Question 367
The following are possible indications for renal replacement therapy:
a. Malignant hyperthermia
b. Lithium toxicity
c. Ethylene glycol toxicity
d. Myasthenia gravis
e. Severe sepsis

Downloaded from https://www.cambridge.org/core. University of Edinburgh, on 19 Aug 2019 at 13:21:27, subject to the Cambridge Core terms of
use, available at https://www.cambridge.org/core/terms. https://doi.org/10.1017/9781108566100.011
418 Chapter 5a: Clinical Anaesthesia Questions

Question 368
Regarding potential complications of regional anaesthesia in pregnancy:
a. NAP3 showed a reduced incidence of permanent harm in obstetric CNS blocks com-
pared with the overall figure
b. Most cases or neurological damage in postpartum women are attributable to epi-
dural use
c. The risk of a permanent nerve injury is quoted as being greater than 1 in 100 000
d. Arachnoiditis develops within 48 hours of a regional technique
e. Combined spinal and epidurals have a disproportionately high incidence of harm to
patients

Question 369
Conditions associated with myasthenia gravis include:
a. Diabetes
b. Pericarditis
c. Polymyositis
d. Hypertrophic cardiomyopathy
e. Factor V Leiden deficiency

Question 370
You are on-call for the ICU. Your ST3 is struggling to gain consistent cardiac output
measurement from an oesophageal Doppler. Regarding oesophageal Doppler cardiac out-
put monitoring:
a. It requires patient demographic data to calculate the cross-sectional area of the
ascending thoracic aorta
b. It can accurately be used in children
c. Oesophageal Doppler probes only measure 70% of cardiac output
d. The probe is approximately 90 cm long with depth markers at 35 cm, 40 cm and 45 cm
e. Correct probe position within the oesophagus is at approximately T5–T6

Question 371
A 20-year-old pedestrian is hit by a car and suffers an open tibia/fibula fracture that requires
fixation. He has a history of severe asthma with two previous intensive care admissions
requiring ventilation and is currently on 40 mg prednisolone, inhaled corticosteroids and
β2-agonists. He is wheezy on your preoperative visit with PEFR <50% predicted. Your
anaesthetic plan should include:
a. Fixation under regional technique
b. Preoperative nebulized salbutamol
c. IV hydrocortisone 50 mg at induction
d. Avoidance of atracurium, diclofenac, morphine and fentanyl
e. Low-dose adrenaline infusion if his airway pressures are high under general anaesthesia

Downloaded from https://www.cambridge.org/core. University of Edinburgh, on 19 Aug 2019 at 13:21:27, subject to the Cambridge Core terms of
use, available at https://www.cambridge.org/core/terms. https://doi.org/10.1017/9781108566100.011
Chapter 5a: Clinical Anaesthesia Questions 419

Question 372
During your follow-up of elective caesarean section patients, one of the patients complains
of a headache. Features suggestive of post-dural-puncture headache include:
a. Cranial nerve palsy
b. Visual disturbance
c. Fever
d. Invariable headache
e. Photophobia

Question 373
The following are correct regarding the appropriate length of time that must elapse prior to
performing a central neuraxial block (CNB) or removing an epidural catheter in patients
with no other coagulopathy:
a. Clopidogrel should be discontinued for at least seven days
b. Prophylactic low molecular weight heparin (LMWH) should not have been adminis-
tered for at least six hours
c. Therapeutic LMWH should not have been administered for at least 12 hours
d. Prasugrel should be discontinued for 7–10 days
e. Aspirin 75 mg should be discontinued for at least 24 hours

Question 374
The following are correct regarding recommendations for the management of postoperative
epidurals:
a. A 20 μm filter should be included within the closed circuit
b. Patients with postoperative epidurals should be situated near the nurses’ station
c. It is the responsibility of the anaesthetist to ensure the postoperative ward is adequately
staffed to manage a patient with an epidural
d. An epidural infusion should be switched off in a patient with a Bromage score of 3
e. An epidural should be switched off in a patient who develops new onset confusion and
dizziness

Question 375
The following statements regarding antineuropathic medications are true:
a. Amitriptyline blocks muscarinic receptors
b. Gabapentin blocks the GABA receptor
c. Duloxetine is a serotonin and noradrenaline reuptake inhibitor
d. Capsaicin is a substance-P inhibitor
e. Carbamazepine is first-line treatment for postherpetic neuralgia

Downloaded from https://www.cambridge.org/core. University of Edinburgh, on 19 Aug 2019 at 13:21:27, subject to the Cambridge Core terms of
use, available at https://www.cambridge.org/core/terms. https://doi.org/10.1017/9781108566100.011
420 Chapter 5a: Clinical Anaesthesia Questions

Question 376
The following methods can be used to detect cerebral ischaemia during carotid
endarterectomy:
a. Transcranial Doppler
b. Electroencephalography
c. Electromyography
d. Carotid stump pressure
e. Bispectral index

Question 377
You have been called to A&E resus to assist with the management of a three-year-old
toddler who has presented with significant burn injury. Which of the following statements
apply?
a. A 5% or more full thickness burn requires transfer to a specialist burns centre
b. Oral fluid resuscitation may be possible in burns <10% body surface area
c. Total fluid requirements are given by the Parkland formula for extensive burns
d. Early enteral feeding improves outcome
e. Suxamethonium should be avoided in RSI of burns patients during initial resuscitation

Question 378
With regard to double lumen endobronchial tubes (DLTs), which of the following are true?
a. For women a size 37–39 Fr is usually appropriate
b. Right-sided tubes are more commonly used than left sided tubes, as they reduce the risk
of obstructing the right upper lobe bronchus
c. In patients with a large thoracic aortic aneurysm a right-sided tube may be preferred
d. The depth of insertion is typically 28–30 cm in adult males
e. The Robertshaw tube has a carinal hook

Question 379
Levosimendan is used in the treatment of severe acute decompensated heart failure.
Concerning levosimendan:
a. It acts in part by increasing duration of calcium binding to troponin C
b. Increased cardiac contractility increases myocardial oxygen demand
c. Increased cardiac output compensates for reduced SVR and prevents hypotension
d. Metabolites are inactive
e. Hypokalaemia may occur

Question 380
One hour following extubation of a patient who has undergone thyroidectomy, a nurse
reports marked stridor. Actions should include:
a. Evacuation of haematoma from the wound
b. Assessment for anaphylaxis

Downloaded from https://www.cambridge.org/core. University of Edinburgh, on 19 Aug 2019 at 13:21:27, subject to the Cambridge Core terms of
use, available at https://www.cambridge.org/core/terms. https://doi.org/10.1017/9781108566100.011
Chapter 5a: Clinical Anaesthesia Questions 421

c. Rapid check of serum calcium concentration


d. Transfer of patient for urgent CT scan of neck and upper thorax
e. Anterior triangle of neck ultrasound

Question 381
The following investigations are necessary in a patient with atrial fibrillation presenting for
elective surgery:
a. 24-hour ambulatory ECG
b. Serum electrolytes
c. 12-lead ECG
d. C-reactive protein
e. Echocardiogram

Question 382
Regarding the diagnosis and management of acute rhabdomyolysis:
a. A urine dipstick can differentiate between haemoglobin and myoglobin
b. It is associated with raised serum calcium level
c. It is commonly associated with cocaine-related admissions
d. It is associated with increased serum urea-to-creatinine ratio
e. Sodium bicarbonate infusion can be useful in severe cases

Question 383
The following would support the diagnosis of severe pre-eclampsia in a pregnant woman
over 20 weeks’ gestation:
a. Systolic BP >140 mmHg or diastolic BP >90 mmHg
b. Proteinuria >1000 mg per 24 hours
c. Seizures
d. Epigastric pain
e. Pulmonary oedema

Question 384
Pyridostigmine is used in the treatment of myasthenia gravis. Regarding pyridostigmine:
a. It acts by decreasing the amount of acetylcholine at the neuromuscular junction
b. Has a peak effect eight hours after administration
c. Does not cross the blood–brain barrier
d. Has a longer duration of action than neostigmine
e. Is administered as a subcutaneous injection

Question 385
Regarding tests of pulmonary function and lung volumes:
a. All lung volumes can be measured with a spirometer
b. In flow–volume loops, the starting point of the loop is the residual volume

Downloaded from https://www.cambridge.org/core. University of Edinburgh, on 19 Aug 2019 at 13:21:27, subject to the Cambridge Core terms of
use, available at https://www.cambridge.org/core/terms. https://doi.org/10.1017/9781108566100.011
422 Chapter 5a: Clinical Anaesthesia Questions

c. Dynamic compression of the airways results in a fixed flow rate during expiration
d. Intrathoracic and extrathoracic obstruction have the same effect on flow–volume loops
e. Predicted peak expiratory flow rates are based on height and age

Question 386
A patient is admitted to the ITU for inotropic support and fluid management. A decision is
made to site a central venous catheter in the internal jugular vein (IJV). With regards to
anatomical relations to the IJV:
a. The internal carotid artery is medial
b. The thoracic duct is anterior
c. The dome of pleura is posterior
d. The omohyoid passes anterior
e. The vagus nerve is posterior

Question 387
With regards to anaesthesia for ophthalmic surgery:
a. Sedation is required in less than 2% of cataract procedures
b. A BMI ≥35 is a contraindication to day-case ophthalmic surgery
c. It is not generally necessary to starve patients prior to ophthalmic surgery under local
anaesthesia
d. Appropriately trained non-medical staff may administer peribulbar blocks
e. Intravenous access is obtained only if sedation is likely to be required

Question 388
During microvascular flap surgery:
a. Haematocrit should be maintained around 40%
b. Sodium nitroprusside can be used to increase flap blood flow
c. Hypervolaemia is recommended to ensure vasodilatation and optimum flap blood flow
d. Hypercapnia is desirable to produce vasodilatation and optimum flap blood flow
e. Remifentanil use is associated with hypotension and reduced flap blood flow

Question 389
With regards to patients undergoing surgery for abdominal aortic aneurysm:
a. Statins should be started one month before intervention and continued indefinitely
b. β-Blockers should be started one week before intervention and continued indefinitely
c. Patients with cardiac risk factors should undergo preoperative cardiopulmonary exer-
cise testing
d. In the case of a ruptured aneurysm the systolic blood pressure should be kept between 50
and 100 mmHg to maintain hypotensive haemostasis
e. Although aspirin may be continued perioperatively, dual antiplatelet therapy must be
stopped at least seven days preoperatively to reduce the risk of bleeding

Downloaded from https://www.cambridge.org/core. University of Edinburgh, on 19 Aug 2019 at 13:21:27, subject to the Cambridge Core terms of
use, available at https://www.cambridge.org/core/terms. https://doi.org/10.1017/9781108566100.011
Chapter 5a: Clinical Anaesthesia Questions 423

Question 390
A 55-year-old male presents for a right nephrectomy. He has a background of chronic lower
back pain and normally takes morphine sulfate 100 mg twice daily. He has declined an
epidural to control postoperative pain. Which of the following would correctly replace his
background opioid requirements over 24 hours?
a. IV morphine 33 mg per 24 h
b. IV morphine 66 mg per 24 h
c. IV morphine 2.75 mg.h–1
d. IV morphine 5.25 mg.h–1
e. SC diamorphine 50 mg per 24 h

Question 391
A 60-year-old woman presents for parathyroid surgery. Prior to induction of anaesthesia:
a. Serum calcium should be less than 4 mmol.l–1
b. CT scanning of the neck is mandatory
c. Screening for other endocrine diseases should be considered
d. All patients should receive steroids
e. All patients should be screened for a long QT interval

Question 392
Conditions associated with phaeochromocytoma include:
a. Multiple endocrine neoplasia 1
b. Multiple endocrine neoplasia 2
c. Von Recklinghausen’s disease
d. Von Hippel–Lindau syndrome
e. Klippel–Trenaunay syndrome

Question 393
A 30-year-old male was brought to the ED one hour following ingestion of antifreeze.
The following options are correct regarding further management:
a. Activated charcoal may be useful in this case
b. Haemofiltration may be indicated
c. He is likely to develop a metabolic acidosis with a high anion gap
d. The toxin involved usually follows first-order kinetics
e. IV fomepizole is indicated based on the above history

Question 394
Obesity affects the pharmacokinetics of intravenous anaesthetic drugs. Which of the
following drugs should have dosing based on ideal body weight?
a. Propofol
b. Rocuronium
c. Vecuronium

Downloaded from https://www.cambridge.org/core. University of Edinburgh, on 19 Aug 2019 at 13:21:27, subject to the Cambridge Core terms of
use, available at https://www.cambridge.org/core/terms. https://doi.org/10.1017/9781108566100.011
424 Chapter 5a: Clinical Anaesthesia Questions

d. Paracetamol
e. Morphine

Question 395
Co-morbidities are affected by fat distribution in obese patients. Regarding an android fat
distribution:
a. It increases likelihood of a difficult airway
b. It increases the likelihood of difficult venous access
c. It increases the likelihood of difficulties in ventilation
d. It increases the likelihood of metabolic co-morbidities
e. It decreases the likelihood of cardiovascular co-morbidities

Question 396
A 68-year-old male presents to the ED with severe dyspnoea and resolved chest pain. His
ECG shows sinus tachycardia with a rate of 130 min–1 and no ST segment changes. His BP is
90/42 mmHg, RR is 44 min–1 and his SpO2 is 92% on FiO2 of 1.0. He is known to be diabetic,
hypertensive and suffers from ischaemic heart disease. He also gives a history of previous
DVT two years ago. Which of the following decisions is appropriate regarding his
management?
a. Immediate thrombolysis if cardiac arrest is imminent
b. Urgent CTPA followed by thrombolysis if pulmonary embolism is detected
c. Stabilize the patient on ICU then request an urgent echo and CTPA
d. Bedside echocardiogram followed by thrombolysis if the right ventricle is dilated
e. Bedside echocardiogram followed by immediate percutaneous coronary intervention if
there is a wall motion abnormality

Question 397
The following statements are correct regarding infective endocarditis:
a. Streptococcus viridans is the most common organism causing infective endocarditis
b. Fungal endocarditis usually requires surgical intervention
c. Anticoagulation is indicated once vegetations are confirmed by echocardiography
d. Streptococcus bovis raises the suspicion of intestinal malignancy
e. At least two sets of blood cultures should be taken before starting antibiotics

Question 398
The following are signs of a developing total spinal:
a. Increasing anxiety or sense of panic
b. Hypotension and bradycardia
c. Tingling in the fingers
d. Sudden whispering voice
e. Respiratory arrest

Downloaded from https://www.cambridge.org/core. University of Edinburgh, on 19 Aug 2019 at 13:21:27, subject to the Cambridge Core terms of
use, available at https://www.cambridge.org/core/terms. https://doi.org/10.1017/9781108566100.011
Chapter 5a: Clinical Anaesthesia Questions 425

Question 399
The following are true in the management of the pregnant trauma patient over 20 weeks’
gestation:
a. Around 5–7% of pregnant women undergo some form of trauma
b. Once the primary survey has been completed then manual displacement of the uterus
must be performed
c. In patients with pelvic fractures there is a fetal mortality incidence of up to 25%
d. Uterine rupture is rare, even with direct abdominal trauma
e. Placental abruption occurs in 3–4% of minor trauma cases

Question 400
You are asked to anaesthetize a term neonate for correction of tracheo-oesophageal fistula
(TOF), which was diagnosed antenatally. Which of the following statements are correct?
a. 50% of patients with oesophageal atresia will have another congenital abnormality
b. Surgery should take place within the first 24 hours of birth
c. The Spitz classification is used to describe the anatomical variations of oesophageal
atresia and TOF
d. Failure to pass an NG tube is commonly the only diagnostic sign
e. IV induction is preferred over gaseous induction

Downloaded from https://www.cambridge.org/core. University of Edinburgh, on 19 Aug 2019 at 13:21:27, subject to the Cambridge Core terms of
use, available at https://www.cambridge.org/core/terms. https://doi.org/10.1017/9781108566100.011
Chapter
Clinical Anaesthesia Answers

5b
Question 1: FFFTF
Malignant hyperthermia (MH) is a rare autosomal dominant condition. If this patient’s
biological father had MH his risk is likely to be 50%. The responsible gene mutation is on
chromosome 19 in the majority of patients, resulting in three abnormal isoforms of the
ryanodine receptors in muscle (plasma cholinesterase is coded for on chromosome 3). Up to
15 relevant mutations at chromosome 19 have been identified and point mutations may
occur, resulting in cases with no relevant family history. The abnormality results in an
abnormal ryanodine calcium channel in muscle that allows excessive calcium to move from
the endoplasmic reticulum into the cytoplasm, with uncontrolled muscle contraction.
Dantrolene is used to treat MH by uncoupling the excitation contraction process and
blocking the ryanodine calcium channel. MH may develop after exposure to triggering
agents, with some reports up to 12 hours post exposure, and can occur after previous
uneventful general anaesthetics.

Question 2: TTFFT
In children a cuffed tube is not always used, in order to prevent tracheal stenosis; an
uncuffed tube can provide a secure airway due to the anatomical variation in children.
A method of detecting CO2 will confirm placement, but continuous capnography is not the
only available method; in prehospital practice a colorimetric device is used.

Question 3: TFFFT
The Third National Audit Project of the Royal College of Anaesthetists (NAP3) investigated
the major complications following central neuraxial block. Staphylococcus aureus was found
to be the most common organism associated with epidural abscesses. The majority of
complications following perioperative central neuraxial block (CNB) occurred with epidur-
als. Vertebral canal haematoma commonly presents with symptoms of leg weakness.
In NAP3 weak legs were a universal symptom in cases of vertebral canal haematoma, but
back pain was rare. The incidence of permanent injury after adult perioperative epidural
was 8.2–17.4 per 100 000. The incidence of paraplegia and death following CNB was found
to be 0.7–1.8 per 100 000.

Question 4: FTTFT
Cyanotic heart disease is a group of illnesses in which the deoxygenated blood travels to the
systemic circulation without entering the pulmonary circulation (right to left shunt).

426
Downloaded from https://www.cambridge.org/core. University of Edinburgh, on 19 Aug 2019 at 13:21:26, subject to the Cambridge Core terms of
use, available at https://www.cambridge.org/core/terms. https://doi.org/10.1017/9781108566100.012
Chapter 5b: Clinical Anaesthesia Answers 427

In coarctation of the aorta there is no alteration of the normal flow but rather stenosis in the
descending thoracic aorta. Tetralogy of Fallot is characterized by right ventricular outflow
obstruction, VSD, aortic root over-riding a high VSD and RV hypertrophy. Thus the blood
is shunted from the right ventricle to the aorta. After birth the pulmonary vascular
resistance (PVR) drops below the SVR making any shunt through ASD or VSD almost
always a left to right shunt. Only when complicated with severe pulmonary hypertension
(Eisenmenger’s syndrome) does the reversal of shunt occur leading to cyanosis in condi-
tions with isolated septal defects.

Question 5: TFFTT
One metabolic equivalent (MET) is equivalent to 3.5 ml.kg–1.min–1 oxygen consumption
and represents the oxygen consumption of an adult at rest. Patients should be able to
perform more than 4 METS to undertake major surgery, which correlates clinically to being
able to climb at least one flight of stairs. MET values of activities range from 0.9 (sleeping) to
23 (running at 22.5 km.h–1).

Question 6: FFFFF
Myasthenic syndrome is a diagnosis related to myasthenia gravis (MG), also known as
Eaton–Lambert syndrome. There are some important features of myasthenic syndrome
distinguishing it from MG. There is decreased release of acetylcholine from the presynaptic
nerve terminal, as opposed to IgG autoantibodies directed at the postsynaptic acetylcholine
receptor seen in MG. Muscle weakness in myasthenic syndrome predominantly affects the
proximal muscles, as opposed to the generalized pattern often with ocular and bulbar
muscle involvement seen in MG. Weakness in MG is typically worse on exertion and
improves with rest and the opposite pattern is true in myasthenic syndrome, with electro-
myography showing an increase in power on titanic stimulation. Patients with myasthenic
syndrome show an increased sensitivity to both depolarizing and non-depolarizing muscle
relaxants. In MG there is increased sensitivity to non-depolarizing muscle relaxants, but
a relative resistance to suxamethonium, with up to twice the normal dose being required.
Acetylcholinesterase inhibitors (such as neostigmine and more commonly pydridostig-
mine) are a mainstay in the pharmacological treatment of MG, but result only in slight
improvement in muscle weakness in myasthenic syndrome. Other features of myasthenic
syndrome not seen in MG include autonomic system disturbance and the depression or
absence of tendon reflexes.

Question 7: TTTFF
Postherpetic neuralgia (PHN) is the term used to describe the painful aftermath of herpes
zoster (HZ) infection, also known as shingles. The diagnosis is given to patients who still
have pain three months or more following HZ. It is the reactivation of varicella zoster virus
(VZV) that gives rise to HZ and it remains in a latent state in spinal and cranial sensory
ganglia until reactivation. Although most people are immune due to childhood vaccination
or exposure to wild-type virus, immunity may be decreased – by disease or immune
suppression – and reactivation occur.

Downloaded from https://www.cambridge.org/core. University of Edinburgh, on 19 Aug 2019 at 13:21:26, subject to the Cambridge Core terms of
use, available at https://www.cambridge.org/core/terms. https://doi.org/10.1017/9781108566100.012
428 Chapter 5b: Clinical Anaesthesia Answers

Risk factors include:


• Older age (it is rare below 50 years)
• Female sex
• Acute pain and rash severity
• Dermatomal pain before rash appears
Most patients experience a painful vesicular eruption in a single dermatome that settles
within three months. However, approximately 20% will develop PHN. The pain is intense
and described as burning, throbbing, stabbing or shooting. It can be continuous or inter-
mittent, and patients often experience allodynia and hypersensitivity. The pain can be very
debilitating and lead to depression and social isolation.

Question 8: TFFFF
Some measures that are part of good intensive care practice also apply to the management of
the potential heart-beating donor, but there are additional measures shown to increase the
viability and number of transplantable organs.
Endocrine dysfunction following brainstem death can contribute to organ failure and
hence hormone replacement may help preserve homeostasis. The hormones commonly
replaced are insulin, methylprednisolone and triiodothyronine. The rationale for using
these hormones is: insulin for treating hyperglycaemia, methylprednisolone to counter
the cytokine-driven inflammatory response and thyroid hormones to improve the function
of transplanted hearts in the recipient.
Donor lungs are susceptible to fluid overload and so considerations may include the
measurement of left-sided filling pressures and avoiding a CVP of >6 mmHg (without
PEEP), which may worsen the alveolar–arterial oxygen gradient. The use of lung protective
ventilation, including a positive end expiratory pressure of 5–10 cmH2O, can be effective in
treating pulmonary oedema and preventing alveolar collapse.
Hypotension is initially managed with volume loading because potential donors often are
often relatively vasodilated, but where vasopressor support is required vasopressin is the
first-line agent. In septic patients doses of vasopressin >2.5 U.h−1 are associated with adverse
outcomes, including cardiac arrest.

Question 9: FFTTF
The Rule of Nines is a quick method used to estimate medium to large-sized burns in adults
(it is not accurate in children). The body is divided into areas of 9% TBSA (see Table 5.9.1).

Table 5.9.1

Head (front and back) = 9%


Anterior chest = 18%
Back = 18%
Each arm = 9%
Each leg = 18%
Perineum = 1%

Downloaded from https://www.cambridge.org/core. University of Edinburgh, on 19 Aug 2019 at 13:21:26, subject to the Cambridge Core terms of
use, available at https://www.cambridge.org/core/terms. https://doi.org/10.1017/9781108566100.012
Chapter 5b: Clinical Anaesthesia Answers 429

For small burns (generally < 5% TBSA) the palmer surface method can be used.
In this method the surface of the patient’s palm, including the fingers, is estimated to be
approximately 0.8–1% of TBSA and can be used to estimate the burn area.

Question 10: TFFTF


Bariatric surgery has been sanctioned by NICE as a recommended treatment for obesity, and
has been shown to cause a maintainable reduction in weight of more than 50% in some cases.
Laparoscopic techniques have a lower morbidity and mortality in the short term; this is
thought to be due to differences in wound healing and postoperative pain causing problems
with respiratory function. According to studies by Brodsky et al., raised BMI in isolation is
not an indicator of difficult intubation, but raised BMI with other signs such as a Mallampati
score of >3 is an indication of a potentially difficult airway. The incidence of OSA in obese
patients is approximately 5%, but a history of daytime somnolence, apnoeic periods or
snoring should be sought, as preoperative CPAP/BiPAP may be helpful. Due to excess limb
weight and positioning, nerve injuries are more common in the obese. Suxamethonium dose
should be based upon actual body weight due to increased plasma cholinesterase activity.

Question 11: TTTFF


RCOA guidelines require a number of features specific to paediatric day surgery. A PICU on
site is not essential unless infants with chronic lung disease are undergoing surgery. Ex-
premature neonates should not undergo day-case anaesthesia unless over 60 weeks post
conception and medically fit. Play specialists are not obligatory, but suitable paediatric
facilities must be available.

Question 12: FTFTT


There are several bedside tests that may predict difficult intubation:
• The inability to protrude the mandibular incisors
• A sternomental distance less than 12 cm
• A thyromental distance less than 6 cm
• Mallampati score 3 or 4
• The presence of buck teeth
• Limited ability to extend the neck
Previous radiotherapy to the head and neck can cause formation of fibrotic tissue and
reduced mobility of tissues, causing difficulty at intubation. Previous tracheostomy forma-
tion or prolonged intubation may result in scarring and tracheal stenosis.
The presence of numerous congenital syndromes, including Pierre Robin, Treacher
Collins and Goldenhar syndrome, plus mucopolysaccharide disorders such as Hurler’s
and Hunter’s syndromes, are associated with difficult intubation. The presence of a high-
arched palate is seen in Marfan’s and Down’s syndromes and may complicate intubation.

Question 13: TFFTF


Thoracic paravertebral blocks provide an ipsilateral somatic and sympathetic nerve block
similar to a unilateral epidural block, especially useful for breast surgery, thoracotomy, in

Downloaded from https://www.cambridge.org/core. University of Edinburgh, on 19 Aug 2019 at 13:21:26, subject to the Cambridge Core terms of
use, available at https://www.cambridge.org/core/terms. https://doi.org/10.1017/9781108566100.012
430 Chapter 5b: Clinical Anaesthesia Answers

patients with rib fractures or open cholecystectomy. The thoracic paravertebral space is
a wedge-shaped area that lies on either side of the vertebral column defined by:
• Parietal pleura anterolaterally
• The vertebral body, intervertebral disc and intervertebral foramen medially
• The superior costotransverse ligament posteriorly
The space is continuous with the intercostal space laterally, epidural space medially and
contralateral paravertebral space via the prevertebral fascia. As the nerves emerge from
intervertebral foramina, they transverse through the paravertebral space where they may be
blocked by local anaesthetics, thereby blocking dorsal and ventral rami and hence the
sympathetic chain.
The block can be inserted with ultrasound guidance, but more commonly is performed
using a landmark technique. C7 is the most prominent cervical spinous process, whilst the
lower tip of the scapula lines up with T7.
Complications of paravertebral blocks include infection, haematoma, local anaesthetic
toxicity, nerve injury and, rarely, total spinal anaesthesia and paravertebral muscle pain
(resembling muscle spasm mainly in young muscular men, especially when larger gauge
Tuohy needles are used).

Question 14: FFTTF


There is no good evidence to support one inotrope over another in cardiogenic shock.
Dobutamine is used frequently, not only because of its positive inotropic effect, but also as
a peripheral vasodilator, reducing the afterload against a failing heart. In this case the BP is
significantly low and will be made worse by dobutamine. Adrenaline is a potent vasopressor
through its action on the α1-receptor, which will support the SVR in this situation.
In diastolic dysfunction the cardiac output is dependent on venous return and filling
pressure, which will be reduced by dobutamine due to vasodilatation. IABP can be quite
helpful in patients with ischaemic cardiogenic shock who are expected to be on multiple
inotropes. It will improve the coronary perfusion during diastole and reduce the afterload
during systole. Noradrenaline is not routinely used as a first-line agent in these cases. Raised
lactate in this case is due to pump failure that is unlikely to be helped by noradrenaline.
However, it can be added later on if the low BP proved to be resistant to either adrenaline or
dopamine as a single agent.

Question 15: TFFTT


Acute fatty liver of pregnancy is a serious condition affecting approximately 5 in every
100 000 pregnancies and has a significant maternal mortality and morbidity. The maternal
mortality rate is now in the region of 10–20%, having been over 85% when the disease was
first identified. It often presents later in pregnancy (after 30 weeks). Common signs and
symptoms include jaundice, abdominal pain, altered mental state, nausea and acute renal
impairment. The features of pre-eclampsia are present in a significant proportion of
patients and it can be difficult to differentiate clinically and biochemically from HELLP
syndrome. There is a higher incidence of AFLP in first pregnancies, multiple pregnancies
and when the fetus is male (3:1 M:F). Radiological appearances can be normal and liver
biopsy remains the gold standard test; it is, however, often contraindicated due to an
underlying coagulopathy. There are risks and benefits of both general and regional

Downloaded from https://www.cambridge.org/core. University of Edinburgh, on 19 Aug 2019 at 13:21:26, subject to the Cambridge Core terms of
use, available at https://www.cambridge.org/core/terms. https://doi.org/10.1017/9781108566100.012
Chapter 5b: Clinical Anaesthesia Answers 431

anaesthesia and both have been used safely, provided none of the usual contraindications
are present.

Question 16: FFTTF


CPET is a very useful test in assessing patients for lung resection. It is not needed for all
patients. In those patients with poor predicted lung function as well as unexplained poor
functional capacity it is indicated. If the peak VO2 is above 20 ml.kg–1.min–1 they can usually
have resection up to pneumonectomy . The ability to climb two flights correlates with a VO2
max of approximately 12 ml.kg–1.min–1. The anaerobic threshold is approximately 55% of
VO2 max in untrained individuals but rises to >80% in trained athletes.

Question 17: FTTFT


Sickle cell disease is an inherited haemoglobinopathy resulting from a mutation on chro-
mosome 11. The mutation causes a pathological amino-acid substitution of valine for
glutamic acid on the β-globin chain of haemoglobin A. This substitution produces haemo-
globin S, which is inherently unstable and can adopt the notorious ‘sickle’ appearance under
®
certain conditions. The Sickledex test is a sickle solubility test and is used in emergency
situations as a rapid screening test. It detects haemoglobin S levels greater then 10%, but is
unable to differentiate between homozygous (sickle cell disease) and heterozygous (sickle
cell trait) conditions. Haemoglobin electrophoresis is the definite distinguishing test. Sickle
cell trait has been shown in numerous studies to confer strong protection against
Plasmodium falciparum malaria. The protective mechanisms are not fully understood.
However, proposed mechanisms include a reduction in parasite growth and enhanced
removal of parasitized cells through acquired or innate immune systems.

Question 18: FTFTT


The tongue is large and the larynx is situated more anteriorly and cephalad (C3–C4).
The epiglottis is large and U-shaped. The cricoid cartilage is the narrowest part of the
upper airway and a small decrease in diameter caused by oedema or stricture formation
following prolonged tracheal intubation may lead to airway obstruction.
Tidal volume is fixed and ventilatory frequency needs to be increased to increase minute
ventilation. Ventilation is mainly diaphragmatic and there are fewer type I muscle fibres, so
infants fatigue earlier. FRC is less than the closing capacity owing to the low elastic recoil of
the chest wall. This, along with the high metabolic requirement, predisposes them to
hypoxia.

Question 19: FFTFF


Recognition of malignant hyperpyrexia (MH) is key: the AAGBI guidance from 2011
suggests unexplained increase in heart rate and end tidal CO2, alongside increased oxygen
requirement, and possibly late-onset temperature rises, should prompt recognition of MH.
Volatile anaesthesia should be discontinued and anaesthesia maintained via intravenous
agents. The patient should be ventilated with 100% oxygen via a clean circuit and preferably
via a dedicated anaesthetic machine that has not been in contact with volatile anaesthetic
agents. While hyperventilation may help control respiratory acidosis, it alone is not

Downloaded from https://www.cambridge.org/core. University of Edinburgh, on 19 Aug 2019 at 13:21:26, subject to the Cambridge Core terms of
use, available at https://www.cambridge.org/core/terms. https://doi.org/10.1017/9781108566100.012
432 Chapter 5b: Clinical Anaesthesia Answers

adequate management. Close liaison with the surgical team is vital – explain that there is an
anaesthetic emergency and that completion of surgery should be expedited, or, if feasible,
surgery abandoned. In this case, conversion to an open procedure may be warranted
depending on surgical experience/expertise. The recommended bolus dose is now
2.5 mg.kg–1 of dantrolene with further 1 mg.kg–1 boluses up to 10 mg.kg–1 . Active cooling
measures need to be taken, but using ice is likely to cause peripheral vasoconstriction that is
counterproductive and should be avoided.

Question 20: TTFTT


Risk factors can be split into a number of categories:
• Delayed gastric emptying (pain, trauma, opioids, alcohol)
• Medical conditions (GORD, hiatus hernia, diabetic neuropathy, neurological/
neuromuscular disease)
• Acute illness (bowel obstruction/ileus, metabolic derangement, impaired consciousness)

Question 21: TFTFF


In 2015/16, 33% of renal transplants in the UK were from living donors. Under UK
guidelines both a consultant surgeon and consultant anaesthetist should be present during
the donor nephrectomy. As long as the renal function tests and urine production are
normal, hypertension is not a contraindication to being a living donor. Diabetes mellitus
is a contraindication to donating, but those with impaired glucose tolerance may be
considered if fasting glucose is <7.0 mmol.l–1, oral glucose tolerance test is <7.8 mmol.l–1
and BMI is <30 with no family history of diabetes. As a nephrectomy is considered a clean
procedure, routine antibiotic prophylaxis is not required. Unless there are specific patient
concerns, routine use of invasive monitoring is not necessary.

Question 22: FTFTF


During intrauterine life, the passage of deoxygenated blood from the pulmonary artery
through the ductus arteriosus (DA) into the aorta stimulates release of prostaglandin E2
(PG E2). This in turn helps to keep the DA patent. At birth, with the first breath, the flow
reverses, with oxygenated blood passing from the aorta to the pulmonary artery due to
changes in pressure in both circulations. This will lead to closure of the DA.
In conditions where the deoxygenated blood bypasses the pulmonary circulation, such as
tetralogy of Fallot, transposition of the great arteries and pulmonary stenosis, keeping the
DA patent is very useful to allow the blood flow to the pulmonary circulation to improve
oxygenation.

Question 23: FTFTF


A 1% solution contains 10 mg.ml–1 of the substrate.
The safe dose of lignocaine is 3 mg.kg–1 plain and 7 mg.kg–1 with adrenaline.
The safe dose for bupivacaine/levobupivacaine is 2 mg.kg–1 (max of 150 mg) regardless of
whether adrenaline is in the mixture.
Prilocaine is safe at 6 mg.kg–1 and 9 mg.kg–1 with adrenaline.

Downloaded from https://www.cambridge.org/core. University of Edinburgh, on 19 Aug 2019 at 13:21:26, subject to the Cambridge Core terms of
use, available at https://www.cambridge.org/core/terms. https://doi.org/10.1017/9781108566100.012
Chapter 5b: Clinical Anaesthesia Answers 433

A 10 ml dose of prilocaine 2% contains 200 mg, which is within the safe dose for
a 60 kg patient. However, it will be inappropriate for this operation as the spinal will be
unnecessarily high and the duration of action will be very short for the procedure
mentioned.
The use of adrenaline is contraindicated near end arteries, e.g. ring blocks or penile
blocks.

Question 24: TTFTT


Intraoperative neurophysiological monitoring is commonly used to test the integrity of
spinal cord functions in scoliosis surgery. Somatosensory evoked potentials (SSEPs) and
motor evoked potentials (MEPs) monitor the response of the spinal cord distally in response
to a stimulus applied proximally. SSEPs are applied at a peripheral nerve and detected using
epidural or scalp electrodes. MEPs are applied using scalp electrodes to the motor cortex
and subsequently detected using epidural electrodes or compound muscle action potentials
(CMAPs). To negate background noise, multiple signals applied over 2–3 minutes are
averaged.
Neuromuscular blocking drugs prevent CMAPs, but may help reduce background noise
when detecting SSEPs. SSEPs and MEPs are depressed by volatile agents, nitrous oxide and
propofol, but opioids have little effect. To identify neurological injury, as opposed to drug-
induced changes in SSEPs and MEPs, it is important to achieve steady state anaesthesia with
minimal changes in the drugs administered after induction. Changes in physiological
parameters such as reductions in blood pressure and temperature may also depress signals.
If changes thought to be due to neurological injury are detected (reduced amplitude,
increased latency or loss of waveform), a ‘wake-up test’ to clinically assess the integrity of
the relevant pathways is necessary.

Question 25: FTTTF


Overall estimated incidence of awareness under general anaesthesia is 1:19 000. This varies
in different settings: with neuromuscular blockade (NMB), the incidence increases to 1:8000
and without NMB it is 1:136 000. The main risk factors identified are:
• Drug factors: neuromuscular blockade, thiopental, total intravenous anaesthesia
techniques
• Patient factors: female gender, age (younger adults, but not children), obesity, previous
accidental awareness under general anaesthesia and possibly difficult airway management
• Subspecialties: obstetric, cardiac, thoracic, neurosurgical
• Organizational factors: emergencies, out of hours operating, junior anaesthetists
ASA grade, physical status, race and use of nitrous oxide are not risk factors for accidental
awareness under general anaesthesia.

Question 26: FFFTF


Guillain–Barré syndrome (GBS) is an acute demyelinating polyneuropathy characterized by
ascending motor weakness preferentially affecting the proximal skeletal muscles. GBS is
often preceded by limb paraesthesia, back pain and in more than half of patients a bacterial
or viral illness, commonly a gastrointestinal or respiratory tract infection. The commonest

Downloaded from https://www.cambridge.org/core. University of Edinburgh, on 19 Aug 2019 at 13:21:26, subject to the Cambridge Core terms of
use, available at https://www.cambridge.org/core/terms. https://doi.org/10.1017/9781108566100.012
434 Chapter 5b: Clinical Anaesthesia Answers

responsible pathogen is Campylobacter jejuni, but others that have been implicated include
Epstein–Barr virus, Mycoplasma pneumoniae and cytomegalovirus.
Autonomic disturbance is common in GBS and this may manifest at any stage as
arrhythmias, wide fluctuations in blood pressure and pulse, urinary retention, ileus and
excessive sweating. A quarter of patients with GBS will require ventilatory support. A vital
capacity of <20 ml.kg–1 is an indication for intubation and ventilation; other indications
include maximal inspiratory pressure of <30 cmH2O, maximal expiratory pressure of
<40 cmH2O or a decrease in any of these three parameters by >30%.
Treatment modalities include those that are supportive therapies, e.g. ventilatory support
and physiotherapy, as well as specific therapies to reduce the inflammatory process, e.g.
corticosteroids, intravenous immunoglobulin and plasmapheresis, which may reduce the
severity and duration of the illness.

Question 27: FFFTF


The underlying pathophysiology in acromegaly is the hypersecretion of growth hor-
mone from the anterior pituitary gland. Patients with acromegaly may present with
a number of systemic manifestations that are of concern to anaesthetists, including
changes to the airway, respiratory and cardiovascular systems, some of which are
detailed here.
Macrognathia, macroglossia and excess soft tissue may make laryngoscopy and tracheal
intubation more difficult. Soft tissue enlargement of the upper airway results in up to 70% of
patients having significant obstructive sleep apnoea. Soft tissue overgrowth peripherally
results in an increased risk of nerve entrapment syndromes necessitating meticulous atten-
tion to patient positioning.
Cardiovascular changes in acromegaly can include refractory hypertension with left
ventricular hypertrophy, ischaemic heart disease, arrhythmias, heart block, cardiomyopa-
thy, and biventricular dysfunction. The onset of acromegaly is typically insidious, with
patients often presenting in middle age with advanced features.
The hypersecretion of other anterior pituitary hormones can occur alongside acromegaly
or as part of other conditions, one of which is Cushing’s disease, attributed to the hyper-
secretion of ACTH from a pituitary corticotroph adenoma. Physical features associated
specifically with Cushing’s disease include exophthalmos secondary to retro-orbital fat
deposits, as well as cervical/supraclavicular fat pads making central venous cannulation
more difficult.

Question 28: FTFFT


Myotonic dystrophy is an autosomal dominant disease and the most common of the
dystonias, the others being myotonia congenita and paramyotonia. It has an incidence of
approximately 1 in 20 000, typically presenting in the second or third decade of life with
death occurring in the fifth or sixth decade.
Stimulation of skeletal muscle is followed by persistent contraction, disease progression is
characterized by progressive muscle atrophy (skeletal, cardiac and smooth). The other
important systemic manifestations include cardiorespiratory complications some of
which are discussed here, but also CNS and endocrine dysfunction. Bulbar palsy increases
the risk of aspiration, and respiratory deterioration can occur due to progressive depression

Downloaded from https://www.cambridge.org/core. University of Edinburgh, on 19 Aug 2019 at 13:21:26, subject to the Cambridge Core terms of
use, available at https://www.cambridge.org/core/terms. https://doi.org/10.1017/9781108566100.012
Chapter 5b: Clinical Anaesthesia Answers 435

of central respiratory drive, as well as weakness of the respiratory musculature.


Cardiovascular complications include cardiomyopathy, atrioventricular block, arrhythmias
and mitral valve prolapse (occurs in 20% of patients).
Factors in the conduct of anaesthesia can trigger persistent muscle contraction; poten-
tial triggers include the use of suxamethonium, reversal with neostigmine and hypother-
mia. Regional anaesthesia does not prevent muscle contraction, but local anaesthetic
injected directly into muscles may help. Other strategies to avoid troublesome spasms
that have been effective in some cases include the use of IV quinine and a slow bolus of IV
phenytoin (3–5 mg.kg–1 ).

Question 29: FTFTF


Cerebral palsy (CP) is a group of neurological disorders characterized by varying degrees of
motor, sensory and intellectual impairment. CP is the most common cause (60%) of motor
impairment in childhood. Two-thirds of patients will have some degree of impaired
intellectual and cognitive function. Up to 50% of patients have either focal or generalized
forms of epilepsy. There is an increased risk of aspiration pneumonitis and consequently
chronic lung scarring because of swallowing difficulties, oesophageal dysmotility, abnormal
lower oesophageal sphincter tone and spinal deformity, which lead to gastro-oesophageal
reflux. In the long term, truncal muscle spasticity can lead to scoliosis, restrictive lung
defects, pulmonary hypertension, and ultimately cor pulmonale and respiratory failure.
Succinylcholine is not contraindicated in patients with CP. Non-depolarizing neuromus-
cular blocking agents are less potent and have a shorter duration of action in patients with
CP owing to the upregulation of ACh receptors. Many anaesthetists choose to perform
a rapid sequence induction to secure the airway in patients with reflux, but there is no
evidence to suggest that this is any safer than a gas induction with the patient inclined at
a 20–30° head-up tilt. A gas induction is often the only option in the ‘veinless’ uncooperative
patient.

Question 30: TFFTF


The dose requirement for induction agents is reduced in the elderly. A contracted blood
volume coupled with reduced protein binding lead to a higher free-drug concentration.
Prolongation of arm–brain circulation time dictates that induction agents should be
administered more slowly. Failure to do this leads to inadvertent overdose, often with
marked cardiorespiratory side effects. Virtually all opioids, IV agents and benzodiaze-
pines exhibit an age-related increase in their elimination half-life, resulting in
a prolonged duration of action. This is attributable to an increased volume of distribu-
tion for lipophilic drugs because of the increase in body lipid content in the elderly, and
a reduction in organ-based elimination. The MAC value of all inhalational anaesthetic
agents is reduced by 20–40% from young adult values. Although ageing is associated
with a reduction in muscle mass, the development of extrajunctional cholinergic
receptors offsets the reduction in the expected dosage of neuromuscular blocking agents
required to produce acceptable intubating conditions. However, the time of onset and
the duration of action are both prolonged because of a reduction in cardiac output and
reduced metabolism. Ageing is associated with a reduction in the carotid baroreceptor
response to a fall in blood pressure. Both IV and inhalational anaesthetic agents further

Downloaded from https://www.cambridge.org/core. University of Edinburgh, on 19 Aug 2019 at 13:21:26, subject to the Cambridge Core terms of
use, available at https://www.cambridge.org/core/terms. https://doi.org/10.1017/9781108566100.012
436 Chapter 5b: Clinical Anaesthesia Answers

impair this response, and also depress cardiac and vascular smooth muscle contractility.
The choice of inhalational anaesthetic has no influence on the risk of POCD, but it has
been suggested that propofol via TIVA may be associated with a lower incidence.

Question 31: TFTFT


It is important to select the appropriate probe for the particular scan required. Probes vary
in their:
• Operating frequency range – the higher the frequency, the better the discrimination of fine
detail, but the lower the penetration
• Physical size
• Width of scanned tissue field – dependent on size of the probe
Linear probes have a flat face and give a parallel-sided scan field approximately 1 mm thick.
They generally operate at frequencies between 5 and 18 MHz.
Hockey-stick probes are smaller and often used in paediatrics.
Curvilinear probes have a curved face giving a fan-shaped scan field. They operate at
lower frequencies, typically 2–5 MHz.
Gain control adjusts brightness. More sophisticated machines employ time gain
compensation.

Question 32: FTFFF


Migraine is more likely to affect females and commonly presents in the second or third
decade. It is can occur with (30%) or without (70%) aura. It has a throbbing, pulsating
quality and can be associated with nausea, vomiting, fatigue, confusion, photophobia and
phonophobia. It is not commonly associated with focal neurological abnormality.
Acute treatment includes simple oral analgesics, plus antiemetics or rectal diclofenac.
The triptans are also used in the acute management, whereas pizotifen is used in the
prophylactic management of migraine. Generally, opioids are not considered useful for
migraine.
In this case the patient is presenting with sudden onset headache with focal neurology and
so neuroimaging should be considered, despite her history of migraine.

Question 33: TTTTT


Many different patient positions are utilized in neurosurgery, including all of those men-
tioned in the question. In the majority of patients, pressure- and positioning-related injuries
are avoidable if due diligence is taken in patient positioning.
Damage to the common peroneal nerve (a branch of the sciatic nerve) can occur in the
lateral position, resulting in foot drop. This nerve is particularly vulnerable because of its
superficial course as it wraps laterally around the fibular head.
In the prone position the upper limbs are slightly flexed, then abducted and externally
rotated; this should be done simultaneously in both arms. The brachial plexus is at risk in
this position and particular care should be taken to ensure that chest supports do not
impinge on the axilla.

Downloaded from https://www.cambridge.org/core. University of Edinburgh, on 19 Aug 2019 at 13:21:26, subject to the Cambridge Core terms of
use, available at https://www.cambridge.org/core/terms. https://doi.org/10.1017/9781108566100.012
Chapter 5b: Clinical Anaesthesia Answers 437

Venous air embolism is a well-recognized complication of the sitting position, the


pathophysiology of which relates to the potential for air entrainment into dural venous
sinuses that are relatively non-collapsible.
The lateral position carries the highest risk of ocular complications in both the dependent
and non-dependent eye, the most of common of which is corneal abrasion. A head ring or
horseshoe headrest if malpositioned can result in direct pressure on the globe, causing
ischaemic optic neuropathy.

Question 34: FTFTT


The Lund–Browder chart is an accurate tool commonly used to measure percentage burns
in children and adults (Table 5.34.1). As opposed to the Rule of Nines, the Lund–Browder
chart compensates for the changes in body proportions in the head, thighs and legs that
occur with age, the head (front and back) making up 17% of total body surface area (TBSA)
in a one-year-old child compared with only 7% in an adult. It has been noted that the
Lund–Browder chart does not take into account obesity, breast size, pregnancy status and
amputated body parts, all of which may affect the calculated body surface area.

Table 5.34.1 The Lund–Browder chart

Area Age 0 1 5 10 15 Adult

Half of head 9.5 8.5 6.5 5.5 4.5 3.5


Half of one thigh 2.75 3.25 4 4.5 4.5 4.75
Half of one lower leg 2.5 2.5 2.75 3 3.25 3.5

The neck (1%), perineum (1%), arms (10% each), torso (13% each side), buttocks (2.5%
each) and feet (3.5% each) are the same %TBSA, regardless of age.

Question 35: FTTFF


Laparoscopy involves the insufflation of the peritoneum with CO2. This gas is chosen
because it is more blood soluble than nitrogen, so it is less likely to cause air embolism, as
it is rapidly absorbed if inadvertent intravascular injection takes place. The diaphragmatic
irritation causes irritation of the phrenic nerve (roots C3–C5), which have a shoulder
dermatomal distribution. Raised ICP (caused by either intracranial neoplasm, head trauma
or hydrocephalus), hypovolaemia and severe valvular or ischaemic heart disease are all
relative contraindications to laparoscopy. Insufflation pressures are typically up to
20 mmHg, as above this point the physiological responses to pneumoperitoneum can
cause a severe fall in cardiac output leading to cardiac arrest. There are some reports of
laparoscopy being combined with epidural or spinal anaesthesia, however a block to T4 is
required and this does not obliterate the referred diaphragmatic pain.

Question 36: FTTTF


Metformin can be continued for surgery (except where contrast media are given in the
context of low glomerular filtration rate). Morning doses of sulfonylureas should be omitted

Downloaded from https://www.cambridge.org/core. University of Edinburgh, on 19 Aug 2019 at 13:21:26, subject to the Cambridge Core terms of
use, available at https://www.cambridge.org/core/terms. https://doi.org/10.1017/9781108566100.012
438 Chapter 5b: Clinical Anaesthesia Answers

for both am and pm lists. GLP1 analogues and gliptins should be paused for the day of
surgery.

Question 37: TTTFF


Tuberculous meningitis is associated with turbid or opalescent CSF, very high protein
(>2 g.l–1), low glucose and lymphocytes. Mumps meningitis may be associated with low
glucose in 20% of cases. Partially treated bacterial meningitis may be associated with
lymphocytes rather that neutrophils, the more classic picture in bacterial infection. CSF
glucose is two-thirds or higher of the serum level and CSF pressure is 6–15 cmH2O.

Question 38: FFTFT


It is important to ensure all patients are adequately starved prior to elective surgery to
reduce the risk of aspiration. The Royal College of Nursing recommends the following as
minimal intervals:
• Water and other clear, non-carbonated drinks, including tea and coffee without milk –
two hours
• Breast milk – four hours
• Food, drinks containing milk, carbonated drinks – six hours
In patients in whom you suspect gastric emptying may be abnormal, consideration should
be given to performing a rapid sequence induction and securing the airway with an
endotracheal tube.

Question 39: FFTFF


Bifascicular block is most commonly used to refer to the presence of a right bundle branch
block co-existing with either an anterior or posterior fascicular block in the left bundle
branch.
In isolated left anterior fascicular block the QRS width is normal: a maximum of 0.12 s.
The only finding is that of left axis deviation. In left posterior fascicular block, right axis
deviation is present without signs of right ventricular hypertrophy. Left anterior fascicular
block is more common than posterior fascicular block; the fascicle has a dual blood supply.
The definition of trifascicular block is more contentious than that of bifascicular block.
It is commonly taken to mean the presence of bifascicular block, that is right bundle branch
block with co-existent left hemifascicular block, with co-existent first-degree atrioventricu-
lar block.

Question 40: FTFTF


In order to standardize the description of permanent pacing and defibrillation systems,
a five-letter international classification system is accepted.The first three letters are always
stated and describe the antibradycardia functions.The last two letters are additional func-
tions and not always stated if absent.
For example, VVI00 describes a pacemaker that:
• V: Paces the ventricle (the chamber paced; can be A – atrial, V – ventricle, D – dual,
0 – none)

Downloaded from https://www.cambridge.org/core. University of Edinburgh, on 19 Aug 2019 at 13:21:26, subject to the Cambridge Core terms of
use, available at https://www.cambridge.org/core/terms. https://doi.org/10.1017/9781108566100.012
Chapter 5b: Clinical Anaesthesia Answers 439

• V: Senses in the ventricle (the chamber sensed; can be A – atrial, V – ventricle, D – dual,
0 – none)
• I: Is inhibited by sensed activity (the response to a sensed beat; can be 0 – none, T –
triggered, I – inhibited, D – dual)
• 0: Is not programmable (programmability; can be 0 – none, P – simple, M –
multiprogrammability, C – communicating, R – rate responsiveness)
• 0: Doesn’t have antitachycardia functions (antitachycardia function; can be 0 – none, D –
dual, P – pace, S – shock)

Question 41: FTTTT


Amniotic fluid embolism (AFE) is a rare (1:50 000) disorder with a mortality of around
20–40%. It was once thought to have a mortality of around 80%, but better identification
and management have reduced this to the current levels. It is thought to be an immune-
mediated response rather than a direct embolic effect. There is no pathognomonic test and
diagnosis is based on clinical signs. It can present atypically with symptoms ranging from
a sense of feeling generally unwell to cardiac arrest; it is possible that it could present with an
isolated coagulopathy. Management is predominantly supportive and the early involvement
of critical care teams is essential to provide the best chance of survival and recovery.
Known risk factors include:
• Maternal age >35
• Induction of labour
• Instrumental or operative delivery
• Placental abnormalities
• Pre-eclampsia/eclampsia

Question 42: FTFTF


For lobectomy, the calculation uses the number of bronchopulmonary segments removed
compared with the total number, 19, in both lungs. Although previous guidelines used
absolute values of FEV1, 1.5 litres for lobectomy and 2 litres for pneumonectomy, this may
lead to false interpretations. For preoperative assessment these values (FEV1, DLCO) should
always be expressed as a percentage of predicted volumes corrected for age, sex and height.
A ppoFEV1 <30% is not an absolute contraindication for lung resection. The patient can
be offered lung resection if they accept the risk involved and potential impact on lifestyle,
such as dyspnoea.

Question 43: FFFFF


Causes of a microcytic anaemia:
• Iron deficiency
• Thalassaemia
• Sideroblastic anaemia

Downloaded from https://www.cambridge.org/core. University of Edinburgh, on 19 Aug 2019 at 13:21:26, subject to the Cambridge Core terms of
use, available at https://www.cambridge.org/core/terms. https://doi.org/10.1017/9781108566100.012
440 Chapter 5b: Clinical Anaesthesia Answers

Question 44: TFTTF


Neonates are obligatory nose breathers and any obstruction causes difficulty in breathing.
Tidal volume is the same as adults (7–10 ml.kg–1), respiratory rate is higher. Dead space is
50% of the tidal volume. Maintenance fluid is 100 ml.kg–1 per 24 h.

Question 45: FFFFF


Anaphylaxis is a type 1, IgE-mediated hypersensitivity reaction. Type 2 hypersensitivity
reactions are cytotoxic immune-mediated reactions via IgG or IgM antibodies reacting to
the presence of antigens. Type 3 immune-complex-mediated reactions involving antigen/
antibody complexes cause activation of complement and neutrophils resulting in tissue
damage – a more gradual process. Type 4 hypersensitivity reactions are delayed reactions
resulting from antigen-specific T cells causing a macrophage-driven response. Anaphylaxis
can occur in anyone, but atopic individuals are more prone. The commonest presentation,
particularly under anaesthesia, is cardiovascular collapse and respiratory compromise
secondary to bronchospasm. Rash is a non-specific sign and may be delayed, or difficult
to ascertain under anaesthesia. Epipens deliver a dose a 0.3 ml of 1 in 1000 adrenaline.
Current ALS guidelines recommend 0.5 ml 1 in 1000 adrenaline IM or in the case of adults
50 μg IV boluses if competent in the IV administration of adrenaline. As the quaternary
ammonium group is commonly found in other drugs, food products and cosmetics,
previous exposure to neuromuscular blockers is not necessary.

Question 46: TFTFT


The majority of complications are secondary to improper pressure point protection or
prolonged positioning. Abdominal compartment syndrome has been described in patients
who have had prone positioning; it is important to allow free movement of abdominal
contents both to prevent this and allow for easier ventilation. Visual loss is as a result of
external orbital pressure.

Question 47: FFFTF


In the UK, conscious sedation is defined as ‘a technique in which the use of a drug or drugs
produces a state of depression of the central nervous system enabling treatment to be carried
out, but during which verbal contact with the patient is maintained throughout the period of
sedation’. The ASA classifies sedation into three levels:
• Minimal sedation: drug-induced state where the patient responds normally to verbal
commands. Cognitive function and physical co-ordination can be impaired, but airway
reflexes, and ventilatory and cardiovascular functions, are maintained
• Moderate sedation: patients respond purposefully to verbal stimuli, which may be
accompanied with light tactile stimulation. This equates to conscious sedation
• Deep sedation: patients respond only to repeated or painful stimulation; can be associated
with significant ventilatory depression
Benzodiazepines can be up to eight times more potent when given after an opioid, therefore
when they are to be used in combination, the opioid should be administered first and the
benzodiazepine titrated carefully after the peak effect of the opioid is seen. The elimination
half-life of midazolam is 1.5–3.5 hours. It is almost completely metabolized in the liver to

Downloaded from https://www.cambridge.org/core. University of Edinburgh, on 19 Aug 2019 at 13:21:26, subject to the Cambridge Core terms of
use, available at https://www.cambridge.org/core/terms. https://doi.org/10.1017/9781108566100.012
Chapter 5b: Clinical Anaesthesia Answers 441

hydroxylated derivatives that are conjugated to glucuronides, these are then excreted in the
urine, thus renal impairment has little effect. The AAGBI recommends that continuous
capnography should be used for all patients undergoing moderate or deep sedation, and
should be available in areas where these patients are recovered. It is appropriate to deliver
oxygen, usually via nasal cannula, to all sedated patients from the administration of the
sedatives until they are ready for discharge from recovery.

Question 48: TTFFT


Factors influencing the development of respiratory distress syndrome:
• Protect:
• Cortisol
• β-adrenergic drugs
• Epidermal growth factor
• Thyroxine
• Prolactin
• Predispose:
• Prematurity
• Male gender
• Ethnicity (Caucasian)
• Genetic predisposition
• Insulin
• Hypoxia
• Hypothermia
• Acidosis
• Hypotension

Question 49: FTTTF


The ulnar nerve is a branch of the medial cord. The musculocutaneous nerve is a branch of
the lateral cord.

Question 50: TFFFT


When oxygen demand exceeds supply, muscle cells begin generating ATP anaerobically. This
process produces lactic acid, which is buffered by bicarbonate and results in an increase in CO2.
The VO2 at the point this occurs is called the anaerobic threshold. Therefore the anaerobic
threshold will not vary with patient motivation, but the peak VO2 will only reach as high as the
effort the patient puts into the test. Anaerobic threshold only reduces slightly with increased
age, but will be reduced in proportion to the degree of organ impairment. An anaerobic
threshold of at least 11 ml.kg–1.min–1 is required to safely undertake significant surgery.

Question 51: TFTTT


Complex regional pain syndrome (CRPS) usually develops after trauma to a limb.
The predominant symptom is pain, but it is also associated with sensory, autonomic,

Downloaded from https://www.cambridge.org/core. University of Edinburgh, on 19 Aug 2019 at 13:21:26, subject to the Cambridge Core terms of
use, available at https://www.cambridge.org/core/terms. https://doi.org/10.1017/9781108566100.012
442 Chapter 5b: Clinical Anaesthesia Answers

motor, skin and bone changes. For a diagnosis of CRPS to be made the patient must meet
the Budapest Diagnostic Criteria (Table 5.51.1).
All the following statements must be met:
• The patient has continuing pain that is disproportionate to the inciting event
• The patient has at least one sign in two or more of the categories below
• The patient reports at least one symptom in three or more of the categories below
• No other diagnosis can better explain the signs and symptoms

Table 5.51.1 Budapest Diagnostic Criteria

Category Sign/symptom

1 ‘Sensory’ Allodynia (pain to light touch and/or


temperature sensation and/or deep somatic
pressure and/or joint movement) and/or
hyperalgesia (to pinprick)
2 ‘Vasomotor’ Temperature asymmetry (>1 °C if counted as
a sign) and/or skin colour changes and/or skin
colour asymmetry
3 ‘Sudomotor/oedema’ Oedema and/or sweating changes and/or
sweating asymmetry
4 ‘Motor/trophic’ Decreased range of motion and/or motor
dysfunction (weakness, tremor, dystonia) and/or
trophic changes (hair/nail/skin)

Hypoaesthesia is not part of the diagnostic criteria.

Question 52: FTTTT


Epilepsy is defined as two or more epileptic seizures with no immediately identifiable
cause. The clinical manifestations of seizures are extremely variable and can range
from behavioural disturbance to life-threatening and unremitting convulsions.
Investigations to determine the cause for a patient presenting with seizures include
electroencephalography (EEG) as well as imaging modalities including CT, MRI,
SPECT and PET scans. Patients with established epilepsy show abnormal epileptiform
activity in up to 50% of cases, but interestingly this is also apparent in 4% of patients
without epilepsy.
It may be necessary to administer anticonvulsant drugs perioperativly, particularly if the
risk of further seizures is high or the seizures are particularly poorly controlled. Of the
commonly used drugs sodium valproate, phenytoin and levetiracetam are available in
intravenous forms.
Status epilepticus (SE) is defined as continuous seizure activity lasting 30 minutes or
intermittent seizures for this period without regaining consciousness. SE carries a mortality
of 25% and has systemic complications that include respiratory failure, pulmonary oedema,
disseminated intravascular coagulopathy, myocardial ischaemia, arrhythmias, cerebral
hypoxia and central venous thrombosis (not an exhaustive list). The prompt management

Downloaded from https://www.cambridge.org/core. University of Edinburgh, on 19 Aug 2019 at 13:21:26, subject to the Cambridge Core terms of
use, available at https://www.cambridge.org/core/terms. https://doi.org/10.1017/9781108566100.012
Chapter 5b: Clinical Anaesthesia Answers 443

of SE is therefore crucial; lorazepam 0.1 mg.kg–1 is commonly recommended as a first-line


agent.

Question 53: FFTTF


The National Network for Burn Care (NNBC) gives the following definitions:
• Burn centre – This level of in-patient burn care is for the highest level of injury complexity
and offers a separately staffed, geographically discrete ward. The service is staffed to the
highest level of critical care and has immediate operating theatre access.
• Burn unit – This level of in-patient care is for the moderate level of injury complexity and
offers a separately staffed, discrete ward.
• Burn facility – This level of in-patient care equates to a standard plastic surgical ward for
the care of non-complex burn injuries.

Question 54: FFTTT


Carcinoid tumours are neuroendocrine tumours that secrete a large amount of vasoactive
peptides such as serotonin, dopamine, corticotrophin, histamine, substance P, neuroten-
sins, kallikrein and prostaglandins. The clinical features of carcinoid crisis are due to the
intravascular release of these hormones. If the primary tumour is found in the gut (as in
67.5% of cases), the vasoactive hormones are metabolized by the liver so rarely cause
systemic effects. For this reason, frequent or severe attacks of flushing, hypotension and
bronchospasm are thought to be due to hepatic metastases or non-GI tumours (e.g.
bronchopulmonary). These hormones, when released into the venous system, cause right-
sided (tricuspid and pulmonary) valvular lesions and fibrous thickening of the endocar-
dium. Left-sided lesions are rare and usually signify right to left shunting or bronchial
carcinoid. Cardiac complications are associated with an increase in perioperative complica-
tions. Tyrosine-containing foods may precipitate an attack due to tumour metabolism, as
may histamine-releasing drugs such as atracurium and morphine. Noradrenaline has been
shown to activate kallikrein in tumours and may lead to synthesis and release of bradykinin,
resulting paradoxically in vasodilatation and worsening hypotension.

Question 55: TFTTT


NICE guidelines recommend ceftriaxone as first-line therapy (unless calcium-containing
solutions are used, in which case cefotaxime should be used). Neuroimaging is indicated if
there is suspicion of raised ICP, an alternative intracranial diagnosis is suspected, focal
neurological signs are present or on expert recommendation. In the UK, suspected or
proven meningitis is a notifiable disease. Young children may present subtle features
suggesting raised ICP, as well as bradycardia, hypertension, reduced conscious level, uni-
lateral pupillary dilatation, tense fontanelles, nausea and vomiting, and papilloedema. NICE
recommend considering intubation if shock persists after more than 40 ml.kg–1.

Question 56: TTFFT


Wilson’s criteria comprise five components: obesity, restricted jaw movement, receding
mandible, the presence of buckteeth and reduced neck movement. Each variable receives
a score between zero and two. A total score greater than two is predictive of a difficult

Downloaded from https://www.cambridge.org/core. University of Edinburgh, on 19 Aug 2019 at 13:21:26, subject to the Cambridge Core terms of
use, available at https://www.cambridge.org/core/terms. https://doi.org/10.1017/9781108566100.012
444 Chapter 5b: Clinical Anaesthesia Answers

intubation. Previous difficult intubation and a reduced thyromental distance are predictive
of difficult intubation, but do not form part of Wilson’s criteria.

Question 57: TTFTT


Ultrasound-guided regional blocks provide faster sensory onset and improved success rates
when compared to landmark-based techniques. There is no current evidence to suggest that
ultrasound use in regional anaesthesia reduces the incidence of neurological complications.
However, there is a reduction in the incidence of vascular puncture and haematoma when
undertaking regional anaesthesia using ultrasound guidance.
There is level 1a evidence to confirm that phrenic nerve blockade is almost always seen
with higher-volume (20 ml or more) local anaesthetic use during brachial plexus block.
Ultrasound guidance allows practitioners to use low volumes.

Question 58: FTTFF


The Goldman Cardiac Risk Index is a multifactorial index of cardiac risk in non-cardiac
surgery (Table 5.58.1). It was developed in 1977 from retrospective data on 1001 patients
undergoing non-cardiac surgery to identify patients at risk from major perioperative
cardiovascular complications. The score is made up from nine independent risk factors,
which have different weightings:
1. Third heart sound/elevated JVP – 11
2. MI in last six months – 10
3. Rhythm other than sinus – 7
4. Ventricular ectopics (>5 per minute) – 7
5. Age >70 – 5
6. Emergency operation – 4
7. Significant aortic stenosis – 3
8. Poor medical condition of patient or bedridden – 3
9. Abdominal or thoracic operations – 3

Table 5.58.1 The Goldman Cardiac Risk Index

Class Score Complication risk (%)

1 0–5 1
2 6–12 7
3 13–25 14
4 26–53 78

Overall the index has high specificity, but low sensitivity.


The index was revised in 1999 by Lee, known as the Revised Cardiac Risk Index, based on
six independent variables (chronic kidney disease, ischaemic heart disease, heart failure,
cerebrovascular disease, insulin-dependent diabetes, intraperitoneal/thoracic surgery). This
has been further superseded by the American College of Surgeons Risk Calculator (2014).

Downloaded from https://www.cambridge.org/core. University of Edinburgh, on 19 Aug 2019 at 13:21:26, subject to the Cambridge Core terms of
use, available at https://www.cambridge.org/core/terms. https://doi.org/10.1017/9781108566100.012
Chapter 5b: Clinical Anaesthesia Answers 445

Question 59: FFFTF


Adrenaline has both inotropic and chronotropic effects, resulting in an increased cardiac
workload. This will eventually increase the oxygen consumption, resulting in lactic acidosis.
Adrenaline also enhances hepatic gylcogenolysis and glycolysis whilst reducing pyruvate
utilization, leading to lactic acid production. The increased heart rate in this scenario gives
a clue that the patient may be helped by a reduction in the adrenaline infusion rate. Starting
vasopressin or increasing the rate of noradrenaline will not add any benefit as both MAP
and urine output are adequate. Dopamine will further increase the heart rate and oxygen
consumption.

Question 60: FTTFT


Cardiac disease remains a significant cause of maternal morbidity and mortality.
Predominant causes of cardiac death are sudden adult death syndrome, myocardial infarc-
tion, aortic dissection and cardiomyopathy. It is the greatest cause of indirect and overall
maternal death. Infarction is the fourth biggest cause of death (behind the others listed
above). Cardiomyopathy can be difficult to diagnose, as its symptoms may be seen normally
in later pregnancy – shortness of breath, swelling, tachycardia and reduced exercise toler-
ance. Dilated cardiomyopathy has a significantly worse prognosis than hypertrophic due to
the additional stress of pregnancy and labour producing a high risk of left ventricular
decompensation. Peripartum cardiomyopathy can present late and should be considered in
any patient with symptoms of cardiac failure up to six months post delivery. Aortic
dissection is still a leading cause of death, with most of those occurring involving the
ascending aorta. Dissection should be excluded in any patient with severe chest pain and
investigation with CT, MRI or USS is appropriate.

Question 61: TFFTF


Thalassaemias have an autosomal recessive pattern of inheritance. Cooley’s anaemia is the
homozygous, major clinical phenotype of β-thalassaemia. It results in a profound anaemia
requiring multiple blood transfusions. Bart’s hydrops fetalis syndrome is the most severe
form of α-thalassaemia, where there is no functional α-allele. This results in formation of
haemoglobin-Bart’s (four γ-chains). Haemoglobin-Bart’s has a very high affinity for oxygen,
resulting in very little tissue oxygen delivery. Bart’s hydrops fetalis syndrome usually results
in intrauterine or early neonatal death, as well as serious maternal risks. A difficult laryngo-
scopy must be anticipated as a consequence of facial changes (for example, maxillary
hypertrophy) from marrow hyperplasia. Haemolysis may be precipitated by oxidant
drugs, including:
• Penicillin
• Prilocaine
• Nitroprusside
• Vitamin K
• Aspirin

Downloaded from https://www.cambridge.org/core. University of Edinburgh, on 19 Aug 2019 at 13:21:26, subject to the Cambridge Core terms of
use, available at https://www.cambridge.org/core/terms. https://doi.org/10.1017/9781108566100.012
446 Chapter 5b: Clinical Anaesthesia Answers

Question 62: TTFFF


Cyanotic heart disease is due to venous blood entering the systemic circulation, having
bypassed the lung. The blood that is pumped out to the body is low in oxygen. Heart defects
like tetralogy of Fallot, transposition of the great arteries, truncus arteriosus, Ebstein’s
anomaly and coarctation of aorta can cause cyanosis. Tetralogy of Fallot is the most
common congenital cyanotic heart disease. The classic form includes four defects, i.e.
large VSD, pulmonary stenosis, right ventricular hypertrophy and an over-riding aorta.

Question 63: TTFTT


Malignant hyperpyrexia (MH) is a rare autosomal dominant condition. The incidence is
approximated at 1:200000, resulting from abnormal ryanodine receptors. Release of cal-
cium through faulty channels results in sustained muscle contraction. This results in muscle
cell damage, with the release of potassium, myoglobin and creatinine kinase. These patients
are therefore at significant risk of acute renal failure and may need renal replacement
therapy. With increased metabolism, temperature and carbon dioxide production, it
would seem logical to expect cutaneous vasodilatation. However, catecholamine surges
result in vasoconstriction initially. Increased metabolic demands of muscle result in diver-
sion of cardiac output from cutaneous vasculature beds to muscle mass. Disseminated
intravascular coagulation is a recognized complication of malignant hyperpyrexia and
there are case reports of compartment syndrome associated with confirmed cases of
malignant hyperthermia.

Question 64: TTFTT


There tends to be some unintended movement of the COETT during change in position and
this may lead to carinal stimulation or endobronchial intubation, which in turn can lead to
atelectasis. Redistribution of blood within the lower limbs can lead to fluid overload in
susceptible patients. Prolonged positioning is a risk factor for the development of VTE and
compartment syndrome of the lower limbs; the sole use of foot stirrups versus combined
calf and foot supports shows no difference in the risk. Alopecia can develop in any position
with the potential for ischemia of the scalp. There is little risk of pressure on the orbit due to
the position (although the risk of damage from the improper use of a facemask or lack of
protective measures for the eye can be a cause of damage in any position).

Question 65: FTFFF


Low-frequency jet ventilation (LFJV) was first described in 1967 by Douglas Sanders. His
technique allowed continuous patient ventilation alongside unrestricted surgical access to the
airway via an open rigid bronchoscope. LFJV is delivered via a handheld device, such as
a Sanders injector or a Manujet, through a short rigid narrow cannula. It can be used in
conjunction with rigid bronchoscopes or laryngoscopes for short surgical procedures, and also
has an important role in emergency airway management via a cricothyroidotomy cannula.
LFJV uses a high-pressure oxygen source and the entrainment of air via the Venturi principle, so
the delivered tidal volume is the sum of the injected and entrained volumes. A jet frequency of
between 8 and 20 min–1 is normally appropriate; this allows time for both chest wall expansion
and passive recoil. The upper airway must be patent to allow adequate exhalation of gases to

Downloaded from https://www.cambridge.org/core. University of Edinburgh, on 19 Aug 2019 at 13:21:26, subject to the Cambridge Core terms of
use, available at https://www.cambridge.org/core/terms. https://doi.org/10.1017/9781108566100.012
Chapter 5b: Clinical Anaesthesia Answers 447

prevent air-trapping and barotrauma. Capnography cannot easily be used, therefore the best
way to assess adequacy of ventilation is to observe the chest wall for expansion and perform
arterial blood gas analysis. When using LFJV for surgical procedures, inhalation agents cannot
be delivered, hence anaesthesia is maintained with TIVA.

Question 66: TTTTT


Achondroplasia is associated with a large tongue and mandible, stenosis of the foramen
magnum, limited neck extension and increased incidence of atlantoaxial instability.
Hurler’s and Hunter’s syndromes are both hereditary, progressive mucopolysaccharidoses,
characterized by progressive generalized infiltration and thickening of the soft tissues. They
are associated with extremely difficult airways for a number of reasons: macroglossia,
tonsillar and adenoidal hypertrophy, narrowing of the nasal passages, short immobile
neck, TMJ involvement, cervical spine instability, and supraglottic and infraglottic soft
tissue thickening. Beckwith–Wiedemann syndrome is the most common overgrowth syn-
drome in infancy, it describes the combination of macroglossia, macrosomia (birth weight
and length greater than the 90th percentile), mid-line abdominal wall defects, ear creases or
pits, and neonatal hypoglycaemia. Goldenhar syndrome (oculo-auriculo-vertebral dyspla-
sia) is characterized by a wide range of congenital anomalies, including micrognathia, facial
asymmetry and hypoplasia, vertebral anomalies causing limited neck movement and ear
malformations with hearing loss. Other anomalies of the oral cavity sometimes seen include
a high arch or cleft palate and abnormalities of the tongue.

Question 67: TTTFF


This preterm baby suffers from severe respiratory distress syndrome. The preterm labour is
mostly due to infection, secondary to prolonged rupture of membranes. Hence antibiotics
should be given in this case. Investigations that would help to confirm the diagnosis include
a chest radiograph, which usually shows symmetrical reticulogranular shadowing and
excludes pulmonary hypoplasia, and an echocardiogram is essential to determine if there
is pulmonary hypertension leading to the higher FiO2 requirement. Pulmonary maturity
can be assessed by measuring the lecithin-to-sphingomyelin ratio from baby’s pharynx or
stomach. Increasing PEEP and the use of high-frequency oscillation are recognized mea-
sures to improve oxygenation rather than increasing FiO2 for longer periods, leading to
oxygen toxicity. Surfactant therapy is usually used as prophylaxis to the preterm within the
first hour of birth. This leads to significant reduction in pneumothorax, bronchopulmonary
dysplasia and mortality.

Question 68: TFFFF


Packed red cells may be stored for up to 35 days with SAGM (saline adenine glucose
mannitol). Platelets can be stored for up to five days at room temperature (20–24 °C) and
up to seven days with bacterial screening. They must be continually agitated to prevent
clumping. Storage is limited to five days due to the risk of bacterial contamination. Platelets
are labelled with ABO and rhesus type. ABO compatibility is preferred, but in adults ABO-
incompatible platelets can be used due to the small volume of plasma containing antibodies.
Cross-matching is not required because a platelet unit contains only 2–5 ml of red cells.
This small red cell volume is capable of creating immunization of the Rh-D antigen and

Downloaded from https://www.cambridge.org/core. University of Edinburgh, on 19 Aug 2019 at 13:21:26, subject to the Cambridge Core terms of
use, available at https://www.cambridge.org/core/terms. https://doi.org/10.1017/9781108566100.012
448 Chapter 5b: Clinical Anaesthesia Answers

therefore Rh compatibility is required for all Rh-negative children and Rh-negative mothers
of childbearing age to prevent the formation of anti-D antibodies.

Question 69: FFFTF


The aim of electroconvulsant therapy (ECT) is to produce a generalized seizure lasting
between 15 and 120 seconds. First described in 1938, during the first three decades of its use
anaesthesia was not employed, but now a general anaesthetic with paralysis is obligatory.
The cardiovascular response to ECT begins with an initial parasympathetic discharge that
can result in bradycardia and hypotension. This is followed by a sympathetic response that
can involve cardiac arrhythmias. Myocardial ischaemia is a risk due to a combination of
increased myocardial oxygen consumption and reduced myocardial oxygen supply.
Common induction agents include propofol and etomidate. Propofol is associated with
quicker emergence, whereas etomidate produces the longest seizure duration and may
reduce the seizure threshold. Neuromuscular blocking agents reduce the risk of serious
injury due to muscular convulsions. Suxamethonium is the most commonly used agent,
usually at a dose of 0.5 mg.kg–1 . Larger doses are sometimes required, for example in cases
of osteoporosis or severe cachexia.

Question 70: FFTFF


The patient is describing the typical symptoms of trigeminal neuralgia (TN). It is defined as
a sudden, usually unilateral, brief stabbing recurrent pain in the distribution of one or more
branches of the trigeminal nerve. TN is usually subdivided into idiopathic and secondary
categories. Idiopathic TN has no obvious cause other than vascular compression. Secondary
TN occurs as a result of a structural lesion such as multiple sclerosis.
The diagnosis of TN can often be made on the basis of the history alone. To image the
trigeminal ganglion and surrounding vessels, MRI and angiography are employed.
The superior cerebellar artery is the more likely vessel to be compressing the trigeminal
nerve. However, not all patients with idiopathic TN have obvious neurovascular contact,
which has led to the development of other theories regarding its pathophysiology.
Carbamazepine is widely used in the management of TN and with some success.
Gabapentin and the lidocaine plaster are also management options, but are not generally
first line. Acyclovir is used in the management of acute herpes zoster.

Question 71: TFFFT


See explanation for Question 28.

Question 72: TFTTF


Myasthenia gravis (MG) is an autoimmune disease characterized by worsening muscle
weakness. In its early stages the muscle weakness may affect the ocular muscles alone, but
as the disease progresses the effects on skeletal muscle are more pronounced and eventually
this may lead to respiratory failure. The pathophysiology is attributed to autoantibodies
specific for the postjunctional acetylcholine receptor, reducing the number of functional
receptors and preventing normal neuromuscular transmission. There are a number of
markers of disease severity that are predictive of the need for postoperative ventilation

Downloaded from https://www.cambridge.org/core. University of Edinburgh, on 19 Aug 2019 at 13:21:26, subject to the Cambridge Core terms of
use, available at https://www.cambridge.org/core/terms. https://doi.org/10.1017/9781108566100.012
Chapter 5b: Clinical Anaesthesia Answers 449

and these include: duration >6 years, pyridostigmine dose >750 mg.day–1, preoperative vital
capacity of <2.9 l and co-existing chronic respiratory disease.
MG sufferers are very sensitive to the effect of non-depolarizing muscle relaxants
(NDMRs) and so a tenth of the usual dose is recommended for intubation and subsequent
maintenance doses. Myasthenic patients are, however, resistant to the effect of suxametho-
nium and so a slightly increased dose (1.5 mg.kg–1 ) is usually needed for rapid sequence
induction. Reversal of NDMRs with a normal dose of neostigmine is possible, but its use
carries the risk of precipitating a cholinergic crisis. The avoidance of NDMRs or the use of
atracurium (spontaneously broken down) is therefore preferable to reversal using
neostigmine.

Question 73: TTTTT


A number of different devices can be used to monitor intracranial pressure; examples
include the extradural fibreoptic probe, subarachnoid screw and external ventricular
drain. These devices measure intracranial pressure, but some can be used to calculate
other indices, including compliance or pressure/volume index and to drain CSF.
The intracranial pressure waveform is similar to the trace produced by an arterial line,
since it corresponds to the pressure changes in large intracerebral vessels, as well as varying
with respiration, reflecting changes in central venous pressure. The three main waveforms
observed are as follows. ‘A’ waves are also known as plateau waves (amplitude 50–100
mmHg, lasting 5–20 min). These represent severely reduced compliance and are most
commonly associated with tumours and cerebral vasodilatation. ‘B’ waves (amplitude
<50 mmHg, occur with a frequency of one per minute) are associated with changes in
respiration and are less useful clinically. A variation of ‘B’ waves known as ‘ramp’ waves is
associated with hydrocephalus. ‘C’ waves (amplitude <20 mmHg, frequency of 4–8
per minute) are not useful clinically and relate to normal physiology: blood pressure and
vasomotor tone. Parameters vary according to the pathology identified, but a sustained rise
in intracranial pressure of >20 mmHg for 30 minutes usually warrants active management.

Question 74: TFTFT


The moorLD12-BI is a laser Doppler blood-flow imaging system used for the non-invasive
mapping of blood flow in an area of skin that has been burnt. NICE guidance supports its
use to guide treatment decisions for patients in whom there is uncertainty about the depth
and healing potential of burn wounds that have been assessed by experienced healthcare
clinicians. It uses a low-power laser beam that scans the burn area and using Doppler
frequency shift calculations displays the blood flow as a colour coded blood-flow image and
a colour video image of the burn wound. Healing potential results based on the blood-flow
image are calculated and reported in three categories: less than 14 days, 14–21 days and
more than 21 days. In burn wounds of intermediate (also known as indeterminate) depth
the moorLD12-BI can be used to demonstrate which areas of any burn wound require
surgical treatment and which do not, enabling decisions about surgery to be made earlier
and for surgery to be avoided in some patients. It has been shown to have a higher accuracy
than clinical assessment, with a variety of criteria, including the ability to predict wound
healing in 14–21 days.

Downloaded from https://www.cambridge.org/core. University of Edinburgh, on 19 Aug 2019 at 13:21:26, subject to the Cambridge Core terms of
use, available at https://www.cambridge.org/core/terms. https://doi.org/10.1017/9781108566100.012
450 Chapter 5b: Clinical Anaesthesia Answers

Question 75: FFTFT


Pulmonary hypertension (PH) is defined as a mean pulmonary arterial pressure over
25 mmHg at rest with a wedge pressure <12 mmHg; moderate PH is said to occur when
the PA pressure is >35 mmHg. The key to safely anaesthetizing these patients is to
maintain coronary perfusion pressure by ensuring a normal or near normal SVR,
avoiding a loss of right ventricular preload, avoiding high PEEP, hypoxia, hypercarbia,
acidosis, hypothermia and pain – all conditions which will increase pulmonary vascular
resistance. Due to their inodilator properties, which may decrease SVR, milrinone and
dobutamine are not recommended for use in PH. Central neuraxial anaesthesia can be
safely used in PH, as long as attention is paid to minimizing the cardiovascular effects
of these techniques. There are no α1-receptors in the pulmonary circulation. Ketamine
may raise pulmonary vascular resistance, but its safe use has been reported in the
literature.

Question 76: TTFFF


A thyroid storm is a hypermetabolic crisis that may occur in thyroid disease or be
precipitated by drugs, radioiodine or surgery. Mortality may be as high as 30%.
Management includes resuscitation and cooling. Specific therapy includes:
• Propranolol, which reduces effects via β-blockade and reduction in T4 to T3 conversion
• Hydrocortisone, which treats relative adrenal insufficiency and blocks T4 to T3 conversion
• Lugol’s solution, which delivers high concentrations of iodine, which suppresses thyroid
hormone synthesis via the Wolff–Chaikov effect (not the Jod–Basedow mechanism).
Amiodarone has a high iodine content and can be a precipitant of a thyroid crisis, so should
be used with great caution. Paracetamol can be used as an antipyretic, but NSAIDs can
displace bound thyroid hormone and should be avoided.

Question 77: TTFFT


The systemic effects of cigarette smoke are widespread, owing to the different chemicals
contained therein. In the respiratory system, airway reactivity is increased, predisposing to
laryngospasm and bronchospasm. There is a greater production of airway secretions
compared to non-smokers, and combined with impaired ciliary function, results in mucous
plugging and sputum retention. Smokers are at risk of developing COPD and lung cancer.
In the cardiovascular system, nicotine acts to increase heart rate, systemic vascular
resistance and blood pressure. Prolonged exposure to cigarette smoke increases the risk of
developing ischaemic heart disease and peripheral vascular disease. In the presence of
carboxyhaemoglobin, the oxyhaemoglobin dissociation curve is shifted to the left, resulting
in impaired release of oxygen to the tissues and the risk of hypoxia. Hepatic enzymes can be
induced by cigarette smoking; this may result in lower levels of drugs than might otherwise
be anticipated. Smoking reduces the risk of postoperative nausea and vomiting.
The mechanism by which this occurs is uncertain.

Question 78: TFFFF


Atrial fibrillation with a spontaneously slow ventricular response is an indication for pace-
maker insertion. The pressing need for surgery must be balanced against the risk of

Downloaded from https://www.cambridge.org/core. University of Edinburgh, on 19 Aug 2019 at 13:21:26, subject to the Cambridge Core terms of
use, available at https://www.cambridge.org/core/terms. https://doi.org/10.1017/9781108566100.012
Chapter 5b: Clinical Anaesthesia Answers 451

compromise to the cardiovascular system if a pacemaker is not inserted preoperatively and


an alternative mechanism of pacing, such as transcutaneous pacing, should be available.
Patients who receive β-blockers as part of their routine management of atrial fibrillation
should take these on the morning of surgery. The risk of developing a rapid ventricular
response in the perioperative period outweighs that of hypotension intraoperatively, which
is usually relatively straightforward to manage with vasoconstrictors or chronotropic
agents.
Digoxin doses should be reduced by a third when an oral dose is converted to intrave-
nous. It is not essential to measure digoxin levels on the day of surgery in patients who have
been established on digoxin long term without a recent dose change; adequate rate control
and an ECG without concerning features of toxicity are acceptable.
Patients undergoing elective surgery are able to take their routine medications – unless
contraindicated – with a small amount of water. There is no need to routinely convert oral
medications to intravenous formulations.

Question 79: TTTFT


Interscalene block provides reliable anaesthesia and analgesia for procedures (open and
arthroscopic) involving the shoulder joint, lateral two-thirds of the clavicle and proximal
humerus. It effectively blocks the proximal nerve roots, distal cervical plexus (supraclavi-
cular nerves) and important nerves such as the suprascapular, which exit proximally from
the plexus. A sterile high-frequency (10–13 MHz) probe is used to scan the neck transversely
between the level of the cricoid cartilage and supraclavicular fossa until an optimal view of
the hypoechoic round images of C5 and C6 nerve roots or upper trunk is obtained. There is
level 1a evidence to confirm that phrenic nerve blockade is almost always seen with
traditional high volume (20 ml or more) blocks, including continuous infusions for post-
operative analgesia.

Question 80: FTTFT


The liver is the principle organ in the body for drug metabolism. The phases of metabolism
are typically split into two and aim to make the drug easier to excrete. Phase 1 reactions are
non-synthetic and involve formation of a new or modified functional group, or cleavage
(commonly oxidation, reduction, hydrolysis or hydration). Phase 2 reactions are synthetic
and involve conjugation with endogenous hydrophilic groups to increase solubility and
hence renal excretion (e.g. glucuronide, sulfate, glycine).
Benzodiazepines undergo oxidation in the liver, whilst morphine undergoes glucuroni-
dation (the most common phase 2 reaction). Morphine is metabolised to morphine-3-
(inactive) and morphine-6-glucoronides (active).
Cytochrome enzymes function as electron transfer agents and are found in many tissues
including the liver, gut and kidneys. They are proteins containing haem as a co-factor.
The term P450 is from the spectrophotometric wavelength absorption maxima for the
enzyme in the reduced state complexed with carbon monoxide (450 nm). These enzymes
are typically found on the inner mitochondrial membrane or in the endoplasmic reticulum,
and are generally terminal oxidase enzymes in electron transfer chains. Cytochrome P450
enzymes are the major enzymes involved in drug metabolism. They are also important in
hormone synthesis and breakdown.

Downloaded from https://www.cambridge.org/core. University of Edinburgh, on 19 Aug 2019 at 13:21:26, subject to the Cambridge Core terms of
use, available at https://www.cambridge.org/core/terms. https://doi.org/10.1017/9781108566100.012
452 Chapter 5b: Clinical Anaesthesia Answers

Question 81: TFTTF


Sepsis and its related complications remain the most common cause of death in the
ICU. Oxygen utilization rather than delivery is the main problem. Normally, antiox-
idant defences act to protect the mitochondria against the production of reactive
oxygen species (ROS). In sepsis this mechanism is overwhelmed by the increased
production of ROS and nitric oxide. This results in an oxidative stress and mitochon-
drial damage leading to impaired ATP production. Although antioxidants have shown
some promising results in the laboratory, there is still little evidence to support their
regular use in humans.

Question 82: TTTFF


It is becoming more common for women with spinal cord injuries to go through labour.
The level of lesion involved can have a significant effect upon potential complications and
labour management. Lesions above T5 have a reasonable likelihood of autonomic dysre-
flexia being present and this can cause severe cardiovascular instability, commonly causing
hypotension, headaches and bradycardia. In extreme cases cardiac arrest is possible. Labour
can be a trigger for this instability and therefore a pre-emptive epidural can be used to
protect against it. The instability can continue for up to 48 hours post delivery so the
epidural should continue into the postdelivery period. Suxamethonium should be avoided
for up to one year post injury to prevent a hyperkalaemic response to its use, although this is
controversial and many clinicians would avoid suxamethonium altogether.

Question 83: TTTTT


Since the majority of pulmonary resection patients have a smoking history, they already
have one risk factor for coronary artery disease. Surgical risk estimate is a broad approx-
imation of 30-day risk of cardiovascular death and myocardial infarction that takes into
account only the specific surgical intervention, without considering the patient’s co-
morbidities. Intrathoracic, non-major surgeries are considered as intermediate-risk proce-
dures, whereas pneumonectomy is considered a high-risk procedure. The surgical stress
increases myocardial oxygen demand. Surgery also causes alterations in the balance between
prothrombotic and fibrinolytic factors, potentially resulting in increased coronary throm-
bogenicity. CT angiography has a high sensitivity for coronary stenosis, but is less specific.
Thus, a patient with a normal CT coronary angiogram can proceed to surgery.

Question 84: FTTFT


The parathyroid glands lie within close anatomical proximity to the thyroid gland. As such,
the parathyroid glands may be accidently damaged during thyroid surgery. Transient
hypocalcaemia may present 36 hours postoperatively in 20% of patients having undergone
thyroidectomy for large multinodular goitre. Permanent hypocalcaemia is rare.
Hypocalcaemia may present with:
• Perioral tingling
• Muscle twitching
• Confusion
• Tetany

Downloaded from https://www.cambridge.org/core. University of Edinburgh, on 19 Aug 2019 at 13:21:26, subject to the Cambridge Core terms of
use, available at https://www.cambridge.org/core/terms. https://doi.org/10.1017/9781108566100.012
Chapter 5b: Clinical Anaesthesia Answers 453

• Layngospasm
• Seizures
• Ventricular arrhythmias
• Trousseau’s sign – carpopedal spasm (may be caused by non-invasive blood pressure cuff
inflation)
• Chvostek’s sign – facial twitch or spasm upon tapping over the facial nerve at the parotid
gland
• Prolonged QT interval
Treatment with calcium replacement should commence immediately. The route of calcium
replacement depends upon serum calcium concentration:
• Serum calcium >2 mmol.l–1 – oral calcium replacement
• Serum calcium <2 mmol.l–1 – intravenous calcium with, e.g. 10 ml of 10% calcium
gluconate. A calcium infusion may be required in severe cases

Question 85: FFFTF


Enzyme functions tend to be poorer in newborns so suxamethonium has a prolonged
action. In addition, 80% of the body weight in a neonate is water so there is a larger
extracellular volume that can dilute suxamethonium.

Question 86: TFTTT


Laparoscopic surgery reduces venous return. Insufflation can cause marked vagal stimula-
tion resulting in bradycardia; coupled with reduced venous return in a dehydrated elective
patient, this can result in marked hypotension. Insufflation should therefore be gradual with
intra-abdominal pressures limited to 20 mmHg. Trochar introduction to the abdominal
cavity may also result in vascular injury, resulting in rapid hypovolaemia. Anaphylaxis
should be amongst the differentials – cardiovascular collapse is a common initial presenting
feature of anaphylaxis under general anaesthesia. Air embolism is a risk with laparoscopic
surgery, especially in the presence of venous injury. However, as CO2 is a relatively soluble
gas, any gas embolism should be short-lived – which is the reason it is the gas of choice when
creating a pneumoperitoneum in laparoscopic surgery. Too small a blood pressure cuff
would cause falsely high blood pressure readings at the extremes of blood pressure.

Question 87: TTFFF


Awareness is most often associated with emergency situations or patients with a higher ASA
grade due to the higher risk of haemodynamic instability. Patients with chronic alcohol and
opiate use are more likely to be aware compared to those with acute intake.
Patients with TIVA are not proven to have higher rates of awareness.

Question 88: FFFTF


The Difficult Airway Society (DAS) rapid sequence induction (RSI) guideline advises:
– 10 N of pressure to the cricoid while the patient is awake and 30 N once consciousness
has been lost

Downloaded from https://www.cambridge.org/core. University of Edinburgh, on 19 Aug 2019 at 13:21:26, subject to the Cambridge Core terms of
use, available at https://www.cambridge.org/core/terms. https://doi.org/10.1017/9781108566100.012
454 Chapter 5b: Clinical Anaesthesia Answers

– That during initial tracheal intubation (Plan A), no more than three attempts should be
made at intubation
– That a second dose of muscle relaxant should not be administered
During anaesthesia, oxygen consumption in a healthy adult stays relatively constant at
approximately 250 ml.min–1. During a RSI, haemoglobin is not important as an oxygen
store, but as an oxygen transporter, therefore anaemia will only cause a small decrease in the
time taken to reach critical hypoxia. However, if the FRC is reduced, anaemia will have
a more significant effect on the time taken for desaturation to occur.

Question 89: FTFFF


The most commonly used irrigation fluid is glycine 1.5% in water, which is hypotonic, with
an osmolality of 220 mmol.l–1. Irrigation fluid is normally absorbed at a rate of approxi-
mately 20 ml.min–1, although absorption can be much greater. The rate of absorption
depends on the infusion pressure of the irrigation fluid and the venous pressure within
the patient; therefore, the pressure of the irrigation must be kept to the minimum that still
allows adequate flow (it should never be at a height of greater than 100 cm). TURP
syndrome is the result of large amounts of irrigation fluid being absorbed through open
venous sinuses, which causes fluid overload and hyponatraemia. TURP syndrome is more
common in procedures lasting over an hour, when the prostate weighs more than 50 g, when
the patient is hypovolaemic/hypotensive (as this creates a greater pressure gradient for
absorption) and when high irrigation pressures are used.

Question 90: TTFTT


The patency of the ductus arteriosus (DA) depends on the high concentration of prosta-
glandin E2. Acidosis and low arterial pO2 are the two main stimuli for PG E2 release.
In pulmonary hypertension, the blood flows from the pulmonary trunk to the aorta before
reaching the lung (deoxygenated blood), leading to release of PG E2, maintaining the DA
open.

Question 91: TTTTF


Local anaesthetic toxicity depends on the volume of the drug injected, as well as the
absorption rate and the site of injection. Intercostal blocks have the highest rate of absorp-
tion after intravenous, then caudal, paracervical, epidural, brachial, sciatic and
subcutaneous.
Signs of LA toxicity depend on the plasma concentration of the individual agent used. For
example: bupivacaine concentration of 2–4 μg.ml–1 and lignocaine level of 10–12 μg.ml–1
can initiate the CNS excitatory stage of LA toxicity. Circumoral numbness and tingling are
the first signs of toxicity, followed by respiratory then cardiovascular signs and symptoms.
Blood samples should be taken to diagnose the plasma levels of the drug used, yet not as
a first line of action. First stop the injection, then deal with the collapsed patient in an ABC
manner. The appropriate action is 100% O2, with constant CVS monitoring and supportive
treatment. Candidates would be advised to familiarize themselves with the AAGBI guide-
lines regarding the use of intralipid.

Downloaded from https://www.cambridge.org/core. University of Edinburgh, on 19 Aug 2019 at 13:21:26, subject to the Cambridge Core terms of
use, available at https://www.cambridge.org/core/terms. https://doi.org/10.1017/9781108566100.012
Chapter 5b: Clinical Anaesthesia Answers 455

Question 92: TFTTF


Cardiopulmonary exercise (CPET) testing requires appropriate monitoring as listed and
two members of staff, one to instruct and look after the patient and the other to watch the
monitoring screen and run the test. The equipment includes a static cycle and a metabolic
cart containing a gas analyzer to enable breath-by-breath measurement of oxygen con-
sumption (VO2) and carbon dioxide production (VCO2). The duration of exercise is
between 6 and 10 minutes.

Question 93: TFFFT


The sciatic nerve is formed from the nerve roots L4 to S3. The two components of the nerve
(tibial and common peroneal) diverge approximately 4 to 10 cm above the popliteal crease
to separately continue their paths into the lower leg.
Blocking the sciatic nerve will provide anaesthesia of the leg below the knee with the
exception of the medial strip of skin, which is innervated by the saphenous nerve.
The inferior or Raj approach to the sciatic nerve requires the patient to lie supine with the
knee flexed to 90°. A line is drawn connecting the greater trochanter to the ischial tuberosity
and needle insertion is at the halfway point in the groove between the hamstring and the
adductor muscles.
The sciatic nerve is usually found at a depth between 40 and 80 mm.
Adrenaline is not advised because of the risk of ischaemia of the sciatic nerve.

Question 94: FFFTF


Dose equivalence of opioid patches:
Table 5.94.1 Buprenorphine

Patch Drug delivery μg.h–1 Oral morphine mg per 24 h

Butrans®5 5 5–10
BuTrans® 10 10 10–20
BuTrans®20 20 25–40

Table 5.94.2 Fentanyl

Patch Drug delivery μg.h–1 Oral morphine mg per 24 h

Fentanyl 12 12 20–60
Fentanyl 25 25 60–100
Fentanyl 50 50 120–200
Fentanyl 75 75 180–300
Fentanyl 100 100 240–400

Downloaded from https://www.cambridge.org/core. University of Edinburgh, on 19 Aug 2019 at 13:21:26, subject to the Cambridge Core terms of
use, available at https://www.cambridge.org/core/terms. https://doi.org/10.1017/9781108566100.012
456 Chapter 5b: Clinical Anaesthesia Answers

Question 95: TTFFF


See explanation for Question 27.

Question 96: TTTFF


See explanation for Question 26.

Question 97: FTTFT


Chlorhexidine gluconate was developed in the UK in the 1950s. It is a cationic, bisbiguanide
molecule that is used widely for skin decontamination prior to procedures. It has good
activity against Gram-positive bacteria, somewhat less activity against Gram-negative
bacteria and fungi, and is minimally active against mycobacteria. It is not sporicidal and
because chlorhexidine is a cationic molecule, its activity can be reduced by natural soaps,
various inorganic anions, non-ionic surfactants and hand creams containing anionic emul-
sifying agents. For skin preparation prior to central neuraxial block, a 0.5% chlorhexidine
preparation is recommended. The antimicrobial activity of chlorhexidine is not seriously
affected by the presence of organic material, including blood. Care must be taken to avoid
contact with the eyes, as concentrations of 1% or greater can cause conjunctivitis.
Chlorhexidine is also ototoxic and direct contact with brain tissue or meninges should be
avoided.

Question 98: TFFFF


Standard ET tubes can be used to isolate one lung in an emergency, and in conjunction with
either bronchial blockers (or unconventionally, Fogarty or Foley catheters), or pulmonary
artery catheters in paediatrics, put down an ET tube. However, for elective surgery this is not
recommended, and an appropriately sized double lumen tube should be used.
The dependent (lowermost) lung is the lung that is being ventilated, so clamping its
pulmonary artery will dramatically worsen hypoxia. Left-sided double lumen tubes are
most commonly used so that the right upper lobe is not accidentally occluded.
Malpositioned ET tubes have been implicated in 30% of deaths in oesophagectomies.
The double lumen tube is rotated 90° on passing through the vocal cords.

Question 99: FFFFF


A low intrinsic thyroid function blunts responses to inotropes and vasopressors – however,
cardiovascular responses are variably affected and cautious initiation is recommended.
Thyroid imaging is not essential, measurement of thyroid function is. Patients are typically
hyponatraemic due to reduced cardiac output and altered renal handling of sodium and
water. Lack of thyroid hormone leads to reduced cardiac index, reduced contractility and
relative bradycardia. Steroid replacement should be considered for these patients, as there is
a risk of concomitant Addison’s being made worse with thyroid hormone administration or
pituitary failure being the cause. Passive rewarming is recommended, but aggressive active
rewarming can lead to vasodilatation and worsening hypotension.

Downloaded from https://www.cambridge.org/core. University of Edinburgh, on 19 Aug 2019 at 13:21:26, subject to the Cambridge Core terms of
use, available at https://www.cambridge.org/core/terms. https://doi.org/10.1017/9781108566100.012
Chapter 5b: Clinical Anaesthesia Answers 457

Question 100: TTTTT


It is important to note the presence of fistulae; intravenous access and blood pressure cuffs
should be sited away from any active fistulae. Patients who are dialysis dependent may be
fluid restricted. This needs to be taken into account when planning perioperative fluid
management. It is therefore also important to note the volume status of the patient and
when they are normally dialyzed, including the date and time of their last session and when
they should next be dialyzed. Elective surgery should usually be planned for the day between
dialysis sessions. Patients with renal disease are at risk of developing peripheral neuropathy.
This should be documented preoperatively, particularly if peripheral nerve blockade is
planned, and care taken when positioning the patient to ensure no further nerve damage
occurs perioperatively.

Question 101: TFFFT


Regional blocks in children are usually performed after general anaesthesia has been
administered. Calculated maximum allowable dose is based on patient ideal body weight
(IBW). Estimating the IBW has particular relevance to the high BMI child.
As the structures of interest in the paediatric patient are relatively superficial, a higher-
frequency probe will be more suitable. Penile block performed using the landmark approach
has reported failure rates of 4–8%. Systemic absorption of local anaesthetic when performing
ilioinguinal and iliohypogastric blocks is higher after ultrasound guidance when compared to
a landmark approach, and careful attention needs to be paid to avoid local anaesthetic toxicity.

Question 102: FTFFF


The commonest cause for liver transplant is posthepatitis C cirrhosis, though acute liver
failure, e.g. post paracetamol overdose, will be prioritized for transplantation. Most cases
involve the transplantation of a whole adult liver from a non-living donor and removing the
diseased organ (orthotopic transplant). Portions of adult livers can be used in paediatrics
and split livers allow one liver to be used for two patients. In living-donor liver trans-
plantation, a portion of a healthy liver is transplanted from the donor to the patient.
Venovenous bypass can be used to allow venous return from the lower part of the body
whilst the liver is being removed and the transplant implanted. Reperfusion syndrome post
reperfusion of the liver graft will cause massive cytokine release and typically is accompa-
nied by a small drop in patient temperature. The anhepatic phase and need for massive
transfusion are also reasons for further intraoperative hypothermia. Typically hypocalcae-
mia is more common during the anhepatic phase as a result of chelation with unmetabolized
citrate. Hepatic artery thrombosis is a real concern in the first few days post transplantation,
but occurs in 0.5–5% of patients. Thrombectomy can be attempted, but retransplantation
may be needed.

Question 103: TTTTF


In June 2012, a panel of experts announced a new definition and severity classification
system for ARDS. It is known as the ‘Berlin Definition’ and aims to simplify the diagnosis of
ARDS and clarify the ambiguity of the old definition. Acute lung injury (ALI) does not exist
anymore, but rather three degrees of severity according to the paO2/FiO2 ratio. Patients with

Downloaded from https://www.cambridge.org/core. University of Edinburgh, on 19 Aug 2019 at 13:21:26, subject to the Cambridge Core terms of
use, available at https://www.cambridge.org/core/terms. https://doi.org/10.1017/9781108566100.012
458 Chapter 5b: Clinical Anaesthesia Answers

a ratio of 200–300 would now have ‘mild ARDS’ rather than ALI. The severity of hypox-
aemia predicts the mortality; being measured at 45% for severe ARDS with a paO2/FiO2
<100. The old definition did not define the ‘acute onset’, which caused confusion in cases of
acute on chronic hypoxaemia. Patients with heart failure can still develop ARDS and hence
there is no more need to exclude a raised PCWP. However, if there is no obvious cause for
ARDS, heart failure shold be excluded as a cause of bilateral lung opacities and pulmonary
oedema. The need for high PEEP predicts neither mortality nor clinical outcome.

Question 104: TTFTF


Cardiac arrests in the pregnant population are increasing in frequency and have a current
incidence of around 1:30 000 pregnancies. This increase is probably caused by the increasing
ages and morbidities of women now going through pregnancy. Perimortem caesarean
section (PMCS) is now widely considered to be the best chance for a successful resuscitation
of the mother, but should aim to be performed within 5 minutes of any arrest. This is,
however, difficult to achieve practically. If PMCS is performed it should be done so where
the woman is being treated and ideally not involve moving her to theatre. Cardiac arrests
occur for a variety of reasons and can occur ‘out of the blue’ during an otherwise apparently
normal pregnancy and labour. There is no change to standard ALS protocols for the
defibrillation of patients.

Question 105: TFFTT


Lung volume reduction surgery (LVRS) is a viable option for a select group of emphysema
patients. General inclusion criteria include:
• Age <75 years
• FEV1 between 15 and 35% of predicted
• paCO2 <55 mmHg (7.3 kPa)
• Prednisone requirement <20 mg.day–1
• PAPsys <50 mmHg
An ideal anatomical precondition for LVRS is a lack of homogeneity of the lung structure,
where normal lung tissue and severely destroyed, over-distended tissue are present in the
same lung. If it is homogeneous and the FEV1 is <20% there is high morbidity and mortality.

Question 106: FTFTT


There are five subtypes of atlantoaxial subluxation that may result from rheumatoid disease.
Anterior subluxation is the most common and can affect up to 80% of patients. It results
from the C1 vertebra moving anteriorly on the C2 vertebra. The transverse ligament and
apical ligaments are destroyed as a consequence, risking spinal cord compression by the
odontoid peg. Anterior subluxation is made worse by neck flexion, whereas posterior
subluxation (affecting less than 5% of patients) is made worse by neck extension.
In posterior subluxation, destruction of the odontoid peg may cause posterior movement
of the C1 vertebra on the C2 vertebra. Vertical subluxation affects between 10 and 20% of

Downloaded from https://www.cambridge.org/core. University of Edinburgh, on 19 Aug 2019 at 13:21:26, subject to the Cambridge Core terms of
use, available at https://www.cambridge.org/core/terms. https://doi.org/10.1017/9781108566100.012
Chapter 5b: Clinical Anaesthesia Answers 459

patients and results from the destruction of the lateral masses of the C1 and C2 vertebrae.
The cervicomedullary junction is then compressed by subluxation of the odontoid peg
through the foramen magnum. Other subtypes of atlantoaxial subluxation include lateral⁄
rotatory (affecting 5–10% of patients) and subaxial (rare).

Question 107: FFTFT


Speed of induction with inhalational agents is increased with high inspired concentration,
increased alveolar ventilation, small FRC, low blood–gas partition coefficient, low cardiac
output and second gas effect.
Second gas effect: administration of rapidly absorbed gas, such as nitrous oxide, given in
high concentration together with a volatile agent of lower solubility produces an increasing
alveolar concentration of the second agent thus promoting its absorption.

Question 108: FFTFT


All the volatile anaesthetic agents and suxamethonium are known to cause malignant
hyperpyrexia (MH) in susceptible individuals. Nitrous oxide, however, does not, but
should be used through a clean circuit, ideally via an anaesthetic machine reserved
solely for MH patients. Ketamine and etomidate are safe to use. Patients are referred to
the specialist centre in Leeds for fresh muscle biopsy that is exposed to halothane and
caffeine. In susceptible patients muscle contraction occurs at lower concentrations than
normal.

Question 109: TFFTF


A disposable four-electrode sensor is placed on the forehead. The frontal EEG signal is
converted by an algorithm to a dimensionless index of the depth of anaesthesia. A reading
between 0 and 100 is produced; zero represents no electrical activity, 100 being awake.
The manufacturers recommend a number between 40 and 60.
BIS monitors are not validated for use in paediatrics, especially in those below the age of
one, despite data suggesting EEG patterns in those over five are similar to adults.

Question 110: FFTFT


The most commonly used irrigation fluid in the UK is 1.5% glycine solution in water.
The ideal irrigation fluid for use during TURP is clear, electrically non-conductive, non-
haemolytic, non-toxic, sterile, not metabolized and cheap. No fluid that fulfils all these
criteria exists; other irrigation fluids include mannitol 5% and sorbitol 3.5%. Glycine is an
inhibitory neurotransmitter, it is metabolized in the liver and kidneys by oxidative deami-
nation to glyoxylic acid and ammonia.

Question 111: FTTFF


The ductus arteriosus normally closes spontaneously at 24 hours (90% by 60 hours). Isolated
PDA should always be treated. Treatment options depend on the size of the PDA. Small
PDA can be treated by indomethacin to prevent infective endocarditis. Moderate PDA

Downloaded from https://www.cambridge.org/core. University of Edinburgh, on 19 Aug 2019 at 13:21:26, subject to the Cambridge Core terms of
use, available at https://www.cambridge.org/core/terms. https://doi.org/10.1017/9781108566100.012
460 Chapter 5b: Clinical Anaesthesia Answers

could be treated by device occlusion via catheterization to prevent pulmonary vascular


disease. Very large PDA is treated by ligation through left thoracotomy, to treat heart
failure. Left to right shunts lead to heart failure due to sequestration, while right to left
shunts lead to cyanosis due to bypassing the pulmonary circulation.

Question 112: TTFTT


Fresh frozen plasma (FFP) must be transfused to an ABO-compatible donor due to the
presence of antibodies in the plasma. The usual FFP starting dose is 10–15 ml.kg–1. For
children up to the age of 16 born after 1995 it is obtained from the USA to reduce the risk of
transmitting variant Creutzfeldt–Jakob disease. The FFP is also treated with methylene blue
to inactivate viruses within the transfusion.

Question 113: FTFFF


The coeliac plexus is the main junction for autonomic nerves supplying the upper
abdominal organs and consists of bilateral coeliac ganglia with a network of interconnect-
ing fibres. The greater (T5–T10), lesser (T9–T11) and least (T11–T12) splanchnic nerves
provide the major contributions to the plexus with input from the vagus and phrenic
nerves also.
The plexus lies anterior to the aorta on either side of the body of L1 and posterior to the
pancreas.
Alcohol is usually preferred to phenol for coeliac plexus block due its greater ability to
diffuse through tissues. Diarrhoea and hypotension are common side effects and damage to
the L1 nerve root may occur. Paraplegia, either through direct trauma or spasm of the artery
of Adamkiewicz, has been quoted as 1 in 683 in one large case study.

Question 114: FTFFF


Intraoperative sedation and general anaesthesia using a scalp block for analgesia or regional
anaesthesia as the sole technique are the anaesthetic options for an awake craniotomy.
The term ‘awake craniotomy’ can be misleading, since varying levels of sedation or anaes-
thesia may be used during the procedure. The patient is always fully awake for the process of
‘cortical mapping’, during which lesion resection takes place, in order to minimize the risk
of damage to the eloquent cortex.
Patient selection is vital to the success of this surgery; absolute contraindications include
patient refusal, inability to lie still and lack of cooperation, e.g. acute confusion. Relative
contraindications include an inability to lie flat, having a cough, anxiety, a language barrier
and learning difficulties.
In order to anaesthestize the scalp, six nerves need to be blocked bilaterally, these are the
supratrochlear, supraorbital, zygomaticotemporal, auriculotemporal, and the lesser and
greater occipital nerves. The greater auricular nerve and third occipital nerve may need to
be blocked in some patients, making a total of eight on each side at most. To block the
greater auricular nerve, local anaesthetic is injected approximately 2 cm posterior to the
auricle at the level of the tragus.

Downloaded from https://www.cambridge.org/core. University of Edinburgh, on 19 Aug 2019 at 13:21:26, subject to the Cambridge Core terms of
use, available at https://www.cambridge.org/core/terms. https://doi.org/10.1017/9781108566100.012
Chapter 5b: Clinical Anaesthesia Answers 461

Question 115: FFTTF


Table 5.115.1 Burns depth

Nomenclature Traditional Depth Clinical features


nomenclature

Superficial First degree Epidermis Painful, blanchable


Superficial–dermal Second degree Through epidermis Painful, blisters,
to upper layers of blanchable, hair
dermis follicles present
Deep-dermal Second degree Into deep dermis ± Pain, does not
but not entire blanch, some hair
dermis follicles present
Full thickness Third/fourth degree Extends to Painless, black
subcutaneous eschar, no hair
tissue follicles

Question 116: FFTTT


The key message is to not make any assumptions about what is acceptable, and clarify
exactly what each individual will accept in the way of treatment. Some Jehovah’s Witnesses
will accept cell salvage, but not all. The AAGBI has produced guidance on good practice and
mandates that consultant surgeons and anaesthetists should be directly involved in the care
of these patients. In an emergency, anaesthetists are obligated to treat Jehovah’s Witnesses,
but this is not true for elective surgery. However, they must refer Jehovah’s Witnesses to
a colleague who is willing to anaesthetize them. AAGBI guidance makes provision for
Jehovah’s Witnesses to change their advance directive under regional anaesthesia.
Tranexamic acid may be used to decrease bleeding.

Question 117: TFFFF


AAGBI day-case guidelines state specific cut-offs for age and BMI are not appropriate in
assessing patients for day-case surgery suitability. Asthma, obesity and epilepsy may be well
suited to the principles of regional anaesthesia and short-acting techniques, as well as
minimal disruption to daily routine. Oral intake should be able to resume within a few
hours.

Question 118: TTFTF


In advanced renal disease, gastric emptying is delayed, residual volume is greater and pH
lowered. This increases the risk of aspiration and a rapid sequence induction may be
considered where a history of reflux is elucidated. Platelet count is usually normal, however
platelet function may be impaired with decreased adhesiveness and aggregation. Standard
tests of coagulation are often normal, however bleeding time may be prolonged. Atracurium
is often considered to be the muscle relaxant of choice in patients with severe renal disease
owing to its termination of action by Hoffman degradation. However its duration of action

Downloaded from https://www.cambridge.org/core. University of Edinburgh, on 19 Aug 2019 at 13:21:26, subject to the Cambridge Core terms of
use, available at https://www.cambridge.org/core/terms. https://doi.org/10.1017/9781108566100.012
462 Chapter 5b: Clinical Anaesthesia Answers

may still be altered in renal disease owing to the alteration in ion concentrations such as
magnesium, which can prolong the duration of block. So it is recommended, as for all
patients, that neuromuscular monitoring is utilized to ensure adequate reversal prior to
emergence and extubation. Patients with renal dysfunction are likely to be relatively fluid
overloaded and hypoalbuminaemic. Hypoproteinaemic states result in an increased fraction
of free drug within the plasma and thus a higher concentration of free drug with heavily
protein-bound drugs. Hyperkalaemia is a common electrolyte abnormality seen in renal
dysfunction.

Question 119: FTTFF


The evidence for a reduction in thromboembolic complications comes mainly from the
orthopaedic literature. Studies in abdominal surgery have shown only a non-significant
trend towards reduction of thromboembolic complications. Epidural anaesthetic blocks
afferent nociceptive input and reduces pain and sensitization. This reduces the incidence of
chronic postsurgical pain. The reduced stress response occurring with a regional technique
results in less postoperative immunocompromise with consequent reduced potential for the
spread of micrometastases at the time of surgery. When epidural analgesia is used intra-
operatively in abdominal surgeries, it reduces the depth of anaesthesia required and hence
would have a beneficial effect in reducing postoperative cognitive dysfunction. The debate
on whether intrathecal blocks are safer when performed on awake or anaesthetized patients
in unresolved. Available evidence is low-level and conflicting.

Question 120: FTFFF


Cardioplegia can be either blood- or crystalloid-based and typically has 20 mmol.l–1
potassium, 16 mmol.l–1 magnesium and procaine. Rapid infusion of approximately 1 l
renders the heart asystolic. Cold cardioplegia at 4 °C provides further myocardial protection
again ischaemia. Cardioplegia is instilled into the ascending aorta (as long as the aortic valve
is competent) with a cross clamp on the aorta distal to the cardioplegia cannula, and runs
into the coronaries. If the aortic valve is incompetent, the cardioplegia can be instilled
directly into the coronaries or can be retrograde instilled via the coronary sinus. The aim is
for the cold, hyperkalaemic solution to cause depolarization and arrest in diastole,
which is a relaxed state and, therefore, at a cellular level, metabolically less active.
On cardiopulmonary bypass, the blood is typically pumped through roller pumps which
provide non-pulsatile flow. Pulsatile flow may be more physiological, but is more expensive
to produce and has shown little benefit (ECMO circuits typically use centrifugal pumps).
As blood comes in contact with a large prothrombotic surface in the bypass circuit, it is
essential that the patient is fully heparinized, aiming for an ACT >400 s, prior to going on to
cardiopulmonary bypass.

Question 121: FTTFT


Enteral feeding is more physiological and hence should be considered before parenteral
feeding. Whilst it is almost impossible to overfeed using the enteral route, overfeeding is
more common with the parenteral route. Enteral feed protects the gut against bacterial
translocation and hence may reduce the risk of hospital-acquired infection. Postpyloric
feeding should only be considered if gastric absorption is problematic. Although early

Downloaded from https://www.cambridge.org/core. University of Edinburgh, on 19 Aug 2019 at 13:21:26, subject to the Cambridge Core terms of
use, available at https://www.cambridge.org/core/terms. https://doi.org/10.1017/9781108566100.012
Chapter 5b: Clinical Anaesthesia Answers 463

feeding is advocated for its benefits, it is certainly better to underfeed than overfeed. It is not
advisable to attempt to match the calculated caloric requirement. Aiming for a high caloric
intake during the acute phase of critical illness may be associated with a less favourable
outcome.

Question 122: TTFFF


Diabetes is increasing in incidence in the general population and there is also
a corresponding increase in gestational diabetes. Risk factors for gestational
diabetes include BMI over 30 (therefore routinely screened for), first-degree relative
with diabetes or certain ethnic subgroups (South Asian, Black Caribbean). With pre-
existing diabetes, management should ideally commence preconception to ensure the
risks of pregnancy-associated diabetes are reduced. Risks include increases in miscarriage,
pre-eclampsia, stillbirth and birth trauma. Whilst the manufacturers of oral hypoglycae-
mic agents suggest avoiding their use in pregnancy for safety purposes, NICE guidelines
recommend they are used and believe them to have an appropriate safety record (Clinical
Guideline 63). During labour, blood glucose should be maintained between 4 and 7
mmol.l–1. Where this is not possible or if the patient is a Type I diabetic, insulin and
dextrose should be used. Whilst there is an increase in larger babies and an associated risk
of birth trauma and obstetric interventions being required, epidurals are not specifically
recommended to women with diabetes.

Question 123: FFTFF


Hypoxic pulmonary vasoconstriction (HPV) is a reflex contraction of vascular smooth
muscle in the pulmonary circulation in response to low regional partial pressure of oxygen
(paO2). Animal studies show that at low doses almitrine enhances HPV by a vasoconstrictor
effect specific to pulmonary arteries. Iron attenuates HPV and also greatly reduces the
enhanced response normally seen after prolonged hypoxic exposure. The common intra-
venous anesthetic agents show no inhibition of HPV.

Question 124: TFFFF


An advance directive made in the presence of a witness is a legally binding document.
The patient must be over 18 years of age and have capacity at the time of writing. Patients are
encouraged to carry a copy of their advance directive, as well as keeping copies with their
general practitioner, family and friends. The directive must specify the decision applied to
a specific treatment if the individual’s life is at risk and if any of these conditions are not met,
the directive may be invalid. A doctor who knowingly breaches the terms of the directive
may face criminal charges and referral to the General Medical Council. In an emergency
where the patient cannot express their own wishes, an advance directive should be obtained
as soon as possible. However, where such a directive cannot be obtained, life-saving
treatment should not be withheld. The views of family and friends may be sought where
possible, but no individual can refuse treatment on behalf of a patient. Involvement of the
hospital legal team and Jehovah’s Witness liaison, and documentation of all discussions and
decisions are paramount in such situations.

Downloaded from https://www.cambridge.org/core. University of Edinburgh, on 19 Aug 2019 at 13:21:26, subject to the Cambridge Core terms of
use, available at https://www.cambridge.org/core/terms. https://doi.org/10.1017/9781108566100.012
464 Chapter 5b: Clinical Anaesthesia Answers

Question 125: FTTFT


Cleft lip and palate (CLP) is one of the commonest congenital deformities. Two-thirds
involve the lip with or without the palate and the remainder the palate alone. Cleft lip is
unilateral in 80% of cases and occurs on the left in over 70% of cases. CLP is more common
in males. Chronic rhinorrhoea is common due to food reflux into the nasal passages and
may present with recurrent URTI. Snoring, apnoea during feeds or protacted feeding time
may indicate chronic airway obstruction. Difficult mask ventilation is unusual. Difficult
laryngoscopy and intubation are strongly associated with retrognathia and bilateral cleft lip,
due to protruding maxilla. Nasopharyngeal airways are effective and well tolerated and
should be inserted in patients with high risk of postoperative airway complications before
emergence. Avoid an oropharyngeal airway due to risk of disrupting the surgical repair.

Question 126: TTFFT


Suxamethonium apnoea occurs due to abnormal levels of functional plasma cholinesterase,
coded for on chromosome 3. Plasma cholinesterase may be present, but not functional.
In E1f:E1f there is reduced or no activity of plasma cholinesterase. In 1957 Kalow and
Genest described mixing plasma with benzoylcholine and measuring the light emitted. They
established that normal plasma and benzoylcholine emitted light at a specific wavelength.
Dibucaine inhibits this reaction, and the percentage inhibition by a set amount of dibucaine
compared to a reference value is known as the dibucaine number. Normal is 80 and reduced
values reflect lack of functional plasma cholinesterase in plasma samples. Low dibucaine
number is therefore a reflection of plasma cholinesterase abnormality. The risks of transfus-
ing blood products outweigh the benefits. Given time, the neuromuscular block will reverse.
Fresh frozen plasma should only be given in emergency situations following consultation
with a consultant haematologist. Methotrexate is a cause of acquired plasma cholinesterase
deficiency.

Question 127: FTFTT


Perioperative hypothermia is split into three portions:
1. Preoperative (up to an hour prior to arriving in theatre suite)
2. Intraoperative
3. Postoperative (up to 24 hours after entry to recovery area)
Core temperature should be measured preoperatively and warming devices should be
employed if temperature falls below 36 °C. NICE guidelines on perioperative hypothermia
suggest groups at high risk of inadvertent hypothermia include patients with:
• ASA grade ≥II
• Preoperative temperature <36 °C
• Combined general and regional anaesthesia
• Undergoing major or intermediate surgery
• At risk of cardiovascular complications

Downloaded from https://www.cambridge.org/core. University of Edinburgh, on 19 Aug 2019 at 13:21:26, subject to the Cambridge Core terms of
use, available at https://www.cambridge.org/core/terms. https://doi.org/10.1017/9781108566100.012
Chapter 5b: Clinical Anaesthesia Answers 465

Question 128: FFTFF


The trachea divides into the left and right main bronchi at the level of approximately T5.
The right main bronchus is wider, shorter (approximately 2.5 cm) and more vertically
angled when compared with the left. The right main bronchus gives off the right upper lobe
bronchus (which divides into the apical, anterior and posterior segments), the right middle
lobe bronchus (which divides into the lateral and medial lobes), and the bronchus of the
apical segment of the lower lobe (which divides into superior, anterior basal and lateral
basal). The lower lobe bronchus continues downwards giving off the medial, anterior, lateral
and posterior basal segments. The left main bronchus is longer (approximately 5 cm),
narrower and more obliquely angled than the right. The left main bronchus gives off the left
upper lobe bronchus, this divides into the superior division (which divides into the apical,
posterior and anterior segments) and the lingular bronchus (divides into superior lingular
and inferior lingular segments). The left lower lobe bronchus gives off the apical, anterior
basal, medial basal (although can arise with the anterior basal), lateral basal and posterior
basal.

Question 129: FTFFT


This neonate’s most likely diagnosis is tetralogy of Fallot. This is due to the presence of
cyanosis (secondary to an over-riding aorta), a murmur over the pulmonary area (suggest-
ing a right ventricular outflow tract obstruction) and evidence of right ventricular hyper-
trophy on the ECG. The degree of the right ventricular outflow obstruction (RVOT) will
dictate the speed of onset of his cyanosis. Within 72 hours 90% of all PDAs will close, leading
to significant cyanosis in cases of severe RVOT obstruction. Treatment should aim to
improve pulmonary blood flow. This could be achieved medically by prostaglandin infusion
until a more definitive shunt is established surgically. In the modified Blalock–Taussig
operation, an anastomosis is established between the subclavian and the ipsilateral pul-
monary artery.

Question 130: TFTTF


The following events are applicable for all incidents that occur on or after 1 April 2015 when
providing NHS care:
• Wrong site surgery
• Wrong implant/prosthesis
• Retained foreign object post procedure
• Mis-selection of a strong potassium-containing solution
• Wrong route administration of medication
• Overdose of insulin due to abbreviations or incorrect device
• Overdose of methotrexate for non-cancer treatment
• Mis-selection of high-strength midazolam during conscious sedation
• Falls from poorly restricted windows
• Chest or neck entrapment in bedrails
• Transfusion or transplantation of ABO-incompatible blood components or organs
• Misplaced naso- or orogastric tubes
• Scalding of patients

Downloaded from https://www.cambridge.org/core. University of Edinburgh, on 19 Aug 2019 at 13:21:26, subject to the Cambridge Core terms of
use, available at https://www.cambridge.org/core/terms. https://doi.org/10.1017/9781108566100.012
466 Chapter 5b: Clinical Anaesthesia Answers

Wrong site surgery includes a surgical intervention performed on the wrong patient or the
wrong site. It includes wrong level spinal surgery and wrong site block unless being under-
taken as a pain control procedure. Retention of a foreign body post procedure does not
include items inserted before the procedure that are not subject to the formal counting/
checking process, such as a throat pack inserted in the anaesthetic room. Misplacement of
naso- or orogastric tube itself is not a never event; however, failing to recognize misplace-
ment and administering feed down a misplaced tube is.

Question 131: TFFTT


Radiofrequency (RF) procedures use high-frequency alternating current to alter pain path-
ways. They are used to help in the management of a number of chronic pain states. There are
two more commonly used types:
• Continuous RF (CRF) – this produces a thermal lesion in a target nerve
• Pulsed RF (PRF) – short bursts of RF that do not produce significant heat interrupt pain
signal transmission in a mechanism that is not fully understood.
In CRF, passing an alternating current produces molecular oscillation causing a heating effect
that can ablate nerves. Temperatures of 80–90 °C are reached. Current is concentrated at the
needle tip, causing very localized heating. Pulsed RF does not produce heat and may be less
effective, but is also thought to be associated with fewer potential side effects. Continuous RF
does not differentiate between sensory and motor nerves and adverse effects may occur if
care is not taken. Thermal lesions are not permanent, as coagulated nerves will regenerate.

Question 132: FFFTF


A large number of drugs with varied mechanisms of action are used in the treatment of
Parkinsons’ disease (PD). The stage and severity of the disease, its progression, the patient’s
side-effect tolerance and ease of compliance influence the choice of drugs. In the early stages
of the disease there may be no medication required, but dopamine agonists, e.g. ropinorole
or apomorphine, or alternatively monoamine oxidase B (MAO-B) inhibitors, e.g. selegeline,
are often the first to be introduced. Amantadine is another first-line drug; its mechanism of
action remains uncertain and it is hence classified as an atypical agent. Levodopa is
a commonly used agent in established PD; it is a precursor of dopamine, but unlike
dopamine is able to cross the blood–brain barrier. Levodopa is often administered with
dopamine decarboxylase inhibitors, e.g. benseraside, to minimize the peripheral side effects
of dopamine, e.g. tachycardia, nausea and vomiting. With disease progression and sustained
use, levodopa has less effect, producing ‘off periods’, and side effects may become more
pronounced producing peak dose dyskinesias. Other drugs may have a role in minimizing
off periods, including MAO-B inhibitors, dopamine agonists or catecholamine-O-methyl-
transferase inhibitors that prevent dopamine breakdown, e.g. entacopone.

Question 133: TFFTT


The Parkland formula arose as a result of regression analysis studies of resuscitative volumes
in adult burns patients at the Parkland Hospital in Dallas, Texas. The formula is 4 ml.kg–1
body weight × %TBSA burn and describes the volume of Ringer’s lactate to be given in the
first 24 hours. Half is given in the first eight hours (from the time of injury, not admission)

Downloaded from https://www.cambridge.org/core. University of Edinburgh, on 19 Aug 2019 at 13:21:26, subject to the Cambridge Core terms of
use, available at https://www.cambridge.org/core/terms. https://doi.org/10.1017/9781108566100.012
Chapter 5b: Clinical Anaesthesia Answers 467

and the other half in the remaining 16 hours. Hartmann’s solution is generally used in the
UK in preference to 0.9% saline to avoid the hyperchloraemic metabolic acidosis that can
occur after the administration of large volumes of saline.
In case c. the initial rate of crystalloid resuscitation would be:
4 × 80 × 40 = 12 800 ml to be given in 24 hours
Therefore 6400 ml to be given in eight hours
As the burn occurred 1 hour ago the 6400 ml should now be given in the remaining seven
hours
6400 ÷ 7 = 914.3 rounded up to 915 ml.h–1.
It is important to remember that the formula acts only as an initial guide and trends in
physical and biochemical parameters such as heart rate, blood pressure, urine output,
electrolytes and haematocrit will be required to tailor management of the individual patient.

Question 134: TFTFT


Normal intra-abdominal pressure should be a reflection of intrathoracic pressure – in health
it should be zero or slightly subatmospheric, though it may rise slightly during IPPV.
Abdominal compartment syndrome is defined as an intra-abdominal pressure greater
than 25 mmHg and should be decompressed. If it is left untreated, mortality approaches
100%, compared to 20% if decompressed early or 40–60% with late decompression.
Abdominal compartment syndrome causes basal atelectasis and a decrease in cardiac out-
put, both of which will increase the shunt fraction. Patients with suspected abdominal
compartment syndrome should be aggressively volume-loaded to maintain organ perfusion
pending urgent surgical decompression.

Question 135: TFTFF


Back and limb pain is common in Guillain–Barré syndrome (GBS) and may be difficult to
treat. Pupillary dilatation is not a feature of GBS, although cardiovascular autonomic
dysfunction is seen is 70%. Cranial nerves may be involved in 15%, but this often follows
limb weakness; early cranial nerve involvement should prompt consideration of
Miller–Fischer variant or botulism. Ventilation is required in 10–30%.

Question 136: TTTTT


Diabetes can lead to glycosylation of soft tissues, resulting in reduced head and neck
movement and hence difficulty in intubation. The prayer sign is used as a predictor: the
inability to place both palms together with wrists at 90° extended is predictive of difficult
intubation. Diabetes predisposes to neuropathy. Peripheral neuropathy is of importance to
the anaesthetist as the patient may be at an increased risk of developing further nerve
damage perioperatively and care should be taken to avoid this. Autonomic dysfunction may
result in delayed gastric emptying, lack of physiological tachycardia and impaired heat
regulation. Other complications of diabetes include ischaemic heart disease, renal disease,
peripheral vascular disease and diabetic eye disease.

Downloaded from https://www.cambridge.org/core. University of Edinburgh, on 19 Aug 2019 at 13:21:26, subject to the Cambridge Core terms of
use, available at https://www.cambridge.org/core/terms. https://doi.org/10.1017/9781108566100.012
468 Chapter 5b: Clinical Anaesthesia Answers

Question 137: TFFFT


Benzodiazepines (BZDs) remain the first line of treatment in acute seizures. There is no
evidence that one form of BZD is superior to another. Phenytoin is usually the second line of
treatment, but its administration should not be delayed in order to check levels if the patient
is in status epilepticus. Patients who fail to respond to both BZD and phenytoin usually
require general anaesthesia. There is no evidence to show that thiopentone is better than
propofol or vice versa. Propofol is usually preferred due to its short context-sensitive half-
life; however, propofol infusion syndrome is a potential problem if the infusion continues
beyond 48 hours. If the patient is paralyzed, continuous EEG is mandatory to exclude
seizure activity.

Question 138: TFTTT


With the exception of a raised temperature, these are all absolute contraindications for
epidural insertion. An epidural can usually be inserted in a patient with a raised tempera-
ture, provided that there are no other abnormal markers of infection and the woman is
clinically well. If there is any suspicion of systemic sepsis then an epidural should not be
performed.

Question 139: TFFTF


Duchenne muscular dystrophy follows an X-linked recessive pattern of inheritance, with an
incidence of 1:3500 live male births. Pathophysiologically the abnormality is an absent or
abnormal dystrophin protein, which results in a weak sarcolemma. Chronic muscle fibre
necrosis, degeneration and regeneration follow. Degeneration affects cardiac, smooth and
skeletal muscle. Presentation is usually between the ages of three and five years with
a waddling gait and calf muscle pseudohypertrophy. Progressive wasting and weakness
affects the proximal, rather than distal, muscles. Cardiac or respiratory complications
usually result in death in the second or third decades of life.

Question 140: TTFFF


Pierre Robin anomaly is congenital malformation of craniofacial development and the
sequence describes a triad of micrognathia, glossoptosis and cleft palate. Congenital
anomalies of the heart (VSD, ASD, PDA, coarctation of aorta and choanal atresia) have
been reported. Airway obstruction is primarily due to a posterior-situated tongue, which
significantly reduces the size of the posterior pharyngeal airway. Upper airway obstruction
improves and intubation becomes easier with age, due to mandible growth. Surgical airway
or tracheostomy is rarely needed and can prove difficult. Obstruction often improves by
placing the child prone or lateral. Once the child is fully awake, IV or NCA opioids can be
administered. Close monitoring postoperatively for 12 hours for detection of airway
obstruction or bleeding is required.

Question 141: FFFTF


Incidence data for anaesthetic agents come predominantly from a French epidemiological
survey from the mid-1990s. National incident reporting in the UK attempts to capture data

Downloaded from https://www.cambridge.org/core. University of Edinburgh, on 19 Aug 2019 at 13:21:26, subject to the Cambridge Core terms of
use, available at https://www.cambridge.org/core/terms. https://doi.org/10.1017/9781108566100.012
Chapter 5b: Clinical Anaesthesia Answers 469

on anaphylaxis, but this may be incomplete. Suxamethonium causes over 40% of reactions
caused by neuromuscular blockers. However, this figure may change with increasing
availability of sugammadex, resulting in increased usage of rocuronium (incidentally
there have now been case reports of anaphylaxis to sugammadex). Etomidate causes the
fewest anaphylactic reactions of all induction agents, thiopentone the most. Colloids cause
just fewer than 5% of all reactions under anaesthesia. Morphine is the opiate most com-
monly attributed to anaphylactic reactions. Synthetic opioids cause relatively fewer reac-
tions than morphine.

Question 142: FTFTT


One definition of massive transfusion is the replacement of a patient’s total blood volume
within 24 hours. Complications of massive transfusion include:
• Dilutional coagulopathy
• Hypocalcaemia – due to binding with citrate
• Hyperkalaemia – potassium concentration in stored blood increases during storage
• Hypothermia – due to storage of blood at 4 °C
• Alkalosis – due to metabolism of citrate to bicarbonate
An acidosis is potentially possible, but is related to poor tissue perfusion not directly due to
large transfusion of blood products.

Question 143: TTTTT


TURP syndrome classically occurs due to excessive absorption of irrigation fluid through
open prostatic vessels during a TURP procedure, although it can be seen with other
procedures that involve irrigation with electrolyte-free solutions. The clinical features
observed are mainly due to intravascular hypervolaemia and acute electrolyte disturbances,
including dilutional hyponatraemia, but additional features can occur due to glycine and its
metabolites. The severity depends on the volume of fluid absorbed, in addition to the
underlying condition of the patient. The clinical features that are observed in TURP
syndrome are shown in Table 5.143.1.

Table 5.143.1

System Clinical features

CVS Hypertension
Hypotension
Bradycardia
Tachycardia
Dysrhythmias
Angina
Shock
CNS Confusion
Agitation
Seizures
Blindness
Coma

Downloaded from https://www.cambridge.org/core. University of Edinburgh, on 19 Aug 2019 at 13:21:26, subject to the Cambridge Core terms of
use, available at https://www.cambridge.org/core/terms. https://doi.org/10.1017/9781108566100.012
470 Chapter 5b: Clinical Anaesthesia Answers

Table 5.143.1 (cont.)

System Clinical features


Respiratory Pulmonary oedema
Respiratory distress and failure
Electrolyte Hyperglycaemia
Hyponatraemia
Hyperammonaemia
Hypo-osmolality
Haematological Haemolysis
Anaemia
Renal Acute renal failure

Question 144: FFTTT


Hirschsprung’s disease can be associated with neurological, cardiovascular, urological and
gastrointestinal abnormalities. Trisomy 21 is seen in 10%, other associated anomalies
include Waardenburg syndrome, congenital hypoventilation syndrome ‘Ondine’s curse’,
congenital deafness, MEN type 2A and neuroblastoma.
Perioperative risks include:
• Intestinal obstruction leading to regurgitation and aspiration
• Vomiting and diarrhoea may result in electrolyte and fluid imbalance
• Enterocolitis might happen, leading to dehydration and septic shock
• Intestinal ischaemia, perforation and peritonitis leading to increase in third-space losses
Emergency operative treatment is only required if the child is in septic shock secondary to
bowel perforation or enterocolitis.

Question 145: TFTFT


Measurements include work rate in watts. Cardiac output and arterial-mixed oxygen
differences increase linearly with VO2 in most patients until a peak oxygen extraction
ratio of 75% is reached. During exercise, local mechanisms increase the oxygen extraction
ratio in most patients to 75%, but after an operation the oxygen extraction ratio is only 30%.
Therefore a relative 2.5 times increase in cardiac output is required postoperatively to
achieve the same oxygen delivery seen during exercise. The nine-panel plot graphs are
numbered 1–9 from top left to bottom right. Graphs 1,4,7 examine ventilation, graphs 2,3,5
relate to the cardiovascular system and graphs 6,8,9 demonstrate the ventilation–perfusion
relationships.

Question 146: TTFTF


The transversus abdominis plane (TAP) block is an effective and safe adjunct in the multi-
modal approach to postoperative pain relief for abdominal surgery.

Downloaded from https://www.cambridge.org/core. University of Edinburgh, on 19 Aug 2019 at 13:21:26, subject to the Cambridge Core terms of
use, available at https://www.cambridge.org/core/terms. https://doi.org/10.1017/9781108566100.012
Chapter 5b: Clinical Anaesthesia Answers 471

Innervation of the anterolateral abdominal wall does indeed arise from the anterior rami
of spinal nerves T7 to L1. These include:
• The intercostal nerves (T7–T11)
• The subcostal nerve (T12)
• The iliohypogastric and ilioinguinal nerves
The TAP is the fascial layer between the internal oblique and the transversus abdominis
muscles. The anterior rami described above pass through this plane. The anatomical land-
mark-based approach to the TAP block requires insertion of the needle within the lumbar
triangle of Petit. This is situated between the lower costal margin and iliac crest and is bound
anteriorly by the external oblique muscle and posteriorly by the latissimus dorsi. As a blunt
needle passes though the fascial extensions of the external and internal oblique muscles two
‘pops’ are felt and a large volume of local anaesthetic deposited. The ultrasound approach
has become more common, allowing for more accurate deposition of local anaesthetic.
Complications of the TAP block are rare. Case reports of hepatic injury caused by right-
sided TAP blocks exist. Injury to other viscera is possible. A femoral nerve block is possible
as the transversalis fascial plane is continuous with the fascia iliaca. From here spread to the
femoral nerve can occur.

Question 147: TTFFF


Considerations in providing anaesthesia for magnetic resonance imaging can broadly be
classified as those that relate to the patient population, those relating to the environment
(often remote site) and those pertaining to monitoring and equipment, some aspects of
which are covered here. Equipment is classified as being MR safe, conditional or unsafe.
Where it is marked as MR conditional this indicates that it is safe to use in a specified
environment/under specific conditions of use and if sourced locally it is highly likely to be
suitable for use. The ECG is particularly prone to interference due to induction currents in
the leads that can cause spikes in the waveform, T wave and ST segment abnormalities –
making arrhythmias difficult to detect despite using an appropriate monitor. Specific ECG
electrodes for use with MRI are required and leads that minimize induction currents are
short and of high impedance. MRI-compatible pulse oximiters (and ECG leads) utilize fibre-
optic cables (not electrical cables); there have been several reports of severe burns inflicted
by induction currents due to the use of standard pulse oximiter probes. A long sampling line
for capnography can result in a waveform delayed for up to 20 seconds.

Question 148: TTTFT


Anaesthetists are regularly called upon to assess patients who have suffered burns to the
head and face. The clinical decision as to whether or not they require endotracheal intuba-
tion to protect an at-risk airway is not always straightforward. There are, however, specific
things one should look for in the history, examination and investigations that can aid this
evaluation:
Suspect airway burn if:
• Burns occurred in an enclosed space
• There is stridor, hoarseness or cough
• There are burns to face, lips, mouth, pharynx or nasal mucosa
• There is soot in sputum, nose or mouth

Downloaded from https://www.cambridge.org/core. University of Edinburgh, on 19 Aug 2019 at 13:21:26, subject to the Cambridge Core terms of
use, available at https://www.cambridge.org/core/terms. https://doi.org/10.1017/9781108566100.012
472 Chapter 5b: Clinical Anaesthesia Answers

• There is dyspnoea, decreased level of consciousness or confusion


• There is hypoxaemia (low pulse oximetry saturation or arterial oxygen tension) or
increased carbon monoxide levels (>2%)

Question 149: FTTFT


Threatened miscarriage is defined as vaginal bleeding at less than 24 weeks’ gestation, and in
some cases the patient may not know they are pregnant. Presentation with abdominal sepsis
secondary to infected retained products of conception is a rare but recognized entity. Serum
β-hCG levels will halve every two days if there is loss of the trophoblast (i.e. a complete
abortion). Only rhesus-negative women will require anti-D therapy, as rhesus-positive
women will not mount a response to Rh antigen if there is mixing. Uterine perforation
may occur as a complication of surgical management of miscarriage.

Question 150: FFFFF


Cardiac dysrhythmias associated with hypothermia often revert spontaneously with
rewarming. Although judicious use of fluids is important, as patients rewarm, vasodilata-
tion occurs and so fluid requirements are often large. Adrenaline and other drugs should be
withheld in the arrested patient until the core temperature is 30 °C, adrenaline should then
be given at double the time interval (i.e. 6–10 minutes) until the patient’s temperature is
above 35 °C. A pulmonary artery flotation catheter would be the gold standard for core
temperature monitoring, but is rarely a priority in resuscitation. Only 0.5% of drowning
victims have C-spine injury.

Question 151: FTFTT


Anticoagulation is an important consideration for cardioversion of arrhythmias, espe-
cially atrial fibrillation. The risk of atrial thrombus formation increases the longer AF
has been present. As such, if AF has been present for more than 48 hours the patient
must be either fully anticoagulated, or the presence of thrombus ruled out on
echocardiogram.
Following three attempts at cardioversion the risk of damaging the myocardium out-
weighs the chances of a successful cardioversion and consideration of an alternative treat-
ment should be sought. When the patient is compromised it is appropriate to make all
attempts to determine the underlying rhythm. The administration of drugs such as adeno-
sine can assist in slowing the heart rate to aid this process. Attempting to cardiovert a sinus
tachycardia is dangerous. Electrolytes, particularly potassium and magnesium, should be in
the normal range prior to elective cardioversion. Potassium should ideally be greater than
4 mmol.l–1. Failure to correct electrolytes increases the risk of failure to successfully
cardiovert the patient. In the emergency situation, every effort should be made to rapidly
correct such problems; this is likely to be confounding the underlying arrhythmia.
Cardioversion is most likely to be successful when the pads are in the anteroposterior
position, however in an emergency, anterolateral is acceptable and cardioversion should not
be delayed for pad replacement.

Downloaded from https://www.cambridge.org/core. University of Edinburgh, on 19 Aug 2019 at 13:21:26, subject to the Cambridge Core terms of
use, available at https://www.cambridge.org/core/terms. https://doi.org/10.1017/9781108566100.012
Chapter 5b: Clinical Anaesthesia Answers 473

Question 152: TFTTF


High spinal cord injury may result in sympathetic denervation leading to a drop in SVR and
CO. Unopposed vagal stimulation will cause bradycardia that should respond to atropine.
Initial management should focus on fluid resuscitation and early use of vasoconstrictors.
Phenylephrine should be avoided as it has an unopposed agonistic effect on α1-receptors
resulting in a reflex bradycardia.

Question 153: FFFFT


Whilst the use of propofol is increasing in frequency as an induction agent for caesarean
section (CS), it is neither the most common nor the recommended choice. Thiopentone
remains the most widely used agent, usually given with suxamethonium for muscle relaxa-
tion. There has been an increasing use of rocuronium for muscle relaxation since the
introduction of sugammadex as a rapid reversal agent, but this has been restricted by
a lack of availability, additional cost and unfamiliarity with its use. No muscle relaxants,
be they depolarizing or non-depolarizing, cross the placenta in meaningful quantities.
Opiates are best avoided until the fetus is delivered due to the risk of respiratory depression.
Short-acting opiates, however, are frequently given on induction, in cases where the risk of
increasing maternal hypertension on laryngoscopy is high. Skin incision should occur once
the airway has been secured and appropriate anaesthesia delivered. Cricoid pressure is
difficult to perform due to the positioning of the patient with lateral tilt and can make the
laryngoscopic view worse. Thirty newtons is an appropriate force to apply to an anaesthe-
tized patient.

Question 154: FTFTT


Pneumonectomy has a higher incidence of mortality (8–10% for pneumonectomy versus
2% for lobectomy). Postoperative respiratory complications are associated with higher
mortality, hence tight perioperative fluid management is essential. Right-sided pneumo-
nectomy carries more risk than a left-sided pneumonectomy. Cardiac herniation into the
postpneumonectomy space can cause haemodynamic instability and needs urgent manage-
ment. Extrapleural pneumonectomy is currently typically performed for local control of
malignant pleural mesothelioma.

Question 155: TFFTT


Sickle cell disease is an autosomal recessive haemoglobinopathy, which can lead to pro-
gressive vaso-occlusive damage to the spleen. This functional state of asplenia makes
patients more prone to infection. Recommended vaccinations for adults with sickle cell
disease include:
• Seasonal influenza
• Pneumococcal
• Haemophilus influenzae B
• Meningococcal C
• Hepatitis B

Downloaded from https://www.cambridge.org/core. University of Edinburgh, on 19 Aug 2019 at 13:21:26, subject to the Cambridge Core terms of
use, available at https://www.cambridge.org/core/terms. https://doi.org/10.1017/9781108566100.012
474 Chapter 5b: Clinical Anaesthesia Answers

Question 156: TTTFT


There are a number of differentials for a noisy airway in a child, some of which are benign
such as a croup. Epiglottitis is a life-threatening emergency caused by bacterial infection of
the epiglottis, aryepiglottis and arytenoids. In children, it is almost always due to infection
with Haemophilus influenzae type B (Hib), although it can be caused by β-haemolytic
streptococci, staphylococci and pneumococci. Epiglottitis is distinguished from croup by
its fulminant abrupt onset and the toxic appearance of its victims. It occurs usually in
children aged two to six years, with a peak incidence at three years. Clinically, there is an
abrupt onset of high fever, sore throat, dysphagia, stridor and drooling. Typically, the child
looks ill and may prefer to sit leaning forwards, mouth open and with tongue and jaw
protruding in order to open the airway, described as the ‘tripod position’.

Question 157: TFFFF


Immediate management involves early recognition, calling for senior help and securing the
airway with an endotracheal tube and ventilating with 100% O2. As the sequelae of
anaphylaxis may include swelling of the airway, early intubation is a prudent measure.
Ideally it would be best to avoid the administration of neuromuscular blockers as they carry
an anaphylaxis risk, however many would use them to rapidly secure the airway.
The AAGBI and ALS guidelines recommend IV adrenaline in 50 μg boluses.
As anaesthetists we should be familiar with this drug and competent to administer it
intravenously in a fully monitored environment. If, however, you are new to anaesthesia
or have limited experience in administrating IV adrenaline, then administer adrenaline
0.5 ml 1:1000 intramuscularly. As colloids can precipitate anaphylaxis, crystalloids should
be the resuscitation fluid of choice. Hydrocortisone, chlorpheniramine and salbutamol are
all considered secondary measures and are not part of the immediate management.

Question 158: TTFTF


Transfusion related acute lung injury (TRALI) typically develops six hours post transfusion.
Non-cardiogenic pulmonary oedema results in dyspnoea, hypoxaemia, tachycardia, fever
and hypotension. Radiographically, appearances are of bilateral pulmonary infiltrates
characteristic of pulmonary oedema. Myalgia and headache are classically features of non-
haemolytic febrile reactions.

Question 159: TTTFT


A ‘spray-as-you-go’ technique allows topical local anaesthetic to be applied under direct
vision to the airway (either via the side-port of the scope, or through an epidural catheter
threaded out of the end of the scope) as it is visualized with the fibrescope. Studies
comparing the use of 2% and 4% lidocaine for this technique have found that both produced
adequate intubating conditions for awake-fibreoptic intubation. Nebulized lidocaine 2–4%
for 15–30 minutes can also produce effective anaesthesia of the oral cavity and trachea,
although the density of anaesthesia produced can be quite variable, and some patients will
maintain a cough reflex. The superior laryngeal nerve (SLN) supplies sensory innervation to
the base of the tongue, the posterior surface of the epiglottis, the aryepiglottic folds and the
arytenoids. Blockade of the SLN alone is not normally adequate as a solo technique to allow

Downloaded from https://www.cambridge.org/core. University of Edinburgh, on 19 Aug 2019 at 13:21:26, subject to the Cambridge Core terms of
use, available at https://www.cambridge.org/core/terms. https://doi.org/10.1017/9781108566100.012
Chapter 5b: Clinical Anaesthesia Answers 475

awake intubation, but is very effective when used in combination with topical anaesthetic
techniques. A translaryngeal block, which involves a cricoid puncture through which 2–3 ml
of 2–4% lidocaine is injected, provides a further technique for effectively anaesthetizing the
larynx.

Question 160: TFFFT


Local anaesthetics (LAs) are basic drugs so they are more unionized in a local pH above their
pKa. Hence an alkaline medium would favour their entrance into the cells. Bicarbonate has
been used to improve LA drug absorption. Adrenaline causes vasoconstriction and hence
decreases the washout of the drug from the site injected. Clonidine is an α2-agonist that has
an inhibitory role. This will potentiate the effect of the local anaesthetics to block the nerve
conduction. Local infection creates an acidic medium that will render the local anaesthetic
more ionized and decrease its capability to cross the lipid layer of the nerve tissue.

Question 161: TTTFT


All except malaria are the minimum recommended tests by the World Health Organization
for blood donation testing. Many countries conduct other tests based on local needs such as
anti-T cell lymphotrophic virus, cytomegalovirus, West Nile virus and Chagas disease.

Question 162: FTFFF


Spinal cord stimulation (SCS) leads are placed in the dorsal epidural space and are
connected to a programmable pulse generator that is implanted subcutaneously.
The leads activate the large diameter afferents of the dorsal columns that inhibit nociceptive
small-diameter afferent transmission rostrally. Pain relief may continue to be improved for
a period of time after the cessation of electrical stimulation, which suggests long-lasting
neuromodulation of neurotransmission, for example involving adenosine and GABA
release.
Indications for SCS:
• Failed back surgery syndrome causing limb pain
• Complex regional pain syndrome
• Neuropathic pain secondary to peripheral nerve damage
• Ischaemic pain
• Refractory angina
• Brachial plexopathy
Failed back surgery syndrome causing axial spine pain is less likely to respond.
Contraindications for SCS:
• Psychological unsuitability, e.g. significant depression
• Bleeding disorder
• Sepsis
• Presence of a demand pacemaker or implantable defibrillator (relative contraindication)
• Immunosuppression
Care should be taken when performing neuraxial blocks in these patients. Lead damage and
infection are possible and the use of fluoroscopic guidance should be considered.

Downloaded from https://www.cambridge.org/core. University of Edinburgh, on 19 Aug 2019 at 13:21:26, subject to the Cambridge Core terms of
use, available at https://www.cambridge.org/core/terms. https://doi.org/10.1017/9781108566100.012
476 Chapter 5b: Clinical Anaesthesia Answers

Question 163: TTTTT


Venous air emboli (VAEs) are commonly associated with neurosurgery, particularly in the
seated position, but not exclusively so. In essence, any procedure performed at a level above
the heart with the potential for air entrainment into veins can result in a VAE and so it is
important for the anaesthetist to maintain a high index of suspicion. Classically, profound
cardiovascular collapse leads swiftly to cardiac arrest and fatality; the presentation of VAE
can, however, be much more subtle or even clinically undetectable. The presenting features
are largely dependent on the amount and rate of air entrainment. The auscultation of
a ‘millwheel’ murmur using a precordial or oesophageal stethoscope is rarely useful, as it
signifies imminent cardiac arrest; nonetheless this sign is pathognomonic of VAE. Though
clearly not routinely used, the most sensitive monitor for VAE is transoesophageal echo-
cardiography (volumes of 0.02 ml.kg–1), ten times more sensitive than the next most
sensitive, precordial Doppler (0.2 ml.kg–1 volumes). Other useful methods for detection
of VAE include monitoring of end tidal carbon dioxide concentration, the use of
a pulmonary artery catheter, transcranial Doppler and monitoring central venous pressure.

Question 164: TTFTF


The North American Symptomatic Carotid Endarterectomy Trial (NASCET) and the
European Carotid Surgery Trial (ECST) established that carotid endarterectomy is highly
beneficial for patients with a high degree of carotid artery stenosis. The exact method in
which the degree of stenosis was measured differed between trials. In the ECST the
estimated normal lumen diameter at the site of the lesion, based on a visual impression of
where the normal arterial wall was before development of the stenosis, was used. In the
NASCET the diameter of a visible portion of disease-free ICA distal to the stenosis was used.
Current NICE Guidance 68 recommends referring patients for carotid endarterectomy who
have stable neurological symptoms from acute non-disabling stroke or TIA who have
symptomatic carotid stenosis of 50–99% according to the NASCET criteria, or 70–99%
according to the ECST criteria. The GALA trial was a parallel-group, multicentre, rando-
mized controlled trial of 3526 patients with symptomatic or asymptomatic carotid stenosis.
Patients were randomized to general or local anaesthesia for their surgery. The two groups
did not significantly differ for quality of life, length of hospital stay, stroke, myocardial
infarction or death between randomization and 30 days after surgery.

Question 165: FFTTT


Intrathecal opiates are a useful adjunct to local anaesthesia and many different opiates and
opioids have been used via this route. The principle problem with intrathecal morphine
historically has been that preservatives have caused neuronal damage, however preserva-
tive-free morphine is now available for intrathecal use. Fentanyl is less likely to cause late
respiratory depression as it is highly lipophilic and therefore has a short duration of action.
Opiates bind to G-protein-linked receptors in laminae I and II of the dorsal horn and
decrease the release of GABA in a calcium-dependent process. The more hydrophobic/more
lipophilic the opiate, the more potent it is. Due to its high lipid solubility in comparison with
morphine, fentanyl has a larger volume of distribution in the CSF.

Downloaded from https://www.cambridge.org/core. University of Edinburgh, on 19 Aug 2019 at 13:21:26, subject to the Cambridge Core terms of
use, available at https://www.cambridge.org/core/terms. https://doi.org/10.1017/9781108566100.012
Chapter 5b: Clinical Anaesthesia Answers 477

Question 166: FTTFT


AAGBI guidelines recommend a series of criteria for less serious urgent surgical cases to be
managed on a day-case basis. Spinal anaesthesia may be successful when used appropri-
ately – acute urinary retention and postural hypotension may be later complications.
Agreed follow-up protocols and audit are important components of successful day surgery
emphasized by the guideline. Elderly people frequently become somewhat confused follow-
ing surgery and in some cases this may be best treated by avoiding admission; discharges
where appropriate are recommended in these cases.

Question 167: TFTTT


Pregnancy per se does not delay gastric emptying, however the gastric contents may be of
increased volume and more acidotic. Labour prolongs gastric emptying significantly.
Sympathetic stimulation, including pain, stress and anxiety, will all prolong gastric empty-
ing, sometimes far in excess of what is considered normal.
Glycopyrrolate acts as an antagonist at acetylcholine receptors. Acetylcholine is involved
in facilitating gastric emptying. Acute alcohol intake impairs gastric emptying.

Question 168: FTTFF


A recent systematic review concludes there is no convincing evidence that regional anaes-
thesia hinders diagnosis of acute compartment syndrome; rather it may facilitate detection
by heralding breakthrough pain in the presence of continuous peripheral nerve block.
In pre-existing nerve injury, individual risk–benefit analysis is essential. Modifications
such as using a less potent local anaesthetic, reducing volume, concentration, or both and
avoiding vasoconstrictors may minimize risk from new or progressive neurological com-
plications. Before block performance, detailed neurological examination and documenta-
tion of findings are essential for medicolegal purposes. When rivaroxaban is used in
a prophylactic dose, central neuraxial blocks can be performed 18 hours after the last dose.
Pain is an unreliable symptom for diagnosing acute compartment syndrome, demon-
strating poor sensitivity (19%). Absence of motor response to peripheral nerve stimulation
does not confirm that the needle has not been in contact with the nerve.

Question 169: TFTFF


ICU-acquired weakness can involve critical illness polyneuropathy, critical illness myopa-
thy and critical illness neuromyopathy. These conditions usually develop about a week after
ICU admission. The syndrome is characterized by generalized symmetrical flaccid weak-
ness. Cranial nerves are usually spared. Other causes of weakness should be excluded before
the diagnosis is made. Most of the patients will still be dependent on the ventilator at the
time of diagnosis.

Question 170: FTTTF


There are many physiological changes that occur during pregnancy, many of which poten-
tially alter the conduct of anaesthesia. There is an increase in the incidence of difficult
intubation, but it is not as high as stated, being closer to 1:250–400. There is a reduction in

Downloaded from https://www.cambridge.org/core. University of Edinburgh, on 19 Aug 2019 at 13:21:26, subject to the Cambridge Core terms of
use, available at https://www.cambridge.org/core/terms. https://doi.org/10.1017/9781108566100.012
478 Chapter 5b: Clinical Anaesthesia Answers

plasma cholinesterase activity of around 25%, which can prolong the action of suxametho-
nium. There is also an unrelated increased sensitivity to non-depolarizing relaxants.
Cerebrospinal fluid (CSF) volume is reduced in pregnancy due to the enlargement of
epidural venous plexuses and there is a reduction in the required local anaesthetic doses
in both spinal and epidural anaesthesia. The pressure of the CSF is raised when compared to
prepregnancy levels and can reach 60–70 mmHg during contractions; it is therefore not
advisable to advance a Tuohy needle during a contraction! Oxygen consumption is
increased and oxygen reserve reduced in pregnancy, which is why rapid desaturation can
occur in a woman undergoing a general anaesthetic, despite full preoxygenation. Whilst an
increase in oxygen consumption of 60–70% would not be unusual during labour, it is closer
to 20% in a non-labouring woman at term.

Question 171: TTFFF


Carbimazole is a prodrug, being rapid and completely converted to methimazole during
passage thought the liver. Carbimazole prevents synthesis of new T3 and T4, however stored
T3 and T4 are unaffected. As such, treatment with carbimazole to render a patient euthyroid
requires weeks rather than days. Inhibition of the enzyme thyroid peroxidase inhibits
oxidation of iodide to iodine. Carbimazole crosses the placenta and may cause fetal
hypothyroidism. Carbimazole increases the vascularity of the thyroid gland and thus should
be omitted on the day of thyroid surgery.

Question 172: FTFFT


The lateral soft tissue neck film is the single most useful study. Patients with supraglottitis will
exhibit thickening and rounding of the epiglottis (the ‘thumb’ sign), with loss of the vallecular
air space. In contrast to viral croup, the signs of respiratory obstruction in epiglottitis are
unlikely to be relieved by administration of nebulized epinephrine. At laryngoscopy, the
epiglottis will be red and swollen and the arytenoids and other supraglottic tissues inflamed,
so that the glottic opening may be extremely difficult to visualize. The key is to secure the
airway under inhalation anaesthesia with 100% oxygen. Anaesthesia will normally be induced
in the sitting position and IV cannulation is only attempted once the child is sufficiently
obtunded. A tracheal tube 0.5–1.0 mm (ID) smaller than normal for the child’s age should be
chosen to assist passage through the partially obstructed glottic opening.

Question 173: TFTTF


As well as blocking fast-acting sodium channels, local anaesthetics may act by stabilizing
lipid membranes and in excessive doses inhibit aerobic metabolism and ATP production in
the mitochondria. There is typically a biphasic response with CNS excitability – circumoral
tingling, tinnitus and dizziness followed by seizures and reduced level of consciousness.
Cardiovascular collapse and arrhythmias may occur concurrently. Symptoms can be rela-
tively non-specific, so a high index of suspicion is always needed. Intralipid is thought to act
by providing a ‘sink’ for local anaesthetics, reducing the free concentration in the plasma
and possibly at a mitochondrial level as well. Carnitine is an endogenous amino acid that is
necessary to transfer long-chain fatty acids into the mitochondria for metabolism.
Deficiency is associated with increased risk of local anaesthetic toxicity. Toxicity may
occur at any time following infiltration.

Downloaded from https://www.cambridge.org/core. University of Edinburgh, on 19 Aug 2019 at 13:21:26, subject to the Cambridge Core terms of
use, available at https://www.cambridge.org/core/terms. https://doi.org/10.1017/9781108566100.012
Chapter 5b: Clinical Anaesthesia Answers 479

Question 174: TTTTT


Transfusion-associated graft versus host disease is a rare but fatal complication of transfu-
sion. Although blood is leucodepleted, the presence of residual white cells can trigger the
reaction in susceptible patients. Irradiation of the blood inactivates these white cells.
Patient groups include those:
• With congenital immune deficiency (DiGeorge syndrome, Wiskott–Aldrich syndrome)
• With Hodgkin’s disease
• Undergoing transfusions from first/second-degree relatives
• Undergoing intrauterine transfusions
• Undergoing neonatal exchange transfusions
• Receiving HLA-selected platelet units
• Receiving purine analogues
• Who receive stem cell transplants
• Who receive granulocyte transfusion

Question 175: FFTFF


Aspiration under general anaesthesia occurs at a rate of 1 in 600 emergency GAs, and 1 in
3000–6000 elective GAs. The lobes affected depend on the gravitational flow of the gastric
contents and therefore vary depending on the position of the patient. In the supine position
the lobes most commonly affected are the posterior segment of the right upper lobe and the
superior segment of the right lower lobe. Following aspiration, pneumonia will develop in
approximately 20–30% of cases, with Gram-negative organisms being most commonly
implicated. Antibiotics should only be prescribed if there is strong evidence of pneumonia,
therefore empirical administration is not recommended. Routine use of steroids is also not
recommended.

Question 176: FTTFT


Eisenmenger’s syndrome is characterized by pulmonary hypertension with MAP of the
pulmonary artery >25 mmHg at rest and >30 mmHg with exercise. This leads to reversibility
of an existing left to right shunt to a right to left one with subsequent central cyanosis.
Pulmonary hypertension could also result from iatrogenic increase in pulmonary blood flow
after corrective cardiac surgery (e.g. Blalock–Taussig shunt, Waterston shunt, Potts shunt).
A study by Salehian et al. reported that left ventricular dysfunction (defined as left ventri-
cular ejection fraction, LVEF <50%), right ventricular hypertrophy, and signs and symptoms
of heart failure predict mortality in patients with Eisenmenger. However, surgical correction
of the cardiac defect is unlikely to be successful in reversing the pulmonary hypertension.

Question 177: FFFTF


Blood group O is the universal donor, as the red cells do not have antigens on the red cell
surface to trigger an antigen–antibody response in the recipient’s blood. Blood group AB is
the universal recipient as these people have both A and B antigens on their red cell surfaces
and therefore do not contain either A or B antibodies in their plasma and so will not form
antigen–antibody complexes in their plasma. A recipient may form antibodies to the rhesus
antigen and therefore B– recipients should receive B– or O– blood. If the patient is rhesus

Downloaded from https://www.cambridge.org/core. University of Edinburgh, on 19 Aug 2019 at 13:21:26, subject to the Cambridge Core terms of
use, available at https://www.cambridge.org/core/terms. https://doi.org/10.1017/9781108566100.012
480 Chapter 5b: Clinical Anaesthesia Answers

positive they can receive rhesus positive or rhesus negative blood. In an emergency,
O– blood should be used, so no A, B or rhesus antigens are present in the transfused
blood to trigger an antigen–antibody reaction.

Table 5.177.1 The eight blood groups

GROUP A GROUP B GROUP AB GROUP O

Antibodies present Anti-B Anti-A None Anti-A and Anti-B


Antigens present A antigen B antigen A and B antigens None

Question 178: FFTFT


Methadone is a synthetic opioid that has been used in the management of heroin addiction
and withdrawal for decades. It has also found a place in the management of cancer and
chronic pain. It is a μ-receptor agonist and NMDA-receptor antagonist. It has high, though
variable, oral bioavailability (35–100%). In doses >100 mg per 24 hours, corrected QT (QTc)
interval prolongation and torsades de pointes have been reported. Risk factors for QTc
prolongation in patients taking methadone include:
• Female gender
• Concomitant administration of other QTc-prolonging drugs
• Electrolyte imbalance
• Structural or ischaemic heart disease
• Congenital long QT syndrome
• Liver impairment
• Renal impairment
For day-case surgery the best strategy is to allow the patient to remain on their usual
methadone dose and then provide additional analgesia as normal.
In the presence of a normal ECG and the absence of any other risk factors, ondansetron
should still be safe to administer and so I have marked this as true. Please feel free to
disagree!

Question 179: TTTTT


Parkinson’s disease can be idiopathic or have a number of other causes; the common
underlying pathology is loss of dopaminergic transmission in the substantia nigra. Some
of the better-recognized causes are listed as answers to this question and a more compre-
hensive list of answers are classified under subheadings in Table 5.179.1

Downloaded from https://www.cambridge.org/core. University of Edinburgh, on 19 Aug 2019 at 13:21:26, subject to the Cambridge Core terms of
use, available at https://www.cambridge.org/core/terms. https://doi.org/10.1017/9781108566100.012
Chapter 5b: Clinical Anaesthesia Answers 481

Table 5.179.1 Causes of Parkinson’s disease

Degenerative: Idiopathic
Multisystem degeneration (parkinsonism-plus)
Familial parkinsonism
Pharmacological: Drugs affecting dopamine synthesis, storage or
release, e.g. reserpine
Drugs affecting the dopamine receptor, e.g.
prochlorperazine
Structural: Normal pressure hydrocephalus, tumour, trauma
Toxic: MPTP (1-methyl-4-phenyl-
1,2,3,6-tatrahydropyridine), carbon monoxide,
cyanide, manganese
Metabolic: Hypoparathyroidism
Infective: Encephalitis lethargica, acquired immunodeficiency
syndrome
Vascular: Multi-infarct dementia

Question 180: TTTTF


Sub-Tenon’s blocks for eye surgery have increased in popularity due to a reduced risk
of major complications compared to sharp needle techniques. There are, however,
certain occasions when one may have to think twice before performing a sub-Tenon’s
block:
• Scleral explants and previous retinal surgery may make sub-Tenon’s blocks difficult or
impossible
• Active local infection
Certain ocular cicatricial conditions such as ocular pemphigoid or Stevens–Johnson syn-
drome require conjunctival preservation to prevent progression to symblepharon and sub-
Tenon’s blocks in these cases should be avoided if possible.

Question 181: TTFFT


CSEs can be used in various different surgical specialties and have been proven to decrease
the total dose of drugs given via the epidural route, and also have been shown to improve
analgesia and muscle relaxation in obstetric analgesia in comparison to epidural-only
techniques. Both mid-line and paramedian techniques can be used, and needle-through-
needle and separate injection techniques have been described. The needle-through-needle
technique has a higher failure rate of the spinal component, but the rate of PDPH has been
quantified at between 0.6 and 2.5% (i.e. the same as epidural analgesia alone).

Downloaded from https://www.cambridge.org/core. University of Edinburgh, on 19 Aug 2019 at 13:21:26, subject to the Cambridge Core terms of
use, available at https://www.cambridge.org/core/terms. https://doi.org/10.1017/9781108566100.012
482 Chapter 5b: Clinical Anaesthesia Answers

Question 182: TTTTF


AAGBI Guidelines clearly state a number of requirements for patients who have undergone
discharge. It advises analgesia should be started before regional blocks have worn off.

Question 183: FFTFF


The preoperative management of diabetes is complex. Consideration needs to be given to
the type of diabetes the patient has, their medication and the type of surgery they are
undergoing.
Ideally, patients with diabetes should be placed first on the operating list in order to
minimize starvation times, but there are circumstances where this is not possible or
appropriate. Equally, it is good practice to measure the blood glucose within the hour
prior to the patient coming to theatre. Patients with Type I diabetes may require a variable
rate intravenous insulin infusion if their starvation period is prolonged. However, a short
procedure with minimal starvation time and rapid resumption of oral intake post procedure
may make this unnecessary. Type II diabetes is more complex to manage: patients could be
on one of many different regimens. The important consideration is to maintain blood
glucose between 4 and 12 mmol.l–1 (ideally 6–10 mmol.l–1).

Question 184: TFFTF


The French gauge is the same as the Charriere gauge. It is commonly used for the sizing of
catheters and double lumen tubes. French gauge = 3 × diameter (and therefore the larger the
gauge, the larger the catheter). Hence a 39Ch tube is 13 mm in external diameter. This varies
from the other common medical gauge used which is the wire gauge, e.g. in IV cannulae.
This compares how many cannulae can fit into a standard-sized wire or tube and hence the
larger the gauge, the smaller the actual cannula. The Carlens tubes were the first double
lumen tubes, produced in 1949, with a carinal hook and one lumen running anterior to the
other. Today the reusable Robert Shaw red rubber tubes are still in use, whilst many other
centres have moved onto disposable PVC tubes. Bronchial blockers can also be used and are
particularly useful in difficult anatomy and in paediatric patients. Left-sided tubes are
generally preferred to minimize the risk of right upper lobe (RUL) hypoventilation as the
RUL comes off proximally and at a sharp angle from the right main bronchus and is
therefore at risk of being obstructed by the tube.

Question 185: TFFFT


Septic shock is defined as sepsis-induced hypotension that is not responding to fluid
resuscitation. Vasopressors should be started once the vascular system has been adequately
filled. Short synacthen test is no longer recommended and low-dose steroids should be
started early if the patient’s BP is poorly responding to fluid and vasopressors. In ventilated
patients, the inspiratory plateau pressure is the key value rather than the peak pressure. This
should be maintained below 30 cmH2O to reduce the risk of lung injury. Activated protein
C has been withdrawn from the market following the preliminary results of the PROWESS-
SHOCK trial. Although ScvO2 >70% can be misleading, being due to poor oxygen extraction
rather than delivery, it is still one of the goals recommended by Surviving Sepsis Campaign.

Downloaded from https://www.cambridge.org/core. University of Edinburgh, on 19 Aug 2019 at 13:21:26, subject to the Cambridge Core terms of
use, available at https://www.cambridge.org/core/terms. https://doi.org/10.1017/9781108566100.012
Chapter 5b: Clinical Anaesthesia Answers 483

Question 186: FTTTT


Whilst warfarin remains the most frequently used anticoagulant, direct oral anticoagulants
(DOACs) are being prescribed more frequently. DOACs’ direct mechanism of action
produces a relatively quick onset and offset, which can be advantageous when compared
to warfarin. Apixaban and rivaroxaban are direct inhibitors of factor Xa. Dabigatran is a
thrombin inhibitor.
The American Society of Regional Anesthesia recommends an interval of 5 half-lives
before neuraxial blocks should be attempted, leaving up to 3.1% of the drug in the system.
This equates to 72 hours for both apixaban and rivaroxaban and 3–5 days for dabigatran,
depending on creatinine clearance.

Question 187: TFTFT


The 2011 BTS guidelines recommend referring any patient with three or more risk factors to
a cardiologist for optimization.
Risk factors:
• High-risk type of surgery (includes all thoracic surgery)
• Ischaemic heart disease
• History of congestive cardiac failure
• History of cerebrovascular disease
• Insulin therapy for diabetes
• Preoperative serum creatinine >177 mmol.l–1
Table 5.187.1 Risk factors and the risk of major cardiac complications

Number of factors Risk of major


cardiac complications (%)

0 0.4
1 1
2 7
≥3 11

Question 188: TFTTF


Causes of vitamin B12 deficiency:
• Inadequate dietary intake (e.g. vegans)
• Impaired absorption (intrinsic factor deficiency, pernicious anaemia, post gastrectomy,
terminal ileum resection, Crohn’s disease, bacterial overgrowth, chronic tapeworm
infestation, drugs e.g. metformin)
• Congenital abnormalities in metabolism

Downloaded from https://www.cambridge.org/core. University of Edinburgh, on 19 Aug 2019 at 13:21:26, subject to the Cambridge Core terms of
use, available at https://www.cambridge.org/core/terms. https://doi.org/10.1017/9781108566100.012
484 Chapter 5b: Clinical Anaesthesia Answers

Question 189: FFTTF


Croup is a common paediatric respiratory illness involving inflammation and narrowing of
the subglottic region of the larynx, frequently precipitated by viral infections. 65–75% of
cases are due to parainfluenza (types 1, 2, 3); most severe cases are due to influenza A. It has
a gradual onset and manifests variously as a barking cough, hoarseness, stridor and
respiratory distress, with or without concomitant symptoms of viral upper respiratory
infection. Children primarily between the ages of six months and three years are affected.
Management includes humidification, nebulizers, steroids and racemic adrenaline.
Antibiotics are only required if there is a secondary bacterial infection.

Question 190: TFTTT


Heavy bupivacaine is preferred as it gives a more predictable block ascent compared to plain
bupivacaine. This is especially true in pregnancy where the gravid uterus leads to raised
intra-abdominal pressure and engorged epidural veins. Obesity is also a risk factor for high
block for similar reasons. Epidural top-up within the past 30 minutes is a risk factor, which
some would consider a relative contraindication to spinal anaesthesia due to the risk of high
spinal block. Fine-gauge spinal needles are thought to be a risk factor due to increased
infiltration pressures, compounded by cephalad angulation. The volume of local anaesthetic
is less important than the mass in terms of risk of high spinal block.

Question 191: TFFTF


The plasma concentration of sodium is 131–143 mmol.l–1. It is the major plasma cation
contributing to plasma osmolarity.
The relative sodium concentrations in IV fluids are as follows:
• 0.9% NaCl = 154 mmol.l–1
• Hartman’s solution = 131 mmol.l–1
• 5% dextrose = 0 mmol.l–1
• 8.4% sodium bicarbonate = 1000 mmol.l–1
• 4% dextrose/0.18% sodium chloride = 30 mmol.l–1
All of the crystalloids other than sodium bicarbonate have a pH less than physiological
(sodium bicarbonate is higher).

Question 192: FFFFF


After initial insertion, a tract usually forms by seven to ten days. Tracheostomies are
described in terms of their internal diameter, although the outer diameter is also often
printed on the tracheostomy. Tracheostomies can be double or single cannula, double
cannula tracheostomies possessing an inner tube, which can be removed and cleaned,
hence reducing the risk of secretions obstructing the lumen. Tracheostomies can possess
fenestrations (in both the inner and outer tubes), which in a cuffed tracheostomy lie above
the cuff. By deflating the cuff of these fenestrated tracheostomies during spontaneous
respiration, increased airflow can pass through the upper airways, helping to aid speech.
Percutaneous tracheostomies have been shown to have a lower infection rate than surgical
tracheostomies. The procedure-related mortality with percutaneous tracheostomy has been
quoted as <0.5%.

Downloaded from https://www.cambridge.org/core. University of Edinburgh, on 19 Aug 2019 at 13:21:26, subject to the Cambridge Core terms of
use, available at https://www.cambridge.org/core/terms. https://doi.org/10.1017/9781108566100.012
Chapter 5b: Clinical Anaesthesia Answers 485

Question 193: TTTFT


Hypoplastic left heart syndrome (HLHS) was first described by Lev in 1952 and named in
1958 by Noonan and Nadas. The essential malformation is the underdevelopment or
absence of the left ventricle. Features include: atretic, hypoplastic or stenotic aortic and
mitral valves, patent foramen ovale and a large patent ductus arteriosus to supply blood to
the systemic circulation. The ventricular septum is usually intact.

Question 194: FTFTF


The minimum patient identifier information is last name, first name, date of birth and unique
identification number, such as hospital number or NHS number. Blood administration sets have
a 170–200 μm integral mesh filter and these should be used for all blood product administrations.
Acute transfusion reactions present within 24 hours of transfusion. They include:
• Febrile non-haemolytic transfusion reactions, usually clinically mild
• Allergic transfusion reactions, ranging from mild urticaria to life-threatening angio-
oedema or anaphylaxis
• Acute haemolytic transfusion reactions such as ABO incompatibility
• Bacterial contamination of blood unit
• Transfusion-associated circulatory overload (TACO)
• Transfusion-related acute lung injury (TRALI)
TRALI is most common after transfusion of plasma-rich blood components such as FFP or
platelets. Implicated donors are usually females sensitized during a previous pregnancy.
Since the UK Blood Services switched to using male donors for producing FFP, resuspending
pooled platelets in male plasma and screening female apheresis platelet donors for leucocyte
antibodies has resulted in a significant fall in both reported cases and mortality from TRALI.

Question 195: FFTFF


The varicella zoster virus (VZV) is a double-stranded DNA herpes virus, and it is the
reactivation of this virus that causes herpes zoster (HZ), or shingles. About 20% of patients
with HZ will go on to develop postherpetic neuralgia (PHN). In the acute phase of HZ, the
use of antiviral drugs such as acyclovir, which control viral DNA replication, and therefore
reduce the severity of pain and rash, may markedly reduce the number of patients that go on
to suffer from PHN. For those unfortunate enough to get PHN, first-line medical manage-
ment will involve the use of one or more of the gabapentinoids such as gabapentin, tricyclic
antidepressants and topical lidocaine in the form of a plaster, not the gel. Opioids such as
oxycodone and morphine can significantly reduce the pain of PHN. Their use may be
limited by their side-effect profile, particularly as this will be an elderly age group and the
use of opioids in non-cancer pain is controversial.

Question 196: TFTTT


See explanation for Question 6.

Downloaded from https://www.cambridge.org/core. University of Edinburgh, on 19 Aug 2019 at 13:21:26, subject to the Cambridge Core terms of
use, available at https://www.cambridge.org/core/terms. https://doi.org/10.1017/9781108566100.012
486 Chapter 5b: Clinical Anaesthesia Answers

Question 197: TTFFT


Cerebral hyperperfusion syndrome is characterized by the triad of ipsilateral migraine-like
headache, seizures and transient neurological deficits in the absence of cerebral ischaemia
after a successful carotid endarterectomy. Other symptoms that have been described include
motor disturbance, abnormal speech, nausea, psychotic disorders, visual disturbance and
ataxia. The symptoms may occur up to a few weeks post revascularization, but are usually
seen within the first few days. It is thought to be caused by an increase in cerebral blood flow
as a result of impaired autoregulation and an increase in systemic blood pressure, and may
be fatal once intracerebral haemorrhage occurs. Treatment strategies are directed towards
regulation of the blood pressure and limitation of rises in cerebral perfusion.

Question 198: FFFFF


The National Burn Care Referral Guidance issued by the National Network for Burn Care
(NNBC) makes separate distinctions for referral or discussion with each of the three levels
of burns services: burn facility, burn unit and burn centre.
The guidance recommends referring adults to a burn centre if they have:
• ≥40% TBSA burn
• ≥25% TBSA burn with inhalational injury
It also recommends giving special consideration to referring those >65 years with ≥25%
TBSA to a burns centre.
Adults with the above injuries should all be referred to a burns unit with the exception of
those who are pregnant, who should be discussed with a burns unit.

Question 199: FTTFT


Regional anaesthesia can be a useful adjunct to minimize postoperative pain, however it is
important to pick the correct block as an incorrect one exposes the patient to all the risks of
the block with none of the benefits, even if it is performed perfectly. For a Pfannenstiel or
‘bikini-line’ incision, the dermatomes to block are T12–L1, which can be achieved with
iliohypogastric, ilioinguinal or transversus abdominis blocks. Rectus sheath blocks are used
to block the terminal branches of the 9th to 11th intercostal nerves, and will not provide
analgesia below the level of the umbilicus. Lumbar plexus blocks are used in hip replacement
surgery as they effectively block the femoral, lateral cutaneous, obturator and saphenous
nerves.

Question 200: TTFFT


Aminoglycosides, quinolones and non-depolarizing muscle relaxants can all exacerbate
myasthenia gravis. If required, NDMRs should be used at reduced dose (typically one-
tenth normal) and clear adequacy of reversal should be checked before extubation.
Myasthenic patients tend to be resistant to suxamethonium due to occupation of acetylcho-
line receptors by antibodies. Edrophonium is used to substantiate the diagnosis clinically; if
it has no effect, this may be due to a cholinergic crisis rather than an incorrect diagnosis.

Downloaded from https://www.cambridge.org/core. University of Edinburgh, on 19 Aug 2019 at 13:21:26, subject to the Cambridge Core terms of
use, available at https://www.cambridge.org/core/terms. https://doi.org/10.1017/9781108566100.012
Chapter 5b: Clinical Anaesthesia Answers 487

Question 201: TFFTF


Pioglitazone belongs to the class of drugs known as thiazolidinediones. They act to increase
hepatic sensitivity to insulin and increase the clearance of glucose. Acrabose inhibits
intestinal α-glucosidases, delaying the absorption and digestion of carbohydrates.
Sitagliptin is a drug from the family of dipeptidylpeptidase-4 inhibitors. They are given as
oral medication and act by increasing insulin secretion and reducing glucagon secretion.
Glucagon-like peptide-1 mimics such as exenatide and liraglutide are given by subcuta-
neous injection. They increase insulin secretion, impair glucagon secretion and delay gastric
emptying. Metglitinides, such as repaglinide and nateglinide, have a short duration of action
and as such are given prior to meals. They act on the β-cells of the pancreas to increase
insulin secretion.

Question 202: TFFTF


The Child–Pugh Score was originally developed in 1973 and used to predict the operative
mortality in patients with bleeding varices. It has since been modified and is now used to
determine prognosis in patients with chronic liver disease/cirrhosis and the necessity of
liver transplantation. The score is made up of five criteria:
• Grade of encephalopathy
• Ascites
• Serum bilirubin
• Serum albumin
• Prolongation of prothrombin time
Each category ranges from 1 to 3 points, and thus a total score of 5 to 15.

Table 5.202.1 Classification of chronic liver disease

Child–Pugh class Score One-year survival (%)

A 5–6 100
B 7–9 80
C 10–15 45

Critics of the score claim that it relies heavily on clinical examination, which leads to
inconsistencies in score. The Model for End Stage Liver Disease (MELD) is a newer scoring
system that uses only weighted laboratory values (bilirubin, INR and creatinine) to predict
survival.

Question 203: FTFFF


Dialysis and filtration are the two basic systems for different types of renal replacement
therapy (RRT) that are available. Haemodialysis can be intermittent or continuous whilst
haemofiltration is always continuous. Other variants include haemodiafiltration, slow
continuous ultrafiltration (SCUF) and sustained low-efficiency dialysis (SLED). All of
these systems incorporate a semipermeable membrane. Dialysis follows Fick’s law of diffu-
sion where molecules move down their concentration gradient. In filtration, the molecules

Downloaded from https://www.cambridge.org/core. University of Edinburgh, on 19 Aug 2019 at 13:21:26, subject to the Cambridge Core terms of
use, available at https://www.cambridge.org/core/terms. https://doi.org/10.1017/9781108566100.012
488 Chapter 5b: Clinical Anaesthesia Answers

move by convection down a pressure gradient, hence it resembles the renal glomerulus.
Continuous RRT is the preferred method in ICU because it provides better haemodynamic
stability and is better tolerated than intermittent dialysis. Disequilibrium syndrome occurs
in chronic renal patients following haemodialysis. It is characterized by neurological
features secondary to possible cerebral oedema. Haemodialysis rapidly removes small
molecules like urea, which can result in a reverse osmotic shift allowing water to move
into brain cells.

Question 204: TTTTT


All of the statements are possible means of providing anaesthesia and analgesia for an
emergency caesarean section. The method chosen should be determined by the clinical
background of the patient, relative or absolute contraindications, available equipment, the
abilities or experience of the anaesthetist/surgeon involved and patient choice. Whilst local
anaesthetic infiltration is very rarely performed and should only be done by an appropri-
ately experienced person with great care, it may be indicated if an individual has contra-
indications to both general and regional anaesthesia.

Question 205: FFTFF


Arrhythmias are a common complication of pulmonary resection surgery and the incidence
is 30–50%. The most common arrhythmia is AF. At present, diltiazem is the most useful
drug for post-thoracotomy arrhythmia prophylaxis. Two factors in the early post-
thoracotomy period interact to produce atrial arrhythmias:
1. Increased flow resistance through the pulmonary vascular bed due to permanent (lung
resection) or transient (atelectasis, hypoxemia) causes, with attendant strain on the right
side of the heart.
2. Increased sympathetic stimuli and oxygen requirements, maximal on the second
postoperative day as patients begin to mobilize.
Digoxin does not prevent arrhythmias after pneumonectomy or other intrathoracic
procedures.

Question 206: FFTFF


Recognition, planning and preparation for a potentially difficult airway is key to the
perioperative management of all obese patients. A ramped position involves positioning
the patient with the tragus of the ear level with the sternum. A ramped position reduces risk
of a difficult laryngoscopy and improves respiratory mechanics, thus aiding oxygenation
and ventilation to maximize safe apnoea time. PEEP improves preoxygenation and reduces
intra- and postoperative atelectasis. Depth of anaesthesia monitoring is recommended to
titrate anaesthesia. Short-acting agents are recommended, including desflurane, propofol
and remifentanil.

Question 207: TFFTT


See explanation for Question 29.

Downloaded from https://www.cambridge.org/core. University of Edinburgh, on 19 Aug 2019 at 13:21:26, subject to the Cambridge Core terms of
use, available at https://www.cambridge.org/core/terms. https://doi.org/10.1017/9781108566100.012
Chapter 5b: Clinical Anaesthesia Answers 489

Question 208: FFFTF


While pulmonary embolism is possible following immobility from a fractured neck of
femur, the timing of desaturation makes bone cement implantation syndrome (BCIS)
a stronger possibility. This results in multiple microemboli of fat, bone and methyl metha-
crylate monomers showering the lungs, producing embolic effects and immune system
activation. The features of BCIS include hypoxia, hypotension, raised pulmonary vascular
resistance (units are dyn.sec.cm–5) and cardiac arrhythmias. The desaturation may only be
short-lived or may worsen and result in cardiac arrest. There is no specific treatment other
than good supportive care.
Risk factors include:
• ASA III–IV
• Osteoporosis
• Pre-existing cardiac disease
• Pre-existing pulmonary hypertension

Question 209: FFTTF


Cell saver use is recommended with:
• Expected large blood loss (>1 l)
• Preoperative anaemia/increased risk of bleeding
• Rare blood groups/antibodies
• Patient refusal of allogenic blood transfusion
Potential complications include:
• Electrolyte imbalance
• Air embolism
• Infection/pyrexia
• Fat embolism
• Microembolism
• Haemolysis (which can result in renal failure due to damage created by free haemoglobin)
• Coagulopathy (due to loss of platelets and clotting products)
• Salvaged blood syndrome (increased capillary permeability causing lung and renal injury)

Question 210: TTFTT


Fifteen percent of Down’s syndrome patients have atlantoaxial instability which may be due
to structural abnormalities of C1, C2 or laxity in the transverse ligament; 2% of patients may
display signs of cervical cord compression. Caudal anaesthesia is an appropriate analgesic
technique for herniorrhaphy and the presence of Down’s syndrome has no special contra-
indications to this. As long as cardiac involvement has been ruled out, a preoperative ECG is
unnecessary. Down’s syndrome patients have an increased prevalence of both
Hirschsprung’s disease and OSA.

Question 211: TFTTF


Oxygenated blood returning to the left atrium cannot flow into the left ventricle because of
the mitral valve anomaly. Therefore, pulmonary venous blood must cross the atrial

Downloaded from https://www.cambridge.org/core. University of Edinburgh, on 19 Aug 2019 at 13:21:26, subject to the Cambridge Core terms of
use, available at https://www.cambridge.org/core/terms. https://doi.org/10.1017/9781108566100.012
490 Chapter 5b: Clinical Anaesthesia Answers

septum through a patent foramen ovale, which is present in most babies. The right
ventricle must then pump this mixed blood to both the pulmonary and the systemic
circulations, which are connected in parallel, rather than in series, by the ductus arter-
iosus. The amount of blood that flows into each circulation is based on the resistance in
each circuit.

Question 212: TFFTF


To optimise the yield and quality of salvaged blood, a large-bore suction tip should be
used (minimum 4 mm, e.g. Yankauer sucker). To reduce haemolysis, the vacuum
pressure should always be set as low as practicable. Cell salvage is recommended by
NICE Guidelines for use in radical prostatectomy and can now be used routinely
during surgery for urological malignant disease, however aspiration of blood from
around the tumour site should be avoided to minimize contamination of salvaged
blood with malignant cells. The main concern surrounding the use of cell salvage in
obstetric haemorrhage is the risk of reinfusing fetal contaminants with the associated
theoretical risk of causing an amniotic fluid embolus. However, there are, to date, no
proven cases in the literature and the use of cell salvage in obstetrics is approved by
NICE. The blood is sucked away from the operative site by a dual lumen tube, which
mixes the blood immediately with an anticoagulant (e.g. heparin or citrate) into
a reservoir. Intraoperative cell salvage should be temporarily discontinued when sub-
stances not licensed for intravenous use are present within the surgical field. Example
substances that should not be aspirated into the cell salvage system include iodine,
topical clotting agents and orthopaedic cement.

Question 213: FTFTT


Sedation for uncomfortable or painful procedures provides many advantages to the patient,
including reducing anxiety and pain and a degree of amnesia. However, sedation is by no
means a procedure performed without risk and in inexperienced or untrained hands it can
prove disastrous, with the potential for loss of the airway, hypoventilation and cardiovas-
cular instability. As such, it requires the same detailed assessment as is performed prior to
undertaking general anaesthesia.

Question 214: FTTTT


Silas Weir Mitchell, a neurologist, was the first to describe the symptoms of complex
regional pain syndrome (CRPS) in soldiers injured during the American Civil War.
There are two types, I and II, and there is no clinical difference between them except
for known peripheral nerve damage in CRPS type II. The predominant symptom is
pain, but it is also associated with sensory, autonomic, motor, skin and bone changes.
CRPS can be further subdivided into that which is maintained by (sympathetically
maintained pain, SMP) or independent of (sympathetically independent pain, SIP) the
sympathetic nervous system.
A postulated mechanism for this is that nerve injury stimulates growth of sympathetic
axons around the dorsal root ganglia of the central nervous system, so sympathetic activity
can initiate activity in sensory fibres.

Downloaded from https://www.cambridge.org/core. University of Edinburgh, on 19 Aug 2019 at 13:21:26, subject to the Cambridge Core terms of
use, available at https://www.cambridge.org/core/terms. https://doi.org/10.1017/9781108566100.012
Chapter 5b: Clinical Anaesthesia Answers 491

Recommendations for the treatment of CRPS are summarized below:


• Education and self-management strategies.
• Specialist physiotherapy and occupational therapy. This should be initiated early and
include desensitization, gradual weight bearing, stretching, and functional and fine motor
exercises.
• Cognitive behavioural therapy, particularly for patients slow to improve and in distress.
• Pharmacological agents, such as amitriptyline and gabapentin, are used, often in
conjunction with conventional analgesics. Treatment with corticosteroids has been tried,
as has intravenous pamidronate, particularly for patients in the early stages.
• Blocks of the sympathetic ganglia using local anaesthetic are used, diagnostically to
differentiate between SMP and SIP, and therapeutically.
• CRPS is an indication for spinal cord stimulation.
Intravenous regional sympathetic blocks are not recommended due to a lack of evidence,
however they are still used by some pain specialists.

Question 215: TTTFT


You should be familiar with the updated NICE Guidelines for the management of patients
with head injury. This includes the safe transfer from hospital to a neurosurgical unit. This
document stipulates indications for intubation and ventilation of patients with brain injury
as follows:
• GCS 8 or less
• Significantly deteriorating conscious level (i.e. fall in motor score of two points or more)
• Loss of protective laryngeal reflexes
• Hypoxaemia (paO2 <13 kPa on oxygen)
• Hypercarbia (paCO2 >6 kPa)
• Spontaneous hyperventilation causing PaCO2 <4.0 kPa
• Bilateral fractured mandible
• Copious bleeding into the mouth (e.g. from skull base fracture)
• Seizures
An abnormal CT head scan is not in itself an indication for intubation and ventilation, as
this decision should be informed primarily by the clinical features listed above.

Question 216: FFTFF


The National Network for Burn Care (NNBC) issue guidance on burns care referral.
Specialized burns services are stratified into three levels of service: burn centres, burn
units and burn facilities.
Burn centres offer the highest level of care and offer a separately staffed, geographically
discrete ward, skilled to the highest level of critical care with immediate operating theatre access.
Children with the following burns require referral to a specialized burn centre:
• ≥30% TBSA (≥15% if under one year old)
• ≥20% TBSA if full thickness burns
• All those predicted to require assisted ventilation specifically for their burn injury for
more than 24 hours
• Any child who is physiologically unstable as a result of their burn

Downloaded from https://www.cambridge.org/core. University of Edinburgh, on 19 Aug 2019 at 13:21:26, subject to the Cambridge Core terms of
use, available at https://www.cambridge.org/core/terms. https://doi.org/10.1017/9781108566100.012
492 Chapter 5b: Clinical Anaesthesia Answers

Question 217: TFFTT


This is a ‘bread and butter’ question! Propofol is thought to have a slight intrinsic antiemetic
effect and volatile agents are proemetic. The incidence of PONV increases from age five
onwards and decreases around the menopause. Other patient factors include female gender,
and a history of PONV or motion sickness. Surgical factors include type and duration of
surgery (middle/inner ear, squint and gynaecological being the worst), and anaesthetic
factors include use of volatiles, nitrous oxide, opiates, relative dehydration (fasting and
lack of IV fluids). Bowel handling may cause an ileus, which may lead to PONV.

Question 218: TFFTT


There are many reports of anaphylaxis manifesting only as hypotension, initially when
under anaesthesia. H2-blockers are not recommended by current guidelines.
Recommended sampling is immediate, one to two hours and 24 hours. A number of agents
may supplement initial resuscitation: salbutamol, magnesium and aminophylline may be
needed for bronchospasm, metaraminol infusions have been used to improve hypotension
not responsive to adrenaline infusions.

Question 219: TTTTT


The half life of carboxyhaemoglobin is short and so stopping smoking for even a few hours
will improve the detrimental effect on the oxyhaemoglobin dissociation curve, improving
oxygen release to the tissues. For this reason, and to reduce the reactivity of the airways,
smoking cigarettes should be avoided for several hours preoperatively. Impaired ciliary
function and increased sputum production are recognized side effects of cigarette smoking.
In the postoperative period, due to manipulation of the airway, sputum production is
increased and thus so is the risk of lower respiratory tract infection.

Question 220: FTFTF


Continuing insulin in Type I diabetics is important to prevent ketoacidosis. Patients under-
going a procedure that will require them to miss more than one meal should be started on
a variable rate intravenous insulin infusion (VRIII). Patients who are expected to resume oral
consumption quickly following the procedure are unlikely to require a VRIII. In patients
receiving a VRIII, 0.45% saline with 5% glucose is recommended as the substrate fluid.
In those patients not requiring VRIII, Hartmann’s solution is recommended in preference
to 0.9% saline to avoid hyperchloraemic metabolic acidosis. Glucose-containing solutions
should be avoided unless the patient is hypoglycaemic, in order to avoid precipitating
hyperglycaemia. Management of patients with diabetes should follow that of an enhanced
recovery programme where applicable. High carbohydrate drinks should be omitted pre-
operatively in patients requiring insulin to avoid precipitating hyperglycaemia. Long-acting
insulin should be continued in the perioperative period alongside a variable rate insulin
infusion.

Question 221: FFTTF


Percutaneously placed tracheostomy tubes are less likely to be dislodged when compared to
the surgical ones. This is because surgical tracheostomy involves the dissection of all tissue and

Downloaded from https://www.cambridge.org/core. University of Edinburgh, on 19 Aug 2019 at 13:21:26, subject to the Cambridge Core terms of
use, available at https://www.cambridge.org/core/terms. https://doi.org/10.1017/9781108566100.012
Chapter 5b: Clinical Anaesthesia Answers 493

muscle layers and either creating a flap in the anterior wall of the trachea or completely
removing the anterior part of one of the tracheal rings. The percutaneous technique depends
on dilating the tissue rather than dissection. This means that the passage created is tighter and
therefore it will be harder to reinsert a tube if dislodged within the first few days. For the same
reasons, the percutaneously inserted tracheostomy tube is more likely to have a tamponading
effect on the surrounding tissue and reduce the risk of bleeding. A meta-analysis published in
Critical Care in 2006 showed a significant reduction in infection rate with percutaneous
tracheostomy. Cervical spine injury is a contraindication for the percutaneous but not the
surgical technique, however neck extension should be avoided.

Question 222: TTFFT


Many opioids have been added to neuraxial solutions to improve the quality of block and
provide postoperative analgesia. They work in three areas, the dorsal horn of the cord, travel
in CSF to modulate descending inhibitory pathways and being absorbed systemically for
a central effect (minimal). Different opioids do these to varying degrees. Highly lipophilic
opioids are absorbed more quickly, provide more rapid onset, but work for a shorter
duration. They are associated with less risk of delayed respiratory depression. Clonidine
can be used both intrathecally and epidurally with local anaesthetics and opioids.
It improves the sensory block, but has no effect on the motor component. By alkalinizing
lidocaine its speed of onset is increased, but there is no effect on duration of action.

Question 223: TTFTT


The most important predictor of paO2 during one-lung ventilation (OLV) is the paO2 during
two-lung ventilation, specifically the intraoperative paO2 during two-lung ventilation in the
lateral position prior to OLV. If the operative lung has little perfusion preoperatively due to
unilateral disease, the patient is unlikely to desaturate during OLV. The left lung being 10%
smaller than the right, there is less shunt when the left lung is collapsed. Other factors being
equal, patients with more severe airflow limitation on preoperative spirometry will tend to
have a better paO2 during OLV than patients with normal spirometry.

Question 224: FFTTF


Acromegaly is a multisystem disease resulting from excess production of pituitary growth
hormone. Airway difficulties may arise from:
• Mandibular enlargement
• Macroglossia
• Epiglottis enlargement
• Vocal cord thickening
• Reduced size of laryngeal aperture
• Thickened pharyngeal and laryngeal soft tissues
• Thyroid gland enlargement

Question 225: TFFTT


Aspiration of foreign bodies has a peak incidence at one to three years of age. A sudden
onset of coughing, gagging and choking is suggestive of foreign body aspiration.

Downloaded from https://www.cambridge.org/core. University of Edinburgh, on 19 Aug 2019 at 13:21:26, subject to the Cambridge Core terms of
use, available at https://www.cambridge.org/core/terms. https://doi.org/10.1017/9781108566100.012
494 Chapter 5b: Clinical Anaesthesia Answers

A foreign body at or above the vocal cords can cause complete obstruction of the upper
airway, stridor or a change or loss of voice, and the cough and dyspnoea with which
foreign bodies lower down the airway present. Partial obstruction of a lower airway
may cause air trapping behind the foreign body (ball and valve effect) with pneu-
mothorax, surgical emphysema and pneumomediastinum as possibilities. In this situa-
tion, the usual inspiratory chest X-ray can appear normal; however, an expiratory film
may reveal air trapping. Depending on the cause, location and duration of airway
blockage, collapse and consolidation of a lobe or an entire lung with bronchial breath-
ing, inspiratory crackles and expiratory wheeze may be found on examination. Without
a clear history of choking, the symptoms can be difficult to differentiate from acute
asthma. Rigid bronchoscopy with the patient breathing spontaneously under deep
inhalation anaesthesia supplemented with topical lidocaine will confirm the diagnosis
and allow for removal of the foreign body. Spontaneous ventilation is preferable to
positive pressure ventilation to avoid pushing any foreign body further in to the
bronchial tree.

Question 226: FTFFF


Airway fire during laser surgery is a rare but serious complication. Laser resistant tubes
minimize the risk of airway fire and cuffs filled with a mix of saline and methylene blue may
aid in extinguishing a fire and identifying a cuff leak. If a fire occurs, ventilate initially with
air only, until it is extinguished (saline and soaked swabs) and then administer 100%
oxygen. The endotracheal tube must be changed due to likely damage and debris can be
removed from the oropharynx and laryngeal inlet at laryngoscopy. The surgical site is most
commonly affected, and inhalational injury is rare. In cases of severe, prolonged airway fire,
a bronchoscopy should be undertaken to rule out inhalational injury and distal damage, as
well as arterial blood gas analysis and chest X-ray. In these cases a low-approach surgical
tracheostomy may be warranted. Dexamethasone should be given, with a low threshold for
admission to HDU/ITU.

Question 227: TFFTF


Although the synthetically derived colloids are formed from natural substances they are not
classed as naturally occurring due to the manufacturing processes they have to go through.
Both human albumin and plasma are classed as blood products and are issued by the
blood bank.

Question 228: TFTFF


The superior laryngeal nerve provides sensory innervation to the:
• Base of the tongue
• Posterior surface of the epiglottis
• Aryepiglottic folds
• Arytenoids
The recurrent laryngeal nerve provides sensory innervation to the:
• Trachea
• Vocal cords

Downloaded from https://www.cambridge.org/core. University of Edinburgh, on 19 Aug 2019 at 13:21:26, subject to the Cambridge Core terms of
use, available at https://www.cambridge.org/core/terms. https://doi.org/10.1017/9781108566100.012
Chapter 5b: Clinical Anaesthesia Answers 495

The glossopharyngeal nerve provides sensory innervation to the:


• Posterior third of the tongue
• Vallecula
• Anterior surface of the epiglottis (lingual branch)
• Walls of the pharynx (pharyngeal branch)
• Tonsils (tonsillar branch)

Question 229: TFFTT


In preterm infants, the most common source of intracranial haemorrhage is the germinal
matrix, which lies in the lateral ventricles. Epidural haemorrhage is rare in newborns
because the middle meningeal artery, which is the usual source of epidural bleeding,
moves freely away from displacements of the skull in this age group. Cranial ultrasono-
graphy is the investigation of choice because of its availability, low cost and high resolution
for bleeding. CT scans are discouraged, because the effect of radiation on the immature
central nervous system is not known.

Question 230: FTFFF


The accuracy of the thermodilution technique can be influenced by a number of factors:
• Intracardiac shunt leading to differences in right and left ventricular output
• Right-sided regurgitant valve lesions can underestimate cardiac output
• Variations in blood temperature affect measurements
• The timing of bolus injection during the respiratory cycle because positive pressure
ventilation produces beat-to-beat variation in right ventricular stroke volume
The Fick principle states that the amount of a substance taken up by an organ per unit time is
equal to the arterial minus venous concentrations of the substance multiplied by blood flow.
Using pulmonary artery flotation catheters with a thermodilution technique for cardiac output
calculations, a known volume of cold saline is injected through the proximal port into the right
atrium. The temperature change is measured by a thermistor (not thermocouple) at the catheter
tip located in the pulmonary artery. The balloon at the distal end of the catheter holds
approximately 1.5 ml. Normal pulmonary capillary wedge pressure is 4–12 mmHg.

Question 231: FFTFT


Table 5.231.1 Red flags: the features, signs or symptoms that indicate
serious spinal pathology

Features Previous history of malignancy


Age <16 or >50 with new onset pain
Unexplained weight loss
Intravenous drug use
Previous steroid use
Recent serious illness/infection
Sign Saddle anaesthesia
Reduced anal tone
Generalized neurological deficit
Progressive spinal deformity
Downloaded from https://www.cambridge.org/core. University of Edinburgh, on 19 Aug 2019 at 13:21:26, subject to the Cambridge Core terms of
use, available at https://www.cambridge.org/core/terms. https://doi.org/10.1017/9781108566100.012
496 Chapter 5b: Clinical Anaesthesia Answers

Table 5.231.1 (cont.)

Symptoms Pain not relieved by rest or worse at night


Thoracic pain
Fevers/rigors
General malaise
Urinary retention
Cauda equina Abrupt back pain with pain/weakness in legs
syndrome Urinary retention
Perianal sensory loss
Erectile dysfunction
Reduced anal tone

Question 232: FTFTF


Pathophysiological changes and their incidences following brainstem death include: hypo-
tension (81%), diabetes insipidus (65%), disseminated intravascular coagulation (28%),
cardiac arrythmias (27%), pulmonary oedema (18%) and metabolic acidosis (11%). Organ
preservation is a key element in the successful transplantation of organs from heart-beating
donors and so an understanding of the pathophysiological changes listed, and how they
arise and might best be managed, is useful. The aetiology of some of the systemic changes is
described here. Brainstem death is often preceded by a pathological rise in intracranial
pressure, leading to hypertension and bradycardia. As the brainstem is rendered ischaemic,
a sympathetic storm ensues, with the release of a massive amount of catecholamines and free
radicals with the potential for damaging transplantable organs. Myocardial ischaemia and
arrhythmias commonly begin in this phase. Ultimately infarction of the brainstem vaso-
motor centres leads to relative vasodilatation and hypotension. Pulmonary oedema can be
neurogenic, related to a generalized inflammatory response or underlying pulmonary/
cardiovascular pathology. Failure of the hypothalamic–pituitary–adrenal axis leads to
reduced circulating T3 with resultant anaerobic metabolism and lactate formation that
contribute alongside poor peripheral perfusion to metabolic acidosis. Lack of ADH
results in diabetes insipidus; this in turn produces dehydration and further metabolic
derangement.

Question 233: FTFFT


Inhalation injury carries a 73% incidence of progression to respiratory failure and a 20%
incidence of acute respiratory distress syndrome (ARDS). Heat, particulates, systemic toxins
and respiratory irritants all combine to cause damage to the airways and lung parenchyma.
Carbon monoxide binds to haemoglobin with about 250 times the affinity of oxygen.
It shifts the oxygen dissociation curve to the left, impairing oxygen delivery, and inhibits
oxygen binding to cytochrome oxidase, disrupting aerobic metabolism. The half-life of
carbon monoxide is about three to four hours in air, 30–90 minutes in 100% oxygen and
15–23 minutes in hyperbaric oxygen (at 2.5 atmospheres). Cyanide combines with trivalent
iron in the mitochondrial cytochrome A3 complex, inhibiting cellular respiration.

Downloaded from https://www.cambridge.org/core. University of Edinburgh, on 19 Aug 2019 at 13:21:26, subject to the Cambridge Core terms of
use, available at https://www.cambridge.org/core/terms. https://doi.org/10.1017/9781108566100.012
Chapter 5b: Clinical Anaesthesia Answers 497

Hydroxocobalamin (50 mg.kg–1) binds the cyanide and is excreted by the kidney. Amyl
nitrate and sodium thiocyanate are alternative treatment options. A study of 90 children in
Texas found that nebulization of heparin and N-acetylcysteine in children with massive
burn and smoke inhalation injuries resulted in a significant decrease in incidence of
reintubation and reduced mortality. A patient intubated for an inhalational injury should
not have their endotracheal tube cut; it is usually wired to a stable tooth to prevent
migration.

Question 234: TTTTF


All of the above drugs are excreted into breast milk with the exception of rocuronium –
it is a large, ionized molecule that is water soluble so will not be excreted. All of the
above drugs will not harm the baby via ingestion when given in therapeutic doses to the
mother.

Question 235: TTFTF


Randomized trials have shown both plasma exchange and intravenous immunoglobulin to
be equally effective. The American Association of Neurologists does not recommend a
combination. Steroids have been shown to have no benefit. Analgesia is important, as many
patients suffer with severe neuropathic pain. Broad-spectrum antibiotics have no specific
role in GBS itself, but secondary infection should be treated as necessary.

Question 236: FFFTT


This gentleman has poorly controlled, chronic hypertension. It is inappropriate to proceed
with elective surgery owing to the exaggerated pressor response he may suffer, increasing
the risk of perioperative myocardial ischaemia. It is appropriate to take a full history,
seeking to establish whether he has symptoms suggestive of ischaemia, heart failure, renal
dysfunction or malignant hypertension. Any alteration in symptoms should prompt per-
formance of an ECG. Urea and electrolytes should be checked to ensure that any change in
blood pressure is not due to an underlying renal problem. Once the above has been
completed it is appropriate to refer this gentleman back to his GP for optimization of his
blood pressure control. Surgery can be considered once his blood pressure has been
stabilized for a period of approximately six weeks. Referral to cardiology is unnecessary
unless there has been a change in symptoms or the patient has symptoms suggestive of
malignant hypertension. The administration of intravenous antihypertensives is not indi-
cated in this scenario, since the patient is symptom-free.

Question 237: FTTTF


About 20–30% of cardiac output goes to the liver; 70% of that is via the portal vein and 30%
from the hepatic artery. Macroscopically, whilst the liver is described as having a right and
left lobe, functionally it is more accurately described by the Couinaud (or French) system,
which divides it into eight subsegments. Each segment has its own vascular inflow, outflow
and biliary drainage. Microscopically, each liver lobe is split into hexagonal hepatic lobules
with plates of hepatocytes radiating from a central hepatic vein. At the corners of the lobules
are the portal tracts. Portal tracts (which are also misleadingly called portal triads) contain

Downloaded from https://www.cambridge.org/core. University of Edinburgh, on 19 Aug 2019 at 13:21:26, subject to the Cambridge Core terms of
use, available at https://www.cambridge.org/core/terms. https://doi.org/10.1017/9781108566100.012
498 Chapter 5b: Clinical Anaesthesia Answers

branches of the hepatic artery, portal vein, bile ducts, vagus nerve and lymphatics. The liver
is involved in protein metabolism – in particular albumin, globulins, coagulation factors,
complement, transferrin, haptoglobulins, caeruloplasmin, plasma cholinesterase and
α1-antitrypsin. Vitamins A, D, E, K, B12 and folate are stored in the liver. ALP is increased
by up to four times during normal pregnancy because of placental secretion. ALT, AST and
LDH generally do not increase in pregnancy. Albumin is often decreased due to
haemodilution.

Question 238: FTTTT


The main pathological features of life-threatening asthma are gas trapping and the genera-
tion of intrinsic PEEP. To avoid further hyperinflation, one should aim to reduce the minute
volume ventilation. Reducing the I:E ratio provides longer expiratory time for the
obstructed bronchi. This, together with limited or no PEEP helps in reducing the incidence
of gas trapping. Patients tolerate hypercapnia well as long as a pH ≥7.2 is maintained.

Question 239: TFFFT


Epilepsy is the most prevalent pre-existing neurological disease in pregnant women and is
still the most common cause of seizures. Eclampsia is, however, the most common cause of
seizures in the peripartum period and should therefore be considered in a patient who is
fitting, even if known to be epileptic. Regional anaesthesia in benign intracranial hyperten-
sion is not usually contraindicated unless there is a new blockage or shunt in situ that could
be damaged. Myasthenia gravis increases in severity in around 40% of patients and this
tends to be a postpartum phenomenon. Multiple sclerosis patients have an increased relapse
rate in the first few postpartum months, but it is unrelated to epidural use. Regional
techniques are often contraindicated in neurofibromatosis patients due to CNS tumours.
If they can be used safely after imaging then there is an increased likelihood of a patchy
block with an epidural.

Question 240: FFFFT


Premedication in children is a useful adjunct to general anaesthesia, resulting in a calm
induction and a degree of amnesia for the child.
Appropriate doses of premedication in children include:
• Oral midazolam 0.5 mg.kg–1
• IV midazolam 0.1 mg.kg–1
• Oral ketamine 3 mg.kg–1
• IM ketamine 1–2 mg.kg–1
• Oral clonidine 3 μg.kg–1

Question 241: TFTTT


Myotonic dystrophy follows an autosomal dominant pattern of inheritance, with the
abnormality lying on chromosome 19. Muscles are in a hyperexcitable state secondary to
abnormal sodium or chloride channels. Common features of the disorder include incom-
plete muscle relaxation and muscle wasting. Aspiration may be a consequence of bulbar

Downloaded from https://www.cambridge.org/core. University of Edinburgh, on 19 Aug 2019 at 13:21:26, subject to the Cambridge Core terms of
use, available at https://www.cambridge.org/core/terms. https://doi.org/10.1017/9781108566100.012
Chapter 5b: Clinical Anaesthesia Answers 499

muscle weakness. Stimulants for myotonias include hypothermia and shivering, as well as
mechanical and electrical stimulation.

Question 242: TFTTT


Systemic inflammatory response syndrome (SIRS) in children is defined as an age-
appropriate change in heart rate, respiratory rate and white cell count. Fever, tachycardia
and vasodilatation are common in children. However, when combined with a change in
mental status, septic shock should be suspected. Neisseria meningitidis, Haemophilus influ-
enzae and Streptococcus pneumoniae are the most common causes of sepsis in children over
three months. Tachycardia is an early sign of shock, but a non-specific one. Hypotension is
more specific, but is a late sign of shock. A normal respiratory rate associated with increased
work of breathing means the patient has respiratory distress. A combination of oxygen
utilization (ScvO2) and perfusion pressure is the most useful and these are relatively easily
measured variables. In septic shock, however, the patient may remain vasodilated and have
a normal capillary refill time or ‘flash’ capillary refill until late in the disease process.
Intravenous fluids, crystalloid in 20 ml.kg–1 aliquots over 5–10 minutes, should be used to
maintain target heart rate and BP. ABCDs should be reassessed after each bolus. Gasping
respiration, bradypnoea or apnoea, bradycardia and hypotension are all late signs preceding
cardiac arrest. Indications for intubation and ventilation include increased work of breath-
ing, hypoventilation, cardiovascular collapse and impaired mental status.

Question 243: FFTTT


Over 95% of people have the normal plasma cholinesterase alleles. The rarest form is
homozygous fluoride resistant (E1f:E1f) with an incidence of approximately 1:150000.
The incidence of E1s:E1s is approximately 1:10000. There are ethnic variations in gene
carriage. E1u:E1s also has a dibucaine number of 80.

Question 244: TFTTF


The femoral vein lies within the femoral triangle. The superior border is formed by the
inguinal ligament, medial border by adductor longus and lateral border by the sartorius.
The femoral vein is a continuation of the popliteal vein and becomes the external iliac vein at
the inguinal ligament. It is medial to the artery and nerve, residing within a sheath with the
artery, with the nerve outside. As the vein is traced distally it becomes more posterior to the
artery, increasing the risk of arterial puncture.

Question 245: FTFFF


Extracorporeal lithotripsy (ESWL) uses a repetitive high-energy focused acoustic pulse,
generated outside the body, to fragment renal/ureteric stones. ESWL is most effective for
stones between the sizes of 4 mm and 20 mm in diameter that are located in the proximal
ureter. However, it can still be used for those located in the kidney and more distally along
the renal tract, but with lower success rates. Renal stones are most commonly formed from
calcium oxalate (40–70%), other compositions include calcium phosphate, mixed oxalate/
phosphate, struvite and uric acid. A permanent pacemaker is not a contraindication to
lithotripsy, but the lithotriptor should be kept at least 15 cm away from the pacemaker, the

Downloaded from https://www.cambridge.org/core. University of Edinburgh, on 19 Aug 2019 at 13:21:26, subject to the Cambridge Core terms of
use, available at https://www.cambridge.org/core/terms. https://doi.org/10.1017/9781108566100.012
500 Chapter 5b: Clinical Anaesthesia Answers

shock-wave pulses should be timed with the ECG and the pacemaker rate modulation
should be deactivated (seek advice from pacemaker technician). When the shock waves
reach an air/water interface, energy is released, therefore it is advisable to use loss-of-
resistance to water rather than air when siting an epidural.

Question 246: TFTFT


In preterm infants, apnoea is inversely related to the gestational age. Apnoea is either central
or obstructive. The following problems of the premature infant can lead to apnoeic episodes
in the neonatal period:
• Immaturity of the respiratory centre
• Intracranial bleed
• Hydrocephalus
• Infection
• Anaemia
• Metabolic disturbances
• Temperature instability
• Medications (e.g. maternal GA drugs or opiates)

Question 247: TTTTT


All the answers to this question are true. Chronaxie does indeed vary depending on the type
of nerve:
• Aα myelinated fibres (motor) 50–100 ms
• Aδ myelinated fibres (sensory) 150 ms
• C unmyelinated fibres (sensory) 400 ms

Question 248: FTTFF


The verbal rating scale (VRS), numerical rating scale (NRS) and visual analogue scale (VAS)
are simple pain scales used to measure acute pain intensity. The VRS asks for one response
from either no pain, mild pain, moderate pain or severe pain. It is simple to use but
insensitive to small changes in pain intensity. The NRS has numbers zero (no pain) to ten
(worst pain imaginable), making an 11-point scale (sorry!). Again, it is simple to use and
relatively insensitive to small changes in pain intensity. The VAS is often used in research
and is sensitive to small changes. Patients are presented with a 10 cm line with ‘no pain’ at
one end and ‘worst pain imaginable’ at the other and asked to cross the line at an appro-
priate point. Multidimensional pain scales are used to assess chronic pain and include the
McGill Pain Questionnaire and the Brief Pain Inventory. Pain can be assessed in the
cognitively impaired using the behavioural pain assessment scale that assesses facial expres-
sion, restlessness, muscle tone, vocalization and consolability. The Children’s Hospital of
Eastern Ontario Pain Scale (CHEOPS) is an observational measure of postoperative pain in
children.

Question 249: FTTFT


There are in excess of 40 different systems for classifying the presenting features of
subarachnoid haemorrhage. The three most commonly used systems are the World

Downloaded from https://www.cambridge.org/core. University of Edinburgh, on 19 Aug 2019 at 13:21:26, subject to the Cambridge Core terms of
use, available at https://www.cambridge.org/core/terms. https://doi.org/10.1017/9781108566100.012
Chapter 5b: Clinical Anaesthesia Answers 501

Federation of Neurosurgeons (WFNS) classification (grade 1–5), the Hess and Hunt
classification (grade 1–5) and the Fisher classification, which is based on CT findings
alone. These systems are of value in prognosticating as well as in delineating which patients
may benefit from early or later intervention. The WFNS and Hunt and Hess scales are
described in Tables 5.249.1 and 5.249.2.

Table 5.249.1 WFNS

Grade GCS Motor defecit

1 15 −
2 13–14 −
3 13–14 +
4 7–12 −/+
5 3–6 −/+

Table 5.249.2 Hunt and Hess

0. Unruptured aneurysm
1. Asymptomatic/mild headache or neck stiffness, mortality 0–5%
2. Severe headache, neck stiffness, cranial nerve palsy, mortality 2–10%
3. Mild focal defecit, lethargy, confusion, mortality 8–15%
4. Stupor, hemiparesis, early decerebrate rigidity, mortality 60–70%
5. Deep coma, decerebrate posture, mortality 70–100%

Question 250: TTTTF


In 2005, as part of ‘Saving Lives’, the Department of Health issued guidance on high-impact
interventions (HIIs). These are evidence-based interventions that relate to key clinical
procedures or care processes that can reduce the risk of infection if performed appropri-
ately. It is a legal requirement for acute hospitals and other care providers to comply with
the Code of Practice for Prevention and Control of Healthcare Associated Infections and
programmes such as ‘Saving Lives’ provide the tools and resources for trusts to achieve this.
The high-impact interventions were rewritten in 2007 and again in 2010 and now give
guidance and tools on various procedures, including:
• Central venous catheter care
• Peripheral venous cannula care
• Renal haemodialysis
• Prevention of surgical site infection
• Ventilator-associated pneumonia
• Urinary catheter care
• Clostridium difficile infection

Downloaded from https://www.cambridge.org/core. University of Edinburgh, on 19 Aug 2019 at 13:21:26, subject to the Cambridge Core terms of
use, available at https://www.cambridge.org/core/terms. https://doi.org/10.1017/9781108566100.012
502 Chapter 5b: Clinical Anaesthesia Answers

• Cleaning and decontamination


• Chronic wounds
• Enteral feeding
• Blood cultures

Question 251: FFFFF


Sterilization can be performed at any point in the menstrual cycle, but female sterilization
(1:200 failure rate) is less reliable than both vasectomy (1:2000 failure rate) and the Mirena ®
IUD (1:1000 failure rate). It can be carried out on nulliparous women, however they must be
completely sure that they want to be sterilized.

Question 252: TFTTT


Risk factors for the need for postoperative ventilation in myasthenia gravis include:
• Co-existent respiratory disease
• FVC <2.9 l
• Diagnosis of >6 years
• Major body cavity surgery
• Pyridostigmine dose > 750 mg.day–1

Question 253: FTTTF


First-line agents in those under 55 years are ACE inhibitors. In patients over 55 years or in
black patients of African or Caribbean family history of any age, a calcium-channel blocker
should be commenced. Inadequate control with either an ACE inhibitor or a calcium-
channel blocker should prompt introduction of the other agent. Diuretics are considered
third-line agents after ACE inhibitors and calcium-channel blockers. Thiazide-like diuretics
such as chlortalidone or indapamide are the preferred agents of choice, moving away from
traditional thiazides such as bendroflumethiazide.

Question 254: TTFTT


The patient is septic, as evident by the tachycardia and tachypnoea prior to induction.
Inadequate resuscitation prior to induction resulted in her BP falling precipitously. Fluid
resuscitation followed by vasopressor infusion is the first line of management in this case.
Controlling the source of sepsis is essential and in this case surgical exploration and
debridement is the best option. Clindamycin is usually the antibiotic of choice in these
severe cases. Blood glucose control is part of the sepsis management bundle.

Question 255: FTTFT


Non-obstetric surgery is required in up to 2% of pregnancies, the most common procedure
being appendicectomy. There is a higher incidence of spontaneous miscarriage and preterm
labour in those undergoing surgery. The aim of perioperative management is to prevent fetal
hypoxia by maintaining maternal physiological parameters under relatively tight control.

Downloaded from https://www.cambridge.org/core. University of Edinburgh, on 19 Aug 2019 at 13:21:26, subject to the Cambridge Core terms of
use, available at https://www.cambridge.org/core/terms. https://doi.org/10.1017/9781108566100.012
Chapter 5b: Clinical Anaesthesia Answers 503

Most anaesthetic agents are considered safe to use and whilst they do affect fetal heart rate and
variability they are not known to cause direct fetal distress and are not teratogenic. Nitrous
oxide is best avoided due to its effects on methionine synthetase, and ketamine should not be
used as it increases uterine tone and could cause fetal asphyxia. Studies have shown that there
is no increased risk to the fetus with laparoscopic surgery, provided appropriate care is taken.
Routine CTG monitoring is an area of some controversy. Evidence suggests that there is no
improvement in fetal outcome when it is used and in addition there may be practical problems
with restricted access. Despite this, many recommend its use, if available, to detect overt fetal
distress. Most importantly, midwives and obstetricians should be involved early and have
agreed a management plan prior to any intervention.

Question 256: TFTTF


Risk factors for development of rheumatoid arthritis include:
• Female gender
• Cigarette smoking
• HLA-DR4 subtype
• Seropositive for rheumatoid factor
• Unidentified viral or bacterial agents
• A family history of rheumatoid arthritis
• Allergies and intolerances to food
• Altered gut flora
• Psychological stress
• Exposure to heavy metals

Question 257: TTFTT


Craniopharyngioma is the third most common brain tumour of non-glial origin. They are
usually detected when they start to produce symptoms by pressing on structures such as the
pituitary gland, optic nerves, optic chiasm, blood vessels or brain. Stunted growth, delayed
puberty, diabetes insipidus, decreased blood pressure, visual disturbances, headache, vomit-
ing and disorders of temperature regulation are common. Disturbances of thyroid and
adrenal function must be quantified before surgery. Diabetes insipidus frequently occurs
perioperatively. If treated with desmopressin, it is important to identify the prescribed dose,
normal urine volumes and serum electrolytes. MRI is useful to determine the extent and
spread of tumour and also aids planning for surgery.

Question 258: TFFFT


A definitive airway would be beneficial in this case since a person with advanced airway
skills is present. If it was not possible to intubate the patient, then maintaining a patent
airway with an LMA would be acceptable. It is important to remember that safe doses are
a guide only and other factors should always be considered. These include patient factors
(pregnancy, renal failure, liver failure, malnutrition) and site of infiltration. This patient
would be at high risk of local anaesthetic toxicity due to his renal impairment and local
anaesthetic absorption being relatively rapid from sciatic nerve blocks. There is no indica-
tion for propofol in this case – there is no seizure activity and intralipid should be used to
manage local anaesthetic toxicity. Atropine is no longer recommended in ALS protocols and

Downloaded from https://www.cambridge.org/core. University of Edinburgh, on 19 Aug 2019 at 13:21:26, subject to the Cambridge Core terms of
use, available at https://www.cambridge.org/core/terms. https://doi.org/10.1017/9781108566100.012
504 Chapter 5b: Clinical Anaesthesia Answers

this patient is on the shockable side of the ALS algorithm. An incident form should be
completed for an adverse incident with actual patient harm – if intralipid is used this should
also be reported on www.lipidregistry.org.

Question 259: TTFTF


There are three waves and two descents in a normal CVP trace:
• a wave: atrial contraction
• c wave: bulging of tricuspid valve into atrium
• x descent: atrial relaxation
• v wave: atrial filling prior to tricuspid valve opening
• y descent: passive atrial emptying
The a-wave is most commonly affected by pathological changes, absent in atrial fibrillation
and enlarged in tricuspid and pulmonary stenosis and pulmonary hypertension. The v-wave
enlarges in tricuspid regurgitation. Cannon waves are large waves that do not correspond to
the normal waves and are present in complete heart block or junctional arrhythmias.

Question 260: FFTTF


Midazolam is a water-soluble imidazobenzodiazepine that has a structure that is dependent
on the surrounding pH. At low pH the diazepine ring is open, giving a water-soluble ionized
molecule, at a pH over 4 the ring closes, giving a lipid-soluble unionized molecule.
Midazolam has a pKa of 6.5, therefore at physiological pH it is 89% unionized. It is
presented as a clear colourless solution with a pH of 3.5; due to its water-solubility it does
not cause pain on injection. It has an oral bioavailability of approximately 40%. Midazolam
is almost completely metabolized in the liver by hydroxylation, derivatives are then con-
jugated with glucuronides and excreted renally. The metabolites are active.

Question 261: TTFFT


Apnoea is very common in preterm babies, and often associated with bradycardia. The risk
is increased in premature babies following general anaesthesia or the use of sedation.
Premature babies may have a history of apnoea that can be central, obstructive or mixed
in aetiology. Various definitions are used for postoperative apnoea, but a cessation of
respiration for 15–20 seconds, which may be associated with desaturation and bradycardia,
is commonly used. These episodes are usually self-limiting or require mild stimulation of
the baby to encourage respiration. Apnoea is more common in the more premature babies
and occurs mainly in the first 24 hours postoperatively.
Risk factors are:
• Age less than 56–60 weeks PGA
• Neurological disease
• Anaemia
• General anaesthesia
• Sedation
• Opiate use
• Known respiratory disease, e.g. bronchopulmonary dysplasia
• Preoperative oxygen requirement

Downloaded from https://www.cambridge.org/core. University of Edinburgh, on 19 Aug 2019 at 13:21:26, subject to the Cambridge Core terms of
use, available at https://www.cambridge.org/core/terms. https://doi.org/10.1017/9781108566100.012
Chapter 5b: Clinical Anaesthesia Answers 505

Question 262: FFTFF


Increasingly, surgeons are using recurrent laryngeal nerve stimulators to protect the nerves,
so techniques avoiding neuromuscular blockade are becoming more common. Arterial lines
may be placed if indicated for individual patients, but are not mandatory. Head-up
positioning aids venous drainage of the surgical site. Awareness is not specifically associated
with thyroid or endocrine surgery and depth of anaesthesia monitoring guidelines do not
specifically refer to its use in this circumstance. Thyroid hormone levels are not so labile as
to require intravenous liothyronine supplementation.

Question 263: TTFTT


Cervical spinal cord injury at the level of C1–C3 can result in immediate paralysis of all
respiratory muscles with consequent apnoea and immediate death unless the patient is
intubated and ventilation applied. The patient will remain ventilator dependent unless
a diaphragm stimulator is used. A spinal cord injury at the level of C3–C5 can result in
a varied degree of impairment of diaphragmatic contraction, and while ventilation may be
necessary in the acute stages, it is eventually possible to wean the majority of patients from
ventilation, although this may be a long process. With C6–C8 spinal cord injuries, expira-
tion is entirely a passive process, as the intercostal and abdominal muscles are paralyzed
while the diaphragmatic innervation remains intact. Respiratory failure is rare with these
lesions but secretion retention can be problematic. Overall, of all patients with cervical cord
injuries, one-third will require intubation and ventilation at some point.

Question 264: TTTTT


The Cataract National Dataset publication in 2009 that extracted data on 55 567 cataract
patients from November 2001 to July 2006 found that sharp needle techniques (peribulbar
and retrobulbar blocks) had a 2.5-fold increased risk of serious complications (defined as
sight- or life-threatening) compared with sub-Tenon’s cannula techniques. Despite being
a generally safer technique, it is important to remember that sub-Tenon’s blocks do carry
the risk of serious complications, and scleral perforation, central spread of local anaesthetic,
retrobulbar haemorrhage, optic neuropathy, hyphaema and extraocular muscle paresis have
been reported. Minor complications such as conjunctival chemosis and subconjunctival
haemorrhage were 2.3 times more common with sub-Tenon’s blocks than ‘any sharp needle
technique’ (peribulbar, retrobulbar or combinations of the two).

Question 265: FFFFT


Congenital diaphragmatic hernia accounts for 8% of all major congenital abnormalities and
90% of all defects are located in the posterolateral diaphragm, with 80% of all defects on the
left side of the diaphragm. The defect is thought to occur, paradoxically, due to hypoplastic
lung development causing inhibition of development of the fetal diaphragm rather than vice
versa. There is no benefit of caesarean section over vaginal delivery in these cases, though
delivery should be planned so that paediatric resuscitation and intensive care facilities are
available immediately postdelivery. Once delivered, the child should be intubated rather
than bag-valve-mask ventilated, as this may cause distension of the stomach and further
difficulty in ventilation. A large bore NG tube should be passed to decompress any

Downloaded from https://www.cambridge.org/core. University of Edinburgh, on 19 Aug 2019 at 13:21:26, subject to the Cambridge Core terms of
use, available at https://www.cambridge.org/core/terms. https://doi.org/10.1017/9781108566100.012
506 Chapter 5b: Clinical Anaesthesia Answers

intrathoracic abdominal contents. O2 saturations of >85% should be aimed for during


resuscitation due to the fetus being acclimatized to lower paO2 and the risk of causing
further lung damage by barotrauma from overdistension. A period of 24–48 hours of
stabilization on PICU should be planned, as there are frequently deteriorations in post-
surgical thoracic compliance and paCO2.

Question 266: FTTTF


Hypothermia is associated with coagulopathy and acute pancreatitis. Poor peripheral perfusion
can lead to a raised lactate. Immunosuppression and poor wound healing also occur. Trials have
failed to demonstrate the benefit of cooling for traumatic brain injury and this is not recom-
mended by international guidelines (although prevention and treatment of pyrexia are).

Question 267: FFTFF


Table 5.267.1 The revised North American Society of Pacing and Electrophysiology (NASPE) and British Pacing
and Electrophysiology Group (BPEG) pacemaker nomenclature

Position I II III IV V

Category Chamber Chamber Response Programmability, rate Multi-site


paced sensed modulation pacing
O none O none O none O none O none
A atrium A atrium T trigger P simple A atrium
V ventricle V ventricle I inhibit programmability V ventricle
D dual (A + V) D dual (A + V) D dual M multiprogrammability D dual (A + V)
(T+ I) C communicating
R rate modulation

Question 268: TTTTT


The patient suffers from refractory hypoxaemia as a result of the ARDS developed second-
ary to acute pancreatitis. The main principle in management of ARDS is to reduce further
lung injury by allowing for permissive hypercarbia and hypoxaemia. All of the above are
recognized strategies that are used as rescue therapies in refractory hypoxaemia. The aim of
the furosemide infusion is to offload the lungs of the excess water, thus improving the
ventilation/perfusion mismatch. Paralysis can be useful if the patient is not synching well
with the ventilator. Prone positioning takes the weight of the heart and mediastinum off the
lungs, allowing for better recruitment of collapsed alveoli. But it comes at the cost of possible
facial oedema, pressure sores and endotracheal tube dislodgement. HFOV allows for better
recruitment at a pressure higher than the mean airway pressure (MawP), but with the
advantage of using very small tidal volumes and reducing the atelectrauma that results from
the cyclical inflation–deflation of alveoli by conventional ventilation. Nitric oxide reduces
the pulmonary artery pressure, improving blood flow to the better-ventilated alveoli.

Question 269: TFTFT


There is an increase in difficult intubation in the obese pregnant population. Positioning of,
and access to, the patient is often difficult and may require extra or specialist equipment and

Downloaded from https://www.cambridge.org/core. University of Edinburgh, on 19 Aug 2019 at 13:21:26, subject to the Cambridge Core terms of
use, available at https://www.cambridge.org/core/terms. https://doi.org/10.1017/9781108566100.012
Chapter 5b: Clinical Anaesthesia Answers 507

tables. There are often similar problems with applying monitoring equipment and gaining
IV access. Whilst positioning of obese patients may be difficult, there is no requirement to
increase pelvic tilt. Obesity is associated with an increased risk of almost every complication
of pregnancy and delivery. Any intervention that could reduce the requirement for post-
operative opiate use should probably be used unless contraindicated. Multimodal analgesia
will help reduce the risk of opiate-induced respiratory compromise. There is no increase of
local anaesthetic toxicity in the obese. There is an increased blood flow requirement of
2–3 ml.min–1 per 100 g to extra body fat and this requires a corresponding increase in
cardiac output to compensate.

Question 270: FTTFF


Cushing’s syndrome causes a vast array of systemic features, some of which are listed below:
• Altered appearance – moon face/central obesity with buffalo hump
• Depression
• Psychosis
• Insomnia
• Amenorrhoea/oligomenorrhoea
• Poor libido
• Thin skin/easy bruising/purple striae on abdomen and thighs
• Hirsutism/acne
• Proximal muscle wasting
• Osteoporosis
• Growth arrest in children
• Back pain
• Polyuria/polydipsia
• Hypertension
• Type 2 diabetes
• Hypernatraemia
• Hypokalaemia
Treatment is dependent on the cause. Metyrapone is an inhibitor of cortisol synthesis, and
may be used to reverse the systemic features of the syndrome. Metyrapone can be used as
a pretreatment prior to surgery or in cases not amenable to surgery. Nelson’s syndrome is
a postoperative risk of adrenalectomy, caused by MSH secretion resulting in hyperpigmen-
tation and compression of parapituitary structures.

Question 271: TFFTF


Predictors for difficult laryngoscopy are the same in obese patients as for non-obese patients.
Rather than intubation being difficult in obese patients, it is usually facemask ventilation and
maintenance of adequate oxygenation that is problematic. The strongest predictors of
a difficult intubation in an obese patient are:
• Male gender
• Large neck circumference
• Limited neck mobility
• High Mallampati score
• Obstructive sleep apnoea

Downloaded from https://www.cambridge.org/core. University of Edinburgh, on 19 Aug 2019 at 13:21:26, subject to the Cambridge Core terms of
use, available at https://www.cambridge.org/core/terms. https://doi.org/10.1017/9781108566100.012
508 Chapter 5b: Clinical Anaesthesia Answers

Question 272: TFTTF


An ECG is mandatory; unfortunately anaesthetists miss 25% of abnormalities. A CPET can
be performed on a bicycle or hand-crank wheel for those with arthritis. However,
a dobutamine stress test can be used in patients with severe arthritis. Closing capacity
encroaches on tidal volume when supine after 65 years of age. Oxygenation is improved by
adopting the sitting position. Silent aspiration is common due to reduced cough reflexes and
increased oesophageal reflexes. The risk of postoperative confusion is high and avoidance of
general anaesthetic is desirable.

Question 273: TFTFF


Benzodiazepines, thiopentone or propofol boluses can be used to manage seizures asso-
ciated with local anaesthetic toxicity. Boluses of 20% intralipid should be repeated at five-
minute intervals at a dose of 1.5 ml.kg–1 over one minute up to a maximum of three boluses.
The infusion rate can be doubled from 15 to 30 ml.kg–1.h–1, but the maximum total dose
should not exceed 12 ml.kg–1. Cardiac arrest may be refractory to treatment and therefore
resuscitation attempts may need to be prolonged. AAGBI guidance suggests that recovery
may take over one hour, so good-quality CPR should continue for at least this length of time.

Question 274: TTFFT


Use of intrathecal opioids is a widely accepted method of analgesia and where combined
with minimally invasive surgery it has allowed for the development of enhanced recovery
programmes. Intrathecal opioids primarily act at G-protein-coupled opioid receptors
within the dorsal horn of the spinal cord. Side effects are no longer thought to be dose
dependent and include:
• Sedation
• Sweating
• Delayed gastric emptying
• Urinary retention
• Pruritus
• Nausea and vomiting
• Respiratory depression

Question 275: FTTFT


Propofol is metabolized in the liver by glucuronidation to form a propofol-glucuronide, and
by sulfo- and glucuro-conjugation of the hydroxylated metabolite via cytochrome P450. All
metabolites are inactive and excreted in the urine. The clearance of propofol exceeds hepatic
blood flow, suggesting some additional extrahepatic metabolism. Renal failure does not
prolong the action of propofol. Midazolam is almost completely metabolized in the liver by
hydroxylation, derivatives are then conjugated with glucuronides and excreted in the urine.
Midazolam has an active metabolite 1-α-hydroxymidazolam which can accumulate in
those with renal failure and has been linked to prolonged sedation.

Downloaded from https://www.cambridge.org/core. University of Edinburgh, on 19 Aug 2019 at 13:21:26, subject to the Cambridge Core terms of
use, available at https://www.cambridge.org/core/terms. https://doi.org/10.1017/9781108566100.012
Chapter 5b: Clinical Anaesthesia Answers 509

Dexmedetomidine is a selective α2-agonist that provides anxiolysis and sedation without


significant respiratory depression. Dexmedetomidine is metabolized in the liver to methyl
and glucuronide conjugates, which are excreted in the urine. Although renal impairment
does not significantly alter the pharmacokinetics of dexmedetomidine, the sedative action of
the drug is prolonged in renal disease.
Ketamine is metabolized in the liver by N-demethylation to the active metabolite nor-
ketamine. Norketamine is further metabolized to inactive glucuronide metabolites that are
excreted in the urine. Clonidine is 50% metabolized in the liver to inactive metabolites, the
other 50% is excreted unchanged in the urine, and hence the sedative action of the drug is
increased in the presence of renal failure.

Question 276: TFTTF


Kernicterus is an acquired metabolic encephalopathy of the neonatal period. It is caused by
unconjugated hyperbilirubinaemia that develops either as a result of haemolytic diseases or
because of the inability of the liver to conjugate bilirubin. This is due to a defect in the
enzyme glucuronyl transferase, e.g. in premature babies in whom this enzyme is not fully
functional. Unconjugated (indirect) bilirubin in serum is bound to albumin. Unbound
unconjugated bilirubin crosses the blood–brain barrier as it is more lipid soluble.
Hypoalbuminaemia, low pH, which weakens the albumin–bilirubin bond, and drugs (sal-
icylates, sulfonamides) that compete with bilirubin for albumin binding increase the
amount of unconjugated unbound bilirubin. Maternal anti-D immunization can prevent
Rh incompatibility, reducing the incidence of neonatal jaundice.

Question 277: TTFFF


The sciatic nerve is formed from the nerve roots L4 to S3. The two components of the
nerve (tibial and common peroneal) diverge approximately 4–10 cm above the popliteal
crease to separately continue their paths into the lower leg. Blocking the sciatic nerve
will provide anaesthesia of the leg below the knee with the exception of the medial strip
of skin, which is innervated by the saphenous nerve. The posterior or Labat approach
to the sciatic nerve requires the patient to be in the lateral decubitus position tilted
slightly forward. A line is drawn between the posterior superior iliac spine (PSIS) and
the greater trochanter. Another line, passing through the mid-point of this line and
perpendicular to it, is extended 4 cm caudal and is the needle insertion point. Nerve
stimulation of the tibial nerve produces plantar flexion of the foot, whereas common
peroneal stimulation produces dorsiflexion.

Question 278: TTTFT


Neural blockade with neurolytic agents has been documented for the treatment of pain for
over a century. Their use has the potential to relieve refractory cancer pain, but the potential
for adverse outcomes is significant and so their use has classically been restricted to those
patients where life expectancy is short. They can be used on peripheral nerves, the sympa-
thetic chain and intrathecally. Phenol is commonly used in a 6% solution. It is hyperbaric
and so care needs to be taken with patient positioning. It is painless on injection. Alcohol
comes in 50–100% strengths. It is hypobaric and painful on injection. It is more popular for
coeliac plexus blocks than phenol as it has a greater ability to diffuse through tissues.

Downloaded from https://www.cambridge.org/core. University of Edinburgh, on 19 Aug 2019 at 13:21:26, subject to the Cambridge Core terms of
use, available at https://www.cambridge.org/core/terms. https://doi.org/10.1017/9781108566100.012
510 Chapter 5b: Clinical Anaesthesia Answers

Question 279: TFTFF


There are two types of cardiac stent: drug-eluting and bare metal. Following stent insertion
a period of dual antiplatelet therapy is required to prevent endothelialization of the stent;
typically daily aspirin and clopidogrel are administered. In the case of bare metal stents,
a minimal period of six weeks dual antiplatelet therapy is required, although this may be
extended at the advice of the patient’s cardiologist. Following this, aspirin alone is continued
indefinitely. For drug-eluting stents, the period of dual antiplatelet therapy is at least 12
months. Ideally, elective surgery is postponed until the period of dual therapy has ended.
For surgery that cannot be delayed, consideration must be given to the balance of the risk of
bleeding perioperatively and the risk of stent thrombosis; this should be undertaken as
a multidisciplinary decision between cardiologist, surgeon and anaesthetist. Where bleeding
risk is low and the risk of thrombosis is high, it may be appropriate to continue dual therapy
or consider a bridging therapy. Clopidogrel is a contraindication to central neuraxial
blockade; aspirin is not.

Question 280: TFTFT


Suxamethonium causes an increase in intraocular pressure (IOP) of approximately 8–10
mmHg for about 5–10 minutes after its administration. This rise occurs after the initial
decrease in IOP caused by the induction agent (unless this is ketamine!), but the pressure
does not usually reach preinduction values, although this cannot always be relied upon.
Rocuronium causes a slight decrease in IOP, whereas atracurium causes no change.
Metoclopramide may cause a slight increase in IOP lasting about 30 minutes, whereas
ondansetron has no effect on IOP when given prior to induction.
Increase IOP:
• Ketamine
• Succinylcholine
• Metoclopramide
Decrease IOP:
• Alfentanil, fentanyl, morphine, remifentanil, sufentanil
• Thiopentone, propofol, etomidate
• Halothane, isoflurane, sevoflurane, enflurane, desflurane
• Rocuronium, vecuronium, pancuronium
No change in IOP:
• Oral benzodiazepines, midazolam
• Anticholinergics
• Atracurium
• Nitrous oxide
• Neostigmine

Question 281: TTFFF


Gastroschisis is a condition where bowel herniates through a defect to the right of the
umbilicus. It is distinct from exomphalos in that it does not involve the umbilicus, and as
a result the bowel is not covered by a sac and is directly exposed to amniotic fluid. This leads

Downloaded from https://www.cambridge.org/core. University of Edinburgh, on 19 Aug 2019 at 13:21:26, subject to the Cambridge Core terms of
use, available at https://www.cambridge.org/core/terms. https://doi.org/10.1017/9781108566100.012
Chapter 5b: Clinical Anaesthesia Answers 511

to the formation of a reactive inflammatory peel and mesenteric thickening. There is a 3:2
male:female incidence, and risk factors include low maternal age, low parity, maternal
smoking, use of decongestants and aspirin. Due to the lack of covering of the bowel, in
contrast to exomphalos, closure of the gastroschisis defect is performed within hours of
birth. If there are signs of gut ischaemia at delivery then the neonate is taken to theatre
directly as a surgical emergency. Due to the effects of gut distension, nitrous oxide is not
recommended as a carrier gas for anaesthetizing these cases, and in cases where anasto-
moses or stomas are formed for an associated bowel stenosis or malrotation, it can put
additional stress on the suture line. Feeding is delayed postoperatively until the volume of
NG aspirates decreases and becomes less bilious, and due to the exposure of the gut to
amniotic fluid, full feeding may take up to ten weeks to establish. Therefore, TPN may need
to be considered early in these cases.

Question 282: FTTFF


Metformin should be given as normal in a patient on a morning or afternoon list if the
patient is expected to resume eating and drinking soon after surgery. The perceived risk of
developing lactic acidosis is negligible in this setting and outweighed by the risk of hyper-
glycaemia. Pioglitazone should be continued perioperatively when it is anticipated only one
meal will be missed. Meglitinides should be omitted on the morning of surgery in patients on
the morning list. Patients undergoing surgery in the afternoon should receive their morning
dose of meglitinide if they eat breakfast in the morning. Glucagon-like peptide-1 inhibitors
should be omitted on the day of surgery regardless of whether the patient is on the morning
or afternoon list. Sulfonyureas should be omitted in the morning for a patient on the
morning list and in the morning and afternoon for patients on the afternoon list.

Question 283: TTFTT


Patients who remain ventilated for more than 48 hours are at great risk of developing
ventilator-associated pneumonia. The best prevention is to avoid ventilation in the first
place. Daily sedation holding and assessment for extubation is essential. Other interventions
that are well supported by the literature include: hand hygiene, chlorhexidine oral rinse, head
elevation, subglottic and oral suctioning of secretions. The usefulness of selective gut decon-
tamination remains controversial, as well as antibiotic-impregnated endotracheal tubes.

Question 284: FFTTF


When optimizing for phaeochromocytoma surgery, α-blockade is started first, as unop-
posed β-blockade can lead to paradoxical hypertension (due to loss of β-mediated vasodi-
latation and thus increased SVR) or heart failure due to decreased inotropy in the face of
high afterload. Α blockade should be introduced gradually to allow for organ adaptation to
hypertension. Agents such as labetalol have β-activity greater than α, so should not be used
alone, but have some advocates for use once α-blockade is established. The presence of
postural drop is used to suggest adequate α-blockade. As catecholamines are insulin
antagonists, hyperglycaemia and secondary diabetes should be sought. Echocardiography
is important if there is likely to be cardiac disease, to seek left ventricular hypertrophy and/
or catecholamine-related cardiomyopathy.

Downloaded from https://www.cambridge.org/core. University of Edinburgh, on 19 Aug 2019 at 13:21:26, subject to the Cambridge Core terms of
use, available at https://www.cambridge.org/core/terms. https://doi.org/10.1017/9781108566100.012
512 Chapter 5b: Clinical Anaesthesia Answers

Question 285: FFTFT


The number of maternal deaths from thromboembolism has significantly reduced since the
Royal College of Obstetricians and Gynaecologists produced new guidelines in 2004. Better
identification of those at risk combined with better thromboprophylaxis has been the cause
of this drop. There are many risk factors but obesity seems to be the most important and
there are now specific guidelines published by the RCOG for the management of women
with obesity in pregnancy. In cases of suspected PE, investigations include V/Q scan or CT
pulmonary angiography (CTPA). In pregnant women there is an improved diagnostic rate
with V/Q scans than in non-pregnant women, and CTPA results can be inferior due to
difficulty interpreting them in the presence of increased cardiac output. Current guidelines
would suggest that 60 mg of enoxaparin is appropriate for thromboprophylaxis in a 92 kg
woman. Regional anaesthesia can usually be performed 12 hours after a prophylactic dose
and 24 hours after a treatment dose of heparin. If there is any doubt then discussion with
a haematologist is recommended.

Question 286: TTTFT


Paracetamol overdose is the most common cause of acute liver failure (ALF) in the UK.
In this scenario, the patient is suffering from hyperacute liver failure (HALF), defined as
encephalopathy occurring within seven days of the overdose. Cerebral oedema occurs
frequently with HALF, and ICP monitoring is necessary. Monitoring cerebral oxygenation
using jugular venous bulb oxygen saturation measurement guides further management.
Hypertonic saline infusion is sometimes indicated to maintain serum sodium level of
145–155 mmol.l–1. This helps to reduce cerebral oedema. Prophylactic antibiotics, as well
as antifungals, reduce the incidence of infection.

Question 287: TTFFT


Propofol is a useful agent for providing procedural sedation. Its rapid redistribution leads to
a rapid recovery. It can be administered as intermittent boluses or as a target-controlled
infusion, starting at 1.5 μg.ml–1 target plasma concentration and adjusting accordingly.
Co-administration of benzodiazepines or opioids is not contraindicated; however, the
effects are likely to be synergistic with propofol and therefore an exaggerated response
should be anticipated.

Question 288: FTTTT


A pedicle flap involves the surgical release of an area of tissue, which is then rotated around
its neurovascular pedicle, without interruption of blood flow. In a free flap the neurovas-
cular pedicle is removed from the donor site and transplanted to a new location with
microvascular anastomosis. Hence the flap will be denervated and have no intrinsic sympa-
thetic tone. Primary (as soon as blood flow stops during harvesting of the flap) and
secondary ischaemia (which occurs after flap transfer and reperfusion and is a result of
flap hypoperfusion) are risks of a free flap. Secondary ischaemia generally presents with
massive intravascular thrombosis and significant interstitial oedema and is more harmful to
the flap than primary ischaemia. Pain can contribute to this by causing a respiratory
alkalosis, decreasing plasma carbon dioxide levels and causing release of catecholamine,

Downloaded from https://www.cambridge.org/core. University of Edinburgh, on 19 Aug 2019 at 13:21:26, subject to the Cambridge Core terms of
use, available at https://www.cambridge.org/core/terms. https://doi.org/10.1017/9781108566100.012
Chapter 5b: Clinical Anaesthesia Answers 513

both of which cause vasoconstriction. Blood flow is usually reduced to half the original flow
and may take days or weeks to return to normal.

Question 289: FFTTF


To perform a sub-Tenon’s block, after ensuring the patient is consented, adequately mon-
itored and that resucitation equipment and drugs are available, 0.5% proxymetacaine or 0.4%
oxybuprocaine drops are given to provide topical anaesthesia to the eye. 5% povidone iodine is
used to clean the conjunctiva and the skin around the eye (10% povidone iodine has been
shown to be toxic to the cornea). After the placement of a lid speculum to keep the eye open,
the sub-Tenon’s injection is given as described (commonly using a blunt 19 G, 2.54 cm sub-
Tenon’s canula). Gentle constant pressure only is applied over the closed lid as massaging the
eye has been shown to increase intraocular pressure to up to 400 mmHg.

Question 290: FFTFF


This patient has evidence of extrinsic asthma: history of atopy and likely occupational
exacerbants (a GP/occupational health review is recommended for this). She is on step 3 of 5
of the stepwise management protocol according to revised BTS guidance. The quoted
incidence of bronchospasm under anaesthesia is approximately 2% with continuation of
their normal therapy. Ketamine does have bronchodilating properties, which makes it, for
some, the induction agent of choice in the management of life-threatening asthma.
However, ketamine’s side-effect profile makes it unsuitable for day-case anaesthesia.
The advised dose of magnesium in asthma is 2 g over 20 minutes.

Question 291: TTFFF


Although enteral feed is cheaper and more physiological, it is not without disadvantages.
Problems with absorption, high gastric residual volumes and vomiting are common.
Aspiration is always a risk with the nasogastric route. It neither eliminates the risk of
refeeding syndrome, nor reduces the incidence of hyperglycaemia.

Question 292: FTTTT


Laryngospasm is a complication that an anaesthetist should be confident managing and
identifying patients at risk will help to anticipate and manage this critical incident. Too light
a plane of anaesthesia risks precipitating laryngospasm. This occurs particularly at times of
stimulation, such as during instrumentation of the airway and surgical incision. Extubation
also provides a further opportunity for laryngospasm as the child lightens from anaesthesia.
Childhood obesity is associated with increased risk of airway complications. Smoking and
upper respiratory tract infections result in irritable airways with increased secretions.
Laryngospasm should be managed in the first instance with high-flow oxygen aided by
the application of positive airway pressure. Should this fail propofol boluses or neuromus-
cular blockade may be required.

Question 293: TTTFF


Endovascular aneurysm repair (EVAR) is a minimally invasive technique for the treatment
of abdominal aortic aneurysms. It involves a stent-graft being inserted through a small

Downloaded from https://www.cambridge.org/core. University of Edinburgh, on 19 Aug 2019 at 13:21:26, subject to the Cambridge Core terms of
use, available at https://www.cambridge.org/core/terms. https://doi.org/10.1017/9781108566100.012
514 Chapter 5b: Clinical Anaesthesia Answers

incision in the femoral artery and placed in position in the aorta under X-ray guidance. It is
recommended as a treatment option for patients with unruptured infrarenal abdominal
aortic aneurysms for whom surgical intervention is considered appropriate. The decision on
whether EVAR is preferred over open surgical repair should be made jointly by the patient
and their clinician after assessment of a number of factors, including aneurysm size and
morphology, and patient age, general life expectancy and fitness for open surgery. EVAR is
associated with reduced blood loss, reduced length of hospital and ITU stays, but increased
rates of reintervention and complications. Although two randomized controlled trials
found a reduction in 30-day mortality with EVAR compared to open repair there was no
significant difference in medium-term follow-up.

Question 294: FFFTT


One metabolic equivalent represents oxygen consumption of 3.5 ml.kg–1min–1.
In general, patients with a performance status less than 4 METs are considered to be
at an increased risk of morbidity and mortality perioperatively. The Canadian
Cardiovascular Society angina score is used to assess the severity of angina-type chest
pain (Table 5.294.1).

Table 5.294.1 The Canadian Cardiovascular Society angina score

Grade I – no limitation of ordinary activity, e.g. able to walk, climb stairs


Grade II – slight limitation of ordinary activity, e.g. activity limited after a heavy meal
Grade III – moderate limitation of ordinary activity, e.g. climbing one flight of stairs
Grade IV – marked limitation of ordinary activity, e.g. angina may be present at rest

Although the data relating to time frames for delaying elective surgery are perhaps out-
dated by the change in use of cardiac stents and β-blockers, the best advice remains that
elective surgery should be postponed for six months following an MI. Preoperative cardi-
opulmonary exercise testing can be advantageous in risk stratification in many patients, not
just those with ischaemic heart disease. An anaerobic threshold less than 11 ml.kg–1min–1 is
seen as the cut-off for major surgery and serious consideration should be given to whether
surgery can be postponed and the patient’s physical status improved.

Question 295: FFTFF


Although HFOV can improve oxygenation in patients with refractory hypoxaemia, this
is not translated into improved mortality or morbidity. Small tidal volumes that are
equal to or less than the dead space volume are used at a very high frequency (3–15
Hz). The aim is to keep the alveoli open throughout the respiratory cycle, thus reducing
the shearing effect of cyclical opening of alveoli during conventional IPPV. This will
usually be at the cost of hypercapnia and respiratory acidosis. It has been shown that
patients tolerate hypercarbia well and most clinicians do not pay much attention to the
paCO2 as long as pH ≥7.2. Hypotension is a common complication on starting HFOV,

Downloaded from https://www.cambridge.org/core. University of Edinburgh, on 19 Aug 2019 at 13:21:26, subject to the Cambridge Core terms of
use, available at https://www.cambridge.org/core/terms. https://doi.org/10.1017/9781108566100.012
Chapter 5b: Clinical Anaesthesia Answers 515

due to reduced venous return and it usually indicates that the patient is hypovolaemic.
Pneumothorax is another complication, which can be difficult to diagnose. The need for
heavy sedation and paralysis on HFOV is unnecessary and the patient can be allowed to
breathe spontaneously as long as they can synchronize with the HFOV. FiO2 ≤0.40 is
not sufficient to go back to conventional ventilation, unless the mean airway pressure is
less than 24 cmH2O as well.

Question 296: FFTTF


Post-dural-puncture headaches (PDPH) occur in 0.5–1.0% of women after an epidural
or spinal. Headaches usually occur within 72 hours, but most commence between
24 and 48 hours. Classically, the headache is frontal or occipital and increases on
sitting or standing. Associated symptoms can include nausea and vomiting, neck
stiffness, hearing and visual disturbance, and photophobia. Cranial nerve palsies have
also occurred in more rare cases. The headache is thought to be caused by leaking CSF
producing a drop in intracranial pressure and subsequently causing traction upon
intracranial structures. Management of PDPH can be conservative, pharmacological
or involve an epidural blood patch (EBP). Whilst bed rest does alleviate symptoms,
there is no evidence that IV fluids or caffeine are effective and they can cause significant
side effects. Gutsche’s test involves the patient lying at 45° and applying pressure to
their RUQ; temporary alleviation of the headache occurs due to the subsequent increase
in ICP. Synacthen has been trialled as a treatment, as it is thought to relieve pain
through increased CSF production. ACTH and 5-HT agonists have both been trialled,
but no evidence supports their use as yet. EBP was initially thought to have a success
rate of >90% but it is now thought to be nearer to 50% for a single procedure.

Question 297: TTTTT


In Addison’s disease there is destruction of the adrenal cortex, resulting in decreased or
absent secretion of glucocorticoids and decreased secretion of mineralocorticoids. There are
subsequent biochemical abnormalities, as detailed in the question.

Question 298: TTTFT


One in three women and one in twelve men will have osteoporosis over the age of 50 years.
Risk factors for women include lack of oestrogen, caused by early menopause (before age 45)
and early hysterectomy (before the age of 45), particularly when both ovaries are removed.
Risk factors for men include low levels of the male hormone, testosterone (hypogonadism).
For men and women: long-term use of high-dose corticosteroid tablets, family history of
osteoporosis (mother or father), and liver and thyroid disease. Other risk factors include
malabsorption problems (coeliac disease, Crohn’s disease, gastric surgery), long-term
immobility, heavy drinking or smoking. A bone density scan, called a dual-energy X-ray
absorptiometry (DXA) scan, is used to measure the density of bones. Bisphosphonates are
non-hormonal drugs that help maintain bone density and reduce fracture rates. Calcium
and vitamin D supplements can reduce the risk of hip fracture. Hormone replacement
therapy (HRT) is oestrogen replacement for women at the menopause, which helps main-
tain bone density and reduce fracture rates for the duration of therapy. Selective oestrogen
receptor modulators are drugs that help to maintain bone density and reduce fracture rates

Downloaded from https://www.cambridge.org/core. University of Edinburgh, on 19 Aug 2019 at 13:21:26, subject to the Cambridge Core terms of
use, available at https://www.cambridge.org/core/terms. https://doi.org/10.1017/9781108566100.012
516 Chapter 5b: Clinical Anaesthesia Answers

specifically at the spine. Testosterone therapy is testosterone replacement for men with low
testosterone levels to help maintain bone density.

Question 299: TTTFT


The management of complete heart block is a common Primary and Final question.
Given her history of ischaemic heart disease and no information given regarding
preoperative ECG it would be logical to assume an ischaemic cause.
The atrioventricular node is supplied by the right coronary artery in up to 90% of
people. According to the ALS bradyarrhythmia algorithm, this patient is displaying
adverse signs and so atropine should be given. Adrenaline or isoprenaline infusions
could also be considered in the patient’s management. There is no indication of
β-blocker overdose and so glucagon would not be useful in this case. As the thorax is
well out of the operative field it may be easier to establish transcutaneous pacing via
defibrillator pads to establish haemodynamic stability. Meanwhile, a discussion with
cardiology regarding the suitability for transvenous pacing can take place.

Question 300: FTTTF


Catheter fracture most often occurs from repeated ‘pinching’ between the clavicle and first
rib. The distal portion has the potential to embolize and cause damage to pulmonary or
cardiac structures. Chylothorax occurs following damage to the thoracic duct following
attempted siting of a CVC on the left-hand side. If significant it may cause haemodynamic
compromise; however, the damage would normally be evident at time of injury in this case.

Question 301: TFTTT


Potential and actual airway problems that could lead to obstruction are considered as
contraindications to conscious sedation in a child, these include: stridor, snoring, large
tongue and small mandible. Apnoeic episodes related to brain damage or drugs would also
be considered as a contraindication. Risk of aspiration of gastric contents, such as may occur
with abdominal distension and large NG aspirates, would be another contraindication.
In certain circumstances epilepsy would contraindicate conscious sedation, these would
include seizures requiring rectal diazepam within the last 24 hours or more than once in two
weeks, seizures related to major neurological disease or raised ICP, seizures within the last
four hours, failure to regain full consciousness following a recent seizure, seizures requiring
resuscitation and fitting related to sedation. Severe liver, metabolic or renal disease, and
severe respiratory disease or failure would contraindicate conscious sedation in a child.

Question 302: TTFTT


Sedation is defined as a state of reduced consciousness from which the patient can be roused.
By contrast, anaesthesia is a state from which the patient cannot be roused and often
includes loss of airway reflexes. Conscious sedation refers to the ability to maintain verbal
contact with the patient at all times, setting it apart from deep sedation or anaesthesia, when
verbal contact is lost. One of the primary aims of sedation is to ensure maintenance of
airway reflexes. However, sedation and anaesthesia form a spectrum and it is easy for
sedation to become anaesthesia, particularly in inexperienced hands.

Downloaded from https://www.cambridge.org/core. University of Edinburgh, on 19 Aug 2019 at 13:21:26, subject to the Cambridge Core terms of
use, available at https://www.cambridge.org/core/terms. https://doi.org/10.1017/9781108566100.012
Chapter 5b: Clinical Anaesthesia Answers 517

Question 303: TTFFT


Typically all babies younger than 30 weeks’ gestation or weighing less than 1.5 kg at birth are
screened for the condition. Certain high-risk babies weighing more than 1.5 kg who are
born after 30 weeks should also be screened.
In addition to prematurity, other risks factors may include:
• Neonatal apnoea
• Heart disease
• High pCO2
• Infection
• Low blood pH
• Low blood oxygen
• Respiratory distress
• Bradycardia
• Transfusions

Question 304: FFTTT


In 2000 a large literature search of PubMed and the Cochrane database used 141 studies
published between 1968 and 1999, and gave the following conclusions with regards to local
anaesthetic blocks for cataract surgery. Peribulbar and retrobulbar blocks produce equally
good akinesia and pain control during cataract surgery. There was moderate evidence
indicating superiority of pain control using subconjunctival/sub-Tenon’s approaches com-
pared with retrobulbar block. More recent reviews have also concluded that retrobulbar and
peribulbar blocks produce equally effective pain relief and akinesia, and sub-Tenon’s block
produces superior pain relief to topical anaesthesia for cataract surgery. A study by Parker
et al., comparing sub-Tenon’s and peribulbar blocks in 168 patients having cataract surgery,
found the sub-Tenon block less painful to perform, no significant difference in pain during
surgery, but better akinesia with a peribulbar block. The anaesthetic technique most likely to
cause severe complications is retrobulbar injection.

Question 305: TFTFF


In the context of burns and polytrauma, this patient must be assumed to have severe injuries
in addition to the obvious burn, and ketamine is an appropriate induction agent for these
cases. Immediate escharotomies are not required in this case as the burns are not circum-
ferential, but they may be required later if the burns contract and ventilation becomes
problematic. Cling film is an appropriate dressing as it prevents moisture loss, painful
blowing of air across the burn and visualization of the burn by medical staff without
repeated dressing and redressing. Pulse oximetry may be affected by the presence of carbon
monoxide and cyano compounds from the incomplete combustion of home furnishings, so
may over- or under-read. Immediate examination, assessment and management of trau-
matic injuries are the immediate clinical priorities, with burns care being done afterwards.

Question 306: FFFFF


Exomphalos is a congenital abnormality resulting from a failure of the intestines to migrate
back into the abdominal cavity during development. It is thought to be associated with

Downloaded from https://www.cambridge.org/core. University of Edinburgh, on 19 Aug 2019 at 13:21:26, subject to the Cambridge Core terms of
use, available at https://www.cambridge.org/core/terms. https://doi.org/10.1017/9781108566100.012
518 Chapter 5b: Clinical Anaesthesia Answers

a large umbilical ring in the fetus. The defect is not limited to intestines, as the liver, spleen
and ovaries are commonly found in the sac. The viscera are covered by a membrane
consisting of Wharton’s jelly, peritoneum and amnion. The incidence of exomphalos is
approximately 1:13 000 compared to 1:6000−1:10 000 live births with gastroschisis. Infants
with exomphalos are usually allowed to deliver at term unless there is some other associated
abnormality that requires preterm delivery. They should be delivered in a tertiary facility
with neonatal surgical facilities. NG tubes should be inserted to facilitate bowel decompres-
sion post delivery.

Question 307: TTFFF


Anaphylaxis to neuromuscular blocking drugs (NMBDs) is more common in Norway than
Sweden: this may be due in part to pholcodine, present in Norwegian but not Swedish cough
syrups, cross-sensitizing with NMBDs. A negative skin test does not eliminate the risk of
anaphylaxis, but guidelines recommend the use of that NMBD if one is essential.
Anaphylaxis to halogenated vapours has not been reported.

Question 308: FTFFF


The perioperative management of pacemakers differs in the emergency and elective setting.
Whilst for elective surgery the pacemaker function should be checked and, if appropriate,
deactivated prior to surgery, in an emergency this is seldom possible.
Safe management of a patient with a pacemaker includes a discussion with the surgeons
regarding the need for use of diathermy intraoperatively. Bipolar is safer than monopolar,
but if monopolar is required, the plate should be placed away from the pacemaker, ensuring
the diathermy and the plate do not surround the pacemaker. Diathermy should be used for
short bursts only. The use of magnets to deactivate pacemakers is outdated and may render
them non-functional, and as such should be avoided. Similarly, an alternative method of
pacing, such as transcutaneous pacing, should be available.

Question 309: TTTTT


It is almost impossible to differentiate between the different types of ICU-acquired weakness
(ICUAW) except by clinical investigation. Creatine kinase will be elevated in critical illness
myopathy (CIM) rather than in critical illness polyneuropathy (CIP). Muscle biopsy will be
abnormal in both types. In CIP, the muscle will show features of denervation and reinner-
vation. In CIM, the biopsy will show signs of degeneration. In CIP, a nerve biopsy is not
necessarily abnormal. In CIP, the nerve conduction studies will show a reduced compound
muscle action potential (CMAP) and a reduced sensory nerve action potential (SNAP), but
normal conduction velocities and latencies.

Question 310: TTFTT


Pre-eclampsia affects around 4.9 in every 10 000 pregnancies in the UK and is a significant
cause of morbidity and mortality. When diagnosed, the primary aims of treatment are to
control the blood pressure and to prevent progression to seizures. The first-line IV anti-
hypertensive is labetolol, but if this is ineffective, the patient is asthmatic or has cardiac
disease then hydralazine is the treatment of choice. Magnesium is used to prevent seizures

Downloaded from https://www.cambridge.org/core. University of Edinburgh, on 19 Aug 2019 at 13:21:26, subject to the Cambridge Core terms of
use, available at https://www.cambridge.org/core/terms. https://doi.org/10.1017/9781108566100.012
Chapter 5b: Clinical Anaesthesia Answers 519

and is usually given as a loading dose of 4 g followed by an infusion of 1 g.h–1. Plasma levels
should be monitored, as toxic levels can have serious consequences. Levels >2.5 mmol.l–1
produce ECG changes, >5 mmol.l–1 cause loss of tendon reflexes, >7 mmol.l–1 cause
respiratory arrest and >12 mmol.l–1 can produce cardiac arrest. Calcium gluconate 10%
10 ml IV would be an initial treatment for toxicity followed by diuresis to excrete the
magnesium.

Question 311: FTFFT


The Jehovah’s’ Witness religion is a Christian movement founded in the North Eastern USA
in the 1870s. Jehovah’s Witnesses interpret the Bible literally, and thus have strong beliefs
regarding the rejection of blood transfusions and certain other blood products.
The acceptability of certain products and procedures has changed with time, such that at
the time of writing the below (Table 5.311.1) are generally held to be true.
Table 5.311.1

Generally not accepted May be accepted (‘matters Generally accepted


of conscience’)

Whole blood Red cell fractions Cardiopulmonary bypass


Packed red cells Haemoglobin (human, animal Renal dialysis
Plasma or synthetic, e.g haemopure) Acute hypervolaemic
Autologous predonation Platelet factor 4 haemodilution
White cells White cell fractions Recombinant erythropoietin
Platelets Interferons Recombinant factor VIIa
Interleukins Vaccinations
Plasma fractions
Albumin
Immunoglobulins
Cryoprecipitate
Clotting factors
Epidural blood patch
Cell saver

Question 312: TFFFT


Dementia is a condition in which there is a gradual loss of brain function; it is a decline in
cognitive/intellectual functioning. There are four main symptoms: loss of memory – parti-
cularly short-term memory, problems with speech and understanding, changes in person-
ality and behaviour, and difficulty with skills. Short-term memory is affected before long-
term memory in people with dementia. Long-term memory is affected in more severe forms
of dementia. Dementia is more common with age (approximately 20% of people over age 80
have dementia), but it is not normal with age. Many of the causes of dementia are associated
with a failure of cholinergic transmission; therefore, patients are very sensitive to anti-
cholinergic drugs. Although difficulties with speech and language can be a feature of
dementia, they are usually slowly progressive. Change in personality is a feature of dementia

Downloaded from https://www.cambridge.org/core. University of Edinburgh, on 19 Aug 2019 at 13:21:26, subject to the Cambridge Core terms of
use, available at https://www.cambridge.org/core/terms. https://doi.org/10.1017/9781108566100.012
520 Chapter 5b: Clinical Anaesthesia Answers

and is a sign of frontal lobe involvement. Agitation can occur in both delirium and
dementia – only the speed of onset is of assistance in differentiating delirium from
dementia.

Question 313: TFFFF


There are a number of definitions for massive transfusion, including statement
a. The context of the situation should guide whether your hospital’s massive transfusion
policy is activated. This woman is already bleeding heavily and will need operative delivery,
and so should trigger a massive transfusion policy. Baseline FBC/clotting profiles should be
taken with repeat samples and sent regularly to the blood bank to help guide transfusion
requirements. AAGBI guidance recommends targeting a platelet count of >75 × 109 l–1. This
woman needs urgent delivery, but resuscitation should be started before induction, as this
may precipitate hypovolaemic cardiac arrest with loss of sympathetic tone. Cell salvage is
not contraindicated in obstetric cases and is endorsed by NICE guidance. Leucocyte
depletion filters should be used, with a separated suction system for liquor.

Question 314: TTFTT


The epidural space is bordered inferiorly by the sacrococcygeal ligament, laterally by the
pedicles and intervertebral foraminae, posteriorly by ligamentum flavum, facet joints and
laminae, and anteriorly by the posterior longitudinal ligament, vertebral body and discs.
It contains:
• Fat
• Paired spinal nerves
• Dural sac (ending at approximately S2)
• Blood vessels (venous plexus and spinal arteries)
• Lymphatics
• Connective tissue

Question 315: FFTFT


Conscious sedation is defined as: ‘drug-induced depression of consciousness, similar to
moderate sedation, except verbal contact with the patient is always maintained’. For
painless imaging in children weighing less than 15 kg, oral chloral hydrate up to a dose
of 100 mg.kg–1 is suitable; IV cannulation is not necessary with this oral sedation regime.
Fasting is not required for sedation with nitrous oxide or conscious sedation, which
utilizes a technique with a wide margin of safety. Ketamine, at a dose of 2 mg.kg–1 IV or
5–10 mg.kg–1 IM, is a suitable technique for use in the ER for painful procedures. At this
dose, airway reflexes are usually maintained and recovery is expected within 90 minutes.
For dental procedures, conscious sedation can be administered by nasal nitrous oxide or
by IV midazolam, at a starting dose of 25–50 μg.kg–1, titrated to effect. Both of these
techniques require the co-operation of the child, so if they are unsuccessful, advanced
sedation techniques or general anaesthesia will be required.

Downloaded from https://www.cambridge.org/core. University of Edinburgh, on 19 Aug 2019 at 13:21:26, subject to the Cambridge Core terms of
use, available at https://www.cambridge.org/core/terms. https://doi.org/10.1017/9781108566100.012
Chapter 5b: Clinical Anaesthesia Answers 521

Question 316: TTTFT


The fetus cannot make glucose from glycogen and is wholly dependent on a supply of
glucose via the placenta. During the first few days of life, there is a risk of:
• Hypoglycaemia
• Hypocalcaemia
• Retinopathy of prematurity
• Hirschsprung’s disease

Question 317: TTFFF


Six electrodes are placed on the patient’s chest and neck. The resistance to current flowing
from the outermost to innermost electrodes is measured. The bioimpedance is indirectly
proportional to the total volume of thoracic fluid. Changes in cardiac output will change the
volume of aortic blood and will be reflected in a change in transthoracic electrical bioimpe-
dance. Interference by diathermy may limit its intraoperative use. Transthoracic electrical
bioimpedance is also very sensitive to movement and therefore unlikely to be accurate in
awake patients.

Question 318: FTFTT


Suppression of the hypothalamopituitary axis has been reported. Clinically this manifests
itself as reduced libido and erectile dysfunction. Opioid-induced immunosuppression has
been recognized. Morphine, fentanyl, remifentanil and, to a lesser extent, methadone have
been shown to possess significant immunosuppressive activities. The most prevalent effects
of this have been reported as increased susceptibility to infection after surgery or trauma,
decreased tolerance to malignant disease, with increased likelihood of secondary spread and
an increased chance of HIV infection in drug abusers.

Question 319: TFFTF


Acute spinal cord injury is associated with head injuries in a quarter to two-thirds of cases.
The population most susceptible to spinal cord injury are men aged 15–35 and the most
frequent cause is road traffic accidents. Various systemic physiological responses can
ensue in the weeks following acute spinal cord injury as follows. In the immediate
postinjury period, hypertension, arrhythmias and peripheral vasoconstriction are seen.
Sympathetic disruption with ‘spinal shock’ may then ensue if the cardioaccelerator fibres
(lesions above T1) are disrupted, producing bradycardia and hypotension (lesions above
T6). Autonomic hyper-reflexia typically occurs at an interval of four to six weeks post
injury. Autonomic hyper-reflexia is characterized by an exaggerated autonomic response
to seemingly innocuous stimuli below the level of the lesion, e.g. urinary catheterization.
Flaccid paralysis is seen for up to two to three weeks post injury and is followed by spastic
paralysis. Central cord syndrome results in motor weakness affecting the arms to a greater
extent than the legs.

Downloaded from https://www.cambridge.org/core. University of Edinburgh, on 19 Aug 2019 at 13:21:26, subject to the Cambridge Core terms of
use, available at https://www.cambridge.org/core/terms. https://doi.org/10.1017/9781108566100.012
522 Chapter 5b: Clinical Anaesthesia Answers

Question 320: TFTFF


There are many perioperative anaesthetic factors that can have an influence on the success
or failure of flap surgery. The patient should be kept normothermic with a core to peripheral
difference of <1 °C. The ambient temperature in theatre should ideally be 22–24 °C.
Invasive arterial monitoring is recommended for free-flap surgery as it allows precise and
continuous monitoring of arterial pressure, as well as blood gases, with haematocrit
measurement. Central venous pressure monitoring can be used for cardiac output manage-
ment, but is not recommended as routine. Vasopressors are generally avoided, although
there is little evidence that systemic administration has adverse effects on flap blood flow.
Hypotension is usually secondary to fluid loss or vasodilatation and should be treated with
fluid therapy, although studies of dobutamine used in pedicle flaps in breast surgery have
shown an increase in flap blood flow.
Goals for flap surgery:
• Normothermia (Δt <1 °C)
• Hyperdynamic circulation – high cardiac output and low systemic vascular resistance
• Normal systolic blood pressure (>100 mmHg)
• Hematocrit 30%
• Urine output >1 ml.kg–1.h–1
• SpO2 >94% for the first 24 hours
• Effective analgesia

Question 321: FFTTF


Although there is a higher rate of congenital cardiac disease in Down’s syndrome, desflur-
ane is not contraindicated and nitrous oxide has the same contraindications as in the rest of
the population. Unless a patent foramen ovale has been diagnosed or is suspected, nitrous
oxide is as safe to use in Down’s syndrome as the rest of the population. According to the
Mental Health and Capacity act, everyone should be presumed able to consent until proven
otherwise, however Down’s syndrome patients by definition are not able to consent.
Consent should be documented on Form Four, though every effort should be made to
explain the procedure to the patient as a matter of good practice. There is no increase in the
rate of malignant hyperthermia, and regional techniques can be used at the discretion of the
anaesthetist, but usually as an adjunct to general anaesthesia.

Question 322: FTFFF


Left ventricular work index is increased, but myocardial oxygen consumption remains
largely unchanged. Cardiac index is increased, heart rate is largely unchanged and both
systemic and pulmonary vascular resistances fall. It is levosimendan that increases troponin
C sensitivity to calcium (amongst other actions).

Question 323: TFTTF


Hypokalaemia may predispose to the development of rapid ventricular rate with resulting
compromise of blood pressure and haemodynamic instability. A potassium level lower
than the laboratory reference range needs to be corrected prior to surgery; at a level as low
as 2 mmol.l–1 the patient is likely to require intravenous supplementation and appropriate

Downloaded from https://www.cambridge.org/core. University of Edinburgh, on 19 Aug 2019 at 13:21:26, subject to the Cambridge Core terms of
use, available at https://www.cambridge.org/core/terms. https://doi.org/10.1017/9781108566100.012
Chapter 5b: Clinical Anaesthesia Answers 523

cardiac monitoring. The presence of a right bundle branch block requires no specific
treatment per se. However, a general enquiry as to the patient’s functional status and
a history of ischaemic heart disease should be explored. A slow ventricular rate in an
unmedicated patient with AF constitutes a concerning feature. Elective surgery should be
postponed and the patient referred to cardiology for consideration of a pacemaker.
Patients with atrial fibrillation should take their usual rate-controlling medication prior
to elective surgery to reduce the risk of the development of fast AF in the perioperative
period. Patients with persistent AF are likely to be receiving an antiplatelet or anti-
coagulant agent. Whilst it is necessary to stop warfarin preoperatively in many circum-
stances, aspirin can often be continued and so its administration is not a contraindication
to proceeding with surgery.

Question 324: FTFFT


Two recent large randomized controlled trials, the ATN trial and the RENAL trial, have
both failed to show a benefit from using a flow rate more than 25 ml.kg−1.h−1. The optimal
timing for initiating RRT is still controversial. Although early initiation of RRT is preferred
by many intensivists, there is no good evidence to support it. Phenytoin is a highly protein-
bound drug and hence is not affected by RRT unless the patient’s protein level is low enough
to result in an increase in the free fraction of the drug. The replacement fluid can either be
infused through the arterial site, i.e. prefilter (predilution method), or through the venous
site, i.e. postfilter (postdilution method). The predilution technique can help in flushing the
haemofilter and diluting the blood, thus reducing the risk of clot formation. However, this
will be at the expense of reducing solute clearance and an increase in flow rates may be
needed.

Question 325: FFFTF


Dental damage under anaesthesia is the most common source of complaint and litigation
encountered by anaesthetists. The Royal College of Anaesthetists’ guidance quotes a risk of
1:4500. There is no evidence of an increased risk during emergency intubation, although the
risk is 20 times greater in patients deemed difficult to intubate (Table 5.325.1). Maxillary
incisors are the most commonly damaged. The left central maxillary incisor is most
vulnerable from the flange of the laryngoscope blade if it is used as a fulcrum.

Table 5.325.1 Risk factors for dental damage

Anaesthetic factors Dental factors

Difficult airway predictors: Any restorative dental work


• Prominent incisors Pre-existing dental disease
• Mallampati 3/4 Periodontitis
• Reduced C-spine movement Isolated teeth
• Receding mandible Mixed dentition (children)
• BMI >35
Direct laryngoscopy
Placement of double-lumen tubes
Suctioning of oropharynx

Downloaded from https://www.cambridge.org/core. University of Edinburgh, on 19 Aug 2019 at 13:21:26, subject to the Cambridge Core terms of
use, available at https://www.cambridge.org/core/terms. https://doi.org/10.1017/9781108566100.012
524 Chapter 5b: Clinical Anaesthesia Answers

Question 326: FFTFT


The woman in question has had a primary postpartum haemorrhage (PPH) and may have
had significant blood loss. The most likely cause of a primary PPH is uterine atony (>75%).
Other causes include retained tissue, coagulopathy and trauma. The so-called 4 Ts are Tone,
Tissue, Thrombin and Trauma. She could lose up to 30–40% of her circulating volume
before becoming hypotensive, with tachycardia the only abnormal sign. Careful assessment
of the patient’s cardiovascular status should be performed. Uterotonic drugs and fluids
would be an appropriate choice in most cases as first-line treatment. If non-responsive to
this then she would probably need to go to theatre for further investigation and treatment.
Whilst arterial monitoring would probably be advantageous it should not delay definitive
treatment in theatre. At least two wide-bore cannulae should be in place and an urgent
blood cross-match performed (O– may be required if there is likely to be a delay). The safest
form of anaesthetic in this situation would be a general anaesthetic, as she may be
significantly hypovolaemic or may have an underlying coagulopathy; there may also be
a prolonged surgical time depending on the action required.

Question 327: TFFFF


This is a difficult legal area. A Gillick-competent child may consent to a blood transfusion,
despite the wishes of their parents. Children may also refuse, however the courts may over-
rule such refusal. The Jehovah’s Witness parents may refuse a blood transfusion or proce-
dure. In an elective situation where blood transfusion may be needed, a Specific Issue Order
may be obtained via the High Court. This order allows transfusion to proceed without
removing all parental authority. In an emergency situation where parents refuse life-saving
treatment, failure to give such treatment to a child may lead to criminal prosecution of the
doctor. A second medical opinion should be sought, and all discussions and decisions
documented.

Question 328: FFFTT


Electroconvulsant therapy (ECT) is used in the treatment of severe depression, mania and
catatonia. The aim of ECT is to induce a generalized seizure lasting between 15 and 120
seconds and recorded using electroencephalography. Two electrodes are applied either
bilaterally or unilaterally, the former being the more common. Relative contraindications
include myocardial ischaemia, recent cerebrovascular event, deep vein thrombosis (until
anticoagulated), raised intracranial pressure and severe osteoporosis. The presence of
a permanent pacemaker incurs a low risk, though internal defibrillators should be deacti-
vated. Electroconvulsant therapy is considered safe in pregnancy.

Question 329: TFTFF


The spinal nerve divides into a dorsal and a ventral ramus. The dorsal rami are not blocked
by intercostal nerve blocks. Interpleural blocks are effective in cholecystectomy, but not
thoracotomies. Cryoanalgesia causes chronic post-thoracotomy pain and hence is not
recommended. The paravertebral space is not well defined superiorly, but inferiorly defined
by the psoas major muscle.

Downloaded from https://www.cambridge.org/core. University of Edinburgh, on 19 Aug 2019 at 13:21:26, subject to the Cambridge Core terms of
use, available at https://www.cambridge.org/core/terms. https://doi.org/10.1017/9781108566100.012
Chapter 5b: Clinical Anaesthesia Answers 525

Question 330: FTFTT


The standard adhered to whilst undertaking sedation is analogous to that when performing
a general anaesthetic. Starvation times are identical for both procedures and full resuscita-
tion equipment should be immediately available at all times, including during the recovery
period. Monitoring throughout the procedure should be that of the minimal mandatory
monitoring set as determined by the AAGBI, that is ECG, oxygen saturations and blood
pressure. It is important to counsel the patient as to what to expect during the procedure:
what they are likely to remember, what they can expect to feel and whether they will be
expected to perform any tasks. It is also important to maintain verbal contact and reiterate
these points during the procedure to avoid unexpected movement at a critical point.

Question 331: TFTTT


A tracheostomy placed above the second tracheal ring is more likely to result in subglottic
stenosis and therefore most operators try to aim between the second and third, or the third
and fourth tracheal rings. A multiple dilator technique was common practice a decade ago.
However, now the technique of choice consists of a one-step dilation using a curved dilator
with a hydrophilic coating. Some operators prefer using a longitudinal incision, as it may
reduce the risk of bleeding, however either incision is acceptable practice. If there is no risk
of aspiration, some intensivists prefer to change the ETT to an LMA rather than with-
drawing the ETT and risking compromise of the airway during the procedure.

Question 332: FTTFF


Osteoarthritis is a degenerative process that affects articular cartilage, resulting from joint
trauma due to biochemical stresses, joint injury, neuropathy or muscle atrophy. One or
several joints can be affected. Pain is usually present on motion, but is relieved by rest.
The knees and hips are the most common sites of involvement. Bony enlargements called
Heberden’s nodes are seen at the distal interphalangeal joints of the fingers. There may be
degenerative changes in the vertebral bodies and the intervertebral discs, which can be
complicated by protrusion of the nucleus pulposus and compression of the nerve roots.
Degenerative changes are most significant in the mid to lower cervical spine and in the lower
lumbar area. Maintaining muscle function is important for cartilage integrity and pain
reduction. Pain relief can be achieved by application of heat, simple analgesics and anti-
inflammatory drugs. Transcutaneous nerve stimulation and acupuncture can be effective in
some patients. There is minimal inflammatory reaction in the joints and systemic cortico-
steroids have no place in the treatment of osteoarthritis. Joint replacement surgery is
recommended when pain is persistent and disabling, or significant limitation of joint
function is present.

Question 333: FFTFF


In venous air embolism the bubble can cause right ventricular outflow tract obstruction.
Smaller air emboli reaching the lung precipitate an inflammatory response. Paradoxical
movement of air from the venous circulation to the arterial is high risk in a quarter of
patients with patent foramen ovale, but can also occur via transpulmonary gas shunting.
Tissues that are especially sensitive to hypoxia, e.g. brain, are at high risk in these

Downloaded from https://www.cambridge.org/core. University of Edinburgh, on 19 Aug 2019 at 13:21:26, subject to the Cambridge Core terms of
use, available at https://www.cambridge.org/core/terms. https://doi.org/10.1017/9781108566100.012
526 Chapter 5b: Clinical Anaesthesia Answers

circumstances, with ischaemia resulting in oedema. This may result in delayed recovery
from anaesthesia and postoperative cognitive dysfunction. The Valsalva manoeuvre would
be useful in this case as neurosurgery in the sitting position may not be amenable to
positioning the patient in the left lateral position. The Valsalva manoeuvre will increase
venous pressure, reducing air entrainment, which, together with flooding the field with
saline and surgical compression, helps minimize further air entrainment. Management is
then supportive.

Question 334: FTFTF


CVC are not suitable for rapid fluid administration due to the fact that the lumens are small
and long and should not be chosen as a first-line device in times of fluid resuscitation.
The commonly used sedative drugs can be given via peripheral cannulae and are not an
independent indication for siting a CVC. A central with its distal lumen in the superior vena
cava will aspirate central venous blood, mixed venous blood can only be sampled in the
pulmonary artery.

Question 335: TTFFF


Signs of severe LA toxicity include severe agitation or loss of consciousness, change in
mental state, tonic–clonic seizures, cardiovascular collapse and arrhythmias (sinus brady-
cardia, conduction blocks, asystole and ventricular tachyarrhythmias). The Association of
Anaesthetists of Great Britain and Ireland have produced a guideline detailing the manage-
ment of severe LA toxicity. A good candidate should be familiar with the initial manage-
ment and the importance of administration of lipid emulsion.

Question 336: FTFFF


This patient is receiving clopidogrel, which makes him inappropriate for central neuraxial
blockade before seven days from stopping his medication. The use of a peripheral nerve
block is relatively safe in this case and will reduce the amount of opioids used periopera-
tively. Femoral and sciatic nerve blocks only are not enough for this surgery, as the lateral
cutaneous nerve of the thigh would not be covered. Postponing the surgery for 24 hours to
optimize heart rate control may be appropriate, however a delay of seven days would most
likely worsen outcome in this group of patients.

Question 337: FFFTF


Chronic postsurgical pain (CPSP) is a significant problem following surgery. The incidence
of CPSP varies between different surgical procedures. There is no formal definition, but the
following criteria should be met:
• The pain must develop after a surgical procedure
• The pain is of at least two months’ duration
• Other causes for the pain have been excluded
• The possibility that the pain is from a pre-existing condition has been excluded
The incidence of CPSP for various procedures is:

Downloaded from https://www.cambridge.org/core. University of Edinburgh, on 19 Aug 2019 at 13:21:26, subject to the Cambridge Core terms of
use, available at https://www.cambridge.org/core/terms. https://doi.org/10.1017/9781108566100.012
Chapter 5b: Clinical Anaesthesia Answers 527

• Amputation 30–85%
• Thoracotomy 5–67%
• Mastectomy 1–57%
Risk factors for the development of CPSP are:
• Preoperative pain
• Younger age
• Psychological factors
• Surgical technique
• Acute postoperative pain
• Genetic predisposition
The results of pre-emptive analgesia with, for example, gabapentinoids are inconclusive.
In breast cancer management, adjuvant treatment with chemotherapy and radiotherapy
can itself be associated with the development of neuropathic pain.

Question 338: FTTTF


An abdominal aortic aneurysm (AAA) is an enlargement of the aorta to at least 1.5 times its
normal diameter or greater than 3 cm diameter in total. The main risk factors for AAA include
increasing age, high blood pressure, smoking and family history of AAA. NICE guidance 167
recommends elective surgery for patients with aneurysms greater than 5.5 cm in diameter and
with aneurysms larger than 4.5 cm in diameter that have increased in size by more than 0.5 cm
in the past six months. Patients with aneurysms of less than 5.5 cm should be followed up with
ultrasonographic surveillance. In 2011 the European Society for Vascular Surgery issued
guidance on the management of AAAs and made the following recommendations:
• Females should be considered for surgery when the maximum aortic diameter reaches
5.2 cm.
• All symptomatic patients should have their aneurysm repaired on the next available
elective operating list and should be offered endovascular aneurysm repair (EVAR), which
has a lower operative mortality for symptomatic aneurysms than open repair.
• Those with aneurysms greater than 9 cm should be considered for inpatient treatment.
A national screening programme has been implemented covering men aged over 65 years.

Question 339: FFFTT


Pyloric stenosis is the commonest surgical presentation within the first six months of life
and is a medical as opposed to a surgical emergency. The child should be adequately
resuscitated prior to surgery, with particular attention paid to fluid status, serum electro-
lytes (especially K+ and Cl–) and pH. Vomiting leads to the loss of H+, Cl– and K+ ions from
the stomach, as well as dehydration. The kidney attempts to retain water by the
renin–aldosterone–angiotensin system, which causes the retention of sodium at the expense
of renal potassium loss. The kidney also attempts to retain H+ ions to restore a normal
plasma pH, and this also is at the expense of K+. Hypokalaemia forces Na+ to exchange
preferentially for H+, which causes the paradoxical acidic urine in the presence of plasma
alkalaemia. Only when total body Cl– is restored is there sufficient Cl–concentration in the
glomerular filtrate to permit tubular exchange with HCO3– and excretion of bicarbonate
in the urine, which will restore the serum pH to normal. When the urinary sodium

Downloaded from https://www.cambridge.org/core. University of Edinburgh, on 19 Aug 2019 at 13:21:26, subject to the Cambridge Core terms of
use, available at https://www.cambridge.org/core/terms. https://doi.org/10.1017/9781108566100.012
528 Chapter 5b: Clinical Anaesthesia Answers

is >20 mmol.l–1, adequate ECF equilibration is said to have occurred. The typical picture is
a hypochloraemic metabolic alkalosis with hypokalaemia. As the gastric outlet rather than
inlet is obstructed, NG insertion should not be any more difficult than normal and is
mandated so that the stomach can be drained prior to induction of anaesthesia.

Question 340: TFFTF


Although venoconstriction may play a role in noradrenaline’s actions, inadequate fluid
resuscitation will lead to reduced cardiac output and myocardial ischaemia. There is robust
evidence to discount low-dose dopamine regimes and these should not be used.
Dobutamine may be effective in improving tissue perfusion if there is cardiac dysfunction
and low cardiac index. The target MAP recommended is >65 mmHg.

Question 341: TTTTT


Third-degree heart block is seen when there is complete dissociation of conduction between
the atria and the ventricles. It can be transient, in response to profound vagal stimulation, or
as a result of ischaemia, either to the atrioventricular node or the bundle of His. It is often
chronic and in this instance, insertion of a permanent pacemaker is likely to be necessary.
If surgery is urgent, consideration needs to be given to commencing an isoprenaline
infusion, inserting a transvenous pacing wire or performing transcutaneous pacing,
depending on the circumstances.

Question 342: TTFFF


Cytochrome P450 (CYP) enzymes use haem to enable oxidation and as such are part of phase 1
metabolism in the liver. There are numerous isoforms of CYP, which are divided into families
(identified by a number, e.g. CYP2), subfamilies (identified by a letter e.g. CYP2D) and individual
genes (identified by a number, e.g. CYP2D6). Activity of certain CYP varies significantly between
people due to genetic polymorphism and hence may have profound clinical consequences,
especially when multiple drugs are given to a patient. Polymorphisms (especially in CYP2D6)
may mean that certain drugs are ineffective in people (e.g. antidepressants and neuroleptics)
whilst in some prodrugs will be extensively activated (e.g. codeine forming morphine). Most
CYPs, especially 3A4, can be induced (the notable exception is 2D6). Common inducers are
antimicrobials (rifampicin), anticonvulsants (carbamazepine, phenytoin, phenobarbitone), sul-
fonylureas and St John’s wort. Similarly, many agents inhibit CYP – HIV protease inhibitors,
isoniazid, valproate, keto/fluconazole, erythromycin, sulfonamides, metronidazole, omeprazole
and grapefruit juice. A vast number of drugs are metabolized by CYP3A4 (e.g. opioids, benzo-
diazepines, local anaesthetics, haloperidol and the oral contraceptive pill). Combining these
drugs with inducers or inhibitors of the enzyme may have profound effects.

Question 343: FFTFT


The best management of hyperglycaemia in ICU has been the subject of debate over the last
two decades. Recent large trials have shown no benefit for tight glycaemic control (blood
glucose (BG) = 4–8 mmol l–1) over conventional control (BG <10 mmol l-1). NICE-
SUGARE, the largest trial to date, has shown a significant increase in the incidence of
hypoglycaemia in the tight control group. Most intensivists now target a BG <10 mmol l–1

Downloaded from https://www.cambridge.org/core. University of Edinburgh, on 19 Aug 2019 at 13:21:26, subject to the Cambridge Core terms of
use, available at https://www.cambridge.org/core/terms. https://doi.org/10.1017/9781108566100.012
Chapter 5b: Clinical Anaesthesia Answers 529

for most groups of critically ill patients. One exception may be post-cardiac-surgery
patients, where several units still prefer a tighter control. Capillary measurement of BG
can be inaccurate, especially in a hypoperfused state. Patients with well-established diabetes
lose the normal counter-regulatory responses to hypoglycaemia. This is analogous to the
management of hypertensive crisis. Wide variability in BG levels leads to higher oxidative
stress than sustained hyperglycaemia.

Question 344: FFFTT


Warfarin is the most effective treatment to prevent mechanical valve thrombus formation; it
is, however, able to cross the placenta and is teratogenic. It is therefore usually avoided until
the 13th week of pregnancy with heparin cover until this time. It can then be used until the
third trimester when the woman will often be swapped back on to heparin several weeks
before her due date. Neither form of heparin (unfractionated or low molecular weight)
crosses the placenta and they are generally safe to use during the whole of the pregnancy.
It is possible for either of them to be used as the sole agent throughout. Stillbirth and
miscarriage rates are the same whichever choice of thromboprophylaxis is used. If heparin is
stopped with sufficient time (depending on dose) then an epidural may be sited, however the
period without anticoagulant cover must be minimized to prevent thrombus formation.
Discussion with obstetricians, haematologists and cardiologists is advisable in advance to
create an appropriate management plan.

Question 345: FFFTF


The classic tetrad of clinical features associated with Parkinson’s disease are bradykinesia,
rigidity, tremor and postural instability.

Question 346: FTTTT


The dose requirement for induction agents is reduced in the elderly. A contracted blood
volume coupled with reduced protein binding leads to a higher free-drug concentration.
Prolongation of arm–brain circulation time dictates that induction agents should be
administered more slowly. Failure to do this leads to inadvertent overdose, often with
marked cardiorespiratory side effects. Virtually all opioids, IV agents and benzodiazepines
exhibit an age-related increase in their elimination half-life, resulting in a prolonged dura-
tion of action. This is attributable to an increased volume of distribution for lipophilic drugs
because of the increase in body lipid content in the elderly, and a reduction in organ-based
elimination. The MAC value of all inhalational anaesthetic agents is reduced by 20–40%
from young adult values. Although ageing is associated with a reduction in muscle mass, the
development of extrajunctional cholinergic receptors offsets the reduction in the expected
dosage of neuromuscular blocking agents required to produce acceptable intubating con-
ditions. However, the time of onset and the duration of action are both prolonged because of
a reduction in cardiac output and reduced metabolism. Ageing is associated with
a reduction in the carotid baroreceptor response to a fall in blood pressure. Both IV and
inhalational anaesthetic agents further impair this response, and also depress cardiac and
vascular smooth muscle contractility.

Downloaded from https://www.cambridge.org/core. University of Edinburgh, on 19 Aug 2019 at 13:21:26, subject to the Cambridge Core terms of
use, available at https://www.cambridge.org/core/terms. https://doi.org/10.1017/9781108566100.012
530 Chapter 5b: Clinical Anaesthesia Answers

Question 347: TFTFF


Upper respiratory tract infections may range from mild viral infections with predominantly
coryzal symptoms to severe bacterial infections (rarer since immunization programmes and
effective antimicrobials). An anaesthetist must have an awareness of upper respiratory tract
infections, causes and their management, as well as potential implications for anaesthesia.
Haemophilus influenzae B causes epiglottitis and can result in severe sepsis requiring
resuscitation once the airway is safely secured. Incidence has greatly reduced in the UK
following effective immunization programmes. Adenoviruses and parainfluenza viruses can
also cause upper airway inflammation and irritation. Retropharyngeal abscess is rare in the
UK, but still relatively common in the developing world, most commonly due to Group
A Streptococcus and Staphylococcus aureus. Croup is managed supportively with humidified
oxygen, nebulized adrenaline and intravenous or oral dexamethasone or nebulized budeno-
side. Ventilatory support is required in less than 5% of cases of croup.

Question 348: TFTTF


Table 5.348.1 Descriptions of needles

Quinke: Short bevel, cutting tip with distal orifice


Hustead: Modified Tuohy; shorter, more angulated and less sharp bevel
Sprotte: ‘Atraumatic’ modified pencil-point tip with larger lateral orifice
Whitacre: Elongated pencil-point tip with lateral orifice
Weiss: Modified Tuohy; shortened bevel with addition of T-shaped wings

Question 349: TTFTT


The classic LMA (cLMA) produces a relatively low pressure seal with the pharynx (median
20 cmH2O), thus limiting the use of this device in situations where there is a risk of
regurgitation of gastric contents and when controlled ventilation is required with high
inflation pressures. The i-Gel is a second-generation supraglottic airway device (SAD) that
was launched in 2007. It is made of a soft, gel-like polymer, does not require inflation and is
designed to accurately fit the perilaryngeal anatomy. It is a single-use device, possesses
a narrow-bore oesophageal drain tube and an integral bite-block. When positioned cor-
rectly, the i-Gel does not penetrate as far into the oesophageal inlet as with other LMAs, and
therefore its oesophageal seal is relatively low. The ProSeal LMA is another second-
generation SAD; it consists of a posterior inflatable cuff, a reinforced airway tube, an
oesophageal drain tube, an integral bite-block and an introducer. When positioned cor-
rectly the tip of the oesophageal drain tube (which forms the tip of the ProSeal LMA) sits
over the oesophageal inlet, forming a high-pressure oesophageal seal. Paediatric sizes of the
ProSeal LMA are available (1.5–2.5).

Downloaded from https://www.cambridge.org/core. University of Edinburgh, on 19 Aug 2019 at 13:21:26, subject to the Cambridge Core terms of
use, available at https://www.cambridge.org/core/terms. https://doi.org/10.1017/9781108566100.012
Chapter 5b: Clinical Anaesthesia Answers 531

Question 350: TFTFT


Risk factors for neonatal hypocalcaemia are:
• Low birth weight and prematurity
• Intrauterine growth retardation
• Infants of insulin-dependent diabetics
• Infants with birth asphyxia associated with prolonged, difficult deliveries
Late neonatal hypocalcaemia occurring five to ten days after birth is usually due to ingestion
of cow’s milk, which contains high levels of phosphorus. It is not seen in breast-fed infants
because human breast milk has a lower phosphate content. The high phosphate content of
cow’s milk, in conjunction with decreased phosphate excretion associated with renal
immaturity, can cause hyperphosphataemia and secondary hypocalcaemia in the neonate.
Electrocardiographic evaluation may show prolongation of the QT interval or heart block in
severe cases. Oral calcium therapy is used in asymptomatic patients and as follow-up to
intravenous (IV) calcium therapy. IV treatment is usually indicated in patients having
seizures, those who are critically ill, and those who are planning to have surgery.
Intraoperative hypotension is a prompting sign to correct hypocalcaemia on the table.

Question 351: FFFFT


The normal axial globe length in an adult is <25 mm. Patients with axial myopia who have
eye blocks have a greater risk of globe perforation than those with a normal axial length.
In a series published by Duker et al. the perforation rate was 1 in every 140 needle blocks
(retrobulbar and peribulbar, 9:1 ratio of perforations, respectively) performed in eyes with
an axial length greater than 26 mm. Other studies have reported a penetration rate of 1 in
760 to 1 in 16 000.
When using sharp-needle local anaesthesia, the risks of globe perforation can be
reduced by:
• Insertion of the needle based on anatomical principles
• Patient co-operation
• Peribulbar rather than retrobulbar block
• Using a needle of maximum length 25 mm
• Awareness that ocular and orbital anatomy may be disturbed by pre-existing ocular
pathology and surgery

Question 352: FFTTF


NEC is commonly referred to as a disease of survivors as it usually manifests in preterm
infants who have survived complications of prematurity such as respiratory distress syn-
drome and cardiac abnormalities. It can, however, present in full-term infants who usually
have an associated cardiac defect. Paradoxically, the outcome for term infants who develop
NEC without cardiac defects is worse than those who develop NEC with a cardiac abnorm-
ality. It develops within three to ten days of delivery and can be caused by bacterial or viral
pathogens (Escherichia coli, Enterobacter, Pseudomonas aeruginosa, Clostridium perfringens,
coronavirus, rotavirus and enterovirus).

Downloaded from https://www.cambridge.org/core. University of Edinburgh, on 19 Aug 2019 at 13:21:26, subject to the Cambridge Core terms of
use, available at https://www.cambridge.org/core/terms. https://doi.org/10.1017/9781108566100.012
532 Chapter 5b: Clinical Anaesthesia Answers

Question 353: TTFFT


Coronary vasoconstriction occurs due to the direct effects of noradrenaline, but secondary
metabolites increase coronary blood flow overall. Both placental and splanchnic flows are
reduced, which can lead to fetal hypoxia. Monoamine oxidase (MAO) plays an important
role in metabolism and MAO inhibition leads to extreme sensitivity to the drug. It is
recommended as first line in sepsis guidelines as a vasopressor.

Question 354: TTTTT


Important features in any patient with a pacemaker in situ include: the reason for insertion
(including date and place inserted), the mode of pacemaker function and the date of most
recent battery and pacemaker function check. In addition, it is necessary to check electro-
lytes preoperatively since derangement may alter the capture of the pacemaker. It is also
necessary to perform a 12-lead ECG to assess the underlying rhythm, presence of pacemaker
activity and evidence of electrical capture.

Question 355: TTTTF


Supine chest radiographs can fail to detect early pneumothoraces. Ultrasound can be
superior to X-rays in this particular setting. Failure to visualize the lung sliding against
the pleural line indicates the presence of air between the parietal and visceral pleurae. CT
scan remains the gold standard for diagnosing pneumothorax. Apical pneumothoraces may
require the insertion of an intercostal chest drain (ICD) in the second intercostal space at the
mid-clavicular line. Rapid drainage of pleural fluid can lead to lung re-expansion pulmon-
ary oedema. After drainage of pneumothorax, some clinicians choose to clamp the ICD to
look for recurrent accumulation of air in case of a small leak. There is no additional benefit
from clamping the drain at the time of removal.

Question 356: TFFFF


Sepsis is now a leading cause of maternal death and the most common pathogen is
community-acquired β-haemolytic group A Streptococcus causing genital tract sepsis.
Normal pregnancy-related physiological changes often mimic or mask the signs of SIRS
(systemic inflammatory response syndrome) and sepsis, meaning that the early detection of
sepsis, which is crucial to improving morbidity and mortality, can often be difficult or
delayed. During normal pregnancy there is a mild respiratory alkalosis, which is compen-
sated for with a mild metabolic acidosis. Due to this pre-existing state, any acidosis
developing in response to an underlying sepsis is poorly compensated for. Antibiotics
should be commenced within one hour of the diagnosis or suspicion of sepsis.
In diagnosed sepsis, piperacillin–tazobactam plus gentamicin is an appropriate choice for
treatment unless the patient is penicillin-allergic. There is no role for high-dose steroids in
sepsis management, although low dose steroids (<300 mg hydrocortisone per day) can be
given if shock is refractory to fluids and vasoactive drugs.

Downloaded from https://www.cambridge.org/core. University of Edinburgh, on 19 Aug 2019 at 13:21:26, subject to the Cambridge Core terms of
use, available at https://www.cambridge.org/core/terms. https://doi.org/10.1017/9781108566100.012
Chapter 5b: Clinical Anaesthesia Answers 533

Question 357: FFFFF


Table 5.357.1 Pharmacological treatment of Parkinson’s disease

Class Mechanism of action Examples

Dopamine precursor Crosses the blood–brain barrier, where it Levodopa


is converted to dopamine. Administered
with a dopa decarboxylase inhibitor to
minimize systemic breakdown
Dopamine agonist Acts by mimicking the action of Ropinirole
dopamine at the dopamine receptor Apomorphine
Pramipexole
Pergolide
Bromocriptine
Monoamine oxidase Inhibit the action of the enzyme Selegiline
type B inhibitors monoamine oxidase, preventing the Rasagiline
breakdown of dopamine in the central
nervous system
Catechol- Smooths out the end-of-dose ‘off’ Entacapone
O-methyltransferase periods seen with levodopa/dopa Tolcapone
inhibitor decarboxylase inhibitor treatment
Anticholinergics Antagonize the unopposed cholinergic Orphenadrine
pathways, which have excitatory effects
Atypical Mechanism of action is not fully Amantadine
understood

Question 358: TFTTF


Postoperative cognitive disturbance is a common and disabling consequence of anaes-
thesia and surgery in the elderly. The risk of prolonged postoperative cognitive dys-
function is 10% following major surgery in patients of more than 60 years of age.
Unlike delirium, it is not associated with changes in level of consciousness and it does
not fluctuate over the course of the day. Due to the subtle nature of POCD, only the
patient’s relatives may recognize the problem, and neuropsychological testing is neces-
sary for its detection. Risk factors include:
• Increasing age
• Duration of anaesthesia
• Episodes of hypoxaemia
• Lack of patient education
• A second operation
• Postoperative infections
• Respiratory complications

Downloaded from https://www.cambridge.org/core. University of Edinburgh, on 19 Aug 2019 at 13:21:26, subject to the Cambridge Core terms of
use, available at https://www.cambridge.org/core/terms. https://doi.org/10.1017/9781108566100.012
534 Chapter 5b: Clinical Anaesthesia Answers

It has features of dementia and confusional states, which continue after the immediate
postoperative period. Disturbance of cerebral perfusion and cellular oxygenation is
a contributory factor. Alterations of central acetylcholine and catecholamine levels, as
well as central steroid effects from the stress response, play a role. The choice of inhalational
anaesthetic has no influence on the risk of POCD, but it has been suggested that propofol via
TIVA may be associated with a lower incidence.

Question 359: FFFFF


Although there is a strong link between smoking and COPD, one should not assume that
this man has a smoking history because of the diagnosis – there may be previous occupa-
tional exposure or genetic factors such as α-1 antitrypsin deficiency. COPD is a spectrum
that encompasses both emphysema and chronic bronchitis. The inhaler regime tends to
reflect reversibility of obstruction rather than disease severity – a functional assessment of
the patient should be made: exercise tolerance/Duke’s activity status score, etc. Baseline
arterial blood gases on air would be useful preoperatively, but oxygen should not be
withheld in all these cases. It should be prescribed to targeted saturations and its require-
ment reviewed daily on the ward according to national guidance. Its use should take account
of other possible co-morbidities, in particular cardiac. NSAIDs are not absolutely contra-
indicated in COPD – establish if they have been used before and ensure there are no other
contraindications (renal impairment/ischaemic heart disease).

Question 360: FTFFT


There are a number of contraindications to regional anaesthesia, some of which are absolute
and some relative.
The absolute contraindications are:
• Patient refusal
• True allergy to local anaesthesia
• Local infection at site of entry/septicaemia
The relative contraindications are:
• Coagulopathy
• Communication difficulty/dementia
• Hypovolaemia
• Fixed cardiac output states
• Demyelinating medical conditions
• Spinal deformity
• Previous back surgery

Question 361: TTTFT


The effect that the epidural block has on the cardiovascular system is determined by the level
of the block and the degree of sympathetic blockade. Sympathetic fibres from T5–L1
maintain sympathetic tone; hence blockade of these fibres causes both arterial and venous
vasodilatation. This vasodilation can lead to reduced venous return and cardiac output,
which in turn can cause an increase in vagal tone; this is seen clinically as hypotension
without an increase in heart rate or bradycardia. Unopposed vagal activity, by blockade of

Downloaded from https://www.cambridge.org/core. University of Edinburgh, on 19 Aug 2019 at 13:21:26, subject to the Cambridge Core terms of
use, available at https://www.cambridge.org/core/terms. https://doi.org/10.1017/9781108566100.012
Chapter 5b: Clinical Anaesthesia Answers 535

the splanchnic sympathetic fibres, also has an effect on the gut, leading to increased
peristalsis and secretions. Under normal circumstances renal blood flow is maintained by
autoregulation, therefore an epidural should not have a major effect on renal function. As an
epidural blockade develops, different types of nerve fibres are blocked at different rates:
sympathetic fibres first, then pain/temperature fibres, then fibres involved in proprioception
and lastly motor fibres.

Question 362: FFFTT


Amputees tend to have significant co-morbidities, which goes some way to explaining why
mortality following emergency surgery is as high as 22%. Limb amputation is associated
with severe postoperative pain. Injury to bone, soft tissue and nerves leads to a mixed
nociceptive/neuropathic pain picture resulting in an array of postoperative pain syndromes.
Effective perioperative pain control is important in reducing the risk of chronic stump and
phantom pain. Persistent stump pain is not uncommon and can be secondary to infection,
arterial insufficiency, osteomyelitis and haematoma. Phantom pain following amputation is
very common, occurring in up to 80%. Risk factors for the development of phantom pain
include:
• Severe preoperative pain
• Bilateral amputation
• Stump pain
• Repeated limb surgeries
• Increasing age
Prolonged perineural blockade (>80 hours) may be associated with a significant reduction
in the incidence of phantom pain.

Question 363: FFFTT


The preoperative clinical status of a patient can have a large influence on the success or
otherwise of a free flap. Important factors include nutritional status, tobacco usage and the
presence of underlying co-morbidities (e.g. cardiopulmonary disease and peripheral vas-
cular disease). Poor nutritional status can impede wound healing and recovery. Although
diabetes mellitis is not associated with an increased incidence of thrombosis or flap failure,
acute and chronic hyperglycaemia is associated with vascular leakage, increasing the risk of
tissue oedema. If tissue oedema is formed this will increase extravascular pressure with
a consequent negative effect on the vascular diameter of the free flap. Chronological age is
neither a contraindication for surgery nor a risk for postoperative morbidity and flap failure.
A retrospective analysis of transverse rectus abdominis myocutaneous (TRAM) flaps found
an increased incidence of flap failure in patients with a BMI >30. Smoking is
a contraindication for TRAM-flap breast surgery reconstruction due to the high incidence
of complications.

Question 364: FFTTT


Down’s syndrome is due to trisomy 21 in >90% of cases, however other chromosomal
abnormalities can occur, with a translocation abnormality being the greatest risk factor for
subsequent children developing Down’s. There is no association with cleft palate, though an

Downloaded from https://www.cambridge.org/core. University of Edinburgh, on 19 Aug 2019 at 13:21:26, subject to the Cambridge Core terms of
use, available at https://www.cambridge.org/core/terms. https://doi.org/10.1017/9781108566100.012
536 Chapter 5b: Clinical Anaesthesia Answers

enlarged tongue, tonsils, adenoids and micrognathia may make intubation difficult. There is
a 50% rate of thyroid disorders and 5–10% incidence of epilepsy amongst Down’s syndrome
patients.

Question 365: TTFTF


In HOCM, septal hypertrophy can lead to dynamic left ventricular outflow restriction, so the
ventricle behaves similarly to that found in AS (although more often the coronary arteries
are normal, unlike in senile aortic stenosis). Thus positive inotropic agents can exacerbate
outflow obstruction. Although prevention of pulmonary oedema is important, adequate
filling is necessary for optimal ventricular filling. Noradrenaline preserves SVR in the face of
a limited (fixed) cardiac output, maintaining organ perfusion. Excessive heart rate limits
diastolic filling and so a relatively slow rate is associated with optimum cardiac output.

Question 366: FTTFT


The indications for elective insertion of a cardiac pacemaker include:
• Third-degree heart block with a slow ventricular rate (less than 40 beats per minute)
• A history of syncope attributed to heart block
• Ventricular pauses greater than 3 s
Asymptomatic third-degree heart block carries a lesser indication for pacing, but it is still
appropriate prior to elective surgery.
Pacing is indicated in the presence of chronic bifascicular or trifascicular block if there is
associated intermittent third-degree heart block or the presence of Mobitz type II block, if it
is associated with syncope.

Question 367: TTTTT


Renal replacement therapy (RRT) is an effective method to induce hypothermia. Different
drugs and toxins can be removed by RRT, such as lithium, methanol, ethylene glycol,
barbiturates, aspirin, metformin, aminoglycosides, cephalosporins and most penicillins.
However, it is not useful for digoxin, tricyclics or phenytoin toxicity. It can also be used
to provide plasmapheresis for autoimmune conditions. Renal replacement therapy can be
useful in removing some water-soluble inflammatory mediators in sepsis. Although some
studies have shown a reduction in vasopressor requirement, this has not been yet translated
into improved survival.

Question 368: TFFFT


The Third National Audit Project (NAP3) looked at the reported major complications of
central neuraxial blockade and it did demonstrate that the incidence of permanent harm
was less in the obstetric population than the perioperative population. It also showed
a worryingly high incidence of harm to patients who had undergone combined spinal and
epidural anaesthesia (CSE). Whilst these accounted for less than 6% of all central neuraxial
blocks they were involved in over 13% of the reports of harm to patients. The majority of
incidences of neurological damage discovered post partum can be attributable to obstetric
causes, however any possible neurological damage should always be investigated thor-
oughly, as the consequences of misdiagnosis can be severe. The risk of permanent nerve

Downloaded from https://www.cambridge.org/core. University of Edinburgh, on 19 Aug 2019 at 13:21:26, subject to the Cambridge Core terms of
use, available at https://www.cambridge.org/core/terms. https://doi.org/10.1017/9781108566100.012
Chapter 5b: Clinical Anaesthesia Answers 537

injury is not as rare as 1 in 100 000, however it is now thought to be less than historically
quoted. NAP3 suggests the risk is between 1 in 24 000 and 1 in 54 000. Arachnoiditis is a very
rare but devastating complication that can occur weeks or potentially months following
a central neuraxial block.

Question 369: FFTFF


Myasthenia gravis is an autoimmune condition associated with IgG autoantibodies against
postsynaptic acetylcholine receptors at the neuromuscular junction. This leads to a decrease
in the number of functional receptors at the neuromuscular junction. Patients tend to
present with symptoms of myasthenia gravis once the number of normal nicotinic acetyl-
choline receptors is reduced to 30%.
Other autoimmune diseases associated with myasthenia gravis include:
• Systemic lupus erythematosus
• Ulcerative colitis
• Hyperthyroidism
• Hypothyroidism
• Pernicious anaemia
• Rheumatoid disease
• Scleroderma
• Psoriasis
• Vitiligo
• Pemphigus
• Polymyositis
• Dermatomyositis

Question 370: FFTTT


An estimate of the descending aorta’s cross-sectional area is taken from either
a nomogram – using height, weight and age – or using M Mode ultrasound.
Adult nomogram parameters include:

Age: 16–99 years


Weight: 30–150 kg
Height: 149–212 cm
A correction factor is required during cardiac output measurement because only approxi-
mately 70% cardiac output passes through the descending aorta, as the aorta has already
supplied the coronary arteries and branches from the aortic arch to the head, neck and
upper limbs.

Question 371: TTTFF


In brittle asthmatics a regional technique is safer if this is acceptable to the patient. This
patient would benefit from nebulized salbutamol, given the background history, current
wheeze and poor PEFR. Steroid supplementation is required perioperatively. Fentanyl does
not cause histamine release and so is safe to use in brittle asthmatics, and can be used as
a PCA if there are concerns over morphine. Low-dose adrenaline, while theoretically

Downloaded from https://www.cambridge.org/core. University of Edinburgh, on 19 Aug 2019 at 13:21:26, subject to the Cambridge Core terms of
use, available at https://www.cambridge.org/core/terms. https://doi.org/10.1017/9781108566100.012
538 Chapter 5b: Clinical Anaesthesia Answers

beneficial, is not recommended. Circulating catecholamines will already be high following


the trauma, and treatment would be more appropriate with inhaled/nebulized β2-agonists
via the anaesthetic circuit, increasing depth of anaesthesia and magnesium sulfate. Don’t
forget to give 100% oxygen, exclude equipment causes and call for help early! There should
be a low threshold for requesting a HDU bed in this case.

Question 372: TTFFT


Post-dural-puncture headaches (PDPHs) usually develop within seven days of a regional
procedure and are often self-limiting. They are occasionally associated with more significant
complications, including subdural haematoma, seizures and saggital sinus thrombosis.
Features include fronto-occipital postural headaches associated with nausea, photophobia,
neck stiffness, tinnitus, visual disturbance and cranial nerve palsies.

Question 373: TFFTF


The thienopyridines clopidogrel and prasugrel both irreversibly reduce platelet action by
inhibiting the ADP receptor on the platelet surface and thus prevent activation of the
glycoprotein IIb/IIIa complex. Due to the irreversible nature of their action, platelets are
affected for their life-span (approximately seven days). Current recommendations state that
clopidogrel should be stopped for at least seven days before performing a central neuraxial
block (CNB) and prasugrel for seven to ten days. Aspirin exerts its antiplatelet action by
irreversibly inhibiting COX, thus its effects also last for the life-span of the platelet. Low-
dose aspirin at 75 mg, when given in isolation, does not increase the risk of developing
a vertebral canal haematoma, therefore it does not need to be discontinued. LMWH exerts
its anticoagulant effect primarily by inhibiting factor Xa. Current guidelines suggest allow-
ing 12 hours to elapse between administering a prophylactic dose of LMWH and perform-
ing CNB/epidural catheter removal, and 24 hours in the case of therapeutic doses of
LMWH.

Question 374: FTTTT


Best practice in the management of epidural analgesia in the hospital setting includes:
• Placing the patients close to the nurses’ station, thus ensuring close supervision
• Ensuring the ward is sufficiently staffed to maintain safe management of the epidural
• Providing a closed system between the pump and the patient, containing an antibacterial
filter
The Bromage scale is considered an acceptable tool for the assessment of motor blockade.
New onset confusion or dizziness may indicate local anaesthetic toxicity; therefore the
epidural infusion should be switched off while the patient is assessed. The pore size of
epidural catheter filters is typically 0.2–2 μm.

Question 375: TFTFF


Amitriptyline blocks serotonin and noradrenaline reuptake pumps. It also has anticholi-
nergic properties, which are responsible for some of its side effects (e.g. dry mouth, blurred
vision).

Downloaded from https://www.cambridge.org/core. University of Edinburgh, on 19 Aug 2019 at 13:21:26, subject to the Cambridge Core terms of
use, available at https://www.cambridge.org/core/terms. https://doi.org/10.1017/9781108566100.012
Chapter 5b: Clinical Anaesthesia Answers 539

Gabapentin is a structural analogue of GABA and binds to the α2-δ-subunit of voltage-


sensitive calcium channels, reducing calcium influx.
Capsaicin is an active ingredient in chilli peppers and provokes a typical burning
sensation. It is a transient receptor potential vanilloid (TRPV)-1 channel agonist. TRPV-1
is a non-selective cation channel preferentially expressed on small-diameter sensory neu-
rones, especially on nociceptors. Carbamazepine blocks voltage-sensitive sodium channels.
It is first-line therapy for pain associated with trigeminal neuralgia.

Question 376: TTFTF


The techniques used for neurological monitoring during CEA are:
• Electroencephalography (EEG)
• Transcranial Doppler ultrasound (TCD)
• Carotid stump pressure (SP)
• Somatosensory evoked potential (SSEP)
• Cerebral oximetry (rSO2)
• Conjunctival oxygen tension (pcjO2)
• Cerebral blood-flow measurement with ¹³³Xe
• Locoregional anaesthesia (LRA)
EEG, TCD, SP and SSEP are more commonly used. Cerebral oximetry and conjunctival
oxygen tension have been shown to be neither sensitive nor specific for predicting the need
for an intraoperative shunt, and measurement of 133Xe would be difficult to implement in
routine operating lists.

Question 377: TTFTF


The principles of paediatric burns resuscitation are broadly the same as in adults, following
an ABC approach. Any full-thickness burn of 5% or more requires specialist tertiary
management, as do partial thickness burns of 10% or more, and burns involving the airway,
face, perineum, hands or feet. In patients with less than 10% burns, oral fluid replacement is
possible, but above this limit there is a substantial capillary leak, which requires IV
replacement. Note that the Parkland formula does not need to be used for burns in children
less than 10% BSA and burns in adults less than 15% BSA. Parkland also only applies to burn
fluid-loss replacement; one must never forget the hourly maintenance fluid requirement is
in addition to the fluids calculated by the Parkland formula. The catabolic state caused by
burns means that the basal metabolic rate (and hence calorie consumption) is increased
substantially, so enteral feeding (especially with a glutamine-rich diet) is advocated early
and has been shown to decrease days spent on a ventilator. Suxamethonium is safe in early
burns as it takes 48–72 hours for extrajunctional ACh receptors to develop, which are
responsible for the exaggerated hyperkalaemic response.

Question 378: TFTTF


Double lumen tubes (DLTs) are available in various sizes, for men 39–41 Fr are usually
suitable, for women 37–39 Fr. For children a 26 Fr is the smallest commercially available;
this is appropriate for a child of 25–35 kg. Depth of insertion is typically 28–30 cm in adult
men, which equates to a position where the bronchial cuff is just visible below the carina.

Downloaded from https://www.cambridge.org/core. University of Edinburgh, on 19 Aug 2019 at 13:21:26, subject to the Cambridge Core terms of
use, available at https://www.cambridge.org/core/terms. https://doi.org/10.1017/9781108566100.012
540 Chapter 5b: Clinical Anaesthesia Answers

Left-sided DLTs are most commonly used, even for right-sided procedures; this is due to the
fact that right-sided tubes can easily obstruct the right upper lobe bronchus, leading to
inadequate ventilation. In a patient with a large thoracic aneurysm compressing the left
main bronchus, a right-sided tube is preferred to help prevent the risk of a traumatic
haemorrhage. The Robertshaw DLT does not possess a carinal hook. Double lumen tubes
that do incorporate a carinal hook include the Carlen, the Gordon–Green and the White.

Question 379: TFFFT


Other mechanisms include stimulating ATP-sensitive K+ channels. Contractility is increased
without increasing myocardial oxygen demand. Although cardiac output is increased, the
reduced SVR leads to hypotension in some patients. Its main metabolite is OR-1896, which is
active for a prolonged period. Hypokalaemia may occur in some patients.

Question 380: TTTFF


The potential causes of airway compromise following thyroid surgery are legion.
Assessment should be as for any airway crisis. Problems can include wound haematoma,
which should be managed by reopening the wound. This may still need to be followed by
reintubation as it is associated with extensive tissue oedema. Recurrent laryngeal nerve
palsies and hypocalcaemia (from parathyroid damage) should also be considered. Transfer
to CT with an ‘at risk’ airway cannot be recommended and ultrasound evaluation is
currently not established in management protocols.

Question 381: FTTFF


A 12-lead ECG is important to confirm the underlying rhythm and assess for further
conduction or structural abnormalities and signs of previous ischaemia. A 24-hour ECG
may be necessary in the assessment of a patient with suspected paroxysmal AF, but is not
necessary prior to elective surgery in a patient with established AF. An echocardiogram may
be necessary if there is suggestion of structural abnormality on the ECG or symptoms
suggestive of heart failure, but in cases of isolated atrial fibrillation, it is not mandatory.
Serum electrolytes should be checked prior to surgery; derangement can precipitate
arrhythmias perioperatively. Inflammatory markers may be helpful in determining the
cause of new onset AF during a hospital admission but have no role in the preoperative
period.

Question 382: FFTFT


Acute rhabdomyolysis is characterized by a fivefold increase in serum creatinine and the
presence of myoglobinuria as a result of skeletal muscle damage. A urine dipstick cannot
differentiate between haemoglobin and myoglobin. Electrolyte disturbance seen in the acute
phase involves hyperkalaemia, hyperphosphataemia and hypocalcaemia, as the calcium is
driven into the cells. Hypercalcaemia can occur later due to secondary hyperparathyroid-
ism. Cocaine is a common cause of both traumatic and non-traumatic rhabdomyolysis; the
latter is due to prolonged vasoconstriction of intramuscular arteries, resulting in ischaemia.
Whilst both serum urea and creatinine are increased, the ratio is reduced. Urine

Downloaded from https://www.cambridge.org/core. University of Edinburgh, on 19 Aug 2019 at 13:21:26, subject to the Cambridge Core terms of
use, available at https://www.cambridge.org/core/terms. https://doi.org/10.1017/9781108566100.012
Chapter 5b: Clinical Anaesthesia Answers 541

alkalinization may have a role in renal protection. Sodium bicarbonate is also useful in the
management of severe acidosis and hyperkalaemia.

Question 383: FTFTT


Pre-eclampsia is a multisystem disorder of pregnancy of unknown aetiology. It is thought
that there is abnormal trophoblastic invasion of the spiral arteries leading to a high vascular
resistance within the placental bed instead of the low resistance system in a normal
pregnancy. The altered production of vasoactive substances, including thromboxane, causes
an increase in SVR and corresponding damage to maternal organs other than the placenta.
It occurs after 20 weeks of gestation and it is characterized by hypertension and proteinuria.
Severe pre-eclampsia would be suggested by a systolic BP >170 or a diastolic >110 mmHg
and proteinuria >1 g over 24 hours. Clinical features include:
• Oliguria <400 ml over 24 hours
• Cerebral irritability
• Headache
• Visual disturbance
• Epigastric or RUQ pain
• Pulmonary oedema
• HELLP syndrome (Haemolysis, Elevated Liver enzymes and Low Platelets)
Seizures would indicate that the patient has eclampsia, which can be described as seizures
occurring on a background of pre-eclampsia.

Question 384: FFTTF


Pyridostigmine is classified as an anticholinesterase. Anticholinesterases act by antagoniz-
ing the enzyme acetylcholinesterase. Acetylcholinesterase hydrolyzes acetylcholine at the
neuromuscular junction postsynaptic membrane. Thus, pyridostigmine increases the
amount of available acetylcholine at the neuromuscular junction. Pyridostigmine does
not cross the blood–brain barrier. In comparison to neostigmine, pyridostigmine has
a slower onset, but a longer duration of action. Pyridostigmine acts within 30 minutes of
administration, with peak effect at two hours and a half-life of four hours. Dosages of
30–120 mg orally 4–12 hourly are used (maximum 1.2 g per day). However, the frequency
of administration is patient-specific. Whilst pyridostigmine is usually very effective in the
initial stages of myasthenia gravis, effectiveness decreases with time and other treatments
are then required.

Question 385: FFTFF


Functional residual capacity, residual volume and total lung capacity cannot be measured by
spirometry. Measurement of flow–volume loops starts with forced expiration following
a vital capacity breath and so the starting point on the left of the loop is the total lung
capacity and the loop starts positively with expiratory flow, ending at residual volume before
negative inspiratory flow. During expiration, the intrathoracic pressure rises, resulting in
compression of the airways such that there is a maximal expiratory flow rate that is
independent of effort, giving a straight descending line on the loop. Intrathoracic

Downloaded from https://www.cambridge.org/core. University of Edinburgh, on 19 Aug 2019 at 13:21:26, subject to the Cambridge Core terms of
use, available at https://www.cambridge.org/core/terms. https://doi.org/10.1017/9781108566100.012
542 Chapter 5b: Clinical Anaesthesia Answers

obstruction is more likely to affect expiration, allowing more gas flow in inspiration due to
negative intrathoracic pressures, whereas extrathoracic obstruction is more likely to impede
inspiration. Peak flow reference ranges are gender specific based on age and height.

Question 386: TFTTF


The internal jugular vein (IJV) runs down from the jugular foramen and joins the sub-
clavian vein behind the clavicle to form the brachiocephalic vein. It lies within the carotid
sheath with the vagus nerve between the major vessels.
Anatomical relationships to the IJV are:
• Medial: internal carotid artery, common carotid artery, thoracic duct
• Anterior: omohyoid
• Posterior: vagus nerve, sympathetic chain, dome of pleura

Question 387: TFTFF


Data from 55 567 cataract operations extracted from the National Cataract Dataset pub-
lished in 2009 reported a low rate (1.4%) of sedation for cataract surgery. Local anaesthesia
was used in 95.5% of procedures, with topical anaesthesia alone in 22.3%. The vast majority
of patients are suitable for day-case surgery and neither age nor weight need be an issue in
this respect. Living alone, either with or without a telephone, and distance from the hospital
do not contraindicate day-case surgery under local anaesthesia without sedation, provided
there is adequate support for the patient. It is generally not necessary to starve patients prior
to ophthalmic surgery under local anaesthesia. Although there are theoretical reasons for
believing that a period of fasting prior to local anaesthesia might be appropriate, there have
been no reported cases of aspiration under local anaesthesia during cataract operations.
Although appropriately trained, professionally regulated and indemnified non-medical staff
may administer subconjunctival or sub-Tenon’s blocks for selected, ambulatory cataract
surgery, peribulbar or retrobulbar needle blocks should only be performed by medically
trained personnel. Intravenous access is essential with peribulbar and retrobulbar blocks
and when intraoperative sedation is used and is recommended for long/complex cases, sub-
Tenon’s blocks and patients with poor general health.

Question 388: FFFFF


The guiding principle of anaesthesia for microvascular flap surgery is to maintain adequate
flap blood flow. With the assumption that flap blood flow is laminar, the determinants are
governed by the Hagen–Poiseuille equation:
Q = Pπr4/8ηl
where Q = laminar flow rate, P = pressure, r = radius, η=viscosity and l = length.
The optimum haematocrit for O2 delivering capacity and reduced blood viscosity has
been suggested as 30%, with viscosity increasing greatly over a haematocrit of 40%.
Hypervolaemic haemodilution was traditionally used for this type of surgery but a greater
appreciation of the risks of tissue oedema and the demonstration of increased medical
complications and length of hospital stay with excessive fluid administration has led to more
goal-directed fluid therapy. Animal studies of sodium nitroprusside in latissimus dorsi free
flaps showed that using sodium nitroprusside to achieve a decrease in systemic vascular

Downloaded from https://www.cambridge.org/core. University of Edinburgh, on 19 Aug 2019 at 13:21:26, subject to the Cambridge Core terms of
use, available at https://www.cambridge.org/core/terms. https://doi.org/10.1017/9781108566100.012
Chapter 5b: Clinical Anaesthesia Answers 543

resistance (SVR) and arterial pressure of 30% caused a severe reduction in free flap blood
flow. Hypercapnia causes sympathetic stimulation and reduces erythrocyte deformity,
whilst hypocapnia increases vascular resistance and decreases cardiac output. Hyperoxia
has been shown to cause a reduction in microvascular flow. Remifentanil provides vasodi-
latation, rapid control of blood pressure, intraoperative analgesia and reduces muscle
relaxant requirements.

Question 389: TFFTF


In 2011, the European Society for Vascular Surgery published guidelines on the manage-
ment of abdominal aortic aneurysm (AAA), making use of newly available evidence on
pharmacological treatments, types of imaging and preoperative assessment and operative
interventions. Statins have been shown to reduce morbidity and mortality when given
perioperatively for vascular surgery and should be started one month before surgery and
continued indefinitely. β-blockers are recommended for patients with ischaemic heart
disease and those who have myocardial ischaemia on stress testing, but should be started
one month before surgery and not commenced immediately perioperatively. A formal
assessment of cardiac risk, including an ECG, should be performed in all patients under-
going surgery for an abdominal aortic aneurysm. Those with a positive cardiac history or
cardiac risk factors should have a transthoracic echocardiogram and be considered for
pharmacological stress testing or myocardial perfusion scanning prior to AAA repair.
Minimally invasive AAA repair can be carried out under dual antiplatelet therapy after
drug-eluting coronary stenting.

Question 390: FTTFT


When a patient over time requires greater amounts of drug to achieve the original degree of
effect, this is known as tolerance. It is common in patients taking opioids and can occur in as
little as two weeks. This patient may be physically dependent on opioids. If the patient in this
question were to suddenly stop taking his morphine he would suffer from a withdrawal
reaction. The same is true if his baseline oral opioid intake was not calculated and replaced
appropriately following surgery (Table 5.390.1). At this point thought can be given to
providing appropriate analgesia – for a nephrectomy this will certainly mean the adminis-
tration of further opioids, perhaps in the form of a PCA.

Table 5.390.1 Conversion calculations of oral to parenteral morphine or diamorphine

Drug Dose equivalent to 10 mg oral morphine

IM morphine 5 mg
IV morphine 3.3 mg
SC morphine 5 mg
SC diamorphine 2.5 mg
Epidural morphine 1 mg
Intrathecal morphine 0.1 mg

Downloaded from https://www.cambridge.org/core. University of Edinburgh, on 19 Aug 2019 at 13:21:26, subject to the Cambridge Core terms of
use, available at https://www.cambridge.org/core/terms. https://doi.org/10.1017/9781108566100.012
544 Chapter 5b: Clinical Anaesthesia Answers

Question 391: FFTFF


Serum calcium is recommended to be less than 3 mmol.l−1, or lower if there is cardiac or
renal disease. The parathyroid glands are small and are rarely associated with a difficult
airway (in contrast to thyroid disease). Hyperparathyroidism is associated with MEN1 and
2, which may be present with other endocrine disorders of concern to the anaesthetist.
Steroids are indicated for hypocalcaemia related to malignancy. Hypocalcaemia is asso-
ciated with long QTc, although a preoperative ECG is not indicated for all patients for
parathyroid surgery.

Question 392: FTTTF


Phaeochromocytomas are neuroendocrine tumours secreting adrenaline and/or noradrena-
line and other catecholamine substances. They are said to follow the rule of 10s: 10%
malignant, 10% multiple, 10% extra-adrenal, 10% familial. Familial syndromes also include
Carney’s triad, paraganglioma syndromes and some families show inheritance without an
identified syndrome. MEN1 is classically a triad of parathyroid, pituitary and pancreatic
tumours. Klippel–Trenaunay is a syndrome of venous disease, port wine stains, skeletal
enlargement and AV malformations.

Question 393: FTTFT


Antifreeze is made up of ethylene glycol, an alcohol, that is metabolized by the enzyme
alcohol dehydrogenase. Its metabolism follows zero-order kinetics. Its toxicity is character-
ized by severe metabolic acidosis with a high anion as well as a high osmolar gap. Activated
charcoal is ineffective and has no role in management. Dialysis may be indicated because of
the severe acidosis and renal failure. Ethylene glycol serum level takes a long time to be
measured. Treatment should not be delayed if there is a history of ingestion or if the osmolar
gap is high with a suspicious history. The main treatment involves blocking alcohol
dehydrogenase using either ethanol or fomepizole. Ethanol is cheap and widely available,
but comes with the side effects of alcohol. Ethanol serum level should be closely monitored
during treatment. Fomepizole has a much safer profile with a wide therapeutic index, but is
expensive.

Question 394: TTTTT

Table 5.394.1

Drugs dosed by ideal body weight Drugs dosed by total body weight

Propofol Fentanyl
Rocuronium Midazolam
Vecuronium Atracurium
Paracetamol Cisatracurium
Morphine Succinylcholine
Thiopental Mivacurium
Alfentanil Neostigmine
Remifentanil Sufentanil

Downloaded from https://www.cambridge.org/core. University of Edinburgh, on 19 Aug 2019 at 13:21:26, subject to the Cambridge Core terms of
use, available at https://www.cambridge.org/core/terms. https://doi.org/10.1017/9781108566100.012
Chapter 5b: Clinical Anaesthesia Answers 545

Question 395: TFTTF


An android or ‘apple-shaped’ pattern of fat distribution leads to fat deposition centrally and
viscerally. Males generally exhibit an android fat distribution, whereas females are generally
gynoid or ‘pear-shaped’, with more peripheral fat distribution. Peripheral obesity leads to
fat deposition outside of body cavities and thus is associated with less co-morbidity. Central
obesity can be defined as a waist circumference greater than 88 cm in a woman and 102 cm
in a man, or a waist-to-height ratio greater than 0.55. Intra-abdominal or visceral fat is
highly metabolically active, and thus increases the risk of co-morbidities such as metabolic
syndrome, hypertension, dyslipidaemia, insulin resistance, thrombosis and cardiovascular
disease.

Question 396: TFFTT


The most likely diagnosis is either a massive PE or a significant MI resulting in acute heart
failure. The patient has presented within the six-hour window for thrombolysis. However,
he is too unstable to be transferred safely to the CT scanner. Echocardiography is a useful
tool to exclude PE if the patient is in shock. For stable patients, CTPA should be the test of
choice. A massive PE will result in a dilated right atrium and right ventricle. Sometimes the
embolus can be seen in the pulmonary artery. In cases of MI, echocardiogram is also useful
to detect wall motion abnormalities. Thrombolysis without imaging is only acceptable if
cardiac arrest is imminent. Although the patient needs stabilization on the ICU, this should
not delay the diagnosis and management of a periarrest condition.

Question 397: FTFTT


Staphylococcus aureus is currently the most common isolated organism in infective endo-
carditis (IE). Fungal endocarditis is difficult to isolate and usually requires surgical inter-
vention. Although thrombosis is a key factor in the development of IE, there is no evidence
to support the benefit of anticoagulation. Streptococcus bovis is usually associated with bowel
malignancy. Serial blood cultures should be obtained before starting antibiotics.
Administering antibiotics before taking blood cultures reduces the recovery rate of bacteria
significantly.

Question 398: TTTTT


Total spinals most frequently occur due to the topping up with local anaesthetic of an
incorrectly placed epidural catheter, although they can occur as a complication of certain
nerve and plexus blocks. An ascending block will produce a variety of symptoms. Once
above T4, cardiac sympathetic fibres are affected and hypotension and bradyarrhythmias
are likely. Hands and arms will become weak, with increasing numbness and accessory
respiratory muscles affected once C6–C8 is reached. Above C5 the diaphragm will be
affected and respiration will be impaired enough to require immediate respiratory support.
If the local anaesthetic spreads intracranially, then unconsciousness ensues and a true total
spinal results. This requires immediate recognition and appropriate cardiorespiratory
support under general anaesthetic until the spinal block has receeded.

Downloaded from https://www.cambridge.org/core. University of Edinburgh, on 19 Aug 2019 at 13:21:26, subject to the Cambridge Core terms of
use, available at https://www.cambridge.org/core/terms. https://doi.org/10.1017/9781108566100.012
546 Chapter 5b: Clinical Anaesthesia Answers

Question 399: TFTTT


It is true that around 7% of women undergo some form of trauma during pregnancy.
The majority of these events are relatively minor, but they can still have significant
consequences as the majority occur in the final trimester and therefore the early involve-
ment of obstetricians is necessary to aid in the recognition and management of obstetric
complications. Physiological parameters and the response to treatment will vary from the
standard trauma management and need to be taken into account. Initial management
should follow ATLS guidelines, but with appropriate alterations, the most important
being that the patient is tilted laterally as soon as possible, which in some circumstances
may involve the tilting of an immobilized patient on a spinal board. After the primary
survey is completed, an assessment of fetal status can be undertaken, if safe to do so. Pelvic
injuries do have an associated incidence of fetal mortality of up to 25%. Uterine rupture is
unusual but placental abruption reasonably common, even in minor trauma cases.
Concealed blood loss is a major concern in all cases.

Question 400: TTFTF


Of patients with oesophageal atresia, 92% will have a tracheo-oesophageal fistula (TOF) and
50% of patients will have another associated congenital abnormality, most commonly
structural cardiac defects. Because of the risk of aspiration of both saliva from the upper
pouch and gastric acid from a lower pouch (depending on where the fistula is located),
surgical correction is performed within 24 hours of birth where possible, once any cardiac
abnormalities have been identified. Surgery is very successful in most cases, and the Spitz
classification is used as a prognostic tool for survival. It is based on birth weight and the
presence or absence of congenital cardiac defects and not the anatomical variations of OA/
TOF. The most frequent signs of OA are failure to clear saliva, repeated episodes of
coughing/choking, desaturation after birth or respiratory distress after repeated attempts
at feeding. Failure to pass an NG tube may confirm the diagnosis, but it is not the only sign
that there is an atresia. Due to the presence of TOF, gaseous induction with the baby
spontaneously breathing is the preferred induction technique, as positive pressure ventila-
tion may increase the risk of gastric distension, even with perfect technique.

Downloaded from https://www.cambridge.org/core. University of Edinburgh, on 19 Aug 2019 at 13:21:26, subject to the Cambridge Core terms of
use, available at https://www.cambridge.org/core/terms. https://doi.org/10.1017/9781108566100.012
Index

Answers denoted in bold face

abdomen fire during laser surgery antifungal drugs 206, 258


anterior wall 114, 151 385, 494 antineuropathic drugs
inguinal canal 115, 152–153 alcohol consumption 209, 419–420, 538–539
pathology 115, 153 262–263 antiplatelet agents 201, 249
surface anatomy 114, alfentanil 192, 233 antisialagogue drugs 205, 257
151–152 allergies, to anaesthesia 31, antistatic footwear 269, 305
abdominal aortic aneurysms 96–97. see also antibiotics 206–207, 258–259
411, 422–423, 527, 543 anaphylaxis antibodies 30, 96
accidental awareness altitude antidiuretic hormone (ADH)
339–340, 433 anaesthesia 293, 326–327 15, 67–68
ACE inhibitors 197–198, 243 environmental changes antifreeze ingestion 423, 544
acetylcholine 11, 18, 59, 72 25, 84 aprotinin 201, 250
receptors 20, 75 physiological changes 7, 24, arm. see also upper limb
acetylcholinesterase inhibitors 49–50, 83 arrhythmias, postsurgical
180, 213 alveolar ventilation/gas 380, 488
acid–base balance 15, 66–67 equation 7, 48 arterial cannulae 270–271, 306
acromegaly 340, 355–356, 385, γ-aminobutyric acid (GABA) arterial trace, damping
434, 456, 493 receptors 12, 59–60 271–272, 307
action potential, nerve cells 11, amiodarone 198–199, 244 arterioles, constriction 4,
57–58 amniotic fluid embolisms 43–44
acute fatty liver of pregnancy 343, 439 aspiration risk factors 338–339,
(AFLP) 337, 430–431 anaerobic respiration 13, 61 374, 432, 479
acute kidney injury (AKI) anaerobic threshold 345, 441 aspirin 190, 229
407–408, 523 anaesthesia. see also local asthma 400, 513
acute lung injury (ALI) 9, anaesthesia acute severe 204, 255
53–54 mechanisms 188, 226 anaesthesia 418–419,
acute respiratory distress monitoring 275, 310 537–538
syndrome (ARDS) total intravenous 184, life-threatening 388, 498
357–358, 401, 457–458, 220–221 atenolol 198, 243–244
514–515 analgesics 189–193, 228–229 atlantoaxial subluxation 358,
acute respiratory failure 9, anaphylaxis 344, 440 458–459
53, 54 causes 366, 403–404, atmosphere 264, 299–300
Addison’s disease 401, 515 468–469, 518 atracurium 193, 235
adenosine 198, 244 management 370, 383, 492 atrial fibrillation 352, 407, 421,
adrenal cortex 28, 92 angiosarcoma excisions 341, 450–451, 522–523, 540
adrenal medulla 29, 92–93 435–436 atrioventricular node (AVN)
advance directives 362, ankle 1, 35
463–464 anatomy 129, 172 atropine 194–195, 237–238
adverse drug reactions 181, 214 nerve blocks 131, 175 auditory evoked potentials
afterload 3, 41 antacids 205, 256 19, 74
agonists/antagonists 179, antecubital fossa 124–125, awake craniotomy anaesthesia
211–212 166–167 360, 460–461
airways antiarrhythmic drugs 180, 198, awake fibreoptic intubations
assessments 347, 443–444 213, 244 (AFOI) 370, 474–475
burns 368, 471–472 antiemetic drugs 205, 256 axilla 124, 164–165
difficulties 349, 447 antiepileptic drugs 203, 253 axillary nerve blocks 125, 167
547
Downloaded from https://www.cambridge.org/core. University of Edinburgh, on 19 Aug 2019 at 13:21:27, subject to the Cambridge Core terms of
use, available at https://www.cambridge.org/core/terms. https://doi.org/10.1017/9781108566100.013
548 Index

bendroflumethiazide 199, brachial plexus 121–122, cell salvage 381, 382, 490
246–247 160–161 cellular respiration 13, 61–62
benzodiazepines 186, blocks 345, 441 central neuraxial block (CNB)
224–225, 393 nerves 122, 124, 161, 165 200, 248, 334–335, 419,
bile salts 27, 87–88 brain 426, 538
bilirubin 27, 88 anatomy 16, 70, 119–120, central venous catheter (CVC)
bioavailability, drugs 182, 157–158 402, 410, 516, 526
216–217 blood flow 17, 70–71 central venous pressure (CVP)
biological signals 295, 329 blood supply 120, 158–159 2, 39–40
bispectral index (BIS) 19, injury, intubation/ cerebral hyperperfusion
74–75 ventilation 382–383, 491 syndrome 379, 486. see
monitoring 358–359, 459 ischaemia 420, 539 also brain
blood–brain barrier 17, 70 neurotransmitters 16, 69–70 cerebral palsy (CP) 340–341,
drugs 182, 215–216 brainstem death 386–387, 496 381, 435, 489
blood donation screening breast milk 387, 497 cerebrospinal fluid (CSF)
370–371, 475 breathing systems, in analysis 342–343, 438
blood flow anaesthesia 281–283, cervical plexus 99, 133–134
arterioles/capillaries 5, 45 316–317 deep blocks 103, 138
cerebral 17, 70–71 bronchi 106–107, 142 chemistry laws 269, 305
coronary 3, 41 anatomy 363, 465 chemoreceptors 10, 55–56
lungs 7, 48–49 blood supply/lymph chi-squared tests 268, 303
blood groups 30, 95, 374, drainage 107, 143 children. see paediatrics
479–480 buffers 14, 65–66 Child–Pugh score 380, 487
blood plasma 31, 96 Bunsen’s solubility coefficient chlorhexidine 356, 456
blood pressure 290–291, 325 chronic obstructive pulmonary
automated non-invasive burn centres/units 346, 443 disease (COPD) 416,
measuring 272, 307 burns 379, 387, 486, 496 534
intra-arterial monitoring children 383, 403, 420, 491, chronic postsurgical pain
272, 307 517, 539 (CPSP) 411, 526–527
measurement 271, 306 thermal 360, 461 chronic respiratory failure 9
physiological control 3, total body surface area cisatracurium 193, 234
42–43 percentage 336, 428–429 Clark electrodes 289, 323
blood products 202, 251–252, cleft lips/palates 362–363,
349–350, 447–448 caesarean section 369, 376, 464
transfusion 202, 252 380, 488 clopidogrel 200, 248–249
blood supply calcitonin 29, 93 closed/sealed systems
foot 129–130, 173 calcium homeostasis 29, 93–94 291–292, 326
lower limb 128, 171 carbimazole 207–208, 261, coagulation
placenta 131–132, 176 373, 478 drugs 200, 248
upper limb 123, 163–164 carbocisteine 204, 255–256 pathways 30, 95–96
uterus 131–132, 176 carbon dioxide transport, cocaine 209, 263
blood transfusions 366, blood 10, 55 codeine 191, 231
373–374, 378, 469, cardiac. see also heart coeliac plexus blocks 115, 153,
479, 485 action potential 1, 34–35 360, 460
bodily fluids 13, 62 cycle 2, 35–39 cognitive dysfunction, elderly
body temperature 265, 300 cardiopulmonary exercise 32, 97–98
perioperative 278, 363, testing (CPET) 355, 367, colligative properties, solutions
464–465 455, 470 291, 325
measurement 278, 312 cardiovascular changes combined spinal–epidural
physiological changes 25, 84 elderly 32, 98 anaesthesia (CSE) 375,
bone cement implantation pregnancy 21, 78 481–482
syndrome (BCIS) 381, carotid arteries complete heart blocks 402, 516
489 endarterectomy 103, 138 complex regional pain
Bourdon gauges 271, 307 sheath 103, 138 syndrome 345–346, 382,
bowel tumours 346, 443 stenosis 371, 476 441–442, 490–491
Bowman’s capsule 14, 64 categorical data 268, 303 compliance, lungs 5, 45–46

Downloaded from https://www.cambridge.org/core. University of Edinburgh, on 19 Aug 2019 at 13:21:27, subject to the Cambridge Core terms of
use, available at https://www.cambridge.org/core/terms. https://doi.org/10.1017/9781108566100.013
Index 549

computerized tomography dipyridamole 200–201, 249 ECT (electroconvulsive


(CT) 283–284, 318 direct arterial line pressure therapy) 350, 408–409,
conduction, heart 1, 33–34 monitoring 275, 310 448, 524
congenital heart diseases 359, direct current cardioversion EEG (electroencephalography)
459–460 368, 472–473 19, 73–74
context-sensitive half-life discharge, day-surgery patients Eisenmenger’s syndrome
184, 220 375, 482 374, 479
contractility 3, 41 distributions, statistical 267, elderly patients
coronary arteries. see also heart 302, 303 anaesthesia 395–396, 508
blood flow 3, 41 diuretics 199, 246 anaesthetics 413, 529–530
stents 397, 510 diving 25, 83–84 physiology 31–32, 97, 98
corticosteroids 208, 261 dopexamine 195, 239 electrical currents, direct/
COX-2 inhibitors 191, 230 Doppler effect 297, 331 alternating 281, 314–315
CPEX testing, thoracic surgery dorsal column 18, 72 electrical equipment
patients 337–338, 431 dose–response curves class I 269–270, 305
cranial nerves 116–117, 178–179, 211 class II 270, 305
154–156 double lumen tube (DLT) 376, components 280, 314
craniopharyngioma 392, 503 420, 539–540 plugs 280, 314
croup 377, 484 Down’s syndrome 381–382, transformers 280, 314
Cushing’s syndrome 395, 507 417, 535–536 electrical nerve stimulation
CVP waveform, abnormalities anaesthesia 407, 522 390, 500
393, 504 doxapram 208–209, 262 electromagnetism 280,
cyanotic heart disease 334–335, drug–receptor interactions 313–314
348, 426–427, 446 179, 212 end tidal carbon dioxide
cytochrome P450 enzyme drugs monitoring 279–280, 313
inhibitors 181, 214 action mechanisms 180–181, endocarditis, infective 424,
cytochrome P450 inducers 213–214, 215 545
412, 528 adverse reactions 181, 214 endocrine system. see
antiepileptic 203, 253 hormones
damping 272, 307–308 bioavailability 182, 216–217 endovascular aneurysm repair
arterial trace 271–272, 307 blood–brain barrier 182, 400–401, 513–514
day-case units 372, 477 215–216 energy 285
day surgery 361, 461 combinations 181, 214–215 energy metabolism 26, 84–86
dead space 5, 46 crossing placenta enoximone 407, 522
defibrillators 276, 310–311 182–183, 217 enteral feeding 400, 513
dementia 405, 519–520 delivery to cells 178, 183, epidural analgesia
dental damage under 210–211, 218 abdominal surgery 361, 462
anaesthesia 408, 523–524 elimination 183, 219–220 catheter removal 419, 538
dermatomes, arm 122, genetic differences in labour 189–190, 228,
161–163 metabolism 184–185, 405, 520
desflurane 292, 326 221–222 physiological effects 416,
diabetes 207, 259–260 interactions 181, 215 534–535
medications 342, 437–438 metabolism 182, 183–184, postoperative management
mellitus 379–380, 487 217–218, 352, 451–452 419, 538
perioperative considerations repeated doses 180, 212–213 pregnancy 365, 468
384, 492 Duchenne muscular dystrophy epidural space 117, 156
preoperative considerations 365–366, 468 epiglottitis 369–370, 373,
365, 375–376, 398, ductus arteriosus 354, 454 474, 478
467–468, 482, 511 dyspnoea, severe 424, 545 epilepsy/epileptic seizures 346,
dialysis dependence 357, 457 365, 442–443, 468
diamorphine 192, 232–233 ECG (electrocardiography), erythropoietin 30, 94
diaphragm 108–109, 144–145 pregnancy 22, 79 etomidate 185, 223
diaphragmatic hernia, ECG/EEG/EMG potentials exercise, healthy 24, 83
congenital 394, 505 295, 329 exomphalos 403, 517–518
diet 14, 28, 64–65, 90–91 eclampsia 21, 77–78 explicit recall 354, 453
diffusion, gas 9, 54

Downloaded from https://www.cambridge.org/core. University of Edinburgh, on 19 Aug 2019 at 13:21:27, subject to the Cambridge Core terms of
use, available at https://www.cambridge.org/core/terms. https://doi.org/10.1017/9781108566100.013
550 Index

exponential functions cylinders 296, 330 heparin 200, 247–248


275–276, 310 diffusion 9, 54 high-frequency oscillatory
extracorporeal shock-wave ideal 266, 301 ventilation (HFOV) 401,
lithotripsy (ESWL) laws 265–266, 293–294, 514–515
389–390, 499 301, 327 high-impact interventions,
pipelines 296–297, 330 guidance/tools 391, 501
facial nerves 117, 155–156 gastric secretions 27, 88–89 high spinal anaesthesia 121,
female sterilization 391, 502 gastrointestinal tract 113, 160, 377, 484
femoral triangle 127, 169–170 150–151 hip hemiarthroplasty
femoral vein, anatomy 389, 499 gastroschisis 397–398, 510–511 410–411, 526
fentanyl 192, 233 genetic differences, drug Hirschsprung’s disease
fetal/fetus handling 184–185, 367, 470
circulation 132, 176 221–222 hormones 29, 93
haemoglobin 24, 82 glomerular filtration rate pituitary glands 28, 91–92
term 24, 82–83 (GFR) 14, 65 placenta 23, 80
Fick principle 40–41, 495 glomerulus 14, 64 thyroid 29, 94
Fick’s law of diffusion 291, 325 glucocorticoids, exogenous humidifiers 287, 322
flap surgery 406–407, 522 208, 261 humidity
free tissue 417, 535 glycopyrrolate 195, 238 absolute 287, 321–322
microvascular 422, 542–543 goitre 104, 139–140 relative 287, 322
flow Goldman cardiac risk index hyoscine 195, 238–239
laminar 269, 273–274, 347, 444–445 hyperbaric gases 25, 83–84
304, 309 Guillain–Barré syndrome hyperglycaemia 412, 528–529
pressure 294, 328 (GBS) 53, 340, 356, 365, hypertension
tube 5, 45 387, 433–434, 456, guidelines 391, 502
turbulent 4, 44, 269, 274, 467, 497 uncontrolled 387–388, 497
304, 309 hypertrophic obstructive
flowmeters 269, 274–275, haemofiltration 183, 218–219 cardiomyopathy (HOCM)
304–305, 310 haemoglobin 8, 50–51, 52 417, 536
fluid manometers 271, fetal 82 hypocalcaemia 353, 452–453
306–307 haemorrhage 16, 69 hypoglycaemic agents 207, 260
fluid resuscitation, burns half-life, context-sensitive hypoplastic left heart syndrome
patients 364, 466–467 184, 220 378, 382, 485, 489
foot halothane 187–188, 226 hypothalamus 19, 74
anatomy 129, 172–173 hand muscles 124, 165–166 hypothermia 27, 89–506,
blood supply 129–130, heart 1–4. see also cardiac; 368, 394
173 coronary arteries perioperative 278, 363,
muscles 129, 173 anatomy 109–112, 145–148 464–465
nerves 130, 173 blocks 343, 438 hypoxaemia 9, 54–55, 394–395
foreign body aspiration, congenital diseases 359, hypoxic pulmonary
children 385, 493–494 459–460 vasoconstriction 362, 463
forest plots 268, 303 disease in pregnancy hysterectomy, Pfannenstiel
Frank–Starling law 2, 40, 43 348, 445 incision 379, 486
fresh frozen plasma (FFP) ischaemic disease 401, 514
359–360, 460 muscle structure 1, 33 ICU-acquired weakness 372,
functional residual capacity output formulae 3, 40–41 404, 477, 518
(FRC) 6, 46–47, 277–278 output monitoring 273, 275, ideal gases 266, 301
furosemide 199, 244–246 308, 309, 310 ilium 114, 151
rate 3, 41 infraclavicular blocks 125, 167
G proteins 12, 60–61, 179–180 risk index scoring infrared spectrophotometry
gallbladder 112–113, 149 376–377, 483 288, 322
gas 266, 301 surgery 361, 462 inguinal canal 115, 152–153
chromatography 287–288, heat 264–265, 300. see also inhalational anaesthetic
295–296, 329–330 temperature potency 187, 226
concentrations 288, 289, capacity 285–286, 320 inhalational induction,
322–323 latent 265, 300–301 children 358, 459

Downloaded from https://www.cambridge.org/core. University of Edinburgh, on 19 Aug 2019 at 13:21:27, subject to the Cambridge Core terms of
use, available at https://www.cambridge.org/core/terms. https://doi.org/10.1017/9781108566100.013
Index 551

inotropic agents 195–196, transplants 339, 432 nerves 127–128, 170–171


239–240 reflexes 130, 173–174
insulin 94–260, 207 laminar flow 273–274, 309 thigh muscles 126–127,
intercostal chest drain (ICD) laparoscopic cholecystectomy 169
415, 532 353–354, 374, 453, 480 3-in-1 blocks 131, 175
intercostal spaces 106, 141–142 laparoscopic surgery 342, lumbar dermatomes 130, 174
nerve supply 106, 142 437 lumbar plexus 118, 156–157
intermittent positive pressure laryngospasm 400, 513 blocks 131, 175
ventilation (IPPV) 7, 49 laryngotracheobronchitis Lund–Browder chart 437
internal jugular vein (IJV) 377, 484 lung-volume reduction surgery
422, 542 larynx 100–101, 134–136 (LVRS) 358, 458
interosseous needles 132, nerve injury 104, 140 lungs. see also respiration
176–177 lasers 286–287, 321 borders 107, 143
interscalene blocks 125, 167, latent heat 265, 300–301 compliance 5, 45–46
352, 451 laxatives 205, 256–257 non-respiratory functions
intraabdominal compartment leakage current standards 270, 8, 50
syndrome 364–365, 467 305–306 surfaces 107, 142
intra-arterial blood pressure left ventricular end diastolic surfactant 24, 47, 82
monitoring 272, 307 volume (LVEDV) 4, 43 volumes 5, 45, 277, 312
intracranial pressure leg. see lower limb lymphatic system 4, 44–45
waveforms 351, 449 levosimendan 196, 240, upper limb 123–124, 164
intralipids 189, 228 420, 540
intraocular pressure 397, 510 limb amputation 416–417, 535. magnesium ions 203–204,
drugs 203, 253–254 see also lower limb; upper 254
intrathecal opioids 371–372, limb magnesium sulfate 203, 254
396, 476–477, 508 linear regression 266, 301 magnetic resonance imaging
intravenous fluids 201–203, liver 26, 87 (MRI) 283, 317,
252–253, 377, 484 anatomy 112–113, 149 367–368, 471
intubation difficulties 336–337 functions 388, 497 malignant hyperpyrexia (MH)
ionizing radiation 293, 327 pregnancy 22, 79–80 334, 338, 348, 358–359,
ischaemic heart disease, transplantations 357, 457 431–432, 446, 459
preoperative assessment Lloyd–Davis position mass spectrometry 288–289,
401, 514 348–349, 446 322, 323
isoflurane 186–187, 225 local anaesthesia maxillary nerve blocks 104,
isotopes, medical 293, 327 bolus volume 339, 432–433 138–139
improved absorption MDMA 209, 263
Jehovah’s Witnesses 370, 475 mechanical work 284, 318
advance directives 362, maximum doses 188, 227 median nerve injuries 125–126,
463–464 metabolism 188, 227 167–168
alternatives to blood nerve function 189, 227–228 meningitis, children
transfusion 360–361, 461 speed of onset 188, 226–227 346–347, 443
consent for children 408, 524 strengths/preparations mercury thermometers 278,
unacceptable products 188–189, 227 312–313
404–405, 519 toxicity 189, 354–355, 373, metabolic equivalents (METs)
jejunum 114, 151 396, 410, 454–455, 335, 427
joule 268–269, 304 478–479, 508, 526 metaraminol 196, 240–241
jugular vein, internal 103, 138 loop of Henle 15, 68 microcytic anaemia 344,
low-frequency jet ventilation 439–440
ketamine 185, 222–223 (LFJV) 349, 446–447 microshocks 270, 305
kidney 15, 66 lower limb midazolam 504
anatomy 112, 148–149 blood supply 128, 171 migraines 341, 436
blood flow 13–14, 63–64 fascia 126, 168–169 minimum alveolar
disease 361, 461–462 femoral triangle 127, concentration (MAC)
failure 396, 508–509 169–170 187, 225
physiology 13, 62–63 muscle groups 126, 169 minute ventilation 8, 51–52
sodium handling 16, 69 muscles 127, 169 miscarriages 368, 472

Downloaded from https://www.cambridge.org/core. University of Edinburgh, on 19 Aug 2019 at 13:21:27, subject to the Cambridge Core terms of
use, available at https://www.cambridge.org/core/terms. https://doi.org/10.1017/9781108566100.013
552 Index

mivacurium 194, 236 cranial 116–117, 154–156 one-lung ventilation (OLV)


moorLD12-BI 351, 449–450 facial 117, 155–156 356, 384–385, 456, 493
morphine 191, 231–232 foot 130, 173 ophthalmic needle blocks
motor control 19, 73 lower limb 127–128, 414, 531
motor evoked potentials (MEP) 170–171 ophthalmic surgery,
339, 433 orbit 102, 136–137 anaesthesia 403, 422,
mouth 102, 137 spinal 121, 159–160 517, 542
muscles upper limb 122–123, 163 opioids
composition 20, 76 neuraxial adjuvants, obstetric long-term use 406, 521
foot 129, 173 anaesthesia 384, 493 patches, dose equivalence
hand 124, 165–166 neuromodulation techniques 355, 455–456
movement 20, 76 18, 72 postoperative 423, 543–544
myasthenia gravis (MG) 335, neurolytic agents 397, 509 optical isomers 178, 210
350–351, 379, neuromuscular blocks 194, orbital nerves 102, 136–137
448–449, 486 236 orbital cavity 101, 136
associated conditions nerve stimulator 273, osmosis 291, 325
418, 537 308–309 osteoarthritis (OA) 401–402,
myasthenic syndrome non-depolarizing 272, 308 409–410, 515, 525
378–379, 486 neuronal action potentials 11, oxygen concentration analysis
myotonic dystrophy 340, 57–58 289, 290, 324
388–389, 434–435, 498 neuroreceptors 12, 60 oxygen therapy 8, 52–53
perioperative management neurosurgery positioning oxyhaemoglobin dissociation
350, 448 341–342, 436–437 curve 10, 55
myxoedema coma 356–357, neurotransmitters 11–12, 16,
456–457 58–59, 69–70 pacemakers 276–277, 343, 394,
Never Events 363–364, 415, 438–439, 506, 532
neck veins 99, 133 465–466 indicators for insertion
necrotizing enterocolitis newborns. see neonates 417, 536
(NEC) 414, 531–532 nicotine 209, 262 perioperative management
necrotizing fasciitis nifedipine 198, 243 404, 518
391–392, 502 nitrates 197, 242 paediatrics
negative exponential processes nitrous oxide 187, 225 airways 132, 176, 338, 431
268, 303–304 non-parametric tests body surface area 342, 437
neonates 267–268, 303 calculations 132, 177
apnoea 390, 500 noradrenaline 414–415, day-case services 336, 429
cyanosis 363, 465 532 regional anaesthesia 357,
glucose metabolism 406, 521 normal distributions 267, 457
hypocalcaemia 414, 531 303 pain measurement scales
intracranial haemorrhage nose 102, 137 390, 500
386, 495 NSAIDs 190–191, 229–230 pain, physiology 17, 71–72
kernicterus 396, 509 nutritional support, critically ill pancreas 30, 94–95
nervous system 23, 81–82 patients 361–362, 462–463 anatomy 113, 149–150
postoperative apnoea paracetamol 190, 228–229
393, 504 obesity 380–381, 488 overdose 399, 512
respiratory distress anaesthetic drugs 545, paramagnetic analyzers
syndrome 345, 441 423–424 287, 322
resuscitation 344, 440 android fat distribution parasympathetic nervous
suxamethonium 353, 453 424, 545 system 116, 117, 154,
thermogenesis 23, 81 bariatric surgery 336, 429 156
tracheo-oesophageal fistula intubation difficulties parathyroid hormone 103,
425, 546 395, 507 137–138
neostigmine 194, 237 in pregnancy 395, 506–507 parathyroid surgery,
Nernst equation 10, 57 oesophageal Doppler cardiac anaesthesia 423, 544
nerves output monitoring paravertebral nerve blocks 337,
brachial plexus 122, 124, 418, 537 429–430
161, 165 Ohm’s law 281, 315 parenteral nutrition 400, 513

Downloaded from https://www.cambridge.org/core. University of Edinburgh, on 19 Aug 2019 at 13:21:27, subject to the Cambridge Core terms of
use, available at https://www.cambridge.org/core/terms. https://doi.org/10.1017/9781108566100.013
Index 553

Parkinson’s disease 364, postoperative ventilation prolonged depolarizing


374–375, 480–481 391, 502 neuromuscular blockade
clinical features 413, 529 postpartum bleeding 408, 524 184, 221
drugs 415, 533 power 284–285, 318 prone positioning
patient-controlled analgesia pre-eclampsia 22, 80, 404, 421, 344–345, 440
drugs used 192–193, 234 518–519, 541 propofol 185, 186, 222, 224,
in labour 189–190, 228 predicted postoperative FEV1 399, 512
safety features 192, 233–234 percentage (ppoFEV1%). prostaglandin infusion
Penaz technique 270, 306 343–344, 439 339, 432
pericardium 112, 148 prednisolone 204, 255 pterygopalatine fossa 104,
pethidine 21, 78 pregnancy 138–139
pH balance 15, 66–67 acute fatty liver 337, 430–431 pulmonary artery catheters
pH electrodes, arterial blood anatomical changes (PAC) 289–290, 324,
gas analyzers 278–279, 313 131–132, 175–176 386, 495
phaeochromocytoma 398, 423, blood flow 21, 77 pulmonary function tests 277,
511, 544 cardiac arrests 358, 458 421–422, 541–542
phenylephrine 196–197, 241 cardiac disease 348, 445 pulmonary hypertension (PH)
phenytoin 203, 253 cardiovascular changes 351, 450
phosphodiesterase inhibitors 21, 78 pulmonary resection 353, 452
195, 239 ECG 22, 79 pulmonary vascular resistance
Pierre Robin syndrome eclampsia 21, 77–78 (PVR) 197, 242
366, 468 epidural analgesia 365, pulse oximetry 290, 324–325
pituitary glands 28, 91–92 468 pyloric stenosis 411, 527–528
pituitary surgery 355–356, 456 hepatobiliary changes 22, pyridostigmine 421, 541
placenta 79–80
blood supply 131–132, 176 massive haemorrhage/ racemic mixtures 178, 210
drugs crossing 22, 78–79, transfusion 405, 520 radial nerve injuries 126, 168
182–183 mechanical heart valve radiofrequency (RF)
hormones 23, 80 412–413, 529 denervation, lumber spine
membrane 21, 77 non-obstetric surgery 364, 466
plasma, blood 31, 96 392, 502 Raman effect 288, 323
plasma potassium 16, 68–69 obesity 395, 506–507 randomization, treatments
plenum vaporizers physiological changes 266–267, 301–302
292–293, 326 372–373, 477–478 rapid sequence induction (RSI)
pleurae 108, 144 pre-existing neurological 334, 354, 453–454
nerve supply 107–108, diseases 388, 498 reflexes
143–144 pre-existing spinal cord lower limb 130, 173–174
pneumonectomy 369, 473 injury 352–353, 452 stretch 20, 76–77
pneumotachograph 274, regional anaesthesia 418, upper limb 122, 161
309–310 536–537 regional anaesthesia
popliteal fossa 128–129, related diabetes 362, 463 children 357, 457
130–131, 171–172, 174 renal tract changes 23, 80–81 contraindications 416, 534
nerve blocks 131, 174–175 respiratory changes 22, 79 religious objections, naturally
positron emission tomography sepsis 415, 532–533 occurring fluids/drugs
(PET) 284, 318 thromboembolic disease 385–386, 494
post-dural-puncture headaches 398–399, 512 remifentanil 191–192, 232
401, 419, 515, 538 trauma 425, 546 renal replacement therapy
postherpetic neuralgia (PHN) preload 3, 41 (RRT) 380, 407–408,
335–336, 378, pressure 417–418, 487, 523, 536
427–428, 485 cylinders 294–295, 328–329 renal tract, changes in
postoperative cognitive flow 294, 328 pregnancy 23, 80–81. see
dysfunction (POCD) preterm infants also kidney
415–416, 533–534 apnoeic changes 23, 81 respiration. see also breathing
postoperative nausea and assisted ventilation 349, systems; lungs
vomiting (PONV) 447 acute failure 9, 53, 54
383, 492 prokinetics 204–205, 256 acute injury 9, 53–54

Downloaded from https://www.cambridge.org/core. University of Edinburgh, on 19 Aug 2019 at 13:21:27, subject to the Cambridge Core terms of
use, available at https://www.cambridge.org/core/terms. https://doi.org/10.1017/9781108566100.013
554 Index

respiration (cont.) skull, anatomy 118–119, 157 and pressure 292, 326
blood flow 7, 48–49 smoking, ill-effects 209, 262, thalassaemia 348, 445–446
blood supply/lymph 351–352, 383–384, therapeutic index 179, 212
drainage 107, 143 450, 492 thermal burns 360
conditions 277, 312 sodium ion channels 11, 58 thiazide diuretics 197, 242–243
dead space 5, 46 somatosensory evoked thigh
high altitude 24, 83 potential (SSEP) 339, 433 muscles 126–127, 169
mechanics 6, 47 sound, properties 297, 331 nerves 127–128, 170–171
nerve supply 107–108, space travel, physiological thiopentone 182, 185–186,
143–144 changes 26, 84 223–224
old age 32, 98 spinal anaesthesia 413, 530 third-degree heart blocks
pregnancy 22, 79 spinal blood supply 121, 159 412, 528
work of 6, 46 spinal cord 120, 159 thoracic spinal cord injuries
respirometer, Wright’s acute injury 406, 521–522 368–369, 473
274, 309 levels 393–394, 505 thoracotomy, pain relief 409,
resting membrane potential neurotransmitters 16, 69–70 524–525
(RMP) 10, 56–57 stimulation 371, 475–476 3-in-1 blocks, lower limb
retinopathy, premature infants spinal nerve blocks, high 121, 131, 175
403, 517 160, 377, 484 throat 100, 134
rhabdomyolysis, acute 421, spinal nerves 121, 159–160 thromboelastography (TEG)
540–541 spinal pathology red flags 386, 201, 250–251
rheumatoid arthritis 392, 503 495–496 thyroid
ribs 106, 141–142 spinothalamic tract 18, 72–73 glands 102, 137
risk, perioperative 347, spironolactone 199, 247 goitre 104, 139–140
444–445 starvation 26, 86–87 hormones 29, 94
rocuronium 193, 234–235 starvation times, preoperative surgery 104, 140
rotameters 274–275, 310 343, 372, 438, 477 thyroidectomy
statistics 266–268, 301–304 intraoperative management
sacral dermatomes 130, 174 stimulants 208, 209, 393, 505
sacral plexus 118, 157 261–262, 263 marked stridor 420–421,
sacrum 118, 156 subarachnoid haemorrhage 540
salbutamol 204, 254–255 (SAH), grading systems partial 104, 139
saline, perioperative use 390–391, 500–501 thyrotoxic storm 351, 450
202, 251 sub-Tenon’s blocks 104, 375, thyroxine 207, 260–261
scavenging systems 281, 394, 399–400, 513 tobacco smoking see also
315–316 sugammadex 194, 236–237 smoking
sciatic nerve blocks 355, superior laryngeal nerve blocks tocolytics 206, 257–258
396–397, 455, 509 386, 494 total intravenous anaesthesia
second messengers 12, 60 supraclavicular nerve blocks 184, 220–221
sedation 382, 402–403, 409, 125, 167 total spinal blocks 424–425,
490, 516, 525 supraglottic airway devices 545–546
adults 345, 440–441 413–414, 530–531 trachea 105, 140
children 388, 405–406, surfactants 6, 47 tracheostomies 105–106,
520–521 suxamethonium 193–194, 140–141, 377–378, 384,
sepsis 347–348, 411–412, 235–236 484, 492
445, 528 suxamethonium apnoea 184, percutaneous 409, 525
children 389, 499 221, 363, 389, 464, 499 train-of-four stimulation
pathophysiology 352, 452 swallowing 28, 90 273, 308
septic shock 376, 482–483 Swan–Ganz technique 275, 310 tramadol 191, 230–231
sevoflurane 187, 225–226 sympathetic nervous system tranexamic acid 201, 249–250
SI units 264, 299 115, 153–154 transcutaneous oxygen
sickle cell disease 338, 369, measurement 289, 323
473–474 temperature. see also body transducers 294, 328
sinoatrial node (SA node) 1, 35 temperature; heat transfusion-related acute lung
skeletal myofibrils 20, 76 critical 293, 327 injuries (TRALI) 370, 474
skin flaps 399, 512 measurements 264–265, 300 transplantable organs 336, 428

Downloaded from https://www.cambridge.org/core. University of Edinburgh, on 19 Aug 2019 at 13:21:27, subject to the Cambridge Core terms of
use, available at https://www.cambridge.org/core/terms. https://doi.org/10.1017/9781108566100.013
Index 555

transthoracic impedance 273, antecubital fossa 124–125, veins, neck 99, 133
309, 406, 521 166–167 venous air embolism 371, 410,
transversus abdominis plane axilla 124, 164–165 525–526
(TAP) blocks 367, blood supply 123, 163–164 ventilation, minute 8, 51–52
470–471 dermatomes 122, 161–163 ventilation/perfusion ratio 6,
trauma, regional anaesthesia lymphs 123–124, 164 47–48
372, 477 nerves 122–123, 124, 161, ventilator-associated
trigeminal neuralgia (TN) 163, 165 pneumonia 398, 511
350, 448 reflexes 122, 161 ventricular fibrillation (VF)
turbulent flow, tube 4, 44, small muscles 124, 165–166 337, 392–393, 430, 503
274, 309 upper muscles 124, 165 visceral pleura, nerve supply
TURP syndrome 354, 359, upper respiratory tract 107–108, 143–144
366–367, 469–470 infections 413, 530 vision control 101, 136
type I errors, statistics 267, ureters 112, 148–149 vitamin B12 deficiency
302–303 uterotonic drugs 205–206, 257 377, 483
type II errors, statistics 267, uterus, blood supply vomiting 27, 89
303 131–132, 176 warfarin 199–200, 247
Wilson’s criteria 347, 443–444
ulnar nerve injuries 126, vacuum insulated evaporators work and power 285, 318–319
168 297, 331 Wright’s respirometer 274,
ultrasound-guided regional Valsalva manoeuvre 3, 41–42 309
blocks 347, 444 vascular smooth muscles 4, 44
ultrasound imaging 297–298, vasodilators 197, 241–242 X-rays 283, 317–318
331–333, 341, 436 vasopressin 196, 241
underwater physiology 25, 83 Vaughan-Williams zero-order kinetics
upper limb classification 198, 244 181–182, 215

Downloaded from https://www.cambridge.org/core. University of Edinburgh, on 19 Aug 2019 at 13:21:27, subject to the Cambridge Core terms of
use, available at https://www.cambridge.org/core/terms. https://doi.org/10.1017/9781108566100.013
Downloaded from https://www.cambridge.org/core. University of Edinburgh, on 19 Aug 2019 at 13:21:27, subject to the Cambridge Core terms of
use, available at https://www.cambridge.org/core/terms. https://doi.org/10.1017/9781108566100.013

You might also like